Download as txt, pdf, or txt
Download as txt, pdf, or txt
You are on page 1of 523

--------------------------------------- 1

Emergency Management:
Evaluation of the Critically
III or Injured Child
The critically ill or injured child must be evaluated
e cervical spine if there is a
rapidly to minimize morbidity and mortality.
inal cord injury
Whether presenting to the physician's office, loca l
rynx with a Yankauer suction
clinic, community hospital, or tertiary care center,
inger sweep is contraindicated
the patient is stabilized by administering basic lifebody may be forced further
support and advanced cardiac life-support measures
nx)
recommended by the American Heart Association .
via the jaw-thrust or chin-lift
Once the patient is clinically stable, a problem list
eving any obstruction caused by
can be generated and the cause of the child's sympt tissues of the neck
toms can be determined.
the midline "sniffing position,"

Immobilization of th
possibility of sp
Clearing the oropha
catheter (blind f
because a foreign
down the orophary
Opening the airway
maneuver and reli
the tongue or sof
Placing the head in
often via a rolle

d-up towel beneath the occiput


(hyperextension o
f the neck may result in obstruc_ _DI_FFERENTJAL DlAGNOSIS ___

tion of the airway)


Provision of 100% ox

ygen via face mask


If indicated, placem
ent of an oral or a nasophaOf the causes of pediatric cardiorespiratory arrest,

ryngeal airway

respiratory etiologies (45%), cardiac etiologies (25%],


and primary central nervous system disorders (20%)
established, air exchange
account for 90% of all cases. The differential diagnosis
e evaluated. Examination of
for children with cardiopulmonary arrest, excluding

Once an airway is
(breathing) should b

neonates, is listed in Table 1-1.


reveals the presence and effec-

chest wall excursion


tiveness of spontane

ous respirations. If spontaneous


respiration is prese
nt with adequate oxygenation,
intubation is not in
dicated. If chest wall excursion is
CLINICAL MANIFESTATIONS
heal tube placement is indi-

not adequate, endotrac

cated (if not alread


y in place to secure the airway).
AND TRE;ATMENT
ger than 8 years, an uncuffed tube

If the child is youn

should be used to re
duce the risk of subglottic edema
Primary Survey
and stenosis. (In ch
ildren younger than 8 years, the
The primary survey (Figure 1-1} involves assessment
cricoid ring is the
narrowest part of the airway and
of Airway, Breathing, Circulation, Disability, and
provides the seal fo
r the uncuffed tube.) The size of
Exposure. The aim is identification of life-threatening
the endotracheal tub
e chosen should equal 4 + (age
conditions. Resuscitative measures are outlined in
in years -5- 4). Blo
od oxygenation (via pulse oximetry
Figure 1-2.
or arterial blood ga
s measurement) and blood CO
2
The goals of airway management are to recognize
level (by arterial o
r venous blood gas measurement)
and relieve obstruction, prevent aspiration of gastric
should be assessed a
nd will guide respiratory therapy.
contents, and promote adequate gas exchange. The
Neonatal intubati
on is traditionally performed
airway is assessed and, if necessary, secured as followswithou:
t premedicat
ion, but intubation of the infant

--------------------------------------- 2
Blueprints Pediatrics
1-0
however, activated charcoal is ineffective in ingesvacuum. Such an overhaul
tions with alcohol, hydrocarbons, iron, and lithium.
the amount of lead dust in the
Specific antidotes exist for several commonly
nts must be temporarily housed
ingested drugs (see Table 2-1).

particle accumulator
invariably increases
air, so the inhabita
elsewhere.
Symptomatic child

ren should be immediately


removed to a lead-fr
ee environment and treated
Prevention
imercaprol (BAL) followed by
Pediatricians have played a major role in decreasing
calcium-disodium (EDTA). Oral
the number and severity of poisonings, including
n alternative in asymptomatic
lobbying for child-resistant medicine bottle caps and
below 60|ig/dL.
incorporating anticipatory guidance into well-child
visits. Specific topics include "childproofmg" the
home, keeping medicines in a lock box, removing
cleaning products from children's reach, and the judicious use of syrup of ipecac.
idelines for screening reflect

with intramuscular d
intravenous edetate
succimer (DMSA) is a
children with levels

Prevention
Recently modified gu
the continuing decre

ase in the incidence of elevated


blood lead levels in

children. Targeted screening is


based on risk assess
ment information gathered during
_LE_A_D_PpJS_ONING
Centers for Disease Control and

well-child visits. The


Prevention recommend

s lead screening at 12 and 24


Lead poisoning is one of the most importanmontht
ts living in areas with many prepreventive health issues in primary care pediatrics.
ally high percentages of
The elimination of lead in house paint (in 1977) anelevated
levels.
gasoline (in 1988) has decreased the average blood
level of lead by 75%. The primary source of lead
today is lead-containing paint present in buildings
constructed before 1950. Children breathe in lead
IDENTS
dust, ingest paint chips, and play in lead-contaminated soil. Although there is no direct correlation
es remain the leading cause of
between blood levels and morbidity, levels of 10 to
all age groups. Most infants
19|_ig/dL are considered borderline, and the term
ain trauma as vehicle occupants,
"lead poisoning" is reserved for levels of 20(0,g/dL or
hildren tend to be injured as
greater.
associated with an increased

s for patien
1950 homes and unusu
d blood lead

_MOTp_R VEHJCLE ACC

Motor vehicle injuri


accidental death in
and adolescents sust
whereas school-age c
pedestrians. Factors
risk of automobile i

njury and death include male


gender, age between
15 and 19 years, warm or inClinical Manifestations
ht or weekend driving, and

clement weather, nig


alcohol intoxication

.
Early symptoms of lead poisoning include irritabilf seat belts and child car seats
ity, hyperactivity, apathy, decreased play, anorexia,
highly effective in reducabdominal pain, constipation, and intermittent vomsevere injury and death. All
iting. Children with chronically elevated lead levels
eat restraint of passengers under
may manifest developmental delay, behavioral prob20 pounds or heavier and 1
lems, attention disorders, and poor school performay ride facing forward, whereas
mance. Acute encephalopathy is the most serious
face the rear. Older children
complication of lead poisoning and is characterized
with lap and shoulder straps
by increased intracranial pressure, vomiting, ataxia,
e air bags are designed primarily
confusion, seizures, and coma.

The routine use o


has been shown to be
ing the incidence of
states require car s
40 pounds. Children
year of age or older
lighter infants must
should remain belted
at all times. Becaus
for adults, children

should ride belted in the back


seat whenever possib
le. There is no evidence that

Treatment
grams are an effective deterrent

driver education pro

to accidents involvi
ng teenage drivers.
The most effective therapy involves removing the
Bike helmets decr
ease the risk of significant closed
poison from the child's environment. Leaded painheat
d trauma due to
traffic accidents involving
should be stripped and surfaces cleaned with highbicycles. In many j
urisdictions, law mandates their
phosphate detergent and a special high-efficiency
use by children. Chi
ldren younger than 10 years

--------------------------------------- 3
100
Blueprints Pediatrics
macroorchidism due to testicular edema, dysmorphic
nomalies, tracheoesophageal
facial features (large jaw and large ears), perseveraeal atresia, and radial or renal
tive speech, and mental retardation (90% of affected
ure to significant levels of serum
males have an IQ between 20 and 49). Some males
constellation of clinical features
with fragile X syndrome have mental retardation as
alcohol syndrome. Typical findthe sole manifestation. Female carriers of the fragile
alpebral fissures, smooth
X chromosome may have a subnormal IQ. Autism
pper lip. Affected infants may
occurs more commonly in children with the fragile
poor growth, developmental
X chromosome than in the general population. There
art disease, and renal anomalies.
is no treatment for the syndrome.
Chromosome 22q11 Deletion
Y POINTS
Syndromes
0% of first trimester spontaneous

vertebral and anal a


fistula with esophag
abnormalities. Expos
alcohol results in a
referred to as fetal
ings include short p
philtrum, and thin u
also have hypotonia,
delay, congenital he
KE
1. Approximately 5
abortions have

chromosomal abnormalities.
Microdeletion of 22ql 1.2 has been found in 90% of
aused by autosomal anomalies are
children with DiGeorge's syndrome, in 70% of chilsevere than those caused by sex
dren with velocardiofacial syndrome, and in 15% of
rmalities.
children with isolated conotruncal cardiac defects.
obtaining chromosomal studies
Although the names of the above-mentioned synation of a suspected chromosomal
dromes are still in use, the more general term
22qll.2 deletion syndrome more appropriately
ple organ system malformations,
encompasses the spectrum of abnormalities found in
elopmental delay or mental retarthese children. Its prevalence in the general popularwise explained, short stature or

2. Birth defects c
generally more
chromosome abno
3. Indications for
include confirm
syndrome, multi
significant dev
dation not othe

tion is 1 per 4000 live births. The deletion can be


ed menarche in girls, infertility or
inherited (8% to 28% of cases), but more typically
ltiple spontaneous abortions,
occurs as a de novo event. However, if a parent has
alia, or advanced maternal age.
the deletion, the risk to each child is 50%. The
microdeletion can be detected using fluorescent in
situ hybridization (FISH) probes. Classic cardiac features of this spectrum of disorders include conotrunRS
cal defects such as tetralogy of Fallot, interrupted
aortic arch, and vascular rings. Other common findc Disorders
ings are absent thymus, hypocalcemic hypoparathymetabolic disorders are rare, colroidism, T-cell mediated immune deficiency, and
esponsible for significant morbidpalate abnormalities. These children usually have
nborn errors of metabolism are
feeding difficulties, cognitive disabilities, and behavt occur when a defective protein
ioral and speech disorders.
pathway at a specific step. Pre-

extremely delay
a history of mu
ambiguous genit

II METABpLIC DISORDE
Approach to Metaboli
Although individual
lectively they are r
ity and mortality. I
genetic diseases tha
disrupts a metabolic
cursors and toxic me

tabolites of excess precursors


accumulate, and prod
ucts needed for normal metabISORDER

olism are deficient.

Certain ethnic groups are at


increased risk for s
pecific metabolic errors.
Some syndromes without a detectable chromosomal
tion and age at onset vary. Urea
abnormality have clinical features that suggest a
ganic acidemias present early in
chromosomal disorder. These syndromes often entelifr
olic decompensation. Fatty acid
into the differential diagnosis of a suspected genetic
ydrate metabolism disorders
disorder. CHARGE is an acronym for a nonrandom
lethargy, encephalopathy, and
association of features including coloboma of the
ow carbohydrate intake or
retina or iris; heart abnormalities; atresia of the
torage disorders are characterized
choanae; retarded growth; genital hypoplasia in
omegaly, splenomegaly, and,
males; and ear abnormalities that can include deafogic deterioration. Findings that
ness. VATER refers to the nonrandom association of
icion for an inborn error of

Clinical presenta
cycle defects and or
e with acute metab
oxidation and carboh
usually present with
hypoglycemia after l
fasting. Lysosomal s
by progressive hepat
occasionally, neurol
should increase susp

--------------------------------------- 4
Cha
101
pter 9 / Genetic Disorders
metabolism include emesis and acidosis after initiasynthesi-

s or breakd

own. Because many different


tion of feeding, unusual odor of urine or sweatenzyme,
ved in glycogen metabolism, the
hepatosplenomegaly, hyperammonemia, early infanclinicat
ons of the GSDs are variable.
death, failure to thrive, developmental regressionTypica,
ions include growth failure,
mental retardation, and seizures. Several important
sting hypoglycemia. The most
disorders are discussed here.
I, von Gierke's disease; type

s are invol
l manifestati
l manifestat
hepatomegaly, and fa
common GSDs are type
II, Pompe's disease;

and type V, McArdle's disease. All


are autosomal recess
ive disorders. Treatment is
Carbohydrate Metabolism Disorders
hypoglycemia while avoiding
Galactosemia
glycogen in the liver.
Galactosemia, the most common error of carbohydrate metabolism, is caused by a deficiency of the
m Disorders
enzyme galactose-1-phosphate uridyltransferase,
resulting in impaired conversion of galactose-1phosphate to glucose-1-phosphate (which caPhenylketonurin
(PKU), the most common of these
undergo glycolysis). Galactose-1-phosphate accumudisordersin 1 in 10,000 live births. PKU
lates in the liver, kidneys, and brain. The disorderesultr
iency of phenylalanine hydroxyoccurs in 1 of 40,000 live births, and inheritance is
t converts phenylalanine to
autosomal recessive.
l phenylalanine intake, patients
Clinical Manifestations
oncentrations of toxic metabo-

designed to prevent
storage of even more
Amino Acid Metabolis
Phenylketonuria
a
, occurs
s from a defic
lase, the enzyme tha
tyrosine. With norma
develop high serum c
lites such as phenyl

acetic acid and phenyllactic acid.


Clinical manifestations are noted within a few days
ons
to weeks after birth. Initial symptoms include
evidence of liver failure (hepatomegaly, direct
id disorders, symptoms of
hyperbilirubinemia, disordered coagulation), renal
p in childhood rather than early
dysfunction (acidosis, glycosuria, aminoaciduria),
manifestations include moderate
emesis, anorexia, and poor growth. Cataracts may
ardation, hypertonia, tremors, and
develop by 2 months of age in untreated children.
Tyrosine is needed for the proInfants with galactosemia are at increased risk of
so the block in the conversion of
Escherichia coli sepsis. Older children have severe
osine results in light complexion.

Clinical Manifestati
Unlike most amino ac
untreated PKU develo
infancy. Neurologic
to severe mental ret
behavioral problems.
duction of melanin,
phenylalanine to tyr
The patient's urine

smells mouse-like.
learning disabilities, whether or not they were treated
in infancy. Affected females have a high incidence
of premature ovarian failure. Detecting reduced

Treatment
Prevention of mental

retardation in PKU is achieved


levels of erythrocyte galactose-1-phosphate uridylg dietary restriction of phenylalatransferase is diagnostic. Laboratory findings include
clude PKU detection on their
a direct hyperbilirubinemia, elevated serum aminocreens. Pregnant women with
transferases, prolonged prothrombin and partial
thromboplastin times, hypoglycemia, and aminoict phenylalanine intake draaciduria. Galactose in the urine is detected by a
he risk of having a child with

by early and lifelon


nine. Most states in
mandatory neonatal s
PKU who do not restr
matically increase t
microcephaly, mental

retardation, and congenital


positive reaction for reducing substances and no
reaction with glucose oxidase on urine test strips.
Treatment
used by a defect in the amino
Eliminate all formulas and foods containing galactose
ay that converts methionine to
(including lactose-containing formulas and breast
The incidence of the cystathiomilk).

heart disease.
Homocystinuria
Homocystinuria is ca
acid metabolic pathw
cysteine and serine.
nine synthase defici

ency is 1 in 100,000 live births.


Glycogen Storage Diseases
used by most states detects
Glycogen is a highly branched polymer of glucose
levels in the blood.
that is stored in liver and muscle. Glycogen storag
ions
diseases (GSDs) are a group of conditions that resulThert
oms in infancy. Clinical manifesfrom deficiency of enzymes involved in glycogen
ing childhood include a Marfan's

The neonatal screen


increased methionine
Clinicael Manifestat
e are no sympt
tations observed dur

--------------------------------------- 5
Blueprints Pediatrics
102
body habitus (long thin limbs and digits, scoliosis,
sing benzoic acid and phenylacsternal deformities, and osteoporosis), dislocated eye
ntion may minimize deleterious
lenses, mild to moderate mental retardation, and vasent is complex and extremely
cular thromboses that result in childhood stroke or
s to maintain.
myocardial infarction.
Treatment

nitrogen excretion u
etate. Early interve
effects, but managem
difficult for parent

Lysosomal Storage Di
sorders
Dietary management is extremely difficult because
restriction of sulfhydryl groups leads to a very lowsomal enzyme causes its substrate
protein, foul-tasting diet. Approximately 50% of
osomes of tissues that degrade
patients respond to large doses of pyridoxine.

Deficiency of a lyso
to accumulate in lys
it, creating a chara

cteristic clinical picture. These


"storage" diseases a
re classified as mucopolysacchariOrnithine Transcarbamylase Deficiency
s, Hunter's, and Sanfilippo's synOrnithine transcarbamylase (OTC) deficiency, a urea
e.g., Niemann-Pick, Krabbe's,
cycle defect, is one of the few inborn errors of metabchs diseases), or mucolipidoses
olism with X-linked inheritance. Amino acid catabond mannosidosis), depending on
lism produces free ammonia that is detoxified to urea
ored material.
through a series of reactions known as the urea cycle.
In the urea cycle, ornithine joins with carbamoylphosphate through the action of OTC to
form citrulline within the mitochondria. When OTC
onidase leads to accumulation of
levels are less than 20% of normal, the nitrogenaran sulfates in tissues and
containing moiety in ornithine cannot be quickly
rine. Typical features include
converted to urea for excretion and, instead, forms
l clouding, exaggerated kyphosis,
ammonia, which results in severe hyperammonemia
umbilical hernia, and congeniwhen the patient consumes protein. Milder forms of
evelopmental regression begins in
the condition are seen in heterozygous females and
fe. Most children with Hurler's synin some affected males.
dolescence.
Clinical Manifestations
Within 24 to 48 hours after the initiation of proteincontaining feedings, the newborn becomes progrescaused by deficiency of the
sively lethargic and may develop coma or seizures as
e, leading to the accumulation of
the serum ammonia level rises. Female carriers may
e classic form does not involve
develop headaches and emesis after protein meals
system. Patients characteristically
and manifest mental retardation and learning disd splenomegaly. Storage of gluabilities. Diagnosis is made by measuring the level of
bone marrow leads to anemia,
orotic acid, a by-product of carbamoylphosphate
ytopenia, and recurrent
metabolism, in the urine.
n. Radiologic changes include

doses (e.g., Hurler'


dromes], lipidoses (
Gaucher's, andTay-Sa
(e.g., fucosidosis a
the nature of the st
Hurler's Syndrome
Deficiency of a-idur
the dermatan and hep
their excretion in u
coarse fades, cornea
hepatosplenomegaly,
tal heart disease. D
the first year of li
drome die in early a
Gaucher's Disease
Gaucher's disease is
enzyme (3-glucosidas
glucocerebroside. Th
the central nervous
have hepatomegaly an
cocerebroside in the
leukopenia, thromboc
episodes of bone pai

the Erlenmeyer flask


shape of the distal femur. A low
Treatment
enzyme level in the
white blood cells confirms the
Treatment centers on an extremely low-protein diediagnosist
. Recombi
nant enzyme therapy improves
and the exploitation of alternative pathways for
most symptoms.

--------------------------------------- 6
Hematology
ANEMIA
esting hemolysis or chronic blood

erythropoiesis, sugg
loss.

Anemia, denned as a hemoglobin concentration (or


hematocrit] two or more standard deviations below
the mean value for age and sex, is not a disease but
Clinical Manifestati
ons
rather a symptom of another disorder. The hemoglobin concentration is relatively high in the newborn
but then declines, reaching a nadir known as the

History
In the young infant,

perinatal history may reveal


physiologic anemia of infancy. This nadir occurs at
maternal transfusion. In the older
approximately 6 weeks in the premature infant and
istory may suggest risk factors for
2 to 3 months in the term infant. Thereafter, the
or folate deficiency anemia. Both
hemoglobin concentration rises gradually during
ia and lead poisoning can manchildhood, reaching adult values after puberty.
of overt or occult bleeding include

twin-to-twin or feto
child, the dietary h
iron, vitamin Bi 2,
iron deficiency anem
ifest as pica. Signs
melena, hematochezia

, hematuria, hematemesis,
abnormal menses, or
epistaxis. The patients' race/ethDifferential Diagnosis
story of splenectomy or chole-

nicity and family hi


cystectomy suggest a

n inherited hemolytic anemia.


Anemia results from decreased red cell production,
Medications can caus
e either bone marrow suppresincreased red cell destruction, or blood loss. Desion or hemolysis. O
ther questions should attempt to
creased red cell production is due to either deficiency
of hematopoietic precursors or bone marrow failure,
fever, weight loss, fatigue, rash,
and increased red cell destruction results from
and cough.
hemolytic disease, which may be due to extracorpuscular or intracorpuscular defects. Blood loss may
be acute or chronic. Table 10-1 outlines the most

elicit a history of
bruising, jaundice,
Physical Examination
Examine the patient

to assess the severity of anemia.


common causes of anemia.
nclude pallor (skin, conjunctiva,
The adjusted reticulocyte count (ARC) is used to

Important findings i
mucosa] and loss of

palmar crease pigmentation.


determine whether there has been an adequate
Comparing the comple
xion of the patient and
erythropoietic response to the given anemia. The
parents is also usef

ul. Tachycardia and postural


ARC is calculated as follows:
changes in heart rat
e and blood pressure are
seen with acute bloo
d loss. Other findings may
Measured Hematocrit
ARC =
congestive heart failure
Expected Hematocrit
, lower extremity edema, tachyx Reticulocyte Count
a (petechiae, purpura), blood

provide evidence of
(hepatosplenomegaly
cardia), pancytopeni
loss (positive stool

guaiac or gastroccult, gross


An ARC less than 2 signifies ineffective erythros (scleral icterus, jaundice,
poiesis. An ARC greater than 2 signifies effective
urine), or innltrative disorders

hematuria), hemolysi
urobilinogen in the

--------------------------------------- 7
.104

Blueprints Pediatrics
(lymphadenopathy, he

patosplenomegaly). If the
TABLE 10-1
t gain, consider anemia
Differential Diagnosis of Common Anemias

child has poor weigh


of chronic disease.

Physical findings that suggest


Defined by Mean Corpuscular Volume
anemia are found in Table
Anemia
Differential Diagnosis
Microcytic Anemias
Iron deficiency
Severe lead poisoning
Thalassemia syndromes
n
Sideroblastic anemia
is to determine whether the
Macrocytic Anemias
decreased production, increased
Megaloblastic
Vitamin B12 deficiency
d loss. Initial laboratory tests
Folate deficiency
nemia include a complete blood
Orotic aciduria
fferential and red blood cell
Nonmegaloblastic
Aplastic anemia
e count, and peripheral blood
Diamond-Blackfan anemia
Bone marrow infiltration
Hypothyroidism
lar volume (MCV) and ad-

a specific cause of
10-2.
Diagnostic Evaluatio
The goal of testing
anemia results from
destruction, or bloo
needed to evaluate a
count with manual di
indices, reticulocyt
smear.
The mean corpuscu
justed reticulocyte

count categorize the disorder into


Liver disease
ytic, or macrocytic anemia, with
Normocytic Anemias

a microcytic, normoc

adequate or inadequa
te red blood cell production.
Inherited hemolytic
Abnormal hemoglobins
ar is used to assess the red and
anemias
Sickle cell disease
phology, and the platelet
Red blood cell enzyme
disorders
hemolysis is suspected, consider

Peripheral blood sme


white blood cell mor
number and size. If
electrolytes, lactat

e dehydrogenase, bilirubin,
G6PD deficiency
Pyruvate kinase
ct and direct), and serum hapdeficiency
en may be detected on urinalyRed blood cell membrane
sphate dehydrogenase (G6PD)
disorders
idered in African-American
Hereditary spherocytosis,
pulations who present with
elliptocytosis

Coombs' test (indire


toglobin. Urobilinog
sis. A glucose-6-pho
assay should be cons
and Mediterranean po
hemolytic anemia. Pe

rform hemoglobin elecParoxysmal nocturnal

trophoresis to diagn

ose suspected hemoglohemoglobinuria


deficiency anemia is high on the
Acquired hemolytic
Antibody-mediated

binopathies. If iron
differential, serum

iron level, total iron binding


anemias
anemias
ferritin level are needed for
Autoimmune hemolytic
anemias
el is indicated if lead poisoning

capacity, and serum


analysis. A lead lev
is contemplated. Fre

e erythrocyte protoporphyrin
Isoimmune hemolytic
obtained quickly and with a small
anemias
Microangiopathic hemolytic
vated FEP levels suggest the disanemias
ration seen with iron defiHemolytic uremic
oning. An elevated erythrocyte
syndrome
s seen in anemia of chronic
Disseminated intravascular
me tests of stool or gastric concoagulation
ointestinal bleeding. If a macroChronic inflammation*
d, both vitamin B^ and red blood
Acute blood loss
re needed.
Splenic sequestration
Chronic renal disease

(FEP) levels can be


amount of blood. Ele
ordered heme incorpo
ciency and lead pois
sedimentation rate i
disease. Positive he
tents indicate gastr
cytic anemia is foun
cell folate levels a

* For anemias of chronic inflammation, 75% are normocytic and 25%Treatment


are microcytic.
Treatment varies dep
ending on the cause of
the anemia. Anemias
that appear most often on
USMLE exams are dis
cussed in the following
sections.

--------------------------------------- 8
C
hapter 10 / Hematology
105
TABLE 10-2
Physical Findings in the Evaluation of Anemia
System
Observation
Skin Hyperpigmentation
congenita
Cafe au lait spots
Vitiligo
Partial oculocutaneous albinism
Jaundice
Petechiae, purpura
mmune hemolysis with autoimmune

Significance
Fanconi's anemia, dyskeratosis
Fanconi's anemia
Vitamin B,2 deficiency
Chediak-Higashi syndrome
Hemolysis
Bone marrow infiltration, autoi
thrombocytopenia, hemolytic u

remic syndrome
Erythematous rash
Butterfly rash
Head Frontal bossing
deficiency, chronic subdural
Microcephaly
Eyes Microphthalmia
Retinopathy
Optic atrophy
Blocked lacrimal gland
Kayser-Fleischer ring
Blue sclera
Ears Deafness
Mouth Glossitis
Angular stomatitis
Cleft lip
Pigmentation
nal blood loss)
Telangiectasia
od loss)
Leukoplakia
Chest Shield chest or widespread nipples
Murmur
hemolysis
Abdomen Hepatomegaly
chronic disease, hemangioma,
Splenomegaly
(early) thalassemia, malaria,

Parvovirus, Epstein-Barr virus


SLE
Thalassemia major, severe iron
hematoma
Fanconi's anemia
Fanconi's anemia
Sickle cell disease
Osteopetrosis
Dyskeratosis congenita
Wilson's disease
Iron deficiency
Osteopetrosis
B12 deficiency, iron deficiency
Iron deficiency
Diamond-Blackfan syndrome
Peutz-Jeghers syndrome (intesti
Osler-Weber-Rendu syndrome (blo
Dyskeratosis congenita
Diamond-Blackfan syndrome
Endocarditis: prosthetic valve
Hemolysis, infiltrative tumor,
cholecystitis
Hemolysis, sickle cell disease,
lymphoma, Epstein-Barr virus

, portal hypertension

Nephromegaly
Absent kidney
Extremities Absent thumbs
Triphalangeal thumb
Spoon nails
Beau line (nails)
e illness
Dystrophic nails
Rectal Hemorrhoids
Heme-positive stool
Nerves Irritable, apathy
Peripheral neuropathy
and E, lead poisoning
Dementia
E
Ataxia, posterior column signs
Stroke
nocturnal hemoglobinuria
SLE, systemic lupus erythematosus.

Fanconi's anemia
Fanconi's anemia
Fanconi's anemia
Diamond-Blackfan syndrome
Iron deficiency
Heavy metal intoxication, sever
Dyskeratosis congenita
Portal hypertension
Intestinal hemorrhage
Iron deficiency
Deficiency of vitamins B,, B12,
Deficiency of vitamins B12 and
Vitamin B12 deficiency
Sickle cell disease, paroxysmal

--------------------------------------- 9
106

Blueprints Pediatrics
or toddler who recei

ves a diet exclusively comprised


KEY POINTS
low-iron formula. Nutritional
1. Anemia is not a disease but rather a symptom of
also occur during adolescence
another disorder.
spurt coincides with a diet with
2. Anemia results from decreased red cell producent. This is a particular problem
tion, increased red cell destruction, or blood loss.
s because of iron loss during
3. The mean corpuscular volume and adjusted
reticulocyte count categorize the disorder into a
aused by blood loss can also occur
microcytic, normocytic, or macrocytic anemia,
renatal iron loss can occur from
with adequate or inadequate red blood cell
sion or from twin-to-twin transproduction.
eeding may result from obstetric

composed of milk or
iron deficiency can
when a rapid growth
suboptimal iron cont
in adolescent female
menses.
Iron deficiency c
in young children. P
fetomaternal transfu
fusion. Perinatal bl
complications such a

s placental abruption or placenta previa. Postna


tal blood loss may occur from
obvious sources such
as surgery or trauma or may be
I MICROCYTIC ANEMIAS WITH
idiopathic pulmonary hemo-

occult, as occurs with


siderosis, parasitic

infestations, and inflammatory


DECREASED RED BLOOD CELL

bowel disease.

PRODUCTION
Clinical Manifestati
ons
Hypochromic microcytic red blood cells indicate
is usually asymptomatic. With
impaired synthesis of the heme or globin compoency (hemoglobin 6-8 g/dL), the
nents of hemoglobin. Defective heme synthesis may
exia, irritability, apathy, and easy
be the result of iron deficiency, lead poisoning,
chronic inflammatory disease, pyridoxine deficiency,
sical examination, the anemic

Mild iron deficiency


moderate iron defici
infant develops anor

fatigability. On phy
infant may have skin

and mucous membrane pallor,


or copper deficiency. Defective globin synthesis is
tomatitis, and koilonychia (spoon
characteristic of the thalassemia syndromes. Iron
y also have tachycardia and a
deficiency anemia, the thalassemia syndromes, and
rmur at the left upper sternal
anemia of chronic disease are the most common
ith severe anemia (hemoglobin
causes of hypochromic, microcytic anemias. Lead
ll show signs of congestive heart
poisoning, which may cause a mild hypochromic,
de tachycardia, an S
3, carmicrocytic anemia, is discussed in detail in
aly, distended neck veins, and
Chapter 2.

glossitis, angular s
nails). The child ma
systolic ejection mu
border. The infant w
less than 3 g/dL) wi
failure, which inclu
diomegaly, hepatomeg

pulmonary rales.
Laboratory findin
gs typical for the microcytic
Iron Deficiency Anemia
Table 10-3. Bone marrow ex-

anemias are found in

amination is not cli


nically indicated to confirm the
Iron deficiency, the most common cause of anemidiagnosisa
, but whe
n performed demonstrates
during childhood, is usually seen between 6 and 2micronormoblasti4
c hyperplasia of the erythroid line.
months of age. Nutritional iron deficiency develops
when rapid growth and an expanding blood volume

Treatment

put excessive demands on iron stores. Dietary risk


n deficiency anemia, without
factors include extended exclusive breast feeding
ve heart failure, is treated with 3
(more than 6 months) without iron supplementatemental iron per day. The retiction, consumption of low-iron formula preparationsulocyt,
increase within 2 to 3 days, and
early institution of low-iron iron-containing solids,
increase at a rate of approxiexcessive whole milk intake, and the absence of iromateln
day. Continue therapy for 8

Mild to moderate iro


evidence of congesti
o 6 mg/kg/day of el
e count will
the hemoglobin will
y 0.3 g/dL per

supplements. The iron present in breast milk is mucweekh


moglobin has returned to normal
more bioavailable than the iron in cow's milk. Ascortostores. If the hemoglobin has not
bic acid enhances the absorption of non-heme iron,
lly after one 1 month of therapy
whereas tea decreases its absorption.
een established, consider
Iron deficiency anemia can occur as early as 3
chromic microcytic anemia.
months of age in the premature infant who has inadAlthougan tolerate remarkable degrees
equate iron stores at birth. It can occur in the infant
y if the decline in hemoglobin is

s after the he
replenish tissue
increased substantia
and compliance has b
other causes of hypo
h infants c
of anemia, especiall

--------------------------------------- 10
Chapter 10 / Hematology
TABLE 10-3

107

Laboratory Findings for the Common


Plumbism
ROW
t
MCV
4
RBC no.
4
FEP
TT
HgbA 2
NL
Iron

Iron
Chronic
Deficiency
Disease
T
NL
1
NL4
i
4
T
t
4
NL
I

NL

Microcytic Anemias
Thalassemia

Thalassemia

Trait

Major

NL

NL

NL

NL

P-t

P-t

CC-NL

a-NL

NL

TIBC

NLt
NL
NLT
NL
NL4
% saturation
4
NL
t
NL
4
Ferritin
4
NL
T
NL
NLT
FEP, free erythrocyte protoporphyrin; hgb, hemoglobin;TIBC, total iron-binding
capacity; T, increased; idecreased; NL, normal.
gradual, infants with severe anemia must be transerrors in the transc
ription or translation of [3-globin
fused very slowly with small (3-5mL/kg) aliquots
mRNA and leads to re
duced synthesis of [3-globin
of packed red blood cells to avoid causing cardiac
chains. Thalassemia
syndromes are compared in
decompensation.
Table 10-4.
The number of del

eted a-globin genes determines


the hematologic cons
equences of alpha thalassemia.
KEY POINTS
be cis or trans. Cis deletions

These deletions can


occur when two a-glo

bin genes are deleted from one


1. Iron deficiency anemia, the thalassemia syndromes, and anemia of chronic disease are the
trans deletions signify a single
most common causes of hypochromic, microcytic
on on each of the two chromo-

chromosome, whereas
a-globin gene deleti
somes. Different rac

es and ethnicities have varying


anemias.
d trans deletions of a-globin genes
2. Iron deficiency is by far the most common cause
This factor is discussed later.
of anemia during childhood and is most often
thalassemia, or hemoglobin
seen between 6 and 24 months of age.
rs when all four a-globin genes
3. Mild to moderate iron deficiency anemia, without
to produce any a-globin chains
evidence of congestive heart failure, is treated
tetramers (hemoglobin Bart's).
with 3 to 6 mg/kg/day of elemental iron per day. If
the hemoglobin has not increased substantially
s a high affinity for oxygen
after 1 month of therapy, other causes of
it to the tissue. The result

rates of both cis an


in their population.
Homozygous alpha
Bart's disease, occu
are deleted. Failure
results in y-globin

Hemoglobin Bart's ha
and does not release
is severe anemia, ti

ssue anoxia, heart failure,


hypochromic microcytic anemia should be
generalized edema, and death
considered.
ops fetalis. The cis deletion is

hepatosplenomegaly,
in utero due to hydr
most prevalent in So

utheast Asians.
Hemoglobin H dise
ase results from deletion of
three a-globin genes
. y-Globin chains are only proAlpha and Beta Thalassemia
ormal infants, fetal hemoglobin
Pathogenesis and Clinical Manifestations

duced in utero. In n

(which consists of
two a-globin chains and two yThe thalassemias are hereditary hemolytic anemias
ly predominates at birth. In
characterized by decreased or absent synthesis of one
hemoglobin H disease, the
or more globin subunits of the hemoglobin molecule.
eads to the formation of hemoAlpha thalassemia, caused by deletion of one or more
accounts for 10% to 40% of the
of the four a-globin genes, leads to reduced synthe-

globin chains) usual


newborn infants with
dearth of a-globin l
globin Bart's, which
total hemoglobin. Wi

th the cessation of y-globin synsis of a-globin chains. Beta thalassemia is caused by


of (3-globin synthesis at birth,

thesis and the onset

--------------------------------------- 11
108

Blueprints Pediatrics
TABLE 10-4

Comparison of the Thalassemia Syndromes


Genetic Abnormality

Percent Hemoglobin
Other

HbF
Normal ccp
2-3
Beta thalassemias

Hb A

Hb A2

90-98

2-3

Thalassemia major
P-thal p-thal

0 2-5

95
p-thal+ p-thal+
20-80
Thalassemia intermedia

Very low 2-5


Overlaps with thalassemia major

(varied genetic globin


abnormalities)
Thalassemia minor
P p-thal or p p-thal+
2-10
Alpha thalassemias

90-95

5-7

60-70

2-5

90-98

2-3

90-98

2-3

Homozygous
a-thalassemia
Hb H (p4)
/
Hb Bart (y4)
Hemoglobin H disease
2-5
Hb H 30-40
/
a
Alpha thalassemia minor
2-3
a/ a
a a/
Silent carrier
2-3

a/a a
hemoglobin Bart's diminishes and hemoglobin H
g with alpha thalassemia trait or
(which consists of a |3-globin tetramer) predominates
.
after the first few months of life. Hemoglobin H
can be subdivided into homozyeventually accounts for 30% to 40% of the total
ia major) and heterozygous
hemoglobin, and normal hemoglobin A accounts for
mia minor). Beta thalassemia
approximately 60% to 70% of the total hemoglobin.
from complete absence of p
1This diagnosis is most common in children with
/BO genotype) due to defective
Southeast Asian ancestry.
A or from partial reduction of
Alpha thalassemia trait, also known as alpha tha+ genotype) due to translational
lassemia minor, results from deletion of two a-globin
th beta thalassemia minor, the
genes. This defect manifests with mild anemia,
as one normal (3-globin gene
hypochromia, and microcytosis. Alpha thalassemia
lobin gene.
trait, present in 3% of U.S. blacks, is often confused
a thalassemia major have severe
with mild iron deficiency. The hemoglobin elecsplenomegaly during the first
trophoresis is normal in these children, and the diagreated, bone marrow hyperplasia
nosis is one of exclusion confirmed by documentinang
hernatopoiesis produce characparental microcytosis.
ch as tower skull, frontal bossing,
Those with deletion of only one cc-globin gene are
y with prominent cheekbones,
considered silent carriers for alpha thalassemia, as
lure to thrive is prominent. Death
they have a normal hemoglobin concentration and
rst few years of life due to pronormal red blood cell indices. The condition can be
heart failure if the patient is not
measured only by quantitative measurement of
transfusions. Despite severe
globin chain synthesis or by gene analysis. A carrier
iculocytopenia, reflecting ineffec-

can produce offsprin


hemoglobin H disease
Beta thalassemia
gous (beta thalassem
forms (beta thalasse
major results either
globin synthesis (BO
transcription of mRN
gene product (BVB
errors. The child wi
heterozygous form, h
and one abnormal p-g
Children with bet
hemolytic anemia and
year of life. If unt
d extramedullary
teristic features su
maxillary hypertroph
and an overbite. Fai
occurs within the fi
gressive congestive
supported with blood
anemia, there is ret

--------------------------------------- 12
Chapter 10 / Hematology 109
tive hematopoiesis. Peripheral blood smear reveals
with chelating agents such as
marked hypochromia, microcytosis, anisocytosis,
ause of the constant state of
and poikilocytosis. On hemoglobin electrophoresis,

patients are treated


desferrioxamine. Bec
increased erythropoi

esis, folic acid supplementation is


hemoglobin A is either markedly decreased (BVB
+ recommended for pati
ents not maintained on chronic
or BVBO) or totally absent (BO/BO). On quantitative
transfusion therapy
in order to prevent folate defihemoglobin electrophoresis, hemoglobin F accounts
ciency and megalobla
stic anemia. Bone marrow transfor 95% in the BO/BO genotype and 20% to 80% iplantation
n is curati
ve, but because of its associated
the BVB + genotype. If the diagnosis is in question
morbidity and mortal
ity, this procedure is performed
or the child's hemoglobin electrophoresis is equivoin only a few center
s using HLA-matched sibling
cal, the parental complete blood count, smears,
donors.
and hemoglobin electrophoresis may clarify the
rapy for hemoglobin H disease are
diagnosis.
r beta thalassemia major. The
Children with beta thalassemia minor have only
and chelation therapy depends
a mild hemolytic anemia. On blood smear, the
he anemia.
hypochromia, microcytosis, and anisocytosis are disecessary for alpha or beta
proportionately severe given the degree of anemia.
enetic counselling is recomHemoglobin electrophoresis shows elevation of the
smear of iron deficiency anemia
hemoglobin A 2 level and sometimes a mild elevation
halassemia minor are quite
of hemoglobin F.
ith presumed iron deficiency

Principles of the
the same as those fo
need for transfusion
on the severity of t
No treatment is n
thalassemia minor. G
mended. Because the
and alpha and beta t
similar, the child w
anemia who does not

respond to oral iron therapy


Epidemiology
e compliant should have a hemoAlpha thalassemia is most common in African,
is to rule out beta thalassemia
Southeast Asian, Mediterranean, and Middle Eastern
h alpha thalassemia trait has a
populations. The most severe forms of alpha thaectrophoresis (may have elelassemia, three- and four-gene deletions, are seen in
t's as neonate), whereas the
the Southeast Asian population because of the high
he child with beta thalassemia
prevalence of cis deletions. Beta thalassemia is most
evated hemoglobin A
2 and
often found in populations originating from the

and is believed to b
globin electrophores
minor. The child wit
normal hemoglobin el
vated hemoglobin Bar
electrophoresis of t
minor may show an el
hemoglobin F.

Mediterranean, Middle East, and India.


Treatment
K
EY POINTS
Therapy for children with beta thalassemia major
symptoms of alpha and beta thaconsists of frequent packed red blood cell transfusions

1. The severity of
lassemia depend

s on the level of a- or (3-globin


to ameliorate the anemia and prevent congestive heart
.
failure. These children require 10 to 20mL/kg of
sease and beta thalassemia
leukodepleted red blood cells every 3 to 5 weeks to
ed with red blood cell transfusions,
maintain the hemoglobin above 10 g/dL. This regimen

chain synthesis
2. Hemoglobin H di

eliminates an increased erythropoietic drive, allowing


and folate supplementation. Alpha
normal linear growth and bone development. Supsemia minor usually do not
pression of erythropoiesis also limits the stimulus
nt but may be mistaken for iron
for increased iron absorption, which helps to miniia.
mize iron overload. Splenectomy is considered when

major are treat

iron chelation,
and beta thalas
require treatme
deficiency anem

transfusional requirements exceed 250mL/kg/yr. Iron


overload develops in children with beta thalassemia,
whether they are transfused or not, due to hyperabsease
sorption of dietary iron. When the bone marrow
sease can result from chronic
storage capacity for iron is exceeded, iron accumulates
s, such as inflammatory bowel
in the liver, heart, pancreas, gonads, and skin, producrheumatoid arthritis; chronic
ing symptoms of hemochromatosis. As a result, many
tuberculosis; and malignancy.
thalassemic patients develop cardiomyopathy and
chronic disease is normocytic;
congestive heart failure in their late teens. To miniia of chronic disease have
mize the morbidity associated with iron overload,
of chronic disease results from

Anemia of Chronic Di
Anemia of chronic di
inflammatory disease
disease and juvenile
infections, such as
Typically, anemia of
25% of cases of anem
microcytosis. Anemia

--------------------------------------- 13
Chapter 2 / Poisoning, B
urns, and Injury Prevention
11
should be supervised while walking or playing near
al risk. Nuts account for over 50%
streets.
rations.

children at addition
of foreign body aspi
Differential Diagnos

is
Patients who do not
acutely obstruct their airways
Drowning is a frequent cause of morbidity and morweek after the initial event

may present up to a
with no witnessed ep

isode of choking. Wheezing and


tality in the pediatric population. Incidence peaks
may be mistaken for asthma;

respiratory distress

between 1 and 5 years and again in adolescence.


deration when breath sounds
Rates are twice as high in blacks and three times
higher in boys. Bathtubs are the most common site
te, findings on auscultation in
of drowning in the first year of life. Large buckets
y aspiration are localized to one

pneumonia is a consi
are decreased. Of no
cases of foreign bod
side of the chest on

ly.
and residential pools are particularly dangerous for
toddlers, whereas natural water sources account for
ions
most adolescent injuries. Reliable predictors of outcome include water temperature, time of submerdepending on where the foreign
sion, degree of pulmonary damage, and effectiveness
espiratory tree (Table 2-2). If the
of early resuscitation efforts.
ete, the chest radiograph demonSubmersion for more than 5 minutes in warm
one-sided atelectasis and the heart
water associated with significant aspiration and
affected lung throughout the
minimal response to initial cardiopulmonary resusciycle. However, a partial obstructation (CPR) virtually always results in major disnter during inspiration, and it
ability or death.
l-valve obstruction]. In these
Toddlers and young children must be supervised
ry film may appear normal, but the
at all times while in the bathtub or around pools or
on will show a hyperinfiated
other bodies of water. Residential and commercial
mediastinal shift away from the
swimming pools should be fenced in and have locked
).
gates. CPR training is available to parents through the
American Heart Association and many area hospitals.
Learning to swim is an important preventive measure
but does not take the place of close supervision.
FOREIGN BODY ASPIRATION,
The natural curiosity of children coupled with the
toddler's tendency to put everything in the mouth
make foreign body aspiration a frequent occurrence
in the pediatric population. Most objects and foodstuffs are immediately expelled from the trachea by
coughing. Unfortunately, foreign bodies that lodge
in the upper or lower respiratory tract are more
problematic.
Epidemiology
The highest incidence is noted in children 6 to
36 months old. Aspiration into the lower airways is
much more common than tracheal obstruction;
many objects lodge in the right main stem bronchus
ry film in foreign body aspiration with
because of bronchial anatomy. Inadequate supervi; the obstructed left lung is hyperinfiated,
sion and inappropriate food choices for age place
and mediastinum) are shifted to the right.

Clinical Manifestat
Presentation varies
body lodges in the r
obstruction is compl
strates significant
is drawn toward the
entire respiratory c
tion allows air to e
becomes trapped (bal
cases, the inspirato
x-ray after expirati
obstructed lung with
blockage (Figure 2-1

Figure 2-1

Expirato

partial obstruction
whereas the heart (

--------------------------------------- 14
110
Blueprints Pediatrics
an inability to mobilize iron from its storage sites in
f childhood, parvovirus B19macrophages. The chronic inflammatory state trigsis, and drug toxicity from myelogers cytokines that result in reticuloendothelial
A normocytic anemia also occurs
blockade within the marrow. A modest decrease in
s. Re-equilibration of the total
the survival time of red blood cells and a relatively
erythropoiesis results in the
limited erythropoietin response also contribute to
ammatory states result in anemia
the anemia.
which can be normocytic (75%)

erythroblastopenia o
induced aplastic cri
suppressive agents.
with acute blood los
blood volume before
anemia. Chronic infl
of chronic disease,
or microcytic (25%),

as discussed earlier.
Clinical Manifestations
The anemia is mild in degree (hemoglobin 8-10
stopenia
g/dL). The laboratory findings typical for anemia of
chronic disease are noted in Table 10-3. As in iron
deficiency anemia, the serum iron level is reduced;
stopenia of childhood (TEC) is
in contrast to iron deficiency anemia, the total ironcell aplasia caused by transient
binding capacity is low, and the serum ferritin level
ion. The resulting anemia is
is normal or increased. Bone marrow examination
a specific etiology has not been
shows micronormoblastic hyperplasia and an insually preceded by a viral infeccrease in storage iron, but a decrease in the number
ween the ages of 6 months and
of iron-containing erythroblasts.
incidence at 2 years of age. In

Transient Erythrobla
of Childhood
Transient erythrobla
an acquired pure red
bone marrow suppress
normocytic. Although
identified, TEC is u
tion. TEC occurs bet
5 years, with a peak
contrast to Diamond-

Blackfan syndrome, which is a


Treatment
c pure red cell aplasia, 85% of

congenital macrocyti
cases of TEC occur a

fter 1 year of age, there are no


The anemia resolves when the underlying condition
malies, and fetal hemoglobin and
is treated adequately. Therapy with iron supplements
esent.
is unnecessary unless true iron deficiency is also
present.

other associated ano


i antigen are not pr

Clinical Manifestati
ons
The history and phys
ical examination are unremarkKEY POINTS
ual onset of pallor over the

able except for grad


course of weeks. Per

ipheral smear is normal other


1. Anemia of chronic disease can result from chronic
ia. Bone marrow examination
inflammatory diseases, chronic infections, and
d precursors and normal myeloid

than reticulocytopen
reveals few erythroi
and platelet precurs

ors.
malignancy.
2. Typically, anemia of chronic disease is normocytic;
25% of cases of anemia of chronic disease have
microcytosis.
ually at its nadir at the time of
3. Anemia of chronic disease results from an inability
us recovery occurs within 1 to 2
to mobilize iron from its storage sites in
Red blood cell transfusions are
macrophages.
e patient has signs or symptoms

Treatment
The hemoglobin is us
diagnosis. Spontaneo
months of diagnosis.
necessary only if th
of congestive heart

failure.
NORMOCYTIC ANEMIAS WITH
DECREASED RED CELL PRpDUCTION
EY POINTS

K
1. Transient eryth

roblastopenia of childhood, a norNormocytic anemias result from the failure of the


caused by bone marrow suppresbone marrow to produce adequate numbers of red
uired pure red cell aplasia with a
blood cells due to systemic illness. Bone marrow
at 2 years of age.
function can be impaired by fibrosis, malignant inusually precedes TEC, but no spefiltration, transient marrow failure, or failure to
has been identified.
synthesize erythropoietin (chronic renal disease).
EC is spontaneous.
Transient marrow failure states include transient

mocytic anemia
sion, is an acq
peak incidence
2. Viral infection
cific etiology
3. Recovery from T

--------------------------------------- 15
Chapter 10 / Hematology
NORMOCYTIC ANEMIAS WITH
bodies are usually cold reac-

111
Coombs' test. IgM anti
tive (maximal activi

ty at low temperatures) and are


INCREASED RED CELL_PRODU_CTJON
deemed complete anti
bodies. They agglutinate red
Hemolytic Anemia
vate the complement sequence

blood cells and acti


through C9, causing

lysis of red blood cells. HemolNormocytic anemias with increased red cell produccularly. IgM antibodies are assocition are most commonly caused by hemolytic anemias.

ysis occurs intravas


ated with M. pneumon

iae, Epstein-Barr virus, and


The red blood cell destruction and anemia are sensed
transfusion reaction
s.
by the kidneys, which release erythropoietin to stimuic anemias can be microanlate bone marrow erythropoiesis. Hemolytic anemias
ted intravascular coagulation,
are caused by factors extrinsic to the red cell (extratopenic purpura, hemolytic
corpuscular) or by defects intrinsic to the red cell
ignant hypertension, giant
(intracorpuscular). In general, extrinsic defects are acquired and intrinsic defects are hereditary.
psia, renal graft rejection) or

Nonimmune hemolyt
giopathic (dissemina
thrombotic thrombocy
uremic syndrome, mal

hemangioma, preeclam
can be due to damage

from nonendothelialized surExtracorpuscular anomalies are divided into


art valve, arteriovenous malformaisoimmune, autoimmune, and nonimmune hemolytic
tt syndrome), hypersplenism,
anemias. Isoimmune hemolytic anemia results from
, toxins (snake venom, copper,
antibodies produced by one individual against the
r burns.
red blood cells of another individual of the same

faces (artificial he
tion, Kasabach-Merri
abetalipoproteinemia
arsenic), malaria, o

Intracorpuscular
defects include intrinsic memspecies. ABO or minor antigen incompatibility is an
s hereditary spherocytosis, heredexample of isoimmune hemolytic anemia (see
, hereditary stomatocytosis, and
Chapter 13). In autoimmune hemolytic anemia,
hemoglobinuria (PNH). PNH
abnormal antibodies directed against red blood
puscular disorder that is not inhercells are produced by the patient. Autoimmune

brane defects such a


itary elliptocytosis
paroxysmal nocturnal
is the only intracor

ited. Hemoglobinopat
hies (sickle cell disorders) and
hemolytic anemias can be idiopathic, postinfectious
PD deficiency, pyruvate kinase
(Mycoplasma pneumoniae, Epstein-Barr virus), drugintracorpuscular disorders. Folinduced (penicillin, quinidine, alpha-methyldopa),
ns of hereditary spherocytosis,
or may result from a chronic autoimmune disease
and G6PD deficiency, three of the
(systemic lupus erythematosus) or malignancy
(non-Hodgkin's lymphoma). Therapy for autoimpuscular defects.
mune hemolytic anemia varies depending on the etiology of the hemolysis and the clinical condition of
EY POINTS
the patient. In general, treatment is supportive, with

enzyme disorders (G6


deficiency) are also
lowing are discussio
sickle cell anemia,

most common intracor

K
1. Normocytic anem

ias with increased red cell prothe careful use of packed red blood cell transfusions
and corticosteroids. Autoantibodies react with
t commonly caused by hemolytic

duction are mos


anemias.

virtually all red blood cells, making crossmatching


as are caused by factors extrinsic
difficult. In some severe chronic cases, intravenous
or by defects intrinsic to the red
immunoglobulin, immunosuppressive pharmaal, extracorpuscular defects are
cotherapy, and splenectomy may be indicated.
The antibodies that cause isoimmune and autointracorpuscular defects are

2. Hemolytic anemi
to the red cell
cell. In gener

acquired, and
hereditary.

immune hemolytic anemias may be of the IgG or


r anomalies are divided into
IgM classes. IgG antibodies tend to be warm reacoimmune, and nonimmune
tive (maximal activity at 37C) and are considered
s.
incomplete antibodies. They coat the surface of the

3. Extracorpuscula

red blood cells and fix early complement compor defects include intrinsic

4. Intracorpuscula

isoimmune, aut
hemolytic anemia

membrane defec
ts, hemoglobinopathies, and
nents but cannot agglutinate red blood cells or acti-

enzymopathies.

vate the complement cascade through the entire


hemolytic sequence. Hemolysis occurs extravascularly because of trapping of the opsonized red blood
cells by macrophages in the reticuloendothelial
tosis
system. IgG antibodies are associated with autoimtosis is caused by a defect in red
mune diseases, lymphomas, and viral infections.
supporting proteins (spectrin,
These antibodies are identified by the direct
protein). The defect leads to a loss

Hereditary Spherocy
Hereditary spherocy
blood cell membrane
ankyrin, or band 3

--------------------------------------- 16
112

rinte Pediatrics

of membrane fragments without a proportional loss

Sickle Cell Disease

of volume. Therefore, microspherocytes (small spherical red blood cells with a high volume-to-surface

Pathogenesis

ratio) form. Microspherocytes are less deformable


is an autosomal recessive disorder
than normal red blood cells, so they are trapped and
valine-for-glutamine substitution

Sickle cell disease


that results from a
in the sixth amino a

cid position of the beta (i-globin


destroyed in the microvasculature of the spleen. Inhertion alters the structure of the
itance is usually autosomal dominant, but 25% of cases
are due to new mutations or autosomal recessive forms.
which, under conditions of
Clinical Manifestations
tes aggregation of hemoglobin

chain. This substitu

hemoglobin molecule,
deoxygenation, promo
into long polymers t

hat distort the red blood cell into


Hereditary spherocytosis varies greatly in clinical
ling shortens red blood cell surseverity, ranging from an asymptomatic, wellts in a chronic hemolytic anemia.
compensated, mild hemolytic anemia discovered
ause microvascular obstruction,
incidentally to a severe hemolytic anemia with
e ischemia and infarction. The
growth failure, splenomegaly, and chronic transfuis accentuated by hypoxia, acision requirements in infancy necessitating early
decreased temperature, and dehysplenectomy. The newborn with this disorder may
of the two (3-globin genes is
present with severe unconjugated hyperbilirubinedual has sickle cell trait, which is
mia caused by hemolysis. Infants and children may
te without clinical consequence.
present with pallor and splenomegaly. Occasionally,
es have the genetic substitution,
patients may present with aplastic crisis after parygous for hemoglobin S and
vovirus B19 infection. Because of chronic hemolysis,
ase. Sickling disorders of varying
teenagers develop gallstones and cholecystitis. Physfrom hemoglobin S existing in
ical examination reveals pallor, scleral icterus, and
er abnormal hemoglobins
mild to moderate splenomegaly. Laboratory studies
LosAngeies, OArab) or thalassemias (B +
demonstrate mild normocytic anemia, reticulocyto-

a sickle shape. Sick


vival time and resul
Sickled cells also c
which leads to tissu
sickling phenomenon
dosis, increased or
dration. If only one
affected, the indivi
the heterozygous sta
If both p-globin gen
the patient is homoz
has sickle cell dise
severity also result
combination with oth
(hemoglobin C, D
or BO thalassemia).

sis, and indirect hyperbilirubinemia. During an


aplastic crisis, the anemia becomes severe and

Epidemiology

reticulocytopenia occurs. Diagnosis is confirmed by a


affects 1 in 625 African Americans,
positive osmotic fragility test.
ommon autosomal recessive dis-

Sickle cell disease


making it the most c
order in that popula

tion. It also occurs in those of


Treatment
Saudi Arabian descent.

Greek, Italian, and

Treatment includes folic acid supplementation to


meet the needs of increased red blood cell turnover
ons and Management
and red blood cell transfusions during an aplastic

Clinical Manifestati

crisis. Splenectomy alleviates anemia, reticulocytosis,


cell trait are generally asympto-

Children with sickle


matic. Rarely, an in

dividual will exhibit painless


and scleral icterus, although microspherocytes
ability to properly concentrate
persist. Splenectomy should be deferred until after 6
ria). Patients with sickle cell trait
years of age because of the higher the risk of sepsis
ckle cells on peripheral blood
from encapsulated organisms in young children.
n electrophoresis provides

hematuria and the in


the urine (isosthenu
occasionally have si
smear, but hemoglobi
the definitive diagn

osis. Typically, hemoglobin elecKEY POINTS


55% to 60% hemoglobin A, 40%

trophoresis reveals
to 45% hemoglobin S,

and 2% to 3% hemoglobin
1. Hereditary spherocytosis is caused by a defect in
to detect the trait for genetic
the major supporting proteins of the red blood
cell membrane.
l trait, sickle cell disease causes
2. The defect leads to a loss of membrane fragments
mortality. Quantitative hemoand the formation of rigid microspherocytes,
is shows 0% hemoglobin A, 85%
which are prone to hemolysis.
2% to 3% hemoglobin A
2, and
3. Diagnosis is confirmed by a positive osmotic
F. In most cases, diagnosis
fragility test.
screening tests. The highly

A 2. It is important
counselling.
Unlike sickle cel
severe morbidity and
globin electrophores
to 95% hemoglobin S,
up to 15% hemoglobin
is made from newborn
variable clinical ma

nifestations of sickle cell disease

--------------------------------------- 17
Chapter 10 / Hematology
TABLE 10-5

113

Clinical Manifestations of Sickle Cell Anemia*


Manifestation Comments
Anemia
therapy for chronic hemolysis
Aplastic crisis

Chronic, onset 3-4 mo of age; requires folate


Parvovirus infection, acute and self-resolvin

g
Sequestration crisis

Massive splenomegaly, shock; treat with trans

fusion
Hemolytic crisis
Dactylitis

May be associated with G6PD deficiency


Hand-foot swelling in early infancy

Pain crisis
Microvascular painful vasoocclusive infarcts
of muscle, bone, lung, intestines
Cerebral vascular accidents
Large- and small-vessel sickling and thrombos
is (stroke); requires chronic transfusion
Acute chest syndrome
Infection and/or infarction, severe hypoxemia
, infiltrate, dyspnea, rales
Chronic lung disease
Pulmonary fibrosis, restrictive lung disease,
cor pulmonale
Priapism
Causes eventual impotence; treat with transfu
sion, oxygen, or corpora cavernosato-spongiosa shunt
Ocular

Retinopathy

Gallbladder disease

Bilirubin stones; cholecystitis

Renal
ating deficit; nephropathy
Cardiomyopathy

Hematuria, papillary necrosis, renal-concentr

Infections
ve infection due to encapsulated

Functional asplenia; increased risk of invasi

Heart failure (fibrosis)

bacteria, such as 5. pneumoniae, H. influe


nzae, and N. meningitidis; Salmonella and
Staphylococcus aureus osteomyelitis; severe
Mycoplasma pneumonia; transfusionacquired infections
Growth failure, delayed puberty May respond to nutritional supplements
* Clinical manifestations with sickle cell trait are unusual but include renal
papillary necrosis (hematuria), sudden death on exertion, intraocular hyphema
extension, and sickling in unpressurized airplanes.
CMV, cytomegalovirus; EBV, Epstein-Barr virus; G6PD, glucose-6-phosphate deh
ydrogenase.
result from anemia, infection, and vasoocclusion
Viral suppression of
red blood cell precursors in the
(Table 10-5).
bone marrow, most of
ten by parvovirus B19, precipAt approximately 4 months of age, when the peritates aplastic cris
is. Exposure of a patient with sickle
centage of hemoglobin F diminishes and the percell disease and con
comitant G6PD deficiency to
centage of hemoglobin S rises, the child with sicklae
n oxidative stress
results in acute hemolysis supercell disease develops a progressive hemolytic anemia.
imposed on a chronic
hemolytic anemia (hyperheThe anemia of sickle cell disease is a chronic, wellmolytic crisis]. Me
dications or infection usually cause
compensated, severe anemia that is rarely transfusion
the acute hemolysis.
Splenic sequestration, aplastic
dependent. Common manifestations of the anemia
crisis, and hyperhem
olytic crisis are often treated
include pallor, jaundice, splenomegaly in infancy, a
with red blood cell

transfusion. Because of the


systolic ejection murmur, and delayed sexual develpresence of chroni
c hemolytic anemia, gallstone
opment and growth. Splenic sequestration, aplastic
formation and cholec
ystitis are common during
crisis, and hyperhemolytic crisis all superimpose
adolescence.
acute life-threatening declines in hemoglobin conAs the sickled ce
lls traverse the spleen, they cause
centration on the chronic compensated anemia omicrovasculaf
r obs
truction, infarction, and fibrosis of
sickle cell disease. In splenic sequestration, rapid
the spleen. This pro
cess, known as autoinfarction,
splenic engorgement caused by trapping of red bloocaused
s the spleen t
o gradually regress in size; by the
cells may lead to hypovolemic shock. Sequestration
age of 4, the spleen
is no longer palpable. More
typically occurs between 6 months and 2 years of ageimportant.
, autoinf
arction diminishes the capability

--------------------------------------- 18
114
Blueprints Pediatrics
of the spleen to filter encapsulated bacterial organsion protocols to mi
nimize the risk of future stroke.
isms and places the infant at great risk for overPriapism most typica
lly occurs in boys between 6 and
whelming infection from Streptococcus pneumoniae o2r
0 years of age. The
child develops sudden painful
Haemophilus influenzas. Any infant or child who haengorgemens
t of t
he penis that will not subside. Acute
sickle cell disease and fever (temperature greater
chest syndrome, stro
ke, and priapism are treated by
than 38.5C) must be evaluated immediately. Alexchange transfusion t
o decrease the percentage of
though the child likely has a benign viral infectionhemoglobi,
n S to b
elow 30% in an attempt to miniinvasive bacterial infection must be excluded. To
mize vasoocclusion.
minimize the risk of life-threatening infection, chilBy adolescence, t
he effects of chronic myocardial
dren with sickle cell disease start penicillin prophyrnicrovascular obs
truction and infarction are evident
laxis at approximately 4 months of age and receive
by an enlarged hyper
trophic heart. Many adults
vaccinations. Both the H. influenzae type b (Hib) aneventualld
y succumb
to congestive heart failure from
heptavalent pneumococcal conjugate (Prevnar) vacprogressive myocardi
al damage. Abdominal crisis
cines are given at the 2-, 4-, and 6-month visits and
results from rnicrov
ascular obstruction of the intestithen again between 12 months and 15 months of agena.
l circulation. Pa
tients present with ileus and
The 23-valent pneumococcal polysaccharide vaccine
rebound tenderness,
mimicking an acute abdomen.
(PPV) should be administered at 2 years of age and
The pain may be fami
liar to the patient and readily
then again at 4 to 6 years of age. Penicillin prophyrecognized as "crisi
s pain." Abdominal pain consistent

laxis is continued until at least 5 years of age.


with the child's nor
mal pain constellation during
Vasoocclusive crises result from rnicrovascular
crisis may warrant a
period of observation with
infarcts, may occur in any organ or tissue of the body,
hydration and analge
sic administration. If the aband are commonly precipitated by infection, cold
dominal pain is not
typical for the patient during
exposure, dehydration, venous stasis, and acidosisvasoocclusiv.
e cris
is, surgical consultation should be
Dactylitis, or hand-foot syndrome, is symmetrical
obtained.
painful swelling of the dorsal surface of the hands
and feet caused by avascular necrosis of the
metacarpal and metatarsal bones. Dactylitis occurs at
KE
Y POINTS
4 to 6 months of age and is the earliest clinical man1. Sickle cell dis
ease is an autosomal recessive disorifestation of vasoocclusive disease in the sickle cell
der that result
s from an amino acid substitution
patient. In older children, pain crises most often
on the (i-globi
n chain.This substitution results in
localize to the long bones of the arms, legs, vertebral
an alteration o
f the structure of the hemoglobin
column, and sternum. Pain crises last from 2 to 7 days
molecule, which
, under conditions of deoxygenaand are treated with nonsteroidal anti-inflammatory
tion, promotes
aggregation of hemoglobin into
drugs and narcotics. Hydroxyurea maintenance
long polymers t
hat distort the red blood cell into
therapy decreases the number and severity of vasooca sickle shape.
clusive crises. Avascular necrosis of the femoral heads,
2. Sickling shorte
ns red blood cell survival time and
another vasoocclusive manifestation in bone, typiresults in a ch
ronic hemolytic anemia.
cally occurs in the adolescent population.
3. The clinical ma
nifestations of sickle cell anemia
Microvascular obstructive disease can also occur
result from ane
mia, infection, and vasoocclusion.
in the lungs, central nervous system, penis,
myocardium, and intestine. Acute chest syndrome, a
vasoocclusive crisis within the lungs, is often caused
Glucose-6-Phosphate
by pulmonary infection and infarction. Patients
present with hypoxia, respiratory distress, and pulDehydrogenase Defici
ency
monary infiltrates. Oxygen, analgesia, antibiotics, and
G6PD deficiency, the
most common red blood cell
exchange transfusion are used to maximize respiraenzyme defect, is tr
ansmitted as an X-linked recestory status and minimize further pulmonary damage.
sive trait. The lack
of this enzyme in the hexose
Similarly, occlusion of the large cerebral vessels
monophosphate shunt
pathway results in depletion
results in stroke. Patients present with mental status
of nicotinamide aden
ine dinucleotide phosphate
changes, seizures, and focal paralysis. Because of the
(NADPH) and the inab
ility to regenerate reduced
high risk of recurrence, children who have had a
glutathione, which i

s needed to protect the red blood


stroke are placed on chronic red blood cell transfustress.

cell from oxidative

--------------------------------------- 19
Chapter 10 / Hematology
115
The most common forms of G6PD deficiency are
marrow to compensate, so the
the A" and Mediterranean variants. The A~ variant,
ay be low for the first 3 to 4
found in approximately 10% of African Americans in
the United States, is associated with an isoenzyme
G6PD deficiency is made by
that deteriorates rapidly, with a half-life of 13 days.
DPH formation on G6PD assay.
The Mediterranean variant occurs predominantly in
ormal in the setting of acute,
persons of Greek and Italian descent; its isoenzymsevere
cause the most deficient cells
is extremely unstable, with a half-life of several
Repeating the test at a later
hours.
t is in a steady-state condition,
When there is an oxidative stress on the red blootestind
f males with suspected G6PD
cell, exposed sulfhydryl groups on the hemoglobideficiencyn
rforming electrophoresis to identify
are oxidized, leading to dissociation of heme and
facilitate diagnosis.
globin moieties, with the globin precipitating as

ability of the bone


reticulocyte count m
days.
The diagnosis of
finding deficient NA
G6PD levels may be n
e hemolysis be
have been destroyed.
time when the patien
g the mother o
, and pe
the precise variant

Heinz bodies. Damaged red cells are removed

Treatment

from circulation by the reticuloendothelial system;


eficiency associated with acute
severely damaged cells may lyse intravascularlysever.
ed to avoid drugs that initiate
Known oxidants include sulfonamides, nitrofuranis supportive, including packed
toin, primaquine, and dimercaprol. Hemolysis may
fusion during significant cardioalso be precipitated by fava beans and infection.
and vigorous hydration and

Patients with G6PD d


e hemolysis ne
hemolysis. Treatment
red blood cell trans
vascular compromise
urine alkalinization

to protect the kidneys against


Clinical Manifestations
ated free hemoglobin.
The classic course of G6PD deficiency is episodic

damage from precipit

stress- or drug-induced hemolytic anemia. Patients


with the A" variant have a limited hemolysis confined
EY POINTS
to the older red blood cell population. Recovery
hate dehydrogenase deficiency,

K
1. Glucose-6-phosp

occurs as young red blood cells with enzyme activred blood cell enzyme defect,
ity sufficient to resist oxidative stress emerge from

the most common

the bone marrow. Hemolysis is most common in


as an X-linked recessive trait.
males who possess a single abnormal X chromosome.
s enzyme in the hexose monophosHeterozygous females who have randomly inactihway results in a depletion of
vated a higher percentage of the normal gene may
ability to regenerate reduced
become symptomatic, as may homozygous females
ich is needed to protect the red
with the A~ variant. One percent of Africanoxidative stress.
American females are A" variant homozygous.

is transmitted
2. The lack of thi
phate shunt pat
NADPH and an in
glutathione, wh
blood cell from

Patients with the Mediterranean variant have hemolysis that destroys most of their red cells and may
require transfusions until the drug is eliminated from
WITH
their bodies. The neutrophils of patients with the
_PR_OD_UCTION
most severe degrees of G6PD deficiency demonstrate
defective oxidative killing because of the depletion
re subdivided according to the
of NADPH, which serves as an electron donor to thpresence
e of megaloblastosis, a marker of
membrane-bound oxidase that produces bactericidaineffectivl
thesis within a red blood cell
oxygen species.
megaloblastic anemia include
On peripheral blood smear, the red cells appear tvitamio
late deficiency, drugs that interfere
have "bites" taken out of them (blister cells). The
sm (phenytoin, methotrexate,
bitten areas result from phagocytosis of Heinz bodies
etabolic disorders (orotic
by splenic macrophages. During hemolytic episodesaciduria,
lonic aciduria, Lesch-Nyhan synphysical examination reveals jaundice and dark
nemias without megaloblastourine (caused by hemoglobinuria and high levels osif
marrow failure and include bone
urobilinogen). Laboratory tests reveal elevated inomes (Diamond-Blackfan syndirect bilirubin and lactate dehydrogenase and low
emia, idiopathic aplastic anemia,
haptoglobin. Initially, the hemolysis exceeds the
nduced anemias (azidothymi-

--------------------------------------- 20
116

sprints Pediatrics

MACROCYTIC ANEMIAS
DECREASED RED CELL

Macrocytic anemias a
e or absenc
e DNA syn
precursor. Causes of
n B 12 and fo
with folate metaboli
trimethoprim), and m
, methylma
drome). Macrocytic a
s result from bone
marrow failure syndr
drome, Fanconi's an
preleukemia), drug-i

dine, valproic acid, carbamazepine), chronic liver


f urinary excretion is minimal, an
disease, and hypothyroidism.
ic factor is given. Normal urinary

urinary excretion. I
oral dose of intrins
excretion after intr

insic factor confirms the diagnosis


of pernicious anemia
. Inadequate urinary excretion
Megaloblastic Macrocytic Anemias
or suggests bacterial overgrowth.

after intrinsic fact


Antibiotics are give

n, and if vitamin B,
Vitamin B 12 Deficiency

2 urinary

Vitamin 812, coenzyme for 5-methyl-tetrahydrofoases, the patient has bacterial


late formation, is needed for DNA synthesis. It is

excretion then incre

found in meat, fish, cheese, and eggs. Dietary vitamin

Treatment

overgrowth.

B]2 deficiency is rare in developed countries except


in the breast-fed infant whose mother is a vegan with
orms of vitamin B
12 deficiency,
poor attention to dietary sources of vitamin B
]2.
f bacterial overgrowth and fish
Another cause of vitamin B
12 deficiency is selective
intramuscular vitamin B
12.The
or generalized malabsorption. Vitamin B
]2 combines
nse is rapid, with marrow mega-

Treatment for most f


with the exception o
tapeworm, is monthly
erythropoietic respo
loblastosis improvin

g within hours, reticulocytosis


with intrinsic factor, which is produced by gastric
parietal cells, and is absorbed in the terminal ileum.
appearing by day 3 o
f therapy, and anemia resolving
Transcobalamin II then transports vitamin B
12 to the within 1 to 2 months
.
liver for storage. The availability of vitamin B
12 is
reduced by any condition that alters intrinsic factor
Folate Deficiency
production, interferes with intestinal absorption,
iver, green vegetables, cereals, and
or reduces transcobalamin II levels. Disorders such
ted to tetrahydrofolate, which
as congenital pernicious anemia (absent intrinsic

Folate is found in l

factor), juvenile pernicious anemia (autoimmune


synthesis. Because folate stores
destruction of intrinsic factor), and transcobalamin II
, deficiency may develop within

is required for DNA

cheese and is conver

are relatively small


1 month and anemia w

ithin 4 months of deprivadeficiency result in vitamin B


!2 deficiency. Other
lude inadequate dietary intake,
causes include ileal resection, small bowel bacterial

tion. Etiologies inc


impaired absorption

of folate, increased demand for


overgrowth, and infection with the fish tapeworm
folate metabolism. Dietary defiDiphyllobothrium latum.

folate, and abnormal

ciency of folic acid


is unusual in developed countries.
Clinical Manifestations
infants fed goat's milk, evapoThe effects of vitamin B ]2 deficiency include glossisterilized milk or formula; each
tis, diarrhea, and weight loss. Neurologic sequelae
e content. Malabsorptive states
include paresthesias, peripheral neuropathies, and, in
as inflammatory bowel disease
the most severe cases, dementia, ataxia, and/or posn cause folate deficiency.
terior column spinal degeneration. Vitiligo is the
folate occurs with an increased
main dermatologic manifestation.
ll turnover (hyperthyroidism,
Megaloblastic changes on peripheral blood smear
emolysis, malignancy). Relative
include ovalocytosis, neutrophils with hypersegy develop if the diet does not
mented nuclei (more than four per cell), nucleated
ate to meet these needs. Cerred blood cells, basophilic stippling, and Howell-Jolly
drugs (phenytoin, phenobarbital)
bodies. The mean corpuscular volume is usually
e metabolism.
greater than lOOfL. Intramarrow hemolysis, also

Children at risk are


rated milk, or heathas inadequate folat
of the jejunum, such
and celiac sprue, ca
Increased demand for
rate of red blood ce
pregnancy, chronic h
folate deficiency ma
provide adequate fol
tain anticonvulsant
interfere with folat

known as ineffective erythropoiesis, results in eleClinical Manifestati


ons
vated levels of serum lactate dehydrogenase, indirecSpecifit
c symptoms a
re often absent, although pallor,
bilirubin, and serum iron. In severe cases, megaglossitis, malaise,
anorexia, poor growth, and reloblastic anemia may be accompanied by leukopenia
current infection ma
y be seen. Unlike vitamin B
]2
and thrombocytopenia.
deficiency, neurolog
ic disease is not associated with
Diagnosis is confirmed by a subnormal serum levefolatl
e deficiency. L
aboratory findings include low red
of vitamin B] 2. In nondietary deficiency, the Schilling
blood cell folate an
d normal serum vitamin B
12 levels.
test helps delineate pernicious anemia from bacterial
Megaloblastic change
s on peripheral blood smear and
overgrowth. In this test, an oral dose of radiolabeled
bone marrow aspirate
are the same as those noted
vitamin B I2 is given, and its absorption is checked by
with vitamin B ]2
deficiency.

--------------------------------------- 21
Chapter 10 / Hematology

117

Treatment
in F, and fetal i antigen is
It is imperative not to misdiagnose B
12 deficiency as
ells. Twenty-five percent of
folate deficiency, because treatment with folate may
ated congenital anomalies that
result in hematologic improvement and allow foincludr
re, web neck, cleft lip, shield chest,
progressive neurologic deterioration. Treatment with
umb. These children are at high
1 mg of folate given orally each day for 1 to 2 monthriss
ter in life.
will treat the anemia and replenish body stores. Clini-

an elevated hemoglob
present on the red c
patients have associ
e short statu
and triphalangeal th
k for leukemia la

Treatment
cal response is rapid, following a time course similar
of patients respond to high-dose
to that of vitamin B )2 replacement therapy. Children
py but must receive therapy
with chronic hemolytic conditions require continued

Seventy-five percent

folate supplementation.
who do not respond to steroid

indefinitely. Those

corticosteroid thera

treatment are transf


usion dependent and are at risk
for the complication
s of iron overload.
KEY POINTS
Anemia
1. Megaloblastic macrocytic anemias reflect ineffecanemia is an acquired failure of
tive DMA synthesis and can result from vitamin
em cells that results in panB12 and folate deficiency, drugs that interfere with
der may result from exposure
folate metabolism, and some rare metabolic
e, phenylbutazone), drugs
disorders.
lfonamides), infectious agents
2. Vitamin B12 is a coenzyme needed for DNA synr ionizing radiation. Often an etithesis. Dietary vitamin B12 deficiency is rare in
identified, and the case is classideveloped countries, because vitamin B 12 stores

Idiopathic Aplastic
Idiopathic aplastic
the hematopoietic st
cytopenia. The disor
to chemicals [benzen
(chloramphenicol, su
(hepatitis virus), o
ologic agent is not
fied as idiopathic.

are large.The usual cause of vitamin B12 deficiency


is malabsorption.
ons
3. Folate is converted to tetrahydrofolate, which is
r from pancytopenia, and bone
required for DNA synthesis. Because folate stores
als a hypocellular marrow.
are relatively small, deficiency may develop

Clinical Manifestati
These patients suffe
marrow aspirate reve

Treatment
within 1 month and anemia within 4 months of
ilymphocyte globulin is tem-

Antithymocyte or ant

deprivation.
but serum sickness is a nearly
4. Neurologic sequelae of vitamin B 12 deficiency
t and relapse is common. Highinclude paresthesias, peripheral neuropathies,
are often used in combination
and, in the most severe cases, dementia, ataxia,
lobulin. Cyclosporin A has been
and posterior column spinal degeneration.
ses, but hepatic dysfunction,
5. Misdiagnosis and treatment of vitamin B 12 defiand immunosuppression limit
ciency as folate deficiency may result in hematologic improvement while allowing progressive
marrow transplantation is the

porarily effective,
universal side effec
dose corticosteroids
with antithymocyte g
effective in some ca
renal insufficiency,

its usefulness. Bone


sole effective treat

ment; without transplantation,


neurologic deterioration.
within 3 months of diagnosis.

80% of patients die


If transplantation i

s being considered, it is important


to minimize transfus
ions to reduce exposure to
potentially sensitiz
ing blood products. Neutropenic
Nonmegaloblastic Macrocytic Anemias
for serious bacterial infection

patients are at risk


and usually require

antibiotics when they develop


Diamond-Blackfan Syndrome

fever.

Diamond-Blackfan syndrome is an autosomal recessive, pure red cell aplasia of unknown etiology.
Fanconi's Anemia
Clinical Manifestations
Fanconi's anemia is
an autosomal recessive disorder
The anemia develops shortly after birth but is not
that results in panc
ytopenia. Commonly associated
usually detected until later, when symptoms developcondition;
s include
pigmentary changes and skeletal,
90% of cases are observed within the first year of liferenal.
, and developm
ental abnormalities. The disorder
Infants present with mild macrocytosis and reticuloresults from defect
ive DNA repair mechanisms that
cytopenia. On hemoglobin electrophoresis, there ileas
d to excessive c
hromosomal breaks and recombi-

--------------------------------------- 22
118

rints Pediatrics

nations. These chromosomal anomalies are found in


ASIIS
all cells of the body, not just in hematopoietic stern

_.DISORDERS pF HEMpST

cells. The mean age at onset of pancytopenia is 8


quires the integrity of the
years, and it almost always occurs before age 10.
lets, and soluble clotting factors.
Clinical Manifestations
s can result from abnormal
Common signs include hyperpigmentation and cafe
ation, which occurs in platelet
au lait spots, microcephaly, microphthalmia, short
clot formation, which is noted in

Normal hemostasis re
blood vessels, plate
Bleeding derangement
hemostatic plug form
disorders; aberrant
defects of the coagu

lation cascade; or with vascular


stature, horseshoe or absent kidney, and absent
thumbs. Hematologic manifestations include prolar anomalies that result in
gressive pancytopenia. Macrocytosis is universal
editary defects of collagen syneven before the onset of anemia, and hemoglobin F
s syndrome), acquired disorders
is seen on hemoglobin electrophoresis. Approxion (vitamin C deficiency,
mately 10% of children with Fanconi's anemia will
tis (Henoch-Schonlein purpura,
develop leukemia during adolescence.
ciated with abdominal pain,
Diagnosis is confirmed by demonstrating increased
chromosomal breakage with exposure to diepoxybu, and purpura, classically distribtane or other agents that damage DNA.
ks and lower extremities.
Treatment

abnormalities.
Examples of vascu
bleeding include her
thesis (Ehlers-Danlo
of collagen producti
scurvy), and vasculi
or HSP). HSP is asso

arthritis, nephritis
uted over the buttoc

Patients frequently require red blood cell transfusions


Platelet Disorders
and antibiotics to treat anemia and infections. Some
patients transiently respond to androgens. Corticoan be either quantitative or qualsteroids are often given with the androgens to
n abnormal hemostatic plug forcounterbalance androgen-induced growth acceleraabnormalities are detected by
tion. Bone marrow transplantation is the treatment
r platelet estimate on peripheral
of choice if an HLA-matched donor can be found.
qualitative disorders are deBecause of chromosomal sensitivity, the preparative
ng time or platelet aggregation
radiation and chemotherapeutic regimen must be
penia, defined as a platelet
modified because normal protocols result in severe
mm
3, is the most common
morbidity and mortality.
eeding. A low platelet count can

Platelet disorders c
itative and result i
mation. Quantitative
the platelet count o
blood smear, whereas
tected by the bleedi
studies. Thrombocyto
count below 150,000/
cause of abnormal bl
result from inadequa

te production or increased
destruction of plate
lets. Platelet production is evaluated by assessing th
e number of megakaryocytes in
the bone marrow aspi
rate.
KEY POINTS
t production can result from
1. Macrocytic anemias without megaloblastosis
marrow or bone marrow supresult from bone marrow failure and include bone
w failure states causing thrommarrow failure syndromes (Diamond-Blackfan
disorders causing pancytopenia
syndrome, Fanconi's anemia, idiopathic aplastic
diopathic aplastic anemia,
anemia, preleukemia), drug-induced anemias,
topenia-absent radius (TAR)
chronic liver disease, and hypothyroidism.
t-Aldrich syndrome. TAR syn2. Diamond-Blackfan syndrome is an autosomal
congenital megakaryocytic
recessive pure red cell aplasia. Associated anomtosomal recessive disorder in
alies include short stature, web neck, cleft lip,
ia develops in the first few
shield chest, and triphalangeal thumb.
hen resolves spontaneously after
3. Idiopathic aplastic anemia is an acquired failure
sient leukocytosis is common and
of the hematopoietic stem cells that results in
mia. Deformity of the radii is
pancytopenia.
tt-Aldrich syndrome is an X4. Fanconi's anemia is an autosomal recessive disoracterized by hypogammaglobuder that results in pancytopenia and pigmentary,
thrombocytopenia. Bone
skeletal, renal, and developmental abnormalities.
on is curative. Etiologies of

Decreased platele
failure of the bone
pression. Bone marro
bocytopenia include
(Fanconi's anemia, i
leukemia), thrombocy
syndrome, and Wiskot
drome, also known as
hypoplasia, is an au
which thrombocytopen
months of life and t
1 year of age. Tran
often suggests leuke
pathognomonic. Wisko
linked disorder char
linemia, eczema, and
marrow transplantati
thrombocytopenia cau

sed by bone marrow suppres-

--------------------------------------- 23
Chapter 10 / Hematology
119
sion include chemotherapeutic agents; acquired viral
O157:H7, that bind to endothelial
infections (HIV, Epstein-Barr virus, measles); conause of endothelial cell injury,
genital infections, including toxoplasmosis, syphilis,
lotting and platelet activation.
rubella, cytomegalovirus, and parvovirus B19; and
olytic anemia results from
certain drugs (anticonvulsants, sulfonamides, quinired cells as they pass through

as Escherichia coll
cells cause HUS. Bec
there is localized c
Microangiopathic hem
mechanical injury to

dine, quinine, thiazide diuretics). Acquired postnatal


endothelium, and thrombocyinfections, with the exception of HIV, and drug reacplatelet adhesion to the
tions usually cause transient thrombocytopenia,
Sixty percent to 80% of
whereas congenital infections may produce proansiently require dialysis. Most
longed suppression of bone marrow function.
acute phase and recover normal
Thrombocytopenia caused by shortened platelet
TP, platelet consumption precipsurvival is much more common than thrombocyactor or the lack of an inhibitory
topenia caused by inadequate production. Platelet
the primary process. There is
destruction is most commonly immune mediated.
of fibrin, which causes red cell
Thrombocytopenia in the newborn can result from

the injured vascular

isoimmune or autoimmune antibodies. Isoimmune


et survival can also result from
IgG antibodies are produced against the fetal
s seen with giant hemangiomas
platelets when the fetal platelet crosses the placenta
ypersplenism most commonly
and presents itself to the maternal immune system.
sickle cell anemia, thalassemia
If there is an antigen on the fetal platelet that does
disease, and portal hypernot exist on the maternal platelet, it will be recoglists the common causes of
nized as foreign and isoimmune antibodies will be
ing the neonatal, infant, and
created against the antigen. Maternal antiplatelet

Diminished platel

topenia results from


damaged endothelium.
patients with HUS tr
children survive the
renal function. In T
itated by a plasma f
factor appears to be
moderate deposition
destruction.

platelet trapping, a
and hypersplenism. H
occurs secondary to
syndromes, Gaucher's
tension. Table 10-6
thrombocytopenia dur
childhood periods.

antibodies then cross the placenta, causing destruction of the fetal platelet. This disorder is known as
neonatal isoimmune thrombocytopenic purpura.
The maternal antiplatelet antibody does not produce
Y POINTS
maternal thrombocytopenia. Autoimmune IgG
atic plug formation occurs in

KE
1. Abnormal hemost
platelet disord

ers.
antibodies are transferred to the fetus through the
ers can be either quantitative or
placenta when the mother has idiopathic thrombod thrombocytopenia is the most
cytopenic purpura, systemic lupus erythematosus, or
abnormal bleeding.
drug-induced thrombocytopenia. In all three cases,
a caused by shortened platelet
maternal autoantibodies cross the placenta and
h more common than thrombocyattack the fetal platelets. In contrast to isoimmune
by inadequate production and is

2. Platelet disord
qualitative, an
common cause of
3. Thrombocytopeni
survival is muc
topenia caused

antibodies, autoimmune antibodies also result in


e antibodies, autoimmune antimaternal thrombocytopenia. After birth, infants with
roangiopathic hemolytic anemias.
severe isoimmune or autoimmune thrombocytope-

due to isoimmun
bodies, and mic

nia may be treated with corticosteroids or intravenous immunoglobulin until the maternal
antiplatelet antibodies dissipate. A detailed discustopenic Purpura
sion of childhood idiopathic thrombocytopenia pur-

Idiopathic Thrombocy

pura (ITP) appears later in this chapter.


mbocytopenia for which a cause
Microangiopathic hemolytic anemias also cause
results from the development
thrombocytopenia by decreasing platelet survival.
bodies that bind to the platelet
Microangiopathic disorders include disseminated
body-coated platelets are then
intravascular coagulation (DIG), hemolytic-uremic
iculoendothelial system. Rarely,
syndrome (HUS), and thrombotic thrombocytopenic
nting symptom of an autoimpurpura (TTP). DIG is discussed later. HUS, characs systemic lupus erythematosus
terized by a microangiopathic hemolytic anemia,

ITP refers to a thro

renal cortical injury, and thrombocytopenia, is a


ons
major cause of acute renal failure in children.
resent 1 to 4 weeks after a viral
Verotoxin-producing gram-negative organisms, such
onset of petechiae and ecchy-

Clinical Manifestati

is not apparent. ITP


of antiplatelet anti
membrane. These anti
destroyed in the ret
ITP may be the prese
mune disease, such a
or HIV infection.

Children typically p
illness with abrupt

--------------------------------------- 24
12

rints Pediatrics
not life-threatening

. Patients who survive severe


TABLE 2-2
Signs and Symptoms of Foreign Body Aspiration
with significant scarring and

burns are often left


disability.

Location of Obstruction

Associated Signs and


Symptoms
Epidemiology

Trachea
Total obstruction
Acute asphyxia, severe
s are scald injuries. Flame burns,
retractions with poor
by smoke inhalation, are less
chest wall movement
for most deaths. A typical
Extrathoracic, partial
Inspiratory and expiratory
tric burn involves a young child

The majority of burn


usually accompanied
frequent but account
scenario for an elec

stridor, retractions
Intrathoracic, partial
Expiratory wheeze;
aterial into a wall socket or an

putting conductive m
infant sucking on th

e connected end of an extension


there is frequently

cord. Contact burns

result from direct contact with


Main stem bronchus

inspiratory stridor as
well
Cough and expiratory
wheeze; there may be
blood-tinged sputum

a hot surface.
Risk Factors
Boys and children yo

unger than 5 years are at the


Lobar/segmental bronchus
Decreased breath sounds
rn injury.
over affected lobe,
wheezing, rhonchi

greatest risk for bu

Clinical Manifestati
ons
The evaluation of se
verity is based on body surTreatment
Partial-thickness burns are

face area and depth.


divided into first-d

egree and second-degree burns.


Foreign bodies must be removed from the airway to
nvolve only the epidermis; the
alleviate symptoms. Rigid bronchoscopy is the treater but does not blister. Firstment of choice. Thereafter, prognosis depends on
heal within a week with no
the degree of lung damage, which is directly related
econd-degree burns may be superto time interval to diagnosis. Most patients recover
lf the depth of the dermis) or deep
quickly with minimal sequelae.
he dermis but leaving appendages

First-degree burns i
skin is red and tend
degree burns usually
residual scarring. S
ficial [less than ha
(involving most of t
such as sweat glands

and hair follicles intact). Superficial partial-thick


ness burns resolve in a few weeks
Prevention
with little scarring
; deep second-degree injuries result
Infants are not developmentally prepared to protect
ing and may require skin grafting.
their airways from small morsels of food, includextend into the subcutaneous
ing hard candy, nuts, and popcorn. Small toys, coins,
ed into third-degree and fourthbuttons, and balloons should be kept out of the
-degree burns involve fascia,

in significant scarr
Full-thickness burns
tissue and are divid
degree burns. Fourth
muscle, bone, or joi

nt tissue. Both are nontender due


toddler's reach. Choking is covered in basic CPR
classes; however, the effectiveness of the Heimlich
issue loss. Because of the commaneuver or back blows is limited in instances of
the epithelium, these burns

to sensory nervous t
plete destruction of

lower tract aspiration.


Specific injury sites and patterns

require skin grafts.


are characteristic o

f abuse (Figure 2-2}.


BURNS
Burns are the third leading cause of injury in chilbe placed immediately in
dren, behind motor vehicle accidents and drownings,
vered with wet gauze or cloth.
and are the second most frequent cause of accidento gentle cleansing, silver sultal death. An estimated 15% of burns are the result
), and dressing changes twice a
of abuse. Fortunately, the great majority of burns are
ialization occurs. Burns that are

Treatment
Burned areas should
lukewarm water or co
Minor burns respond
fadiazine (Silvadene
day until re-epithel

--------------------------------------- 25
120

Blueprints Pediatrics
Treatment
TABLE 10-6
Approximately 80% of

the cases of acute ITP resolve


Causes of Thrombocytopenia
6 months. Some cases, howNeon ate
ng or chronic.
Maternal IIP,* SLE, drugs, preeclampsia
icant bleeding or severe thromboIsoimmune*
count less than 20,000] is treated
Congenital megakaryocytic hypoplasia
ids, intravenous immunoglobu(thrombocytopenia absent radius)
-D immune globulin (in RhGiant hemangioma
These measures all decrease the
Sepsis*

spontaneously within
ever, become relapsi
Clinically signif
cytopenia (platelet
with high-dose stero
lins (IVIG), or anti
positive children).
duration of severe t

hrombocytopenia by decreasing
DIG
e of antibody-coated platelets in
Congenital infections

the rate of clearanc


the reticuloendothel

ial system but do not diminish


Infant
tiplatelet antibodies. None of
Wiskott-Aldrich syndrome

the production of an
these measures affec

ts the long-term outcome of ITP.


Viral infections*
ned as thrombocytopenia conMedications

Chronic ITP, defi


tinuing for more tha

n 12 months after an acute ITP


Malignancies (leukemia, neuroblastoma)
Hemolytic-uremic syndrome

episode, is treated

with IVIG or splenectomy or both.


Sepsis
with IVIG have been effective
IIP
omy, whereas splenectomy
Childhood
70% to 80% of cases of chronic
IIP*
ases in which steroids, IVIG, antiMedications*
nd splenectomy have failed,
Aplastic anemia
th azathioprine or cyclophosLeukemia*
eresis may be indicated.
Hypersplenism (thalassemia, Gaucher's disease,
inhibits fibrinolysis, is used to
portal hypertension)
pistaxis and oral bleeding.
Sepsis
SLE
Virus-induced hemophagocytic syndrome
IIP with autoimmune hemolytic anemia (Evan's
Y POINTS
syndrome)
AIDS
mbocytopenic purpura results

Repeated treatments
in delaying splenect
induces remission in
ITP. In refractory c
D immune globulin, a
immunosuppression wi
phamide and plasmaph
Amicar, a drug that
treat uncontrolled e

KE
1. Idiopathic thro

from autoimmune
antibody formation against
"Common.
IIP, idiopathic thrombocytopenic purpura; SLE, systemic lupus host platelets.
erythematosus; DIG, disseminated intravascular coagulation; AIDS,2. Approximat
ely 80% of cases of acute ITP resolve
acquired immunodeficiency syndrome.
spontaneously w
ithin 6 months. Some cases,
however, become
relapsing or chronic.
3. Clinically sign
ificant bleeding or severe thrombocytopenia (plat
elet count less than 20,000) is
moses on the skin and bleeding of the mucous memtreated with hi
gh-dose steroids, intravenous
branes. Severe bleeding occurs after trauma. Spontaimmunoglobulins
, and anti-D globulin.
neous internal hemorrhage, though rare, has been
4. Chronic ITP is
treated with intravenous
noted with platelet counts below 10,000/mm
3.
immunoglobulins
or splenectomy or both.
Other than thrombocytopenia, the complete
Splenectomy ind
uces remission in 70% to 80% of
blood count is normal. Large platelets are seen on
the cases of ch
ronic ITP.
peripheral blood smear, and serology reveals antiplatelet antibodies. Diagnosis of ITP does not require
a bone marrow aspirate. However, if there are atypDisseminated Intrava
scular Coagulation
ical findings on either the complete blood count or

peripheral blood smear, marrow examination is indis a balance between hemorcated to exclude leukemia and idiopathic aplastic
. In DIC, this balance is altered
anemia. In ITP, bone marrow aspiration reveals
that the patient has activation
normal myeloid and erythroid elements and an
(thrombin) and fibrinolysis
increased number of megakaryocytes.
al injury, release of thrombo-

Normal homeostasis i
rhage and thrombosis
by severe illness so
of both coagulation
(plasmin). Endotheli

--------------------------------------- 26
Chapter 10 / Hematology
121
plastic procoagulant factors into the circulation, and
lation Cascade
impairment of clearance of activated clotting factors

Defects of the Coagu

Coagulation disorder
s can be inherited or acquired.
directly activate the coagulation cascade. Intravascurited defects are hemophilia
lar activation of the coagulation cascade leads to

The most common inhe

fibrin deposition in the small blood vessels, tissue


ebrand's disease, whereas
ischemia, release of tissue thromboplastin, consumpis an important acquired cotion of clotting factors, and activation of the fibri-

A and B and von Will

nolytic system. Coagulation elements, especially

Hemophilia A and B

vitamin K deficiency
agulation defect.

platelets, fibrinogen, and clotting factors II, V, and


VIII, are consumed, as are the anticoagulant proteins,
ed by deficiency of factor VIII
especially antithrombin III, protein C, and plasmino000 males, whereas hemophilia
gen. Acute and chronic conditions associated with
r IX deficiency and is found in 1

Hemophilia A is caus
and occurs in 1 in 5
B results from facto
in 25,000 males. Bot

h are X-linked recessive disorDIG include sepsis, burns, trauma, asphyxia, maligting factors are coded on autosonancy, and cirrhosis.

ders. All other clot

mal chromosomes and


are thereby inherited in an
Clinical Manifestations
he lack of factor VIII or IX
The bleeding diathesis is diffuse, with bleeding from
e production of thrombin, which
venipuncture sites and around indwelling catheters.
ion of the primary fibrin clot by
Gastrointestinal and pulmonary bleeding can be
brinogen to fibrin and stabilizes
severe, and hematuria is common. Thrombotic

autosomal fashion. T
causes a delay in th
catalyzes the format
the conversion of fi

the fibrin by activa


ting factor XIII.
lesions affect extremities, skin, kidneys, and brain.
Both ischemic and hemorrhagic stroke can occur.
ons
The diagnosis of DIG is a clinical one bolstered by
tor replacement regimens, hemolaboratory evidence. Thrombocytopenia is evident,
ndistinguishable clinically, and the
along with prolonged prothrombin time (PT) and
order is determined by the degree
partial thromboplastin time (PTT). Fibrin split prod. Children with mild hemophilia
ucts and d-dimers are elevated. Fibrinogen and factor
factor) require significant
V and VIII levels are low. The peripheral blood smear
eding, and spontaneous bleedreveals schistocytes, which are classically seen with
Patients with moderate hemomicroangiopathic disease.
normal factor) require moderate

Clinical Manifestati
Other than their fac
philia A and B are i
severity of each dis
of factor deficiency
(5% to 49% of normal
trauma to induce ble
ing does not occur.
philia (1% to 5% of
trauma to induce ble

eding episodes. Severe hemoTreatment


ith less than 1 % of normal factor)
The treatment of DIG is supportive. The disorder

philiacs (children w

that caused DIG must be treated, and hypoxia,


bleeding and will bleed with

may have spontaneous


very minor trauma. M

ild hemophilia may go undiacidosis, and perfusion abnormalities need to be


rs, whereas severe hemophilia
corrected. If bleeding persists, the child should be
ing infancy. Hemophilia is chartreated with platelets and fresh frozen plasma, which
eous or traumatic hemorrhages,
replaces depleted clotting factors. Heparin may be
useful in the presence of significant arterial or venous
neous, intramuscular, or within

agnosed for many yea


manifests itself dur
acterized by spontan

which can be subcuta


joints (hemarthroses

). Life-threatening internal hemthrombotic disease unless sites of life-threatening


rauma or surgery. In newborns
bleeding coexist.
re may be intracranial bleeding

orrhage may follow t


with hemophilia, the
secondary to traumat

ic delivery or after circumcision; otherwise, ble


eding complications are uncomKEY POINTS
r of life. Circumcision should be

mon in the first yea


avoided in boys with

a family history of hemophilia.


1. Disseminated intravascular coagulation results
hemophilia, the PTT is profrom severe illness, causing activation of both

In both forms of
longed. In hemophili

a A, factor VIII coagulant activcoagulation (thrombin) and fibrinolysis (plasmin).


2. Intravascular activation of the coagulation
whereas in hemophilia B, factor IX
cascade leads to fibrin deposition in the small
le 10-7 compares hemophilia A,

ity (VIII:c) is low,


activity is low. Tab
hemophilia B, and vo

n Willebrand's disease.
blood vessels, tissue ischemia, release of tissue
thromboplastin, consumption of clotting factors,
and activation of the fibrinolytic system.
is to prevent long-term crippling

Treatment
The goal of therapy
orthopedic injuries

due to hemarthroses. Most

--------------------------------------- 27
122

Blueprints Pediatrics
TABLE 10-7

Comparison of Hemophilia A, Hemophilia B, and von Willebrand's Disease


Hemophilia A Hemophilia B von Wi
llebrand's Disease
Inheritance X-linked X-linked
Autosomal dominant
Factor deficiency Factor VIII Factor IX
von Willebrand factor and VIII:C
Bleeding site(s) Muscle, joint, Muscle, joint,
Mucous membranes, skin,
surgical surgical
surgical, menstrual
PT Normal Normal
Normal
APTT Prolonged Prolonged
Prolonged or normal
Bleeding time Normal Normal
Prolonged or normal
Factor VIII coagulant activity (VIII:C) Low Normal
Low or normal
vWF:Ag Normal Normal
Low
vWF:Act Normal Normal
Low
Factor IX Normal Low
Normal
Ristocetin-induced platelet agglutination Normal Normal
Normal or low
Platelet aggregation Normal Normal
Normal
patients require periodic or regular infusions of factor
Testing of blood
products for HIV and hepatitis
VIII or IX to raise their factor levels high enough to
viruses did not begi
n until the mid-1980s, and as a

stop the bleeding. While plasma-derived factors were


result, many hemophi
liacs contracted the viruses.
used in the past, recombinant factors VIII and IX
Between 1979 and 198
4, 90% of hemophiliacs who
are now available. For mild to moderate bleeding
received plasma-deri
ved factor products became
episodes, such as hemarthroses, a 40% factor level is
HIV seropositive. Ac
quired immunodeficiency synappropriate. For life-threatening bleeding, levels of
drome is the most co
mmon cause of death in older
80% to 100% of normal factors VIII and IX are necpatients with hemoph
ilia. Newer pooled concenessary. Desmopressin acetate (DDAVP), a synthetic
trates are safer, an
d all recombinant preparations are
vasopressin analogue, releases factor VIII from
completely safe from
viral agents.
endothelial cells. When administered, it triples or
Another significa
nt complication of therapy is the
quadruples the initial factor VIII level of a patient
formation of inhibit
ors, which are IgG antibodies
with hemophilia A but has no effect on factor IX
directed against tra
nsfusion factors VIII and IX.
levels. If adequate hemostatic levels of factor VIII can
Inhibitors arise dur
ing therapy in 15% of those
be achieved with DDAVP, it is the initial treatment
patients with factor
VIII deficiency and in 1% of
of bleeding for those afflicted with mild to moderthose with factor IX
deficiency. The treatment of
ate hemophilia A. Since DDAVP is an antidiuretic
bleeding patients wi
th an inhibitor is difficult.
hormone analogue, hemophiliacs who frequently use
For low-titer inhibi
tors, options include continuous
DDAVP should be monitored for hyponatremia
factor VIII infusion
s or administration of porcine
caused by water retention. Mild acute bleeding
factor VIII. For hig
h-titer inhibitors, it usually is
episodes can be treated in the home once the patiennecessart
y to admin
ister a product that bypasses the
has attained the appropriate age and the parents have
inhibitor, such as a
ctivated prothrombin complex
learned how to administer recombinant factor VIII
concentrates or reco
mbinant factor Vila. The use of
or IX or DDAVP. Bleeding associated with surgery,
frequent high doses
of prothrombin complex contrauma, or dental extraction can be anticipated,
centrates, and espec
ially of the activated products,
and excessive bleeding can be prevented with approparadoxically incr
eases the risks of thrombosis, which
priate replacement therapy. Aminocaproic acid
has resulted in fata
l myocardial infarction and stroke.
(Amicar), an inhibitor of fibrinolysis, may help treat
Induction of immune
tolerance with continuous
oral bleeding after a dental procedure. It is generally
antigen exposure plu
s immunosuppression may be
given before and after the procedure.
beneficial.

--------------------------------------- 28

Chapter 10 / Hematology

123
PTT, which results f

rom the effect of vWF deficiency


KEY POINTS
on factor VIII activ
ity. In Table 10-7, the findings in
1. Hemophilia A results from a deficiency of factor
nd's disease are compared with
VIII, and hemophilia B from a lack of factor IX.
A and B.
Both disorders are inherited in an X-linked reces-

classic von Willebra


those in hemophilia
Treatment

sive fashion.
Willebrand's disease depends
2. Other than their factor replacement regimens,
leeding. DDAVP, which stimuhemophilia A and B are indistinguishable clinivWF from endothelial cells, is the
cally, and the severity of each disorder is deterfor bleeding episodes in most
mined by the degree of factor deficiency.
ith type 3 disease (who have
3. Hemophilia is characterized by spontaneous
r with severe bleeding not reor traumatic hemorrhages, which can be
n be treated with a virally
subcutaneous, intramuscular, or within joints

The treatment of von

(hemarthroses). Life-threatening internal hemorining concentrate (Humate P].


rhage may follow trauma or surgery.
also be used, but it cannot be

attenuated vWF-conta

on the severity of b
lates the release of
treatment of choice
patients. Patients w
no vWF to release) o
sponding to DDAVP ca

Cryoprecipitate may
virally attenuated.

Hepatitis B vaccine should be


given before exposur
e to plasma-derived products.
As in all bleeding d
isorders, medications that
alter platelet funct
ion, such as aspirin, should be
von Willebrand's Disease

avoided.

von Willebrand's disease is caused by deficiency of


von Willebrand factor (vWF), an adhesive protein
that connects subendothelial collagen to activated
EY POINTS
platelets and also binds to circulating factor VIII, protecting it from rapid clearance, von Willebrand's
s disease is caused by deficiency
disease is classified on the basis of whether vWF
nd factor, an adhesive protein that
is quantitatively reduced but not absent (type I),
othelial collagen to activated
qualitatively abnormal (type 2; dysproteinemia), or

1. von Willebrand'
of von Willebra
connects subend
platelets and a

lso binds to circulating factor VIII,


absent (type 3].
rom rapid clearance.

protecting it f
2. The clinical ma

nifestations of mild to moderate


Clinical Manifestations
s disease are similar to those of
The clinical manifestations of von Willebrand's
a and include mucocutaneous
disease are similar to those of thrombocytopenia and
axis, gingival bleeding, cutaneous
include mucocutaneous bleeding, epistaxis, gingival
enorrhagia.
bleeding, cutaneous bruising, and menorrhagia. In
illebrand's disease, factor VIII defisevere von Willebrand's disease, factor VIII deficiency
rofound and the patient may also
may be profound and the patient may also have manions similar to hemophilia A.
ifestations similar to those of hemophilia A. If there
is little or no vWF in the blood to bind factor VIII,
eatment of choice for the majority
factor VIII is cleared quickly from the circulation,
sodes in patients.
resulting in factor VIII deficiency. Approximately

von Willebrand'
thrombocytopeni
bleeding, epist
bruising, and m
3. In severe von W
ciency may be p
have manifestat

4. DDAVP is the tr
of bleeding epi

85% of patients with von Willebrand's disease have


classic type 1 disease, which results in mild to moderate deficiency of vWF.
y
Laboratory testing includes measurement of
(factors II, VII, IX, and X] and
the amount of protein, usually accomplished by
rs (protein C and protein S) are
immunologic detection of vWF antigen (vWF:Ag)
iver and depend on vitamin K for
and vWF activity (vWF:Act). vWF activity is meavitamin K is deficient, coagulasured functionally by the ristocetin cofactor assay
tamin K deficiency often occurs
(vWF:RCoF), which uses the antibiotic ristocetin to
tion, especially with cystic
induce vWF to bind to platelets. The patient typically
tibiotic-induced suppression of
has a prolonged bleeding time, due to the effect of
that produce vitamin K. Overdose
vWF deficiency on platelet activity, and a prolonged
that interferes with vitamin K

--------------------------------------- 29
124

Blueprints Pediatrics

metabolism, causes deficiency of vitamin K-depen-

Vitamin K Deficienc
Coagulation factors
antithrombotic facto
synthesized in the l
their activity. When
tion is impaired. Vi
because of malabsorp
fibrosis and with an
intestinal bacteria
of coumadin, a drug

dent factors. Similarly, maternal use of coumadin or

TABLE 10-8
Differentiation of

Vitamin K Deficiency, Liver


anticonvulsant therapy (phenobarbital, phenytoin)

Disease, and DIG

may also result in vitamin K deficiency in the


newborn. The most common disorder resulting from
Vitamin K
Liver
DIG
vitamin K deficiency is hemorrhagic disease of the
Deficiency
Disease
newborn, which occurs in neonates who do not

Laboratory
Test

PT
t
T
t
receive intramuscular vitamin K at birth.
nl
J/ to nl
i
Clinical Manifestations
nl
U
Although most newborn infants are born with
nl
nltot
I
reduced levels of vitamin K-dependent factors, only
a few develop hemorrhagic complications. Because
T
nl
nl to T
breast milk is a poor source of vitamin K, breast-fed

Platelets
Fibrinogen
Factor VIII
Fibrinogen
degradation
products

infants who do not receive prophylactic vitamin K


on the first day of life are at the highest risk for hem4
4.
1 to nl

Factor VII
nl, normal.

orrhagic disease. Peak incidence is at 2 to 10 days of


life. The recommended preventive dose of vitamin K
is 1.0 mg given intramuscularly. The disorder is
marked by generalized ecchymoses, gastrointestinal
hemorrhage, and bleeding from the circumcision site
and umbilical stump. Affected neonates are at risk for
EY POINTS
intracranial hemorrhage.
tors II, VII, IX, and X and antithromBoth the PTT and PT are prolonged in vitamin K
rotein C and protein S are synthedeficiency, because factors of both extrinsic and
intrinsic pathways are affected. Prolongation of the
ver and depend on vitamin K for

K
1. Coagulation fac
botic factors p

sized in the li
their activity.

PT is a more sensitive test for vitamin K deficiency


disorder resulting from
because most infants have transient prolongation of
iency is hemorrhagic disease of the

2. The most common


vitamin K defic

the PTT at birth. The coagulopathy seen with hemoccurs in neonates who do not
orrhagic disease may be confused with liver disease

newborn, which

or DIG, both of which have a prolonged PT and


K at birth.

receive vitamin
3. The coagulopath

y seen with hemorrhagic disease


decreased factor VII level. Table 10-8 differentiates
with liver disease or DIC, both of
vitamin K deficiency, liver disease, and DIG by laboolonged PT and decreased factor
ratory data.

may be confused
which have a pr
VII level.

Treatment
Nutritional disorders and malabsorptive states
respond to parenteral administration of vitamin K.
Fresh frozen plasma or prothrombin complex concentrate, which is a mixture of coagulation factors II,
VII, IX, and X, is indicated for severe bleeding.

--------------------------------------- 30
Immunology,
Allergy, and
Rheumatology
IMMUNOLOGY
upper respiratory tract infections,

history of frequent
including otitis med

ia, sinusitis, and pneumonia.


The immune system, composed of specialized cells
and molecules, is responsible for recognizing and
is
neutralizing foreign antigens. Specific complex interactions produce adaptive inflammatory responses
agammaglobulinemia occurs in
and defend against infection. Immunodeficiency
ter 6 months of age as matersyndromes increase susceptibility to infection,
dy levels fall. These patients do
malignancy, and autoimmune disorders (Table 11-1).
es and have virtually no B cells.
Unfortunately, even a "normal" immune reaction may
susceptibility to encapsulated
result in undesirable consequences, such as tissuels with this disorder are prone to

Differential Diagnos
X-linked (Bruton's)
males and appears af
nally derived antibo
not produce antibodi
In addition to their
organisms, individua
severe, often life-t

hreatening enterovirus infections.


damaging inflammation, life-threatening anaphylaxis,
mmunodeficiency is an inheror graft rejection.
ogammaglobulinemia with equal

Common variable i
ited disorder of hyp

distribution between
the genders. Infections are
usually less severe;
however, the incidences of lymDisorders of Humoral Immunity
disease are increased in
B cells produce antibodies, the primary effectors of
iciency is the mildest and most
humoral immunity. Antibodies are a vital component
ncy syndrome. Serum levels
of the immune system, particularly in defense against
y classes are usually normal.
extracellular pathogens such as encapsulated bactelly to viral infections but are
ria. A variety of antibodies activate complement,
bacterial infections of the respiserve as opsonins, inhibit microbial adherence to
mucous membranes, and neutralize various toxins
inal, and urinary tracts.
and viruses. As a group, B-cell immunodeficiency

phoma and autoimmune


these patients.
Selective IgA def
common immunodeficie
of the other antibod
Patients react norma
more susceptible to
ratory, gastrointest
Diagnostic Evaluatio

n
syndromes are the most commonly encountered in
pediatric practice.
ment of total and fractionated

Quantitative measure
serum immunoglobulin

levels is an important screening test for specifi


c deficiencies and for panhyClinical Manifestations
Antibody titers generated

pogammaglobulinemia.
against tetanus, dip

htheria, and pneumococci after


History and Physical Examination
antibody function.
A history of recurrent infections with encapsulated
organisms such as Haemophilus influenzae and Streptococcus pneumoniae and failure to respond to appropriate antibiotic therapy is suggestive of a primary
rapy are appropriate antibiotic
B-cell deficiency. In addition, there is sometimes a
maglobulin administration.

immunization assess

Treatment
The mainstays of the
use and periodic gam

--------------------------------------- 31
126

Blueprints Pediatrics
TABLE 11-1

Clinical Characteristics of Immune Component Deficiencies


Conditions
Sequelae
Laboratory Tests
Disorders of humoral
Frequent recurrent, pyogenic infections with
Quantitative and qualitative
immunity
extracellular encapsulated organisms
immunoglobulin levels

Frequent bacterial otitis media, sinusitis, an


d
pneumonia
Phagocytic disorders
Recurrent skin infections, abscesses with cata
lase- Absolute neutrophil count
positive bacteria, Pseudomonas, fungi
(ANC)
Abscess formation in the lung, liver, and lymp
h
DCF assay
nodes
Adhesion and chemotactic
Difficult-to-clear bone, joint infections
bacterial assays
Disorders of cell-mediated
Frequent, recurrent infections with
Absolute lymphocyte count
immunity
opportunistic or low-grade organisms and
(ALC)
viruses
Mitogen stimulation
Increased incidence of autoimmune disorders
response
and malignancies
Delayed hypersensitivity
Anergy
skin testing
Complement disorders
Recurrent bacterial infections with pyogenic,
Total hemolytic complement
extracellular, encapsulated organisms
(CH 50)
Increased susceptibility to meningococcal dise
ase
Assays of the classical and
Increased incidence of autoimmune disease
alternative pathways
Intravenous immunoglobulin (IVIG) provides antiKE
Y POINTS
body replacement and has revolutionized the treatment of humoral immunodeficiency syndromes.
ciency syndromes encountered

1. Most immunodefi
in pediatrics a

re humoral.
2. Humoral immunod
eficiency predisposes patients
to infection wi
th encapsulated organisms.
Transient Hypogammaglobulinemia
munoglobulin studies and antiof Infancy

3. Quantitative im

body liters aga


inst vaccine toxins are abnormal in
Although maternal IgG is actively transported across
umoral immune dysfunction.
the placenta and is protective throughout the first
ntibodies to patients with humoral
few months of life, neonates are considered relatively
y.
immunocompromised hosts. All serum immuno-

patients with h
4. IVIG provides a
immunodeficienc

globulin classes are present at birth, but most do not


reach adult levels until early to middle childhood.
diated Immunity
Over the first 6 to 8 weeks of life, maternally derived

Disorders of Cell-Me

immunoglobulins decrease and are replaced by the


t immune responses, primarily
child's growing production. Thus, infants are partin of interleukins. In addition,
cularly susceptible to infection at 6 to 12 weeks, their
tors of cell-mediated immunity,
immunologic nadir.
ense against intracellular and
Transient hypogammaglobulinemia of infancy is a
ions. Certain subclasses are
recognized disorder in which acquisition of normal
umor and viral-infected cells.
infant immunoglobulin levels is delayed. Although
ctional T cells are at increased
some of these patients are subsequently diagnosed
disorders. T-cell diseases generwith other primary immunodeficiencies, most evenantly greater morbidity and mortually develop normal immunity.
ims than humoral disorders alone;

T cells modulate mos


through the secretio
they are major effec
important in the def
opportunistic infect
capable of killing t
Patients with dysfun
risk for autoimmune
ally impart signific
tality to their vict

--------------------------------------- 32
Chapter
127
ergy, and Rheumatology
survival beyond childhood is rare. DiGeorge's synverity. Affected patients display
drome, a congenital disorder, and human immunolity to both traditionally virulent
deficiency virus, an acquired one, both represent
fections.
T-cell immunodeficiencies. Patients with near-total
mmunodeficiency disease
thymic hypoplasia are highly susceptible to opporarly devastating disorder charactunistic infections from organisms such as fungi and
al deficits in both humoral and
Pneumocystis carinii.
ty. Patients are susceptible to a

11 / Immunology, All
range of clinical se
increased susceptibi
and opportunistic in
Severe combined i
(SCID) is a particul
terized by substanti
cell-mediated immuni
wide range of infect

ions and usually present with


Clinical Manifestations
n the first few months of life.
History and Physical Examination
have an absolute lymphocyte
An example of a disorder of cell-mediated immunity
, which can be noted on a
is DiGeorge's syndrome (chromosome 22qll delerrow transplantation has been
tion). These patients present early in infancy with
py is now being studied as a
disease unrelated to the immune system (e.g., con-

multiple illnesses i
These patients will
count less than 2800
routine CBC. Bone ma
curative; gene thera
possible alternative

treatment.
genital heart disease, hypocalcemic tetany from
asia is a rare autosomal recessive
thymic hypoplasia). Other structures and organs
ed by variable humoral and cellderived from the branchial pouches during embryocits, cerebellar ataxia, and
genesis may be malformed as well, including the ears
giectasia (small dilated vessels
and face. The severity of the immunodeficiency is
the bulbar conjunctiva and skin
extremely variable.
nce of malignancy, especially
Diagnostic Evaluation
ma and gastric carcinoma, is

Ataxia-telangiect
disorder characteriz
mediated immune defi
oculocutaneous telan
easily visible along
surface). The incide
non-Hodgkin's lympho
increased. No specif

ic therapy is available; most


Absolute lymphocyte counts are normal or moderair-bound by puberty and die
ately decreased. T-cell function, measured by in vitro
mitogen stimulation and intradermal delayed hyperyndrome is an X-linked recessensitivity testing, is absent or significantly comproand T-cell immunity, atopic
mised. No thymic shadow is seen on chest x-ray.
mbocytopenia. Survival to
Fluorescent in situ hybridization (FISH) testing of
cause of bleeding, infections, and
chromosome 22 detects the 22 gU deletion.
ies.
Treatment
The immunodeficiency of DiGeorge's syndrome has
tic Immunity
been successfully treated with both thymic and bone
marrow transplantation. Initial therapy should be
nsible for removing particulate
aimed at repairing associated congenital heart defects
d and tissues by ingesting and
and maintaining normocalcemia.
nisms. These cells must be able

patients are wheelch


prematurely.
Wiskott-Aldrich s
sive disorder of Bdermatitis, and thro
adulthood is rare be
associated malignanc
Disorders of Phagocy
Phagocytes are respo
matter from the bloo
destroying microorga
to adhere to the end

othelium, move through the


tissues to their sit
e of action, engulf the harmful
KEY POINTS
intracellularly. Chronic granulo-

matter, and kill it


matous disease (CGD)

is the most common


1. Patients with cell-mediated immune dysfunction
f phagocytic immunity.
are susceptible to autoimmune disorders and to
opportunistic infections from organisms such as
ons
Pneumocystis carinii.
2. Persistent hypocalcemic tetany or aortic arch
Examination
anomalies suggest DiGeorge's syndrome.
by chronic or recurrent

inherited disorder o
Clinical Manifestati
History and Physical
CGD is characterized
pyogenic infections

caused by bacterial and fungal

pathogens that produ


ce catalase. Although the most
Combined Immunodeficiency
disorder is inherited as an XSyndromes
mal inheritance has also been

common form of this


linked trait, autoso
reported. Abscesses

and granuloma formation occur


Combined humoral and cell-mediated immunoliver, spleen, lungs, skin, and gasdeficiencies tend to be inherited and manifest a wide
Failure to thrive, chronic diarrhea,

in the lymph nodes,


trointestinal tract.

--------------------------------------- 33
128
Blueprints Pediatrics
and persistent candidiasis of the mouth and diaper
implicated as a contributing
area are common. Affected individuals are also at
s, asthma, allergic rhinitis, and
increased risk for opportunistic infections and disllergic reactions range from mild
seminated viral disease.
and are never considered adaptive.
Diagnostic Evaluation

Allergies have been


factor in anaphylaxi
atopic dermatitis. A
to life-threatening
Allergic Rhinitis

White blood cell count typically ranges between


10,000 and 20,000/uL with 60% to 80% polymor-

Pathogenesis

phonuclear cells. Leukocyte chemotaxis is normal.


The hallmark abnormality is the inability of affected
a type 1 hypersensitivity immune
cells to produce an oxidative burst resulting in hydroental allergens, including air-

Allergic rhinitis is
response to environm
borne pollens, anima

l dander, mold, house mites,


gen peroxide. The Nitroblue tetrazolium test (NET)
te smoke, and certain foods. The
and the 2,7-dichlorofluorescin assay (DCF) are labinds to IgE on mast cells in the
oratory studies performed to detect this reduction
act, with subsequent release of
reaction.
rs. Allergic rhinitis is the most

cockroaches, cigaret
offending allergen b
upper respiratory tr
inflammatory mediato
frequent cause of ch

ronic or recurrent clear rhinorTreatment


c population.
All patients with CGD should receive daily prophy-

rhea in the pediatri

lactic trimethoprim-sulfamethoxazole. Judicious

Epidemiology

antibiotic therapy during infections is critical.


Seasonal allergic rh
initis, or hay fever, is limited to

Gamma interferon therapy has been shown to


n and is uncommon before 5
reduce the incidence of serious infection. Bone
ollens are common during early
marrow transplantation is not as successful as in
grass pollens, which are detected
other immunodeficiency syndromes.
er. Ragweed season starts in the

months of pollinatio
years of age. Tree p
spring, followed by
until the early summ
late summer and pers

ists until the first frost. PerenKEY POINTS


s year round, usually in response
1. Chronic granulomatous disease is characterized
ns, especially the dust mite.
by chronic or recurrent infections due to catalaseproducing bacteria or fungi. In particular, patients

nial disease persist


to household allerge

Risk Factors
Atopy and genetic pr

edisposition are the major risk


can develop frequent skin and liver abscesses.
allergen exposure also increases
2. Gamma interferon reduces the incidence of
bsequent disease.
serious infection.

factors. Early heavy


the likelihood of su

Clinical Manifestati
ons
History
Disorders of Complement Immunity
ic rhinitis are plagued with nasal
Although quantitative deficiencies of virtually all
watery rhinorrhea, and sneezing.
complement components have been described, they
conjunctivitis is common. Unreare less common than the classes of disease menip produces frequent coughing or

Patients with allerg


congestion, profuse
Associated allergic
lenting postnasal dr
throat clearing. Pat

ients may also complain of being


tioned earlier. Patients with complement disorders
current brief awakenings at
have increased susceptibility to bacterial infections

drowsy because of re
night.

and a higher incidence of rheumatologic disease. In


particular, deficiencies of the terminal complement

Physical Examination

components C5 to C8 increase the likelihood of


nasal mucosa appears boggy
Neisseria meningitidis infections.
acteristic features of allergic

On examination, the
and bluish. Two char
rhinitis are allergi

c shiners (dark circles that develop


under the eyes secon
dary to venous congestion] and
_ ALLERGY __ __the allergic salut_
(a horizontal crease across the

e
middle of the nose d

ue to a constant upward wiping

An allergic reaction is an undesirable immune). Because of the severe conmediated response to an environmental stimulus.
ay become obligate mouth

motion with the hand


gestion, patients m

--------------------------------------- 34
Chapter 11 / Immunology, All
ergy, and Rheumatology
129
breathers, and a gaping mouth may be seen on phystructive sleep apnea may
ical exam.
Differential Diagnosis
Y POINTS
Infectious rhinitis is much more common than aller-

become involved, obs


ensue.
KE
1. Allergic rhinit

is may be seasonal or perennial.


gic rhinitis in infants and toddlers and is often
is should be considered in any child
mucopurulent. Sinusitis causes chronic rhinorrhea
recurrent rhinorrhea and upper
and postnasal drip associated with facial tenderness,
ct symptoms.
cough, and/or headache. When a nasal foreign body
rs" and the "allergic salute" are
is present, the discharge is usually unilateral, thick,
physical features of allergic rhinitis.
and foul-smelling. Occasionally other diagnoses
histamine blockers and nasal
should be considered, including cystic fibrosis, espes are the mainstays of treatment.
cially if nasal polyps are seen.
Diagnostic Evaluation
Usually, a careful history confirms the diagnosis.
Patients who do not respond favorably to a trial of
in detail in Chapter 20. A sigsecond-generation (nonsedating) antihistamines may
of cases of asthma is allergic in
require further workup. Elevated serum and
equently associated with asthma
nasopharyngeal eosinophil levels and intranasal allere molds, dust mites, pet dander,
gen challenges may support the diagnosis; however,
lens, foods, and cockroach
direct skin testing is the preferred method for
voidance is the first step in
specific allergy testing.
Other therapies are discussed in

2. Allergic rhinit
with chronic or
respiratory tra
3. "Allergic shine
characteristic
4. Nonsedating H,topical steroid

Asthma
Asthma is discussed
nificant proportion
nature. Allergens fr
exacerbations includ
cigarette smoke, pol
antigens. Allergen a
effective treatment.
Chapter 20.

Treatment
The most effective treatment for any allergic
condition is allergen avoidance. Switching to airconditioning in the summer (rather than keeping the
onsists of allergic rhinitis, asthma,
windows open) affords some protection to patients
s (eczema). Atopic dermatitis is

Atopic Dermatitis
The allergic triad c
and atopic dermatiti
a chronic, relapsing

and remitting inflammatory skin

with pollen allergies. Limiting the amount of humidallergens. Eczema usually appears
ity in the home can decrease the presence of dust
ts upward of 10% of the pediatric
mites and various fungi. Eliminating animal dander
predilection is the highest risk
and limiting exposure to cigarette smoke are also
helpful.
Pharmacotherapy is an important adjunct if avoidons
ance is not possible. Hi-histamine blockers are the
sists of a pruritic, erythematous,
mainstay of treatment. They are now available in
lar reaction that progresses to
nonsedating formulations approved for use in chiland lichenification. In infants
dren greater than 2 years of age. Intranasal cromolyn
, the eruption involves the
is helpful as a preventive medication if taken prior to
the arms and legs, the wrists, the
the onset of symptoms. Nasal topical steroids are very
the diaper area is invariably
effective treatments with minimal side effects.
predominate in older age groups,
Topical and inhaled sympathomimetics (the most
wrists, and ankles. The diagnosis
popular being pseudoephedrine) are useful for shortis primarily clinical, based on
term therapy only and, if taken improperly, may
amination, and response to treatresult in severe rebound congestion. Allergy shots are
al diagnosis includes contact
painful, time-consuming, risky, and expensive; they
asis, a chronic nonallergic skin
are indicated only for severe symptoms not controlled with conventional pharmacotherapy.
r 5).
Occasionally, congestion is so severe that children
become exclusively mouth-breathers, which leads to
tment is to terminate the "itchdental malocclusion. If the tonsils and adenoids
Patients should try to keep their

reaction to specific
in infancy and affec
population. Genetic
factor.
Clinical Manifestati
The typical rash con
weeping papulovesicu
scaling, hypertrophy
younger than 2 years
extensor surfaces of
face, and the scalp;
spared. Flexor areas
as well as the neck,
of atopic dermatitis
history, physical ex
ment. The differenti
dermatitis and psori
disorder (see Chapte
Treatment
The mainstay of trea
scratch-itch" cycle.

--------------------------------------- 35
Chapter 2 / Poisoning, B
urns, and Injury Prevention
13
Cigarette
the great majority are preschool-

under 1 year of age;


Immersion scald

Light buib
that 10% of emergency room visits

ers. It is estimated
involving children y

ounger than 5 years result from


abuse. Parents, the
mother's boyfriend, and stepparents are the most fr
equent perpetrators.

Reports of sexual
abuse have skyrocketed over
the past few decades
. The abuse may occur at any age.
Relatives and family
acquaintances account for most
cases; molestation b
y strangers is uncommon. In 80%
of reports, the vict
ims are girls; most are abused by
Iron Curling iron
, or other male family members.

stepfathers, fathers
Male sexual abuse is

probably under-recognized.
Figure 2-2
Burn injury patterns consistent with abuse.
Neglect results i
n more deaths than physical and
sexual abuse combine
d. It is the most common cause
severe, circumferential, extensive (more than 10%
of failure to thrive
in childhood.
to 15% of the body), or that involve the face, hands,
ccurs in a fourth of children
perineum, or feet require more specialized care.
mes. The mortality rate is 5%.
Treatment includes appropriate management of
airway, breathing, and circulation issues; effective
electrolyte and fluid therapy to account for increased
fluid loss; prevention of infection; pain management;

Continued abuse o
returned to their ho

Risk Factors
Abuse and neglect oc

cur at all socioeconomic


optimization of cosmetic recovery; and early mobillevels but are more
prevalent among the poor. Chility and rehabilitation.
dren with special ne
eds (mental retardation, cerebral
palsy, prematurity,
chronic illness) are at particular
Prevention
have themselves suffered abuse,

risk. Caretakers who


who are alcohol or s

ubstance abusers, or who are


Installing and maintaining smoke detectors and
under extreme stress
are more likely to abuse or
decreasing water heater thermostat settings are the
neglect.
two most successful preventive measures for avoiding burn injury. All sleepwear for children should be
constructed of flame-retardant material. Smoking
is
cessation decreases the likelihood that matches or
lighters will be left where children can experiment
ted abuse are subsequently
with them. Parents should be counseled to practice
ld protective services. Cuts and
escape routes and reinforce the "stop, drop, and roll"
ely to represent abuse if found in

Differential Diagnos
Most cases of suspec
substantiated by chi
bruises are more lik

low trauma areas, su


ch as the buttocks or back. Care
technique for extinguishing fire.
should be taken to d
ifferentiate bruises from
Mongolian spots, whi
ch commonly occur in these
areas. Fractures tha
t occur before ambulation are
_ CHILD ABUSE AND NEGLECT
usually inflicted. O
ccasionally, osteogenesis imperInjuries intentionally perpetrated by a caretaker that
ken for abuse. Skin conditions
result in morbidity or mortality constitute physical
tigo may mimic cigarette burns

fecta has been mista


such as bullous impe
or other forms of ab

use.
abuse. Sexual abuse is defined as the involvement of
a child in any activity meant to provide sexual gratification to an adult. Failure to provide a child with
Clinical Manifestati
ons
appropriate food, clothing, medical care, schooling,
and a safe environment constitutes neglect.

History
An injury that is in

consistent with the stated history


coupled with delay i
n obtaining appropriate medical
Epidemiology
ts abuse. Age-inappropriate
Almost half the children who are brought for
knowledge are consistent with
medical attention as a result of physical abuse are
s of physical or sexual abuse may

care strongly sugges


sexual behavior and
sexual abuse. Victim

--------------------------------------- 36
130
ints Pediatrics
skin well hydrated by avoiding hot water and strong
d intolerance (an undesirable
or fragrant soaps. Tight clothing and heat may preion] and true food allergy,
cipitate exacerbations. Moisturizers are the mainstay
immune mechanisms. Exof treatment, followed by the use of topical cortiogic adverse food reaction
costeroids for areas of inflammation. In the most
uced tachycardia and lactose
severe cases, other topical immunomodulators
have been used, including tacrolimus. Severe
chronic eczema may be complicated by bacterial
superinfection.

tinguish between foo


nonimmunologic react
which is mediated by
amples of nonimmunol
include caffeine-ind
intolerance.
Epidemiology
Eighty percent of al

l food allergies present during the


Urticaria and Angioedema
The overall prevalence of food allerUrticaria and angioedema are classic type 1 hyper-

first year of life.


gies is also higher

in children (5%-8%) than in adults


sensitivity reactions. Urticaria describes the typical
few foods are represented;
raised edematous hives on the skin or mucous memproteins, soy, wheat, and fish

(l%-2%). Relatively
peanuts, eggs, milk
account for over 90%

of reported cases. Exclusive


branes resulting from vascular dilation and increased
lay presentation unless the
permeability. The lesions itch, blanch, and generally
the offending proteins regularly.
resolve within a few hours to days. Angioedema is a
s with atopic dermatitis also
similar process confined to the lower dermis and subcutaneous areas; the depth results in a welldemarcated area of swelling devoid of pruritus, erythema, or warmth. Although acute urticaria and
ons
angioedema occur frequently in the pediatric
Examination
population, chronic forms are rare.
including daily records of intake

breastfeeding may de
mother is ingesting
One-third of patient
have a food allergy.
Clinical Manifestati
History and Physical
A detailed history,
and symptoms, is ess

ential for the diagnosis. True


Clinical Manifestations
resent with isolated cutaneous
The diagnosis is based on a detailed history of recent
estinal symptoms, respiratory
exposures or changes in the patient's environment.
hreatening anaphylaxis. SympThe multiple allergens and conditions associated
ring weaning are particularly
with urticaria and angioedema include foods, medllergies.
ications, infections, and some systemic illnesses.
Clinical manifestations may be delayed as long as 48
hours after the initial encounter. Hereditary forms
n

food allergies can p


reactions, gastroint
symptoms, and life-t
toms that develop du
suggestive of food a

Diagnostic Evaluatio
Although skin testin

g may be helpful in some cases,


exist; patients with hereditary angioedema have an
acebo challenge-food challenge
inherited Cl esterase inhibitor deficiency. Often the
standard. Several foods are elimetiology remains a mystery.
ent's diet for a period before

the double-blind, pl
is the current gold
inated from the pati
testing. Then the fo

ods are disguised and tested, alterTreatment


, over several days. A challenge
Treatment depends on severity, which ranges from
ve if signs and symptoms recur
mild to life-threatening (i.e., swelling around the
airway). Subcutaneous epinephrine is the treatment
of choice in emergency situations, followed by intravenous diphenhydramine and steroids. Oral antihistamines, sympathomimetics, and occasionally oral
iminating the offending food
steroids are appropriate in milder cases.
ildren with severe, widespread

nating with placebos


is considered positi
after ingestion.
Treatment
Treatment entails el
from the diet. In ch

allergies, elementa
l hypoallergenic formulas are available. Cow's milk, s
oy, egg, and wheat allergies are
Food Allergies
er avoidance of the offending
Pathogenesis

usually outgrown aft

food. Oral challenge


s can be conducted safely to reinSteady advances have been made in food allergy
troduce the food. Ho
wever, nut and fish allergies
research over the past decade. It is important to disusually persist.

--------------------------------------- 37
Chapter 11 / Immunology, All
ergy, and Rheumatology
131
KEY POINTS
1. Peanuts, eggs, milk, soy, wheat, and fish account
in Juvenile
for the overwhelming majority of food allergies.
is
2. Signs and symptoms of food allergy in infants
toms
Systemic Symptoms
include irritability, diarrhea, and failure to thrive.
3. The double-blind, placebo challenge-food chalFatigue
lenge is the gold standard of diagnosis.
Anorexia

TABLE 11-2
Signs and Symptoms
Rheumatoid Arthrit
Joint-Related Symp

Morning stiffness
Night pain
Rheumatoid nodules

Failure to thrive
Guarding
Rash
Refusal to bear we
ight

Irritability
Deformity
Lymphadenopathy
Hepatosplenomegaly
RHEUMATOLOGY

Rheumatology involves the diagnosis and treatment


of a variety of loosely related chronic, recurrent,
arthritic and connective tissue disorders. Most are
thought to result from misdirected host defense
ons
mechanisms; the immune system fails to recognize

Clinical Manifestati

"self" antigens and initiates an inappropriate inflamExamination


matory response against the host. Usually, autoantimanifestations are listed in Table

History and Physical


Noteworthy clinical
11-2.

bodies are produced and may be recovered from


A constitutes about 10% to 20%
plasma or tissue samples, assisting with diagnosis.
urs equally in boys and girls. It

Systemic-onset JR
of all cases and occ
presents with high s

piking fevers and a salmoncolored, evanescent


rash prior to the onset of joint
Juvenile Rheumatoid Arthritis
Pathogenesis
dren will appear quite ill during

symptoms. These chil


a febrile episode; l

ymphadenopathy and hepatoJuvenile rheumatoid arthritis (JRA) consists of a


en present on exam. Elevated
group of immunologic disorders characterized by
platelet counts, anemia, and a
chronic synovitis. The American College of Rheumaate are characteristic. Tests for
tology has established the following criteria for the
(ANA) and rheumatoid factor
diagnosis of JRA:
ve. These patients do not develop

splenomegaly are oft


white blood cell and
high sedimentation r
antinuclear antibody
are generally negati
chronic iritis.

Age younger than 16 years


presents in 30% to 40% of chilArthritis in at least one joint for 6 consecutive
ivided into early-onset and lateweeks
uciarticular JRA, the patient may
Arthritis as defined by the presence of limitation
to four joints and primarily
of range of motion, tenderness or pain on motion,
e joints such as knees and
or increased warmth
nt of these patients will have a
Exclusion of other causes of arthritis
dicates an increased risk

Pauciarticular JRA
dren with JRA. It is d
onset disease. In pa
have involvement in up
has symptoms in larg
ankles. Eighty perce
positive ANA, which in
factor for the devel

opment of uveitis.
Epidemiology
JRA, the involvement of five or
JRA, as is true of most rheumatologic conditions,
red for diagnosis. Girls predomioccurs more commonly in girls. Patients may be
f 3:1. Joint involvement may
afflicted early or late in childhood or in adolescence.
rge joints as well as the tem-

In polyarticular
more joints is requi
nate, with a ratio o
include small and la
poromandibular joint

and cervical vertebrae.


Risk Factors
y be present in some patients
Many patients have a positive family history for other
disease similar to adult
rheumatologic disorders. Certain HLA types also
. Fewer patients have a positive
have been associated with increased risk of disease

Rheumatoid factor ma
who go on to develop
rheumatoid arthritis
ANA, but those that

do continue to be at risk for


(e.g., HLA-B27 and pauciarticular JRA).

uveitis.

--------------------------------------- 38
132
Blueprints Pediatrics
Differential Diagnosis
dy immune complexes become
Virtually any rheumatologic disorder can present
ls of small arteries, resulting in

them. Antigen-antibo
deposited in the wal
inflammation and nec

rosis. This immune complex


initially with isolated arthritis. Other conditions to
sic pathologic lesion responsible for
consider include septic arthritis, toxic synovitis, Lyme
disease, and reactive arthritis. Noninflammatory
al manifestations.
causes of limb and joint pain are discussed in detail
in Chapter 19.

vasculitis is the ba
the extensive clinic
Epidemiology
SLE usually appears

in late childhood or adolescence


Diagnostic Evaluation
and is far more comm
on in females. AfricanSynovial fluid analysis typically reveals a white blood
c patients tend to have more
cell count of 10,000 to 20,000/u.L and high protein
with decreased glucose and complement levels.
Radiographs reveal soft tissue swelling early; later,

American and Hispani


severe disease.
Clinical Manifestati

ons
bony erosion and narrowing of the joint spaces occur.
History and Physical
Examination
Treatment
is based on clinical criteria

The diagnosis of SLE


(Table 11-3). The on

set may be precipitous and


Treatment consists of medical management with
inflammatory-suppressive drugs (nonsteroidal antior insidious with a slow, steady
inflammatory drugs, immunosuppressive drugs,
se, and weight loss are frequent
steroids, etc.) and physical therapy. Functional or
aints. Arthritis of the hands,

rapidly progressive,
course. Fever, malai
constitutional compl
wrists, elbows, shou

lders, knees, and ankles produces


cosmetic surgery is generally delayed until growth is
complete.
on to the physical signs; in fact,
Most patients with JRA experience little permais neither erosive nor deforming.

pain out of proporti


the arthritis of SLE
Central nervous syst

em involvement may present at


nent disability and remain in remission for long
periods. Severe involvement often leads to joint
urse of the disease.

any time over the co

destruction and deformity and may result in a leg


s the most common clinical manlength discrepancy. Children with pauciarticular
ten present at diagnosis. The World

Lupus nephritis i
ifestation and is of
Health Organization

classifies renal involvement as


disease may develop iridocyclitis and vision loss. Systemic JRA is associated with pulmonary, hepatic,
renal failure extremely rare),
central nervous system, and cardiac disorders.
20%, renal failure rare), focal pro-

normal (type I, 6%,


mesangial (type II,
liferative (type III

, 23%, renal failure uncommon),


diffuse proliferativ
e (type IV, 40%, progressive renal
failure common, high
mortality), and membranous
KEY POINTS
al failure uncommon).
1. Juvenile rheumatoid arthritis is characterized by
chronic synovitis and is classified according to
degree of involvement (systemic, pauciarticular,
polyarticular).
2. Anti-inflammatory drugs and physical therapy are
lassification of Systemic Lupus
the mainstays of treatment.

disease (type V, ren

Criteria for the C


Erythematosus
Malar (butterfly)

rash
Discoid lupus ras
h
Photosensitivity
Systemic Lupus Erythematosus
Oral or nasal muc
ocutaneous ulcerations
Pathogenesis
tis
Systemic lupus erythematosus (SLE) is characterized
by widespread connective tissue inflammation and
carditis
arteriolar vasculitis. SLE develops when the immune
system somehow begins to recognize "self" nuclear
rology
proteins, cytoplasmic contents, and connective tissue
ear antibody test
as "foreign" and attempts to neutralize or remove

Nonerosive arthri
Nephritis
Encephalopathy
Pleuritis or peri
Cytopenia
Positive immunose
Positive antinucl

--------------------------------------- 39
Chapter
133
ergy, and Rheumatology
Diagnostic Evaluation
Anemia, leukopenia (with a predominance of neutrophils), and thrombocytopenia are characteristic.
n inflammatory disease involvComplement levels, including C3, C4, and CH50,
s of the skin, striated muscle, and
are generally depressed or falling, especially during

11 / Immunology, All
Dermatomyositis
Pathogenesis
Dermatomyositis is a
ing the small vessel

active disease. A positive antinuclear antibody test is


trointestinal tract. Immune
very sensitive but not necessarily specific. Elevation
ted in the walls of arterioles,
in double-stranded DNA antibodies parallels disease
ules, leading to inflammation,
severity, especially renal disease. Other autoanti, and fibrinous repair. Polymyosibodies, including antiphospholipid and anticardimmatory muscular condition
olipin antibodies, may be present. Circulating anti-Ro
s, occurs less frequently in the
and anti-La antibodies in an SLE-affected mother
.
may cause congenital heart block in her fetus.
Treatment
in the later childhood years.
Treatment is long term and multifactorial. Careful
gic conditions, dermatomyoattention must be paid to nutritional status and fluid
in females.
balance. Limiting sun exposure and using appropriate
sun block improves skin problems. Aggressive charritable, and the condition seems
acterization and treatment of renal disease, including
h viral illnesses in some cases.
renal biopsy and frequent imaging, are invaluable in
minimizing morbidity. Hypertension is a relatively
ons
common complication that is usually well controlled
Examination
with conventional therapy.
story of malaise, fatigue, weight
Anti-inflammatory therapy remains the mainstay
nt fevers. Progressive muscle
of pharmacologic treatment. Oral prednisone is prescribed as needed for maintenance therapy; highic and shoulder girdle muscle
dose oral or intravenous pulse therapy is preferable
y the characteristic violaceous
during acute exacerbations. Other immunosuppreselids, hands, elbows, knees, and
sants, such as cyclophosphamide, are helpful in treatnches the diagnosis. Gottron's
ing lupus nephritis. Hydroxychloroquine may be
ristic lesions resembling scaly
used to treat mucocutaneous symptoms.
on the extensor surfaces of the
Overall, prognosis and quality of life are improvts of the fingers, the elbows, and
ing. Renal disease produces the most significant moress may advance to involve the
bidity; renal failure is the leading cause of death after
, and muscle groups used for
infection. Libman-Sacks endocarditis is a serious
n, and respiration. Longcardiac complication. Most patients with SLE do not
n eventually results in calcium

occasionally the gas


complexes are deposi
capillaries, and ven
ulceration, bleeding
tis, a similar infla
without skin finding
pediatric population
Epidemiology
Onset usually occurs
Like most rheumatolo
sitis is more common
Risk Factors
Predisposition is he
to be associated wit
Clinical Manifestati
History and Physical
Patients report a hi
loss, and intermitte
weakness of the pelv
groups accompanied b
dermatitis of the ey
ankles virtually cli
papules are characte
erythematous papules
interphalangeal join
the knees. The weakn
anterior neck, trunk
swallowing, phonatio
standing inflammatio

achieve normal life expectancy.


(calcinosis cutis], cutaneous

deposits in the skin


striation, scarring,

and significant muscle atrophy.


KEY POINTS

Diagnostic Evaluatio

n
1. SLE consists of widespread connective tissue
boratory abnormality is marked
inflammation and vasculitis.
reatinine kinase, an enzyme
2. The diagnosis of SLE is clinical.
le breakdown. Specific elec3. Lupus nephritis is the most common clinical mantologic results are characteristic
ifestation, resulting in significant morbidity.
um acute-phase reactant levels
4. Typical laboratory findings include falling completation rate, C-reactive protein)
ment levels and positive antinuclear antibody and
se severity.
double-stranded DNA antibody titers.
5. The disease usually responds to immunosuppressant therapy.
f rest, appropriate physical

The most striking la


elevation of serum c
released during musc
tromyography and his
of the disorder. Ser
(erythrocyte sedimen
correlate with disea
Treatment
Treatment consists o
therapy, and immunos

uppressants. As long as muscle

--------------------------------------- 40
Blueprints Pediatrics
T34
enzyme levels remain high, activity is limited and
complaints, painful
erythematous skin nodules,
the primary aim of therapy is to prevent contracpurpura, hypertens
ion, hematuria, abdominal pain,
tures with positioning and splints. High-dose predencephalopathy,
and neuropathy. The fingers and
nisone is prescribed in an attempt to control thtoee
s become gangreno
us in extreme disease. The
inflammatory response. Once evidence of musclerythrocyte
e sedime
ntation rate is invariably elevated
destruction begins to abate, steroid doses are taperedurind
g active disea
se. Diagnosis rests on signature
and strengthening exercises are gradually addedvascula.
r lesions on
biopsy. Corticosteroids and
Patients whose disease does not respond to oraimmunl
e suppressan
ts are the mainstays of therapy.
steroids may require intravenous pulse steroids oPrognosir
s is fair;
mortality is related to renal or
second-line agents including methotrexate, intracardiac complication
s.
venous immunoglobulin, cyclophosphamide, and
Wegener's Granulomat
osis
cyclosporine.
Oropharyngeal, chest wall, and respiratory muscle
Wegener's granulomat
osis, a rare, necrotizing vasweakness predisposes patients to aspiration. Respiraculitis, typically p
resents with a triad of involvement
including the upper

and lower respiratory tracts and


tory failure necessitating mechanical ventilation is
isease causes a necrotizing vasrare. Most children diagnosed with dermatomyositis
is is further supported by the
recover with no permanent disability within a few
antineutrophil antibodies
years. About 10% progress to wheelchair dependence
taining pattern). Treatment is
and premature death.
on with corticosteroids or

kidneys. This rare d


culitis. The diagnos
presence of C-ANCA [
with a cytoplasmic s
with immunosuppressi
cyclophosphamide.
Henoch-Schonlein Pur

pura
KEY POINTS
pura is an immunologically

Henoch-Schonlein pur
mediated vasculitis

involving the gastrointestinal


1. Dermatomyositis is an inflammatory disease of
and kidneys. It occurs in young
the small vessels of the skin, striated muscle, and
he winter months, and may be
gastrointestinal tract.
2. The weakness begins in the proximal extremity
or group A streptococcal upper
muscle groups and is accompanied by a characn. Gastrointestinal involvement
teristic violaceous dermatitis.
nt, including abdominal pain, vom3. Serum creatinine kinase levels are markedly
per and lower tract bleeding.
elevated.
ay progress to acute renal

tract, skin, joints,


children, peaks in t
preceded by a viral
respiratory infectio
is usually significa
iting, ileus, and up
Glomerulonephritis m
failure. The charact

eristic nonthrombocytopenic pur4. The weakness may progress to involve the respiar rash over the buttocks and
ratory and oropharyngeal muscles.
almost always observed. Treat-

puric or maculopapul
lower extremities is
ment is supportive;

corticosteroids have not been


found to be particul
arly helpful. The prognosis for
Vasculitides
4 to 6 weeks is excellent. Long-

full recovery within


term complications p

arallel the severity of renal


A number of other connective tissue diseases, including polyarteritis nodosa, Wegener's granulomatosis
and Henoch-Schonlein purpura, present with vasKawasaki'e is a systemic vasculitis characterculitis as the primary manifestation. Kawasaki's
lymphadenopathy, and mucocudisease, a vasculitis postulated but not proven ttaneouo
t occurs almost exclusively in infants
be infectious in origin, is limited to the pediatric
nd is more common in males.
population.
gy has been suggested but never

involvement.
Kawasaki',s Disease
s diseas
ized by high fever,
s lesions. I
and young children a
An infectious etiolo
confirmed. Current c

riteria for diagnosis are noted in


Clinical Manifestations and Treatment
Table 11-4.
Polyarteritis Nodosa
The most serious
complications are cardiac,
Insidious in onset, variable in symptomatologyincludin,
g coronary
vasculitis and concurrent or
waxing and waning, polyarteritis nodosa often provedelayes
d aneurysm f
ormation. Prognosis is tied to
difficult to diagnose. Signs and symptoms may
severity of cardiac
involvement; cardiac instability
include any of the following: vague systemic
can produce arrhyth
mias, infarction, or congestive

--------------------------------------- 41
Chapter 11 / Immunology, All
ergy, and Rheumatology
135
TABLE 11-4
ibed during the acute course as an
Criteria for Diagnosis of Kawasaki's Disease
FVIG therapy administered over

Aspirin is prescr
antiplatelet agent.

2 to 3 days results
in profound improvement. Both
Fever for 5 days or more, together with four of the
treatments significa
ntly reduce the risk of coronary
following five signs on physical exam (or by history):
artery aneurysms.
1. Bilateral conjunctivitis
2. Changes of lips and oral cavity (dry, red, fissured lips
KE
Y POINTS
or strawberry tongue)
3. Changes of peripheral extremities (erythema or
1. Henoch-Schonlei
n purpura is characterized by
indurative edema of hands and feet)
abdominal pain,
vomiting, gastrointestinal bleed4. Polymorphous rash (primarily on trunk)
ing, and nonthr
ombocytopenic purpura over the
buttocks and lo
wer extremities.
5. Acute nonpurulent swelling of cervical lymph node
2. Kawasaki's dise
ase presents with high fever, lymto >1.5cm in diameter
phadenopathy, a
nd mucocutaneous lesions.
3. High-dose aspir
in therapy and IVIG reduce the risk
heart failure within days of presentation. Aneurysrns
of coronary art
ery aneurysms in Kawasaki's
and coronary artery disease persist and may result in
disease.
death months to years later.

--------------------------------------- 42
Infectious
Disease
Remarkable advances in the diagnosis, management,
and prevention of infectious diseases have occurred
during the past century. New techniques for diagno-

VACCINATIONS
Routine Immunizatio

ns
sis include fluorescent antibody testing, polymerase
chain reaction (PCR), and imaging modalities
involves stimulating an indisuch as magnetic resonance imaging (MRI). Specific
em to develop a rapid protectreatment of bacterial illnesses began with the
future infectious exposures. A
introduction of sulfonamides in the 1930s and
or part of either a weakened or
penicillin in the 1940s. Newer classes of antibacrt of the organism. Table 12-1
terial agents include semisynthetic penicillins,
d version of the current vaccina-

Active immunization
vidual's immune syst
tive response during
vaccine contains all
nonviable form or pa
contains a simplifie
tion guidelines reco

mmended by the American


tetracyclines, macrolides, fluoroquinolones, aminos.
glycosides, carbapenems, and four generations of
g history of safe use and imprescephalosporins. Antifungal, antiviral, and antiparasitic agents have also been developed. Other antiratio, vaccines should be held or
infectives include specific antibodies, intravenous
ircumstances. Table 12-2 lists
immunoglobulin, phagocyte stimulating factors, and
e contraindications to vaccine
interferons. Vaccines have led to a dramatic decline
ome common misconceptions.
in certain infectious diseases. Smallpox was
eradicated worldwide in 1977, and indigenous

Academy of Pediatric
Despite their lon
sive cost-to-benefit
delayed in certain c
absolute and relativ
administration and s

Additional Vaccinati
ons
poliomyelitis was eliminated from the United States
ital, iatrogenic, or functional
in 1979. The annual incidence of measles,
mumps, rubella, diphtheria, pertussis, tetanus, and
isease) asplenia should receive the
Haemophilus influenzae type b meningitis has been
th pneumococcal (conjugate
decreased by more than 98% in the United States by
vaccines. A yearly influenza
vaccine use.
ed for children between 6 and
Unfortunately, new pathogens continue to
r patients with chronic pulemerge; for example, human immunodeficiency virus
uding asthma), cardiac disease,
(HIV) was unheard of 20 years ago. Equally disse and for patients receiving

Children with congen


(e.g., sickle cell d
meningococcal and bo
and polysaccharide)
vaccine is recommend
24 months old and fo
monary disease (incl
or sickle cell disea
immunosuppressive th

erapy.
concerting is the rapid emergence of resistance to
known antibiotics (e.g., methicillin- and vancomycinresistant Staphylococcus aureus and penicillinresistant Streptococcus pneumoniae). Thus, after 100
RIGIN
years of progress against infectious diseases, the
current challenges are every bit as formidable as at
unknown origin" (FUO) implies

_ fEVER OF UN KNOWN O
The phrase "fever of

the beginning of the last century.


uration (>10 days), documented

fever of prolonged d
temperature greater

than 38.3C (101F) on multiple occasions, and u


ncertain etiology. FUO usually is

--------------------------------------- 43
Ch
apter 12 / Infectious Disease
TABLE 12-1

137

ization Schedule
Age

nmunizations

Birth

BV(1)

2 mo
V(1)
4 mo
V (2)
6mo a
V (3)
6-18mo

BV(2)

DTaP(1)

Hib (1)

IPV(l)

PC

DTaP (2)

Hib (2)

IPV (2)

PC

DTaP (3)
3V (3)

12-15 mo
V (4) MMR(1)
>12mo
15-18mo

Hib (3)

PC
IPV (3)

Hib (4)

PC

Varicella
DTaP (4)

4-6 yr

DTaP (5)b
IPV (4)
MMR (2)
' Influenza vaccine also is recommended annually for children aged 6 months to
24 months and for all children >6 months with chronic pulmonary,
cardiovascular, metabolic, or sickle cell disease.
b Tetanus-diphtheria vaccine is given at age 11 years and then every 10 years
thereafter.
The numbers in parentheses indicate the number in the sequence of immunization
s. DTaP, diphtheria, tetanus, and acellular pertussis vaccine; HBV,
hepatitis B virus vaccine; Hib, Haemophilus influenzae type b vaccine; IPV, in
activated polio virus vaccine; MMR, measles, mumps, rubella vaccine; PCV,
conjugated seven-valent pneumococcal vaccine.
TABLE 12-2
Contraindications to and Precautions Regarding Vaccination
Absolute Contraindications Precautions (Relative Contraindications)
Not Contraindications
Severe allergic reaction (e.g., anaphylaxis) Shock/hyporesponsive episode < 48
hours Mild illness with or without
after a previous vaccine dose
after previous dose of DTaP
low-grade fever
Known severe immune deficiency (MMR;
Fever > 40.5C within 48 hours of pre

vious Current antibiotic therapy


varicella)
Encephalopathy within 7 days of
e of Positive PPD
administration of the previous

dose of DTaP
Seizure < 3 days after previous dos
DTaP

dose (DTaP)
Pregnancy (MMR; varicella)
th or Prematurity*

Moderate to severe acute illness wi


without fever

* Premature infants should be vaccinated according to chronologic age. Hepatit


is B vaccine should be delayed until the child weighs more than 2000 g if
the mother is HBsAg negative.
DTaP, diphtheria, tetanus, and acellular pertussis vaccine; HBsAg, hepatitis
B surface antigen; MMR, measles, mumps, rubella vaccine; PPD, purified
protein derivative.
caused by a common pediatric infection with an
ocky Mountain spotted fever,
atypically prolonged time course.
e disease, brucellosis, lep-

scratch disease, R
ehrlichiosis, Lym
tospirosis, tular

emia, endocarditis, septic arthritis,


Differential Diagnosis
tra-abdominal abscess, enteric

osteomyelitis, in
fever, tuberculos

is, HIV, opportunistic infection


FUO in the pediatric population is usually a common
disease: Systemic juvenile
disorder presenting in an uncommon manner, rather
tis, systemic lupus erythethan a "zebra." Diagnostic considerations include the
following:
ia, lymphoma, neuroblas-

Connective tissue
rheumatoid arthri
matosus
Malignancy: Leukem
toma

Infection: Sinusitis, hepatitis, cytomegalovirus


bowel disease, Kawasaki's
(CMV), Epstein-Barr virus [EBV], parasites, catver, thyrotoxicosis, sarcoidosis,

Other: Inflammatory
syndrome, drug fe

--------------------------------------- 44
138
Blueprints Pediatrics
familial dysautonomia (Riley-Day syndrome), and
KE
Y POINTS
factitious fever
1. The phrase "fev
er of unknown origin" implies fever
of prolonged du
ration (>10 days), documented
Clinical Manifestations

temperature gre

ater than 38.3C (101F) on multiple occasions,


and uncertain etiology.
History
The age and gender of the patient narrow the difcaused by a common pediatric
ferential diagnosis. Inflammatory bowel disease and
an atypically prolonged time
connective tissue disorders are uncommon in
al exam, and initial laboratory
younger children. Autoimmune disorders occur more
urther evaluation. ,
frequently in females. Sexual history, travel history,
current medications, exposure to animals, tick bites,
antecedent illness, trauma, and family history are
important areas of inquiry. A thorough history
SIS __ _ __
and physical exam (usually after repeated encounters) will reveal the diagnosis in more than half
esence of bacteria in the blood.
of the children in whom a cause of the fever is
r described as occult if it occurs
found.
child without any obvious source

2. FUO is usually
infection with
course.
3. History, physic
studies guide f

BACTEREMIA AN D SEP

Bacteremia is the pr
Bacteremia is furthe
in a well-appearing
of infection. The ri

sk of occult bacteremia is highest


Physical Examination
ren between 2 and 24 months
Conjunctivitis, the absence of tears, rashes, lymgreater than 39.0C and leukophadenopathy, joint tenderness, oral ulcers, thrush,
y of episodes are due to Strepto-

(1.5%-2.5%) in child
of age with a fever
cytosis. The majorit
coccus pneumoniae an

d resolve spontaneously. Rarely,


heart murmurs, organomegaly, masses,
tenderness, cutaneous manifestations
pigmentation, etc.), joint findings,
is implies bacteremia with evichanges may suggest a specific cause
response (tachypnea, tachycarfurther evaluation.
ed organ perfusion. Affected

abdominal
(rash, hyperand mental status

meningitis occurs.

and guide

dence of a systemic

In contrast, seps

dia, etc.) and alter


children appear quit

e ill and may develop shock.


The cause of sepsis
varies by age. In neonates, group
Diagnostic Evaluation
ric gram-negative bacilli, and
The initial evaluation can be performed in the outes are most prevalent. In older

B streptococci, ente
Listeria monocytogen
children, S. pneumon

iae predominates, followed by


patient setting in older, well-appearing children.
is. Less common causes include
Neonates and ill-appearing children require
s, Salmonella sp., Pseudomonas
hospitalization.
dans streptococci. The evaluation
Screening laboratory tests include complete blood

Neisseria meningitid
Staphylococcus aureu
aeruginosa, and viri

count and differential, serum electrolytes, blood urea


spected sepsis includes cultures

of the child with su


from the blood, urin

e, and occasionally CSF. A chest


nitrogen (BUN) and creatinine levels, liver function
respiratory signs or symptoms are
tests, alkaline phosphatase, and urinalysis. Bacterial
atment with a third-generation
cultures should be obtained from specimens of
ccasionally) vancomycin is
blood, urine, stool, and possibly cerebrospinal fluid
(CSF). Stool may also be sent for viral antigen deteciate supportive measures.
tion and parasite examination. Additional tests to
consider include erythrocyte sedimentation rate
(ESR), C-reactive protein, antinuclear antibodies,
and specific serologic tests such as antibody studies
for cat-scratch disease and EBV. A chest x-ray and
skin testing for tuberculosis are typically performed.
ed acute infection of the middle
More expensive and invasive studies may be ware physician visits than any other
ranted based on screening results. In about 25% of
he middle ear is normally sterile; a
cases, no etiology is determined and the children
le eustachian tube allows fluid
recover without sequelae.
ddle ear into the nasopharynx

x-ray is obtained if
present. Empiric tre
cephalosporin and (o
coupled with appropr
.- MEDIA._ __
Pathogenesis
Suspected or confirm
ear accounts for mor
pediatric illness. T
patent but collapsib
drainage from the mi

--------------------------------------- 45
Chapt
139
er 12 / Infectious Disease
but normally prevents the retrograde entry of upper
externa (inflammation of the
respiratory flora. In children, the angle of entry,
also causes ear pain; however, the
short length, and decreased tone of the tube may
hould appear normal on
allow for retrograde flow and increased susceptibil. The pain of otitis externa is
ity to infection.
ulation of the external ear. A

of ear pain). Otitis


external ear canal)
tympanic membranes s
physical examination
exacerbated by manip
tympanic membrane th

at is erythematous without
Epidemiology
isease may be caused by vigorous

any other signs of d


crying and should no

t be considered OM.
Otitis media (OM) is most common in children 6 to
36 months of age. By 3 years of age, 80% of all children in the United States have had at least one
episode of otitis media, and 50% have had at least
untreated children with acute
three episodes. About 30% of cases of acute OM are
inical resolution by 7 to 14 days,
caused by viruses but may be complicated by bacter than 95% of those treated

Treatment
Approximately 80% of
otitis media have cl
compared with greate

rial superinfection. The other 70% represent bacterwith antimicrobials.


Most clinicians recommend
ial infections, most commonly S. pneumoniae (50%
antibiotics for acut
e OM to prevent progression to
of all bacterial episodes), nontypeable Haemophilus
chronic otitis media
and reduce the risk of rare but
influenzae (30% of all bacterial episodes), and
serious sequelae inc
luding mastoiditis and meningiMoraxella catarrhalis (10% of all bacterial episodes).
tis. The pathogens r
esponsible for acute OM generChronic suppurative OM is more likely to be caused
ally respond to high
-dose oral amoxicillin (80-90
by Staphylococcus aureus, Pseudomonas aeruginosa, omg/kg/dar
y for 5 t
o 10 days); resistant organisms may
mixed pathogens.
require amoxicillinclavulanate or second- or thirdgeneration cephalosp
orins. Tympanocentesis relieves
severe pain, may ide
ntify the pathogen (in neonates,
Risk Factors
immunocompromised ch
ildren, or children who fail
Caretaker smoking, bottle feeding, day-care attenantibiotic therapy),
and treats complications such as
dance, allergic disease, craniofacial anomalies,
mastoiditis. When mo
re than four infections have
immunodeficiency, and pacifier use all predispose
occurred within a yea
r (or three in 6 months), the
children to OM.
child should be refe
rred for possible tympanostomy
tube placement. The
use of daily prophylactic antibiotic therapy is in d
ispute. Antibiotic therapy is not
Clinical Manifestations
recommended for trea
tment of otitis media with
History and Physical Examination
effusion.
In addition to c
onductive hearing loss, complicaChildren may have local or systemic complaints or
tions of frequent ep
isodes of OM include tympanic
both, including ear pain, fever, and fussiness. Acute
membrane perforation
, excessive scarring (tymOM is frequently preceded by symptoms of upper
respiratory infection (cough, rhinorrhea, congestion).
On physical examination, the affected tympanic
K
EY POINTS
membrane appears bulging, opaque, and erythematous or yellow with an aberrant light reflex.
1. The three most
common bacteria implicated in
Pneumatic otoscopy reveals decreased tympanic
acute OM are S
. pneumoniae, nontypeable H.
membrane mobility.
influenzae, an
d Moraxella catarrhalis.
2. In acute otiti
s media, the tympanic membrane is
bulging, opaqu
e, and erythematous or yellow.
Differential Diagnosis
Mobility is di

minished as assessed by pneumatic


Otitis media with effusion is diagnosed when there
is apparent fluid behind the tympanic membrane
ubes should be considered for
(reduced mobility on pneumatic otoscopy) or the
recurrent episodes of acute OM.
tympanic membrane is immobile, but there is no eviurrent infection predisposes to perdence of inflammation (tympanic membrane is not
ive hearing loss.
red or yellow, there is no fever, there is no evidence

otoscopy.
3. Tympanostomy t
children with
4. Chronic or rec
manent conduct

--------------------------------------- 46
14

Blueprints Pediatrics

act out by abusing others, attempting suicide, runninexamineg


d for Neis
seria gonorrhoeae, Chlamydia iraaway, or engaging in high-risk behaviors. Abuse places
chomatis, and other
sexually transmitted diseases.
children at an increased risk for poor school perforOther studies includ
e blood tests for syphilis and
mance, low self-esteem, and depression.
human immunodeficien
cy virus.
Physical Examination
Treatment
As with burns, the location and pattern of injury may
strongly suggest abuse (Figure 2-3). Bruises, fractures,
are required by law to report

Health care workers


any suspicion of chi

ld abuse or neglect to state


or lacerations in different stages of healing occur in
Victims should be immediately
chronic or repeated abuse. Spiral fractures in young

protection agencies.

children are virtually diagnostic; rib and skull fracomes and placed in protective

removed from their h


custody at a hospita

l or a state facility. Many family


tures frequently result from abuse as well. Vigorous
s that focus on social support
shaking may lead to acceleration/deceleration injury,
including subdural hemorrhage. Eighty-five percent
are being evaluated across the
of infants who have been shaken will have retinal or
t to provide children with safer

intervention program

and parenting skills


country in an attemp
home environments.

vitreous hemorrhages, or both. Permanent vision loss


may result if these hemorrhages are left untreated.
SUDDEN INFANT DEATH
Diagnostic Evaluation

SYNDROME

A skeletal survey and bone scan reveal areas of


n infant death syndrome (SIDS)
past injury that may not be evident on physical
pected death of an infant for
examination. Children with extensive bruising should

By definition, sudde
consists of the unex

which the etiology r


emains unclear despite a thorundergo coagulation studies to rule out hematologic
tmortem evaluation. The cause
abnormalities. When sexual abuse is suspected, rectal,
own but is thought to be
oral, vaginal, and urethral specimens should be

ough history and pos


of SIDS remains unkn

related to delayed m
aturation of brainstem respiratory control and aro
usal mechanisms.
Risk Factors
Although multiple fa
ctors have been associated with
an increased risk fo
r SIDS, none has proven prognostic value (Table
2-3). More cases are reported
during the winter mo
nths. African-American infants
are twice as likely
(and American Indians three
times as likely) to
die of SIDS than the general
population.
TABLE 2-3
SIDS: Risk Factor
s
Prone sleeping po
sition

Intrauterine drug exposure


Belt

Hand

Prematurity

Deficient prenatal care


Apnea
Low socioeconomic status
Buckle
Perinatal asphyxia

Maternal smoking
Age 2-4 mo

Figure 2-3

Body marks consistent with abuse.

--------------------------------------- 47
1.40
rints Pediatrics
panosclerosis), cholesteatoma formation, and chroniWhec
ns are noted on the palms and
suppurative OM. Most spontaneous perforations dusolee
ally on the buttocks), the more

n similar lesio
s (and occasion

to OM resolve within 24 to 72 hours without speinclusiv-foot-and-mouth disease is


cific treatment. Labyrinthitis and mastoiditis may
result from direct spread of organisms into the inner
ear and mastoid air cells, respectively.

e name hand
used.
_

YNGITIS
..SINUSITJS

.STREPTOCpCCAL PHAR

Pathogenesis
Group A beta-hemolyt

ic streptococci (Streptococcus
The maxillary and ethmoid sinuses are present at
pyogenes) are the mo
st important cause of bacterial
birth; the sphenoid and frontal sinuses develop latepharyngitisr
. Antim
icrobial therapy for streptococcal
in childhood. The spectrum of pathogens responsible
disease is recommend
ed because of the frequency of
for sinusitis is virtually identical to that for OM.
suppurative (periton
sillar abscess, retropharyngeal
abscess) and nonsupp
urative (rheumatic feyer, postSinusitis is often difficult to diagnose in a young child
since the classic symptoms of headache, facial pain,
streptococcal glomer
ulonephritis) complications.
and sinus tenderness may be absent or difficult to
articulate. Acute bacterial sinusitis has two common
clinical presentations: (1) persistent respiratory
Epidemiology
symptoms (>10-14 days), including either nasal dis"Strep throat" affli
cts older children and adolescents;
charge (clear or purulent) or a daytime cough, anid
t is rare before ag
e 3. The organism is spread person
(2) severe symptoms including high fever and puruto person through in
fected oral secretions.
lent nasal discharge for at least 3 days. The differential diagnosis includes viral upper respiratory tract
infections, allergic rhinitis, and nasal foreign body.
Clinical Manifestati
ons
Computed tomography (CT) is quite reliable at
History and Physical
Examination
detecting mucosal thickening, air-fluid levels, and
Classic symptoms inc
lude sore throat, fever,
opacification but is not routinely required for diagnosis. Antibiotic coverage is similar to that for OM,
headache, malaise, n
ausea, and occasionally abdomialthough treatment should continue for 10 to 21
nal pain. Physical e
xamination reveals enlarged, erythematous, exudative
tonsils and tender cervical
days. Persistent infections may require surgical
lymphadenopathy. Pet
echiae may be present on the
drainage. Complications are uncommon but include
soft palate. Rhinorr
hea, hoarseness, and coughing,
bony erosion, optic neuritis, orbital cellulitis, and
intracranial extension. Children with recurrent or
the hallmarks of vir
al upper respiratory tract infecchronic sinusitis should be evaluated for cystic fibrotions, are notably a
bsent. The diagnosis of scarlet
fever is made when a
characteristic erythematous,
sis, ciliary dyskinesia, or primary immune deficiency.
"sandpaper-like" ras
h accompanies the fever and

pharyngitis. The ras


h appears on the neck or trunk,
spreads to the extre
mities, and may desquamate 10
to 14 days later.
Herpangina is a symptom complex caused by
enteroviruses, most notably coxsackie A. It is typiDifferential Diagnos
is
cally diagnosed during the spring and summer in
Differentiating vira
l pharyngitis and infectious
younger children. Initially, the patient develops a
mononucleosis from s
treptococcal pharyngitis may
high fever and very sore throat. On examinationb,
e impossible based
on clinical symptoms; definitive
characteristic vesicular lesions that progress to ulcerdiagnosis
s requires
either throat culture or antigen
are scattered over the soft palate, tonsils, and
detection test for g
roup A streptococci.
pharynx. Primary herpetic gingivostomatitis (caused
by herpes simplex virus) presents in a similar manner,
Diagnostic Evaluati
on
although the lesions are generally more widespread
over the gums, lips, and niucosa. Herpangina is selfTherapeutic decis
ions should be based on throat
e or rapid anti
limited (5 7 days) and requires no specific therapycultur.
gen detection test results. The

--------------------------------------- 48
Chapt
er 12 / Infectious Disease 141
specificity of most rapid antigen tests is greater
arthritis predominate; longthan 95% (compared with throat culture), so falsets from valvular destruction
positive test results are rare. The sensitivity of rapid
al or aortic valve insufficiency
antigen tests ranges from 80% to 90%, meaning falsepisodes respond favorably to
negative results occasionally occur.
flammatory drugs, and cardiac

cardiac murmur, and


term morbidity resul
with consequent mitr
or stenosis. Acute e
antibiotics, anti-in
management. ARF may

recur after the initial episode.


Treatment
ls diagnosed with ARF should

Therefore, individua
receive prophylactic

penicillin therapy to prevent


Patients with documented group A streptococcal
pharyngitis should receive a 10-day course of oral
coccal glomerulonephritis may
penicillin (or a single dose of intramuscular benzathine penicillin G) to prevent acute rheumatic fever.
A streptococcal pharyngitis or

recurrent ARF.
Acute poststrepto
follow either group
scarlet fever and is

not prevented by antibiotic


Erythromycin, azithromycin, and clindamycin are
nifestations follow infection by

therapy. Clinical ma

acceptable alternatives for children allergic to peniabout 10 days and in


clude hematuria, edema, oligcillin. The treatment of scarlet fever is identical to
on. Complement (C3) levels are
that for streptococcal pharyngitis.
sts of penicillin therapy and
Acute rheumatic fever (ARF) occurs 3 to 4 weeks

uria, and hypertensi


low. Treatment consi
diuretics; steroids

are rarely indicated. In contrast to


after streptococcal pharyngitis in a small percentage
majority of affected children
of untreated patients. ARF is an inflammatory con-

affected adults, the


recover without rena

l sequelae.
dition involving connective tissues of the heart
(carditis, valvular destruction), joints (migratory polyarthritis), and central nervous system (transient
chorea). Diagnosis rests on fulfilling the Jones criteEY POINTS
ria (Table 12-3). Initially, fever, dyspnea, chest pain,

K
1. Children with p

haryngitis should not be treated


with antibiotic
s empirically since most episodes
will be due to
viruses.Therapeutic decisions
TABLE 12-3
should be based
on throat culture or rapid
Revised Jones Criteria for the Diagnosis of Acute
on test results.
Rheumatic Fever

antigen detecti
2. Acute rheumatic

fever involves the heart, joints,


Major Manifestations
Carditis
tococcal glomerulonephritis may
Polyarthritis
kin or pharyngeal infection and is
Sydenham's chorea
y antibiotic therapy.
Erythema marginatum
Subcutaneous nodules
Minor Manifestations
Clinical

and brain.
3. Acute poststrep
follow either s
not prevented b

_ .MONONUiCLJEpSlS _
Fever
Arthralgia
Laboratory
Elevated C-reactive protein or erythrocyte

Pathogenesis
Infectious mononucle

osis is an acute self-limited


sedimentation rate
ur during primary infection with
P-R interval prolongation on electrocardiogram

illness that may occ


Epstein-Barr virus o

r cytomegalovirus.
Additional Criteria
Supporting evidence of preceding streptococcal

infection
Positive throat culture for group A streptococci or
Positive rapid antigen test or
by exchange of infected saliva
Increased streptococcal antibody titer*
ssing disease"). Most infections

Epidemiology
Transmission occurs
(hence the term "ki

are acquired in ear


ly childhood, resulting in asympDiagnosis of acute rheumatic fever requires two major or one major
and two minor criteria plus supporting evidence of antecedent grouptomatic inf
ection or mild illness. In contrast, the
A streptococcal infection.
mononucleosis syndr
ome occurs most frequently
Antibody tests include antistreptolysin-O (ASO), anti-DNase B,
antihyaluronidase, or antistreptokinase.
among individuals i
nfected in late childhood or
adolescence.

--------------------------------------- 49
142

Blueprints Pediatrics

Clinical Manifestations
vations of hepatic transaminases.

mild to moderate ele


A heterophile antibo

dy test allows rapid detection of


History and Physical Examination
EBV-associated monon
ucleosis (but not CMV) in the
The predominant symptom is usually a severe,
outpatient setting;
however, it has limited sensitivity
exudative pharyngitis. Fever, profound fatigue, and
in children younger
than 8 years. Specific serologic
generalized lymphadenopathy occur. Other manifesavailable for EBV (Figure 12-1]
tations include splenomegaly, palatal petechiae, jaun-

antibody testing is
and CMV.

dice, and rash. Although the pharyngitis usually


resolves within a week, the malaise may last much
longer.

Treatment
The disorder is typi

cally self-limited, requiring only


supportive care. Act
ivity restrictions (i.e., no contact
Differential Diagnosis
sports) are advised
until any associated splenomegaly
Classic mononucleosis caused by EBV accounts

resolves because of
the possibility of splenic rupture.
for most cases. In CMV infection, typical signs of
Rare but serious
complications include upper
mononucleosis are present in only half the patients.
airway obstruction,
splenic rupture, and meningoenOther infectious agents that cause similar symptoms
nt treatment with anipicillin
include Toxoplasma gondii, human herpesvirus 6, and
characteristic (but harmless)
HIV. Pharyngitis caused by group A streptococci or

cephalitis. Concurre
often precipitates a

rash. Immunocompromi
sed individuals are at risk for
adenovirus is difficult to distinguish from that of
severe disseminated
disease and lymphoproliferative
mononucleosis without laboratory studies. Pancy-

disorders.

topenia in the presence of the clinical manifestations


listed previously suggests malignancy.
K
EY POINTS
Diagnostic Evaluation
1. Classic mononuc
leosis is due to Epstein-Barr virus.
Leukocytosis or leukopenia may be present; lymstations of mononucleosis include
phocytes account for more than 50% of leukocytes,
ngitis, generalized lymphadenoand at least 10% are usually atypical lymphocytes.
nd fatigue.
Most children develop mild thrombocytopenia and
Primary infection
Convalescent serology
640
320
IgG
73
O
JD
nuclear_antigen
40

2. Clinical manife
exudative phary
pathy, fever, a

20
0
2 4

4 6

Weeks

Months
Time following onset of illnes

s
Figure 12-1

Appearance of antibodies during Epstein-Barr virus infection.

--------------------------------------- 50
Chapt
er 12 / Infectious Disease 143
CROUP
The term croup refers to virus-induced inflammation
of the subglottic tissues, resulting in a syndrome of
upper airway obstruction. Croup usually is due to
parainfluenza virus, but can also be caused by other
viruses, such as influenza and respiratory syncytial
virus (RSV). It is most pronounced in young children
because of the narrow caliber of the airway below
the vocal cords (subglottic region), but also afflicts
adolescents and adults. Incidence peaks during the
spring and late fall. At its most severe, the disease
progresses to partial or total airway obstruction.
Clinical Manifestations
History and Physical Examination
Children typically develop a hoarse voice, barky
("seal-like") cough, and stridor, which may progress
to respiratory distress. Many children have a prodrome consisting of low-grade fever and rhinorrhea
12 to 24 hours prior to the onset of stridor. Respiratory compromise varies from minimal stridor with
agitation to severe distress with tachypnea, hypoxia,
nasal flaring, retractions, and impending airway
obstruction.
Differential Diagnosis
The differential diagnosis of upper airway obstruca 3-year-old. Note the "steeple sign"
tion includes epiglottitis, bacterial tracheitis, foreign
ttic narrowing.
body aspiration, anaphylaxis, and angioneurotic
edema.

Figure 12-2-Croup in
indicative of subglo

respond well to cool


night air or humidity. In the
Diagnostic Evaluation
dorous infants receive oral,

emergency room, stri


intravenous, or intr

amuscular steroids; occasionally,


The diagnosis usually is made on the basis of clinical
findings. Neck radiograph reveals a tapered, narrow
inephrine is used to shrink
subglottic airway in the anteroposterior view (steeple
ding respiratory failure and

nebulized racemic ep
airway mucosa. Impen

sign) (Figure 12-2) or widening of the hypopharynx


onstitute medical emergencies
in the lateral view. These findings are present in 50%
cordingly (see Chapter 1).
of cases and do not correlate with disease severity.
Direct laryngoscopy in the operating room is recommended in cases when the diagnosis is unclear and
EY POINTS
the child is critically ill (see "Epiglottitis").

airway obstruction c
and are addressed ac

K
1. Children with c

roup develop a hoarse voice, barky


("seal-like") c
ough and stridor, which may progress
Treatment
distress.
Most children with croup never become symptovere stridor caused by croup are
matic enough to prompt a visit to the pediatrician.
eroids and nebulized epinephrine.
They are usually treated at home; cough and stridor

to respiratory
2. Infants with se
treated with st

--------------------------------------- 51
144
rints Pediatrics
_ EPIG LOTTITIS
Pathogenesis
Epiglottitis consists of inflammation and edema of
the epiglottis and aryepiglottic folds. It is considered
a life-threatening emergency because of the propensity of the swollen tissues to result in sudden and irreversible airway occlusion.
Epidemiology
H. influenzae type b (Hib) was the most common
cause in the past, but cases due to S. pneumoniae
and group A streptococci increasingly are reported.
Because of routine administration of the Hib vaccine
since the late 1980s, the incidence of epiglottitis has
decreased dramatically. Most cases occur during the
winter months in children 3 to 5 years old.
Risk Factors
Failure to receive Hib vaccination is the greatest risk
factor for epiglottitis.
Clinical Manifestations
History and Physical Examination
Fever, sore throat, hoarseness, and stridor develop
itis in a 4-year-old with massive edema of
over 1 to 2 days. On examination, the child has a
kened aryepiglottic folds, and effacement of
toxic appearance and is in severe respiratory distress.
The child with impending airway obstruction drools
and leans forward with chin extended to maximize
airway patency.
the face of rapidly progressive

Figure 12-3

Epiglott

the epiglottis, thic


the valleculae.
cannot be secured in
obstruction. Intrave

nous ampicillin-sulbactam proDifferential Diagnosis


piric coverage until blood and
The differential diagnosis is similar to that for croup.

vides appropriate em
throat cultures take

n in the operating room provide


identification and a
ntibiotic sensitivity profile of the
infecting organism.
Diagnostic Evaluation
Lateral neck radiographs show "thumb-printing" of
the epiglottis (Figure 12-3). Though radiographs may
Y POINTS
aid in diagnosis, they are not recommended since
a life-threatening emergency.
they delay appropriate care.
ient has a toxic appearance, with

KE
1. Epiglottitis is
2. The typical pat
drooling and se

vere, progressive respiratory


Treatment
is is suspected, the child should be
The child with suspected epiglottitis requires timely
the operating room for endotratransportation to the operating room and emergent
n and direct visualization under
endotracheal intubation. Emergency cricothyroidotsia.
omy may be performed if an endotracheal airway

distress.
3. When epiglottit
transported to
cheal intubatio
general anesthe

--------------------------------------- 52
Chapt
er 12 / Infectious Disease

145
lethargic. Hypoxia i

s common in severely affected


BRONCHIOLITIS

patients.

Pathogenesis
Bronchiolitis is an acute viral lower respiratory tract
is
infection that results in an inflammatory obstruction

Differential Diagnos

of the peripheral airways. There is a predominantly


ted with bronchiolitis may be
lymphocytic infiltrate into the peribronchial and
uish from reactive airway disease
peribronchiolar epithelium that promotes submudy in older infants. Causes of
cosal edema. Intraluminal mucous plugs and cellular
f wheezing include vascular
debris accumulate due to impaired mucociliary
is, and ciliary dyskinesia.
clearance.
n
Epidemiology
tured from nasal secretions;

The wheezing associa


difficult to disting
or airway foreign bo
recurrent episodes o
rings, cystic fibros
Diagnostic Evaluatio
The virus may be cul
however, immunofiuor

escence of nasopharyngeal
Respiratory syncytial virus causes 65% of cases, while
iral antigens is a rapid and practiparainfluenza, influenza, and adenovirus are respon-

aspirate to detect v
cal alternative. Che

st x-ray should be obtained for ill


sible for the remaining 35%. Bronchiolitis occurs
and for those with recurrent or
between November and April. At least half of all chil.
dren are infected with RSV during the first year of

or hypoxic patients
unexplained wheezing

life, and recurrent infections are common. Between


0.5% and 1% of all children with bronchiolitis

Treatment

require hospitalization. The disease is most severe in


ring children require hospitalinfants younger than 3 months, former premature
th oxygen saturation greater than
infants, and children with underlying heart or lung
tory distress, good fluid intake,
disease.
and good follow-up may be

Hypoxic or ill-appea
ization. Children wi
94%, minimal respira
reliable caretakers,
treated as outpatien

ts.
Most infants requ
ire only supportive care for their
Risk Factors
self-limited illness
. The benefit of bronchodilators
Children with chronic lung disease, congenital heart
is controversial. While betadisease, and congenital or acquired immunodeficienntly improve respiratory sympcies are more susceptible to severe disease. Predictors
pear to shorten the duration of
of severe illness include respiratory rate greater than
zation. Alpha-adrenergic agents
70/minute, hypoxia, atelectasis on chest radiograph,
given by inhalation, may be
and history of preterm birth.
the hospitalized infant. Cortico-

and corticosteroids
agonists may transie
toms, they do not ap
illness or hospitali
such as epinephrine,
more beneficial for
steroids have not be

en clearly shown to affect the


course of the diseas
e; however, administration early
Clinical Manifestations
ness may benefit those infants with

in the course of ill


a familial predispos

ition to reactive airways disease.


History
ral agent that suppresses viral
The acute illness lasts for 5 to 10 days, followed by
ity, may shorten symptoms,
gradual recovery over the next 1 to 2 weeks. Infected
e considered in patients with
neonates may develop life-threatening apnea. Infants
ung disease or immunosuppresinitially develop fever, cough, and rhinorrhea followed by progressive respiratory distress. Household
ravenous immunoglobulin with
contacts usually have upper respiratory symptoms.

Ribavirin, an antivi
RNA polymerase activ
and its use should b
underlying chronic l
sive conditions.
RespiGam, an int
high RSV antibody co

ncentration, and palivizumab,


an injectable RSV mo
noclonal antibody, provide
Physical Examination
and are recommended during the
Findings on exam include fever, tachypnea, and mild
tients at risk for severe disease
to severe respiratory distress. Wheezing, rhonchi,
premature infants).
crackles, and accessory muscle use during respiration
te for hospitalized patients is
(tugging) may be noted. Ill infants may be restless or
hildren with congenital heart

passive prophylaxis
winter months for pa
(especially former
The mortality ra
approximately 1%. C

--------------------------------------- 53
146
Blueprints Pediatrics
defects, chronic lung disease, and immunodeficienccontaiy
m, which may be detected by
fare particularly poorly. Patients with documented
staining, PCR, or culture. The
RSV bronchiolitis have more airway hyperresponis normal, but nonspecific infilsiveness later in life than the general populationtrate;
cause versus effect has not been elucidated.

n the organis
fluorescent antibody
chest x-ray usually
s may be seen.
Treatment

KEY POINTS
Young infants with s
evere disease should be hospi1. Bronchiolitis is a self-limited but potentially severe talized to manage ap
nea, cyanosis, hypoxia, and
infection in infants, especially those with underly- feeding difficulties
. Erythromycin shortens the duration of illness if g
iven early in the catarrhal phase.
ing conditions.
After the coughing p
aroxysms begin, antibiotics do
2. Bronchiolitis is due to lower airway obstruction
and, therefore, most children develop wheezing or
not affect the cours
e of illness but are recommended
rhonchi.
to decrease the peri
od of infectivity. A 14-day course
3. Apnea is a frequent presentation in neonates.
completely eradicate
s the organism from the
nasopharynx and resp
iratory tract. Household and
other close contacts
require chemoprophylaxis with
erythromycin.
PERTUSSIS
The acellular per
tussis vaccine is 95% effective
against severe illne
ss, although at least one-third of
immunized individual
s are susceptible to mild infecInfection with Bordetella pertussis causes upper restions later in life.
piratory tract infection and persistent cough in adults
but may result in life-threatening respiratory disease

in neonates and infants.


K
EY POINTS
Clinical Manifestations
pertussis is the long, stridulous
History and Physical Examination
er the paroxysmal cough.
Patients with pertussis are almost invariably afebrile.
th a predominance of atypical

1. The "whoop" in
inspiration aft
2. Leukocytosis wi
lymphocytes is

typical of pertussis.
The classic presentation in young children is "whoopice is erythromycin.
ing cough." The catarrhal phase consists of 1 to 2
weeks of low-grade fever, cough, and coryza. Then
comes a 2- to 4-week paroxysmal phase characterized by paroxysms of cough followed by sudden

3. The drug of cho

^PNEJJMJDNJA ___ ....


, ...
inhalation, which produces the characteristic whoop.
Post-tussive emesis is common. Most symptoms
Pathogenesis
remit during the convalescent phase, but the cough
may last for 2 to 8 weeks. Newborns with severPneumonie
a refers
to an acute inflammatory process
disease may present with apnea or the typical paroxoccurring in the l
ungs. It may be infectious or
ysmal cough followed by choking and progressivnoninfectiouse
.
cyanosis. The characteristic "whoop" is absent in very
young infants since they cannot generate sufficient
negative inspiratory force. Facial petechiae and scleral
Epidemiology
hemorrhages often develop as a consequence of
The age of an immuno
competent child suggests an
forceful coughing.
etiologic organism (
Table 12-4). Viruses are the most
common cause of pneu
monia in young children.
Chlamydia trachomati
s pneumonia manifests at 2 to
Diagnostic Evaluation
3 months of age in i
nfants born to women with
Laboratory evaluation usually reveals leukocytosis
untreated genital C.
trachomatis infection. S.
(>30,000 white blood cells per U.L) with a predomipneumoniaeshou
ld be considered in any comnance of lymphocytes. Nasopharyngeal secretions
munity-acquired lowe
r respiratory tract infection.

--------------------------------------- 54
Chapt
er 12 / Infectious Disease
TABLE 12-4

147

Causes of Pneumonia by Age


<1 Month 1 to 6 Months
School Age/Adolescent

6 Months to 5 Years

Group B streptococci
RSV
influenza,Mycoplasma pneumoniae

RSV, parainfluenza,
adenovirus

Gram-negative enteric bacilli Influenza


niae
Chlamydia pneumoniae
Staphylococcus aureus
Parainfluenza
ae
Streptococcus pneumoniae
Cytomegalovirus
Streptococcus pneumoniae
s
Influenza
Listeria monocytogenes
Chlamydia trachomatis
ulosis
Mycobacterium

Streptococcus pneumo
Haemophilus influenz
Staphylococcus aureu
Mycobacterium tuberc

tuberculosis
Haemophilus influenzae
Group A streptococci
Staphylococcus aureus
Staphylococcus aureus
RSV, respiratory syncytial virus.
Mycoplasma pneumoniae pneumonia is uncommon iann
t onset of fever, chills, dyspnea,
children younger than 5 years. Less common bacteranypical. Productive cough is more
ial causes include nontypeable H. influenzae, Scommo.
patients. M. pneumoniae and C.
aureus, and group A streptococci.
present initially with fever,

d lethargy. Abrup
d chest pain is t
n in older
pneumoniae pneumonia
headache, and myalgi

a. These symptoms gradually


subside over 5 to 7
days, while coughing increases
Risk Factors
eeks or more.
Conditions associated with an increased risk of bac-

and persists for 2 w

terial pneumonia include the following:

Physical Examination
Any indication of re

spiratory distress can signal pneuChronic lung disease, including cystic fibrosis
nea and dyspnea are most
Neurologic impairment with swallowing dysfunccommonout of proportion to fever is an
tion
eumonia in the young child.
Gastroesophageal reflux with aspiration of gastric
rackles suggest involvement
contents
the lung, characteristic of viral
Upper airway anatomic defects (tracheo-sophageaol
oniae, C. pneumoniae, C. trafistula, cleft palate)
Focal findings such as focal
Hemoglobinopathies (including sickle cell diseasecrackle)
d breath sounds, dullness to perImmunodeficiency or immunosuppressive therapcussiony
d bronchophony suggest pneu-

monia, although tachyp


. Tachypnea
important clue to pn
Diffuse wheezing and c
of multiple areas of
r atypical (M. pneum
chomatis] pneumonia.
s or decrease
, egophony an
monia of bacterial o

rigin. Cyanosis is uncommon

except in severe dis


ease. Approximately 10% of
Clinical Manifestations
umoniae infection develop a
History
matous and maculopapular and

patients with M. pne


rash, usually erythe
occasionally erythem

a rnultiforme.
Viral pneumonia develops gradually over 2 to 4 days.
It is usually preceded by upper respiratory symptoms
such as cough, rhinorrhea, postnasal drip, coryza, and
sis
low-grade fever. Infants with pneumonia caused by

Differential Diagno

C. trachomatis are afebrile and have conjunctivitis


ore common in the pediatric
and a staccato cough. Infants and young children
other conditions with similar
with bacterial pneumonia may present with nonpresentationscluding congestive heart failure,
specific constitutional complaints, including fever,
s, pulmonary embolism, sarirritability, poor feeding, vomiting, abdominal pain,
ry or metastatic malignancy.

Pneumonia is much m
population than are
, in
chemical pneumoniti
coidosis, and prima

--------------------------------------- 55
Blueprints Pediatrics
148
Diagnostic Evaluation
Y POINTS
A thorough history and physical examination suggest
the diagnosis. Sputum culture is not likely to be
s the most common cause of bachelpful, since pediatric patients generally do not
a. Amoxicillin or ampicillin is the
produce sputum samples. Chest x-ray remains an
oice.
excellent test for defining the extent and pattern of
hould be considered in older chilinvolvement and assessing related complications (i.e.,
cents. Macrolide antibiotics are

KE
1. S. pneumoniae i
terial pneumoni
treatment of ch
2. M. pneumoniae s
dren and adoles
the treatment o

f choice.
pleural effusion, pneumatocele). Bacterial pneumoinfiltrate on chest radiograph may
nia causes lobar consolidation. Diffuse interstitial
ologic agent
infiltrates suggest viral or atypical pneumonia,
though children with Mycoplasma pneumonia may
have lobar consolidation. Aspiration pneumonia is
typically located in the right middle or right upper
lobe. C. trachomatis pneumonia can be diagnosed by
direct fluorescent antibody testing of conjunctival or
nasopharyngeal specimens. M. pneumonias infection
may be diagnosed by PCR of specimens obtained by
can infect the leptomeninges

3. The pattern of
suggest the eti

MENINGITIS
Pathogenesis
Almost any pathogen

nasopharyngeal swab or by specific antimycoplasmal


uid. Viral meningitis is typically
IgM antibody determination. Cold-agglutinin titers
ed illness; bacterial meningitis is a
are elevated not only in M. pneumoniae infections but
dition associated with substantial
also in many cases of viral pneumonia and some cases
ity. The term aseptic meningiof bacterial pneumonia.
eal inflammation caused by an

and cerebrospinal fl
an acute, self-limit
life-threatening con
morbidity and mortal
tis refers to mening
antigenic stimulus o

ther than pyogenic bacteria (e.g.,


enterovirus or Eorre
lid).
Treatment
Therapy depends on the most likely pathogen. In the
outpatient setting, amoxicillin is appropriate for
most cases of bacterial pneumonia when antibiotics
of meningitis depends on age
are thought to be necessary. Amoxicillin-clavulanic
es and children younger than
acid or a second- or third-generation cephalosporin
st risk for bacterial meningitis.
may be necessary when H. influenzae or S. aureus
meningitidis are the most
are suspected. Erythromycin, clarithromycin, or
rganisms. Hib vaccine has
azithromycin is recommended for "walking pneumoinfluenzae type b meningitis
nia" caused by M. pneumoniae or C. pneumoniae.
. Both infants and older children
Erythromycin is used to treat infants with infection
ngitis caused by enteroviruses
caused by C. trachomatis.
se of viral meningitis).
Any child with persistent hypoxia or moderate to
ate primarily in the late summer
severe respiratory distress requires hospitalization.
meningitis, caused by Borrelia
Intravenous ampicillin is appropriate initial therapy
affects school-age children and
for hospitalized children with suspected bacterial
uses of meningitis and meninpneumonia, though second- or third-generation
de EBV, Bartonella henselae
cephalosporins are often used because of concern
), M. pneumoniae, M. tuberculosis,
regarding resistant S. pneumoniae. Most viral infecanoformans.
tions are self-limited.
The most frequent complication is development
of a pleural effusion large enough to compromise
respiratory effort. Pleurocentesis with possible chest
terial meningitis are the same as
tube placement provides rapid relief. Empyema
cause most cases follow hematogeresults when purulent fluid from an adjacent lung
invasion (nonhematogenous)
infection drains into the pleural space. Lung abscesses
f trauma, mastoiditis, sinusitis, and

Epidemiology
The likely etiology
(Table 12-5). Neonat
3 years are at highe
S. pneumoniae and N.
common responsible o
nearly eliminated H.
in the United States
are at risk for meni
(the most common cau
Enteroviruses circul
and early fall. Lyme
burgdorferi, usually
adolescents. Rare ca
goencephalitis inclu
(cat-scratch disease
d Cryptococcus ne
Risk Factors
Risk factors for bac
those for sepsis, be
nous seeding. Direct
occurs as a result o

may complicate anaerobic infections.


the scalp or skull. In the neonate,

anatomic defects in

--------------------------------------- 56
Chapt
er 12 / Infectious Disease
TABLE 12-5

149

Causes of Meningitis by Age


<1 Month 1 to 2 Months
School Age/Adolescent
Group B streptococci
Escherichia coli
oniae
Streptococcus pneumoniae
Escherichia coli
Streptococcus pneumoniae
dis
Neisseria meningitidis
Other gram-negative bacilli Enteroviruses
Enteroviruses
Herpes simplex virus
Haemophilus influenzae
i
Borrelia burgdorferi
type b*

2 Months to 6 Years

Listeria monocytogenes
zae

Haemophi/us influen

Group B streptococci

Streptococcus pneum
Neisseria meningiti
Enteroviruses
Borrelia burgdorfer

type b*
Streptococcus pneumoniae
* Rare in immunized populations.
low birth weight, prolonged rupture of membranes,
is
and chorioamnionitis predispose to septicemia and

Differential Diagnos

meningitis; myelomeningocele also increases the risk.


gnosis includes encephalitis,

The differential dia


which may develop co

ncurrently or subsequently
(see Chapter 15). Ot
her conditions that may present
Clinical Manifestations
cal picture include drug intoxicaHistory
, recent anoxia or hypoxia, primary

with a similar clini


tion or side effects
or metastatic centra

l nervous system (CNS) maligViral meningitis is preceded by a nonspecific proocarditis with septic embolism,
drome including fever, malaise, sore throat, and myalgias. Children then develop nausea, vomiting,
age/hematoma, malignant hyphotophobia, irritability, lethargy, headache, and stiff
elination disorders.
neck. Unless complicated by encephalitis, symptoms
of enteroviral meningitis generally resolve over 2 to

nancy, bacterial end

intracranial hemorrh
pertension, and demy

4 days and may improve after lumbar puncture.


n
Lyme meningitis is characterized by low-grade fever,
iagnostic. Cell counts and dif-

Diagnostic Evaluatio
Lumbar puncture is d
ferential, Gram stai

n, glucose, and protein levels


headache, stiff neck, and photophobia developing
. Blood and CSF cultures
over the course of 1 to 2 weeks. Cranial nerve palsies
Bacteria are detected on Gram
may occur. In bacterial meningitis, the prodromal

should be determined

phase is absent and the fever is generally quite high.


s of bacterial meningitis. PCR
Mental status changes, focal neurologic signs, ataxia,
s simplex virus (HSV),
seizures, and shock are not uncommon.
yme disease are available and are

should be obtained.

stain in 80% of case


assays for CSF herpe
enteroviruses, and L
highly sensitive and

specific. CSF findings that sugPhysical Examination


logy are described in Table 12-6.
Infants may present with a bulging fontanelle. In
ld not be attempted in a child
older children, signs of increased intracranial pressure
c deficits until an expanding
include cranial nerve palsies and papilledema.
ded by CT or magnetic resonance
Nuchal rigidity and positive Kernig's (flexion of the
the potential for brainstem herleg at the hip with subsequent pain on knee extenaindications include cardiopulsion) and Brudzinski's (involuntary leg flexion on
nd skin infection overlying the
passive neck flexion) signs are markers for meningeal
.
irritation. These findings are rarely present in chil-

gest a specific etio


Lumbar puncture shou
with focal neurologi
mass lesion is exclu
imaging, because of
niation. Other contr
monary instability a
lumbar puncture site

dren younger than 1 year. Patients with meningitis


caused by N. meningitidis may present with petechial

Treatment

or purpuric skin lesions. Arthralgias are common


f uncomplicated viral meningiwith meningococcal meningitis. An erythema
hospitalization is generally not
migrans rash may accompany Lyme meningitis.
ial meningitis cannot be excluded,

When the diagnosis o


tis is unequivocal,
necessary. If bacter

--------------------------------------- 57
Chapter 2 / Poisoning, B
15
urns, and Injury Prevention
Differential Diagnosis
Y POINTS
Cases that initially appear to be SIDS may in fact
result from infection, congenital heart disease, meta-

KE
1. Together, accid

ents and injuries are the most


bolic disorders, accidental trauma, or abuse.
pediatric morbidity and

common cause of
mortality.
2. Poisoning is us

ually accidental in toddlers and


Prevention
adolescents.
Since the "Back To Sleep" campaign was initiated
places a child at high risk for

intentional in
3. Lead poisoning
developmental d

elay and behavior problems.


by the National Institutes of Health, the incidence of
of seat belts and car seats is
SIDS has decreased by 43%. Infants should be placed
e in reducing the incidence of
on their backs while sleeping. Contrary to popular
nd death.
belief, home apnea monitors do not decrease the likedence that driver education prolihood of SIDS.
fective deterrent to accidents

4. The routine use


highly effectiv
severe injury a
5. There is no evi
grams are an ef
involving teena

ge drivers.
6. Certain pattern
s of injury or burns suggest abuse.
7. Babies should b
e put to bed on their backs.
8. Home apnea moni
tors do not decrease the likelihood of SIDS.

--------------------------------------- 58
Blueprints Pediatrics

150

TABLE 12-6
Cerebrospinal Fluid Findings Suggesting a Specific Etiology
for
Meningitis in Childhood
CSF Parameter

Bacterial

Viral

White blood cells (per mm3)

>1200

<500

Neutrophils

>75%

<50%*

Protein

Tt

Normal or T

Lyme
<100
<30%
Normal or T
Glucose

I or -Ixl

Normal

Normal
'Neutrophils may predominate early in the course of viral
meningitis; mononuclear cells usually
predominate in Lyme meningitis.
CSF, cerebral spinal fluid; T, mild increase; TT, moderat
e or severe increase; 4., mild decrease; 4-i,

moderate or severe decrease.


the patient should be hospitalized for intravenous

GASTROENTERITIS

antibiotic therapy.
Vancomycin plus a third-generation cephalosporin
rhea by a variety of mechanisms.
(cefotaxime or ceftriaxone) achieve therapeutic
cteria invade intestinal tissue
levels in the CSF and provide broad-spectrum cov-

Pathogens cause diar


For example, some ba

directly, whereas ot
hers secrete injurious toxins
erage of the most likely pathogens in infants and
stion. Viruses, parasites, and proolder children. Neonates should be treated with
le of inflicting disease. Excessive
ampicillin to treat group B streptococci and L. monodration, inadequate nutrition,
cytogenes; cefotaxime is added to treat gram-negative
rmalities, all of which are poorly
pathogens. Once an organism and its susceptibility

before or after inge


tozoa also are capab
stooling causes dehy
and electrolyte abno

tolerated in infants
and small children.
pattern are available, antibiotic coverage may be
adjusted. The course of therapy for bacterial meningitis is usually 10 days. Exceptions include meninClinical Manifestati
ons
gococcal meningitis (5-7 days), Lyme meningitis
(14-28 days), and neonates (14-21 days).

History

The current mortality rate for bacterial meninginclude information about


tis is 30% for neonates and less than 5% for infants
mily members, recent travel,
and older children. However, 15% to 30% of patients
ne status, day-care attendance,
experience some persistent neurologic deficit, most
ater, contact with animals, duracommonly hearing loss, developmental delay, motor
ver, and number, color, and charincoordination, seizures, and hydrocephalus. Morbid-

The history should i

ity and mortality are higher after infection with


acterial causes of gastroengram-negative organisms.
onella spp., Shigella spp., E. coll,

symptoms in other fa
medication use, immu
source of drinking w
tion of symptoms, fe
acter of stools.
The most common b
teritis include Salm
Yersinia enterocolit

ica, and Campylobacter jejuni;


Vibrio cholerae may
be acquired during travel to
KEY POINTS

India, Africa, or th
e Middle East and from eating
1. Meningitis may be septic (bacterial) or aseptic.
st shellfish. Patients with bac2. Immunization with Hib vaccine has dramatically
ent with fever, significant abdomdecreased the incidence of childhood meningitis;
se, and tenesmus; vomiting is less
conjugate pneumococcal vaccine (Prevnar) is
ontain mucous and may be
likely to result in decreased frequency of pneuixed with blood. Occasionally,
mococcal meningitis among infants.
losis present with neurologic
3. Lumbar puncture is invaluable in the diagnosis
argy, seizures, mental status
and treatment strategy of meningitis.
ue to a neurotoxin elaborated by
4. New PCR-based assays facilitate the diagnosis of
ella spp. are capable of invading
HSV, enteroviral, and Lyme central nervous system
causing extraintestinal disease,
infection.
and osteomyelitis (particularly

undercooked Gulf Coa


terial diarrhea pres
inal cramping, malai
common. The stools c
guaiac positive or m
children with shigel
manifestations (leth
changes), possibly d
the organism. Salmon
the bloodstream and
including meningitis
in children with sic

kle cell disease). Shigella dysente-

--------------------------------------- 59
Chapt
er 12 / Infectious Disease
151
riae and E. coli O157:H7 produce an enterotoxin
evaluation if bacterial disease is
(Shiga or Shiga-like toxin) associated with hemolytic
stool culture results take several
uremic syndrome, a serious complication consisting
in determining the need for
of microangiopathic hemolytic anemia, nephropathy,
e is a history of antibiotic use, stool
and thrombocytopenia. Almost 25% of individuals
Clostridium difficile toxins A and
infected with Y. enterocolitica develop subsequent
ting is available for rotavirus. If
erythema nodosum. Patients with chronic giardiasis
is suspected, multiple stool
are at risk for failure to thrive resulting from ongoing
nt times should be examined for
malabsorption.
cent antibody detection in stool
In some patients, particularly those with Yersinia,
diagnose G. lamblia infection.
severe pain localizes to the right lower quadrant,
y be indicated if the diarrhea
creating a "pseudoappendicitis" picture.
no etiology has been found.
In cholera, the stools quickly become colorless
and flecked with mucus, termed "rice-water" stools.

the time of initial


suspected. Bacterial
days but are helpful
antibiotics. If ther
should be tested for
B. Rapid antigen tes
G. lamblia infection
samples from differe
cysts. Immunofluores
can also be used to
Endoscopic biopsy ma
becomes chronic and

Treatment
Severe diarrhea leading to hypovolemic shock may
develop in hours to a few days.
es oral rehydration whenever
Rotavirus is the major cause of nonbacterial gasparenteral therapy may be
troenteritis in infants and toddlers in the Western
ases. Antidiarrheal agents should
world. Infections peak between January and April.

Treatment incorporat
possible; aggressive
required in severe c

be avoided.
Complaints include profuse diarrhea, vomiting, and
t is a febrile infant younger than
low-grade fever. Severe diarrhea may lead to severe
cs should generally be withheld
dehydration, acidosis, and electrolyte disturbances.
lts. Antibiotic therapy prolongs
Giardiasis is the most commonly reported paraand should be reserved for
sitic disease in the United States. More water-related
Antibiotics may enhance the
outbreaks of diarrhea are due to Giardia lamblia than
pment of hemolytic uremic synany other organism. The illness presents with frewith diarrhea caused by E. coli
quent, foul-smelling, watery stools that rarely contain
s persist once culture results
blood or mucus; abdominal pain, nausea, vomiting,
c therapy should be considered.
anorexia, and flatulence often accompany the diarthoxazole is usually effective
rhea. Symptoms generally resolve within 5 to 7 days,
sis. Erythromycin is the treatment
although some cases linger for more than a month.
uni. Patients with C. difficile

Unless the patien


12 months, antibioti
pending culture resu
Salmonella shedding
systemic infections.
likelihood of develo
drome among patients
.O157:H7. If symptom
are known, antibioti
Trimethoprim-sulfame
in treating shigello
of choice for C. jej
enterocolitis usuall

y improve with suspension of


Physical Examination
but if treatment is warranted,
The main goals of the physical examination are estitreatment of choice. Patients
mating the degree of dehydration and ruling out the
also be treated with oral
need for abdominal surgery.

antibiotic therapy,
metronidazole is the
with giardiasis may
metronidazole.
As long as the pa

tient does not develop hypovolemic shock, progn


osis for full recovery is excelDifferential Diagnosis
hreatening cases, appropriate
Acute diarrhea in childhood is usually due to infecnt permanent sequelae.
tion. Other conditions associated with diarrhea
include majabsorption, antibiotic use, cystic fibrosis,

lent. Even in life-t


management may preve

EY POINTS
and inflammatory bowel disease.
1. Infectious diar
rhea may be bacterial, viral, or
parasitic.
Diagnostic Evaluation
2. Careful fluid a
nd electrolyte management is the
Electrolyte and renal function studies (Na
+, K+, Cl~,
treatment in infectious diarrhea.
HCO 3~, BUN, creatinine) guide replacement therapy
higellosis may present with mental
in significantly dehydrated children. Abdominal radiographs are generally normal or nonspecific. Blood,
and E. coli O157:H7 have been assomucus, and fecal leukocytes suggest a bacterial origin
olytic uremic syndrome.
for the illness. Blood culture should be performed at

most important
3. Children with s
status changes.
4. S. dysenteriae
ciated with hem

--------------------------------------- 60
.152

i-ints Pediatrics
ing HBV, HCV, and HD

V. Risk factors for HAV and


HEPATITIS
HEV include foreign
travel, poor sanitation, and
contact with other c
hildren in day care.
Pathogenesis
Acute hepatic inflammation in children can be due
to a large number of infectious and noninfectious
Clinical Manifestati
ons
causes. Viruses that are primarily hepatrophic
include hepatitis A virus (HAV), hepatitis B virus

History

(HBV), hepatitis C virus (HCV), hepatitis D virus


infants are usually asymptomatic.
(HDV, formerly delta hepatitis) and hepatitis E
cute hepatitis include anorexia,
virus (HEV). Features of HAV, HBV, and HCV are
iting, jaundice, dark urine,
compared in Table 12-7.
low-grade fever. Children with

Perinatally infected
Clinical signs of a
nausea, malaise, vom
abdominal pain, and
HAV and HEV may have

diarrhea. However, a wide

range of severity ex
ists, and as many as 30% to 50%
Epidemiology
are asymptomatic. HBV and

of infected children
HCV infection are us

ually silent, in that the patient


HAV and HEV are acquired via fecal-oral transmission. HBV, HCV, and HDV are transmitted by pertoms unless chronic infection

complains of no symp
has caused significa

nt hepatic damage.
cutaneous or mucosal exposure to infectious body
fluids and by vertical transmission from an infected
Physical Examination
mother to her infant. HDV, or delta antigen, consists
of single-stranded RNA. It is a "defective" virus in
jaundice are noted in some chilthat it requires the presence of an active HBV infecof children with HBV, and 20%
tion to replicate. HBV and HCV can persist for many
ith HCV. Hepatomegaly and
years following acute infection. This "carrier state"
tenderness may be present. A
is associated with development of hepatocellular
h may be present early in the
carcinoma.
Risk Factors
sis
Intravenous drug users, those who have unprotected
us, and other viral infections can
sex with multiple partners, and those who receive
s, but other organ systems are
blood transfusions are at increased risk of contract-

Scleral icterus and


dren with HAV, 50%
to 30% of children w
right upper quadrant
benign-appearing ras
course.
Differential Diagno
EBV, CMV, enterovir
also cause hepatiti
usually involved.

TABLE 12-7
Viruses Responsible
Feature

for Hepatitis; Comparison and

Summary

Hepatitis A

Hepatitis B

RNA

DNA

15-45

45-180

Late incubation to early

When HBsAg seropositi

Hepatitis C
Virus type
RNA
Incubation (days)
7-180
Period of infectivity
ve
Unknown

symptomatic state
Fulminant hepatitis
1%
Chronic hepatitis
%-50%
50%

<1%

1%-3%

No

5%-1 0% of adults; 25
of infants; 90% of

neonates

whose mothers are


HBeAg+
s,

Diagnostic evaluation
Anti-HAV IgM
Anti-HCV antibody, HCV

HBsAg, HBeAg, anti-HB

anti-HBc, anti-HBe
PCR
anti-HBc, total antibody to hepatitis B core antigen; anti-HBe, total antibod
y to hepatitis B e antigen; anti-HBs, total antibody to hepatitis B surface
antigen; HAV, hepatitis A virus; HBeAg, hepatitis B e antigen; HBsAg, hepatit
s B surface antigen; HCV, hepatitis C virus.

--------------------------------------- 61
Chapt
er 12 / Infectious Disease
153
Diagnostic Evaluation
e 12-8 present the clinical
Liver enzymes are uniformly elevated in hepatitis.
markers important in diagnos-

Figure 12-5 and Tabl


course and serologic
ing HBV disease stag

e. Anti-HBs heralds resolution


Because the clinical manifestations are so similar, speof the illness and c
onfers lifelong immunity.
cific serologic tests are indispensable for securing an
accurate diagnosis (Tables 12-7 and 12-8). The presresent in both acute and

HCV antibody is p
chronic infection. H

CV RNA can be detected by


ence of anti-HAV IgM antibody confirms HAV infecPCR within 1 week of
infection, whereas the
tion (Figure 12-4). Tests are also available to detect
antibodies to the delta antigen.
infection to antibody response

"window period" from


for HCV may be as lo

ng as 12 weeks. Therefore, the


Three different particles may be found in the
presence of HCV RNA
in the absence of antibody
serum of patients infected with HBV. The Dane
particle is the largest, made up of a core antigen
cute infection. Recovery is char-

response indicates a
acterized by disappe

arance of HCV RNA from the


(HBcAg) and envelope antigen (HBeAg) surrounded
blood.
by a spherical shell of HBsAg ("surface") particles.

Treatment
TABLE 12-8
Both active and pass
ive forms of immunization are
Commparison of Disease States in
on the source of infection. HAV
Hepatitis B Virus
mmended for all children in
Test
Acute HBV Resolved HBV Chronic HBV
ited States where infection is

available, depending
immunization is reco
some parts of the Un
more likely. HAV imm

unoglobulin will prevent clinHBsAg +


ministered within 14 days of
Anti-HBs -

ical disease when ad

exposure. The HBV va


ccine series is recommended
Anti-HBc +
he United States. Infants of infected
HBeAg

for all infants in t

mothers should recei

ve both the vaccine and the


Anti-HBe -

HBV immunoglobulin a
t delivery to prevent
anti-HBc, total antibody to hepatitis B core antigen; anti-HBe, totalthe disea
se and, most important, development of the
antibody to hepatitis B e antigen; anti-HBs, total antibody to hepatitis B
surface antigen; HBeAg, hepatitis B e antigen; HBsAg, hepatitis Bcarrier state
. Alpha-interferon has shown promise in
surface antigen; HBV, hepatitis B virus.
IgG-anti-HAV

^ Serum
,'
'

transaminases
Fecal
excretion
Serum N |gM-anti-HAV
Iransaminases1234
567

3 6 9 12

2 3

Incubation period
Weeks Months
12

Years

3456

24
Time after exposure

onths after exposure


Figure 12-4
The course of acute hepatitis A.

M
Figure 12-5

The course

of acute hepatitis B.
Modified from Shulman ST, Phair JP, Sommers HM.The Biologic and Clini-Modified f
rom Shulman ST, Phair JP, Sommers HM.The Biologic and Clinical Basis of Infectious Diseases, 4th Ed. Philadelphia: W.B. Saunders, 1992:cal
Basis of Infectious Diseases, 4th Ed. Philadelphia: W.B. Saunders, 1992:
315,319.
315,319.

--------------------------------------- 62
154
rints Pediatrics
treating patients with chronic HBV hepatitis; studies
in children are less encouraging. There is no specific
treatment for HDV. Alpha-interferon has been effective in preventing conversion from acute to chronic
y a sexually transmitted disease
HCV hepatitis. Only supportive care is available to
tion with the spirochete
HEV-infected individuals.
The prognosis for patients with hepatitis depends
on the virus responsible.

Pathogenesis
Syphilis is primaril
resulting from infec
Treponema pallidum.
Epidemiology

HAV: Very few patients develop fulminant hepatitis, but the mortality rate among those who do is
atric population may be acquired
almost 50%.
ngenital syphilis) or through

Syphilis in the pedi


transplacentally (co
sexual contact. The

incidence of syphilis has


HBV: HBV may persist as chronic hepatitis, and
the last several years. Cointhe course may be relatively benign or more
exually transmitted diseases is
severe. Chronic persistent hepatitis B is characterized by little cellular inflammation and usually
resolves within a year. Chronic active hepatitis is
more aggressive, progressing to cirrhosis and
increasing the risk of hepatocellular carcinoma.
Chronic infection is more likely among infected
other with untreated infection
children than adults.
enital syphilis. Adolescents and

increased sharply over


fection with other s
common.
Risk Factors
Neonates born to a m
are at risk for cong
adults who have unpr

otected sex with an infected


HDV: When HDV and HBV are acquired simulr primary syphilis.
taneously, the recipient is at greater risk for more
severe chronic hepatitis B and fulminant hepatitis
associated with a higher mortality rate. When an
ons
individual is infected with HDV on top of preexisting HBV, acute exacerbation, and an accelerated
Examination
course result. The risk of progressing to cirrhotic
infants with congenital
liver disease is increased when HDV is present.
who survive are often asympto-

partner are at risk fo

Clinical Manifestati
History and Physical
Approximately 40% of
syphilis die. Those

HCV: Half of those infected with HCV develop


elop symptoms within 1 month
chronic hepatitis with an increased risk for
s with congenital syphilis may
cirrhosis.
plenomegaly, mucocutaneous
HEV: HEV does not appear to result in chronic
phadenopathy, and the charhepatitis.
," a bloody, mucopurulent nasal

matic at birth but dev


if untreated. Infant
have hepatomegaly, s
lesions, jaundice, lym
acteristic "snuffles
discharge. Other fin

dings include deafness and


retardation.
Syphilis acquired
through sexual contact progresses through thre
e stages. After a 2- to 4-week
KEY POINTS
nfected individuals enter the
1. HAV and HEV are spread via fecal-oral transmishilis, characterized by the classic
sion. HBV, HCV, and HDV are transmitted through
lation site: a well-demarcated,
infected bodily fluids.
less genital ulcer with an
2. Clinical signs of acute hepatitis include anorexia,
use the lesion heals spontanausea, malaise, vomiting, jaundice, dark urine,
6 weeks, patients with primary
abdominal pain, and low-grade fever. However, a
t seek medical attention.
wide range of severity exists, and as many as 30%
is is frequently manifested by
to 50% of infected children are asymptomatic.
gic involvement coinciding
3. Liver enzymes are uniformly elevated in hepatitis.
f the spirochete throughout
Because the clinical manifestations are so similar,
ows the primary stage directly,
specific serologic tests are indispensable for
cre is still present. The typical
securing an accurate diagnosis.
eralized (including the soles and

incubation period, i
primary stage of syp
chancre at the inocu
firm, strangely pain
indurated base. Beca
neously within 3 to
syphilis often do no
Secondary syphil
widespread dermatolo
with dissemination o
the body. Onset foll
often while the chan
rash consists of gen
palms), erythematous

macules (3-1 Omm) that

--------------------------------------- 63
Chapt
er 12 / Infectious Disease
155
progress to papules. Some patients also develop
t neurosyphilis, but positive
systemic symptoms including fever, malaise, pharynic. Infants may develop radigitis, mucosal ulcerations, and generalized lyrnes of the long bones. Anemia
phadenopathy; patchy alopecia and thinning of the
may also develop in

vated protein sugges


CSF VDRL is diagnost
ographic abnormaliti
and thrombocytopenia

lateral third of the eyebrow have also been associated

untreated infants.

with secondary syphilis. Symptoms of secondary


syphilis resolve in 1 to 3 months.
Tertiary syphilis develops years after primary
exposure and is rare in the pediatric population.
n G remains the treatment of
Granulomatous lesions called gummas destroy
of infection and fully eradicates
surrounding tissues, especially in the skin, bone,
e body. Doxycycline may be
heart, and CNS. Unfortunately, tertiary syphilis may
re allergic to penicillin.
occur without any previous primary or secondary

Treatment
Parenteral penicilli
choice for any stage
the organism from th
used for those who a

manifestations.
Differential Diagnosis
EY POINTS

K
1. Syphilis may be

transmitted transplacentally or
Syphilis is one of the great masqueraders, a disease
with a wide spectrum of presentation. The presence
ongenital syphilis present with
of the rash, if characteristic, greatly aids in diagnosis.
atosplenomegaly, mucocutaneous

sexually.
2. Neonates with c
"snuffles," hep
lesions, jaundi

ce, and lymphadenopathy.


Diagnostic Evaluation
re diagnosed in the secondary

3. Most patients a
stage of syphil

is, when widespread dermatologic


Chancre scrapings (and mucosal secretions in
are present.
infected neonates) demonstrate rapidly mobile
R are excellent screening tests
organisms moving in a corkscrew-like motion under
false positives.
dark-field microscopy. Aspiration of enlarged lymph
cillin G is the treatment of choice.
nodes may also yield the organism. Both the VDRL

manifestations
4. The VDRL and RP
but may produce
5. Parenteral peni

(developed by the Venereal Disease Research Laboratory of the U.S. Public Health Service) and the
rapid plasma reagin (RPR) are excellent blood
screening tests for high-risk populations, providing
rapid, inexpensive, quantitative results. Both are nonIMPLEX
treponemal tests for antibodies to a lipoidal molecule
rather than the organism itself. Both are considered

li GENITAL HERPES S
VIRUS INIFECTION

highly sensitive when titers are high or when the test


ly results from infection with
is complemented by historical or physical evidence
type 2. Small mucosal tears or
of the disease. However, infectious mononucleosis,
ulated with the virus, usually
connective tissue disease, endocarditis, and tubercuty. Genital herpes is one of the

Genital herpes usual


herpes simplex virus
skin cracks are inoc
during sexual activi
most common sexually

acquired diseases; 10% to


losis may all result in false-positive VDRL and RPR
a history suggestive of prior
results. By contrast, treponemal tests, such as the flution. Transmission of HSV from
orescent treponemal antibody absorption (FTA-ABS)
the time of birth may result in
and microhemagglutination test (MHA-TP), are
n in the newborn.
much less likely to produce false positives, unless

20% of adults have


genital herpes infec
mother to infant at
devastating infectio

Lyme disease is present. A positive screening VDRL


or RPR coupled with a positive FTA-ABS in a
ions
newborn or sexually active adolescent is virtually
diagnostic of untreated syphilis. Nontreponemal tests
Examination
may become negative after treatment, whereas treubation period (5-14 days),
ponemal studies remain positive for life.
itching progress to vesicular,
Neonates with suspected congenital syphilis
ons. These burst to form painful,
require lumbar puncture. CSF pleocytosis and eleheal without scarring. Fever,

Clinical Manifestat

History and Physical


After a variable inc
genital burning and
often pustular lesi
shallow ulcers that

--------------------------------------- 64
Blueprints Pediatrics
.156
pharyngitis, headache, and malaise may accompany
ons, History, Physical
the primary episode. After acquisition, the virus
gnostic Evaluation
ascends peripheral nerves to dorsal root ganglia,
where it may lie latent or recur periodically. Recuris of PID is based on the presrences have fewer symptoms than the primary
ed and one of several supporting

Clinical Manifestati
Examination, and Dia
The clinical diagnos
ence of three requir
symptoms:

episode, and asymptomatic shedding may occur.


Required: Abdominal
pain and tenderness, cervical
Diagnostic Evaluation
, adnexal tenderness
Giant multinucleated cells with intranuclear incluure greater than 38C, leukosions are found in scrapings from the ulcerous base.

motion tenderness
Supporting: Temperat
cytosis, elevated

ESR, greater than 20 white blood


cells per high-po
wer field or intracellular gramHSV may be cultured from the active lesions or
ci on endocervical smear, white
asymptomatic infected individuals in 1 to 4 days;
and/or bacteria on culdocentesis,
rapid antigen testing is also available.
by ultrasound
Treatment
is
Acyclovir (topical or parenteral) diminishes the
nditions and intra-abdominal
length of both symptoms and shedding but does not
ed in the differential diagnosis:
eradicate the organism. It has limited efficacy in
recurrent episodes. Continued prophylactic use of
pregnancy, ruptured ovarian
oral acyclovir has been shown to prevent or reduce
tion
the frequency of recurrences.
ndicitis, pyelonephritis, in-

negative diplococ
blood cell count
inflammatory mass
Differential Diagnos
Other gynecologic co
pathology are includ
Gynecologic: Ectopic
cyst, septic abor
Nongynecologic: Appe
flammatory bowel

disease
_.PELVIC JNFJLAMM/gpRY DISEASE _
Pathogenesis
ould be treated for both N.

Treatment
Patients with PID sh
gonorrhoeae and C. t

rachomatis. A single dose of a


Pelvic inflammatory disease (PIDJ is a constellation
neration cephalosporin, such as
of signs and symptoms related to the ascending
ime, is sufficient to eradicate N.
spread of pathogenic organisms from the lower
female genital tract to the cervix, endometrium, and
loxacin, ofloxacin, or spectinofallopian tubes.
well. Azithromycin eradicates

long-acting third-ge
ceftriaxone or cefix
gonorrhoeae', ciprof
mycin may be used as
C. trachomatis in o

ne large dose; alternatively, a 7-day


course of doxycyclin
e is appropriate. Significant
Epidemiology
ore extensive courses of therapy.

infections require m
Patients who are adm

itted to the hospital for severe


Over 1 million cases of PID occur annually in the
essure instability should receive
United States. The etiology is generally polymicrobiotics, including both a thirdbial, with Chlamydia trachomatis and Neisseria
orin and doxycycline, and pregonorrhoeae being by far the most common isolates.
Barrier contraceptive methods are protective. N. gonf infected women will become
orrhoeae or genital C. trachomatis infection in a preingle episode of PID. Other gynecopubertal child strongly suggests sexual abuse.
include increased risks for

vomiting or blood pr
therapy with IV anti
generation cephalosp
ventive education.
Twenty percent o
infertile after a s
logic complications

ectopic pregnancy,
dyspareunia, chronic pelvic pain,
Risk Factors
s capable of invading the bloodRisk factors include age (adolescence), sexual interorgan system. Joint involvement
course with multiple partners, unprotected interarthritis may affect only one
course, and preexisting mucosal sexually transmittejoind
yarticular and migratory with
disease.
vitis and skin lesions. Although C.

and adhesions.
N. gonorrhoeae i
stream and thus any
is most common. The
t or may be pol
associated tenosyno

--------------------------------------- 65
Chapt
er 12 / Infectious Disease
157
trachomatis seldom causes systemic illness, untreated
Infection is usually asymptoindividuals may go on to develop Reiter's syndrome
hin, white, foul-smelling dis[a constellation of urethritis, conjunctivitis, and
"fishy" odor when mixed with
arthritis). Fitz-Hugh-Curtis syndrome, a form of perThe clinical diagnosis is based
ihepatitis, is a known complication of infection with
much more common in sexually
either organism.
appearance and odor of dis-

data remain unclear.


matic except for a t
charge that emits a
potassium hydroxide.
on patient history (
active females), the
charge, a vaginal pH

greater than 4.5, and characteristic "clue" cells o


n the wet prep (squamous epithelial
KEY POINTS
borders caused by adherent bac1. C. trachomatis and N. gonorrhoeae are the most
a single dose of metronidazole
common isolates in pelvic inflammatory disease.
e infection. Concurrent antibiotic
2. The diagnosis of PID is clinical, based on history,
rtners seems to have no effect
physical examination, and supporting laboratory
results.
3. A long-acting third-generation cephalosporin (for
N. gonorrhoeae) and azithromycin (for C. traasis is not a sexually transmitted
chomatis) constitute appropriate outpatient
re colonized with Candida;
therapy in mild infections.
h as antibiotic use, pregnancy,

cells with "smudged"


teria). Once again,
effectively cures th
treatment of male pa
on recurrence rates.
Vaginal Candidiasis
Vulvovaginal candidi
disease. All women a
however, factors suc
diabetes, immunosupp

ression, and oral contraceptive


use predispose women
to candidal overgrowth
I N FECTIONS

(moniliasis). Signs

and symptoms include a thick


white vaginal discha

rge with vaginal itching and


burning. Yeast and p
seudohyphae are apparent on
Trichomoniasis, bacterial vaginosis, and Candida
h potassium hydroxide. Overvaginitis are all bothersome but relatively benign
tifungal creams are safe and genvaginal infections collectively manifested by changes
casionally, oral antifungal agents
in the amount and character of vaginal secretions.
Clinical Manifestations and Treatment

wet prep treated wit


the-counter local an
erally effective. Oc
are required.
K

EY POINTS
Trichomoniasis
Trichomoniasis results from sexually transmitted
is diagnosed by demonstrating
Trichomonas vaginalis, a mobile flagellated protozoan.
nads on fresh wet prep and is
Most infected individuals remain asymptomatic,
tronidazole.
although urethritis is not uncommon in men. Typical
osis, often caused by M. hominis,
symptoms in women include a malodorous, frothy
cted when the vaginal pH is
gray discharge and vaginal discomfort. Some patients
5 and clue cells are seen on wet
also develop dysuria and vague lower abdominal pain.
zole is effective treatment.
The cervix and vaginal mucosa may be either normal
or visibly irritated and inflamed. A fresh wet prep of
the vaginal fluid reveals polymorphonuclear leukoD
cytes and the characteristic motile trichomonads.
Metronidazole, in a single 2-gram oral dose, is the
treatment of choice for patients and their partners.
Bacterial Vaginosis
that infects and kills CD4 T lymBacterial vaginosis, long thought to be harmless, is
in progressive immunodeficiency.
now known to increase the risks of PID, chorioamunt for 1% to 2% of the total in
nionitis, and premature birth. Bacterial vaginosis is
ost infections in children are
caused by Gardnerella spp., Mycoplasma hominis, anacquired
or perinatally (80%); smaller
various anaerobic organisms. The epidemiology of
blood product transfusions and
the disease suggests sexual transmission, although the
HIV produces a wide range of

1. Trichomoniasis
motile trichomo
treated with me
2. Bacterial vagin
should be suspe
greater than 4.
prep. Metronida

HIV AND ACQUIRE

Pathogenesis
HIV is a retrovirus
phocytes, resulting
Pediatric cases acco
the United States. M
d in utero
numbers result from
sexual transmission.

--------------------------------------- 66
Blueprints Pediatrics
158
clinical manifestations in children, the most severe
he child is considered to have
of which is acquired immunodeficiency syndrome
he absolute CD4 count.
(AIDS}.

conditions occurs, t
AIDS regardless of t

Differential Diagnos
is
Epidemiology
eat masquerader" because of its
The risk of HIV transmission from a seropositive
n; the virus can affect any organ
mother to her fetus is approximately 20% to 30%.
are often nonspecific.
Treatment of infected pregnant women with antiretrovirals during the third trimester, followed by
treatment of the infant for the first 6 weeks of life,
n
has been shown to reduce the vertical transmission
positive mothers are always

HIV has become a "gr


variable presentatio
system, and symptoms

Diagnostic Evaluatio
Infants born to HIVseropositive for mat

ernally derived IgG antibodies to


rate to about 8%. Asymptomatic HIV-positive
enzyme immunoassays used for
women may not realize that they are infected and,
lations are not helpful in children
therefore, often do not receive therapy. IV drug abuse
age. If the mother is HIV posiis reported in approximately 70% of infected
the infants' blood should be
women. The disease is more common in urban pop1 month, and 4 months of age, and
ulations, lower socioeconomic classes, and racial
e should be sent at 1 month of
minorities.
of HIV DNA PCR and HIV

the virus; thus, the


screening older popu
before 18 months of
tive, HIV DNA PCR of
performed at birth,
blood for HIV cultur
age. The combination
culture will detect

98% of all positive infants by 1


Risk Factors
e than 99% by 4 months of age.
Risk factors include birth to an HIV-positive mother,
birth to a woman who uses IV drugs and shares
needles, and birth to a woman with multiple sexual
partners who does not practice safe sex. Other
consists of antiretroviral drugs
groups at risk include patients who received multiine) and ddl (didanosine), prople units of blood products (e.g., hemophiliacs)
d trimethoprim-sulfamethoxazole
before March 1985, victims of sexual abuse, and adoPCP, the most common and
lescents who engage in high-risk behavior.
c infection. New pharmacologic

month of age and mor


Treatment
The standard of care
such as AZT (zidovud
tease inhibitors, an
prophylaxis against
serious opportunisti
therapies have drast

ically improved the chances of


living longer with H
IV infection.
Clinical Manifestations
History and Physical Examination
HIV may present in infants and children with any
EY POINTS
one or several of the following signs and sympions in children are acquired in
toms: generalized lymphadenopathy, hepatomegaly,
tally (80%); smaller numbers result
splenomegaly, failure to thrive, recurrent or chronic

K
1. Most HIV infect
utero or perina
from blood prod

uct transfusions and sexual


diarrhea, oral candidiasis, parotitis, and developmental delay. Respiratory manifestations include lymHIV-positive mothers are always
phoid interstitial pneumonia and Pneumocystis carinii
r maternally derived IgG antibodies
pneumonia (PCP). Regression in developmental
hus, the enzyme immunoassays used
milestones, progressive encephalopathy, and CNS
lder populations are not helpful in
lymphoma are unfortunate neurologic complica18 months of age.
tions. Cardiomyopathy and nephropathy also occur.
ons of pediatric HIV are varied.
Recurrent, often severe, bacterial and opportunistic
asymptomatic or present with
(fungal, disseminated HSV or CMV, and Mycobacral of the following signs and
terium avium) infections are the hallmark of the
pathy, hepatomegaly,
acquired helper T-cell immunodeficiency.
ailure to thrive, recurrent or
PCP, CNS lymphoma, and lymphocytic interstitial
a, oral candidiasis, parotitis, and
pneumonitis (LIP) are considered AIDS-defining illelay.
nesses in the pediatric population. When any of these

transmission.
2. Infants born to
seropositive fo
to the virus; t
for screening o
children before
3. The manifestati
Children may be
any one or seve
symptoms: adeno
splenomegaly, f
chronic diarrhe
developmental d

--------------------------------------- 67
Chapt
er 12 / Infectious Disease

159
introduced into the

skin by a tick bite and subseVIRAL INFECTIONS


lymphatics and blood vessels.
OF CHILDHOOD

quently spread via the

They invade and mult


iply within the endothelial and
Viral infections are quite common in the infant
of blood vessels, causing generand young child but decrease with age because of
ry that leads to hypoalbuminemia,
acquired immunity. Several viral illnesses that are freand hypotension.
quently encountered in the pediatric population are

smooth muscle cells


alized vascular inju
edema, hypovolemia,

not usually seen in adults. Many of these present

Epidemiology

with characteristic rashes that permit reliable clinical


en between April and
diagnosis. Live attenuated vaccines are routinely adfested areas of the south Atlantic
ministered to prevent measles, mumps, rubella, and

RMSF occurs most oft

varicella (chickenpox). Roseola and erythema infectes. Despite the name, none of

and southeastern sta

September in tick-in

tiosum are generally benign in children. Table 12-9


eporting RMSF is near the Rocky
describes the typical presentations and complications

the top ten states r


Mountains.

of these viral illnesses in children, which are discussed further in Chapter 5.

Risk Factors
The most significant

risk factor is residence in or


travel to an endemic
area during times of the year
ROCKY MOUNTAIN
SPOTTED

when ticks are active.

FEVER

Pathogenesis
ons
Rocky Mountain spotted fever (RMSF) is a tickExamination
borne disease caused by Rickettsia rickettsii, a gramo 14 days after a tick bite.
negative intracellular bacterium. Rickettsiae are
en are nonspecific and include
TABLE 12-9

Clinical Manifestati
History and Physical
Symptoms develop 2 t
Initial symptoms oft

Presentations and Complications of Childhood Viral Illnesses


Virus
Exanthem
Complications
Measles
Confluent, erythematous
vitis,Pneumonia, myocarditis,
maculopapular rash
cal
encephalitis; rare: subacute
that starts on head and
e)
sclerosing panencephalitis
progresses caudally
Mumps
None
Orchitis, pancreatitis; rare:

Other Features
Coryza, cough, conjuncti
Koplik's spots (on buc
mucosa early in diseas

Swollen salivary glands,


especially parotid gla

nds
meningitis, encephalitis
Rubella
Similar to measles but
or
Polyarticular arthritis or
does not coalesce
thy
arthralgias; rare: encephalitis
Roseola (human
Maculopapular
ash Febrile seizures; rare:
herpesvirus 6)
meningoencephalitis
Erythema infectiosum
Facial erythema giving
in Arthritis; rare: encephalitis
(fifth disease;
"slapped cheeks"
parvovirus B19)

appearance followed
by spread to extremities
in reticular pattern

Suboccipital and posteri


auricular lymphadenopa
High fever resolves as r
appears
Transient aplastic crisis
child with
hemoglobinopathy

Chickenpox (varicella) Pruritic, erythematous


ve, newSecondary bacterial
macules evolve to
ions
infection; rare: pneumonia,
vesicles and then crust
e
cerebellar ataxia,
over; begins on face and
y
encephalitis, hepatitis
spreads to extremities

As initial lesions resol


crops form so that les
in different stages ar
observed simultaneousl

--------------------------------------- 68
Cardiology
EVALUATION OF THE
r and delivery; and neonatal
CYANOTIC NEONATE
ained. Exactly when the child

tions; history of labo


course should be obt
developed cyanosis i

s critical, because certain conCyanosis is a physical sign characterized by blue


s present at birth, while others
mucous membranes, nail beds, and skin. Cyanosis
one month to present themselves.
results from an absolute concentration of deoxycal examination should focus on

genital heart defect


may take as long as
The initial physi
the vital signs and

cardiac and respiratory examinagenated hemoglobin of at least 3.0g/dL. Factors that


vidence of right, left, or biventricinfluence whether cyanosis will appear include the

tions, looking for e

hematocrit, which reflects the absolute concentration


ular congestive hear
t failure and respiratory distress.
of hemoglobin, and the factors that affect the O
2 Blue or dusky mucous
membranes are consistent
dissociation curve (pH, PCO
2, temperature, level of with cyanosis. Evalu
ate for rales, stridor, grunting,
2,3-diphosphoglycerate, and ratio of adult to fetal
flaring, retractions
, and evidence of consolidation or
hemoglobin). Cyanosis should not be confused with
effusion on pulmonar
y examination. On cardiovasacrocyanosis, which is blueness of the extremities
cular examination, t
he precordial impulse is paldue to peripheral vasoconstriction noted in the first
pated, and the clini
cian should evaluate for systolic
or diastolic murmurs
, the intensity of Si, S
2 splitting
24 to 48 hours of life. Neonates with acrocyanosis
abnormalities, and t
he presence of an S
3 or 84 gallop,
have pink mucosal membranes.
ejection click, open
ing snap, or rub. Examination of
Differential Diagnosis
ld focus on the strength and

the extremities shou


symmetry of the puls

es in the upper and lower


The causes of cyanosis in the newborn are of cardiac,
e of edema, and cyanosis of the
pulmonary, neurologic, or hematologic origin. The
enomegaly may be consistent
incidence of structural heart disease is about 8 in
ar or biventricular heart failure.
1000 live births, and severe congenital heart disease

extremities, evidenc
nail beds. Hepatospl
with right ventricul

occurs in approximately 1 in 400 live births. Pulmonary disorders may lead to cyanosis as a result of
n
primary lung disease, airway obstruction, or extrinsic
ial evaluation of the cyanotic
compression of the lung. Neurologic causes of
ine whether the cyanosis is
cyanosis include central nervous system dysfunction
c in origin. An electrocardiogram
and respiratory neuromuscular dysfunction. Table 3aph, and hyperoxia test should
1 delineates the causes of cyanosis in the neonate.
ition, preductal and postductal

Diagnostic Evaluatio
The goal of the init
neonate is to determ
cardiac or noncardia
(ECG), chest radiogr
be performed. In add
oxygen saturation, a

nd four extremity blood pressures should be docu


mented.
Clinical Manifestations
should be carried out in neonates
History and Physical Examination
oximetry reading less than 95%,
A complete birth history that includes maternal
circulatory collapse. The hyperoxia
history; prenatal, perinatal, and postnatal complicaaining a baseline right radial

A hyperoxia test
with a resting pulse
visible cyanosis, or
test consists of obt

--------------------------------------- 69
Blueprints Pediatrics
160
fever, chills, headache, malaise, and myalgias. The rash
begins on the third or fourth day and consists of erythematous, maculopapular lesions that progress to
been reported across the

Epidemiology
Although cases have
country, most occur

in southern New England, southform petechiae or purpura (corresponding to a widew Jersey, eastern Pennsylvania,
spread small-vessel vasculitis). It characteristically
Minnesota, and Wisconsin. The
appears initially on the wrists and ankles and spreads
sease is highest among children
proximally to involve the trunk and head over several
hours. Typically, the palms and soles are involved as
well. The rash is absent in 5% of children. Approximately 30% of children have some impairment of
mental status.
reased occupational or recre-

eastern New York, Ne


Maryland, Delaware,
incidence of Lyme di
5 to 10 years old.

Risk Factors
Individuals with inc

ational exposure to
tick-infested woodlands in
endemic areas are at
highest risk of Lyme disease. An
Differential Diagnosis
Meningococcemia and measles (especially atypical
eed for more than 48 hours to

infected tick must f


transmit B. burgdorf

eri.
measles) may be confused with RMSF. Ehrlichiosis,
another tick-borne rickettsial infection, is usually
associated with neutropenia; rash is present in up to
ons
50% of children.

Clinical Manifestati
History
Most patients do not

recall a tick bite. The clinical


Diagnostic Evaluation
d on the stage of the disease

manifestations depen
early localized, ear

ly disseminated, or late. Erythema


Although immunofluorescent staining of skin bioptation of early localized disease,
sies taken from rash sites may demonstrate the organof the tick bite 7 to 10 days after
ism, there is no reliable diagnostic test that becomes
egins as a red macule or papule
positive early enough in the course of the disease to
guide therapy. Thus, the clinician must maintain a
rm a large, annular, erythematous

migrans, the manifes


appears at the site
the bite. The rash b
and progresses to fo
lesion with central

clearing that is about 15 cm in


high suspicion for the disease. Antibodies to confirm
esion often is accompanied by
the clinical diagnosis are detectable approximately
laise, headache, arthralgias, and
10 days after symptom onset. Key laboratory features
eminated Lyme disease (3-5
include thrombocytopenia, elevated liver enzymes,
e) may manifest as multiple eryhypoalbuminemia, and hyponatremia.
s (anywhere on the body),

diameter. The skin l


low-grade fevers, ma
myalgias. Early diss
weeks after tick bit
thema migrans lesion
cranial nerve palsy,

meningitis, and carditis (heart


block). The most com
mon manifestation of late
Treatment
ks after tick bite) is arthritis,
Treatment with doxycycline is effective. Mortality is
e knee.
higher in those receiving treatment more than 5 days
after symptom onset and those with gastrointestinal
symptoms.
earlier, may be present. Children

Lyme disease (>6 wee


usually involving th
Physical Examination
The rash, described
with early dissemina

ted Lyme may have multiple


erythema migrans les
ions, facial nerve palsy, or signs
of meningitis. Child
ren with Lyme arthritis may have

_ .LyjyiE_DISEASE
oint.
Pathogenesis
Lyme disease is a tick-borne illness resulting from
is
infection with the spirochete Borrelia burgdorferi.
gnosis depends on the presentaThe disease was first described 20 years ago in Lyme,
is atypical, it may be confused
Connecticut; isolation of the causative organism ocorme or erythema marginatum
curred several years later.
ever). The differential diagnosis

a swollen and tender j


Differential Diagnos
The differential dia
tion. When the rash
with erythema multif
(seen in rheumatic f

--------------------------------------- 70
Chapt
er 12 / Infectious Disease
161
of arthritis also includes juvenile rheumatoid arthriTreatmen-

tis, reactive arthritis, and Reiter's syndrome. The


differential diagnosis of Lyme meningitis includes
ocalized Lyme disease prevents
other causes of aseptic meningitis.
nd late disease, including menin-

Treatment of early l
early disseminated a
gitis and arthritis.

Younger children can be treated


with amoxicillin or
cefuroxime. Penicillin-allergic
Diagnostic Evaluation
ted with erythromycin. Children

children can be trea


older than 8 years s

hould receive oral doxycycline


For the most part, early localized Lyme disease is
hildren with severe arthritis or
a clinical diagnosis, based on suggestive history and
c involvement warrant parenteral
the characteristic rash on physical examination. The
organism cannot be reliably cultured from the skin
se penicillin G or ceftriaxone.
lesions, blood, and other body fluids. Lyme IgM titer
remit within 30 days of
is elevated several weeks after the tick bite. Lyme
to be due to residual infection,

for 14 to 30 days. C
cardiac or neurologi

therapy with high-do


Symptoms that do not
therapy are unlikely
and another cause sh

ould be sought.
PCR of CSF (or joint fluid) reliably diagnoses Lyme
meningitis (or arthritis). Cardiac involvement, in the
EY POINTS
form of conduction abnormalities, is rare but can be
diagnosed by electrocardiogram in conjunction with
h, erythema migrans, may be
supporting history and antibody studies.
fever, headache, and arthralgia.

1. The typical ras


accompanied by

Testing for Lyme disease in the presence of vague


treated with amoxicillin in chilor nonspecific complaints is not helpful; falsean 8 years, and with doxycycline
positive test results can occur. Antibodies to B.
en. Lyme meningitis requires
burgdorferi cross-react with other infectious agents,

2. Lyme disease is

particularly other spirochetes, including syphilis,


hen treated appropriately, does
although VDRL and RPR remain negative in patients
ic symptoms.
with Lyme disease.

3. Lyme disease, w

dren younger th
in older childr
ceftriaxone.

not cause chron

--------------------------------------- 71
Chapter
Neonatology
_BJRTH Apgar Scoring
Neonatal Mortality
ion, a rapid scoring system based

The Apgar examinat

on physiologic re
sponses to the birth process, is an
The late fetal and early neonatal period is the time
excellent method
for assessing the need for neonatal
of life exhibiting the highest mortality rate of any age
resuscitation. It
is not generally useful as a prognosinterval. The perinatal mortality rate refers to fetal
tic tool. The Apga
r scoring system is shown in Table
deaths occurring from the 20th week of gestation
13-1. At 1 and 5
minutes after birth, each of five
until the 7th day after birth. Intrauterine fetal death
physiologic parame
ters is evaluated. Full-term infants
(i.e., stillbirth) represents 40% to 50% of the perinawith a normal card
iopulmonary transition will have
tal mortality rate.
a total score of
8 to 9 at 1 and 5 minutes. An Apgar
The neonatal mortality rate includes infants who
score of 0 to 3 in
dicates either cardiorespiratory
die between birth and 28 days of life. Modern neonaarrest or a condi
tion resulting from severe bradycartal intensive care has delayed the mortality of many
dia, hypoventilat
ion, and/or central nervous system
newborn infants who have life-threatening diseases,
depression. Most
low Apgar scores are due to diffiso that they survive beyond the neonatal period only
culty in establis
hing adequate ventilation and not to
to die of their original diseases or of complications oprimarf
y cardiac
pathology.
therapy sometime after the 28th day of life. This
delayed mortality occurs during the postneonatal
period, which begins after 28 days of life and extends

to the end of the first year of life.


The infant mortality rate includes both the neonatal and the postneonatal periods and is expressed a
a
the number of deaths per 1000 live births. The infant

Cephalohematoms

A cephalohematoma
is a traumatic subperiosteal
mortality rate in the United States declined in 1999
y involving the parietal bone)
to 7.1 per 1000 live births. The rate for Africans suture lines. The scalp hematoma
American infants in 1990 was a distressing 14.6 per
1000 live births. There were 27 countries with lower
lly firm without discoloration of
infant mortality rates.
may not become apparent until

hemorrhage (usuall
that does not cros

is characteristica
overlying skin and
hours to days afte

r delivery. Predisposing factors


include large head
size, prolonged labor, vacuum
extraction, and fo
rceps delivery. Spontaneous resoluKEY POINT
tion occurs over s
everal weeks. Two percent of the
1. The 1999 infant mortality rate in the United States
anize, calcify, and form a central
was higher than 27 other countries, and Africancalvarium. Cephalohematoma disAmerican infants were twice as likely to die
t in an indirect hyperbilirubinemia
during the first year of life.
rapy, especially in a premature

hematomas will org


depression in the
solution may resul
requiring photothe
infant.

--------------------------------------- 72
Chapter 13 / Neonatology
TABLE 13-1

163

Apgar Scoring System


Physical

0 Points

1 Point

No pulse

<100

2 Points
Exam Evaluated at
1 and 5 Minutes
Heart rate
>100

Respiratory effort
No respirations
Vigorous cry
Color
Pale, cyanotic
Pink throughout
Muscle tone
Absent
mities
Active
Reflex irritability
Absent
Active cry and avoidance
Caput Succedaneum
and the wrist is flexed. When there
A caput succedaneum is a diffuse, edematous, and
flex in the right arm, and the

Irregular, weak cry


Cyanotic extremities
Weak, slightly flexed extre
Grimace
internally rotated,
is an absent Moro re
right hand grasp is

intact, Erb's palsy should be susoften dark swelling of the soft tissue of the scalp that
nt of these lesions resolve sponextends across the midline and/or suture lines and
s of age, but if the nerve deficit
is commonly found in infants who are delivered
ting may be beneficial.
vaginally in the customary occiput-anterior position.

pected. Ninety perce


taneously by 4 month
persists, nerve graf

Pressure induced from overriding parietal and frontal


bones against their respective sutures causes the
molding associated with the caput. The caput is comEY POINTS
monly seen after prolonged labor in both full-term
ma is a traumatic subperiosteal
and premature infants.
does not cross suture lines.

K
1. A cephalohemato
hemorrhage that
2. A caput succeda

neum is a diffuse, edematous, and


Fractured Clavicle
ling of the soft tissue of the scalp

often dark swel


that extends ac

ross the midline and/or suture


A fractured clavicle is found in 2% to 3% of vaginal
deliveries, and the right clavicle is two times more
res heal without intervention, and
likely to fracture than the left. This predilection
in babies with macrosomia
exists because the right shoulder must move beneath
dystocia.
the pubic symphysis during normal delivery and may
ults from stretching of the fifth and

lines.
3. Clavicle fractu
are most common
and/or shoulder
4. Erb's palsy res
sixth cervical

nerves and should be suspected


get entrapped. Predisposing factors include large size,
n absent Moro reflex of the right
shoulder dystocia, and traumatic delivery. Findings
include swelling and fullness over the fracture site,
ct right hand grasp.
crepitus, and decreased arm movement. Of neonates
with clavicular fracture, 80% have no symptoms and

when there is a

arm and an inta

only minimal physical findings. It is often diagnosed


when a callus is detected at 3 to 6 weeks. Radiograph

PREMATURITY

is not indicated. No specific treatment is necessary.


The parents should be advised to avoid tension on
EW) infants, defined as those
the affected arm.
weights less than 2500 grams (g),

Low-birth-weight (L
infants having birth
represent a dispropo

rtionately large percentage of


neonatal and infant
deaths. Although these infants
Erb's Palsy
ll births, they account for twoInjury to nerves of the brachial plexus results from
al deaths. Very low-birth-weight
excessive traction on the neck, producing paresis.
ghing less than 1500g at birth,
Erb's palsy results from stretching of the fifth and
1 % of all births but account for
sixth cervical nerves. The infant's arm is held in the
ths. In comparison with infants
"waiter's tip" position, where the arm is extended and
ore, LEW infants are 40 times

make up only 7% of a
thirds of all neonat
(VLBW) infants, wei
represent only about
50% of neonatal dea
weighing 2500 g or m

--------------------------------------- 73
Blueprints Pediatrics
T64
more likely to die in the neonatal period, and VLBW
infants have a 200-fold higher risk of neonatal death.
In contrast to the improvements in the overall
Preterm Birth
infant mortality rate, there has not been improvement in the rate of LEW births. This is one reason
that the infant mortality rate of the United States is
the worst of the large, modern, industrialized counetalis
tries. If birth-weight mortality rates are calculated,
fetalis
the United States has one of the highest survival
es
rates, but because of the large number of LEW
infants, the total infant mortality rate remains high.
LEW is caused by premature birth or intrauterine
growth retardation. Maternal factors associated with
having an LEW infant include previous LEW birth,
x
low socioeconomic status, low
achievement, lack of prenatal
than 16 years or greater than
interval between pregnancies,
lness

level of educational
care, maternal age less
35 years, a short time
unmarried status, low

TABLE 13-2
Medical Causes of
Fetal
Fetal distress
Multiple gestation
Erythroblastosis f
Nonimmune hydrops
Congenital anomali
Placental
Placenta previa
Abruptio placenta
Uterine
Bicornuate uterus
Incompetent cervi
Maternal
Preeclampsia
Chronic medical il
Infection (chorio

amnionitis)
prepregnancy weight (less than lOOlb) and/or poor
ially cocaine)
weight gain during pregnancy (less than lOlb), and
African-American race. Maternal use of cigarettes,
of membranes
alcohol, and/or illicit drugs is also associated with
having an LEW infant. Specific medical causes of
preterm birth are listed in Table 13-2.

Drug abuse (espec


Other
Premature rupture
Polyhydramnios
Trauma
Diethylstilbestro

l exposure
KEY POINTS
of postmaturity. The
cause of prolonged pregnancy is
1. Low-birth-weight infants make up 7% of all births
ses.
but account for two-thirds of all neonatal deaths.
2. Very low-birth-weight infants represent 1% of all
births but account for 50% of neonatal deaths.
ons
3. In comparison with infants weighing 2500g or
maturity is characterized by
more, LBW infants are 40 times more likely to die
ad circumference but decreased
in the neonatal period, and VLBW infants have a
this syndrome are distinct from
200-fold higher risk of neonatal death.
l age infants in that they were
4. One reason that the infant mortality rate of the
y went beyond 42 weeks' gestaUnited States is so high is that the rate of LBW
itionally deprived from placenbirths is high. If birth-weight mortality rates are
ommon symptoms include dry,
calculated, the United States has one of the
ose, and wrinkled skin and a
highest survival rates, but because of the large
nce with decreased amounts
number of LBW infants, the infant mortality rate
ues. Conditions that occur more
remains high.
ture infants include meconium
5. LBW is caused by premature birth or intrauterine
ssion at birth, persistent pulgrowth retardation.
of the newborn (PPHN), hypo-

not known in most ca

Clinical Manifestati
The syndrome of post
normal length and he
weight. Infants with
small for gestationa
doing well until the
tion and became nutr
tal insufficiency. C
cracked, peeling, lo
malnourished appeara
of subcutaneous tiss
frequently in postma
aspiration and depre
monary hypertension
glycemia, hypocalcem

ia, and polycythemia.


POSTMATURITY
uld be monitored closely by
Infants whose gestation exceeds 42 weeks are concal profile, and nonstress tests.
sidered postmature and are at risk for the syndrome
t involves preparation for peri-

--------------------------------------- 74

Treatment
Fetal well-being sho
ultrasound, biophysi
Intrapartum treatmen

Chapter 13 / Neonatology
165
natal depression and meconium aspiration. Earlychromosoma,
l abn
ormalities (trisomies or Turner's
feeding to reduce the risk of hypoglycemia ansyndrome)d
, and co
ngenital (especially CNS) malforevaluation for the conditions noted above encompasmatios
n syndromes.
Placental causes include choripostpartum treatment.
onic villitis, chron
ic abruptio placentae, twin-twin
transfusion, placent
al tumor, and placental insufficiency secondary to
maternal vascular disease.
KEY POINTS
Maternal causes of i
ntrauterine growth retardation
1. Infants whose gestation exceeds 42 weeks are
include severe perip
heral vascular diseases that
considered postmature and are at risk for the
reduce uterine blood
flow, such as chronic hypertensyndrome of postmaturity.
sion, diabetic vascu
lopathy, preeclampsia, sickle cell
2. Conditions that occur more frequently in postmaanemia, and cardiac
and renal disease. Other materture infants include meconium aspiration and
nal causes include r
educed nutritional intake, alcohol
depression at birth, persistent pulmonary hyperor drug abuse, cigar
ette smoking, and uterine anomtension of the newborn, hypoglycemia, hypocalalies or uterine con
straint. Uterine constraint is noted
cemia, and polycythemia.
in mothers of small
stature and reduced weight gain
during pregnancy.
Treatment
- JJ^I? AU TJ= RJ N E PROBLEMS _
Infants who are smal
l for gestational age have a high
Small for Gestational Age
e fetal death. Therefore, prenatal
Pathogenesis and Clinical Manifestations
identification, evaluation, and

risk for intrauterin


management includes
monitoring. The stan

dard intrauterine growth retarInfants who are small for gestational age have birth
es a review of obstetric causes,
weights below the 1 Oth percentile for gestational age.
tifiable syndromes, and laboraTwo broad categories of intrauterine growth retardatorcongenital infection. Antepartum
tion have been described: early onset and late onsetfeta.
th serial ultrasound, biophysical
One-third of low-birth-weight neonates infants
t, and oxytocin challenge test is
weighing less than 2500 g are small for gestational
amination of placental flow is
age.
uteroplacental insufficiency

dation workup includ


examination for iden
y evaluation for
l monitoring wi
profile, nonstress tes
often used. Doppler ex
used to determine if

Early-onset, or symmetrical, intrauterine growth


exists. If early del
ivery is being contemplated, deterretardation is thought to result from an insult thaminatiot
n of pulmon
ary maturity is critical. Early
begins before 28 weeks' gestation. The early insult
delivery is necessar
y when it is determined that the
results in a neonate whose head circumferencrise
k to the fetus of
staying in utero is greater than the
and height are proportionately sized and whosrise
k of premature de
livery. Fetal lung maturity can be
weight-for-height ratio is normal. This pattern iaccelerateds
, if n
ecessary, by steroid administration. If
seen in infants whose mothers have severe vasculatherr
e is placental
insufficiency, the fetus may not
disease with hypertension and renal disease otoleratr
e labor and
may require delivery by cesarean
in infants with congenital malformations or chromosection.
somal abnormalities.
take place at a center with a highLate-onset, or asymmetrical, intrauterine growth
se infants who are very small for
retardation starts after 28 weeks' gestation. These
at risk for life-threatening probinfants have a normal, or close to normal, head cirdelivery. The delivery team
cumference with a reduced height and weight. Thshoule
for perinatal asphyxia and/or
weight-for-height ratio is low, and the infant appears
m aspiration, and hypothermia.
long and emaciated. In this type of intrauterine
placenta after delivery for patholgrowth retardation, the neonate initially has a normal
congenital infection or infarction
growth trajectory and follows a normal centile line
etermining the cause of the
and then "falls off" the curve late in gestation.
retardation. The newborn that is

Delivery should
risk nursery, becau
gestational age are
lems at the time of
d be prepared
depression, meconiu
Examination of the
ogy consistent with
may be helpful in d
intrauterine growth
small for gestation

al age should be monitored for


Risk Factors
ycemia, hypocalcemia, hyponaGrowth retardation may result from fetal causes such
a, pulmonary hemorrhage, and
as multiple gestation, congenital viral infections,
y hypertension. Leukopenia,

hypothermia, hypogl
tremia, polycythemi
persistent pulmonar

--------------------------------------- 75
166
rints Pediatrics
neutropenia, and thrombocytopenia may be seen in
k for being large for gestational
infants born to hypertensive mothers. Commencing
f diabetic mothers (class A, B, or C);
feedings as soon as possible minimizes hypoglycemia.
nts; and neonates with transposi-

4. Neonates at ris
age are those o
postmature infa
tion of the gre

at vessels, erythroblastosis fetalis, or

Large for Gestational Age


ann syndrome.
Infants whose weight is greater than 2 standard devio are large for gestational age are
ations above the mean or above the 90th percentile
y large, from large parents or a
are defined as large for gestational age. Neonates at
dilection for large infants.
risk for being large for gestational age are those of
ates are a subcategory of large
diabetic mothers (class A, B, or C); postmature
age infants and have birth weights
infants; and neonates with transposition of the
00 g.They are at significant risk for
great vessels, erythroblastosis fetalis, or Beckwithia.
Wiedemann syndrome. Most infants who are large
for gestational age are constitutionally large, from
large parents or a family with a predilection for large
infants. After birth, the infant should be evaluated for
the disorders just described, as well as birth trauma,
which occurs often in large for gestational age
fined as an amniotic fluid
neonates. The blood sugar of the large for gestational
2 liters; it occurs in 1 in 1000
age infant should be monitored and the child fed
dramnios is associated with preearly, because large for gestational age infants who
have diabetic mothers or who suffer from Beckwithal discomfort, and respiratory
Wiedemann syndrome or erythroblastosis fetalis
en, polyhydramnios is chronic
are prone to hypoglycemia. Obtaining a hematocrit
tational diabetes, immune or nonafter birth is advisable, because large for gestational
is, abdominal wall defects
age neonates have an increased incidence of
troschisis), multiple gestations,
polycythemia.
ural tube defects, and certain
Macrosomic neonates have birth weights greater
of the gastrointestinal tract.
than 4000 g. All macrosomic infants are large for gesngomyelocele are neural tube
tational age, but not all large for gestational age
fetal swallowing, whereas
neonates are macrosomic. Macrosomic infants have
al atresia, diaphragmatic
an increased risk of shoulder dystocia and other birth
late interfere with swallowing and
trauma. Conditions such as maternal diabetes melliid dynamics.
tus, obesity, and postmaturity are associated with an
increased incidence of macrosomia.

Beckwith-Wiedem
5. Most infants wh
constitutionall
family with pre
6. Macrosomic neon
for gestational
greater than 40
shoulder dystoc

Polyhydramnios
Polyhydramnios is de
volume greater than
births. Acute polyhy
mature labor, matern
compromise. More oft
and is seen with ges
immune hydrops fetal
(omphalocele and gas
trisomy 18 or 21, ne
congenital anomalies
Anencephaly and meni
defects that impair
esophageal or duoden
hernia, and cleft pa
gastrointestinal flu

Oligohydramnios
Oligohydramnios is a
ssociated with intrauterine
growth retardation,
amniotic fluid leak, postmaturity,
and congenital anoma

lies of the fetal kidneys.


KEY POINTS
esis results in a specific deforma1. It is useful to divide infants who are small for gesas Potter's syndrome. The syntational age into two categories: symmetrical
ed by club feet, compressed
(early onset) and asymmetrical (late onset or head
, scaphoid abdomen, and diminsparing).
e that is accompanied by pul2. Intrauterine growth retardation may result from
d pneumothorax. Uterine
fetal, placental,or maternal causes.
bsence of amniotic fluid retards
3. Infants who are small for gestational age have a
ients with this condition expire
high risk for intrauterine fetal death; therefore,
re rather than of renal insuffiprenatal management includes identification,
ios increases the risk of fetal
evaluation, and monitoring.
r. This risk may be reduced by

Bilateral renal agen


tion syndrome known
drome is characteriz
facies, low-set ears
ished chest wall siz
monary hypoplasia an
compression in the a
lung growth, and pat
of respiratory failu
ciency. Oligohydramn
distress during labo
normal saline amnioi

nfusion during labor.

--------------------------------------- 76
167
Chapter 13 / Neonatology
KEY POINTS
ons

Clinical Manifestati
Infants infected ear

ly in pregnancy suffer from


1. Chronic polyhydramnios is seen with gestational
encephalitis and present with
diabetes, immune or nonimmune hydrops fetalis,
ephalus, microphthalmia, chorabdominal wall defects (omphalocele and
anial calcifications, and seizures.
gastroschisis), multiple gestations, trisomy 18 or
so appear septic and have jaun21, neural tube defects, and certain congenital
galy, purpura, petechiae, a macanomalies of the gastrointestinal tract.
generalized lymphadenopathy.
2. Oligohydramnios is associated with intrautersymptomatic at birth, 70% will
ine growth retardation, amniotic fluid leak,
uelae, which may include mental
postmaturity, and congenital anomalies of
g disabilities, and chorioretinitis.
the fetal kidneys.
ecome reactivated years after the

intrauterine meningo
microcephaly, hydroc
ioretinitis, intracr
These infants may al
dice, hepatosplenome
ulopapular rash, and
Of infants who are a
suffer long-term seq
retardation, learnin
Ocular disease can b
initial infection, b

oth in healthy and immunocompromised individuals


, resulting in impaired vision or

blindness.
..CpNGENJTALJNFECTIp_NS ___________
the primary means of definitive
Infections of the fetus during the first, second, or
d rise in antibody titer or seroearly third trimester are referred to as congenital
tive to positive indicates the
infections. Classically, they are referred to as
n. In congenital infection, diagTORCH infections, an acronym for toxoplasmosis,
ated by the presence of materTreponema pallidum infection, other infections,
lacental antibody. If the maternal
rubella, cytomegalovirus infection, herpes simplex,
egative, the diagnosis of congeniand HIV. Although it is important to be familiar with
excluded. If maternal and

Serologic tests are


diagnosis. A fourfol
conversion from nega
presence of infectio
nosis may be complic
nally derived transp
antibody status is n
tal toxoplasmosis is
neonate levels are p

ositive, serial studies of antitoxothis acronym, it has several shortcomings. Herpes


al months are necessary to dissimplex and HIV are much more commonly perinatal antibody from congenital
tal (rather than congenital) infections, and there is an
transplacental antibody fall over
ever-expanding list of viruses that could be included
fe, whereas antibody levels from
in the "other" group. The most important congenital
remain stable or rise. A cominfections and their syndromes are discussed in this
) scan of the head may reveal
section. There are many similarities in the congenital
ons in the central nervous system.
syndromes, so focusing on the differences can help
visualized in the cerebrospinal
refine the evaluation. Table 13-3 summarizes diseaseuge preparations or by growth
specific clinical findings and laboratory evaluation.
mice. Typical histopathology or

plasma IgG for sever


tinguish transplacen
infection. Levels of
the first year of li
congenital infection
puted tomography (CT
cerebral calcificati
The parasite may be
fluid by cytocentrif
in inoculated infant
cysts may be identif

ied in biopsy specimens of


involved lung, brain
, bone marrow, or lymph
Toxoplasmosis

node.

Toxoplasmosis is caused by Toxoplasma gondii, an


intracellular protozoal parasite found in mammals
and birds. Members of the cat family are the defini-

Treatment

tive host. Infected cats excrete toxoplasma oocytes iTreatmenn


t include
s both pyrimethamine and sulfaditheir stool, resulting in fecal-oral transmission to
azine, which act syn
ergistically against Toxoplasma.
humans.
These antibiotics in
hibit folic acid, so they are used
There are approximately 3000 cases of congenitainl
conjunction with
folic acid. Corticosteroids are

infection annually in the United States. Only primary


with severe central nervous
infection of the mother, which is usually asymptosystefection.
matic, results in congenital infection. Among the
l-cooked meat and the avoidance
infants of women infected with toxoplasmosis during
areas where cats defecate
the first trimester, less than 20% will be infected, but
oxoplasmosis in pregnant or
their disease will likely be severe. If the mother's
tients. Cat litter should
infection is acquired in the third trimester, as manby
y, because toxoplasma oocytes
as 65% will give birth to an infected neonate, but thare
for the first 48 hours after
infection will be mild or asymptomatic.

reserved for infants


m or ocular in
Ingestion of wel
of cats and soil in
reduce the risk of t
immunocompromised pa
e disposed of dail
e not infectious
passage.

--------------------------------------- 77
.168

Blueprints Pediatrics
TABLE 13-3

Differentiating and Evaluating Some Congenital Infections


Agent
Laboratory Evaluation
Toxoplasma gondii
Toxoplasmosis IgG antibody

Specific Clinical Features


Hydrocephalus with generalized
calcifications; chorioretinitis

followed by IgM, which is more


specific.
Treponema pallidum
Osteochondritis and periostitis;
Nontreponemal test such as RPR or
eczematoid skin rash; snuffles
VDRL, supported by treponemal
test such as IgM FTA-ABS.
Rubella
Eye: Cataracts, cloudy cornea,
Maternal rubella immune status. If
pigmented retina
immune, send infant's IgG and the
Skin:"Blueberry muffin" syndrome
more specific IgM. If IgM is
Bone: Vertical striation
negative, but IgG is positive, viral
Heart: Patent ductus, pulmonary
cultures from urine, cerebrospinal
stenosis
fluid, and throat swabs may isolate
the virus.
Cytomegalovirus
Microcephaly with periventricular
Urine for cytomegalovirus culture or
calcifications;

rapid CMV early antigen test.


hepatosplenomegaly;
chorioretinitis; inguinal hernias
in males; thrombocytopenia
Herpes simplex
from cerebrospinal

Skin vesicles or denuded skin; Viral cultures


keratoconjunctivitis; acute fluid, skin les

ions, conjunctivae,
central nervous system findings urine, bloo
d, rectum, and
such as seizures nasopharynx should grow wi
thin
2-3 days. PCR of CSF.
Direct fluorescent antibody staining
of scraping from skin lesion is
specific but not sensitive.
CMV, cytomegalovirus; CSF, cerebral spinal fluid; FTA-ABS, fluorescent trepon
ema antibody test; PCR, polymerase chain reaction test; RPR, rapid plasma
reagin test; VDRL, Venereal Disease Research Laboratory test.
Syphilis
KEY POINTS
om transplacental transmission of
1. Toxoplasmosis is caused by Toxoplasma gondii, an
Syphilis in the untreated pregintracellular protozoal parasite whose definitive
ransmitted to the fetus at any
host is the cat family.
nsfer is most common during the
2. Only primary infection of the mother, who is
nal infection.
usually asymptomatic, results in congenital

Syphilis results fr
Treponema pallidum.
nant woman may be t
time, but fetal tra
first year of mater

Clinical Manifestat
ions
infection.
3. Infants infected early in pregnancy suffer from
c at birth may exhibit nonimintrauterine meningoencephalitis and present
nemia, thrombocytopenia,
with microcephaly, hydrocephalus, microphitis, hepatitis, osteochondritis,
thalmia, chorioretinitis, intracranial calcifications,
nifestations described in
and seizures.
ife include intermittent fever,
4. Of infants who are asymptomatic at birth, 70%
hondritis, hepatosplenomegaly,
suffer from long-term sequelae, which may
cocutaneous lesions (macuinclude mental retardation, learning disabilities,

Neonates symptomati
mune hydrops with a
leukopenia, pneumon
and rash. Common ma
the first year of l
osteitis and osteoc
lymphadenopathy, mu
lopapular rash on t

he trunk, palms, and soles),


and chorioretinitis.
(snuffles), jaundice, and failure

persistent rhinitis
to thrive. Laborato

ry tests may reveal hyperbiliru-

--------------------------------------- 78
Chapter 13 / Neonatology
169
binemia, a transaminitis, thrombocytopenia, leukocya positive RPR, and the history
tosis, and a Coombs'-negative hemolytic anemia.
s make infection unlikely, it is safe
The late sequelae of congenital syphilis appeatr
of the IgM FTA-ABS and repeat
many years after birth. They include multiple bone
cant rise in titer or any clinical
signs (frontal bossing, saber shins), Hutchinson teeth,
ent. The infant should be treated
mulberry molars, a saddle-nose deformity, rhagades,
ot negative by 6 months of age. For
juvenile paresis, juvenile tabes, interstitial keratitis,
ence of central nervous system
eighth nerve deafness, and Glutton joints (painless
G is given intravenously for 10 to
joint effusions]. These manifestations are rare in the
th central nervous system infecmodern era in which penicillin therapy is used to
h penicillin for 3 weeks. For
treat congenital syphilis.
for infection for whom follow-up
Diagnostic Evaluation
nt with one intramuscular dose

If the infant has


and clinical finding
o await the results
the RPR. Any signifi
signs require treatm
if the serology is n
infants with no evid
disease, penicillin
14 days. Infants wi
tion are treated wit
infants at low risk
is doubtful, treatme
of benzathine penici

llin G can be administered.


Laboratory tests include nontreponemal tests such as
the rapid plasma reagin test (RPR) and the Venereal
EY POINTS
Disease Research Laboratory test (VDRL), and treponemal tests such as the IgM fluorescent treponehilis results from transplacental
mal antibody absorption test (IgM FTA-ABS). If a
T.pallidum.
mother has a positive RPR screening test, a treponeations described in the first
mal test should be used to confirm infection. If infecclude intermittent fever, osteitis
tion is suspected in the mother, the infant needs to
itis, hepatosplenomegaly,
be similarly evaluated. The IgM FTA-ABS test is the
y, maculopapular rash on the
most specific for fetal infection. Radiographs of the
nd soles, persistent rhinitis
long bones may provide evidence of metaphyseal
undice, and failure to thrive.

1. Congential syp
transmission oi
2. Common manifest
year of life in
and osteochondr
iymphadenopath
trunk, palms, a
(snuffles), ja

demineralization or periosteal new bone formation.


ment of syphilis is so benign, an
Dark-field examination of nasal discharge may reveal
be treated if the diagnosis is
treponemes. Cerebrospinal fluid should also be sent

3. Since the treat


infant should
considered.

for RPR and FTA-ABS.


Treatment

Rubella

Pregnant women with primary, secondary, or latent


RNA togavirus. Congenital rubella
syphilis are treated with penicillin.
rare, reflecting the success of
If the infant's serologic test results are negative

Rubella virus is an
syndrome has become
rubella vaccine.

and no symptoms are present, no treatment is necessary. If the serologic test results are positive and the
ions
infant is symptomatic, treat the infant. The asymptomatic infant is treated when any of the following
arily as a result of infection in
conditions exists:
and include heart defects (patent

Clinical Manifestat

Anomalies occur prim


the first trimester

ductus arteriosus,
peripheral pulmonic stenosis,
The infant's titer is three to four times higher than
ventricular septal de
fect, atrial septal defects),
the mother's.
ophthalmologic defe
cts (cataracts, microphthalmia,
The FTA is 3 to 4+.
glaucoma, and chorior
etinitis), auditory deficits (senThe mother has been inadequately treated osorineurar
l deafness)
, and neurologic malformations
untreated.
(microcephaly, meni
ngoencephalitis, and mental
The mother is unreliable and follow-up is doubtfulretardation).
. Sequel
ae of chronic in utero infection
The mother's infection was treated with a drug
are growth retardatio
n, radiolucent bone disease,
other than penicillin.
hepatosplenomegaly,
thrombocytopenia, jaundice,
The mother has had a recent sexual exposure tano
d purple skin lesi
ons ("blueberry muffin spots").
an infected person.
Mild forms of the d
isease can be associated with few
The mother was treated in the last month oof
r no obvious clinica
l manifestations at birth.
pregnancy.
Rubella virus is
most consistently isolated from
The mother has HIV and has been treated for
nasopharyngeal secret
ions and urine. Infants with
syphilis with less than a neurosyphilis regimen.
congenital rubella
may excrete virus for months to

--------------------------------------- 79

Chapters / Cardiology
TABLE 3-1

17

Differential Diagnosis of Cyanosis in the Neonate


Cardiac

Pulmonary

Ductal-independent mixing lesions


disease such as respiratory distress
Truncus arteriosus
meconium aspiration, pneumonia,
Total anomalous pulmonary venous return without obstruction
ent pulmonary hypertension of the
D-transposition of the great arteries3

Primary lung

Lesions with ductal-dependent PBF


uction such as choanal atresia, vocal

Airway obstr

syndrome,
or persist
newborn

cord paral
ysis, or laryngotracheomalacia
Tetralogy of Fallot with pulmonary atresiab
Ebstein's anomaly6
Extrinsic co
mpression of the lungs such as
Critical pulmonic stenosis
pneumothor
ax, chylothorax, or hemothorax
Tricuspid valve atresiabwith normally related great arteries'3 Neurologic
Pulmonic valve atresia with intact ventricular septum
ion such as drug-induced depression
Heterotaxy6
tory drive, postasphyxial cerebral
Lesions with ductal-dependent SBF
n, or central apnea

CNS dysfunct
of respira
dysfunctio
Respiratory

neuromuscular dysfunction such as


Hypoplastic left heart syndrome
Interrupted aortic arch
cular atrophy, infant botulism, or
Critical coarctation of the aorta
yasthenia gravis
Critical aortic stenosis

spinal mus
neonatal m
Hematologic

Tricuspid valve atresia with transposition of the great arteries6Methemoglob


inemia or polycythemia
a A patent ductus arteriosus may improve mixing, especially with an intact ven
tricular septum.
b Most forms.
PBF, pulmonary blood flow; SBF, systemic blood flow.
TABLE 3-2
Interpretation of the Hyperoxia Test
m Hg) at Fio2 = 1.00 PaCo

Pao2 (mm Hg) at Fio2 = 0.21

Pao2 (m

(% Saturation)

(% Satu

ration) (mmHg)
Normal 70 (95)
0)
35
Pulmonary disease 50 (85)
0)
50
Neurologic disease 50 (85)
0)
50
Methemoglobinemia 70 (95)
0)
35
Cardiac disease

>300(10
>150(10
>150(10
>200(10

Parallel circulation3 <40 (<75)


<50 (<8
35
Mixing with restricted PBFb <40 (<75)
<50 (<8
5)
35
Mixing without restricted PBFC 40-60 (75-93)
<150(<1
00)
35
a D-Transposition of the great arteries with intact ventricular septum, D-tran
sposition of the great arteries with ventricular septal defect.
bTricuspid atresia with pulmonary stenosis or atresia, pulmonary atresia or cr
itical pulmonary stenosis with intact ventricular septum, tetralogy of Fallot,
or Ebstein's anomaly.
5)

cTruncus arteriosus; total anomalous pulmonary venous return; single ventricle


, hypoplastic left heart syndrome.
PBF, pulmonary blood flow.
(preductal) arterial blood gas measurement with the
>2 less than 150mmHg on 100%
child breathing room air, Fio 2= 0.21, and then repeatrdiac lesion characterized by coming the measurement with the child inspiring 100%
restricted pulmonary blood flow.
oxygen, Fio 2= 1.00. Interpretation of the hyperoxia test
0 mm Hg on 100% oxygen indicates
is delineated in Table 3-2. A PaO 2 greater than 200 mm
h parallel circulation, or a mixing
Hg on 100% oxygen makes congenital heart disease
ed pulmonary blood flow.

very unlikely. A PaC


oxygen suggests a ca
plete mixing without
A PaO 2 less than 5
a cardiac lesion wit
lesion with restrict

--------------------------------------- 80
170
rints Pediatrics
years. Specific rubella IgM antibody or persistence of
inically inapparent infections.
rubella IgG in the infant is diagnostic.
enital CMV (cytomegalic

develop in 10% of cl
The syndrome of cong
inclusion disease) i

s uncommon, occurring in 5% of
Treatment
ection, and includes intraThere is no specific antiviral chemotherapy. Approdation, purpura, jaundice,
priate treatment of specific defects is recommended.
microcephaly, intracerebral calInfants with congenital rubella are considered contarioretinitis. The calcifications tend
gious until they are 1 year of age, unless they have

infants with CMV inf


uterine growth retar
hepatosplenomegaly,
cifications, and cho
to be periventricula

r. A more common symptomatic


negative nasopharyngeal and urine cultures after 3
auterine growth retardation,
months of age. Rubella vaccination should not be
and persistent jaundice. Severe
given during pregnancy, but inadvertent administraia in premature infants can be
tion carries a very low risk of fetal disease.
enital infection excrete CMV in
KEY POINTS
and saliva, and the virus may be
1. Congenital rubella syndrome has become rare,
re or identified by early antigen
reflecting the success of rubella vaccine.
ne. Additional diagnostic studies
Anomalies occur primarily as a result of infection
of infection include a CT scan
in the first trimester and include heart defects,
ction of intracranial calcifications,
ophthalmologic defects, auditory deficits, and
, long bone films, and chest radineurologic malformations.
umonitis.
2. Sequelae of chronic infection are growth
retardation, radiolucent bone disease,
hepatosplenomegaly, thrombocytopenia,
l therapy exists. Ganciclovir effijaundice, and purple skin lesions ("blueberry
ease has not been demonstrated.

presentation is intr
hepatosplenomegaly,
interstitial pneumon
fatal.
Infants with cong
high titers in urine
grown in viral cultu
detection in the uri
to determine extent
of the head for dete
liver function tests
ograph to detect pne
Treatment
No accepted antivira
cacy in neonatal dis
Newborn hearing scre

ening by brainstem auditory


muffin spots").
important. Repeated evaluations
3. Rubella vaccination should not be given during
se postnatal development of
pregnancy, but inadvertent administration carries
Neonates with congenital CMV
a very low risk of fetal disease.
Cytomegalovirus
Neonatal cytomegalovirus (CMV) infection is
EY POINTS
common, occurring in 1 % of newborns in the United
infection is common in the
States. Higher rates are found in lower socioecoing in 1% of all neonates.
nomic populations. Among fetuses of mothers who
0% of fetuses whose mothers
develop primary CMV infection during pregnancy,
ary CMV infection during pregapproximately 40% become infected, and of those
rience congenital infection; of
infected, only 5% have residual neurologic deficits.
only 5% have residual neurologic
Infection occurs in about 10% of pregnancies with
recurrent or reactivated maternal infection. Neuros in about 10% of pregnancies with
logic sequelae in offspring are more severe after
activated maternal infection.
primary maternal infection; infection following reac-

evoked responses is
are imperative becau
deafness can occur.

K
1. Cytomegalovirus
newborn, occurr
2. Approximately 4
experience prim
nancy will expe
those infected,
deficits.
3. Infection occur
recurrent or re
4. Most cases are

clinically inapparent. Late sequelae


tivation during pregnancy may result in hearing loss
eafness and learning disabilities
and milder developmental problems for the infant.
10% of clinically inapparent
CMV infection acquired during the birth process, via
breastfeeding, or from blood or platelet transfusions
lusion disease occurs in 5% of
has not been associated with neurologic deficits.
V infection and includes intrauter-

such as nerve d
may develop in
infections.
5. Cytomegalic inc
infants with CM
ine growth reta

rdation, purpura, jaundice,


Clinical Manifestations
galy, microcephaly, intracerebral
Most cases are clinically inapparent. Late sequelae
and chorioretinitis.
such as nerve deafness and learning disabilities may

hepatosplenome
calcifications,

--------------------------------------- 81
Chapter 13 / Neonatology
171
shed the virus for some time, and pregnant
apings may reveal multinucleated
health care workers should not take care of infected
fluorescent antibody staining of
infants.
apings from lesions is very specific

smear of vesicle scr


giant cells. Direct
vesicle fluid or scr
but not very sensiti

ve. HSV polymerase chain reaction (PCR) of cerebr


ospinal fluid is both sensitive
Herpes Simplex Virus
e is central nervous system
There are two serotypes of herpes simplex virus
r this diagnosis in any infant
(HSV): HSV-1 and HSV-2. They can both cause
ns or denuded skin, infants with

and specific if ther


involvement. Conside
with vesicular lesio
signs of sepsis, or

in the setting of acute central


severe disease and mortality in the neonate, though
se.
HSV-1 in this setting generally produces milder

nervous system disea

disease. The incidence of neonatal infection is esti-

Treatment

mated to be about 1 in 3500 live births. Most neonatal HSV infection is caused by HSV-2 because it
th acyclovir is indicated for all

Antiviral therapy wi
forms of neonatal he

rpes infection, because even iniaccounts for the majority of genital herpes. The child
ease may disseminate with devasis infected as he or she moves through the vaginal
canal. The majority of neonatal herpes is therefore a

tially localized dis


tating effects.

result of perinatal infection, and true congenital


herpes is rare.
KE
Y POINTS
Clinical Manifestations
erpes simplex virus infection is

1. Most neonatal h
caused by HSV-2

.
Asymptomatic infection is rare. HSV manifests itself
in three discrete constellations of symptoms. Infants
fection is rare. HSV manifests
may have disseminated infection involving the liver
discrete constellations of symptoms.
and other organs (occasionally including the central
e disseminated infection involving
nervous system), localized central nervous system
ther organs (often including the

2. Asymptomatic in
itself in three
Infants may hav
the liver and o
central nervous

system), may have localized


disease, or localized infection of the skin, eye, and
system disease, or may have SEM
mouth (SEM disease). Ocular manifestations include

central nervous

conjunctivitis, keratitis, and chorioretinitis. In about

disease.

one-third of the patients, SEM involvement is the


py with acyclovir is indicated for all
first indication of the infection. Disseminated disease
al herpes infection, because even
may present with findings described for sepsis. Localized disease may disseminate with

3. Antiviral thera
forms of neonat
initially local
devastating eff

ects.
ized central nervous system disease may present with
fever, lethargy, poor feeding, hypoglycemia, disseminated intravascular coagulation (DIG), and irritability, followed by intractable focal or generalized
seizures. Vesicular lesions, when present, are an
Varicella-Zoster Vir
us
important clue to the diagnosis. Symptoms can occur
shortly after birth or as late as 4 weeks after birth.
ring age, 90% are immune
Disseminated disease usually occurs during the first
virus (VZV), so congenital and
2 weeks of life, whereas localized central nervous
re rare. Only 25% of the infants
system disease and SEM disease typically occur
e mothers develop congenital
during the second or third week.
ox.

Of women of childbea
to varicella-zoster
neonatal varicella a
of infected nonimmun
or neonatal chickenp

Neonatal herpetic infections are frequently severe,


with a high mortality rate and significant neurologic
ions
and/or ocular impairment of survivors, particularly in
on in the first and second
those not treated with antiviral therapy.
associated with cutaneous scars,
HSV is cultured easily; viral detection generally
gits or limbs, defects of the eye,
takes 1 to 3 days. Cultures are obtained from skin
tem anomalies, and low birth
vesicles, the mouth or nasopharynx, conjunctiva,
Newborns who acquire VZV
urine, blood, rectum, and cerebrospinal fluid. Tzanck
e perinatal period have a clinical

Clinical Manifestat
Maternal VZV infecti
trimesters has been
abnormalities of di
central nervous sys
weight in newborns.
infection during th

--------------------------------------- 82
172
Blueprints Pediatrics
illness varying from mild to fatal. The acquisition of
cells that causes immunodefitransplacental antibody determines the outcome in
nsmission from mother to infant
infants.
V-infected infants in the world.
Diagnosis of congenital varicella is made by
diatric AIDS cases result from
specific IgM VZV antibody or the persistence omaternaf
ion. Most remaining cases are
significant titers of VZV IgG. Maternal history will
or occur because of sexual transreveal characteristic chickenpox illness during
g factors include mothers with
pregnancy. Neonatal varicella is characterized by difg abuse or sexual contact with
fusely disseminated skin lesions in varying states,
ause of the relatively high
from macules, papules, vesicles, and pustules tprevalenco
avenous drug abuse in inner-city
crusts. Recovery of varicella-zoster virus by culture,
dren are disproportionately
immunofiuorescent staining of scrapings, or Tzanck
ent of pediatric AIDS cases caused
smear of vesicle base scrapings is diagnostic. Direct
sion occur in African-American
immunofluorescence of cells differentiates VZV
Hispanics. Transmission rates of
infection from HSV
to the neonate have been esti-

destruction of these
ciency. Vertical tra
accounts for most HI
Eighty percent of pe
l transmiss
transfusion related
mission. Predisposin
HIV secondary to dru
a male with HIV. Bec
e of intr
areas, minority chil
affected. Fifty perc
by maternal transmis
infants, and 25% in
HIV from the mother
mated at 15% to 30%.

Postnatal transmission of HIV


Treatment
s to infants by means of breast
Infants with congenital varicella do not require isonted.
lation, because they are no longer shedding virus.
Infants with neonatal varicella should be placed in

from infected mother


milk has been docume
Clinical Manifestati

ons
strict isolation for at least 7 days after onset of rash.
Infants born to mothers with onset of varicella 5 or
generally asymptomatic at birth.

Infected infants are


Within the first mon

th, they may develop persistent


more days before delivery require no specific treatthy, and hepatosplenomegaly.
ment other than isolation, if kept in the hospital.
r of life, without appropriate antiInfants whose mothers have onset of varicella within
5 days before delivery, or within 2 days after delivcommon symptoms include recurery, should receive varicella-zoster immune globulin
ctions, severe intractable diarrhea,

thrush, lymphadenopa
During the first yea
retroviral therapy,
rent refractory infe
and failure to thriv

e. It is estimated that 20% of


(VZIG), preferably at birth or within 96 hours.
th congenital/perinatal HIV
Infants with acute varicella in the first week of life
the first year of life, and 60% of
should receive acyclovir for 10 days. Infants who are
n have severe symptomatic
exposed to VZV infection as a result of contact with
nursery personnel should have their immune status
s of age.

untreated infants wi
infection die within
HIV-infected childre
disease by 18 month
Diagnosis of HIV

at birth is difficult because of


verified and, if susceptible, should receive VZIG
. If HIV is suspected and the
within 96 hours of exposure.
ive for HIV, the risk in the child

maternal antibodies
mother is seronegat
is minimal. The dia

gnosis of HIV infection in children born to mother


s who are HIV seropositive can
KEY POINT
re the onset of symptoms through

be established befo
detection of HIV in

peripheral blood by HIV DNA


1. Ninety percent of women of childbearing age are
ulture.
immune to varicella-zoster virus, and only 25% of
the infants of infected nonimmune mothers
develop congenital or neonatal chickenpox.
that maternal antiretroviral

detection and HIV c


Treatment
Studies have shown
therapy in the last

trimester can dramatically reduce


transmission of HIV
to the fetus to less than 10%,
and multidrug regim
ens reduce the risk of transmisHuman Immunodeficiency Virus
Neonates who have HIV-positive
Human immunodeficiency virus (HIV), an RNA
in whom HIV is suspected
retrovirus, causes acquired immunodeficiency synth antiretrovirals. At-risk infants
drome (AIDS). HIV is particularly tropic for CD4actically with trimethoprim-

sion even further.


mothers or mothers
are also treated wi
are treated prophyl

containing cells, which include helper T cellssulfamethoxazol,


to prevent Pneumocystis carinii
monocytes, and macrophages. It is the invasion and
pneumonia.

--------------------------------------- 83
Chapter 13 / Neonatology
173
KEY POINTS
nary tract infection (gram-

bacteremia), and uri


negative bacteremia)

.
1. Eighty percent of pediatric AIDS cases result from
ired sepsis (occurring between
maternal vertical transmission. Most remaining
occurs predominantly among
cases are transfusion related.
the newborn intensive care unit,
2. Transmission rates of HIV from the mother to the
e infants have been colonized
neonate have been estimated at 15% to 30% if
esistant bacteria indigenous to
neither mother nor infant is treated with antie care unit. Frequent treatment
retrovirals.
antibiotics for sepsis and the
3. Maternal treatment dramatically reduces the risk
venous indwelling catheters,
of transmission to the infant.
umbilical vessel catheters, and
4. Within the first month, infected infants may
g devices increase the risk for
develop persistent thrush, lymphadenopathy, and
al or fungal infection. The most
hepatosplenomegaly. During the first year of life,
S. aureus, Staphylococcus
common symptoms among untreated infants
gative bacteria, and Candida
include recurrent refractory infections, severe
intractable diarrhea, and failure to thrive.
cci are the most common cause
5. Treatment involves nutritional support, P. carinii
but the incidence has dramatically
prophylaxis, antiviral therapy, and anti-infective
institution of maternal screening
agents for specific infections.
al antibiotic regimens in culture-

Nosocomially acqu
day 3 and discharge)
premature infants in
because many of thes
with the multidrug-r
the newborn intensiv
with broad-spectrum
presence of central
endotracheal tubes,
electronic monitorin
such serious bacteri
common pathogens are
epidermidis, gram-ne
albicans.
Group B streptoco
of neonatal sepsis,
decreased since the
protocols and prenat
positive mothers. Gr

oup B streptococci are recovered


from the vaginal cul
tures of approximately 25% of
American women at th
e time of delivery.
.NiPNATALJ_N_FECTiqN_ Clinical Manifestations
Most infants with ea
rly-onset sepsis present with

Neonatal Sepsis
spiratory signs such as grunting,
Neonatal sepsis is generally divided into early-onset,
sis at birth. As a result, it is
late-onset, and nosocomial sepsis. Early-onset sepsis,
entiate sepsis from respiratory
occurring from birth to 3 days, can be an overDS) in the initial stages of
whelming multiorgan systemic disease manifested by
n the preterm neonate. Because of
respiratory failure, shock, meningitis (30%), DIG, and
t premature infants with RDS
acute tubular necrosis. Early-onset sepsis is due to
um antibiotics. Common signs
infection by the bacteria in the mother's genitouriy sepsis include poor feeding,
nary tract. These organisms include group B strepnea, ileus, and abdominal distentococci, Escherichia coli, Klebsiella, and Listeria
purpura are noted when DIG is
monocytogenes. Predisposing factors for early-onset
(with possible seizures) is present
sepsis include vaginal colonization with group B
ith early-onset sepsis.
streptococci, prolonged rupture of the membranes
ected early-onset sepsis should
(more than 24 hours), chorioamnionitis, maternal
rospinal fluid sent for culture.
fever or leukocytosis, fetal tachycardia, and preterm
should also be tested for Gram
birth. African-American race and male sex are unexd differential, and protein and
plained additional risk factors for neonatal sepsis.
al complete blood counts are perLate-onset sepsis, occurring between days 3 and
igns of infection. A white blood
28, usually occurs in the healthy full-term infant who
5000 or greater than 40,000, a
was discharged in good health from the normal
nt under 1000, and a ratio of
newborn nursery. Bacteremia leads to hematogenous
of greater than 20% all correseeding that results in focal infections such as
ed risk of bacterial infection.
meningitis (75%, usually due to group B streptococci
also be seen. The chest radior E. coli), osteomyelitis (group B streptococci and
termine the presence of pneuStaphylococcus aureus), arthritis (Neisseria gonord gases should be monitored to
rhoeae, S. aureus, Candida albicans, gram-negative
metabolic acidosis that may

nonspecific cardiore
tachypnea, and cyano
often hard to differ
distress syndrome (R
early-onset sepsis i
this difficulty, mos
receive broad-spectr
and symptoms of earl
emesis, lethargy, ap
tion. Petechiae and
present. Meningitis
in 25% of neonates w
Infants with susp
have blood and cereb
Cerebrospinal fluid
stain, cell count an
glucose levels. Seri
formed to identify s
cell count less than
total neutrophil cou
bands to neutrophils
late with an increas
Thrombocytopenia may
ograph is used to de
monia. Arterial bloo
detect hypoxemia and

--------------------------------------- 84
T74
Blueprints Pediatrics
be due to hypoxia or shock, or both. Blood pressure,
ens are the most common

gram-negative pathog

urine output, central venous pressure, and peripheral


infections, a combination of
perfusion are monitored to determine the need to
micin often is used. Persistent
treat septic shock with fluids and vasopressor agents.
espite antibacterial treatment
The clinical manifestations of late-onset sepsis
psis, which is treated with
include lethargy, poor feeding, hypotonia, apathy,
seizures, bulging fontanelle, fever, and direct hyperbilirubinemia. The evaluation for late-onset sepsis is
Y POINTS
similar to that for early-onset sepsis, with special
attention given to examination of the bones, the labis generally divided into earlyoratory values, and urine culture obtained by sterile
et, and nosocomial sepsis.
suprapubic aspiration or urethral catheterization.
sis (birth to 3 days of life) is due to
Late-onset sepsis may be due to the same pathogens
e bacteria in the mother's genias early-onset sepsis or those usually found in the
, which includes group B streptoolder infant (Streptococcus pneumoniae, Neisseria
ebsiella, and Listeria monocytogenes.
meningitides].
is (3 to 28 days of life) may be
The initial clinical manifestations of nosocomial
ame pathogens as early-onset
infection in the premature neonate may be subtle
se infants presenting late in the
and include apnea and bradycardia, temperature
also may have infections caused
instability, abdominal distention, and poor feeding as
ually found in the older infant
early signs. In the later stages, there may be severe
oniae, N. meningitidis).
metabolic acidosis, shock, DIG, and respiratory
quired sepsis (3 days of life to
failure.
rs predominantly among prema-

bacterial nosocomial
vancomycin and genta
signs of infection d
suggests candidal se
amphotericin B.
KE
1. Neonatal sepsis
onset, late-ons
2. Early-onset sep
infection by th
tourinary tract
cocci, coli, Kl
3. Late-onset seps
caused by the s
sepsis, but tho
neonatal period
by pathogens us
(e.g., S. pneum
4. Nosocomially ac
discharge) occu
ture infants in

the newborn intensive care unit


Treatment
monly caused by S. aureus,
A combination of ampicillin and gentamicin for 10
gram-negative bacteria, and C.
to 14 days is effective treatment against most organisms responsible for early sepsis. Once an organism is
identified and antibiotic sensitivities are determined,
antibiotic therapy may be tailored to treat the infecting organism. If meningitis is present, the treatment
is extended. If gram-negative meningitis is present,
s is transmitted from the
treating with an aminoglycoside and a third-generaected mothers to their newborn
tion cephalosporin is recommended for improved
occurs in about 50% of infants
penetration across the blood-brain barrier. As a result,
fected mothers. Of the infants

and is most com


S. epidermidis,
albicans.

Chlamydia Infection
Chlamydia trachomati
genital tract of inf
infants. Acquisition
born vaginally to in

cefotaxime and amikacin (for synergy) are used to


omatis, the risk of conjunctivitis
treat E. coli or Klebsiella meningitis. Sepsis resulting
he risk of pneumonia is 5% to
from group B streptococcal meningitis and Listeria
is the most commonly
are treated with ampicillin and gentamicin (for
te. A symptomatic infection of
synergy).
rynx, rectum, or vagina of the
The treatment of late-onset neonatal sepsis and
or more than 2 years. Prevalence
meningitis is the same as that for early-onset sepsis.
varies between 6% and 12%
Some centers recommend using cefotaxime in place
of gentamicin for improved coverage of potentially
resistant strep pneumonia. Cefotaxime is effective
ions
therapy against S. pneumoniae and N. meningitidis
al conjunctivitis, ocular congessepsis and meningitis.
charge develop a few days to
The treatment of nosocomially acquired sepsis
birth and last for 1 to 2 weeks.
depends on the indigenous microbiologic flora of the
ng infants caused by C.
particular hospital and their antibiotic sensitivities.
lly an afebrile illness that presents
Because S. aureus (sometimes methicillin resistant),
ks after birth. A repetitive,
S. epidermidis (usually methicillin resistant), and
tachypnea are characteristic but

who acquire C. trach


is 25% to 50%, and t
20%. The nasopharynx
infected anatomic si
the conjunctiva, pha
infant can persist f
among pregnant women
in most populations.
Clinical Manifestat
In neonatal chlamydi
tion, edema, and dis
several weeks after
Pneumonia in you
trachomatis is usua
between 3 and 19 wee
staccato cough and

--------------------------------------- 85
175
Chapter 13 / Neonatology
not always present. Crackles can be present, whereas
end of expiration and allows for
wheezing is less likely. Hyperinflation on chest radili at low intrathoracic pressures.
ograph is prominent. Untreated disease can linger or
of surfactant, the lungs have poor
recur.
sults in progressive atelectasis,

lar collapse at the


opening of the alveo
Because of the lack
compliance, which re
intrapulmonary shunt

ing, hypoxemia, and cyanosis.


Treatment
by mechanical ventilation,
Topical erythromycin may be instilled into the eye at
alveolar capillary leak result in
birth to prevent gonococcal ophthalmia, but this
ne membrane. The membrane
treatment will not reliably prevent neonatal chlamyd is composed of protein and
dial pneumonia. Chlamydial conjunctivitis and
ithelium. The incidence of RDS
pneumonia in young infants are treated with oral ery-

The forces generated


oxygen exposure, and
formation of a hyali
lines the alveoli an
sloughed alveolar ep
increases with decre

asing gestational age. A measure


thromycin for 14 days. Topical treatment of concithin-to-sphingomyelin ratio can
junctivitis is ineffective and unnecessary. The efficacy
ung maturity.
of erythromycin therapy is only 80%, so a second
surfactant is accelerated by
course is sometimes required. A specific diagnosis of
inistration, prolonged rupture
C. trachomatis infection in the infant should prompt
maternal narcotic addiction,
treatment of the mother and evaluation of her sex
c fetal stress caused by placenpartner.
aternal hyperthyroidism, and

of amniotic fluid le
be used to predict l
The production of
maternal steroid adm
of fetal membranes,
preeclampsia, chroni
tal insufficiency, m
theophylline. The pr

oduction of surfactant is delayed


KEY POINTS
perglycemia and hyperinsuline-

by combined fetal hy
mia, as occurs in ma

ternal diabetes.
1. Acquisition occurs in about 50% of infants born
vaginally to infected mothers. Of the infants who

Clinical Manifestati

acquire C. trachomatis, the risk of conjunctivitis is


nfants characteristically present
25% to 50% and the risk of pneumonia is 5% to
ting, nasal flaring, chest wall
20%.
nosis in the first 3 hours of life.
2. In neonatal chlamydial conjunctivitis, congestion,
try on auscultation. The amniotic
edema, and discharge develop a few days to
hingomyelin ratio is less than 2.0,
several weeks after birth and last for 1 to 2 weeks.
erol is absent in the amniotic
3. Pneumonia in young infants caused by C. traconfirmed by chest radiograph that
chomatis is usually an afebrile illness that presents
ticulonodular or ground-glass
between 3 and 19 weeks after birth. A repetitive,
chograms that are consistent
staccato cough and tachypnea are characteristic
asis.
but not always present.
e is a progressive worsening over

Affected premature i

ons
with tachypnea, grun
retractions, and cya
There is poor air en
fluid lecithin-to-sp
and phosphatidylglyc
fluid. Diagnosis is
reveals a uniform re
pattern and air bron
with diffuse atelect
The natural cours
the first 24 to 48 h
ours of life. After the initial insult
to the airway lining
, the epithelium is repopulated
NEONATAL RESPIRATORY
cells, which produce surfactant.
DISEASE
is increased production and

with type II alveolar


Subsequently, there
release of surfactan

t, so that there is a sufficient quanRespiratory Distress Syndrome


es by 72 hours of life. This results

tity in the air spac


in improved lung com

pliance and resolution of resPathogenesis


hich is frequently preceded by an
RDS, or hyaline membrane disease, is the most
tput.
common cause of respiratory failure in newborn
ons associated with RDS include
infants. It occurs in premature infants who are born
al emphysema, pneumothorax,
with immature lungs. In the average child, lung
nd pneumopericardium.
maturity occurs at 32 to 43 weeks' gestation, when
olar epithelial lining produces
surfactant, a phospholipid that lines the alveoli, is
ial emphysema, as air dissects
produced by the type II pneumocytes. RDS is caused
ial spaces and the peribronchial
by deficiency of surfactant. The major function of
sation of gas into the lung
surfactant is to decrease alveolar surface tension and
lung compliance and worsens
increase lung compliance. Surfactant prevents alveo.

piratory distress, w
increase in urine ou
Acute complicati
pulmonary interstiti
pneumomediastinum, a
Rupture of the alve
pulmonary interstit
along the interstit
lymphatics. Extrava
parenchyma reduces
respiratory failure

--------------------------------------- 86
Blueprints Pediatrics
176
Treatment
ion may mimic RDS clinically
The goal of therapy is to provide respiratory support
aph. Until blood culture results
to the infant until spontaneous resolution occurs. All
cs are recommended. Because of
attempts should be made to minimize barotrauma
ia that accompany RDS, intraand damage from high FiO
2.
ge and necrotizing enterocolitis
Conventional therapy for the affected premature
ccur in the neonate with RDS.
infant includes respiratory support with oxygen, conase is the long-term complicatinuous positive airway pressure (CPAP), and/or
ue to prolonged mechanical venmechanical ventilation. Therapy with artificial surature infant with high mean
factant has been shown to improve this condition
high oxygen tensions. Although
dramatically and has significantly decreased the rate
nates requiring mechanical
of neonatal mortality in premature infants. After sursome degree of chronic lung

streptococcal infect
and on chest radiogr
are known, antibioti
the periods of hypox
ventricular hemorrha
are more likely to o
Chronic lung dise
tion of RDS and is d
tilation of the prem
airway pressures and
15% of premature neo
ventilation develop
disease, 50% of prem

ature infants whose birth weight


factant administration, the FiO
2 of oxygen should be
evelop the condition. The pathotitrated to keep the PaO
2 greater than SOmmHg. If
n the lung are termed bronthe FiO 2 exceeds 60%, CPAP can be used to decrease

is less than lOOOg d


logic changes seen i
chopulmonary dysplas

ia (BPD). BPD is a chronic


the time spent in high oxygen concentrations and to
pulmonary disorder c
haracterized by squamous
lessen the need for mechanical ventilation. CPAP is
metaplasia and hyper
trophy of small airways with
also useful in treating apnea that is unresponsive to
nasal cannula stimulation and during the weaning
subsequent alveolar
collapse and air trapping. Infants
process after extubation. Intubation and intermittent
with chronic lung di
sease are chronic carbon dioxide
positive pressure ventilation are used when CPAP has
retainers. Chest rad
iograph abnormalities include
been optimized and the FiO
2 required to keep the
areas of hyperaerati
on and atelectasis. These chronic
PaO 2 greater than SOmmHg exceeds 60%. Other
changes make diagnos
is of a new infiltrate difficult.
indicators that mechanical ventilation is needed
Complications includ
e chronic respiratory insuffiinclude apnea that is unresponsive to CPAP and/or
ciency, requiring ho
me use of continuous oxygen
persistent respiratory acidosis (PaCO
2 greater than 60 therapy; right-sided
congestive heart failure secand pH less than 7.25] on maximum CPAP. In
ondary to pulmonary
hypertension; and pneumothgeneral, CPAP will not be sufficient for neonates with
orax. Weaning the in
fant off oxygen to room air can
birth weights less than lOOOg. As RDS resolves and
take up to 6 months.
Reactive airway disease is
surfactant therapy takes effect, the compliance of the
common and can be se
vere. Sudden infant death synlungs increases dramatically and ventilator paramedrome (SIDS) is more
common in infants with BPD.
ters must be weaned quickly to avoid severe baroLower respiratory in
fections caused by usually
trauma. When amniotic fluid assessment of the
benign viral agents,
most notably respiratory syncypremature infant reveals fetal lung immaturity and
tial virus, may caus
e severe respiratory distress. Some
preterm delivery cannot be prevented, administrainfants recover full
y, but the healing process takes
tion of corticosteroids to the mother 48 hours before
years.
The use of high o
xygen tensions in neonates with
delivery can induce or accelerate the production of
RDS results in the d
evelopment of retinopathy of
fetal surfactant and minimize the incidence of
prematurity in some
infants. Retinopathy of premaRDS.
turity is discussed
fully in Chapter 18.
Very premature neonates who require mechanical
ventilation for long periods of time are at risk for
alveolar rupture and the development of pulmonary
interstitial emphysema, pneumothorax, pneumomediastinum, and/or pneumopericardium. The risk of
K
EY POINTS
barotrauma increases as the duration of mechanical
ventilation increases, mean airway pressure escalates,
Respiratory dis
tress syndrome, or hyaline memand the intermittent mandatory ventilation rate
brane disease,
is the most common cause of respi-

increases. When RDS is very severe, pulmonary


in newborn infants. It occurs in
hypertension may occur, causing a right-to-left shunt
ts who are born at 34 weeks'
at the patent foramen ovale and the ductus arterioss and results from deficiency of
sus. Infants with respiratory distress deserve evaluation for sepsis and pneumonia, because group B

ratory failure
premature infan
gestation or le
surfactant.

--------------------------------------- 87
Chapter 13 / Neonatology

177
suffer from intraute

rine growth retardation. Both


2. Conventional therapy for the affected premature
infant includes respiratory support with oxygen,
ficiency as a common pathway

have placental insuf


for fetal hypoxia. I

nfants born in the breech position


continuous positive airway pressure, and/or
ed risk of meconium in the amnimechanical ventilation.

also have an increas


otic fluid.

3. Therapy with artificial surfactant has been shown


to improve RDS dramatically and has significantly
decreased the rate of neonatal mortality in premature infants.
ons
4. Chronic lung disease is the long-term complicapneumonitis is characterized
tion of RDS and is due to prolonged mechanical
a, and hypercapnia. Diagnosis
ventilation of the premature infant with high
e presence of meconium in the
mean airway pressures and high oxygen tensions.
fluid, combined with symptoms
5. The pathologic changes seen in chronic lung
ess and a chest radiograph that
disease are termed bronchopulmonary dysplasia,
diffuse infiltrates with hyperinwhich is characterized by squamous metaplasia
with meconium aspiration synand hypertrophy of small airways with subseneumothoraces.
quent alveolar collapse and air trapping.

Clinical Manifestati
Meconium aspiration
by tachypnea, hypoxi
is established by th
tracheal or amniotic
of respiratory distr
reveals a pattern of
flation. Of infants
drome, 10% develop p

Treatment
In pregnancies in wh
ich uteroplacental insufficiency
Meconium Aspiration
or suspected, tests of fetal wellPathogenesis

is either documented
being, such as the n

onstress test, biophysical profile,


fetal monitoring, an
d scalp pH sampling, help to
The fetal lung produces fluid that flows out of the
ts at high risk for meconium
lung and contributes to amniotic fluid. Fetal respira-

identify those infan


aspiration.

tory movements are not of sufficient strength to draw


When meconium is
noted, the obstetrician
amniotic fluid into the respiratory tree. Fetal hypoxiasuction,
s the oropha
rynx before delivery of the
however, may trigger the passage of meconium from
thorax. After delive
ry, if the baby appears depressed,
the lower GI tract into the amniotic fluid, and with
the vocal cords are
visualized by direct laryngoscopy,
severe fetal asphyxia and acidosis a gasp reflex may
and an endotracheal
tube is inserted. Suction is
generate adequate force to draw the meconium intapplieo
d to the endo
tracheal tube as it is slowly
the lung. Aspiration of the meconium interferes with
removed. The procedu
re is repeated if significant
gas exchange and obstructs airways by a ball-valve
d. If the baby exhibits poor
mechanism, resulting in ventilation-perfusion missupport by bag-valve mask is
match and pneumothoraces. The resulting hypoxithea
aby who appears vigorous
and acidosis increase pulmonary vascular resistance
ivery does not require
and causes right-to-left shunting of blood across the
ld have routine suctioning of the
patent foramen ovale or the ductus arteriosus or
both. This shunting further worsens the hypoxia and
occurred and the infant is in disacidosis created by aspiration, resulting in a vicious
sts of administration of oxygen
cycle of increasingly severe pulmonary arteriolar
ntilation. The severity of disease
hypertension, respiratory distress, and cyanosis. This
ount of meconium the infant
sequence of events can occur without meconium
e severity of the pulmonary
aspiration as a primary result of chronic fetal
due to the prenatal asphyxia.
hypoxia and is referred to as persistent pulmonary
ia (PaO
2 < SOmmHgJ or severe
hypertension.
2 > 60mmHg), intubation and

meconium is recovere
respiratory effort,
n initiated. A b
immediately upon del
intubation, but shou
oropharynx.
If aspiration has
tress, therapy consi
and/or mechanical ve
is related to the am
has aspirated and th
hypertension present
For persistent hypox
hypercapnia (PCO
mechanical ventilati

on are indicated. If severe


Risk Factors
h conventional ventilation, it is

hypoxia persists wit


likely that PPHN is

present and high frequency venThe risk of meconium aspiration is markedly


acorporeal membrane oxygenaincreased in postmature infants and neonates who

tilation and/or extr


tion (ECMO) may be b

eneficial.

--------------------------------------- 88
178

Blueprints Pediatrics
Risk Factors
KEY POINTS

1. Meconium aspiration syndrome is a disorder


ith meconium aspiration,
caused by perinatal asphyxia. Fetal hypoxia trigmatic hernia, pulmonary

PPHN is associated w
severe RDS, diaphrag
hypoplasia, and neon

atal pneumonia from group B


gers passage of meconium into the amniotic fluid,
oll.
which is likely aspirated in utero and immediately

streptococci or E. c

after birth.
2. Aspiration of the meconium interferes with gas
ons
exchange and obstructs airways by a ball-valve
gested by a history of perinatal
mechanism, resulting in ventilation-perfusion
progressive cyanosis associated
mismatch and pneumothoraces.The resulting
respiratory distress. Often the
hypoxia and acidosis increase pulmonary vascular
pulmonary insufficiency is greater
resistance and causes right-to-left shunting of
chest radiograph; the chest
blood across the patent foramen ovale or the
rmal or abnormal depending on
ductus arteriosus or both.
f the PPHN. Echocardiography
3. The risk of meconium aspiration is markedly
tructural heart disease, evidence
increased in postmature infants (gestational age
ry vascular resistance, and the
greater than 42 weeks) and neonates who suffer
-left shunting at the foramen
from intrauterine growth retardation.
riosus or both. The severity varies

Clinical Manifestati
The diagnosis is sug
hypoxia and rapidly
with mild to severe
clinical severity of
than the findings on
radiograph may be no
the specific cause o
reveals absence of s
of increased pulmona
presence of right-to
ovale or ductus arte
from mild disease wi

th spontaneous resolution to
death from intractab
le hypoxemia. Pulmonary
hypertension usually
resolves within 5 to 10 days of
birth.
Persistent Pulmonary Hypertension of
the Newborn

Treatment

Pathogenesis
maximizing oxygen delivery

Treatment focuses on

and decreasing pulmo


nary arteriolar hypertension.
PPHN, or persistent fetal circulation, is a disorder
Conditions that p
otentiate PPHN include
of term or post-term infants who have experihypoxia, acidosis, h
ypoglycemia, hyperviscosity,
enced acute or chronic hypoxia in utero. The
anemia, and systemic
hypotension. Hypoxia and aciprimary abnormality is the failure of the pulmonary
dosis promote increa
sed pulmonary arteriolar hypervasculature resistance to fall with postnatal
tension, whereas sys
temic hypotension increases
lung expansion and oxygenation. Normally, at birth
right-to-left shunti
ng and tissue hypoxemia.
the systemic vascular resistance rises as a result of
Hypoglycemia results
in ketosis, which exacerbates
cessation of blood flow through the placenta, and
acidosis, and anemia
reduces oxygen delivery to the
pulmonary vascular resistance decreases after the
tissues. Hyperviscos
ity increases pulmonary hyperfirst few breaths. With persistence of the fetal
tension by sludging.
The therapies used to treat
circulation, the pulmonary vascular resistance
PPHN combat the cond
itions that worsen pulcontinues to be high and may in fact be higher than
monary hypertension
and include supplemental
the systemic resistance. This results in shunting of the
oxygen, hyperventila
tion, administration of sodium
deoxygenated blood, which is returning to the righbicarbonatet
, pulmo
nary vasodilators, and support of
atrium, away from the lungs. The right-to-left shunt
systemic blood press
ure.
can occur at the foramen ovale or the ductus arteMild hyperventila
tion to a PaCO
2 less than 40 mm
riosus or both. Because the lungs are bypassed, the
Hg prevents the pulm
onary vasoconstrictive effects
blood is not oxygenated and hypoxemia ensues. Thoe
f a respiratory ac
idosis and results in improvement
hypoxemia and acidosis caused by the right-to-left
in PaO 2. Nitric o
xide relaxes pulmonary arteriolar
shunt only worsens the baseline pulmonary arteriosmooth muscle cells
and has been shown to be effeclar hypertension, resulting in a vicious cycle of
tive in PPHN. Sedati
on facilitates relaxation of the
increasingly severe pulmonary arteriolar hyperteninfant and pulmonary
vasodilation, whereas muscle
sion and cyanosis culminating in cardiopulmonary
paralysis may be nee
ded to assist with hyperventilafailure.
tion. The overall mo
rtality rate associated with

--------------------------------------- 89
179
Chapter 13 / Neonatology
PPHN is 25% in term infants. Infants who require

at high levels the u

nconjugated bilirubin exceeds the


very high ventilator settings, marked by an alveolarbinding capacity of
albumin and the free bilirubin
to-arterial gradient of greater than 600mmHg on
crosses the blood-br
ain barrier and damages the cells
room air, have a high mortality rate and may benefiot
f the brain. In pr
emature infants, much lower levels
from ECMO. ECMO improves the outcomes in the
of hyperbilirubinemi
a may result in kernicterus,
most severely ill patients.
because the more imm
ature the neonate, the more
immature is its bloo
d-brain barrier. Kernicterus is
characterized by a y
ellow staining of the basal ganglia
KEY POINTS
and hippocampus, whi
ch results in widespread cere1. PPHN is seen when there is failure of the pulbral dysfunction. Cl
inical features include lethargy
monary vasculature resistance to fall with postnaand irritability, hy
potonia, opisthotonos, seizures,
tal lung expansion and oxygenation. It occurs in
mental retardation,
cerebral palsy, and hearing loss.
term and post-term infants who have experienced
Most full-term an
d preterm neonates develop a
acute or chronic hypoxia in utero.
transient, unconjuga
ted hyperbilirubinemia during
2. Hypoxemia and acidosis caused by right-to-left
the first week of li
fe. This episode of "physiologic
shunting worsen baseline pulmonary arteriolar
jaundice" is due to
an elevated bilirubin load
hypertension, resulting in a vicious cycle of
(secondary to an inc
reased red blood cell volume,
increasingly severe pulmonary arteriolar hypera decreased red bloo
d cell survival time, and
tension and cyanosis that culminates in cardiopulan increased enteroh
epatic circulation), defective
monary failure.
hepatic uptake of bi
lirubin, inadequate bilirubin
3. The therapies used to treat PPHN include suppleconjugation caused b
y decreased UDP-glucuronylmental oxygen, hyperventilation, administration
transferase activity
, and defective bilirubin excretion.
of sodium bicarbonate, pulmonary vasodilators,
Physiologic jaundice
begins after 24 hours of life, is
and support of systemic blood pressure.
associated with a pe
ak of 12 to 15 mg/dL at day 3 of
life, and returns to
normal levels by the end of the first
week of life. Risk f
actors for developing more severe
NEONATAL GASTROINTESTINAL
physiologic jaundice i
nclude prematurity, maternal
diabetes, and Asian
or Native American ancestry.
DISEASED
The mechanism of
breast milk jaundice, which is
also quite common, i
s not known. Some researchers
Hyperbilirubinemia
have theorized that

it is due to an increase in enteroHyperbilirubinemia manifests as jaundice a yellowhepatic circulation fr


om an unknown maternal factor
ing of the skin, mucous membranes, and sclera. It
in the breast milk.
The infant's peak bilirubin level
occurs when serum bilirubin levels are greater than
tends to be higher a
nd lasts longer than that found
5mg/dL in neonates and greater than 2mg/dL in
with physiologic jau
ndice.
children and adolescents. The two types of hyperAny infant who de
velops hyperbilirubinemia in
bilirubinemia are unconjugated (indirect), which can
the first 24 hours o
f life, has an increase in serum
be physiologic or pathologic in origin, and conjugatebilirubid
n greater th
an 5 mg/dL/day, is jaundiced and
(direct), which is always pathologic. Conjugated
has the risk factors
noted earlier, has prolonged jaunhyperbilirubinemia is defined as the direct fraction of
dice (more than 1 w
eek in the full-term infant or
bilirubin in the blood exceeding 2mg/dL or 15% of
more than 2 weeks i
n the premature neonate), or
the total bilirubin. Bilirubin is a bile pigment formed
has conjugated hype
rbilirubinemia needs to be
from the degradation of heme that is derived
evaluated.
from red blood cell destruction and ineffective
erythropoiesis. Figure 13-1 illustrates normal bilirusis
bin metabolism. Abnormalities in any step in the
ilirubinemia
process may result in unconjugated or conjugated
e
hyperbilirubinemia.
Neonatal hyperbilirubinemia is monitored with

Differential Diagno

great care because elevated levels of unconjugated


ABO/Rh incompatibility, erybilirubin cause kernicterus. Unconjugated bilirubin is
fetalis, drug reaction (penicillin,
normally bound tightly to albumin in the blood, but
oxytocin)

Immune etiology:

Unconjugated Hyperb
Physiologic jaundic
Hemolytic process

throblastosis
sulfonamides,

--------------------------------------- 90
18

rints Pediatrics
The PaO
2 should be measured directly via arterial
the pulmonary vascularity
puncture, though properly acquired transcutaneous
ncreased pulmonary blood flow
oxygen monitor (TCOMJ values for PaO
2 are also
e of D-transposition of the great
acceptable. Pulse oximetry should not be used foarterier
h intact ventricular septum,
interpretation of the hyperoxia test, because a
ema is a manifestation of
neonate given 100% inspired oxygen may have a
onary venous return with

has been ruled out,


becomes the focus. I
suggests the presenc
s (D-TGA) wit
whereas pulmonary ed
total anomalous pulm

PaO 2 of SOmmHg with a pulse oximeter reading oobstructionf


100% [abnormal), or a PaO
2 greater than 300mmHg
gnoses (tricuspid atresia with
with a pulse oximeter reading of 100% (normal). If
at arteries, pulmonic atresia with
a cardiac cause is deemed likely, obtain an echocarintaceptum, critical pulmonic stenosis,
diogram and a cardiology consultation.
lot with or without pulmonary
Pulse oximetry should be documented at predecreased pulmonary vascularity
ductal and postductal sites to assess for differential
lightly enlarged heart size. These
or reverse differential cyanosis. If the preductal
tiated by their axis on ECG
saturation is higher than the postductal saturaabsence of a murmur. Tricuspid
tion, differential cyanosis exists, which results when
ry stenosis or pulmonary atresia
there are normally related great arteries and
erior axis, lying in the 270- to
deoxygenated blood from the pulmonary circularitical pulmonic stenosis and
tion enters the descending aorta through a patent
th intact ventricular septum
ductus arteriosus. Differential cyanosis is seen in
e 0- to 90-degree quadrant. They
persistent pulmonary hypertension of the newborn
y the presence of the loud systolic
(PPHN) and in lesions with left ventricular outflow
d from critical pulmonic stenotract obstruction such as interrupted aortic arch,
alogy of Fallot and tetralogy of Fallot
critical coarctation of the aorta, and critical aortic
ia both have axes in the 90- to
stenosis.
they are distinguished from
In rare cases of reverse differential cyanosis, the
lmonic stenosis murmur noted
postductal saturation is higher than the preductal satot.
uration. This occurs only in children with transposition of the great arteries with left ventricular outflow
obstruction (i.e., critical coarctation of the aorta,
interrupted aortic arch, critical aortic stenosis) or
lesions without adequate
PPHN. Oxygenated blood from the pulmonary cirntact ventricular septum and
culation enters the descending aorta through a patent
oramen ovale) or defects that
ductus arteriosus.
t pulmonary blood flow
When either the hyperoxia test or the preductal/
systemic blood flow may
postductal oxygen saturation measurement, or both,
n Ej (PGEj) infusion to maintain
indicate cardiac disease, the chest radiograph and
s arteriosus until definitive
ECG may be used to delineate which cardiac strucan be accomplished. Rarely, the

.
The remaining dia
normally related gre
t ventricular s
and tetralogy of Fal
atresia) all produce
and normal or only s
defects are differen
and the presence or
atresia with pulmona
is noted for its sup
0-degree quadrant. C
pulmonary atresia wi
both have axes in th
are differentiated b
ejection murmur hear
sis. Similarly, tetr
with pulmonary atres
180-degree quadrant;
each other by the pu
in tetralogy of Fall
Treatment
Newborns with mixing
mixing (D-TGA with i
restrictive patent f
have ductal-dependen
or ductal-dependent
require prostaglandi
patency of the ductu
surgical treatment c

tural defect is the most likely. The chest radiograph


tal heart disease may become
is obtained to determine the size of the heart and
nstable after the institution of
whether the pulmonary vascularity is increased or
linical deterioration after instidecreased. The ECG evaluates the heart rate,
important diagnostic finding that
rhythm, axis, intervals, R-wave progression, and Pnital heart defect as one that has
wave and ST/T wave morphology and helps deterw out of the pulmonary veins or
mine if ischemia, atrial dilatation, or ventricular
that have impaired blood flow
hypertrophy is present.
include hypoplastic left heart
To differentiate among cyanotic congenital hearsyndromt
trictive or intact foramen ovale,
defects that present with a PaO
2less than SOmmHg
tral atresia with restrictive
on the hyperoxia test, the clinician should first
position of the great arteries with
examine the chest radiograph. If massive cardiac
r septum and restrictive foramen
enlargement is noted, Ebstein's anomaly is the most
malous pulmonary venous return
likely diagnosis. Once massive cardiac enlargement

patient with congeni


progressively more u
PGEi therapy. This c
tution of PGEj is an
identifies the conge
obstructed blood flo
left atrium. Lesions
from the left atrium
e with res
other variants of mi
foramen ovale, trans
an intact ventricula
ovale, and total ano
with obstruction.

--------------------------------------- 91
180

rints Pediatrics
Catabolism of
Ineffective
effete red
blood cells
Heme'j75%) Heme(25%)

erythropoiesis
Heme

Biliverdin oxygenase
Biliverdin
transferase
,.--- Bilirubin

Reticuloendothelial
system

Smooth
endoplasmic
reticulum
Bilirubin glucuronideEnterohepatic
circulation of
bilirubin

B-Glucuronidase
Figu

re 13-1

Bilirubin metabolism in the


Fecal bilirubin

ate.
Red cell defects: Structural (spherocytosis, elliptoHypothyroidism, infants of
cytosis), hemoglobinopathy (sickle cell, alphahypopituitarism
thalassemia), enzyme deficiency (G6PD or
pyruvate kinase deficiency)

neon
Endocrine disorders:
diabetic mothers,
Bacterial sepsis

Conjugated Hyperbili
rubinemia
DIG
Extrahepatic obstruc
tion: Biliary atresia, choledoPolycythemia
ledochal cyst, common duct stenoExtravascular blood loss: Bruising from birth
bile syndrome from cystic fibrosis,
trauma (petechiae, cephalohematoma), hemorct compression, pancreatitis
rhage (pulmonary, cerebral)
tic cholestasis: Paucity of
Swallowed maternal blood

cholithiasis, cho
sis, inspissated
extrinsic bile du
Persistent intrahepa
intrahepatic duct

s, benign recurrent intrahepatic


Increased enterohepatic circulation: Intestinal
riohepatic dysplasia
obstruction (pyloric stenosis, duodenal stenosis or

cholestasis, arte
Acquired intrahepati

c cholestasis: Neonatal
atresia, annular pancreas), Hirschsprung's disease,
ial sepsis; congenital infections;
meconium ileus and/or meconium plug syndrome,
nd C; varicella; Epstein-Barr virus;
drug-induced paralytic ileus (magnesium)
kie virus; tuberculosis; lepBreast milk jaundice
iasis), drug-induced cholestasis,
Disorders of bilirubin metabolism: Gilbert's synnutrition cholestasis, cirrhosis, drug
drome, Crigler-Najjar syndrome, and Lucey, neoplasms (hepatoblastonia,
Driscol syndrome
etastases)

hepatitis (bacter
hepatitis A, B, a
echovirus; coxsac
tospirosis; amoeb
total parenteral
or metal toxicity
secondary liver m

--------------------------------------- 92
Chapter 13 / Neonatology 181
Genetic and metabolic disorders: Disorders of
linical Jaundice
bilirubin metabolism (Dubin-Johnson syndrome,
Rotor's syndrome), disorders of carbohydrate
easure total bilirubin
metabolism (galactosemia, fructosemia), disorders

Total bilirubin < 1


2 mg/dl and infant < 24 hours old
of amino acid metabolism (tyrosinemia, hypermeI
thioninemia), disorders of lipid metabolism
ombs' test
(Niemann-Pick disease, Gaucher's disease), chro-

Obtain Co

mosomal disorders (trisomy 18 and 21), metabolic


ity Evaluate direct bilirubin level

ABO/Rh incompatibil
(Hemoliptic proces

s of ^/^
liver disease (Wilson's disease, aj-antitrypsin
immune etiology) D
irect bilirubin > 2mg/dl Direct bilirubin < 2mg/dl
deficiency)
(co
njugated

(unconjugated
t^ hype

rbilirubinemia) hyperbilirubinemia)
Viral/bacteri
al
nfection,
Clinical Manifestations
Evaluate hematocrit
hepatitis
History
Normal or low / High

biliary
obstruction

Evaluate red blood cell Polycythemia


Is the child breast or formula fed? Other important
y and
Normal morphology

Abnormal morpholog
high reticulocyte

count
clues include a history of red cell structural defects,
and reticulocyte count
hemoglobinopathies, or enzyme deficiencies in the
1. Extravascular blood loss

Hemolytic process
1. Red cell defect

s
2. Swallowed maternal blood
family or whether a previous child had an ABO
travascular3 Increased enterohepatic circulation

2. Disseminated in
coagulation

4. Breast milk jaundice


incompatibility. There may be a family history of
5. Disorders of bilirubin metabolism
3. Drug reaction
genetic or chromosomal disorders. Prenatal screens
6. Endocrine disorders
7. Bacterial sepsis
should be reviewed for possible causes of congenital
infection. The length of time the jaundice has been
Figure 13-2

Algorith

m for the evaluation of hyperbilirubinepresent, whether it is worsening or improving, and


mia in the neonate.

associated gastrointestinal or constitutional sympty JP, Stark AR, eds. Manual of Neonatal Care, 3rd ed.
toms should be explored. Also, it is important to ask
n, 1991:301.
whether the stool color has changed (to a gray color)

Modified from Cloher


Boston: Little, Brow

or the urine has darkened.


Physical Examination
In neonates, the examination should focus on the
level of jaundice, because progression is reliably
exchange transfusion
. When to use these treatments
cephalopedal. When jaundice has reached the
depends on the birth
weight of the neonate. Given a
umbilicus, the serum level is approximately 10. If thspecifie
c LEW, Table
13-4 shows the indicated treatpalms and soles are involved, the level is likely greater ment at different le
vels of unconjugated hyperbiliruthan 15.
binemia. When to use
phototherapy in the full-term
neonate is quite con
troversial. No studies show eviDiagnostic Evaluation
dence of encephalopa
thic damage from unconjuBecause the most common causes of unconjugated
gated hyperbilirubin
emia peak levels less than 25
hyperbilirubinemia are physiologic and hemolytic,
mg/dL in the full-te
rm healthy neonate with uncomthe initial evaluation should include a complete
plicated physiologic
jaundice. As a result, there is
blood count with peripheral blood smear and reticmuch debate among
pediatricians as to when to
ulocyte count, a determination of maternal and
begin phototherapy.
Phototherapy converts the
infant blood types, a Coombs' test (direct and indiunconjugated bilirub
in into several water-soluble
rect), and a determination of the conjugated and
photoisomers that ca
n be excreted without conjugaunconjugated fractions of the hyperbilirubinemia.
tion, so it is impor
tant to optimize the infant's hydraFigure 13-2 shows an algorithm for the evaluation of
tion status. Exchang
e transfusion directly removes
hyperbilirubinemia.
the bilirubin from t
he intravascular space and is
usually reserved for
the extremely sick premature
Treatment
neonate with hemolyt
ic disease.
Treatment of conj
ugated hyperbilirubinemia is
The goal in treating unconjugated hyperbilirubinedirected at the unde
rlying cause of the hyperbilirumia is to avoid kernicterus or sublethal bilirubin
binemia. Phototherap
y of conjugated bilirubin

encephalopathy. The two modalities used to decrease


"bronzes" the skin
and takes months to resolve.
unconjugated bilirubin are phototherapy and

--------------------------------------- 93
182

Blueprints Pediatrics
arteriosus, congesti

ve heart failure, umbilical vessel


KEY POINTS
catheterization, pol
ycythemia, and exchange transfu1. Hyperbilirubinemia may be conjugated or unconad of formula has also been
jugated. Conjugated hyperbilirubinemia is always
pathologic, whereas unconjugated hyperbilirubinemia may or may not be pathologic.
ons
2. The two most common causes of unconjugated
be mild to fulminant and
hyperbilirubinemia are physiologic jaundice and
6 weeks of life. The earliest signs
hemolytic disease.
nce with bilious aspirates and
3. Most neonatal unconjugated hyperbilirubinemia
. The patient may develop
is physiologic.
stool, which can become grossly

sion. The osmotic lo


implicated.
Clinical Manifestati
The presentation may
occurs in the first
are feeding intolera
abdominal distention
occult blood in the
bloody. Extreme abdo

minal tenderness with discoloration, hyperglycem


ia, severe metabolic acidosis,
sepsis, shock, DIG,
temperature instability, and inefNecrotizing Enterocolitis
effort (due to severe abdominal

fective respiratory
distention) requirin

g mechanical ventilation are seen


Pathogenesis
Necrotizing enterocolitis (NEC) refers to a process
ases.
of transluminal and mucosal necrosis that is seen in
ations include intestinal stric-

in the more severe c


Long-term complic
tures, which may be

demonstrated by contrast study.


premature infants. The cause is unknown but likely
include leukocytosis, neutropeinvolves a component of ischemia or reperfusion
a, and metabolic acidosis.
injury, followed by translocation of bacteria into the
wall of the intestine. Occasional epidemics in neonatal intensive care units implicate a primary role for

Laboratory findings
nia, thrombocytopeni
Treatment
If necrotizing enter

ocolitis is suspected, feeds should


infection in some instances. Pneumatosis intestinalis

be discontinued imme

diately and a nasogastric tube


results from gas production in the bowel wall. It can
gastric and intestinal decombe detected on abdominal radiography and is pathognomonic for necrotizing enterocolitis.
ntibiotics should be started, and
NEC occurs primarily in premature infants and
d be sent. Abdominal radi-

should be placed for


pression. Systemic a
blood cultures shoul
ographs should be ob

tained at least every 6 hours to


is ultimately diagnosed in almost 25% of very
sis intestinalis, portal air, and
low-birth-weight infants (<1500g). Prenatal factors
Intravenous fluids are adminisassociated with necrotizing enterocolitis include
ck. If free air is seen in the perimaternal age greater than 35, maternal infection
requiring antibiotics, premature rupture of memestinal necrosis is suspected,

monitor for pneumato


free peritoneal air.
tered to prevent sho
toneal cavity or int
surgical interventio

n is indicated. If there is no free


branes (PROM), and cocaine exposure. Perinatal
course of bowel rest and broad
factors include maternal anesthesia, depressed Apgar
generally leads to full recovery.
scoreat 5 minutes, birth asphyxia, RDS, and
hypotension. Postnatal factors include patent ductus

air, a 10- to 14-day


antibiotic treatment

K
EY POINTS
TABLE 13-4
ocolitis refers to a process of

1. Necrotizing enter
acute intestina

l necrosis seen in premature


Hyperbilirubinemia in Low-Birth-Weight
Nebnates
Weight (g) Bilirubin Level (mg/dL)
dical necrotizing enterocolitis

infants.
2. Infants with me
present with fe

eding intolerance, abdominal disConsider


blood in the stool, and dilated
Phototherapy
abdominal radiograph.

Consider Exchange

tention, occult

Transfusion

bowel loops on
3. Pneumatosis int

estinalis is the diagnostic radi<1000


5-7
12-15
1000-1500
7-10
15-18
g. Free peritoneal air is evidence
1 500-2500
10-15
18-20
and an indication for surgical
>2500
>15
>20

ographic findin
of perforation
intervention.

--------------------------------------- 94
Chapter 13 / Neonatology
NEONATAL HEMATOLOGIC
cardiogram may be abnor-

183
gram (EEG) and electro

DISORDERS ........
h often reveals cardiomegaly,

mal. Chest radiograp


increased vascular m

arkings, pleural effusions, and


Polycythemia

interstitial edema.
Long-term complic

ations from neonatal polyPathogenesis


kely in the symptomatic child,
Polycythemia is defined as a greater than normaparticularll
glycemia is present. Neurodevelnumber of red cells in the blood. Neonatal polyes include mild deficits in
cythemia (defined as a venous hematocrit greater
coordination. If cerebral infarcthan 65%) is almost always a consequence of fetal
l palsy and mental retardation are
hypertransfusion. Delayed clamping of the cord after

cythemia are more li


y if hypo
opmental abnormaliti
speech, hearing, and
tion occurs, cerebra
likely.

delivery with consequent transfer of placental blood


to the infant is the most common cause in term

Treatment

infants. A significantly elevated hematocrit leads to


ons may be prevented by treathyperviscosity of the blood, resulting in vascular
infants with partial exchange
stasis, microthrombi, hypoperfusion, and tissue
rth. A partial exchange transfusion
ischemia. Neonatal erythrocytes are less filterable
and replaces it with normal
and deformable than adult erythrocytes, further con-

Long-term complicati
ment of symptomatic
transfusion after bi
removes whole blood
saline or albumin.

tributing to hyperviscosity. Although a central venous


hematocrit of greater than 65% occurs in 3% to 5%
of infants, not all infants have symptoms of hyperEY POINTS
viscosity syndrome.
syndrome, which occurs when the

K
1. Hyperviscosity
hematocrit exce

eds 65%, results in vascular stasis,


Risk Factors
ypoperfusion, and tissue
Infants at risk for polycythemia are post-term and

microthrombi, h
ischemia.

small for gestational age neonates; infants of diabetic


fants appear ruddy and plethoric.
mothers; infants with delayed cord clamping (mater-

2. Polycythemic in

nal-fetal transfusion}; and infants suffering from


ications from neonatal polyneonatal hyperthyroidism, adrenogenital syndrome,
re likely in the symptomatic
the trisomies (13, 18, and 21], twin-twin transfusion
arly if hypoglycemia is also present,
(recipient), or Beckwith-Wiedemann syndrome. In
ld deficits in speech, hearing, and

3. Long-term compl
cythemia are mo
child, particul
and include mi

some infants, polycythemia reflects a compensation


for prolonged periods of fetal hypoxia from placenlycythemia is primarily by partial
tal insufficiency; these infants have increased eryfusion.
thropoietin levels at birth.

coordination.
4. Treatment of po
exchange trans

Clinical Manifestations
Polycythemic infants appear ruddy and plethoric.
Irritability, lethargy, poor feeding, emesis, tremulousness, and seizures all reflect abnormalities of the
e can result from blood loss,
microcirculation of the brain. Acute renal failure
d red blood cell production, or
results from inadequate renal perfusion.
ased erythropoiesis. Blood loss
Hepatomegaly and hyperbilirubinemia are due to
tetric causes, occult blood loss,
poor hepatic circulation and to the increased amounot
s and may occur during the preof hemoglobin that is metabolized into bilirubin.
r neonatal period.
Because of stasis in the pulmonary vessels, pulmonary
of blood loss include abruptio
vascular resistance increases, and PPHN may result.
previa, incision of the placenta
Other complications include necrotizing enterocolition, rupture of anomalous vessels
tis and hypoglycemia. Vascular impairment in the
entous insertion of the cord, or
penis can cause priapism, and the formation of
ating vessels in a multilobed
microthrombi may cause thrombocytopenia. If
of the cord caused by varices
ischemia is severe enough, both electroencephaloture of the cord.

Anemia
Anemia in the neonat
hemolysis, decrease
(physiologic) decre
may result from obs
r iatrogenic cause
natal, perinatal, o
Obstetric causes
placenta, placenta
during cesarean sec
(vasa previa, velam
rupture of communic
placenta), hematoma
or aneurysm, or rup

--------------------------------------- 95
184
Blueprints Pediatrics
Occult blood loss may result from fetomaternal
ons
bleeding, fetoplacental bleeding, or twin-to-twin
story, including questions about
transfusion. Fetomaternal bleeding may be chronic or
olestatic disease, and splenecacute. It occurs in 8% of all pregnancies. The diagortant clues to newborn
nosis of this problem is by Kleihauer-Betke stain of
ic history may identify blood loss
maternal smear for fetal cells.
anemia. The physical examination
Bleeding in the neonatal period may be due to
tiate acute blood loss, chronic

Clinical Manifestati
A complete family hi
anemia, jaundice, ch
tomy, may define imp
disease. The obstetr
as the cause of the
can usually differen

intracranial bleeding, massive cephalohematoma,


nic hemolytic disease. Manifestaretroperitoneal bleeding, ruptured liver or spleen,
loss include shock, tachypnea,
adrenal or renal hemorrhage, gastrointestinal bleedous pressure, weak pulses, and
ing, or bleeding from the umbilicus. Excessive blood
d loss is manifested by extreme
loss may result from blood sampling with inadequate
atocrit. These infants are typireplacement. With acute blood loss, the hematocrit
nd may have congestive heart
is often normal, as is the reticulocyte count.
etalis. Chronic hemolysis is assoHemolysis is manifested by a decreased hematjaundice, and hepatosplenomegaly.
ocrit, increased reticulocyte count, and an increased
bilirubin level. Hemolysis may result from immune
ay be classified by evaluation
mechanisms, hereditary red cell disorders, or
count, bilirubin level, Coombs'
acquired hemolysis. Immune-mediated hemolysis
cell morphology (Table 13-5).
results from Rh incompatibility, ABO incompatibilo identify maternal blood
ity, minor blood group incompatibility (c, E, Kell,
wed by the neonate, and the

blood loss, and chro


tions of acute blood
tachycardia, low ven
pallor. Chronic bloo
pallor and a low hem
cally normovolemic a
failure or hydrops f
ciated with pallor,
Neonatal anemia m
of the reticulocyte
test, and red blood
The Apt test helps t
that has been swallo
Kleihauer-Betke prep

aration determines if fetomaDuffy), and maternal hemolytic anemia from sysas occurred. Ultrasound of the
temic lupus erythematosus. Hereditary red cell disne an intracranial bleed. Laboraorders that result in hemolysis include red blood cell
rents help to determine the
membrane defects (spherocytosis), enzymopathies
lytic process. If a congenital
(G6PD deficiency, pyruvate kinase deficiency], and
ed as the cause of the anemia, the
hemoglobinopathies (sickle cell disease, alpha and
ic tests may be done. Bone
beta thalassemias). Causes of acquired hemolysis
performed in rare cases in which
include bacterial or viral infection, DIG, vitamin E
is suggested.
deficiency, or microangiopathic hemolytic anemia.
Diminished red blood cell production is manifested by a decreased hematocrit, decreased reticutomatic newborns self-correct a
locyte count, and normal bilirubin level. Etiologies
d that iron intake is adequate.
include Diamond-Blackfan syndrome, Fanconi's
eding infants are sent home on
anemia, congenital leukemia, infections (especially
las, iron supplementation is not
rubella and parvovirus), osteopetrosis leading to
ths of age, when reticulocytosis
inadequate erythropoiesis, drug-induced red blood
cell suppression, physiologic anemia, or anemia of
s acute blood loss at birth, imme-

ternal transfusion h
head is used to defi
tory tests on the pa
likelihood of a hemo
infection is suspect
appropriate diagnost
marrow aspiration is
bone marrow failure
Treatment
Healthy, term, asymp
mild anemia, provide
Although nonbreastfe
iron-fortified formu
required until 2 mon
resumes.
If the neonate ha

prematurity.
be obtained, and blood must be
Physiologic anemia of the full-term or premature
crossmatching. If hypovolemic
neonate is due to physiologically decreased erythrocreased venous pressure, pallor,
poiesis. Full-term infants have a nadir of the hemog of volume expander is recglobin level at 6 to 12 weeks, premature infants
type O blood should be
(1200-2400 g) have a nadir at 5 to 10 weeks, and
usion if needed. Albumin and
very LEW neonates (birth weight less than 1200g)
so useful to replete the intravashave a nadir at 4 to 8 weeks. The laboratory manirily. Chronic blood loss and the
festations of physiologic anemia are a decreased
s are generally well tolerated.
hematocrit and a low reticulocyte count. When the
is symptomatic with congestive
infant's oxygen demand increases, erythropoietin will
he or she be transfused. It is recincrease; if iron stores are adequate, the reticulocyte
matocrit in the child with
count will increase and the hemoglobin level will
ry diseases be kept above 35
rise.

diate access should


sent for typing and
shock is present (de
tachycardia), 20mL/k
ommended. Unmatched
available for transf
normal saline are al
cular volume tempora
anemia from hemolysi
Only if the neonate
heart failure should
ommended that the he
cardiac or respirato
to 40.

--------------------------------------- 96
Chapter 13 / Neonatology
TABLE 13-5

185

Classification of Anemia in the Newborn


Reticulocytes Bilirubin Coombs'
Diagnostic Possibilities

RBC Morphology
Test

Normal or -I
Normal
Physiologic anemia of infancy or

Negative Normal

prematurity; congenital
hypoplastic anemia; other
causes of decreased production
Normal or t Normal
Acute hemorrhage

Negative Normal

(fetomaternal, placental,
umbilical cord, or internal
hemorrhage)
Hypochromic microcytes
Chronic fetomaternal

hemorrhage
Positive Spherocytes
Immune hemolysis (blood group
incompatibility or maternal
autoantibody)
Normal or T T
Hereditary spherocytosis

Negative Spherocytes
Elliptocytes

Hereditary elliptocytosis
Hypochromic microcytes
Alpha or gamma
thalassemia syndrome
Spiculated RBCs
Pyruvate kinase deficiency
Schistocytes and RBC
Disseminated intravascular
fragments
coagulation; other
microangiopathic processes
Bite cells (Heinz bodies
Glucose-6-phosphate
with supravital stain)
dehydrogenase deficiency
Normal
Infections; enclosed
hemorrhage
(cephalohematoma)
RBC, red blood cell; I, decreased; T, increased.
Anemia of prematurity is tempered by vitamin E
VOUS
and iron administration in premature formulas. Pre
S
mature infants tolerate hemoglobins of 6.5 to 8.0

NEONATAL CENTRAL NER


SYSTEM-

| DISORDER

g/dL. The level itself is not an indication for transfuApnea of Prematurity


sion. Transfusion should occur if another condition
exists that requires an increased oxygen-carrying
capacity, such as sepsis, necrotizing enterocolitisApne,
ture infant is defined as a cessapneumonia, chronic lung disease, and apnea.
r longer than 20 seconds or a

Pathogenesis
a in the prema
tion of breathing fo
shorter pause associ

ated with cyanosis, pallor, hypoKEY POINTS


te of less than 100 beats/min.

tonia, or a heart ra
Apnea in the full-te

rm neonate is defined as absent

1. Anemia in the neonate can result from blood loss,


than 16 seconds. In the premahemolysis, decreased red blood cell production, or
episodes may be due to central,
physiologically decreased erythropoiesis.
d mechanisms. In central apnea,
2. Neonatal anemia may be classified by evaluation
cessation of air flow and respiraof the reticulocyte count, bilirubin level, Coombs'
chest wall movement, whereas in
test, and red blood cell morphology (see Table
here is respiratory effort and
13-5).
but no air flow. Apnea of pre-

breathing for longer


ture infant, apneic
obstructive, or mixe
there is a complete
tory effort with no
obstructive apnea, t
chest wall movement
maturity usually has

a mixed central and obstructive

--------------------------------------- 97
186
pints Pediatrics
picture. Periodic breathing, which must be differenf respiratory stimulants (caffeine
tiated from apnea, is defined as pauses of 5 to 10
nea of prematurity may also be
seconds followed by a short period of rapid breathmanagesing the mean airway pressure
ing. Periodic breathing is normal.
PAP or intermittent assisted

and administration o
or theophylline}. Ap
d by increa
through the use of C
ventilation. For the

other causes of apnea, treatment


Epidemiology
of the underlying di
sorder usually leads to cessation
Apnea occurs in most infants of less than 28 weekso'
des.
gestation, approximately 50% of infants 30 to 32
aches 34 to 35 weeks' postconweeks' gestation, and in less than 7% of infants 34 to
lerating feeds orally, and has not
35 weeks' gestation.
dycardiac episode for 7 days, the

f the apneic episo


When an infant re
ceptional age, is to
had an apneic or bra

infant is ready to b
e discharged home. The apnea
Clinical Manifestations
monitor sent home wi
th the patient can be disconApnea of prematurity is associated with bradycardia,
tinued when the infa
nt has been apnea-free for 2
which is a heart rate less than 80 beats/min. Bradymonths.
cardia and cyanosis are usually present after 20
seconds of apnea but may occur more rapidly in the
K
EY POINTS
small, premature infant. After 30 to 40 seconds, pallor
and hypotonia are also seen, and the infant may be
1. Apnea in the pr
emature infant is defined as a cesunresponsive to tactile stimulation. A neonate may
sation of breat
hing for longer than 20 seconds or
rouse itself and stop the apneic spell, but more sympa shorter pause
associated with cyanosis, pallor,

tomatic apnea is apparent if a caregiver must touch


heart rate of less than 100
the infant to discontinue the apnea. With hypotonia
and pallor, bag-mask ventilation is required to return
e infant, apneic episodes may be
the child to a normal breathing pattern.
l, obstructive, or mixed mechanism.
A diagnosis of apnea of prematurity is made
or apnea of prematurity includes
after excluding other causes of apnea, which can
a skin-core temperature gradient
be grouped into the following broad categories:
r, supplemental oxygen, tactile
hypoxemia, diaphragmatic fatigue, respiratory center
ministration of respiratory stimudepression, infection, vagal stimulation, airway
the most severe cases, CPAP or interobstruction, and inappropriate environmental temd ventilation.
perature. Hypoxemia may result from anemia, hypovolemia, and congenital heart disease, whereas RDS
and pneumonia can cause diaphragmatic fatigue.
orrhage
Respiratory center depression can occur with metabolic abnormalities (hypoglycemia, hypocalcemia,
hyponatremia), drugs, seizures, or intraventricular
xclusively in preterm infants and
hemorrhage (IVH). Infectious processes such as
g of the germinal matrix, an area
sepsis, necrotizing enterocolitis, and meningitis all
ure that is the site of pluripotent
can cause apnea, whereas gastroesophageal reflux,
o form neurons and glia. Changes
suctioning of the oropharynx, and nasogastric tube
ow have been proposed as a conpassage can cause vagally mediated depression of the
Surges of cerebral arterial flow
respiratory center. Excessive oral secretions, anatomic
res, episodes of hypoxia, apnea,
obstruction, or malposition may result in obstructive
, rapid infusion of colloid, patent
apnea. Apnea is more frequent in the absence of a
nd ECMO. Increased venous presskin-core temperature gradient, and sudden increases
ed with RDS, pneumothorax,
in incubator temperature increase the frequency of
lure, ventilator parameters such as
apneic spells.
sity. IVH is very common

hypotonia, or a
beats/min.
2. In the prematur
due to a centra
3. The treatment f
maintenance of
in the incubato
stimulation, ad
lants, and, in
mittent assiste

Intraventricular Hem
Pathogenesis
IVH is seen almost e
results from bleedin
of immature vasculat
cells that migrate t
in cerebral blood fl
tributing mechanism.
may occur with seizu
respiratory distress
ductus arteriosus, a
sure may be associat
congestive heart fai
CPAP, and hypervisco
among VLBW infants,

and the risk decreases as gestaTreatment


. Approximately 50% of infants
Treatment for apnea of prematurity includes maindence of intracranial bleeding.
tenance of a skin-core temperature gradient in the
ar hemorrhages that are confined
incubator, supplemental oxygen, tactile stimulation,
ix (grade I) or are associated with

tional age increases


under 1500g have evi
Small intraventricul
to the germinal matr

--------------------------------------- 98
Chapter 13 / Neonatology
187
a small amount of blood in the ventricle (grade II)
KE
Y POINTS
often resolve without sequelae. Large IVHs that are
associated with ventricular dilatation (grade III) or
r hemorrhage is seen almost excluwith extension into the brain parenchyma (grade IV)
rm infants and results from bleedare associated with permanent functional impairment
inal matrix.
and hydrocephalus.
0% of hemorrhages occur in the
Posthemorrhagic hydrocephalus is a consequence
fe, and approximately 90% occur
of obstruction of the ventricular outlets (obstructive
t 3 days of life.
hydrocephalus) or of obliteration of the arachnoid
is minimized by preventing premavilli that ultimately absorb the cerebrospinal fluid
f possible, or through the use of
(communicating hydrocephalus). Hydrocephalus
natal resuscitation measures to
may be static, in which case no intervention is made,
mia and rapid cerebral flow
or it may be progressive, requiring the surgical placeilizing the arterial blood
ment of a ventriculoperitoneal shunt.
vascular volume, hematocrit, and

1. Intraventricula
sively in prete
ing of the germ
2. Approximately 5
first day of li
within the firs
3. The risk of IVH
ture delivery i
appropriate neo
minimize hypoxe
changes by stab
pressure, intra
oxygenation.

Clinical Manifestations
hemorrhages result in no longApproximately 50% of hemorrhages occur in the first
Of infants with grade III IVH, 30%
day of life, and approximately 90% occur within the
or and intellectual impairment;
first 3 days of life. Most hemorrhages are asymptoh grade IV IVH, 60% to 80%
matic. If a severe hemorrhage occurs, the neonate
nd intellectual disabilities.
may develop anemia, pallor, hypotension, focal neu-

4. Grades I and II
term morbidity.
to 45% have mot
of neonates wit
develop^motor a

rologic signs, seizures, an acute increase in ventilatory


assistance needs, apnea, and/or bradycardia.
Ultrasonography through the anterior fontanelle is
al seizures are categorized in the
the method of choice to screen for, grade, and follow
IVH. All premature infants with birth weights less

Neonatal Seizures
The causes of neonat

following list.

than 1500 g should have diagnostic ultrasound within


emia, electrolyte abnormali-

Metabolic: Hypoglyc
ties (hypocalcemi

a, hypomagnesemia, hyponathe first week of life.


tremia), inborn e
rrors of metabolism (organic
Treatment
of amino acid metabolism, pyriThe risk of IVH is minimized by preventing prema)
ture delivery if possible, or through the use of approg ingestion, neonatal drug

acidemias, error
doxine deficiency
Toxic: Maternal dru
withdrawal, inadv

ertent local anesthetic poisoning,


priate neonatal resuscitation measures to minimize

bilirubin

hypoxemia by stabilizing the arterial blood pressure,


intravascular volume, hematocrit, and oxygenation.
entricular, subdural, or subThe goal in acute management of IVH is to maintain
age
adequate cerebral perfusion and to control intracereal meningitis, viral encephalitis

Hemorrhagic: Intrav
arachnoid hemorrh
Infectious: Bacteri
Asphyxia: Hypoxic i

schemic encephalopathy
bral pressure. Normal arterial blood pressure is presyndromes: Cerebral dysgeneserved by volume replacement with packed red
blood cells or inotropic support or both. IVH is
abnormalities, phakomatoses
followed by serial ultrasound evaluation, because
sis)
ventriculomegaly occurs before there is an increase
ficult to differentiate from benign
in head circumference. Progressive posthemorrhagic
n neonates with hypoglycemia or
hydrocephalus is treated by placement of a venfants of diabetic mothers, in
triculoperitoneal shunt.
tic withdrawal syndrome, and in
Outcome is dependent on the severity of the
isode of asphyxia. In contrast to
IVH. Grades I and II hemorrhages rarely result iseizuresn
and tremors are sensory dependent,
long-term morbidity. Of infants with grade III IVH,
, and may be interrupted by
30% to 45% will have motor and intellectual impairty. Seizure activity is coarse, with
ment; of neonates with grade IV IVH, 60% to
c activity, whereas jitters are char80% of neonates develop motor and intellectual
very rapid movement. It is often
disabilities.
fy seizures in the newborn period

Genetic/'dysmorphic

sis, chromosomal
(tuberous sclero
Seizures are dif
jitters or clonus i
hypocalcemia, in in
newborns with narco
infants after an ep
, jitters
elicited by stimuli
holding the extremi
fast and slow cloni
acterized by fine,
difficult to identi

--------------------------------------- 99
188
rints Pediatrics
because the infant, especially the LEW infant, usually
ons
does not demonstrate the tonic-clonic major motor
nd perinatal history may shed
activity typical of the older child.
etiology. The diagnostic evaluaSubtle seizures constitute 50% of seizures in newseizures should include a deterborns (both term and preterm]. Subtle seizure activvels of glucose, sodium, calcium,
ity may include rhythmic fluctuations in vital signs,
ia. In the jaundiced neonate,
apnea, eye deviation, nystagmus, tongue thrusting,
ubin level is indicated. When
eye blinking, staring, and "bicycling" or "swimming"
ed, a blood culture and lumbar
movements. Continuous bedside EEG monitoring
ed. If an inborn error of metabcan help identify subtle seizures.
urine organic acids and serum
The movements in focal clonic seizures involve
xamined. Further evaluation
well-localized clonic jerking. These types of seizures
sound or CT scan of the head.
are not associated with loss of consciousness and
ion or head imaging suggest a
are most often provoked by metabolic disturbances.
, appropriate cultures, antibody
Subarachnoid hemorrhage and focal infarct may
PCR should be done. Continualso promote this type of seizure. The EEG is
d EEG monitoring provides the
unifocally abnormal, but the prognosis is generally
good.
defining the type of seizure
Multifocal clonic seizures are characterized by
EEG with pyridoxine infusion
random clonic movements of the limbs. Multifocal
presence or absence of pyridoxanomalies are seen on the EEG, and the prognosis is
eizures result from narcotic withpoor.
ontrolled wean is indicated.

Clinical Manifestati
A careful prenatal a
light on the seizure
tion of infants with
mination of blood le
magnesium, and ammon
measurement of bilir
infection is suspect
puncture are perform
olism is suspected,
amino acids may be e
may include an ultra
If physical examinat
congenital infection
determinations, and
ous bedside video an
best information in
present. Continuous
helps establish the
ine deficiency. If s
drawal syndrome, a c
Treatment

Tonic seizures manifest as extensor posturing with


tonic eye deviation and are most often seen in premary cause of the seizure should
mature neonates with diffuse central nervous system
eated. Correct any metabolic disdisease or IVH. The EEG has multifocal abnormali(hyperammonemia, hyperbiliruties. The prognosis is generally poor.
as the etiology of the seizure,
Synchronized single or multiple slow jerks of
may be used to remove it.
the upper or lower limbs (or both) characterize

If possible, the pri


be identified and tr
turbance. If a toxin
binemia) is isolated
exchange transfusion
Meningitis is treate

d with the appropriate antibiotic


myoclonic seizures. These seizures are noted when
e of an identifiable cause, antithere is diffuse central nervous system pathology,
s used. Agents include phenoand the prognosis is poor. The EEG shows a burst/
(Dilantin), lorazepam (Ativan],
suppression pattern.
). Phenobarbital is standard
Seizures noted in the delivery room may be due
nytoin is used when seizures
to direct injection of local anesthetic into the fetal
barbital level greater than 50
scalp, severe anoxia, or congenital brain malformaoutcome of neonatal seizures is
tion. Seizures due to hypoxic-ischemic encephalopape of seizure and its etiology.
thy (postasphyxial seizures], a common cause of
seizures in the full-term neonate, usually occur 12 to
24 hours after a history of birth asphyxia and are
EY POINTS
often refractory to conventional doses of anticonvulesult from metabolic disturbances,
sant medications. Postasphyxial seizures may also
of metabolism, toxic exposures,
result from metabolic disorders such as hypoain insult, infectious etiologies,
glycemia and hypocalcemia. IVH is a common cause
genetic defects.
of seizures in premature infants and often occurs
es are divided into focal clonic,
from the first to third days of life. Seizures with IVH
nic, tonic, myoclonic, and tonicmay be associated with a bulging fontanelle, hemors.
rhagic spinal fluid, anemia, lethargy, and coma.
ide video EEG monitoring proSeizures after the first 5 days of life may be due to
information in defining the type of
infection or drug withdrawal. Seizures associated
t
with lethargy, acidosis, ketonuria, respiratory alkalos the primary anticonvulsant used
sis, and a family history of infantile death may be due
atal seizures.
to an inborn error of metabolism.

agent. In the absenc


convulsant therapy i
barbital, phenytoin
and diazepam (Valium
primary therapy. Phe
persist with a pheno
mg/L. The long-term
determined by the ty

K
1. Seizures may r
inborn errors
hemorrhagic br
asphyxia, and
2. Neonatal seizur
multifocal clo
clonic seizure
3. Continuous beds
vides the best
seizure presen
4. Phenobarbital i
to manage neon

--------------------------------------- 100
189
Chapter 13 / Neonatology
m NEONATAL DISORDERS OF THE
ntary or ectopic thyroid gland.
ENpOCRJNE SYSTEM
med before therapy commences

presence of a rudime
Scans must be perfor
and the TSH decrease

s. Maternal antibodies can supHypothyroidism

press the newborn th

yroid gland function temporarily so that there is


no uptake by the thyroid gland on
The physical stigmata of congenital hypothyroidism

scan.

in the newborn are often too subtle for physical diagrted within the first month after
nosis, so clinicians rely heavily on diagnostic screenis excellent. The thyroxine dose
ing. All states currently require newborn screening
justed, because too little thyfor hypothyroidism. The sooner treatment is initirsistent hypothyroidism, whereas
ated, the better will be the prognosis for normal
ay result in advanced bone age
intellectual development in the child. In most cases
.
the diagnosis can be made and treatment initiated

If therapy is sta
birth, the prognosis
must be carefully ad
roxine results in pe
too much thyroxine m
and craniosynostosis

within 4 weeks.
The etiology is usually sporadic athyreosis or
thyroid ectopy. Less common is familial goitrous
EY POINTS
hypothyroidism. Children of mothers with Graves'

1. All states curr


ently require newborn screening for
disease who are receiving propylthiouracil have

hypothyroidism.

transient hypothyroidism.
tarted within the first month after

2. If therapy is s
birth, the prog

nosis for normal intellectual develClinical Manifestations


hild is excellent.
Primary hypothyroidism is indicated by a low T

opment in the c
4 level

and an elevated thyroid-stimulating hormone (TSH).


Serum levels should be drawn to confirm abnormal
screening results.
ia
A low T 4 level accompanied by a low TSH value

Neonatal Hypoglycem

The definition of hy
poglycemia in the neonate has
may indicate a physiologically normal thyroid status
decades of debate. Full-term
caused by a low concentration of thyroid-binding
ntly have a transient hypoglobulin (TBGJ. This is frequently observed in preglucose measurements in the
mature infants or may be seen on a hereditary basis.
taneously recover. As a result,
Alternatively, a low T 4 and low TSH with a normal
l definitions of hypoglycemia genTBG level may indicate hypopituitarism or hypo-

been the subject of


neonates will freque
glycemia with blood
30s (mg/dL) and spon
published statistica
erally use a level i

n the mid-3 Os. However, persistent


thalamic deficiency. Hypothalamic deficiency usually
60 should prompt consideration of
is accompanied by growth hormone or corticotropin
pathologic processes.
deficiency, which may cause acute hypoglycemia.
An algorithm for the diagnosis of hypothyroidism is

levels of less than


and evaluation for

Pathogenesis

delineated in Figure 13-3.


Infants with hypogly
cemia may be divided into those
Treatment
and those without hyperin-

with hyperinsulinism

sulinism. Infants w
ith transient hyperinsulinism
If the screening results indicate primary hypothyinclude infants of
diabetic mothers and infants with
roidism, the T 4 and TSH studies should be repeated
Rh hemolytic diseas
e. Infants with protracted hyperand therapy started. Serum T
4 is measured after 5
insulinism include
those who have Beckwith-Wiededays of therapy, and the thyroxine dosage is adjusted
mann syndrome, isle
t cell adenomas, and functional
to keep the T 4 level in the upper half of the normal
hyperinsulinism. In
fants who do not have hyperinrange for age. The TSH concentration may remain
sulinism and have t
ransient hypoglycemia include
elevated for months in some patients because of
those with intraute
rine growth retardation, birth
immaturity of the feedback mechanism. Levothyasphyxia, polycythe
mia, cardiac disease, central
roxine is administered at an initial dose of 10(j,g/kg.
nervous system dise
ase, sepsis, maternal use of proTablets are crushed and given orally.
pranolol, oral hypo
glycemic agents, or narcotic addicBefore therapy commences, a bone age and a
tion. Infants who d
o not have hyperinsulinism but
thyroid scan should be done. An iodinated
123I or have protracted hyp
oglycemia include those with
technetium scan of the thyroid gland evaluates the
neonatal hypopituit
arism or defects in carbohydrate

--------------------------------------- 101
Chapters / Cardiology
19
KEY POINTS
1. The absolute concentration of deoxygenated
hemoglobin, and not the ratio of oxygenated to
deoxygenated hemoglobin, determines the presence of cyanosis.

2. Once cyanosis has been identified, stabilize the


infant, quickly initiate the preliminary workup
(chest radiograph, electrocardiogram, and hyperoxia test) and define whether the lesion is cardiac
or noncardiac in origin.
3. If a cardiac lesion is suspected, determine whether
the defect is ductal dependent for systemic or
pulmonary circulation or a ductal-independent
mixing lesion.
4. Once the infant is stabilized, obtain an emergent
cardiology consultation, an echocardiogram, and,
if indicated, begin PGE, therapy in preparation for
surgical palliation or correction.
Figure 3-1

Truncus a

rteriosus (with right aortic arch).Typical


anatomic and hemodyn
amic findings include: (a) a single
CYANOTIC CONGENITAL HEART
conotruncus giving rise to coronary arter-

artery arises from the


ies (not shown), pul

monary arteries, and aortic vessels; (b)


DISEASE: DUCTAL-INDEPENDENT
ve (quadricuspid shown) with stenosis
MIXING LESIONS
is common; (c) right-sided aortic arch

abnormal truncal val


and/or regurgitation
(occurs in approxima

tely 30% of cases); (d) large conTruncus Arteriosus


ular septal defect; (e) pulmonary artery

oventricular ventric
hypertension with a

large left-to-right shunt (note superior


Truncus arteriosus (Figure 3-1) is a rare form of cyanturation of 60% and pulmonary artery
otic congenital heart disease that consists of a single
85%); (f) complete mixing (of the
arterial vessel arising from the base of the heart from
ry venous return) occurs at the great
which arise the coronary, systemic, and pulmonary

vena caval oxygen sa


oxygen saturation of
systemic and pulmona
vessel level.
Cloherty JP, Stark A

R. Manual of Neonatal Care, 4th ed. Philadelphia:


arteries. In this disorder, there is complete mixing of
98:426.
systemic and pulmonary venous blood in the truncus.
This lesion, along with other conotruncal anomalies

Lippincott-Raven, 19

(tetralogy of Fallot, interrupted aortic arch, VSD,


isolated arch anomalies, and vascular ring), is associated with microdeletion of chromosome 22 (22ql 1
cular hypertrophy on ECG. On
deletion).
rked cardiomegaly, increased

riosus have biventri


chest radiograph, ma
pulmonary vascularit

y, and right aortic arch may be


seen. DiGeorge's syn
drome related to the 22qll
Clinical Manifestations
sult in hypocalcemia.
Moderate cyanosis is present at birth, and congestive

microdeletion may re

heart failure develops in a matter of weeks as the pul-

Treatment

monary vascular resistance falls and shunting across


atal surgical repair is perthe ventricular septal defect begins. On examination,
air involves closing the ventrica systolic ejection murmur is heard at the left sternal
o the oxygenated blood in the left
border, a widened pulse pressure is present, and
through the VSD to the truncal
bounding arterial pulses are palpated. There is a
is interposed between the right
single loud second heart sound on cardiovascular
ary arteries, which are disconexam. Seventy percent of children with truncus artecal vessel.

At most centers neon


formed. Surgical rep
ular septal defect s
ventricle is baffled
valve and a conduit
ventricle and pulmon
nected from the trun

--------------------------------------- 102
190

Blueprints Pediatrics
TSH
on screen

Bone age
Thyroid scan
Repeat T4 & TSH levels
Treat immediately
with levothyroxine
Figure 13-3
Algorithm for the diagnosis of hypothyroidism.
and/or amino acid metabolism. Deficiencies of
phatase deficiency,
fructose intolerance, galacgrowth hormone or corticotropin or both cause
tosemia, and pyruvat
e carboxylase deficiency. Disorhypoglycemia in neonatal hypopituitarism. Defects
ders of amino acid m
etabolism that result in
in carbohydrate metabolism that result in hypohypoglycemia include
methylmalonic acidemia,
glycemia include glycogen storage disease type Ityrosinosis,
, propio
nic acidemia, and maple syrup
glycogen synthetase deficiency, fructose-1,6-diphosurine disease.

--------------------------------------- 103
Chapter 13 / Neonatology
Clinical Manifestations

191
2. Infants who do

not have hyperinsulinism and


The onset of hypoglycemia may occur anywhere
hypoglycemia include those with
from a few hours after birth to several days of age.
owth retardation, birth asphyxia,
Subtle symptoms such as poor feeding, apathy,
ardiac disease, central nervous
lethargy, and hypotonia are most common, but lifeand sepsis and whose mothers
threatening manifestations such as seizures, apnea,
anolol, oral hypoglycemic agents,
and cyanosis may also occur.
In persistent or recurrent hypoglycemia, consider
inborn errors of metabolism. When the infant is
not have hyperinsulinism and

have transient
intrauterine gr
polycythemia, c
system disease,
have used propr

and narcotics.
3. Infants who do
have protracted

hypoglycemia include those with


hypoglycemic, obtain serum for glucose, insulin, cortuitarism and defects in carbohytisol, growth hormone, lactate, and pyruvate levels.
m and in amino acid metabolism.
Serum amino acid screening is indicated if no defininfants, an intravenous push of
itive diagnosis is made; the infant need not be hypotrose is followed by an
glycemic at the time of the sample collection.
ravenous dextrose at a rate of 5 to

neonatal hypopi
drate metabolis
4. In symptomatic
2 ml/kg 10% dex
infusion of int
7mg/kg/min.

Treatment
In asymptomatic infants, oral feedings can be
attempted. If oral feedings are not accepted, an intravenous infusion of maintenance dextrose at 5 to
7mg/kg/min is initiated.
In symptomatic infants, an intravenous push of
stula
2mL/kg 10% dextrose precedes infusion of intra-

CONGENITAL ANOMALIES
Tracheoesophageal Fi

venous dextrose at a rate of 5 to 7mg/kg/min. The


the esophagus develops as
rate is adjusted to keep the blood glucose leveal
e superior portion of the primibetween 60 and 120mg/dL. Rebound hypoglycemia
here is abnormal anastomosis
may occur if the dextrose infusion is abruptly
rior portions of the esophagus,
decreased. Dextrostix values are useful for screening

The lower section of


n elongation of th
tive foregut. When t
of superior and infe
esophageal atresia r

esults. Of neonates with


blood sugar; abnormal values should be verified with
85% have tracheoesophageal
a true blood sugar. When the infant is stabilized, the
our types of tracheoesophageal
dextrose infusion rate is slowly decreased, with
n Figure 13-4. Esophageal atresia
careful monitoring of blood glucose. After dextrose
unts for 85% of the cases of TEF.
infusion is discontinued, the blood glucose leveFortl
tients with TEF have other
should be monitored for 24 hours.
cardiovascular anomalies include
Glucagon in doses of 300|_ig/kg to 1 mg/kg can bpatene
riosus, vascular ring, and coarctaused in conditions with adequate glycogen storestio,
he incidences of imperforate
such as hyperinsulinism. Glucocorticoids are used as
nd duodenal anomalies also are
replacement therapy in infants with hypoadrenalism.
yndrome describes the assoGrowth hormone is helpful in those infants with
, anal, cardiac, tracheal,
growth hormone deficiency. Diazoxide can be
nd limb anomalies.
administered in hyperinsulinemic states and may
ions
serve as a diagnostic technique, because patients with
insulinomas are far less likely to respond to diazoxve excessive oral secretions,
ide than are functional hyperinsulinemic patients.
agging, and respiratory distress.
Pancreatectomy is reserved for intractable hypois noted on ultrasound while
glycemia due to hyperinsulinism. If an isolated tumor
o. Lateral and anteroposterior
is found, it must be removed.
the thoracocervical region and

esophageal atresia,
fistula (TEF). The f
atresias are shown i
with distal TEF acco
y percent of pa
defects. Associated
t ductus arte
n of the aorta. T
anus, malrotation, a
increased. VACTERL s
ciation of vertebral
esophageal, renal, a
Clinical Manifestat

Neonates with TEF ha


inability to feed, g
Polyhydramnios often
the child is in uter
chest radiographs of
abdomen with a Repl

ogle tube in the proximal


KEY POINTS
blind pouch with air in the gas1. Infants with hypoglycemia may be divided into
In esophageal atresia without TEF,
those with hyperinsulinism and those without
the gastrointestinal tract, whereas
hyperinsulinism.
ageal atresia (H type), infants

esophagus reveal a
trointestinal tract.
gas is absent from
in TEF without esoph
may have nonspecific

symptoms for several months,

--------------------------------------- 104
192

Blueprints Pediatrics
Esophageal atresia
H-type TEF

Esophageal atresia

with distal TEF

with no TEF

(4%)
(85%)

(8%)
Esophageal atresia

Esophageal atres

with proximal TEF

with proximal and dis

ia
tal TEF
(2%)
Figure 13-4

(1%)

Types of tracheoesophageal fistulas with relative frequencies.

including chronic cough with feeding and recurrent

Duodenal Atresia

pneumonia.
Duodenal obstruction
may be complete (atresia} or
Treatment
owing to a web, band, or annular

partial (stenosis),
pancreas. Duodenal a

tresia results from a failure of


Placing the infant in a 60-degree head-up prone
position and minimizing disturbance of the infant
ize during the eighth to tenth
are recommended to prevent reflux and aspiration of
Seventy percent of the cases of

the lumen to recanal


weeks of gestation.
duodenal atresia are

associated with other malforgastric contents. To remove swallowed oral secretions


ardiac anomalies and gastroinfrom the proximal esophageal pouch, a Replogle
tube may be placed to suction. The usual corrective
h as annular pancreas, malrotation

mations, including c

testinal defects suc


of the intestines, a

nd imperforate anus. Twenty-five


procedure is division and closure of the TEF and endith duodenal atresia are premato-end anastomosis of the proximal and distal esophia is often associated with
agus. If the distance between esophageal segments is
too long for primary anastomosis, delayed anastomo-

percent of infants w
ture. Duodenal atres

trisomy 21.

sis follows stretching of the upper segment. Strictures


at the anastomosis site require periodic dilation.
ons

Clinical Manifestati
With complete obstru

ction, in utero polyhydramnios


may be present. Afte
r birth, bilious emesis begins
KEY POINT
within a few hours a
fter the first feeding. Abdominal
1. When there is abnormal anastomosis of superior
show gastric and duodenal

radiographs usually

and inferior portions of the esophagus,


roximal to the atretic site. This
esophageal atresia results. Of neonates with
the "double bubble" sign. The
esophageal atresia, 85% have tracheoesophageal
he distal bowel suggests partial
fistula.
ontrast radiographic study of the

gaseous distention p
finding is known as
presence of gas in t
obstruction, and a c
abdomen should be pe

rformed.

--------------------------------------- 105
Chapter 13 / Neonatology
193
Treatment
anced chest drainage is needed to
Treatment is surgical. Mortality is related to premasalveolar pressure gradients.
turity and other associated anomalies.

tive repair with bal


avoid excessive tran

KE
Y POINTS
KEY POINTS
hragmatic hernia results from a

1. Congenital diap
defect in the l

eft posterolateral diaphragm that


1. Duodenal atresia results from a failure of the
l contents to enter the thorax
lumen to recanalize during the eighth to tenth
lung development.
weeks of gestation.
of pulmonary hypoplasia and

allows abdomina
and compromise
2. The combination
pulmonary arter

iolar hypertension makes this


2. Seventy percent of the cases of duodenal atresia
are associated with other malformations, includct lethal in many cases.
ing cardiac anomalies and gastrointestinal defects

congenital defe

such as annular pancreas, malrotation of the


intestines, and imperforate anus.
Omphalocele
Omphalocele results
when the abdominal viscera herniate through the um
bilical and supraumbilical portions of the abdomin
al wall into a sac covered by
Congenital Diaphragmatic Hernia
tic membrane. The defect
Congenital diaphragmatic hernia results from a
d folding of the embryonic disc.
defect in the posterolateral diaphragm that allows

peritoneum and amnio


results from arreste
Large defects may co

ntain the entire gastrointestinal


abdominal contents to enter the thorax and comand spleen. The sac covering the
promise lung development. This defect is commonly
ay rupture in utero or during delivreferred to as a Bochdalek hernia. Ninety percent
f omphalocele is 1 in 6000 births.
of congenital diaphragmatic hernias occur on the

tract and the liver


defect is thin and m
ery. The incidence o

Clinical Manifestati
ons
left side of the diaphragm. The combination of pulmonary hypoplasia and pulmonary arteriolar hyperted in utero, and 10% of infants
tension makes this congenital defect lethal in many
e born prematurely. Diagnosis
cases.
natal ultrasound. Thirty-five

Polyhydramnios is no
with omphaloceles ar
is often made by pre
percent of afflicted

infants have other gastrointestiClinical Manifestations


nal defects, and 20%
have congenital heart defects.
Early symptoms include respiratory distress with
ren with omphalocele have
decreased breath sounds on the affected side and
yndrome (exophthalmos,
shift of heart sounds to the opposite side with a
ism, hyperinsulinemia, and
scaphoid abdomen. Diagnosis is sometimes made via

Ten percent of child


Beckwith-Wiedemann s
macroglossia, gigant
hypoglycemia).

ultrasound while the fetus is in utero. If the diagnosis is not known at birth, a simple chest radiograph

Treatment

will make the diagnosis.


prevent rupture of the sac.

Cesarean section may


Small defects are cl

osed primarily, whereas larger


Treatment
e a staged repair that involves
Because of pulmonary hypoplasia and pulmonary
h prosthetic material.
hypertension, the child must be intubated and venintact omphalocele sac includes
tilated. Sometimes conventional ventilation is not
ttent nasogastric tube suction to
sufficient to provide adequate oxygen delivery and
tinal distention, covering the sac
carbon dioxide excretion; in such cases, highegnated gauze, wrapping the
frequency ventilation or ECMO may be needed to
ile towel to minimize heat loss,
manage the child's pulmonary hypertension. A
on the abdomen with Kling
Replogle tube is placed to minimize gastrointestinal
viscera on the abdominal wall.
distention that would further diminish effective lung

defects often requir


covering the sac wit
Treatment of the
low-pressure intermi
minimize gastrointes
with petrolatum-impr
infant in a dry ster
and wrapping the sac
gauze to support the
There should be no a

ttempt to reduce the sac,


volume. Medical management of the associated pulse rupture of the sac, interfere
monary hypertension was discussed earlier. Operarom the sac, and cause respira-

because this may cau


with venous return f

--------------------------------------- 106
1-94
Blueprints Pediatrics
tory distress. Broad-spectrum antibiotics should be
Palate
given. Surgical consultation should be arranged, and
definitive surgery should be delayed until the infant
is thoroughly resuscitated. Definitive care can be
thout cleft palate occurs in 1 in
postponed as long as the sac remains intact.
ore common in males. UnilatTreatment of the ruptured sac is similar to that of
e result of failure of the ipsilateral
the intact sac, except that saline-soaked gauze is
to fuse with the medial nasal
placed over the exposed intestine, and emergent surcess produces a persistent labial
gical intervention is needed to cover the intestine.
ilateral fusion produces bilateral
KEY POINTS
rs in 1 in 2500 births. Develop-

Cleft Lip and Cleft


Pathogenesis
Cleft lip with or wi
1000 births and is m
eral cleft lip is th
maxillary prominence
prominence. This pro
groove. Failure of b
cleft lip.
Cleft palate occu
ment of the palate p

roper, which includes the hard


1. Omphalocele results when the abdominal viscera
uvula, and maxillary teeth, is comherniate through the umbilical and supraumbiliweek of gestation. This region
cal portions of the abdominal wall into a sac
xillary bone plates that are
covered by peritoneum and amniotic membrane.
by the tongue. As the tongue
2. Omphalocele has a high association with other
r of the mouth and moves
anomalies, including gastrointestinal and cardiac
tes fuse. Failure of the tongue to
abnormalities and Beckwith-Wiedemann
midline palatal clefts.
syndrome.
Gastroschisis
environmental factors play a

palate, soft palate,


pleted by the ninth
develops from the ma
initially separated
descends in the floo
forward, the two pla
descend produces the
Epidemiology
Multiple genetic and
role in the etiology

of the cleft lip. The recurrence


Gastroschisis, by definition, contains no sac; the intes3% to 4%. The risk for a child with
tine is herniated through the abdominal wall lateral
lip is 14%. Genetic factors are
to the umbilicus. The eviscerated uncovered mass is
eft palate, and the recurrence
adherent, edematous, dark in color, and covered by a
that for cleft lip. Cleft palates

risk in siblings is
a mother with cleft
also important in cl
risk is the same as

gelatinous matrix of greenish material. The pathots with chromosomal


genesis of this abdominal wall defect is not clear.
Clinical Manifestations
Polyhydramnios is noted in utero. Sixty percent of
ons
these infants are born prematurely, and 15% have
ated with cleft lip include
associated jejunoileal stenoses or atresias.
defects, and cardiac anomalies.
Treatment
difficulties are not seen in isolated

are common in patien


abnormalities.
Clinical Manifestati
Malformations associ
hypertelorism, hand
In general, feeding
cleft lip.

Treatment of gastroschisis involves placement of a


nasogastric tube to suction, covering the exposed
intestine with saline-soaked gauze, wrapping the
infant in a dry, sterile towel to minimize heat loss,
repaired shortly after birth or
and starting antibiotics to cover for infection caused
onstrates steady weight gain.
by bowel flora. Gastroschisis is a surgical emergency,
is usually undertaken at 12 to 24
and single-stage primary closure is possible in only
he newborn period, respiratory
10% of infants.

Treatment
Most cleft lips are
once the infant dem
Cleft palate repair
months of age. In t
and feeding problem

s may occur. Repositioning the


KEY POINTS
he baby on his or her side should

tongue and feeding t


resolve respiratory

difficulties. Most patients do well


1. Gastroschisis, by definition, contains no sac; the
ipple that has a hole that is longer
intestine is herniated through the abdominal wall
ations after cleft palate repair
lateral to the umbilicus.
iculties, dental disturbances, and
2. Gastroschisis has a lower association with other
dia. Although two-thirds of chilanomalies compared to omphalocele.
ceptable speech, it may have a

with a long, soft n


than usual. Complic
include speech diff
recurrent otitis me
dren demonstrate ac
nasal quality or a

muffled tone.

--------------------------------------- 107
Chapter 13 / Neonatology

195
dandruff shampoo. Fo

r seborrhea of the diaper


KEY POINTS
one cream can be used. If
1. Most cleft lips are repaired shortly after birth or
ion appears, nystatin ointment is
once the infant demonstrates steady weight gain.
2. Cleft palate repair is usually undertaken at 12 to

area, 1% hydrocortis
candidal superinfect
recommended.

24 months of age.
3. In the newborn period, respiratory and feeding
problems may occur with cleft lip or cleft palate.
transient dark blue-black pig-

Mongolian Spots
Mongolian spots are
mented macules seen

over the lower back and buttocks in 90% of Afri


can-American, Indian, and Asian
Neural Tube Defects
re never elevated or palpable and

infants. The spots a


result from infiltra

tion of melanocytes deep into the


Neural tube defects are discussed in detail in
mented areas fade as the child
Chapter 15.
o known long-term problems but

dermis. The hyperpig


ages. They present n
may occasionally be

mistaken for bruises inflicted by


child abusers.
NEONATAL DERMATOLOGIC
PROBLEMS
KE
Y POINTS
Erythema Toxicum Neonatorum
m neonatorum occurs 24 to 72
The rash of erythema toxicum consists of evanescent
th and resolves 3 to 5 days later
papules, vesicles, and pustules on an erythematous
. Fifty percent of full-term babies
base that usually occur on the trunk (but sometimes
oxicum.
appear on the face and extremities). Rash onset
rmal cysts of the nose, chin, and
usually occurs 24 to 72 hours after birth but may be
seen earlier. Gram stain of vesicular contents reveals
atitis appears between 2 and 10
sheets of eosinophils. The lesions resolve over 3 to
nd is commonly called "cradle cap"
5 days without therapy. Fifty percent of full-term
on the scalp.
babies have erythema toxicum. This figure decreases
are benign, transient, dark blueas the gestational age decreases. The cause of the rash
macules seen over the lower
is unknown.
ks in 90% of African-American,

1. Erythema toxicu
hours after bir
without therapy
have erythema t
2. Milia are epide
forehead.
3. Seborrheic derm
weeks of life a
when it appears
4. Mongolian spots
black pigmented
back and buttoc
Indian, and Asi

an infants.
Milia
Milia is characterized by pearly white or pale yellow
epidermal cysts found on the nose, chin, and forehead. The benign lesions exfoliate and disappear

DRUGS OF ABUSE

Fetal Alcohol Syndro


me
within the first few weeks of life. No treatment is
necessary.
common teratogen to which

Alcohol is the most


fetuses are exposed.

Maternal alcohol ingestion


results in a spectru
m of effects in the neonate,
Seborrheic Dermatitis
duction in cerebral function to
Seborrhea is characterized by erythematous, dry,
l syndrome. The amount of alcohol
scaling, crusty lesions. It occurs in areas rich in sebaer appears to correlate with
ceous glands (face, scalp, perineum, and postauricular
the fetus is affected. Fetal
and intertriginous areas). Affected areas are sharply
urs in 1 in 1000 newborns. The
demarcated from uninvolved skin. Seborrhea appears
gher in the Native American
between 2 and 10 weeks and is commonly called
f a higher incidence of alco"cradle cap" when it appears on the scalp.
affects 40% of the offspring of
For severe cradle cap, baby oil is applied to thwomee
more than four to six drinks
scalp for 15 minutes, followed by washing with a
nt.

ranging from mild re


classic fetal alcoho
consumed by the moth
the degree to which
alcohol syndrome occ
incidence is much hi
population because o
holism. The syndrome
n who consume
per day while pregna

--------------------------------------- 108
Blueprints Pediatrics
196
Clinical Manifestations
Features of fetal alcohol syndrome include microperiod, therapy is supportive.
cephaly and mental retardation, intrauterine growth
may be helpful, but frequently
retardation, facial dysmorphisms, and renal and
logic interventions may be
cardiac defects. Facial anomalies include midfacial
age, many of these children have
hypoplasia, micrognathia, a flattened philtrum, short
ds.
palpebral fissures, and a thin vermillion border.
Treatment
Treatment is aimed at minimizing morbidity and
Y POINTS
mortality from renal and cardiac defects and assistplacental insufficiency and fetal
ing the child with mental retardation with activities
is associated with increased rates
of daily living.
abortion, placental abruption,

Treatment
During the perinatal
Sedative medications
soothing nonpharmaco
adequate. At school
special learning nee

KE
1. Cocaine causes
hypoxia, which
of spontaneous
fetal distress,

meconium staining, preterm birth,


intrauterine gr

owth retardation, and low Apgar


KEY POINTS
.

scores at birth
2. Infants may und

ergo withdrawal, characterized by


1. Alcohol is the most common teratogen to which
ncreased tremulousness, state lability,
fetuses are exposed.
consoled, and poor feeding in the
2. Fetal alcohol syndrome affects 40% of the offof life.
spring of women who consume more than four to
six drinks per day while pregnant.
ts include attention and concen3. Features of fetal alcohol syndrome include micros and an increased incidence of
cephaly and mental retardation, intrauterine
lities.
growth retardation, facial dysmorphisms, and
renal and cardiac defects.

irritability, i
inability to be
first few days
3. Long-term defec
tration deficit
learning disabi

Heroin and Methadone


Heroin and methadone
are the two narcotics to
Cocaine
st commonly exposed. About
Cocaine causes maternal hypertension and placental
ent babies are born in the
vasoconstriction with diminished uterine blood flow
ear, and 5000 narcotic-addicted
and fetal hypoxia. These effects are associated with
n methadone treatment proincreased rates of spontaneous abortion, placental
ntenance is prescribed for
abruption, fetal distress, meconium staining, preterm
crease the stress that unrelibirth, intrauterine growth retardation, and low Apgar
nd uncontrolled withdrawal in
scores at birth.
fetus.
Clinical Manifestations
ons
Maternal cocaine use is associated with congenital
associated with congenital anomanomalies, intracranial hemorrhage, and necrotizing
use causes intrauterine growth
enterocolitis. Congenital anomalies include cardiac
eased risk of SIDS, and infant
defects, skull abnormalities, and genitourinary malsyndrome. It is unclear whether
formations. Cocaine-exposed infants have demonfetal growth seen with narcotic
strated abnormalities in respiratory control and have
direct effect of the drug or to
an increased risk of SIDS. Long-term defects include
factors, such as poor maternal
attention and concentration deficits and an increased
incidence of learning disabilities.
al syndrome, which generally
Infants may undergo withdrawal, characterized by
rst 4 days of life, is characterized

which fetuses are mo


10,000 heroin-depend
United States each y
pregnant women are i
grams. Methadone mai
pregnant women to de
able heroin dosing a
utero places on the
Clinical Manifestati
Opiate abuse is not
alies, but maternal
retardation, an incr
narcotic withdrawal
the abnormalities of
abuse are due to the
other environmental
nutrition.
Narcotic withdraw
occurs within the fi

irritability, increased tremulousness, state lability,


r sleeping, a high-pitched cry, diarinability to be consoled, and poor feeding in the first
zing, seizures, poor feeding, and
few days of life.
e risk of neonatal withdrawal

by irritability, poo
rhea, sweating, snee
poor weight gain. Th

--------------------------------------- 109
Chapter 13 / Neonatology
197
is higher with methadone (75%) than with heroin
nzodiazepines. Paregoric and
(50%). Methadone withdrawal tends to be later in
so are used.
onset and more protracted, sometimes lasting as
long as 1 month. Symptoms appear soon after birth,
Y POINTS
improve, and then may recur at 2 to 4 weeks.

phenobarbital, or be
tincture of opium al

KE

1. Heroin and meth


adone are the two narcotics to
Treatment
re most commonly exposed.
The treatment for narcotic withdrawal syndrome
adone are not associated with
attempts to minimize irritability, emesis, and diarrhea
alies, but maternal use does
and to maximize sleep between feedings. Symptoine growth retardation and infant
matic care includes holding, rocking, and swaddling
awal syndrome.
the infant and providing the neonate with frequent
otic-abusing mothers should never
small feedings of a hypercaloric formula.
ne in the delivery room because it
Infants of narcotic-abusing mothers should never
seizures.
be given naloxone in the delivery room because it
awal symptoms unresponsive to
may precipitate seizures. Narcotic withdrawal sympre can be mitigated by the use of
toms unresponsive to nonmedicinal care can be
tions.
mitigated by a controlled wean of morphine,

which fetuses a
2. Heroin and meth
congenital anom
cause intrauter
narcotic withdr
3. Infants of narc
be given naloxo
may precipitate
4. Narcotic withdr
nonmedicinal ca
sedative medica

--------------------------------------- 110
Nephrology and
Urology
The renal system is the primary regulator of body
ally discovered during evaluation
fluid volume, osmolarity, composition, and pH. The
mass. Similar dilation is found

The condition is usu


of a palpable renal

kidneys collect and excrete many waste products oif


ducts, with varying degrees of
metabolism, such as urea and creatinine, and preserve
Most patients die before reachthe ionic equilibrium by conserving or excreting speutosomal dominant form of
cific electrolytes as needed. Infants, in particular, are
sease usually is detected in adultsusceptible to renal challenges. Their kidneys are less
ily history warrants early inveseffective in filtering plasma, regulating electrolytes,
may be very large. Hypertension
and concentrating urine.
al insufficiency develop.
Though the kidneys and urinary tract are separate
systems, they are interrelated; irregularities in one
system may affect the other. Abnormalities may be
anatomic, infectious, cellular, inflammatory, funcON
tional, hormonal, or maturational in nature.
on obstruction (UPJ) is the most
_ RENIAL DYSPLAS]A __
ephrosis in childhood. Pos-

n the hepatic bile


periportal fibrosis.
ing adulthood. The a
polycystic kidney di
hood, unless the fam
tigation. The cysts
and, eventually, ren

URETEROPELVIC JUNCTI
OBSTRUCTION
Ureteropelvic juncti
common cause of hydron
sible causes include

intrinsic fibrosis at the junction


A multicystic kidney, the most common renal dysand ureter, kinking of the ureter,
plasia, consists of numerous noncommunicating,
vessel.
fluid-filled cysts. Affected kidneys are nonfunctional,
but the condition is virtually always unilateral.
Multicystic kidney is one of the two most common
ons
causes of renal masses in the newborn (the other is
hydronephrosis resulting from ureteropelvic junction
s to elevated intrapelvic pres-

of the renal pelvis


or a crossing renal

Clinical Manifestati
The obstruction lead
sure, dilation of t

he renal pelvis and calyces, urinary


obstruction). The diagnosis is confirmed by ultrahematuria, pain, and gradual destrucsound. Most cases undergo spontaneous involution
arenchyma. Occasionally, the conbut some require nephrectomy. Long-term complion prenatal ultrasound; in the
cations, such as hypertension and malignancy, are
enous pyelography and ultrasound
rare.
ostic tests for UPJ.
Polycystic kidney disease is an inherited disorder
that occurs in two forms: the infantile autosomal
recessive type and the adult autosomal dominant
type. In the former, the kidneys appear grossly
normal but the renal collecting tubules are dilated,
by pyeloplasty provides an alterproducing small cysts. Unaffected segments are internsport from the ureter to the
spersed, but in general the kidneys function poorlypelvis.

stasis, infection,
tion of the renal p
dition is detected
infant, both intrav
are sensitive diagn

Treatment
Surgical correction
native route of tra
.

--------------------------------------- 111
Chapter 14 /
Nephrology and Urology
199
i VESJCOyRETERAL REFLUX
on, bladder neck hypertrophy,

causes urethral dilati


mucosal trabeculatio

n, and, not infrequently, vesiVesicoureteral reflux results from incompetence of a


d renal dysgenesis.
valve that normally only allows one-way urine flow
from the ureters to the bladder. In children, the conons
dition is usually bilateral and occurs as a consequence
of insufficient tunneling of the ureters into the subsuspected by detecting
mucosal bladder tissue.
enatal ultrasound or by palpat-

coureteral reflux an
Clinical Manifestati
The disorder may be
hydronephrosis on pr
ing a distended blad

der or renal mass during the


Clinical Manifestations
In older infants, parents may

newborn examination.
note a weak or dribb

ling urinary stream. OccasionThe most frequent presentation is recurrent urinary


is diagnosed in young children
tract infections (UTIs). A voiding cystourethrogram
aluation following a UTI.
(VCUG) detects abnormalities at ureteral insertion
sites and allows classification of the grade of reflux
based on the extent of reflux and associated dilatation of the ureter and pelvis (Figure 14-1). Low

ally, the condition


during radiologic ev

Treatment
Transurethral ablati

on of the obstructing tissue is the


grades of reflux often resolve spontaneously; high
In neonates who are too small
grades produce large, tortuous ureters and gross pyeemporary supravesical diversion
localiectasis and lead to progressive renal injury and
ablation can be performed.
scarring.
the degree of renal and bladder

treatment of choice.
for the procedure, t
is appropriate until
Prognosis depends on
impairment at the ti

me of repair.
Treatment
First-line therapy involves antibiotic prophylaxis
with amoxicillin or nitrofurantoin. In recalcitrant
cases, ureteral reimplantation is performed; the
ureters are surgically tunneled through larger segt common congenital anomaly
ments of submucosa at a more advantageous angle.
in 1 per 500 newborns. Incom-

__HY_POS_PA_DIAS
Hypospadias, the mos
of the penis, occurs
plete development of

the distal urethra leads to malposition of the uret


hral meatus along the ventral
_Pp_STE_RIOR_LI[RETHRAJL VALVES
rotum, or perineum. Proximal

shaft of the penis, sc


hypospadias may caus

e curving of the penis


Occurring only in males, posterior urethral valves
ed anomalies include hernias and
consist of posteriorly situated leaflets within the
Circumcision is contraindicated
prostatic urethra, which result in partial bladder
air requires preputial tissue. The
outlet obstruction. The increased pressure upstream
to extend the urethral meatus to

(chordee). Associat
undescended testes.
because surgical rep
aims of therapy are
the tip of the glans

penis and produce the appearance of a normal ci


rcumcised phallus. Prognosis is
excellent for dista
l lesions; proximal lesions may
require multiple re
visions before an acceptable result
is achieved.
I CRYPTORCHIDISM
Cryptorchidism is d
efined as testes that have not
fully descended int
o the scrotum and, unlike
Grade Grade Grade Grade Grade retracted testes, cannot be manipulated into the
12 3 4 !)
scrotum with gentle pressure. Testes that
remain
Classification of vesicoureteral reflux. Severity outside the scrotu
Figure 14-1
m develop ultrastructural changes,
based on level of reflux and degree of collecting system have impaired sperm pro
duction, and have an
dilatation. increased risk of malignancy. Bilateral cryptorchidism

--------------------------------------- 112
Blueprints Pediatrics
gJH^^USwHJ Primary survey
The Differential Diagnosis for Children with
Airway
Cardiopulmonary Arrest
Breathing
Respiratory Metabolic
Upper airway obstruction Diabetic ketoacidosis
Circulation
Lower airway obstruction Addison's disease
Restrictive lung disease Hyperthyroidism
Disability
Insufficient gas transfer Hypoglycemia
Exposure

Cardiac Hyperkalemia
1
Congenital heart disease Hypocalcemia
1
Primary dysrhythmia Hyponatremia
I
Myocarditis Multisystem
fe-threatening condition?

Li

V
Pericarditis Sudden infant death
^ No
Cardiac tamponade syndrome
Congestive heart failure Drug intoxication*

/
// \L

Central Nervous System Multiple trauma


Meningitis Anaphylaxis

^_
-^
Resuscitation

Secondary survey
Encephalitis Hypothermia
Acute hydrocephalus Se
Ptic shock
Complete examination

Airway maneuvers
Supplemental 02 and

in a head-to-toe fashion
Head trauma Renal
t
History of illness or
Seizure Acute and chronic renal
trauma event
Tumor failure
Past medical history
Hypoxic-ischemic injury
igns,
Laboratory results and

ventilatory suppor
< Venous access and
shock management
( Monitoring vital s
EGG, pulse oximetr

y,

radiographic tests
Gastrointestinal

Abdominal trauma
r bag
Re-evaluation and

urinary catheter o
*0ro-gastric tube

stabilization
Bowel perforation or
Problem list
obstruction
^.
Peritonitis
Dehydration
r
* Narcotics, tricyclic antidepressants, barbiturates, benzodiazepines.
Definitive care

Figure 1-1

Algorithm

of the initial assessment of the pediatric


or child is done with premedication in the following

patient.

rapid-sequence fashion:
s DG, Vaster M, Lappe DG, et al. Golden Hour: The

Modified from Nichol


Handbook of Advanced

Pediatric Life Support. St. Louis: Mosby Yearbook,


1991:2,128.
1. Preoxygenation with 100% oxygen.
2. Administration of a vagolytic drug (e.g., atropine).
3. Administration of a sedative, hypnotic, and/or
opioid drug (e.g., thiopental, versed, fentanyl).
uld be applied, and the patient
4. Application of cricoid pressure.
Rarely, a patient cannot be
5. Administration of a paralyzing dose of a neuroted with a bag and mask, and an
muscular blocking agent (e.g., pancuronium or
cothyrotomy is required to
vecuronium, a nondepolarizing agent; or succinylcholine, a depolarizing agent). If succinylcholine
e assessed by evaluating pulses
is used, a defasciculating dose of a neuromusral), capillary refill, and blood
cular blocking agent should be given before
e of a pulse in the large arteries
administration of succinylcholine (e.g., pancurotient who is not breathing
nium or vecuronium).
rest. In children, heart rate is the

Cricoid pressure sho


should be intubated.
intubated or ventila
emergency needle cri
establish an airway.
Circulation may b
(central and periphe
pressure. The absenc
of an unconscious pa
defines a cardiac ar
most sensitive measu

re of intravascular volume
In the hypotensive, hemodynamically unstable, or
fill is the most sensitive measure
unconscious patient, premedication is not indicated.
ion. Blood pressure fluctuations

status. Capillary re
of adequate circulat

--------------------------------------- 113
20

Blueprints Pediatrics
In this defect, t

he pulmonary and systemic circuits


are parallel rather
than in series. The systemic circuit
(deoxygenated blood)
is recirculated through the
body, whereas the pu
lmonary circuit (oxygenated
98%

blood) recirculates

through the lungs. A mixing


lesion such as an at

rial septal defect, ventricular


septal defect, and/o
r patent ductus arteriosus that
allows mixing of the
systemic and pulmonary circulations is necessary
for survival.
Clinical Manifestati
ons
Cyanosis is present
from birth, the degree varying
with the associated
mixing lesions. In the absence of
mixing lesions, ther
e is pronounced cyanosis, right
ventricular heave, a
nd a single loud S

2 on examination. The presence o

f a systolic murmur indicates the


presence of a VSD or
pulmonic stenosis. The ECG is
normal in the newbor
n; however, right-axis deviation
and right ventricula
r hypertrophy are eventually
seen. The chest radi
ograph reveals increased pulmonary vascular mark
ings in D-transposition with or
Transposition of the great arteries with an intactwithout ventricular
Figure 3-2
septal defect, but if pulmonic
ventricular septum, a large patent ductus arteriosus (on PGE,)stenosis is critic
al, decreased pulmonary vascular
and atrial septal defect (status post balloon atrial septostomy).markings may be
present. Cardiomegaly with "eggNote the following: (a) The aorta arises from the anatomic rightshaped silhouett
e" is often seen on chest radiograph.
ventricle, and the pulmonary artery from the anatomic left ventricle; (b) "transposition physiology," with a higher oxygen saturation in the pulmonary artery than in the aorta; (c) "mixing"Treatment
between the parallel circulations (see text) at the atrial Initial management m
ay include PGE] to keep the
(after balloon atrial septostomy) and ductal levels; (d) shuntingpatent ductus a
rteriosus open and increase aorta
from the left atrium to the right atrium via the atrial septal
defect (not shown) with equalization of atrial pressures; (e)(deoxygenated) to p
ulmonary artery (oxygenated)
shunting. If needed,
the Rashkind balloon atrial sepshunting from the aorta to the pulmonary artery via the ductustostomy can be uti
lized to improve atrial mixing and
arteriosus; (f) pulmonary hypertension due to a large ductus
arteriosus.
relieve severe hypox
ia. Surgical repair, utilizing the
Cloherty JP, Stark AR. Manual of Neonatal Care, 4th ed. Philadelphia:arterial sw

itch procedure, is generally performed


Lippincott-Raven, 1998:426.
k of life.

during the first wee


Total Anomalous Pulm

onary
D-Transposition of the Great Arteries
D-Transposition of the great arteries (Figure 3-2)
onary venous connection
accounts for 5% of congenital heart defects and is the
is a rare lesion in which the
most common form of cyanotic congenital heart
urn is directed to the right
disease presenting in the neonatal period. In this
ly or indirectly through venous
defect, the aorta arises anteriorly from the right venfour variants:
tricle, and the pulmonary artery rises posteriorly
from the left ventricle. There are three basic variants:
cases): Blood drains via a
D-TGA with intact ventricular septum (60%), Do the innominate vein or into the
TGA with ventricular septal defect (20%), and Da
TGA with ventricular septal defect and pulmonic
s): Blood drains into the corostenosis (20%).
ectly into the right atrium

Venous Connection
Total anomalous pulm
(TAPVC) (Figure 3-3)
pulmonary venous ret
atrium either direct
channels. There are

Supracardiac (50% of
vertical vein int
superior vena cav
Cardiac (20% of case
nary sinus or dir

--------------------------------------- 114
200 Blueprints Pediatrics
results in oligospermia and infertility. Term infants
stis is fixed to the posterior
have a 3% to 4% incidence at birth; the rate is much
ing surgery to avoid subsequent
higher (30%) in premature infants.
the testicular or epididymal ap-

The contralateral te
scrotal envelope dur
torsion. Torsion of
pendix resolves spon

taneously.
Clinical Manifestations
_ HYpROCELES ANP_YARJ
CQC_E_LE_S
One or both testes may be positioned in the
abdomen or anywhere along the inguinal canal. Most
are palpable on examination. Ninety percent of
-filled sacs in the scrotal cavity
patients will also have inguinal hernias.
ts of the processus vaginalis.

Hydroceles are fluid


consisting of remnan
Hydroceles that comm

unicate with the peritoneal


cavity may develop i
nto hernias when bowel
Treatment
ath into the scrotum. Commu-

descends along the p


nicating hydroceles

and scrotal hernias should be

By 12 months of age, all but 0.08% of males have


possible to prevent the developbilateral descended testicles. Spontaneous descent
ted hernia. Most noncommuniafter 12 months is unlikely. Surgical repair
(orchiopexy) takes place at 12 to 18 months of age
volute by 12 months of age.
and has a high success rate (99%). Orchiopexy does
efined as a dilated testicular vein
not appear to alter the incidence of malignant degenform plexus resulting from the
eration, but it does render the testis accessible for
s valves responsible for advanc-

repaired as soon as
ment of an incarcera
cating hydroceles in
A varicocele is d
and enlarged pampini
absence of the venou
ing the blood toward

the heart. They become


regular self-examination.
uring adolescence and occur

detectable in boys d
more commonly on the

left. Indications for surgical


repair include sever
e pain, interference with tesTORSION
tion, and ipsilateral testicular
Torsion is a surgical emergency, requiring prompt
varicoceles place the patient at
recognition and correction to prevent loss of the tesinfertility.
ticle. Most patients with testicular torsion lack the
posterior attachment to the tunica vaginalis that keeps
TIpNS
the testis from rotating around the spermatic cord.
Clinical Manifestations
frequent cause of pediatric morClinical manifestations include the acute onset of
y be limited to the bladder (cys-

ticular hormone func


atrophy. Unrepaired
an increased risk of
_ URlNARY TRACT INFEC
Pathogenesis
Bacterial UTIs are a
bidity. Infection ma
titis) or may also i

nvolve the kidney (pyelonephritis).


unilateral scrotal pain; nausea; vomiting; a swollen,
TIs result from ascent of exterior
exquisitely tender testis; scrotal edema; and absent
urinary tract. In children younger
cremasteric reflex. Epididymitis, which is more
ogenous seeding of the kidneys
common during puberty and adolescence, presents
more common.
with a similar clinical picture. Doppler ultrasound is
only marginally helpful in differentiating between
the two conditions and may delay appropriate treatment. Occasionally, the torsion is limited to the testicular or epididymal appendix; localized tenderness
10-fold risk over boys. Although

In older children, U
fecal flora into the
than 2 months, hemat
during bacteremia is

Epidemiology
Girls have almost a
uncircumcised male n

eonates are more prone to


and the "blue dot" sign (on the upper aspect of the
ility alone is not a sufficient indiscrotum) suggest limited involvement.
routine circumcision.
Treatment
Early surgical intervention is critical; 90% of gonads

UTIs, this susceptib


cation for universal
Risk Factors
The most significant

risk factor is the presence of a


survive when detorsion and fixation take place within
ality that causes urinary stasis,
6 hours of onset. Necrotic testes must be removed.
ux.

urinary tract abnorm


obstruction, or refl

--------------------------------------- 115
Chapter 14 /
Nephrology and Urology
201
Clinical Manifestations
re should prompt a 10- to 14-day

Positive urine cultu


course with an appro

priate oral antibiotic. Patients


History and Physical Examination
ing or patients with vomiting
In older children, the signs and symptoms of cystitis
antibiotics must be admitted to
are similar to those in adults and include low-

who are toxic appear


who cannot take oral
the hospital for int

ravenous antibiotics and observagrade fever, frequency, urgency, dysuria, inconnt, these patients may be distinence, abdominal pain, and hematuria. In contrast,
ppropriate oral antibiotic to
pyelonephritis presents with high fever, chills,
nausea, vomiting, and flank pain. Infants warrant
patients with isolated cases of
special attention because a UTI can be the first clin-

tion. With improveme


charged home on an a
finish the course.
The prognosis for
cystitis is excellen

t; morbidity increases with recurical manifestation of an obstructive anomaly or


UTI-related complications are
severe vesicoureteral reflux. Also, infants with a UTI
s, including perinephric abscess,
may have a febrile illness and no localizing signs; irrirenal failure.
tability, lethargy, weight loss, and vomiting are commonly reported.

rent infection. Most


due to pyelonephriti
renal scarring, and

K
EY POINTS
Differential Diagnosis
1. In older childr
en, urinary tract infections result
The differential diagnosis includes external genital
ion of the urinary tract with exteirritation, vaginosis, and pinworm infestation. Adea. Hematogenous seeding is more
novirus can cause a self-limited hemorrhagic cystitis
ldren younger than 2 months.
that does not respond to antibiotics but may be misr than 2 years with a UTI should

from contaminat
rior fecal flor
probable in chi
2. Children younge
undergo radiolo

gic imaging to detect anatomic


taken for a UTI. Lower lobe pneumonia often presents with fever, chills, and flank pain.
causes renal scarring and, with

abnormalities.
3. Pyelonephritis

repeated infect
ions, hypertension or end-stage
renal disease.
Diagnostic Evaluation
Although pyuria, hematuria, and bacteriuria on
urinalysis suggest a UTI, a positive urine culture is
the gold standard for diagnosis. Susceptibility testing
should be performed on the bacterial isolate to
E _
ensure appropriate antibiotic treatment. Current
guidelines recommend that all children under the age
of 24 months undergo renal ultrasound to rule out
s a noninflammatory disorder
vesicoureteral reflux or structural lesions that predispose to infection. Those who do not respond to
on characterized by extreme
appropriate antibiotic therapy within 48 hours
uminemia, hyperlipidemia, and
should also receive a VCUG or radionuclide cystography. In prompt responders, the VCUG or radionuclide cystography is optional.

*..NEPHRpTJC SYNDROM
Pathogenesis
Nephrotic syndrome i
of glomerular functi
proteinuria, hypoalb
edema.
Epidemiology
Nephrotic syndrome m

ay be idiopathic (90%) or
secondary in nature
(Table 14-1). Minimal change
Treatment
disease (MCD) is by
far the most common cause of
Children with cystitis may be treated with an apprimary nephrotic sy
ndrome in the pediatric popupropriate oral antibiotic such as amoxicillin or
lation. Most patient
s present between the ages of 2
co-trimoxazole (Bactrim). Non-toxic-appearing chiland 6 years, and boy
s outnumber girls. Focal segdren with suspected pyelonephritis should be treated
mental glomeruloscle
rosis and diffuse mesangial
with cefixime (orally) or ampicillin plus gentamicin
proliferative glomer
ulonephritis account for the
or cefotaxirne until culture results are available. If the remainder of idiopat
hic cases of nephrotic syndrome
culture is negative, antibiotics may be discontinued.
in children.

--------------------------------------- 116
202

rints Pediatrics
TABLE 14-1

Conditions Associated with Secondary Nephrotic


ent with Glomerulonephritic
Syndrome
romes
Postinfectious glomerulonephritis
s
Nephrotic Syndromes
Acute viral illnesses
Minimal change disease
Hemolytic-uremic syndrome
Congestive heart failure

TABLE 14-2
Diseases That Pres
and Nephrotic Synd
Nephritic Syndrome
IgA nephropathy

Acute poststreptoc

occal
Focal segmental
Constrictive pericarditis
glomerulosclerosis
Bacterial endocarditis
Membranoproliferative
Alport's syndrome
urpura
glomerulonephritis
Renal vein thrombosis
e

Systemic lupus erythematosus


Membranous
Medications
glomerulonephritis
Malignant hypertension

Systemic lupus
erythematosus
Henoch-Schonlein p

Rapidly progressiv

Preeclampsia
cell foot processes
demonstrated by electron microsClinical Manifestations
l glomerulosclerosis is charac-

copy. Focal segmenta


terized by focal sec

tions of distorted glomeruli, with


History and Physical Examination
y and segmental capillary
Patients with early nephrotic syndrome appear quite
creased mesangial cellularity and
well. Periorbital edema is commonly the first abnormembrane thickening are
mality noted. This is followed by lower extremity and
angial proliferative glomeruthen generalized edema and ascites. Anorexia and

mesangial hypertroph
loop destruction. In
glomerular basement
found in diffuse mes
lonephritis.

diarrhea are variably present.


Treatment
Differential Diagnosis
If the clinical pres
entation is consistent with uncomEdema may be renal, hepatic, or cardiac in origin.
hrotic syndrome, strict dietary
Other conditions associated with proteinuria include
oral steroid therapy are appropriexercise, trauma, UTI, dehydration, and acute tubular
not resolve within 8 to 12 weeks
necrosis; however, none of these causes the degree oof
xperiences frequent or severe
protein loss seen in nephrotic syndrome. Of note,
sy is indicated to confirm the
glomerular filtration rate (GFR) and blood pressure
are less likely to be affected in nephrotic syndrome
n prompt remission in most cases
than in the nephritic syndromes (Table 14-2).
ndrome that does not respond

plicated primary nep


salt restriction and
ate. If symptoms do
r if the patient e
relapses, renal biop
diagnosis.
Steroids result i
of MCD. Nephrotic sy
to oral steroids may

require treatment with immune


Diagnostic Evaluation

suppressants such as

cyclophosphamide. Intravenous
albumin followed by
a diuretic such as furosemide
The hallmark of nephrotic syndrome is severe proporary measure to induce diureteinuria. Affected individuals lose more than 40mg
of incapacitating anasarca or
protein/m 2/hr in their urine when averaged over a
atory compromise.
24-hour period, a large proportion of which is
ons, particularly spontaneous
albumin. Because the liver rapidly manufactures
most frequent complications
replacement proteins, large amounts of lipids are
e and usually occur while the
created as well.
suppressant therapy. The progRenal biopsy is indicated for patients outside the
llent; although up to 80% of
typical age range for MCD and those who do not
least once, very few develop any
respond to steroids. True to the disease's name, gross
insufficiency. Unfortunately,
sections in MCD show few if any abnormalities; the
segmental glomerulosclerosis and
only consistent finding is effacement of epithelial
roliferative glomerulonephritis do

can be used as a tem


sis in the presence
edema-related respir
Bacterial infecti
peritonitis, are the
of nephrotic syndrom
patient is on immuno
nosis of MCD is exce
patients relapse at
long-standing renal
patients with focal
diffuse mesangial p

--------------------------------------- 117
Chapter 14 /
203
Nephrology and Urology
not respond well to steroid therapy, and end-stage
ve glomerulonephritis is the
renal disease is common. Renal transplant is not a
a number of glomerulopathies
cure, because both diseases recur in the transplanted
asons, deteriorate over a few
kidney.
enal failure, uremia, enceph-

Rapidly progressi
description given to
that, for unknown re
weeks or months to r
alopathy, and even d

eath. All forms demonstrate


generalized crescent
formation in the glomeruli,
thought to represent
cellular destruction by
1. Nephrotic syndrome is characterized by proteinsequent necrosis and fibrin

macrophages with sub


deposition. Fortunat

ely, rapidly progressive glomeruuria, hypoalbuminemia, hyperlipidemia, and


in children.
edema.
2. Minimal change disease is the most common type
of pediatric idiopathic nephrotic syndrome.
hritides
3. Most cases respond to oral steroid therapy; renal
e thought to be a benign con-

lonephritis is rare

Chronic Glomerulonep
IgA nephropathy, onc

biopsy is recommended for those that do not


dition, is now known
to slowly progress to renal
failure in 25% of ca
ses. C3 levels are normal. Renal
biopsy alone makes t
he diagnosis, demonstrating
mesangial deposits o
f IgA in the glomeruli. GlomeruGLOMERULONEPHRITIS
ed with systemic lupus erythe-

lonephritis associat
matosus is discussed

in Chapter 11.
The term glomerulonephritis implies inflammation
of the glomerular basement membrane. Antigenantibody complexes are formed or deposited in the
ephritides
subepithelial or subendothelial areas; immune medir hereditary nephritis, is caused
ators follow, resulting in inflammatory injury. Hemagene encoding type IV collagen
turia, overt or microscopic, is the hallmark of the
normal glomerular basement
disease. Distinguishing characteristics of the major
e is X-linked, although defecglomerulonephritic syndromes of childhood are disother glomerular basement
cussed next.
can cause similar disease.

Inherited Glomerulon
Alport's syndrome, o
by mutations in the
that result in an ab
membrane. Inheritanc
tive genes encoding
membrane components
Because type IV coll

agen is important in the cochlea,


Alport's syndrome is
associated with sensorineural
Acute Glomerulonephritides
Acute poststreptococcal glomerulonephritis
hematuria is a common cause of
(APGN), the most common glomerulonephritis in
copic and occasionally gross
childhood, occurs sporadically in older children and
ction is normal, and biopsy,
is twice as common in males. Streptococcal infections
reveals thinning of the glomeruinvolving either the throat or the skin (impetigo]
ne. Because transmission is
precede the clinical syndrome by 1 to 3 weeks. Treatasymptomatic microscopic
ing the Streptococcal infection does not prevent
y found in other family members.
APGN. Elevated antistreptolysin-O or anti-DNAase
B titers suggest recent Streptococcal infection. The

hearing loss.
Benign familial
asymptomatic micros
hematuria. Renal fun
though unnecessary,
lar basement membra
autosomal dominant,
hematuria is usuall

Differential Diagno
sis
C3 component of the complement pathway is low.
Renal histology reveals mesangial and capillary cell
agnosis of hematuria, the most
proliferation, inflammatory cell infiltration, and
tion of glomerulonephritis,
granular "humps" of IgG and C3 below the glomerul conditions (infection, trauma,
lar basement membrane.

The differential di
prominent manifesta
includes other rena
malignancy, stones,

cystic disease) and hematologic


Henoch-Schonlein purpura (HSP), a systemic vasbleeding produces false-positive
culitis characterized by purpura, crampy abdominal
imen is collected incorrectly. Both
pain, and arthritis, may progress to a glomerulobin test positive for blood on
lonephritis-type syndrome that is indistinguishable
ever, there are no red blood cells
from IgA nephropathy. Two percent of children with
e examination in the presence of
HSP develop long-term renal impairment.

disorders. Vaginal
results if the spec
hemoglobin and myog
urine dipstick; how
on microscopic urin
only myoglobin.

--------------------------------------- 118
204
Blueprints Pediatrics
Clinical Manifestations
ACI DOS IS _
The initial presentation of glomerulonephritis inubular acidosis (RTA) are characcludes hematuria, azotemia, oliguria, malaise, abremic metabolic acidosis resultdominal pain, edema, and hypertension. Red cell
t renal transport of bicarbonate
casts are invariably present; in fact, the urine is often
described as "tea-colored" by parents. Proteinuria
n tubules are the site of reaboccurs as well but is less prominent than in nephrotic
on. Most bicarbonate filtered
syndrome. The GFR is compromised, leading to
eabsorbed in the proximal
salt and water retention and circulatory overload.
mino acids, glucose, sodium,
Azotemia is marked by increasing serum blood urea
phosphate, and water. In the

RE NAL TU BU L A R

All forms of renal t


terized by hyperchlo
ing from insufficien
or acids. The nephro
sorption and secreti
from the plasma is r
tubule, along with a
potassium, calcium,
distal tubule, the r

emainder of the bicarbonate is


nitrogen and creatinine levels. Sodium and potassium
gen ions are secreted into the
regulation may be temporarily disrupted. Important
itubular capillaries. Defects in
laboratory studies include urinalysis, urine culture,
e compromise the kidney's ability
hemoglobin and platelet counts, coagulation studies,
serum electrolytes, blood urea nitrogen and creatistasis.
nine, streptococcal antibody titers, and complement
levels.

reabsorbed and hydro


tubules from the per
either transport sit
to maintain pH homeo

Differential Diagnos
is
In proximal RTA (typ
e 2), the proximal tubule fails
Treatment
ate from the ultrafiltrate. Distal
Positive streptococcal cultures are treated with
either deficient hydrogen
appropriate antibiotic therapy. Hypertension, when
iltrate (type 1} or impaired

to reabsorb bicarbon
RTA may result from
secretion into the f

present, can be severe, requiring vasodilators, diuretn the face of hyperkalemia


ics, and fluid restriction. Steroids may improve the
sm or pseudohypoaldosteoutcome of rapidly progressive glomerulonephritis.
tal RTA type 4 is the most
Although the clinical manifestations of APGN may
hildren and adults. Most types

ammonia production i
from hypoaldosteroni
ronism (type 4). Dis
common RTA in both c
of RTA can be either

hereditary or sporadic, acute or


take a few months to resolve, the overall prognosis for
lone or as part of a disease
return to normal function is excellent. Patients with
, most patients exhibit proxiother types of glomerulonephritis fare less well.
Virtually all males and 20% of females with Alport's
njunction with Fanconi's synsyndrome progress to end-stage renal disease by
disorder of proximal tubule
middle adulthood. The course of rapidly progressive
in excessive urinary losses of
glomerulonephritis is particularly devastating, with
cids, small proteins, glucose,
most patients becoming dialysis-dependent within a
ter.
few years. Most syndromes eventually recur in a transplanted kidney (APGN is a notable exception).
ons

chronic, occurring a
complex. For example
mal RTA type 2 in co
drome, a generalized
transport resulting
bicarbonate, amino a
electrolytes, and wa
Clinical Manifestati
History and Physical

Examination
KEY POINTS
t proximal RTA type 2 as part
Glomerulonephritic syndromes are inflammatory
e present with failure to thrive;
and characterized by hematuria, azotemia, olisymptoms include chronic
guria, edema, and hypertension.
a, vomiting, anorexia, polydipsia
Specific syndromes include acute poststreptocontraction, and impaired
coccal glomerulonephritis, IgA nephropathy,
(rickets].
hereditary nephritis, rapidly progressive
also presents with metabolic
glomerulonephritis, and systemic lupus
to thrive. Hypokalemia, hypererythematosus-associated glomerulonephritis.
y stones are common. In contrast,
Alport's syndrome is associated with painless
al RTA type 4 occurs in the
hematuria and sensorineural hearing loss.
emia in conjunction with priMost syndromes recur in a transplanted kidney.
poaldosteronism or end-organ

Patients who manifes


of Fanconi's syndrom
associated signs and
acidosis, hypokalemi
and polyuria, volume
vitamin D metabolism
Distal RTA type 1
acidosis and failure
calciuria, and kidne
the acidosis in dist
presence of hyperkal
mary or secondary hy
resistance.

--------------------------------------- 119

Chapter 14 /
Nephrology and Urology
Diagnostic Evaluation

205
KE

Y POINTS
Any patient with hyperchloremic metabolic acidosis
1. All classificat
ions of renal tubular acidosis are
of unclear etiology warrants further workup to rule
characterized b
y hyperchloremic metabolic
out RTA (Figure 14-2).
acidosis.
2. The most common
type in children is distal RTA
type 4, resulti
ng from hyperkalemia (from hypoalTreatment
dosteronism or
pseudohypoaldosteronism) that
Treatment consists of providing children with suffiinterferes with
ammonia production.
cient amounts of an alkalinizing agent (either bicar3. Fanconi's syndr
ome is a generalized disorder of
bonate or citrate] to completely correct the acidosis
proximal tubule
transport with excessive urinary
and restore normal growth. Thiazide diuretics are
bonate, proteins, glucose, elecadministered in proximal RTA to increase proximal

losses of bicar

trolytes, and w
ater.
tubular reabsorption of bicarbonate. Hypokalemia is
4. Alkalizing agen
ts correct the acidosis.
treated concurrently when the alkali is coupled with
potassium as a salt. Hyperkalemia is usually more difficult to correct; furosemide is prescribed unless the
defect results in salt wasting. If RTA is associated with
an underlying condition, the primary disorder must
be treated.

Hyperchloremic Metabolic Acidosis?


(low serum pH; low serum bicarbonate; normal anio
n gap)
Yes
Calculate urine anion gap
(Na+ + K+)-CII
f
Negative (Cf > Na*+K+)

Positive

(Cf < Na++K+)


I
I
No Gl tosses

Hypo-/normokalemia

Hyperkalemia
(diarrhea, etc.)
I
I
1
Urine pH > 5.5
Urine pH < 5.5
No exogenous Ch
salt added
I
I
Distal RTA (type 1)
Distal RTA (type 4)
I
Urine pH < 5.5
I
Proximal RTA (type 2)
Figure 14-2
iology.

Diagnostic workup of hyperchloremic metabolic acidosis of unknown et

--------------------------------------- 120
206

Blueprints Pediatrics

NEPHROGENIC DIABETES
o 40% of patients with

which is seen in 10% t


schizophrenia.

INSIPIpUS

Pathogenesis
Diagnostic Evaluatio
n
Diabetes insipidus (DI) involves a disorder in renal
Patients with nephro
genic DI are unable to concenconcentrating ability. Patients produce up to
trate their urine. D
espite significant dehydration,
400mL/kg/day of very dilute urine regardless of
gravity and osmolarity measurehydration status. DI may be central or nephrogenic
priately low. Figure 14-3 outin origin. In central DI, the production or release of

their urine specific


ments remain inappro

lines the evaluation


of a patient with suspected
antidiuretic hormone is insufficient (see Chapter 6).
nephrogenic DI. Peri
natal testing to detect arginine
Nephrogenic DI arises from end-organ resistance to
vasopressin receptor
gene (AVPR2) mutations is now
arginine vasopressin (antidiuretic hormone), either
available.
from a receptor defect or from medications or other
processes that interfere with aquaporin-2 protein
transport of water at the renal cortical tubules.

Treatment
Acute treatment cons

ists of rehydrating the child,


replacing ongoing ur
inary losses, and correcting
electrolyte abnormal
ities. A low-sodium diet (<0.7
Epidemiology
mEq/kg/day) should b
e coupled with thiazide
Nephrogenic DI may be hereditary or acquired and
e urinary sodium reabsorption.
usually presents within the first several years of life.

diuretics to decreas

The addition of indo


methacin or aspirin may have an
Acquired nephrogenic DI has been associated with
hiazide diuretics in reducing water
polycystic kidney disease, pyelonephritis, lithium
toxicity, and sickle cell disease.

additive effect on t
excretion.
Children with nep

hrogenic DI are at risk for poor


growth and mental re
tardation. The disease is lifelong
but carries a good p
rognosis provided that episodes
of hypernatremic deh
ydration remain limited.
Clinical Manifestations
History and Physical Examination
All patients present with polyuria and compensatory
polydipsia. Other features may include intermittent
tes insipidus suspected
fever, irritability, vomiting, and growth retardation.

Diabe

Most affected children also have a history of recurrent hypernatremic dehydration. Developmental
Elevated serum Osm E
levated serum sodium Very dilute urine
delay may occur as a result of frequent hypernaI __ |
tremic seizures. Some patients manifest no symptoms until they are stressed with illness. Others

T T
Normal blood

glucose Normal serum creatinine


remain completely unable to keep themselves in
No |
I Yes Yes I
| No
fluid balance without continual therapy.
(Diabetes mellitus
) DDAVP test dose (Renal failure)
I
Differential Diagnosis

r
1
No decrease in

Significant decrease
Differentiating central DI from nephrogenic DI is
urine output
in urine output
not possible based on symptomatology alone,
although the former more commonly follows head
trauma or meningitis. Other conditions that may
I

I
Nephrogenic DI

Central DI
present in a similar manner include diabetes

mellitus, RTA, and compulsive water drinking,


ng nephrogenic DI.

Figure 14-3

Diagnosi

--------------------------------------- 121
Chapter 14 /
Nephrology and Urology 207
KEY POINTS
osis of Hypertension
1. Diabetes insipidus is a disorder of urine concentration and can be central or nephrogenic.
2. Clinical manifestations include polyuria, polydipuff size
sia, and growth retardation.

TABLE 14-3
Differential Diagn
Pain, anxiety
Inappropriate c
Essential hyperten

sion
3. Therapy for nephrogenic D! includes a lowsodium diet, thiazide diuretics, and indomethacin
or aspirin.
tis

Renal
Glomerulonephri
Pyelonephritis
Parenchymal (i.

e., cystic) disease


Obstructive uro
pathy
Nephrotic syndr
ome
HYPERTENSION
Blood pressure rises as a child grows, reaching adult
values during adolescence. Hypertension in the pediatric population is denned as blood pressure greater
cranial pressure
than 95th percentile for age on three separate
occasions.

Renal tumor
Renal failure
Renal trauma
Neurologic
Increased intra
Hemorrhage
Brain injury
Familial dysaut

onomia
Drugs and toxins
Differential Diagnosis
Oral contracept
ives
Essential (primary) hypertension is the most
common form in adults. Children are more likely to
manifest secondary hypertension, usually related to
renal disease. Endocrine, vascular, and neurologic
conditions have also been associated with increased
nal hyperplasia
blood pressure (Table 14-3).
e

Corticosteroids
Cyclosporin
Cocaine
Endocrine
Congenital adre
Cushing syndrom
Hyperthyroidism
Pheochromocytom

a
Clinical Manifestations
dism
History

Hyperparathyroi
Hyperaldosteron

ism
Stable or slowly progressive hypertension is unlikely
to cause symptoms. Family history is often positive
the aorta
for hypertension, stroke, or premature heart attack.
mbosis
Patients with secondary hypertension often come to
enosis
medical attention for complaints related to their
nous fistula

SIADH
Vascular
Coarctation of
Renal vein thro
Renal artery st
Large arteriove
Infective endoc

arditis
underlying disease (e.g., growth failure, edema). Past
medical history, state of health, recent medications,
and review of systems for urinary tract symptoms
provide pertinent information.
irway obstruction

Vasculitis
Other
Chronic upper a

Severe hypertension or hypertension that has

Preeclampsia
Neurofibromatos

is
developed over a short period of time can cause
headache, dizziness, and vision changes. Hypertensive encephalopathy is characterized by vomiting,
thermia

Hypercalcemia
Malignant hyper

ataxia, mental status changes, and seizures.


ent porphyria
Physical Examination
f inappropriate antidiuretic hormone secretion.
The most important part of the examination is
obtaining an accurate blood pressure reading. The air
bladder portion of the cuff should encircle the

Hypernatremia
Acute intermitt
SIADH, syndrome o

--------------------------------------- 122
Blueprints Pediatrics
208
patient's arm and be wide enough to cover 75% of
EY POINTS
the upper limb. A cuff that is too small will give a
falsely elevated reading. At least once, the blood
norms are related to age and
pressure should be taken in all four extremities
to exclude aortic coarctation. Particular attention
ssure readings on separate occashould be given to the heart sounds and peripheral
greater than the 95th percentile for
pulses. Poor growth, flank pain, a retroperitoneal
constitute hypertension.
mass, large bladder, or abdominal bruit suggest a
ertension range in severity
renal or renal vascular etiology. Obesity contributes
solute value and rapidity of
to hypertension in a genetically predisposed
patient.
ypertension should have screening

K
1. Blood pressure
gender.
2. Three blood pre
sions that are
age and gender
3. Symptoms of hyp
depending on ab
onset.
4. Children with h
tests to evalua

te renal and cardiac function.

5. The first line


of therapy is diet control, weight loss,
Diagnostic Evaluation
The initial laboratory evaluation should include a
blood pressure, even if maintained
complete blood count, serum electrolytes, blood urea
ange, may compromise cerebral
nitrogen, creatinine, renin level, and urinalysis.
patient with a history of sustained
Doppler ultrasound of the kidneys allows assessment
sure.
of anatomy as well as renal vasculature. Chest radiograph, electrocardiogram, and echocardiogram evaluate heart size and function, whether cardiac deficits
are the cause or the effect of the hypertension.

and exercise.
6. Rapid drops in
in the normal r
perfusion in a
high blood pres

ACUTE RENAL FAI


LURE
Treatment
Renal failure is an
uncommon but potentially lifeThe best treatment of essential hypertension is pren in children. Acute renal failure
ventive health care. High-salt diet, sedentary lifestyle,
abrupt reduction in renal funccigarette use, alcohol abuse, high serum cholesterol
several hours to days, with retenlevels, and obesity compound the disorder and
waste products (azotemia) and
increase the morbidity and mortality. Secondary
e imbalances.
hypertension responds to treatment of the underlying disorder when possible.
Pharmacologic therapy is indicated in patients
is
with persistent or refractory hypertension. Diuretics
may be prerenal, intrarenal/
and beta-blockers are used in younger children;
nal (Table 14-4). Prerenal failure
calcium channel blockers and angiotensin-converting
orm of ARF in children and
enzyme inhibitors are second-line treatment in
l kidney experiences significant
this age group but are effective first-line agents
h the reduction of plasma
in adolescents and adults because of fewer side
or hypoxia. The decreasing
effects.
a (urine output < 400
In patients with severe hypertension, rapid
Most patients completely
decreases in blood pressure compromise organ perl failure unless it is unrecogfusion. Hypertensive crisis is treated with intravenous
tely treated.
nitroprusside or labetalol, which blocks both a, and
insic renal failure results from an
P receptors. Hydralazine and diazoxide are also effecidney itself, such as glomerutive; however, they are reserved for unresponsive
itial nephritis, renal vasculitis, or

threatening conditio
(ARF) consists of an
tion, occurring over
tion of nitrogenous
fluid and electrolyt

Differential Diagnos
The mechanism of ARF
intrinsic, or postre
is the most common f
results when a norma
hypoperfusion throug
volume, hypotension,
GFR produces oliguri
mL/mVday) or anuria.
recover from prerena
nized or inappropria
By contrast, intr
abnormality of the k
lonephritis, interst

hypertension because of their tendency to drop the


is, a poorly understood condition
blood pressure too quickly.
ules become obstructed with
Stroke, heart attack, and renal disease are the most
rarenal conditions usually present
devastating complications of hypertension. Prognosis
ria, although the urine output
depends on the underlying disorder and degree of
iguric renal failure). In
control.
bstructive lesions at or below the

acute tubular necros


in which damaged tub
cellular debris. Int
with oliguria or anu
may be normal (nonol
postrenal failure, o

--------------------------------------- 123
Chapter 14 /
Nephrology and Urology
TABLE 14-4

209

Conditions Associated with Acute


n Prerenal versus Intrinsic

TABLE 14-5
Renal Failure

Typical Findings i
Acute Renal Failur

e
Prerenal

Renal

Postrenal
Diagnostic Index P

rerenal Intrinsic
Hypovolemia
Glomerulonephritis

Obstructive
uropathy

on of sodium <1 >1


Hypotension
Henoch-Schonlein
(PNaXUQ)]XlOO
purpura
o plasma >40 <20
Hypoxia
Renal vein
o
thrombosis
n to plasma >8 <3
Pyelonephritis
atio
Acute tubular
mOsmol/kg H 2O) >500 <350
necrosis

Vesicoureteral
reflux
Nephrolithiasis

Acute interstitial
lasma osmolality >1.5 <1.5
nephritis
vity >1.020 <1.020

Fractional excreti
(%)=[(UNaXPCr)/
Urine creatinine t
creatinine rati
Urine urea nitroge
urea nitrogen r
Urine osmolality (

Urine osmolality/P
Urine specific gra
Plasma urea nitrog

en/Plasma >20 <15


creatinine
PC,, plasma creati
nine concentration; PNa, plasma sodium concentration;
U Cr, urine creati
nine concentration; UNa, urine sodium concentration.
sure and result in a rapidly declining GFR and

hydronephrosis. The lesions may be congenital or


acquired, structural or functional. Patients with
proteinuria, leukocy
tes, and casts also provides useful
complete obstruction will be anuric. Partial obstrucnd plasma urea nitrogen, creatitions may present with normal or increased urine

information. Urine a

nine, osmolarity, an
d sodium can be used to difoutput.
rerenal and intrinsic failure

ferentiate between p
(Table 14-5].
Renal ultrasonogr

aphy is the single best noninvaClinical Manifestations


st for determining the site of

sive radiographic te
obstruction in postr

enal failure, as well as kidney


History and Physical Examination
size and shape and r
enal blood flow. Renal nuclear
A history of recent dehydration, shock, cardiac
l perfusion and functional
surgery, receipt of nephrotoxic medications, strepto-

scans delineate rena

differences. Intrave
nous pyelography, voiding cyscoccal infection, or posterior urethral valves may help
omputed tomography may also
clarify the etiology. Growth failure, bony abnormali-

tourethrogram, and c

be helpful. Renal bi
opsy is indicated when the diagties, anemia, deafness, and previous renal conditions
r or the extent of involvement
suggests acute deterioration superimposed on chronic

nosis remains unclea

is unknown.
renal failure. On physical examination, assess for
dehydration, cardiovascular stability, abdominal tenderness, and abdominal or suprapubic masses. Edema,

Treatment

oliguria, and hypertension are usually evident. FindTreatment consists o


f appropriate fluid management,
ings of congestive heart failure (hepatomegaly, diffuse
crackles on lung examination) require immediate
olyte abnormalities and pH,

correction of electr
protein restriction,

and, occasionally, short-term


intervention.
hemodialysis. The un
derlying abnormality must be
corrected to achieve
total resolution and prevent
recurrence. The prog
nosis of ARF depends on the
Diagnostic Evaluation
length of impairment, and severARF is characterized by hyperkalemia, azotemia, and
sturbance.
metabolic acidosis. Increased blood urea nitrogen and
undergo renal clearance may
creatinine levels signal diminished renal function.
tments in acute or chronic renal
Anemia is variably present. Urinalysis for hematuria,
icity.

underlying etiology,
ity of functional di
Medications that
require dosing adjus
failure to avoid tox

--------------------------------------- 124
Chapter 3 / Cardiology

21
cyanosis or increase

d pulmonary blood flow and mild


cyanosis.
Clinical Manifestati
ons
Without obstruction,
clinical findings are similar to
those of an atrial s
eptal defect. There is an active precordium with a right
ventricular heave, a wide and
fixed split 82 with
a loud pulmonary component, and
a systolic ejection
murmur at the left upper sternal
border. On chest rad
iograph, cardiomegaly is noted
with increased pulmo
nary vascularity. On ECG,
right-axis deviation
and right ventricular hypertrophy are seen. A neon
ate with TAPVC with obstruction presents extrem
ely cyanotic, tachypneic, and
dyspneic. Examinatio
n reveals a right ventricular
heave, a narrowly sp
lit S->, and a ventricular gallop
Treatment
In TAPVC without obs
truction, treatment of con-

gestive heart failur


e is needed initially, and surgical
Figure 3-3
Infradiaphragmatic total anomalous pulmonary
venous connection. Note the following: (a) pulmonary venous redirection of aberr
ant vessels into the left atrium is
confluence does not connect with the left atrium, but descendsnecessary in the f
irst month of life. In TAPVC with
to connect with the portal circulation below the diaphragm. obstruction, the neo
nate should be taken to surgery
This connection is frequently severely obstructed as shown; (b)emergently for co
rrection. PGE] should not be given
obstruction to pulmonary venous return results in significantly
elevated pulmonary venous pressures, decreased pulmonary
uctus arteriosus adds more
blood flow, pulmonary edema and pulmonary venous desatulready flooded pulmonary

because the patent d


blood volume to an a
circuit.

ration (92%); (c) systemic to suprasystemic pressure in the pulmonary artery (in the absence of a patent ductus arteriosus,
pulmonary artery pressures may exceed systemic pressures
when severe pulmonary venous obstruction is present); (d) all

CYANOTIC CONGENITAL

systemic blood flow must be derived via a right-to-left shunt at


the foramen ovale; (e) nearly equal oxygen saturations in allHEART DISEASE: LESI
ONS WITH
chambers of the heart (i.e.,complete mixing at right atrial level),DUCTAL-DEPEND
ENT PULMONARY
with severe hypoxemia (systemic oxygen saturation 60%) and
BLOOD FLQW
low cardiac output (mixed venous oxygen saturation 25%).
Cloherty JP, Stark AR. Manual of Neonatal Care, 4th ed. Philadelphia:Tricuspid A
tresia
Lippincott-Raven, 1998:426.
Tricuspid atresia wi
th normally related great arteries
(Figure 3-4) is a ra
re defect that consists of complete
absence of right atr
ioventricular connection, which
Infradiaphragmatic (20% of cases): Blood drains
asia or absence of the right
via a vertical vein into the portal or hepatic veins
rcent of cases of tricuspid atresia
Mixed (10% of cases): Blood returns to the hearhavt
ntricular septal defect. In chilvia a combination of the above routes
atresia with normally related

leads to severe hypopl


ventricle. Ninety pe
e an associated ve
dren with tricuspid
great arteries, the

ventricular septal defect allows

TAPVC can occur with or without obstruction.


he left ventricle to the right venObstruction occurs when the anomalous vein
pulmonary arteries. The vast
enters a vessel at an acute angle. The presence omajoritr
s with tricuspid atresia with norabsence of obstruction determines whether thermalle
arteries also have pulmonary
is pulmonary venous hypertension and severe
id atresia the systemic venous

blood to pass from t


tricular outflow and
y of patient
y related great
stenosis. In tricusp

--------------------------------------- 125
210

Blueprints Pediatrics
KEY POINTS
pertension. Calcium carbonate

intake to control hy
and activated vitami

n D treat renal osteodystrophy.


1. The cause of ARF in children is usually prerenal,
erythropoietin improve the
but intrarenal and postrenal etiologies are
re occurs and catch-up growth
possible.
optimal caloric intake and nor2. Laboratory findings include azotemia, hyperlic parameters.
kalemia, and metabolic acidosis.
s than 10% of normal renal func3. In addition to managing the inciting condition,
reater than lOmg/dL) require
treatment consists of appropriate fluid manageenal transplant. Peritoneal dialysis,
ment, correction of electrolyte abnormalities and
ed at home, is the standard for
pH, protein restriction, and, occasionally, shortong-term dialysis. Peritonitis, the
term hemodialysis.
cation of peritoneal dialysis, is

Iron and recombinant


anemia. Growth failu
is unlikely despite
malization of metabo
Children with les
tion (a creatinine g
either dialysis or r
which can be perform
children requiring l
most frequent compli
usually due to gram-

positive organisms. Hemodialysis provides close t


o 10% of normal renal function
but is time-consumin
g. Hemodialysis-associated
mortality is low at
specialized pediatric centers, but
IB _CH RON 1C_RENA_L FA] LU RE ___
odialysis include disequilib-

complications of hem
rium syndrome, which

occurs when the serum urea


Chronic renal failure (CRF) implies that renal functoo rapidly, resulting in cerebral
tion has dropped below 30% of normal; function at
ptoms of disequilibrium
10% or less defines end-stage renal disease. The most
dache, nausea, vomiting,
common cause of CRF in the pediatric population is
le cramps, seizures, and coma.

nitrogen level drops


edema. Signs and sym
syndrome include hea
abdominal pain, musc

obstructive uropathy, followed by renal dysplasia,


glomerulonephropathies (particularly focal segd to vascular hemodialysis
mental glomerulosclerosis), and hereditary renal
rombosis, and infection.
conditions.
tion is the ultimate therapy for

Complications relate
include bleeding, th
Renal transplanta
all children with en

d-stage renal disease, and there


are few absolute con
traindications. The donated
Clinical Manifestations
a living related donor or a
History and Physical Examination
ted donor transplants have a better

organ may come from


cadaver; living rela
host and graft survi

val rate.
Growth failure frequently prompts evaluation for
require complex and timerenal disease in the outpatient setting. Subjective
and, as a consequence, often
complaints range from none to polyuria, episodic
e in their quality of life and are
unexplained dehydration, salt craving, anorexia,
opmental and social delays.
nausea, malaise, lethargy, and decreased exercise tolerance. Hypertension and pallor are noted on examination. Long-standing CRF produces rickets.
EY POINTS

Children with CRF


consuming treatment
experience a decreas
predisposed to devel

K
1. Children with g

rowth failure should be screened


Diagnostic Evaluation
se.
Patients with CRF demonstrate many of the same
hronic renal failure includes
laboratory abnormalities seen in ARF, including
ysis, hemodialysis, and renal
azotemia, acidosis, sodium imbalance, and hyper.
kalemia. Anemia is more pronounced in CRF than
ARF.
Treatment
Treatment for CRF includes nutritional, pharmacoontrol is usually achieved
logic, and dialysis therapy. Close monitoring of clin24 and 36 months, although
ical and laboratory status is required. Restrict protein
normal children take signifito prevent worsening azotemia. Restrict sodium
sis is the involuntary loss of urine

for renal disea


2. Treatment for c
peritoneal dial
transplantation

ENURESIS
Successful bladder c
between the ages of
many developmentally
cantly longer. Enure

--------------------------------------- 126
Chapter 14 /
211
Nephrology and Urology
in a child older than 5 years. It may be nocturnal or
daytime, primary or secondary. Primary enuretics are
patients who have never successfully maintained a

Treatment
Behavior modificatio

n programs are moderately


dry period, whereas secondary enuretics are usually
popular method of treatment
dry for several months before regular wetting recurs.
alarm that sounds as soon as

effective. The most


is a nighttime audio
the child starts to

urinate, eventually conditioning controlled bladd


er emptying before enuresis.
Clinical Manifestations
sin acetate (DDAVP; analogous
A careful history and physical examination may
essin] acts to concentrate the
suggest secondary causes for enuresis such as UTI,
he evening, less urine is produced
developmental delay, obstruction, emotional strain,
g the likelihood of wetting.

Intranasal desmopres
to endogenous vasopr
urine. If given in t
overnight, decreasin
With all therapies,

the cure rate is 15% per year after


or inappropriate parental toilet training expectations.
en who remain enuretic past
Primary nocturnal enuresis, which is far more
0% risk of never resolving their
common, is thought to be due to delayed maturational control or inadequate levels of antidiuretic
hormone secretion during sleep.

the age of 5; childr


age 8 years have a 1
symptoms.

--------------------------------------- 127
Neurology
NEURAL TUBE DEFECTS
s occurs below the lesion. In

and sensorimotor los


spina bifida occulta

, the bony vertebral lesion occurs


Failure of closure of the neural tube during the third
f any spinal contents. Birthand fourth weeks of gestation results in a group omarksf
hairy tufts at the base of the back
related disorders called neural tube defects. Materg defect. Although the infant
nal malnutrition, drug exposure, genetic factors, and
neurologically normal,
radiation have all been associated with an increased
e cord is affixed or "tethered" to
risk of neural tube defects. Because failure of closure
the vertebral column grows
results in persistent leakage of alpha-fetoprotein into
, the spinal cord is unable to
the amniotic fluid, the maternal serum alphat position, resulting in scoliosis,
fetoprotein level at 16 to 18 weeks is an excellensphinctet
ion, lower extremity deformities,
screening tool for identifying high-risk pregnancies.
deficits. These defects have a
The incidence of neural tube defects is decreased ihign
th the Arnold-Chiari malformainfants whose mothers receive folic acid supplemenof the hindbrain that poses sig-

without herniation o
, dimples, or
suggest an underlyin
may initially appear
the caudal end of th
the distal spine. As
throughout childhood
ascend into the adul
r dysfunct
and increasing motor
h association wi
tion, a malformation

tation during the early weeks of pregnancy.


drocephalus.

nificant risk for hy


Fetal surgery is

under investigation as a means of


repairing myelomenin
goceles in an attempt to preClinical Manifestations
ory function in these patients.
Abnormalities may occur anywhere along the central
nervous system; the higher the lesion, the more devastating the sequelae. Neonates with anencephaly are
Y POINTS
born with large skull defects and virtually no cortex.
Brainstem function is marginally intact. Many are
ernal serum alpha-fetoprotein
stillborn; others die within days of birth. Encephalo18 weeks of gestation is an excellent
celes are projections of cranial contents through a
al tube defects.

serve motor and sens

KE
1. An elevated mat
level at 16 to
screen for neur

2. The incidence o
f neural tube defects is decreased
bony skull defect, usually in the occipital region.
in infants whos
e mothers receive folic acid supSuch patients manifest severe mental retardation,
plementation ea
rly in the pregnancy.
seizures, and movement disorders. Hydrocephaly is a
frequent complication.
Myelomeningocele, meningocele, and spina bifida
occulta are neural tube defects in the spinal region
associated with bony abnormalities. MyelomeningoHYDROCEPHALUS
celes are protruding sacs of neural and meningeal
tissue, whereas meningoceles contain meninges only.
Pathogenesis
Both are most common in the lumbosacral regionHydrocephalu.
s re
sults when cerebrospinal fluid
Bowel and bladder sphincter dysfunction is the rule(CSF,
) production ou
tpaces absorption, usually

--------------------------------------- 128
Chapter 15 / Neurology
213
secondary to outflow obstruction. In noncommunitricular size, and s
ource of obstruction are clearly
cating hydrocephalus, the block exists somewherdelineatede
. A head
ultrasound may be sufficient in
within the ventricular system, and the ventricleths
e young infant. If
a lumbar puncture is indicated,
above the obstruction are selectively enlarged. Iin
t should not be att
empted if there is any danger of
contrast, all ventricles are proportionately enlarged iherniationn
.
communicating hydrocephalus, which occurs when
the subarachnoid villi are dysfunctional or obliter-

Treatment

ated. Hydrocephalus may result from a congenital


malformation or be acquired later in life.

If the underlying et

iology cannot be corrected, surgical diversion with


a ventriculoperitoneal shunt
Risk Factors
al pressure and relieves the symp-

decreases intracrani
toms. Acetazolamide

decreases CSF production and


Intrauterine infections, bacterial meningitis, intraventhe short term if the hydrotricular hemorrhaging, and subarachnoid bleeds all
re.
interfere with the absorption of CSF at the arachnoid
are fraught with complications,
villi. The process may be permanent if inflammation
ction and infection. Staphyloand scarring occur.

may be effective in
cephalus is not seve
Indwelling shunts
most commonly obstru

coccus epidermidis i
s the most frequently isolated
pathogen. Infected s
hunts must be externalized or
Clinical Manifestations
tion eradication, the shunt is re-

removed. After infec


internalized or repl

aced. Patients with hydrocephalus


History and Physical Examination
lopmental delay, visual impairThe clinical manifestations of hydrocephalus depend
urbances.
on the rate of onset and the patency of the

are at risk for deve


ment, and motor dist

fontanelles. An inappropriate increase in head circumference or bulging anterior fontanelle may be the
Y POINTS
only indication in infants; poor feeding, irritability,

KE

1. Clinical manife
stations of hydrocephalus in infants
lethargy, apnea, and bradycardia often provide addipriately large head circumference,
tional clues. In older patients with acute courses, the
lle, poor feeding, irritability, and
signs are relatively clear and include headaches,
nausea, vomiting, irritability, lethargy, papilledema,
is a late indicator of hydroupward gaze paralysis (the "setting sun sign"], and

include inappro
bulging fontane
lethargy.
2. Cushing's triad

cephalus.
diplopia (third or sixth cranial nerve palsies, or both).
re is contraindicated if hydroClonus, a positive Babinski test, and excessively brisk
sent and herniation is a concern.
deep tendon reflexes are additional neurologic signs.
Cushing's triad, consisting of bradycardia, hypertension, and Cheyne-Stokes respirations, is a late and

3. A lumbar punctu
cephalus is pre

ominous development.
Differential Diagnosis
is a nonprogressive disorder of

CEREBRAL PALSY
Cerebral palsy (CP)
movement and posture

that results from a fixed


Conditions that lead to increased intracranial presre brain. It is the most common
sure without hydrocephalus include intraventricular
children. Most cases occur in

lesion of the immatu


movement disorder in
the absence of ident

ifiable risk factors (i.e., premableed, diffuse brain edema (secondary to traumatic
brain injury, hypoxic-ischemic encephalopathy, or
ia, intrauterine growth retardaencephalitis), abscesses, and tumors, all of which are
n, or trauma). Contrary to earlier
easily differentiated by computed tomography (CT)
d obstetric complications are not
or magnetic resonance imaging (MRI).
ncreased risk of CP.
Diagnostic Evaluation
ons
The CT scan is an important adjunct in the evaluaof CP is spastic CP (pyration of hydrocephalus. Anatomic malformations, venconsequence of injury to motor

turity, birth asphyx


tion, early infectio
speculation, isolate
associated with an i
Clinical Manifestati
The most common form
midal), which is the

--------------------------------------- 129
214

rints Pediatrics
TABLE 15-1
lude oral-motor dysfunction, gas-

tered conditions inc


troesophageal reflux

, and behavior problems.


Topographic Classification of Spastic (Pyramidal)
Cerebral Palsy
Diplegia bilateral lower extremity spasticity
POINT
Quadriplegia all limbs severely involved, lower
a nonprogressive disorder of
extremities more than upper
sture resulting from a fixed
Hemiplegia one side involved, upper extremity more
in.
than lower
Bilateral hemiplegia all limbs severely involved, upper
extremities more than lower

KEY
1. Cerebral palsy is
movement and po
injury to the bra

SEIZURE DISORDE
RS
tracts in the brain. It is characterized by increased
rary disruption of brain function

Pathogenesis
A seizure is a tempo
resulting from abnor

mal, excessive, synchronous


muscle tone in the affected limbs. The disorder is

cerebral neuron disc

harge. A patient is diagnosed


further classified by which limbs are involved (Table
nprovoked seizures become
15-1). Patients with CP are generally hypotonic
re). There are many diseases,
through the first few months of their life, only later
sorders that cause seizures.
developing the characteristic spasticity. It is usually
ents, the etiology remains
very difficult to make the diagnosis until a patient is
failing to meet motor developmental milestones, or
the spasticity becomes apparent on exam. As a
patient's body grows and new developmental tasks
are encountered, the condition may appear to be
will have a seizure sometime
progressive (but is not). Extrapyramidal CP, a
neonates, trauma, hypoxia, and
consequence of kernicterus, is a rare but important
imary causes of seizures. Infecdisorder that has largely been eliminated by advances
izures rank high in infancy and
in the management of hyperbilirubinemia. As the
classification implies, this motor disorder is charactemic disease, hypoglycemia,
terized by choreoathetoid movements and ataxia.
bolic abnormalities, ingestions,

with epilepsy when u


recurrent (two or mo
derangements, and di
In about 50% of pati
undetermined.
Epidemiology
About 5% of children
during childhood. In
infection are the pr
tions and febrile se
young childhood. Sys
electrolyte and meta
and congenital defec

ts can also result in seizure activity. Idiopathic epil


epsy is the most common form
Treatment
hildren and adolescents. One to

diagnosed in older c
2% of the general po

pulation suffer from epilepsy.


A multidisciplinary team approach, including a
general pediatrician, physical and occupational therapists, nutritionist, speech-language therapist, and
social support services, results in optimal therapy
with the goal of maximizing function. Many mediory of febrile seizures are at a
cines have been tried to reduce spasticity (including
risk of epilepsy later in life.
benzodiazepines, dantrolene, and baclofen) with variable success. Recently, however, significant improveons
ments in motor function have been achieved with
amination, and
botulinum toxin motor point blocks.
Some children with cerebral palsy are otherwise
n
cognitively normal, but as the severity of symptoms
eizure disorder is primarily based
increases, so does the risk of associated neurologic
count of the episode and the
deficits. Over half will have cognitive deficits ranging
. Electroencephalogram (EEG)
from learning disabilities to mental retardation. A
ntary and particularly useful in
third develop seizure disorders. Many have hearing
osis, documenting baseline activ-

Risk Factors
Children with a hist
minimally increased
Clinical Manifestati
History, Physical Ex
Diagnostic Evaluatio
The diagnosis of a s
on the historical ac
physical examination
studies are compleme
confirming the diagn

and vision impairments. Other frequently encounffective treatment. Table 15-2

ity, and selecting e

--------------------------------------- 130
Chapter 15 / Neurology
215
TABLE 15-2
is not uncommon. Seizures

bladder incompetence
may also be solely t

onic or solely clonic.


International Classification of Epileptic Seizures
mal, seizures almost always begin
Partial seizures
than 10 years. They are brief,
Simple partial (intact consciousness)
ociated with alterations in conMotor
d is unaware and immediately
Sensory
at hand with no postictal phase.
Autonomic
petit mal seizures can occur
Psychic
day and may interfere with
Complex partial (impaired consciousness)
zation. An EEG demonstrates
Partial seizures with secondary generalization
eneralized, symmetric threeGeneralized seizures
wave pattern.
Absence (typical, atypical)
onsist of abrupt, total loss of posTonic
everal minutes. Myoclonic seizures
Clonic
Tonic-clonic
rks similar to those occasionally
Myoclonic
l subjects while in light sleep.
Atonic
imally impaired, and there is no
Infantile spasms
clonic seizures are common in

Absence, or petit
in children younger
staring episodes ass
sciousness. The chil
returns to the task
Although very brief,
hundreds of times a
learning and sociali
the characteristic g
per-second spike and
Atonic seizures c
tural tone lasting s

are simple, short je


experienced by norma
Consciousness is min
postictal phase. Myo
patients with cerebr

al palsy and degenerative disorders. Akinetic seizu


res are a subclass of myoclonic
seizures; they resem
ble atonic seizures but are
extremely brief.
delineates the current international classification of
Two particularly
devastating generalized seizure
epileptic seizures.
syndromes are infant
ile spasms and Lennox-Gastaut
In partial seizures, only a small focus in one hemisyndrome. Infanti

le spasms, which usually present


sphere is involved. The child remains conscious, and
between 2 and 7 mont
hs of age, are recurrent mixed
there is no postictal phase. Partial seizures may
flexor-extensor spas
ms that last only a few seconds
involve very specific movements or sensations that
but may repeat more
than 100 times in a row. This
remain stable with recurrent episodes. The symptomseizurs
e disorder ma
y be associated with many difare specific to the area of the brain involved and may
ferent neurodevelopm
ental diseases (e.g., mental
be motor, cognitive, affective, or somatosensory. Jackretardation, hydroce
phalus, congenital malformasonian seizures are partial motor seizures in which a
tions). The diagnosi
s is confirmed by a typical EEG
rhythmic twitching begins in one extremity and
pattern known as hyp
sarrhythmia. Both adrenocorti"marches" proximally until the entire limb is
cotropin (ACTH) and
corticosteroid administration
involved. Other partial seizures are more complex
have been shown to c
ontrol the seizures in many
and may result in alteration (but not loss) of conpatients but do not
prevent developmental delay.
sciousness. Semipurposeful movement continues
Infantile spasms may
evolve into Lennox-Gastaut
without direction, or the child may begin lip-pursinsyndromeg
, charact
erized by the frequent occurrence
or picking at his clothes. Occasionally, partial seizures of mixed, generalize
d seizures that are notoriously
progress to generalized convulsions.
refractory to pharma
cologic treatment.
Generalized seizure disorders produce a clinical
syndrome indicative of bilateral hemispheric involveDifferential Diagno
sis
ment, such as impaired consciousness, symmetric
bilateral activity, and a postictal phase of confusion
Febrile seizures do
not represent true epilepsy. They
and lethargy. Tonic-clonic seizures are what most
typically occur in c
hildren 6 months to 5 years old
people think of as typical seizures. The tonic phase
with fevers greater
than 39C. The rapid rise in temis characterized by sustained flexor or extensor conperature, rather tha
n the height of the fever, is the
traction; these episodes are interspersed with cloniimportanc
t determi
nant. A simple febrile seizure lasts
activity, consisting of rhythmic, symmetric, generalless than 10 minutes
, is generalized, and does not
ized contractions of the trunk and extremity muscle
recur within 24 hour
s. Complex febrile seizures last
groups. Breathing may be irregular, although most
longer than 15 minut
es, recur within 24 hours, or
episodes do not progress to cyanosis. Bowel oshor
w signs of focal
ization. Such children should

--------------------------------------- 131
216
rints Pediatrics
receive additional studies and close follow-up or hoscontinuous EEG le
ads, a focus may be discovered
pitalization for observation. Simple febrile seizures
that can be removed
surgically. The risks and benedo not require evaluation beyond determining the
fits of such a proce
dure need to be carefully explored
source of the fever. Caretakers should be counselewitd
h the patient and
family. Another option is the
concerning fever avoidance and seizure precautions.
ketogenic diet. Indu
cing ketosis through a high-fat
Children who are toxic appearing, have meningeal
"ketogenic" diet may
control symptoms in some
signs, an abnormal neurologic examination, or have
children. The vagal
nerve stimulator, approved by
an underlying brain abnormality should not be prethe Food and Drug Ad
ministration in 1997, has
sumed to have had a febrile seizure without ruling
proven quite benefic
ial in some patients.
out more serious etiologies. Significant neurologic
Most children wit
h seizure disorder undergo
deficits resulting from febrile seizures are exceedremission, after whi
ch the medication can be
ingly rare. In most cases, the seizures do not recur
tapered. Unfortunate
ly, this is not true for children
with subsequent febrile episodes.
who have seizure dis
orders as a result of congenital
Essential tremor, spasmus nutans, tics, Tourette's
or acquired brain da
mage.
syndrome, and myoclonus are various movement
disorders that originate in the basal ganglia and may
mimic seizures. Essential tremor begins in infancy or
Emergency Management
of
childhood and may involve the chin, head, neck, and
Status Epilepticus
hands; it usually does not interfere with normal
Status epilepticus i
s defined as a prolonged episode
functions. Spasmus nutans includes head nodding
and rapid, small-amplitude nystagmus as well.
of seizure activity
(greater than 30 minutes) or an
Tourette's syndrome consists of motor and vocal tics;
extended period of r
ecurrent seizures between which
patients often demonstrate obsessive-compulsive
the patient does not
return to consciousness. Status
tendencies and attention deficit hyperactivity disorepilepticus is dange
rous, leading to hypoxia, brain
der. Myoclonic movements are sudden, involuntary
damage, and death. A
irway, breathing, and circulation
should be evaluated
and addressed as necessary.
jerk-like motions similar to startle responses.
Intravenous or recta
l short-acting benzodiazepines
Other conditions that may be confused with
often break the seiz
ure. Usually, a phenytoin loading
seizures include breath-holding spells, syncope,
dose is administered
as well to prevent recurrence;
benign paroxysmal vertigo, and temper tantrums.
phenobarbital is pre
ferred in newborns and young
Pseudoseizures should be suspected in the patient
infants.

with implausible findings (e.g., alert and responsive


during generalized tonic-clonic movements).
K
EY POINTS
Treatment
1. Generalized sei
zures are always associated with
Effective treatment combines education and medonsciousness.
ication. Both the child and the parents should
res show a characteristic threebecome knowledgeable about acute care and local
e and wave pattern on EEG;
emergency medical services.
s show hypsarrhythmia on EEG.
With medication, about 50% of patients will be
s are complex if they last more than
seizure-free. Another 30% will have significant
cur within 24 hours, or show signs
reductions in seizure frequency or intensity or both.
.
There has been a dramatic increase in the number
of medications available for the management of
seizures. The newer medications have a better toxicity profile. Their names, indications, and side effects
are listed in Table 15-3. Conventional anticonvulsants require careful monitoring of serum levels; the
a significant, often preventable
newer drugs do not.
nd mortality. Head injuries in
For patients with poor seizure control on mediaresult from motor vehicle accition (about 20%), additional interventions are availps, falls, or child abuse. Males are
able. By monitoring a patient's seizures with
emales to sustain significant head

impairment of c
2. Petit mal seizu
per-second spik
infantile spasm
3. Febrile seizure
15 minutes, re
of focalization

HEAD TRAUMA
Acute head trauma is
cause of morbidity a
children most often
dents, bicycle misha
twice as likely as f

--------------------------------------- 132
Chapter 15 / Neurology
TABLE 15-3

217

Indications and Side Effects of Anticonvulsants


Medication Indications
Effects/Toxicity
Conventional Drugs
Carbamazepine (Tegretol)
opia, nausea and vomiting, ataxia,

Side

Partial, tonic-clonic

Dipl
l

eukopenia, thrombocytopenia.
Ethosuximide (Zarontin)
, anorexia, leukopenia, aplastic

Absence

Rash
a

nemia.
Phenobarbital (Luminal)

Tonic-clonic, partial

Hype

ractivity, sedation, nystagmus,


a
taxia.
Phenytoin (Dilantin)
, nystagmus, ataxia, drug-induced

Tonic-clonic, partial

Rash
l

upus, gingival hyperplasia, anemia,


l
eukopenia, polyneuropathy.
Valproic acid (Depakote)
totoxicity, nausea and vomiting,

Tonic-clonic, absence,
partial

Hepa
a

bdominal pain, weight loss,


w
eight gain, anemia, leukopenia,
t
hrombocytopenia.
Newer Drugs
Gabapentin (Neurontin)
nolence, dizziness, ataxia, and

Partial

Som
f

atigue.
Lamotrigine (Lamictal)
ziness, ataxia, blurred or double

Tonic-clonic, partial,

Diz

absence, and Lennox-

Gastaut

ision, nausea, vomiting, and rash.


few cases of Stevens-Johnson
s
yndrome have been reported.
Levetiracetam (Keppra)
nolence, asthenia, and dizziness
Tiagabine (Gabitril)
ziness, somnolence, and tremor.

Partial

Som

Partial

Diz
M

ay make absence epilepsy worse.


Topiramate (Topamax)
nolence, fatigue, weight loss,

Tonic-clonic, partial,
Lennox-Gastaut,

Som
a

nd nervousness.
infantile spasms
trauma. Recovery from a head injury depends on thquentle
y associated
with coma, and prolonged rehadegree of the initial injury and factors contributing
bilitation is often
required.
to secondary neuron injury such as hypotension and
Brain hemorrhages
that occur after trauma are
hypoxia. Severe injury is often associated with behavusually subdural or
epidural rather than intraioral changes, motor impairment, and memory probparenchymal (Tab
le 15-4; Figure 15-1).
lems. About 10% of children hospitalized for a
traumatic brain injury will have a seizure, and 35%
ons
of these will go on to have a seizure disorder.
A concussion is denned as a brief loss of con-

Clinical Manifestati

History

sciousness after head injury associated with retroThe source of injury


should be described by the child
grade and anterograde amnesia. Brain injury is
and caretaker separa
tely whenever possible. A history
undetectable, and the neurologic examination
that is not consiste
nt with a given injury is suggesreturns to normal within hours. In contrast, cerebrativl
e of child abuse.
Reports of vomiting, severe
contusions represent a bruise to the brain
headache, and mental
status changes strongly suggest
parenchyma. Diffuse axonal injury results from
increased intracrani
al pressure. Confusion, loss of
shearing forces on the white matter of the brain thaconsciousnesst
, am
nesia, seizures, and visual impairoccur with rapid deceleration of the head. It is frement may also be p
resent after significant injury.

--------------------------------------- 133
218

Blueprints Pediatrics
TABLE 15-4

Differentiating Acute Subdural and Epidural Bleeds


Subdural
Epidural
Location
Between the dura and arachnoid layers
Between the skull and the dura
Symmetry
Usually bilateral
Usually unilateral
Etiology
Rupture of bridging cortical veins
Rupture of middle meningeal
artery or dural veins
Typical injury
Direct trauma or shaking
Direct trauma in the temporal
area
Consciousness
Intact but altered
Impaired-lucid-impaired
Common associated
Seizures, retinal hemorrhages
Ipsilateral pupillary dilatation,
findings
papilledema, contralateral
hemiparesis
Appearance on CT
Biconcave
with contrast

Crescentic

Prognosis
High morbidity; low mortality
High mortality; low morbidity
Complications
Herniation
Skull fracture; uncal herniation
The Glasgow Coma
Score (Table 15-5) provides

a rapid, widely used


, easily reproducible method of
quantifying neurolog
ic function and helps guide
initial therapy.
Diagnostic Evaluatio
n
A B
should be performed in all

Cervical spine films


children with signif

icant head trauma to rule out


Figure 15-1
A. Subdural bleed. B. Epidural hematoma.
nts with loss of consciousness,

cervical injury. Patie


persistent altered m

ental status, focal signs on


neurologic examinati
on, signs and symptoms of
increased intracrani
al pressure, suspected skull fracPhysical Examination
Bradycardia, hypertension, and irregular (Cheyneinjury to the brain should have a
Stokes) respirations form Cushing's triad, the hallt.
mark of increased intracranial pressure. Palpation of

ture, or penetrating

the head may reveal step-off (depressed) skull frac-

Treatment

head CT with contras

tures or a bulging fontanelle. Basilar skull fractures


are characterized by periorbital ("raccoon eyes") or
the severity of the injury.
postauricular ("Battle's sign") bruising, hemotympated head or neck injury should
num, or CSF rhinorrhea or otorrhea. Serial neurok board with appropriate cervilogic examinations track evolving lesions. Cranial
tion in the field. Those with a
nerve function, especially pupil size and reactivity,
ess than 8 generally require
may help localize the injury. Sensory and motor
ion is uncommon in isolated
function is difficult to assess in the patient with
ociated injuries may lead to
impaired mental status, who may respond minimally
hock from hemorrhage, neuroeven to noxious stimuli. Deep tendon and pathologic
nal cord injury, and cardiogenic
reflexes should be assessed in all patients.
l contusion). The goal of sup-

Treatment depends on
Patients with suspec
be placed onto a bac
cal spine immobiliza
Glasgow Coma Score l
intubation. Hypotens
head trauma, but ass
shock (hypovolemic s
genic shock from spi
shock from myocardia

--------------------------------------- 134
Chapter 15 / Neurology
TABLE 15-5

219
KE

Y POINTS
Glasgow Coma Scale
idural hemorrhages are more

1. Subdural and ep
common than int

raparenchymal bleeding when


Activity
Score

Activity*

Eye opening
rauma-related.
Spontaneous
4
Spontaneous
consists of hypertension, bradycarTo speech
3
To speech
al respirations.
To pain
2
To pain
bral perfusion pressure is the goal
None
1
None
are in severe brain injury.
Verbal
Oriented

Coos, babbles

Confused

Irritable

Inappropriate words

Cries to pain

Nonspecific sounds

Moans to pain

None

None

Motor
ly rare in children but may be
Follows commands
6
l hemoglobinopathy, vasculitis,

the injury is t
2. Cushing's triad
dia, and abnorm
3. Optimizing cere
of supportive c

ARTERIOVENOUS

MALFORMATIONS

Strokes are relative


Normal, spontaneous
movements

rcoagulable states, and abnorLocalizes pain


5
tabolism. Congenital vascular
Withdraws to pain
4
ding arteriovenous malformations
Abnormal flexion
3
common cause of intracranial
Abnormal extension
2
diatric population. An AVM
None
1

caused by sickle cel


emboli, trauma, hype

Withdraws to touch

malities of lipid me

Withdraws to pain

abnormalities, inclu

Abnormal flexion

(AVMs), are the most

Abnormal extension

hemorrhage in the pe

None
is an abnormal colle

ction of arteries and veins. It


* Modified for infants.
sical findings consistent with

may present with phy


seizures, acute hemo

rrhage, or a focal mass. Occasionally, a cranial


bruit is present on physical examination. Arteriograph
y allows determination of the site
of the abnormality a
nd feeding vessels. Surgery is
portive therapy is to optimize the cerebral perfusion
cases; however, extensive lesions
pressure, which is the difference between the mean

appropriate in some
are usually treated

by selective embolization.
arterial pressure and the intracranial pressure.
Cerebral edema is the most important complication
in the acute period. Normoxia, normothermia, nor-

.HEADACHES

moglycemia, hyperosmolality, and elevation of the


head of the bed are recommended to minimize
on complaint in the pediatric
intracranial hypertension and secondary brain injury.
portant to rule out dangerous conMild hyperventilation, which reduces cerebral blood
s, intracranial bleeds, meningitis)
flow, is used to decrease intracranial pressure during
patient has more benign tension
the initial phase of therapy. Normoventilation is generally utilized after the initial period of brain swelling
an underdiagnosed condition in
has resolved. Patients with evidence of impending
tion. These severe, recurrent,
herniation may be vigorously hyperventilated and
l headaches may be precipitated
given an osmotic agent such as mannitol to decrease
c food ingestions (e.g., chocolate].
intracranial pressure acutely. Patients with evidence
the pain. Migraines are
of significant cerebral edema require intracranial
y an aura, generally a specific
pressure monitoring with a subdural bolt or
ch as a receding tunnel or flashventriculostomy.
ients give a positive family history.
Children with a history of loss of consciousness or
e associated with transient
abnormal neurologic findings should receive medical
such as aphasia or hemiparesis.
attention and may require hospitalization. Patients
ntly prescribed vasoconstrictor,
who remain asymptomatic for 4 to 6 hours may be
the headache if administered
safely observed at home.

Headaches are a comm


population. It is im
ditions (e.g., tumor
before declaring the
headaches.
Migraines remain
the pediatric popula
pounding, often foca
by stress or specifi
Nausea may accompany
typically preceded b
visual experience su
ing lights. Most pat
Complex migraines ar
neurologic deficits
Ergotamine, a freque
relieves or obviates
early in its course.

--------------------------------------- 135
22

rintg Pediatrics
Clinical Manifestati

ons
Neonates with tricus
pid atresia with normally related
great arteries prese
nt with progressive cyanosis, poor
feeding, and tachypn
ea over the first 2 weeks of life.
95%

On cardiac examinati

on, the harsh holosystolic


murmur of a ventricu
lar septal defect at the left
lower sternal border
and the continuous murmur
of a patent ductus a
rteriosus may be heard. On
ECG, there is a supe
rior axis and left ventricular
hypertrophy. Finding
s on chest radiograph include
normal heart size an
d decreased pulmonary vascular
markings.
Treatment
A child with tricusp
id atresia with normally related
great arteries shoul
d have PGEj started to maintain
pulmonary flow, and
a balloon atrial septostomy
should be performed
if the atrial defect is not adequate. Surgical mana
gement for tricuspid atresia
involves placing a m
odified Blalock-Taussig shunt to
maintain pulmonary b
lood flow. The modified
Tricuspid atresia with normally related great arterFigure 3-4
ies and a small patent ductus arteriosus.Typical anatomic andBlalock-Taussig shu
nt is a Gortex conduit placed
hemodynamic findings include: (a) atresia of the tricuspid valve;between the sub
clavian artery and the pulmonary
(b) hypoplasia of the right ventricle; (c) restriction to pulmonaryartery. Ultim
ately, a cavopulmonary anastomosis
blood flow at two levels: a (usually) small ventricular septal(hemi-Fontan or bi
directional Glenn) is performed
defect and a stenotic pulmonary valve; (d) all systemic venous
return must pass through the patent foramen ovale to reach to provide stable pu
lmonary blood flow. In most
centers, a modified
Fontan procedure is performed to
the left ventricle; (e) complete mixing at the left atrial level, withredirect t
he inferior vena cava and hepatic vein flow
systemic oxygen saturation of 78% (in Fio2 of 0:21), suggesting
balanced systemic and pulmonary blood flow.
into the pulmonary c
irculation.
Cloherty JP, Stark AR. Manual of Neonatal Care, 4th ed. Philadelphia:
Lippincott-Raven, 1998:426.
Pulmonic Atresia wit
h Intact
Ventricular Septum
Pulmonic atresia wit
h intact ventricular septum
(Figure 3-5) is a ra
re defect consisting of pulmonary
return is shunted from the right atrium to the left
valvular and infundi
bular atresia and varying degrees
atrium through the patent foramen ovale or an atrial
of right ventricular

and tricuspid valve hypoplasia. In


septal defect, and the left atrium and left ventricle
is an obligate atrial shunt from
handle both systemic and pulmonary venous return.
ulmonary blood flow is dependent
Oxygenated and deoxygenated blood is mixed in the
rteriosus. Since there is no pulleft atrium. Cyanosis is severe in the neonatal period
right ventricle is hypertensive
and is proportionally related to the amount of puloderate to severe tricuspid regurmonary blood flow. In 30% of cases, there is transatresia with intact ventricular
position of the great arteries, which results in blood
ssociated with coronary
passing from the left ventricle through the ventricunusoid communication. The
lar septal defect to the right ventricular outflow and
y be quite abnormal, with areas
the ascending aorta. Tricuspid atresia with transposiete atresia. In some cases, corotion of the great arteries is often associated with
e dependent on the hypertencoarctation of the aorta or aortic arch hypoplasia.
. If the coronaries are right
Unlike tricuspid atresia with normally related great
dent, any palliative procedure
arteries it is a cyanotic lesion with ductal dependent
e right ventricle may lead to
systemic blood flow.
n and death.

this disorder, there


right to left, and p
on a patent ductus a
monary outflow, the
and there is often m
gitation. Pulmonary
septum may also be a
artery-myocardial si
coronary arteries ma
of stenosis or compl
nary perfusion may b
sive right ventricle
ventricle (RV) depen
that decompresses th
myocardial infarctio

--------------------------------------- 136
220

rints Pediatrics
Pseudotumor cerebri is an uncommon but impor-

TABLE 15-6

tant cause of headaches that typically occurs in overweight adolescent females or in association with
opathy in Children
tetracycline or corticosteroid use. The exam is posiInfection
tive for papilledema. Repeated lumbar punctures,
ders
AIDS encephalopathy
which demonstrate increased opening pressure, may
Encephalitis
alleviate the headaches.
Varicella

Causes of Encephal
Burns
Electrolyte disor
Hyponatremia
Hypernatremia
Hypocalcemia

Mumps
Hypercalcemia
Measles
Hypomagnesemia
Enterovirus
EN CEPH ALOPATH Y_
Cytomegalovirus

Hypermagnesemia

Factitious fever
Herpes simplex encephalitis
To function normally the brain needs adequate blood
Lyme disease
flow, oxygen, energy substrates, removal of metaboli
Tuberculosis
waste, and appropriate electrolyte balance. DisrupReye's syndrome
tion of any of these will lead to generalized cerebral
Metabolic disorders
dysfunction, termed encephalopathy.
Uremia

Hypertension
Hypoxia/ischemica
Hysteria
Toxins
Lead
Illicit drugs

Hypoglycemia
Carbon monoxide
Ketoacidosis
Differential Diagnosis
Environmental toxins

Sedatives
Anticholinergic

s
Parainfectious syndromes
Conditions that may lead to encepholopathy are

Salicylates

listed in Table 15-6. Recent or concurrent febrile


illness is consistent with encephalitis. Focal findings
on examination and seizures are more common with
cologic screening. A
n emergency head CT scan is
herpes simplex encephalitis than other viral etiologies. Reye's syndrome tends to follow an acute viral
s with evidence of increased
illness, especially when aspirin has been administered
e or focal neurologic signs. A

indicated in patient
intracranial pressur
lumbar puncture is a

ppropriate when meningitis or


to the child. Severe headache, vomiting, papilledema,
and alterations in vital signs are characteristic of the
ected and increased intracranial
increased intracranial pressure that always accomled out. Herpes simplex virus
panies this disease. Metabolic disorders typically
acterized by focal slowing or local

encephalitis is susp
pressure has been ru
encephalitis is char
wave pattern changes

in temporal lobe activity on


present with recurrent episodes of mental status
changes that clear when the acute process is core abnormalities on CT.
rected. A careful history may suggest environmental

EEG and temporal lob

exposures or drug use.

Treatment
The therapy for Reye

's syndrome involves close monClinical Manifestations


cose, electrolytes, transaminases,

itoring of serum glu

History and Physical Examination


ts with severe disease require
Encephalopathy is characterized by mental status
intracranial pressure monitoring

and ammonia. Patien


intubation and close
in an intensive care

unit. Treatment of infectious


changes, confusion, odd or inappropriate behavior,
of appropriate antibiotic therapy.
disorientation, a shortened attention span, cognitive

meningitis consists

deficits, hyporesponsiveness, lethargy, stupor, or


are discussed in Chapter 9.

Metabolic disorders
Ingestions are disc

ussed in Chapter 2.
coma. The onset may be rapid or insidious.
Diagnostic Evaluation
KEY POINT
Electrolyte abnormalities, uremia, hypoglycemia,
e is an encephalopathy associated
acidemia, and hyperammonemia (as in Reye's synfunction that has been observed
drome) can be ruled out with simple blood tests. The
n children with viral illnesses who
white blood cell count is elevated in the presence of
n.
infection. Urine and blood should be sent for toxi-

1. Reye's syndrom
with liver dys
occasionally i
receive aspiri

--------------------------------------- 137
221
Chapter 15 / Neurology
_VVEAKNESS.OR PARALYSIS
, most patients continue to

several years; however


experience periodic

exacerbations throughout adultAbnormalities leading to weakness or paralysis, or


rase therapy may relieve all or

hood. Anticholineste
most of the symptoms

in patients with mild involveboth, may occur at any level of the neuromotor axis,
s and other immune suppresfrom the motor cortex and pyramidal tracts to the
autoimmune response. Finally,
anterior horn cell, peripheral nerve, neuromuscular
recognized as a potential
junction, and muscle.
presumably because the
Differential Diagnosis
sensitize the lymphocytes pro-

ment. Corticosteroid
sants help curb the
thymectomy has been
method of treatment,
thymus is thought to
ducing the offending

antibodies.
Guillain-Barre syndrome (GBS) is an acutecular dystrophy (DMD], an
onset, progressive, ascending weakness caused bX-linkey
disease of muscle tissue, is the
autoimmune-mediated demyelination of the periphclassi-

Duchenne-type mus
d recessive
c myopathy. Al

though present at birth, the


eral nerves. It typically develops 7 to 21 days after an
disease presents in
early childhood with motor delay.
acute viral illness. Sensory and autonomic impairWeakness is great
est in the proximal muscle groups,
ments are often present but not prominent. Initiasl
o the patient must
rise from sitting on the floor in
symptoms include numbness of the distal extremitwo steps: first le
aning on the hypertrophied calves,
ties followed by progressive, ascending weaknessan.
d then pushing th
e trunk up with the arms
Deep tendon reflexes wane and disappear. Severity
(Gower's sign). Even
tually, ambulation is lost, the
varies from mild weakness to progressive involvemuscles atrophy, and
contractures develop. Cardiac
ment of the trunk, respiratory muscles, and craniaanl
d cognitive abnor
malities may be present but are
nerves; respiratory muscle involvement may necessiseldom severe. Trea
tment is supportive. Most chiltate mechanical ventilation. A significantly increased
dren become wheelcha
ir bound early in the second
CSF protein level is consistent with GBS. Recovery
decade, with death b
efore age 30 from either respitakes weeks to months, and some patients experience
ratory failure or ca
rdiomyopathy.
permanent lingering disability. Plasmapheresis or
Spinal muscle atr
ophy (SMA), also known as
intravenous immune globulin may hasten resolutionWerdnig-Hoffman.
n disease, is an inherited disorder
Tick paralysis resembles GBS. Certain ticks in the
involving degenerati
on of the anterior horn cells and
Appalachian and Rocky Mountains are capable of
cranial nerve motor
nuclei. The more severe form,
producing a neurotoxin that blocks acetylcholine
SMA type I, becomes
evident in early infancy with
release. The patient recovers completely when thgeneralizee
d hypoto
nia and weakness. SMA type 2
tick is removed from the skin.
presents between 6 a
nd 12 months of age and is
Myasthenia gravis (MG) is an autoimmune disorusually less severe.
Cognitive abilities remain unafder of the neuromuscular junction. Auto-antibodies
fected in both forms
of the illness. No specific
bind to the postsynaptic acetylcholine receptor and
therapy is available
; death occurs from repeated aspiblock its activity. The rate of receptor breakdown alsratioo
n or lung infe
ctions. SMA and DMD are conincreases, so fewer receptors are present. The princifirmed by character
istic changes on EMG and muscle
pal symptoms are easy fatigability and weakness thabiopsyt
.
is exacerbated by sustained activity and improves
a viral illness affecting primarily
with rest. MG typically presents in late childhood or
lls of the spine. There have only
adolescence; the onset may be rapid or insidious, anbeed
polio in the last several years, and
symptoms wax and wane over time. Almost half othef
een related to reversion of the
patients experience ocular muscle involvementliv,

Poliomyelitis is
the anterior horn ce
n a few cases of
y seem to have b
e oral polio vacc

ine to wild type. As a result, killed


resulting in ptosis or diplopia or both. Bulbar weakviruf an injected vaccine is now recness leads to dysarthria and difficulty swallowingommended.
oral vaccine still has an important
Administration of an intravenous anticholinesterase
efforts because it contributes to
results in a transient increase in muscle strength in
ng passively spread.
MG by blocking the breakdown of acetylcholine in
ess the spinal cord result in
the synaptic cleft. Repetitive electrical nerve stimuis below the lesion and constilation (EMG) studies demonstrate a significant fall in
gency. Cervical spinal cord
response strength over several rapid-fire trials. MG
den-onset paresthesias and
may go into complete or partial remission after
ental toxin exposure may induce

s in the form o
. The
role in world health
herd immunity by bei
Tumors that compr
weakness and paralys
tute a surgical emer
injuries produce sud
paralysis. Environm

--------------------------------------- 138
222
Blueprints Pediatrics
acquired neuropathies or myopathies. For example,
nce visual disturbances, changes
infants in certain endemic areas may be exposed to
dropping school grades.
spores of Clostridium botulinum and develop proophy, so named because of its
gressive paralysis from the elaborated toxin, which
n with adrenal insufficiency, is
irreversibly blocks release of acetylcholine at the
eas of demyelination coupled
motor end plate.
ivascular inflammatory reaction.

adolescents experie
in personality, and
Adrenoleukodystr
frequent associatio
characterized by ar
with an intense per
Psychomotor retarda

tion progresses to spasticity,


Clinical Manifestations
and death by early adulthood.

extensor posturing,
Dietary therapy is

controversial; no curative treatDiagnostic workup is tailored by findings on history

ment is available.

and physical examination. Patients with asymmetric


weakness or signs of increased intracranial pressure
KEY POINT
should receive neuroimaging to rule out mass or
hemorrhage. Findings localized to a particular level
trophy is the classic white matter
of the spinal cord require evaluation for cord comisease.
pression or injury. A lumbar puncture is helpful when

1. Adrenoleukodys
degenerative d

infection is suspected.
ATAXIA

KEY POINTS
Ataxia is the inabi
lity to coordinate purposeful
1. Guillain-Barre syndrome is an acute-onset, ascends that affect the cerebellum or
ing, progressive weakness caused by peripheral
y to cause ataxia in children.
nerve demyelination.

movement. Condition
inner ear are likel

2. Myasthenia gravis is an autoimmune disorder of


the neuromuscular junction characterized by easy

Differential Diagno

sis
fatigability and weakness.
ve been known to cause ataxia
3. Gower's sign is classically observed with
cute labyrinthitis. Acute cerebelDuchenne muscular dystrophy.
ow some viral infections by 2 to 3

Viral infections ha
during attacks of a
lar ataxia may foll
weeks and is though

t to be autoimmune in origin.
These children pres
ent with horizontal nystagmus,
NEURODEGENERATIVE
ting, and occasionally dysarDISORDERS
nuchal rigidity are absent, and

postural ataxia, vomi


thria. Headache and
examination of the

CSF is negative.
Ataxia-telangiec
tasia is an autosomal recessive
Neural tissue degeneration can occur at any level of
isorder that presents in toddlers
the nervous system, from the brain cell bodies to the
heelchair dependence. The ataxia
peripheral nerves. Many of the diseases are inherited;
most are progressive and debilitating.
extensive telangiectasis and

neurodegenerative d
and progresses to w

is associated with
immunodeficiency (s

ee Chapter 11).
Friedreich's ata
xia presents later in childhood
Clinical Manifestations and Treatment
axia, weakness, and muscle

with progressive at
wasting. Skeletal d

eformities invariably follow. Most


Neurodegenerative disorders may be divided into
disorders resulting from gray or white matter degendiomyopathy-related heart disease

patients die of car


before the age of 3

0.
eration. Gray matter disorders, which include
etabolic derangements, cerebellar
Tay-Sachs, Gaucher's, and Niemann-Pick diseases,
mors may also cause ataxia.
result from lipid buildup in neuronal cell bodies.

Intoxications, m
hemorrhages, and tu
Otitis media is a f

requent cause of ataxia in young


Hypotonia, mental retardation, and seizures are
common. White matter disorders (leukodystrophiesj

children.

are inherited progressive degenerative diseases resultDiagnostic Evaluati


on
ing from abnormal myelin formation and conduction. They present in younger patients with spasticity
out hydrocephalus, mass lesions,
and developmental milestone loss; older children and
rrhages. A brain MRI is prefer-

Neuroimaging rules
and cerebellar hemo

--------------------------------------- 139
Chapter 15 / Neurology
223
able to head CT, given its superior detail of posterior

TABLE 15-7

fossa structures. Patients with a fever should receive


a lumbar puncture to evaluate for infection. Toxicofibromatosis Type 1
logic screens of blood and urine should be obtained

Diagnosis of Neuro

Two of the followi


ng must be present.
in all cases of acute ataxia. Chronic or recurrent
fe au lait spots, >5 mm in size in
ataxia warrants metabolic and genetic workup.
5 mm in adolescents or adults

1. Six or more ca
children and >1
2. Axillary or in

guinal freckling
3. Two or more Li
sch nodules (hamartomas) in the iris
KEY POINT
rofibromas or one plexiform

4. Two or more neu


neurofibroma

1. The differential diagnosis for ataxia (incoordinasseous lesion, such as sphenoid


tion) includes labyrinthitis, acute cerebellar ataxia,
ataxia-telangiectasia, and Friedreich's ataxia.

5. A distinctive o

dysplasia
6. Optic gliomas
7. Affected first

-degree relative diagnosed based on


the above crite
ria
PHAKOMATOSES
Phakomatoses are neurocutaneous diseases charac-

terized by lesions in the nervous system, skin, and


eyes. Three autosomal dominant conditions are
sions include ash-leaf spots (flat,
described: neurofibromatosis, tuberous sclerosis, and
s), shagreen patches (areas of
von Hippel-Lindau disease. Sturge-Weber disease, a
ning), sebaceous adenomas, and
sporadic disorder, is traditionally included as well.
ar forehead lesions. Neu-

der. Typical skin le


hypopigmented macule
abnormal skin thicke
hyperpigmented macul
roimaging demonstrat

es the distinctive periventricuClinical Manifestations and Treatment


of localized swelling, or "tubers."

lar knob-like areas


Mental retardation a

nd seizures are common. Tumors


of the kidney and he
art, particularly cardiac rhabNeurofibromatosis
common. Treatment consists of
Of the several variants of neurofibromatosis, types 1
y and surgical removal of related
(von Recklinghausen's disease; Table 15-7) and 2
d.
(bilateral acoustic neurofibromatosis) are the most

domyomas, are not un


antiepileptic therap
tumors when indicate

common in children. Patients with von Recklinghausen's disease should receive treatment for the
sease
associated seizures, learning disorders, renovascular
sease is characterized by retinal
hypertension, and scoliosis. Neurofibromas that cause
(usually unilateral), similar vasimpairment may be surgically removed; however,
central nervous system, and assomost will recur.
cluding renal cell carcinoma and
Bilateral acoustic neuromas are the hallmark of
ular lesions respond to laser
type 2 neurofibromatosis. Complications include
treatment exists for the CNS
hearing loss and vestibular disorientation. Brain MRI

von Hippel-Lindau Di
von Hippel-Lindau di
vascular hamartomas
cular lesions in the
ciated neoplasms, in
pheochromocytoma. Oc
therapy; no specific
growths.

demonstrates bilateral eighth cranial nerve masses.


Neurofibromas, meningiomas, schwannomas, and
astrocytomas are also associated with type 2 neurofi-

Sturge-Weber Disease

bromatosis. Cataracts and retinal hamartomas are not


is a progressive neurologic disuncommon. Surgical debulking is appropriate when
h a port-wine stain (nevus flamhearing impairment becomes pronounced. Cochlear
innervated by the first division
implants have restored hearing in some patients.
rve. Affected children manifest

Sturge-Weber disease
order associated wit
meus) over the area
of the trigeminal ne
mental retardation,

seizures, and visual impairment;

Tuberous Sclerosis
p glaucoma. Laser therapy may
Tuberous sclerosis, like neurofibromatosis, is a prostain but does not address the
gressive autosomal dominant neurocutaneous disoron.

about a third develo


"fade" the port-wine
neurologic dysfuncti

--------------------------------------- 140
224

rints Pediatrics
motor delay; associa

ted seizure disorders are not


KEY POINTS
1. Neurofibromatosis type 1 is characterized by mulcontrast, refers to a head cirtiple cafe au lait spots on examination.
han 2 standard deviations above
2. In contrast, the typical skin lesions of tuberous
ly may be the result of a large
sclerosis include ash-leaf spots and shagreen
ioskeletal dysplasias, storage
patches.
ephalus should be explored as
3. Sturge-Weber disease is associated with a portwine stain over the area innervated by cranial
is the premature fusion of one
nerve V, first division (CNV,).
res. It may be idiopathic or occur

uncommon.
Macrocephaly, in
cumference greater t
the mean. Macrocepha
brain; however, cran
diseases, and hydroc
possible causes.
Craniosynostosis
or more cranial sutu
as part of a syndrom

e. Bony growth continues along


the open sutures, re
sulting in an abnormally shaped
head. If early oblit
eration of the sagittal suture
occurs, the child wi
ll have a long head and a narrow
il.SKy.LLAB_NORMALITIES
). In contrast, premature closure

face (dolichocephaly
of the coronal sutur

es results in a very wide face with


Microcephaly describes a head circumference that is
like, skull. The need for and
greater than 2 standard deviations below mean head
ntervention, which consists of
size for age. It often results from genetic abnormalis and retarding their subsequent
ties (e.g., trisomy 21, Prader-Wiili syndrome) or consial. Most defects are repaired
genital insults (maternal drug ingestions, congenital
or cosmetic reasons; those threatinfections, or insufficient placental blood flow).
rowth and development are
Affected children demonstrate both cognitive and

--------------------------------------- 141

a short, almost boxtiming of surgical i


reopening the suture
fusion, is controver
before age 2 years f
ening normal brain g
addressed sooner.

Chapter
Good nutrition is necessary for optimal physical
ied formulas provide appropriate
growth and intellectual development. A healthy diet
ients. Premature infants (<32 weeks)
protects against disease, provides reserve in times of
cifically designed for them, or
stress, and contains adequate amounts of protein,
added fortifier. Newborns feed on
carbohydrates, fats, vitamins, and minerals. Children
very 1 to 2 hours. Neonates norwith vegetarian diets are at risk for vitamin Bi
2 and
10% of their birth weight over the
trace mineral deficiencies. Failure to thrive, obesity,
s; formula-fed babies regain their
and infant feeding intolerance are the most common
he second week of life, whereas
pediatric conditions associated with malnutrition.
may take about a week longer.
In order to assess a patient's nutritional statuHealths
automatically regulate intake to meet
and growth, pediatricians rely on following a
patient's growth chart. Growth charts represent
mulas contain the recommended
cross-sectional data from the National Center for
s and minerals. However, at 4 to 6
Health Statistics of the Centers for Disease Control
n-fortified cereals may be added
and Prevention. Separate growth charts are generated
t. After 6 months of age, other baby
for premature infants and infants with certain genetic
ted, including fruits and vegetables.
disorders, including Down syndrome and Turner's
new foods, only one new product
syndrome. A child's growth should be assessed oveshoulr
uced at a time to look for potential
time. A change in weight greater than two percentile
. Infants 6 months and older may
lines over a 3- to 6-month period should be evalurequirsupplementation, depending on the
ated. Many children will cross percentiles during 9 to
fluoride in their tap water. Whole
18 months of age, as growth begins to be based morcow'e
e introduced at 12 months and
on genetic potential than on maternal nutrition prioshoulr
until 24 months, when skim milk
to birth.
uted.
Pediatricians also assess nutritional status by calculating the ideal body weight (IBW). When the
patient's actual body weight is greater than 20% over
IBW, the patient is considered obese; less than 70%
emy of Pediatrics recommends
of IBW represents severe body wasting.
eeding during the first 6 months of

pared iron-fortif
calories and nutr
need formulas spe
breast milk with
demand, usually e
mally lose up to
first several day
birth weight by t
breast-fed babies
y infants
caloric demand.
All infant for
amount of vitamin
months of age iro
to the infant die
foods may be star
When introducing
d be introd
adverse reactions
e fluoride
concentration of
s milk may b
d continue
should be substit

Breastfeeding
The American Acad
exclusive breastf
life and continua

tion of breastfeeding during the


second 6 months f
or optimal infant nutrition. Studies
*, J.N.FA_NT FEEDING ISSUES__

have shown that b

reast-fed infants have a lower


incidence of infe
ctions, including otitis media, pneuInfant feeding addresses the physical and emotionamonial
ia, and meningitis. Human milk conneeds of both mother and child. Babies triple in
nd viral antibodies (secretory IgA]
weight during the first year. Although breastfeeding
Lactoferrin is a protein found in
is strongly recommended, many commercially preincreases the availability of iron and

, bacterem
tains bacterial a
and macrophages.
breast milk that

--------------------------------------- 142
226
Blueprints Pediatrics
has an inhibitory effect on the growth of Escherichia
Y POINTS
coli. Breast-fed infants in particular may need fluoride supplementation after 6 months. Also, infants
lly lose weight, but should regain
with rare sunlight exposure may be at risk for rickets
by the third week of life.
if the maternal intake of vitamin D is inadequate. In
ein intolerance can lead to feeding
developed countries, mothers with human immuaversion.
nodeficiency virus (HIV] infection or untreated
ture and sudden onset of colic
active tuberculosis or those who are using illegal
uish this condition from feeding
drugs should not breastfeed. Other contraindications
include infants with galactosemia and some maternal
ademy of Pediatrics recommends
medications.
tfeeding during the first 6 months

KE
1. Newborns initia
to birth weight
2. Cow's milk prot
intolerance and
3. The sporadic na
usually disting
intolerance.
4. The American Ac
exclusive breas
of life.

Infant Feeding Intolerance


Feeding intolerance may lead to food aversion and
failure to thrive; the most significant cause is cow's
milk protein intolerance or allergy.
VE
Clinical Manifestations

FAILURE TO THRI
Failure to thrive (F

TT) is defined here as persistent


History and Physical Examination
rd percentile or falling off the
Feeding intolerance may present with any number ogrowtf
factors include low birth weight,
clinical manifestations. Malabsorption is characterstatus, physical or mental disized by poor growth and chronic diarrhea. Colitisability,
er neglect.
indicated by anemia or obvious blood in the stools,
can occur. Allergy may be accompanied by eczema
or wheezing. Other possible symptoms include vomiting, irritability, and abdominal distention.
is
Differential Diagnosis

weight below the thi


h curve. Risk
lower socioeconomic
, and caretak

Differential Diagnos
Most cases of FTT in

developed countries are nonorganic or psychosocia


l in origin; that is, there is no
Infectious gastroenteritis, necrotizing enterocolitis,
coexistent medical d
isorder. The list of organic diagintussusception, intermittent volvulus, celiac diseasenose,
s predisposing
to FTT is extensive, and virtually
cystic fibrosis, chronic protein malnutrition, aspiraall organ systems ar
e represented (Table 16-1).
tion, and eosinophilic enteritis should be considered.
The most common condition mistaken for milk
protein intolerance is colic, which is generally limited
Clinical Manifestati
ons
to infants younger than 3 months. Colic is a synHistory
drome of recurrent irritability that persists for several
hours, usually in the late afternoon or eveningTh.
e caretaker must
be questioned in detail about the
During the attacks, the child draws the knees to thchild'e
s diet, includ
ing how often the child eats, how
abdomen and cries inconsolably. The crying resolvemucs
h at each feedi
ng, what the child is fed, how the
as suddenly and spontaneously as it begins.
formula is prepared,
and who feeds the child. Information regarding dia
rrhea, fatty stools, irritability,
Treatment
vomiting, food refus
al, and polyuria should be docuExclusive breastfeeding during the first year of life
mented. Recurrent in
fections suggest congenital or
eliminates the problem posed by milk protein intolacquired immunodef
iciency. Constitutional growth
erance, except in severely allergic infants. If there is
delay can usually be
diagnosed by family history
no evidence of any underlying disease, many pediaalone. Foreign and
domestic travel, source of water,
tricians recommend a trial of casein hydrolysate forand developmental de
lay are occasionally overlooked
mulas (e.g., Nutramigen or Pregestimil). Because as
topics. The psychoso
cial history includes questions
many as 25% of children with milk protein allergy
concerning the caret
aker's expectations of the child,
are also intolerant of soy, the casein hydrolysate forparental and sibli
ng health, financial security, recent
mulas are a better alternative than soy milk.
major life events, a
nd chronic stressors.

--------------------------------------- 143
Chapter 16 / Nutrition

227
weight alone, wherea

s height and head circumferTABLE 16-1


ed in chronic deficiency. Severely
Differential Diagnosis of Failure to Thrive
y present with lethargy, edema,

ence are also affect


deprived children ma
scant subcutaneous f

at, atrophic muscle tissue,


Cardiac
Congenital heart malformations
r, coarsened hair, dermatitis, and
Endocrine
Diabetes mellitus
retaker-child interaction and
Hypothyroidism
critical. Children who are listless,
Hyperaldosteronism
to the examiner and/or careGastrointestinal
excessively fearful often have
Malabsorption
ocial issues. Findings suggestive
Milk protein intolerance/allergy
neglect (see Chapter 2) should
Gastroesophageal reflux
nted.
Pyloric stenosis
Celiac disease
al examination, with careful

decreased skin turgo


distended abdomen.
Observation of ca
feeding behavior is
minimally responsive
taker, withdrawn, or
contributing psychos
of physical abuse or
be sought and docume

A complete physic
attention for dysmor

phism, pallor, bruising, cleft


Infectious
ckles, heart murmurs, and muscle
HIV
e etiology.
Chronic gastroenteritis

palate, rales or cra


tone, may suggest th

Intestinal parasites
Urinary tract infection

Diagnostic Evaluatio

Neonatal
from the history and physical
Prematurity

Information obtained

Low birth weight


ion of further diagnostic
Congenital or perinatal infection
th FTT should receive a comNeurologic
erum electrolytes, blood urea
Cerebral palsy
ine, and protein and albumin
Mental retardation
ly malnourished children and
Degenerative disorders
ted nonorganic FTT should be
Pulmonary
ital. Adequate catch-up growth
Cystic fibrosis
on on a regular diet is virtually
Bronchopulmonary dysplasia
social FTT.
Chronic aspiration

determine the direct


workup. Any child wi
plete blood count, s
nitrogen and creatin
measurements. Severe
patients with suspec
admitted to the hosp
during hospitalizati
diagnostic of psycho

Respiratory insufficiency
Renal
Y POINTS
Renal tubular acidosis

KE

Chronic renal insufficiency


ht below the third percentile and
Nonorganic
growth curve are both evidence of
Neglect
ve.
Psychosocial
TT in developed countries are
Abuse

1. Consistent weig

Inadequate amount fed


m may be implicated in organic
Incorrect preparation of formula

3. Any organ syste

Other
al exam and screening tests should
Inborn errors of metabolism
search.
Malignancy

falling off the


failure to thri
2. Most cases of F
nonorganic.

FTT; the physic


help focus the

Cleft palate
Congenital immunodeficiency syndromes
OBESITY
Obesity, defined as
actual weight at least 120% of
Physical Examination
s at epidemic proportions in the
Weight, height, and head circumference should be
The cause is simply caloric
plotted out on an appropriate growth chart. Relaexpenditure. The social and psytively recent growth failure is usually limited to
ces of being a "fat" child may be

ideal body weight, i


United States today.
intake in excess of
chological consequen

--------------------------------------- 144
228 Blueprints Pediatrics
particularly damaging to self-esteem at a critical age.
increases the risks
of gallbladder disease, cardiovasObesity is treated by altering dietary habits (limiting
cular disease, and h
ypertension. Sleep apnea, slipped
intake of high-calorie, high-fat foods) and developing
capital femoral epip
hysis, and early-onset puberty in
a regular exercise program. Rather than losing weighfemalet
s are other p
ossible childhood complications.
(which may compromise growth), the goal for overweight children is slowing weight gain until they are

back within the normal growth curve. Patients who


KEY POINT
are morbidly obese, obese adolescents, and children
of obese parents are more likely to become obese
in childhood type 2 diabetes
adults. The recent rise in childhood type 2 diabetes
asing childhood obesity.
reflects increasing childhood obesity. Obesity also

1. The recent rise


parallels incre

--------------------------------------- 145
Oncology
LEUKEMIA
e of this review text. The fol-

*^ x
goes beyond the scop
lowing discussion fo

cuses on ALL and AML.


The leukemias account for the greatest percentage of
by both morphologic and immunocases of childhood malignancies. There are 3000 new
ologic classification is based on
cases of leukemia each year in the United States, and
e lymphoblasts. The LI type lymapproximately 40 children per million are affected.
common (85% of cases) and has
Table 17-1 lists types of childhood cancer and the
s. The L2 type lymphoblast (14%
fraction of the total childhood malignancies that each
type lymphoblast (1% of cases)
accounts for annually.
gnoses. Immunologic classifica-

ALL is classified
logic methods. Morph
the appearance of th
phoblast is the most
a favorable prognosi
of cases) and the L3
have unfavorable pro
tion is based on imm

unophenotype. Non-T, non-B cell


ALL accounts for 80%
of cases and has a good
Pathogenesis
L, which is responsible for 19% of
Leukemia results from malignant transformation
e prognosis, and B-cell ALL, which
and clonal expansion of hematopoietic cells that have
cases, has a very poor prognosis.
stopped at a particular stage of differentiation and are
into seven subtypes by morphounable to progress to more mature forms. Leukemias
cal information using the Frenchare divided into acute and chronic subtypesAmerican-Britis.
FAB) classification system: Ml is
Leukemias are further classified on the basis
a without differentiation, M2
of leukemic cell morphology into lymphocytic
emia with differentiation, M3 is
leukemias (lymphoid lineage cell proliferation) and
ia, M4 is myelomonocytic leunonlymphocytic leukemias (granulocyte, monocyte,
stic leukemia, M6 is erythro-

prognosis. T-cell AL
cases, has a variabl
accounts for 1 % of
AML is classified
logic and histochemi
h (
myeloblastic leukemi
is myeloblastic leuk
promyelocytic leukem
kemia, M5 is monobla

erythrocyte, or platelet lineage cell proliferation).


megakaryoblastic leukemia.
Acute leukemias constitute 97% of all childhood
leukemias and are subdivided into acute lymphocytic leukemia (ALL) and acute nonlymphocytic
leukemia, also known as acute myelogenous
pediatric neoplasm,
leukemia (AML). If untreated, they are rapidly fatal
all cases of childhood acute
within weeks to a few months of diagnosis, but with
times more common in males
treatment they are often curable. Chronic leukemias
more common in white children
make up 3% of childhood leukemias. Chronic
ican children. The incidence of
leukemias in children are always nonlymphocytic.
and 5 years of age.
Unlike those with acute leukemias, these patients
20% of all cases of childhood
may survive without treatment for many months to
s more common in males than
years. Unfortunately, the chronic leukemias evolve
mon in African-American
into forms of acute leukemia that cannot be cured
te children. The incidence of
by available chemotherapy. Because they are so rare
ALL, is fairly constant from
in children, a discussion of the chronic leukemias
hildhood.

leukemia, and M7 is
Epidemiology
ALL, the most common
accounts for 80% of
leukemia. ALL is 1.3
than in females and
than in African-Amer
ALL peaks between 3
AML accounts for
acute leukemia. It i
females and more com
children than in whi
AML, in contrast to
birth through late c

--------------------------------------- 146
Chapter 3 / Cardiology

23
be performed to asse

ss the coronary arteries. If the


coronary circulation
is not RV dependent, then a
right ventricle to p
ulmonary artery conduit or pulmonary valvotomy is
performed to provide antegrade
pulmonary blood flow
. A modified Blalock-Taussig
shunt is also typica
lly performed to augment pulmonary blood flow fu
rther. Depending on the growth
of the right ventric
le and tricuspid valve, a single ventricle, one and a ha
lf ventricle, or two ventricle repair
may be possible. If
the coronary circulation is RV
dependent, the RV is
not decompressed and a modified Blalock-Taussi
g shunt is performed. After mod-

ified Blalock-Taussi
g shunt placement, patients with
a right ventricle de
pendent coronary circulation are
either listed for he
art transplantation or staged to a
Fontan palliation.
Tetralogy of Fallot
Tetralogy of Fallot
(Figure 3-6) is the third most
prevalent cyanotic c
ongenital heart lesion during the
Figure 3-5
Pulmonary atresia with intact ventricular septumneonatal period and a
fter the third week of life
(PA/IVS) in a neonate with a nonrestrictive patent ductus arte-becomes the leadi
ng cause of cyanosis due to conriosus while receiving PGE,. Typical anatomic and hemody- genital heart diseas
e in childhood. The four defects
namic findings include: (a) hypertrophied, hypoplastic right
ventricle; (b) hypoplastic tricuspid valve and pulmonary
an anterior malalignment ven-

Fallot noted include

tricular septal defe


ct, right ventricular outflow tract
annulus; (c) atresia of the pulmonary valve with no antegradeobstruction (50% in
fundibular stenosis, 20% pulflow; (d) suprasystemic right ventricular pressure; (e) pulmonary blood flow via the patent ductus; (f) right-to-left shuntmonary valve ste
nosis, and 30% infundibular stenoat the atrial level with systemic desaturation. Many patientssis and pulmonary v
alve stenosis), right ventricular
have significant coronary abnormalities with sinusoidal or fis-hypertrophy, and
an "overriding" large ascending
tulous connections to the hypertensive right ventricle or sig-aorta.
nificant coronary stenosis (not shown).
Cloherty JP, Stark AR. Manual of Neonatal Care, 4th ed. Philadelphia:
Lippincott-Raven, 1998:426.
ons

Clinical Manifestati
Neonates with tetral

ogy of Fallot are cyanotic


because of right-toleft shunting across the ventricular septal defect an
d decreased pulmonary flow.
Shunting occurs when
the combination of the pulClinical Manifestations
stance and the resistance created
Neonates present at birth extremely cyanotic and
ular outflow tract obstruction
tachypneic. Cardiac examination reveals a tricuspid
l vascular resistance. The degree
regurgitation murmur in the left lower sternal border
rtional to the severity of the right

monary vascular resi


by the right ventric
exceed the periphera
of cyanosis is propo

and the continuous murmur of a patent ductus


tract obstruction. Blood shunted
arteriosus. On ECG, left ventricular hypertrophy anfrod
e pulmonary artery through the
a leftward axis are seen. On chest radiograph,
osus provides additional puldecreased pulmonary markings and left ventricular
eonates present with cyanosis
hypertrophy are seen.
and may have characteristic peri-

ventricular outflow
m the aorta to th
patent ductus arteri
monary blood flow. N
of varying severity
odic episodic cyanos

is and agitation. These episodes


Treatment
n as "tet spells." Tet spells are
PGEi should be started to ensure pulmonary blood
e in right ventricular outflow
flow initially. Prior to any surgery to provide more
ading to an increase in the rightstable pulmonary flow, a cardiac catheterization must
lls may last minutes to hours, may

of cyanosis are know


caused by an increas
tract resistance, le
left shunt. Such spe

--------------------------------------- 147
230

Blueprints Pediatrics

TABLE 17-1
nd anorexia. Approximately 25%

lethargy, malaise, a
of children complain

of bone pain or arthralgias


Distribution of Childhood Cancer by Diagnosis
caused by either leu
kemic infiltration of the perichondral bone or leu
kemic expansion of the marrow
Cancer

Percentage of Total
Pediatric Malignancies

cavity. Progressive

Annually

pallor, ecchymoses o

bone marrow failure leads to


r petechiae (50%), and fever
ALL 23.3
prompt diagnostic evaluation.

(25%), features that


Many children have h

epatosplenomegaly at diagnoCentral nervous system 20.7


Neuroblastoma 7.3
involvement may be seen in the

sis. Extramedullary
central nervous syst

em (CNS), skin, or testicles. CNS


Non-Hodgkin's lymphoma 6.3
Wilms' tumor 6.1
neurologic signs and symptoms,
Hodgkin's lymphoma 5.0
esis, papilledema, and sixth
AML 4.2

infiltration causes
such as headache, em
cranial nerve palsy.

Patients with AML may develop


Rhabdomyosarcoma 3.4
called a chloroma in the spinal
Retinoblastoma 2.9
The lesions have a greenish hue
Osteosarcoma 2.6
nce of myeloperoxidase in the
Ewing's sarcoma 2.1

a soft-tissue tumor
cord or on the skin.
because of the prese

tumors. Of note, pro


myelocytic leukemia (M3 type
Other 16.1
AML) is associated w
ith disseminated intravascular
ALL, acute lymphocytic leukemia; AML, acute myelogenous leukemia.coagulation,
and monoblastic leukemia (M5 type
Adapted from Gurney JG, Severson RK, Davis S, Robinson LL. IncidenceAML) is as
sociated with central nervous system
of cancer in children in the United States. Sex-, race-, and 1-year
age-specific rates by histologic subtype. Cancer 1995;75:2186.involvement and
gingival hyperplasia.
Leukemic dissemi
nation results in bone marrow
failure, reticuloend
othelial system infiltration, bony
involvement, and pe
netration of sanctuary sites. Bone
marrow failure resu
lts from myelophthisis, which
Risk Factors
is the replacement
of the normal hematopoietic
Syndromes with an increased risk for leukemia
row by the leukemic cell
include trisomy 21, Fanconi's anemia, Bloom's syn-

elements in the mar


population.

drome (a chromosomal breakage disorder), ataxiatelangiectasia, X-linked agammaglobulinemia, and


severe combined immunodeficiency. Identical twins
sis
have a 20% risk of leukemia if one twin develops it
agnosis includes aplastic anemia,
during the first 5 years of life. Children with solid

Differential Diagno
The differential di

idiopathic thromboc
ytopenic purpura, Epstein-Barr
tumors, especially Hodgkin's disease and Wilms'
her malignancies, and virus-induced
tumor, who have undergone intense radiation and/or

virus infection, ot

or familial hemopha
gocytic syndromes. Rarely, a
chemotherapy with alkylating agents may develop
isease or rheumatologic disorder
leukemia as a secondary malignancy. Children with

collagen vascular d

mimics the presenti

ng symptoms of leukemia.
congenital bone marrow failure states, such as
Shwachman-Diamond syndrome (exocrine pancreatic insufficiency and neutropenia) and DiamondDiagnostic Evaluati
on
Blackfan syndrome (congenital red cell aplasia), have
an increased risk of leukemia.
ytopenia are present at diag-

Anemia and thromboc


nosis in 90% of cas

es. The anemia is normochromic


The risk factors for AML are the same as those for
ALL. The presence of Fanconi's anemia results in a
reased marrow production of

and normocytic. Dec


red blood cells lea

ds to a low reticulocyte count. The


higher risk for AML than for ALL.
white blood cell co
unt is low (<5000/mm

3) in onethird of patients,

normal (5000-20,000/mm
Clinical Manifestations
ts, and high (>20,000/mm

3) in
one-third of patien
3) in
one-third of patien

ts. Blast cells are frequently seen


History and Physical Examination
, especially if the white blood cell
Symptoms usually develop less than 4 weeks before
high. Bone marrow examination
diagnosis. Initial nonspecific symptoms include
f there are blasts in the peripheral

on peripheral smear
count is normal or
is critical, even i

--------------------------------------- 148
Chapter 17 / Oncology
231
blood, because the morphology of the peripheral
drome is rarely seen
in solid tumors. Acute lysis of
blasts may not reflect the true bone marrow mortumor cells results
in the rapid release of intraphology. It is possible to identify human lymphocytes
cellular contents in
to circulation. This leads to
and granulocytes at different stages of developmenhypocalcemiat
, hy
perphosphatemia, hyperkalemia,
by using specific monoclonal antibodies to define cell
and hyperuricemia. H
yperkalemia can cause cardiac
surface antigens. When this application is combined
arrhythmias. Phospha
te, especially at high serum
with cytochemical histology, molecular probes, and
levels, binds to cal
cium, resulting in precipitation of
cellular morphology, the diagnostic classification,
calcium phosphate in
renal tubules, hypocalcemia,

treatment, and prognosis become more specific.


are processed to uric acid. Hyper-

and tetany. Purines


uricemia can result

in precipitation of uric acid in


renal tubules and re
nal failure. Management of tumor
lysis syndrome inclu
des vigorous hydration, urine
Treatment
acid reduction with allopurinol,
The treatment strategy in leukemia is to (1) treat the
d phosphate reduction. The risk
complications of the leukemia at presentation, (2)
reatest during the first 5 days of
treat the leukemia, and (3) manage the complications
of treatment.
eukemic therapy is instituted in
Managing leukemic complications at presentation
s, each with specific objectives.
involves blood product transfusions and treatment of
on generally lasts 4 weeks, during
infection, hyperviscosity, compressive symptoms, and
ll is achieved. If remission is
metabolic abnormalities. Blood products require irrawill disappear from the bone
diation before infusion to remove donor lymphocytes
lete blood count values will
that may mount a graft-versus-host response against
e goals of consolidation are to
the immunocompromised leukemic host. Neutropeemic cells with further systemic
nia, defined as an absolute neutrophil count less than
nt leukemic relapse within
500/mm 3, predisposes children to serious bacterial
system by giving intrathecal
and fungal infection. The development of fever in a
jectives of maintenance
child with neutropenia warrants careful evaluation
nue the remission achieved in the
for bacteremia or sepsis. A white blood cell count
and to provide additional
greater than 200,000/mm
3 can cause significant
e the leukemia. Discontinuation
hyperviscosity. This is often seen in children with
rs when the patient has
AML whose white blood cell count is greater than
n throughout the prescribed
200,000/mm
3, and in children with ALL whose
e therapy. At the conclusion of
white blood cell count is greater than 400,000/mm
a relapse-free patient is conWithout therapy, hyperviscosity may cause hypoxpatients successfully completemia or stroke from sludging in the lungs and CNS,
phase will have a recurrence of
respectively. The cell count may be lowered using
tial relapse in the bone marrow,
exchange transfusion or leukopheresis. Large collecem, or testes.

alkalinization, uric
diuretic therapy, an
for tumor lysis is g
chemotherapy.
In general, antil
three distinct phase
Induction of remissi
which maximum log ki
achieved, all blasts
marrow, and the comp
return to normal. Th
kill additional leuk
therapy and to preve
the central nervous
chemotherapy. The ob
therapy are to conti
previous two phases
cytoreduction to cur
of chemotherapy occu
remained in remissio
course of maintenanc
3.maintenance therapy,
sidered cured. A few
ing the maintenance
leukemia, with poten
central nervous syst

tions of malignant cells in the mediastinum, common


motherapy is successful in
in T-cell leukemia, compress vital structures, causing
dnisone, vincristine, and Ltracheal deviation or superior vena cava syndrome.
erally used, and depending on
Superior vena cava syndrome is characterized by disin question is of standard or
tended neck veins; swelling of the face, neck, and
nts may be used. In high-risk
upper limbs; cyanosis; proptosis; and Horner's syncline is generally added. Condrome. The mass and the compressive symptoms
intrathecal methotrexate and, in
it creates usually resolve with chemotherapy and
cranial irradiation. Cranial irradiaradiation.
disabilities, especially in young
Tumor lysis syndrome describes a constellation of
somnolence syndrome; and brain
metabolic abnormalities resulting from spontaneous
nces. Maintenance therapy
or treatment-induced tumor necrosis, and is generaptopurine and intramuscular
ally seen in tumors with high growth rates such as
ally lasts 2 years. Local tissue
T-cell ALL or Burkitt's lymphoma. Tumor lysis syne central nervous system or testes

ALL induction che


95% of children. Pre
asparaginase are gen
whether the leukemia
high risk, other age
leukemia an anthracy
solidation includes
high-risk patients,
tion causes learning
children; transient
tumors in rare insta
involves oral 6-merc
methotrexate and usu
relapse of ALL in th

--------------------------------------- 149
.232

Blueprints Pediatrics

is treated with local irradiation and re-induction


chemotherapy. AML chemotherapy is more intensive
EY POINTS
than that used for ALL, and induction regimens tend
account for the greatest percentto include an anthracycline-like drug and cytosine
childhood malignancies.
arabinoside. Myelosuppression is severe, and good
lassified on the basis of leukemic
supportive care is essential. Eighty percent of
into lymphocytic leukemias,
patients with AML achieve initial remission after
ferations of cells of lymphoid
induction chemotherapy, but most patients will
onlymphocytic or myelogenous
relapse within a year.
ch are proliferations of cells of granPrognostic factors have been identified that place
yte, erythrocyte, or platelet lineage.
children with ALL into either the standard-risk or
constitute 97% of all childhood
high-risk category (Table 17-2). Children with stanare subdivided into acute lymphodard-risk ALL have a more favorable prognosis and

K
1. The leukemias
age of cases of
2. Leukemias are c
cell morphology
which are proli
lineage, and n
leukemias, whi
ulocyte, monoc
3. Acute leukemias
leukemias and
cytic leukemia

and acute myelogenous leukemia.


require less intensive therapy. Overall, the initial
common pediatric neoplasm and
white blood count and the age of the patient are the
0% of all cases of childhood acute
most significant variables. L2 or L3 FAB classification

4. ALL is the most


accounts for 8

and the presence of massive organomegaly T-cell


90% of children with leukemia have
leukemia, and central nervous system disease all indiombocytopenia at presentation.
cate poor prognosis.
erapy is instituted in three
In general, the prognosis for AML is worse than
s: induction, consolidation, and
that for ALL. Prognoses vary between subtypes. The

leukemia.
5. Approximately
anemia and thr
6. Antileukemic th
distinct phase
maintenance.

best chemotherapeutic regimens are curative for less


e prognosis for AML is worse than
than one-half of patients with AML. Bone marrow
Standard-risk ALL has an 80% cure
transplantation is curative in up to two-thirds of
the prognoses for AML vary widely
patients with AML.
.
TABLE 17-2
Prognostic Factors in Acute Lymphoblastic
Factor*

7. In general, th
that for ALL.
rate, whereas
among subtypes

Leukemia of Childhood
Favorable (Standard Risk)

Unfavorable (High Risk)


Demographic
Age (years)
<2,>10
Race
African American
Sex
Male
Leukemic Burden

2-9

Initial white blood cell count (per mm3}


>50,000
Hemoglobin (g/dL)
>10
Platelet count (per mm3)
< 100,000
Adenopathy
Present
CNS disease at diagnosis
Present
Mediastinal mass
Present
Hepatosplenomegaly
Marked (>3cm)
LDH
High
Immunologic Factors

< 10,000

Immunoglobulins

Normal IgA, IgG, IgM

White
Female

<7
> 100,000
Absent
Absent
Absent
Mild (<3 cm)
Not high

Low IgA, IgG, IgM


Surface markers
Non-T, non-B cell ALL
T- or B-cell ALL or pre-B
Glucocorticoid receptors
High number
Low number
Response to induction
M2 marrow (5% blasts) on
M3 marrow (25% blasts)
therapy
day 14
on day 14
* Other prognostic factors include cell morphology, histochemistry, cytogenet
ics, and biochemistry.
ALL, acute lymphocytic leukemia; CNS, central nervous system;LDH, lactate deh
ydrogenase.

--------------------------------------- 150
Chapter 17 / Oncology
233
I CENTRAL NERVOUS
omas, or germ cell tumors),
SYSTEM TUMORS
ors in adults are malignant

loblastomas, ependym
whereas most CNS tum
astrocytomas and met

astatic carcinomas.
Central nervous system tumors are the most
common solid tumors in children and are second to
ons
leukemia in overall incidence of malignant diseases.

Clinical Manifestati

In contrast to adults, in whom supratentorial brain


and symptoms of CNS tumors
tumors are more common, brain tumors in children
the child and location of the
are predominantly infratentorial (posterior fossa),
Any CNS tumor may cause
involving the cerebellum, midbrain, and brainstem.
al pressure (ICP) by obstructing
Table 17-3 denotes the location, clinical manifestaflow. Symptoms of increased ICP
tions, and prognosis of CNS tumors in children.
g headaches, vomiting, and
Childhood brain tumors are differentiated further
he is usually present upon awakfrom those in adults in that they are usually lowstanding, and worsens with
grade astrocytomas or embryonic neoplasms (medulg. It is intermittent but recurs
TABLE 17-3

The presenting signs

Location and Manifestations


Tumor Age at
Comments

depend on the age of


tumor (Table 17-3).
increased intracrani
cerebrospinal fluid
include early mornin
lethargy. The headac
ening, improves with
coughing or strainin

of Primary CNS Tumors


Manifestations* 5-Year

Onset

Su

(yr)

(%

rvival
)

Infratentorial
Cerebellar
5-8
20% of all primary
astrocytoma
CNS tumors
Medulloblastoma
3-5
Acute onset of

Ataxia; nystagmus; head tilt; intention 90


tremor
Obstructive hydrocephalus; ataxia, 50
CSF metastasis; spinal cord

symptoms; 20% of all


compression
primary CNS tumors
Ependymoma
2-6
25%-40% supratentorial

Obstructive hydrocephalus; rarely 50


seeds spinal fluid

Brainstem glioma
5-8
Worst prognosis of all

Progressive cranial nerve dysfunction; 30


gait disturbance; pyramidal tract

childhood CNS
and cerebellar signs
tumors
Supratentorial
Cerebral

5-10
Patient may become
astrocytoma
obese after treatment
Craniopharyngioma
7-12
Calcification above

Seizures; headache; motor weakness; 1 0-50


personality changes
Bitemporal hemianopsia; sexual and 70-90
growth retardation

sella turcica;
postoperative diabetes
insipidus common
Optic glioma
<2
Neurofibromatosis in

Poor visual acuity; exophthalmos; 50-90


nystagmus; optic atrophy;

25% of patients
strabismus
Pinealoma

Paralysis of upward gaze (Parinaud's 75


Germ cell line:
syndrome); lid retraction (Collier's
may calcify or
sign); hearing loss; precocious
secrete hCG or

puberty; may seed spinal fluid


alpha-fetoprotein
* All CNS tumors may cause increased intracranial pressure.
CNS, central nervous system; CSF, cerebrospinal fluid; hCG, human chorionic go
nadotropin.

--------------------------------------- 151
234

Blueprints Pediatrics

with increasing frequency and intensity. Obstructive


ead MRI, and in the case of the
hydrocephalus may produce macrocephaly if it
safer. Because of this the head CT
occurs before the sutures have fused. Strabismus
screening tool for the presence of
with diplopia can result from a sixth nerve palsy
cranial mass, and is especially
induced by increased intracranial pressure. Papilhydrocephalus. Head CT is poor
ledema may be detected on funduscopic examinaterior fossa. MRI is utilized to
tion. Cushing's triad (hypertension, bradycardia, and
atomic details on mass size and
irregular respirations) is a late finding.
rgical planning. Head MRI is espeChildren with infratentorial tumors often present
agnosing tumors of the posterior
with deficits of balance or brainstem function
d. Examination of cerebrospinal
(truncal ataxia, problems with coordination and gait,
sential to determine the presence
cranial nerve dysfunction). Because it can result from
ulloblastoma and pinealoma.
increased ICP, a sixth nerve palsy is not considered
a localizing focal neurologic deficit, whereas other
cranial nerve deficits, by definition, localize the lesion
to the brainstem. Head tilt, as a compensation for loss
es of treatment of primary CNS
of binocular vision, is noted with focal deficits of
in Table 17-4.
cranial nerve III, IV, or VI, which cause extraocular
muscle weakness. Nystagmus is usually due to cerebellovestibular pathway lesions, but may also be seen
with a marked visual deficit (peripheral or cortical
EY POINTS
blindness).
s system tumors are the most
Children with supratentorial tumors commonly
umors in children and are second
present either with signs of increased ICP (discussed
overall incidence of malignant
earlier) or seizures. Although most seizures are generalized, less dramatic episodes with incomplete loss
brain tumors in adults, in whom
of consciousness (complex partial seizures) and
tumors are more common, brain
transient focal events without loss of consciousness
ren are predominantly infratentor(partial seizures) are also seen. Personality changes,
fossa), involving the cerebellum,
poor school performance, and change in hand prefbrainstem.
erence suggest a cortical lesion. Endocrine abnormalities are noted with pituitary, hypothalamic, or
pineal tumors. Babinski reflex, hyperreflexia, spasticity, and loss of dexterity occur with either brainstem
MPHQMA
or cortical tumors.

much faster than a h


unstable patient is
is more useful as a
tumor or other intra
useful to assess for
at assessing the pos
further delineate an
location, and for su
cially helpful in di
fossa and spinal cor
fluid cytology is es
of metastasis in med

Treatment
The general principl
tumors are outlined

K
1. Central nervou
common solid t
to leukemia in
diseases.
2. In contrast to
supratentorial
tumors in child
ial (posterior
midbrain, and

.NOISMHOpGiKlN'S LY
Pathogenesis

Differential Diagnosis
mas (NHLs) are a heterogeThe differential diagnosis includes arteriovenous
ses characterized by neoplastic
malformation, aneurysm, brain abscess, parasitic
ature lymphoid cells, which,
infestation, herpes simplex encephalitis, granulomat lymphoid cells of ALL, accutous disease (tuberculosis, cryptococcal, sarcoid),
bone marrow. Just as the immune
intracranial hemorrhage, pseudotumor cerebri, pried into T- and B-cell compartmary cerebral lymphoma, vasculitis, and, rarely,
to T- and B-cell categories.
metastatic tumors.
types in childhood NHL include

Non-Hodgkin's lympho
neous group of disea
proliferation of imm
unlike the malignan
mulate outside the
system can be divid
ments, NHLs fall in
Histopathologic sub
lymphoblastic (T ce

ll), 50%; undifferentiated small


cell (B cell), 30%;
and large cell (T-, B-, or indeterDiagnostic Evaluation
, 20%. Undifferentiated small cell
Computed tomography (CT) and magnetic resoed into Burkitt's and nonnance imaging (MRI) are the procedures of choice
for diagnosing and localizing tumors and other
differs from that in adults in
intracranial masses. A head CT can be performed
ays. Most cases of NHL in chil-

minate cell origin)


NHL can be subdivid
Burkitt's types.
NHL in children
several important w

--------------------------------------- 152
Chapter 17 / Oncology

235
erative disease, sev

ere combined immunodeficiency)


TABLE 17-4
and acquired immunod
eficiency (e.g., AIDS, iatroApproach to Treatment of Childhood CHS
ion in organ and bone marrow
Tumors
s) have an increased incidence of
Treatment
Goals
loom's syndrome and ataxia-

genic immunosuppress
transplant recipient
NHL. Patients with B
telangiectasia also

have a higher incidence of NHL


Surgery Establish diagnosis
iatric population. Malaria and
Debulk and/or resect tumor

than the general ped

Treat increased ICP (ventricular


nfection are believed to be risk

Epstein-Barr virus i
factors for the deve

lopment of Burkitt's lymphoma


shunt, if required)

Radiation Control residual disease

in African countries

.
Control tumor dissemination
Cure
ons
Chemotherapy Adjuvant therapy for malignant

Clinical Manifestati

All childhood NHLs g


row rapidly; as a result,
tumors
Minimize radiation exposure
short. The abdomen is the most
Delay and/or obviate need for
al manifestation of B-cell NHL,
radiation
mediastinum is the primary site
Immunotherapy Adjuvant therapy for malignant
minal involvement can result in
tumors
rgement, pain, ascites, or urinary
Scavenger for minimal residual
astrointestinal obstruction occurs
disease
rves as the lead point for

symptom duration is
common site of initi
whereas the anterior
for T-cell NHL. Abdo
rapid abdominal enla
tract obstruction. G
when the lymphoma se
an intussusception.

Anterior mediastinal masses are


CNS, central nervous system; ICP, intracranial pressure.
Adapted from Pizzo PA, Poplack DG, eds. Principles and practice ofassociated w
ith pleural effusions, airway compropediatric oncology, 3rd ed. Philadelphia: Lippincott-Raven, 1997.mise, and sup
erior vena cava syndrome. Childhood
NHL has a high frequ
ency of dissemination to extranodal sites, such as
the CNS and bone marrow.
dren are diffuse, highly malignant, extremely aggresPeripheral lymph nod
e enlargement can be seen with
sive, and show little differentiation beyond primitive
any type of childhoo
d NHL, and fever and weight
cells. Adult NHL is usually highly differentiated
sent. The progression of disease
and nodular. Distant noncontiguous metastases are
s not follow an orderly
common in childhood NHL, making adult staging

loss may also be pre


in childhood NHL doe

anatomic sequence of
spread as seen with Hodgkin's
systems that depend primarily on nodal involvement

disease.

of little relevance. NHL in childhood resembles ALL


more than it does adult-onset NHL or Hodgkin's
lymphoma. Almost half the cases of NHL in childn

Diagnostic Evaluatio

hood are of T-cell origin, compared with approxiThe evaluation befor


e therapy should include a commately 5% of those in adults.
look for leukocytosis, thrombo-

plete blood count to


cytopenia, and anemi

a. Bone marrow aspiration,


Epidemiology
mbar puncture with cere-

chest radiograph, lu

brospinal fluid cyto


logy, and radionuclide bone scan
Lymphomas are the third most common malignancy
are used to detect d
isseminated disease. Evaluation
in childhood. Approximately 60% of pediatric lymof renal and hepatic
function can be undertaken if
phomas are non-Hodgkin's lymphomas, with the
disseminated disease
is found. Chest and abdominal
remainder being Hodgkin's lymphomas. NHL occurs
CT and ultrasound st
udies may be used to determine
at least three times more frequently in boys than in
the extent of diseas
e. Staging laparotomy with
girls and has a peak incidence between the ages osplenectomf
y and l
iver biopsy is not indicated in
7 and 11 years.
childhood NHL.
Risk Factors

Treatment

Children with congenital immunodeficiency (e.g.,


on staging classification for
Wiskott-Aldrich syndrome, X-linked lymphoprolifilable. It is essential to ascer-

No generally agreedchildhood NHL is ava

--------------------------------------- 153
Blueprints Pediatrics
236
tain whether a patient has local disease (nodaHistopathologil
c s
ubtypes in childhood Hodgkin's
or extranodal), which has an excellent prognosis, odiseasr
e are similar
to those in adults: 40% to 60%
disseminated disease, which has a less favorablnodulae
r sclerosis,
10% to 20% lymphocyte predomiprognosis.
nance, 20% to 40% mi
xed cellularity, and 10%
Systemic disease, occult or overt, is present ilymphocytn
e deplet
ion.
about 80% of children with NHL. Aggressive multidrug chemotherapy with the agents known to be
effective in childhood ALL is the mainstay of
Epidemiology
therapy. Induction produces remission in 90% of
Hodgkin's disease ac
counts for 5% of all cases of
affected children, and maintenance chemotherapy
childhood cancer. Ep
idemiologic studies have identireduces the incidence of relapse. With radiotherapy
fied three distinct
forms of Hodgkin's disease: a childalone, 30% of patients develop leukemic transformahood form (age < 14
years), a young adult form (age
tion and bone marrow relapse. Central nervous

15-34 years), and an


older adult form (age 55-74
system prophylaxis is essential.
e has a bimodal distribution with
Patients with localized disease have a significantly
5 to 30 years of age and after the
better survival rate than that of patients with disoccurs in children younger than
seminated disease. Patients with hyperuricemia or an
3:1 male predominance in the
elevated serum lactic dehydrogenase level are con-

years). Its incidenc


peaks occurring at 1
age of 50. It rarely
10 years. There is a
childhood form of Ho

dgkin's disease.
sidered to have a high tumor load, are at risk for
tumor lysis syndrome, and have a worse prognosis
than those who do not. The long-term survival of alClinical
ions
children with NHL is 50% to 75%.

l Manifestat
History and Physical

Examination
The most common pres
entation is painless, firm lymphadenopathy involvi
ng either the supraclavicular or
KEY POINTS
thirds of patients will also have
1. Non-Hodgkin's lymphomas are a heterogeneous
nopathy. Fever, night sweats,
group of diseases characterized by neoplastic
asionally pruritus are noted in
proliferation of immature lymphoid cells, which,
hrotic syndrome is a rare but
unlike the malignant lymphoid cells of ALL, accug feature of Hodgkin's disease.
mulate outside the bone marrow.
2. NHL in children differs from that in adults in

cervical nodes. Twomediastinal lymphade


weight loss, and occ
30% of children. Nep
recognized presentin

Differential Diagno
sis
several important ways. In contrast to NHL in
adults, most cases of NHL in children are diffuse,
gnosis for Hodgkin's and NHL
highly malignant, extremely aggressive, and show
ymphadenitis, infectious mononulittle differentiation beyond primitive cells.
s, atypical mycobacterial infection,
3. Lymphomas (NHL and Hodgkin's lymphoma) are

The differential dia


includes bacterial l
cleosis, tuberculosi
cat scratch disease

, human immunodeficiency virus


the third most common malignancy in childhood.
mosis, and toxoplasmosis.
Two-thirds of lymphomas are the non-Hodgkin's
type.
4. Patients with localized disease have a significantly
on
better survival rate than that of patients with dis-

infection, histoplas

Diagnostic Evaluati
The hallmark of dia

gnosis is the identification of


seminated disease.
s in tumor tissue. Nonspecific ele-

Reed-Sternberg cell
vations of erythrocy

te sedimentation rate and serum


copper and serum fe
rritin levels may occur and cor.HOJDG KIN'S L.YM PHOM A
ctivity. Abnormalities of renal

relate with disease a


and hepatic functio

n tests may influence the choice


of chemotherapeutic
agents. Autoimmune hemolytic
Pathogenesis
anemia and thromboc
ytopenia are unusual findings,
The cause of Hodgkin's disease is unknown, bubut
t leukocytosis a
nd eosinophilia are often seen.
some indirect evidence suggests an infectiouCutaneous
s antige
n testing reveals anergy and a
agent. Familial clustering has also been observeddiminishe.
d cellul
ar immunity that predisposes the

--------------------------------------- 154
Chapter 17 / Oncology
237
patient to opportunistic infections. Initial chest radiside effects. The ad
dition of radiation to combination
ograph and CT scan define the extent of mediastinachemotherapl
y imp
roves disease-free survival in chiland pulmonary parenchymal involvement. Abdomidren with bulky dise
ase and B subgroup symptoms,
nal CT scan can identify subdiaphragmatic lymph
and also allows for
fewer cycles of chemotherapy.
nodes as well as liver and spleen involvement. A bone
Prognosis varies
from a 90% cure of stage I disease
marrow biopsy should be performed when dissemito a 50% cure of sta
ge IV disease. As in adults, lymnated disease is suspected.
phocyte predominance
has the most favorable prognosis and lymphocyte
depletion the least favorable.
Late complications o
f therapy include secondary
Treatment
malignancies (AML, N
HL) from combined radioTreatment depends on staging. Four stages artherape
azine-containing chemotherapy
described, and for any given stage, patients are
and dysfunction, growth retardafurther subdivided into A or B subgroups dependintiong

y and procarb
regimens, thyroid gl
, and sterility.

on the absence (A) or presence (B) of systemic symptoms. Systemic symptoms are defined as unexplained
weight loss greater than 10% of body weight in the
EY POINTS
preceding 6 months, fever higher than 38C for 3

consecutive days, and night sweats. The approximate


f Hodgkin's disease has a bimodal
stage can be assigned using a combination of clinical
th peaks occurring at 15 to 30 years
and laboratory information, but definitive staging

1. The incidence o
distribution wi

of age and afte


r the age of 50.
often requires exploratory laparotomy. Surgical
llular immunity that predisposes
staging is not indicated unless therapy will be influopportunistic infections is common
enced by the findings. If the patient clearly has dismphoma. Hodgkin's lymphoma
seminated disease (stage III or IV), surgical staging is
red in an otherwise healthy
unnecessary. With surgical staging, as many as 30%

2. A diminished ce
the patient to
in Hodgkin's ly
must be conside

adolescent with
an opportunistic infection.
of patients with stages I and II are reclassified to
higher stages. Approximately 60% of children with
Hodgkin's disease have stage I or II disease. The stages
are as follows:
NEUROBLASTOMA
Stage I: Involvement of a single lymph node region

Pathogenesis

or a single extralymphatic organ.


Stage II: Involvement of two or more lymph node
ignancy of the primitive
regions on the same side of the diaphragm, or
hat form the adrenal medulla and
localized involvement of an extralymphatic organ
thetic ganglia. Neuroblastoma

Neuroblastoma is a mal
neural crest cells t
the paraspinal sympa
can be located in t

he abdomen, thoracic cavity, or


and one or more lymph node regions on the same
inal tumors account for
side of the diaphragm.
Stage III: Involvement of lymph node regions on
ird of which arise from the
both sides of the diaphragm. This may be accompathetic ganglia and two-thirds
panied by localized involvement of an extralymdulla itself. Thoracic masses,
phatic organ or site, involvement of the spleen, or
of the tumors, tend to arise from

head and neck. Abdom

70% of tumors, one-th


retroperitoneal sym
from the adrenal me
accounting for 20%
paraspinal ganglia

in the posterior mediastinum.


both.
Stage IV: Disseminated involvement of the liver,
neck occurs in 5% of cases and

Neuroblastoma of the

bone marrow, lungs, or other non-nodal sites.


cervical sympathetic ganglion.
Most pediatric protocols prescribe multiagent

often involves the

Epidemiology
chemotherapy, alone or in combination with lowdose involved-field radiation therapy. VincristineNeuroblastom,
a ac
counts for 7% of all childhood
prednisone, cyclophosphamide, and procarbazine
cancers and, in chi
ldren, is the most common solid
has been the most commonly used combination of
tumor outside the c
entral nervous system. The median
chemotherapeutic agents, but other four-drug comage at diagnosis is
22 months; more than 50% of chilbinations may be as effective and may have fewer
dren are diagnosed
before 2 years of age, and 90% are

--------------------------------------- 155
238
Blueprints Pediatrics
diagnosed before 5 years of age. There is a slight male
is
predominance. Neuroblastoma accounts for 15% of
gnosis of abdominal neuroblasthe pediatric cancer-related deaths each year.
lesions such as hydronephrosis,

Differential Diagnos
The differential dia
toma includes benign
polycystic kidney di

sease, and splenomegaly and


Risk Factors
h as renal cell carcinoma,
The incidence of neuroblastoma is 1 per 100,000
ma, retroperitoneal rhab-

malignant tumors suc


Wilms' tumor, lympho
domyosarcoma, and ov

arian tumors.
infants. It is associated with Hirschsprung's disease,
fetal hydantoin syndrome, and von Recklinghausen's
disease.
n

Diagnostic Evaluatio
The presence of a ma

ss can be confirmed by CT.


Clinical Manifestations
astoma can be made by patho-

Diagnosis of neurobl
logic identification

of tumor tissue or by the unequivThe clinical manifestations are extremely variable


or cells on bone marrow aspirate
because of the widespread distribution of neural crest
ed urinary catecholamines
tissue and the length of the sympathetic chain.

ocal presence of tum


combined with elevat
(vanillylmandelic ac

id and homovanillic acid).


History and Physical Examination
ry catecholamines, which are

Measurement of urina
breakdown products o

f epinephrine and norepiAbdominal tumors are hard, smooth, nontender

nephrine, is also us

eful for following response to


abdominal masses that are most often palpated in the
cting recurrence. For tumors
flank and displace the kidney anterolaterally and inferiorly. Abdominal pain and systemic hypertension
enal medulla, intravenous
occur if the mass compresses the renal vasculature.
lacement of the kidney

therapy and for dete


arising from the adr
pyelogram shows disp
with minimal distort

ion of the calyceal system.


Respiratory distress is the primary symptom seen in
umor generally results in distorthoracic neuroblastoma tumors. Sometimes the thosystem.
racic variant is asymptomatic, and the tumor is discovered as an incidental finding on chest radiograph
obtained for an unrelated reason. Neuroblastoma
of the neck presents as a palpable tumor causing
urgery and chemotherapy
Horner's syndrome (ipsilateral ptosis, miosis, and
nts have distant metastases at
anhidrosis) and heterochromia of the iris on the
gical resection of the primary
affected side. Sometimes thoracic or abdominal
nodes or selected metastases,
tumors invade the epidural space posteriorly in a

Conversely, Wilms' t
tion of the calyceal

Treatment
Treatment involves s
because 70% of patie
diagnosis. After sur
tumor and any lymph
surgical and radiolo

gic data are gathered to stage the


dumbbell fashion, compromising the spinal cord
and resulting in back pain and symptoms of cord
ined to organ or structure of
compression.
Metastases are common at diagnosis and often
cause the sequelae that lead to tumor diagnosis.
ends beyond structure of origin,
Nonspecific symptoms of metastatic disease include
dline, with (stage IIB) or without

tumor as follows:
Stage I: Tumor conf
origin
Stage II: Tumor ext
but not across mi
(stage IIA) ipsil

ateral lymph node involvement


weight loss and fever. Specific metastatic sequelae
tends beyond the midline, with
include bone marrow failure, resulting in pancytopenia; cortical bone pain, causing a limp (Hutchinson's
ral lymph node involvement
syndrome); liver infiltration, resulting in hepation of tumor to distant lymph
tomegaly (Pepper's syndrome); periorbital infilmarrow, liver, and/or other

Stage III: Tumor ex


or without bilate
Stage IV: Dissemina
nodes, bone, bone
organs (except as

defined in stage IVS)


tration, resulting in proptosis and periorbital
ger than 1 year with dissemecchymoses ("raccoon eyes"); distant lymph node
enlargement; and skin infiltration, causing palpable
to liver, skin, or bone marrow
subcutaneous nodules. Remote effects, such as watery
lvement and with a primary
diarrhea in patients with differentiated tumors that
otherwise be stage I or II
secrete vasoactive intestinal peptide, and opsoclonus-

Stage IVS: Age youn


ination of tumor
without bone invo
tumor that would
Postsurgical rad

iation is used to treat residual local


myoclonus (chaotic eye movements, myoclonic
d metastatic foci, whereas
jerking, and truncal ataxia), have been noted.
in duration and intensity

disease and selecte


chemotherapy varies

--------------------------------------- 156
Chapter 17 / Oncology
239
depending on the stage and biologic features. Regi-

WILMS'TUMOR

mens usually include vincristine, cyclophosphamide,


doxorubicin (Adriamycin), and cisplatin. Sponta-

Pathogenesis

neous regression is common in stage IVS tumors. In


stage IVS, surgical removal of the small primary
from neoplastic embryonal
tumor is indicated to prevent late local recurrence.
etanephros. Wilms' tumor, like
Bone marrow transplantation is often the best
ostulated to evolve through
therapy for extensive stage III and IV disease.
the host genome. Prezygotic
Infants younger than 1 year have the best progce of the first hit is followed by
nosis. Stages I, II, and IVS have a good prognosis,
ic) mutation, the second hit,
whereas stages III and IV have a poor prognosis.
ancy in the tissue rendered susSerum markers associated with a poor prognosis int hit. The most often cited genetic

Wilms' tumor results


renal cells of the m
retinoblastoma, is p
two distinct hits to
(germline) inheritan
a postzygotic (somat
which induces malign
ceptible by the firs
anomaly in Wilms' tu

mor is partial deletion of chroclude elevated neuron-specific enolase, ferritin, and

mosome 1Ipl3.

lactic dehydrogenase. Certain genetic features, such


as N-myc oncogene amplification within the tumor
cells, are associated with a poor prognosis. Each
stage's percentage of new cases annually and it

Epidemiologs

respective 5-year survival rate are as follows:


for 6% of all childhood cancers.
Stage I: 5% of cases at diagnosis and greater than
und in the first 5 years of life
90% survival
) and has equal occurrence in
Stage II: 10% of cases at diagnosis and 75% survival
.
Stage III: 25% of cases at diagnosis and 40% to 70%

This tumor accounts

survival, depending on the success of surgical resection

It is predominantly fo
(mean 3 years of age
both males and females

Stage IV: 60% incidence and 60% survival if age at


diagnosis is less than 1 year, 20% if age at diagnorisk for Wilms' tumor include
sis is older than 1 year and less than 2 years, and
aniridia, Beckwith-Wiedemann
10% if age at diagnosis is greater than 2 years
rophy, macroglossia, omStage IVS: 5% of cases at diagnosis and greater
rinary abnormalities), and
than 80% survival
ies.
KEY POINTS

Risk Factors
Children at highest
those with sporadic
syndrome (hemihypert
phalocele, and genitou
genitourinary anomal

1. Neuroblastoma may occur in the abdomen, thoClinical Manifestati


ons
racic cavity, or head and neck; 70% of children
History and Physical
Examination
present with abdominal tumors.
Most children (85%)
are diagnosed after incidental
2. Neuroblastoma accounts for 7% of newly diagdetection of an asym
ptomatic abdominal mass by the
nosed cases of cancer in children each year.
child's parents whil
e bathing or dressing the child or
3. Neuroblastoma usually occurs in children younger
by the pediatrician
during a routine physical examithan 5 years.
nation. Abdominal pa
in or fever may develop after
4. For abdominal tumors that arise from the adrenal
hemorrhage into the
tumor. Other associated
medulla, the intravenous pyelogram often shows
findings include mic
roscopic or gross hematuria
displacement of the kidney with minimal distortion
(10%-25%) and hypert
ension (25%). Hypertension
of the calyceal system. Conversely, Wilms'tumor
occurs as a result o
f either renin secretion by tumor
generally results in distortion of the calyceal system.cells or compression
of the renal vasculature by the
5. Treatment involves surgery and chemotherapy,
tumor.
because 70% of patients at diagnosis have distant
alities include complete or
metastases.
ridia, hemihypertrophy, and
6. Infants younger than 1 year have the best progies including hypospadias,
nosis. Stages I, II, and IVS have a good prognosis,
eshoe or fused kidneys, ureteral
whereas stages III and IV have a poor prognosis.
tic kidneys, and ambiguous

Associated abnorm
partial sporadic ani
genitourinary anomal
cryptorchidism, hors
duplication, polycys
genitalia.

--------------------------------------- 157
24

Blueprints Pediatrics
strictors, beta-bloc

kers, and volume administration.

Holding the infant o


ver the shoulder and placing
the child in a kneechest position decreases preload
and increases system
ic vascular resistance. Morphine
sulfate suppresses t
he respiratory center, stops
hyperpnea, and dilat
es the pulmonary arteries. Vasoconstrictors raise t
he systemic vascular resistance,
whereas beta-blocker
s are thought to minimize
infundibular spasm.
Volume is added to increase the
systemic blood press
ure, which minimizes right-toleft shunting. Metab
olic acidosis must be corrected,
because it increases
pulmonary vascular resistance
and thereby promotes
right-to-left shunting across
the ventricular sept
al defect. In most institutions,
surgical repair is p
erformed during the first 3 to 6
months of life, or a
fter the first hypercyanotic episode
(tet spell).
Ebstein's Anomaly
Ebstein's anomaly (F
igure 3-7) is an extremely rare
Tetralogy of Fallot.Typical anatomic and hemody-anomaly in which the
Figure 3-6
septal leaflet of the tricuspid
namic findings include: (a) an anteriorly displaced infundibular
septum, resulting in subpulmonary stenosis, a large ventricularvalve is displace
d into the right ventricular cavity and
septal defect and overriding of the aorta over the muscular the anterior leaflet
of the tricuspid valve is sail-like
septum; (b) hypoplasia of the pulmonary valve, main and
and redundant. This
results in a portion of the right
branch pulmonary arteries; (c) equal right and left ventricular
pressures; (d) a right-to-left shunt at ventricular level, with a sys-ventricle
being incorporated into the right atrium.
Functional hypoplasi
a of the right ventricle results,
temic oxygen saturation of 82%.
as well as tricuspid
regurgitation or stenosis or both.
Cloherty JP, Stark AR. Manual of Neonatal Care, 4th ed. Philadelphia:
Lippincott-Raven, 1998:426.
A patent foramen ova
le is present in 80% of neonates
with the anomaly, an
d there is a right-to-left shunt
at the atrial level.
The right atrium is massively
dilated, which may r
esult in supraventricular tachyresolve spontaneously, or may lead to progressive
cardia. Wolff-Parkin

son-White (WPW) syndrome is


hypoxia, acidosis, and death. On cardiac examinaein's anomaly. In severe cases of
tion, a right ventricular heave is often felt and a loud
he majority of the pulmonary
systolic ejection murmur is heard in the left upper
m the patent ductus arteriosus
sternal border due to right ventricular outflow tract
ntricle.
obstruction. The ECG reveals right atrial dilation and
right ventricular hypertrophy, whereas the chest

associated with Ebst


Ebstein's anomaly, t
blood flow comes fro
and not the right ve

Clinical Manifestati
ons
radiograph shows normal heart size with decreased
pulmonary vascular markings. Twenty-five percent of
vere form of the disease present
children with tetralogy of Fallot have a right-sided
ngestive heart failure in the
aortic arch.
fe. The cardiac examination reveals

Neonates with the se


with cyanosis and co
first few days of li
a widely fixed split

Sz, and a tricuspid regurgitant


murmur is heard at t
he left lower sternal border. The
Treatment
bundle branch block with right
The treatment of tet spells is aimed at diminishing
Delta waves due to WPW synright-to-left shunting by increasing systemic vascular
icular tachycardia may maniresistance and decreasing pulmonary vascular resisst radiograph reveals extreme
tance. Tet spells may be treated with supplemental
table right atrial enlargement
oxygen, vagal maneuvers, morphine sulfate, vasoconary vascular markings.

ECG reveals a right


atrial enlargement.
drome and supraventr
fest themselves. Che
cardiomegaly with no
and decreased pulmon

--------------------------------------- 158
Blueprints Pediatrics
240
Differential Diagnosis
tsurgical chemotherapy and

The extent of pos


radiation depends on

tumor stage and histology.


The differential diagnosis of Wilms' tumor includes
s usually include actinobenign lesions such as hydronephrosis, polycystic
, and high-dose doxorubicin.
kidney disease, and splenomegaly, as well as
s include tumor histology (most
malignant tumors such as renal cell carcinoma,
neuroblastoma, lymphoma, retroperitoneal rhabstage. Tumors with favorable
domyosarcoma, and ovarian tumors. Wilms' tumor
lassic nephroblastoma, have an
accounts for one-third of malignant intra-abdominal
rate, regardless of stage. Tumors

Chemotherapy regimen
mycin D, vincristine
Prognostic factor
important) and tumor
histology, such as c
88% overall survival
with unfavorable his

tology, such as anaplastic or sartumors in childhood.


ave a 12% cumulative survival

comatous variants, h
rate. The 4-year ove

rall survival of patients with


Diagnostic Evaluation
is directly related to stage, and

favorable histology
with treatment can b

e expected to be 95% for stage


Screening tests include a complete blood count with
or III, and 80% for stage IV.
differential, liver function tests, electrolytes, BUN,
creatinine, and urinalysis. Radiologic studies include
abdominal ultrasound to establish the presence of an
intrarenal mass, assess renal vasculature, and examine
the contralateral kidney. An abdominal CT scan
EY POINTS
assesses the degree of local extension and involve-

I, 90% for stage II

K
1. Wilms'tumor, li

ke retinoblastoma, is postulated to
ment of the inferior vena cava. CT scans of the chest
two distinct hits to the host
and abdomen are routinely performed to detect
tic (germline) inheritance of the
hematogenous metastases, which are present at
llowed by a postzygotic (somatic)
diagnosis in 10% of patients; the lung is the most
econd hit, which induces
common site of metastatic spread. Radionuclide
bone scan, though not routinely recommended, will
after exploratory laparotomy.
detect metastases to the bone. In children with unfatology rather than its stage is more
vorable histology, a CT scan of the head is required
to exclude CNS metastases.
ognosis. With favorable histology,

evolve through
genome. Prezygo
first hit is fo
mutation, the s
malignancy.
2. Staging is done
3. The tumor's his
important to pr
taking into acc

ount all stages, the aggregate


survival is 88%
.
Treatment
Treatment involves surgery, radiotherapy, and
chemotherapy. Unilateral tumors are treated with
immediate surgical resection of the affected kidney.
When the tumor is bilateral, presurgical chemotherapy or radiation is performed to shrink the tumors in
an attempt to salvage some renal function. Local
ne tumors account for 4% of
tumor excision is then attempted. After nephrecwo forms predominate: Ewing's

BONE TUMORS
Primary malignant bo
childhood cancers. T
sarcoma and osteogen

ic sarcoma.
tomy or surgical resection of the primary tumor and
any lymph nodes or selected metastases, the surgical
and radiologic data are gathered to stage the tumor
as follows:
Stage I: Tumor limited to the kidney and comundifferentiated sarcoma that
pletely excised
bone. The clonal nature of the

Ewing's Sarcoma
Pathogenesis
Ewing's sarcoma is an
arises primarily in

Stage II: Tumor extends beyond the kidney but is


y the consistent translocation
completely excised
to chromosome 22 in affected
Stage III: Residual nonhematogenous tumor conrogenic origin has been suggested
fined to the abdomen
entiated Ewing's sarcoma because
Stage IV: Hematogenous metastases to lung, liver,
location that is found in the
bone, and/or brain
e neuroectodermal tumors of the
Stage V: Bilateral renal involvement at diagnosis
stem.

disease is revealed b
from chromosome 11
cells. A possible neu
for highly undiffer
it has the same trans
cells from primitiv
peripheral nervous sy

--------------------------------------- 159
Chapter 17 / Oncology
241
Epidemiology
or proximal femur have less

of the pelvic bones


favorable outcomes.

Other less favorable features


Ewing's sarcoma is seen primarily in adolescents and
extension, low lymphocyte count,
is 1.5 times more common in males than females. It
is an extremely rare occurrence in African Americans.
actate dehydrogenase.
Unlike osteogenic sarcoma, it occurs in both young

include soft tissue

and elevated serum l

children and adolescents.


KE
Y POINTS
Clinical Manifestations
is an undifferentiated sarcoma
Pain and localized swelling at the site of the primary
tumor are the most common presenting complaints.
marily in bone.

1. Ewing's sarcoma

that arises pri


2. It affects youn

g children and adolescents but is


Unlike osteosarcoma, in which the long bones are
in African Americans.
predominantly involved, flat and long bones are
zed swelling are the most common
equally represented. The most commonly involved
laints.
sites are the femur (20%), pelvis (20%), fibula (12%),
sites for Ewing's sarcoma are
and humerus (10%). Other sites include the tibia,
he bones of the pelvis, which have
ribs, clavicle, and scapulae. In the long bones, Ewing's
able prognosis.
sarcoma usually begins midshaft rather than at the
ends as in osteosarcoma. Systemic manifestations are

extremely rare
3. Pain and locali
presenting comp
4. The most common
the femur and t
the least favor

more common in children with metastases and


include fever, weight loss, and fatigue.

Osteogenic Sarcoma

Differential Diagnosis

Pathogenesis

The differential diagnosis for Ewing's sarcoma


also referred to as osteosarincludes osteomyelitis, eosinophilic granuloma, and
tumor of the bone-producing
osteosarcoma. Metastasis to the bone by neuroblasrcoma arises in either the
toma or rhabdomyosarcoma should be considered in
the periosteum. The primary
younger children with a solitary bone lesion.
ated at the epiphysis of anatomic

Osteogenic sarcoma,
coma, is a malignant
osteoblasts. Osteosa
medullary cavity or
tumor is usually loc
sites that are assoc

iated with maximum growth


Diagnostic Evaluation
ude the distal femur, proximal

velocity, which incl


tibia, and proximal

humerus.
Leukocytosis and an elevated erythrocyte sedimentation rate are often seen. Radiographs characteristi-

Epidemiology

cally reveal a lytic bone lesion with calcified


periosteal elevation (onion skin) or a soft tissue mass,
mainly in adolescence, with a
or both. Biopsy confirms the diagnosis.
of 2:1. Peak incidence occurs

Osteosarcoma is seen
male-to-female ratio
during the maximum g

rowth velocity period.


Treatment
Radiation, chemotherapy, and surgery provide local
ons
control of the primary tumor. If the tumor affects
arcoma, pain and localized
an expendable bone (fibula, rib, or clavicle), comt common presenting complete surgical excision may be warranted. Most
rast to Ewing's sarcoma, systemic
patients with Ewing's sarcoma have micrometastatic
are. Because these tumors occur
disease at the time of diagnosis; as a result,
dolescents, initial complaints
chemotherapy is critical to reduce the size of the
trauma. The most common
primary tumor, treat metastases seen at diagnosis, and
distal femur (40%), proximal tibia
prevent potential future metastases. Specific agents
humerus (10%). Metastases to
used include vincristine, cyclophosphamide, doxoru% of cases. Gait disturbance and
bicin, etoposide, and ifosfamide.
also may be present.
The prognosis is excellent for patients with

Clinical Manifestati
Similar to Ewing's s
swelling are the mos
plaints, but in cont
manifestations are r
most frequently in a
may be attributed to
tumor sites are the
(20%), and proximal
the lung occur in 20
pathologic fractures

Differential Diagnos
is
distal extremity nonmetastatic tumors treated with
chemotherapy and radiation. The 5-year survival rate
gnosis for osteosarcoma includes
is 50% in patients without metastatic disease. Chilign bone tumors, and chronic
dren with metastatic disease at diagnosis or tumors

The differential dia


Ewing's sarcoma, ben
osteomyelitis.

--------------------------------------- 160
242
Blueprints Pediatrics
Diagnostic Evaluation
pse-free survival is greater than
The erythrocyte sedimentation rate and complete
tment of metastatic disease is
blood count are generally normal, whereas the serum
ome patients can be salvaged
alkaline phosphatase level is usually elevated at
therapy and surgical resection
diagnosis and can be used as a marker of treatment
ses. Specific chemotherapeutic
response. Lytic bone lesion with periosteal reaction
atin, doxorubicin, and methotrexis characteristic on radiograph. The periosteal inflamfindings include age less than 10
mation has the appearance of a radial "sunburst" that
>15cm), osteoblastic cell type,
results as the tumor breaks through the cortex and
xial skeleton or humerus, elenew bone spicules are produced. A CT scan of the
dehydrogenase, presence of
chest is essential to detect pulmonary metastases,
an 2 months, and metastatic
which appear as calcified nodules.
Treatment

tion, long-term rela


70%. Aggressive trea
indicated, because s
with high-dose chemo
of pulmonary metasta
agents include cispl
ate. Poor prognostic
years, large tumor (
involvement of the a
vated serum lactate
symptoms for less th
disease.
K

EY POINTS
At diagnosis, 20% of patients have clinically
detectable metastatic disease, and most of the
oma is a malignant tumor of the
remaining patients have microscopic metastatic
osteoblasts.
disease. Various limb salvage surgical procedures that
ises most often during maximum
limit resection to the tumor-bearing portion of the
in the distal femur, proximal tibia,
bone are used initially, but postsurgical chemothererus.
apy dramatically increases disease-free survival. Parg's sarcoma, pain and localized
ticular chemotherapeutic agents include high-dose
e most common presenting commethotrexate, doxorubicin, and cisplatin. The tumor
contrast to Ewing's sarcoma, sys-

1. Osteogenic sarc
bone-producing
2. Osteosarcoma ar
growth velocity
or proximal hum
3. Similar to Ewin
swelling are th
plaints, but in

is relatively resistant to radiation therapy.


tions are rare.
Before adjuvant chemotherapy, survival from
sts of limb salvage surgical proceosteosarcoma was only 20%. Currently, with aggrestherapy.
sive chemotherapy before and after surgical resec-

temic manifesta
4. Treatment consi
dures and chemo

--------------------------------------- 161
J || [ I
phthalmology
VISION SCREENING
] and reduced stereopsis. Treat-

-'.- ; I
amblyopia (see later

ment is aimed at cor


rection of the underlying cause,
Vision screening in children is critical because the
elimination of ambly
opia, and medical or surgical
young eye is part of a dynamic system that may brealignmene
t of th
e eyes.
quickly damaged by visual deprivation. The American Academy of Ophthalmology's recommendations
for vision screening are found in Table 18-1. Children
KE
Y POINTS
older than 8 years can be screened according to adult
guidelines. Children with a history of prematurity,
1. Screening for s
trabismus by means of cover
intrauterine infection, central nervous system disease,
testing should
be included in every pediatric
or family history of ocular disease are at high risk for
health maintena
nce examination.
eye pathology and require more extensive follow-up
2. Early recogniti
on and treatment offer the best
by an ophthalmologist.
means of obtain
ing permanent realignment and
avoiding amblyo
pia.
..STRABISMUS
Strabismus, or misalignment of the eyes, occurs in
AMBLYOPIA
approximately 4% of children. Certain neurologic
Amblyopia, literally
meaning "dull sight," refers to
diseases are associated with an especially high incireduced vision devel
oping in an otherwise normal
dence of strabismus, including cerebral palsy, Down
eye. The condition o
ccurs in 2% to 5% of the general
syndrome, hydrocephalus, and brain tumors. Unilatpopulation. Strabism
us, the most common cause of
eral visual deprivation may also lead to strabismus.
amblyopia, is due to
the suppression of retinal images
from a misaligned ey
e. Visual deprivation due to
Clinical Manifestations
opacities of the opt
ical axis (ptosis, corneal opacity,
cataract) or to uneq
ual refractive errors in the two
The deviating eye of a patient with strabismus maeyey
s (anisometropia

) also results in amblyopia. Other


turn inward (esodeviation), outward (exodeviation),
risk factors include
premature birth and family
upward (hyperdeviation), or downward (hypodeviaor strabismus.
tion). Diagnosis is made using the corneal light reflex
and cover tests.

history of amblyopia

Clinical Manifestati
ons
Treatment
the only sign of amblyopia, and

Subnormal vision is
amblyopia remains a

diagnosis of exclusion.
The most important consequences of untreateUntreated
a leads to permanent vision loss
strabismus, aside from the cosmetic deformity, arane
eopsis.

d amblyopi
d diminished ster

--------------------------------------- 162
244

rints Pediatrics
TABLE 18-1
Pediatric Vision Screening Recommendations of
ology
Age
Examination
Newborn
Corneal light reflex
xes
Red reflexes
bnormality
By age 6 months
Fixation to light or
ion
Monocular occlusion
Corneal light reflex
Cover/uncover test
exes
Red reflexes
abnormality
Age 3-4 yr
Visual acuity
t least 20/40 in each eye and no more
Corneal light reflex
ference between the 2 eyes on vision
Cover/uncover test
Fundus examination

the American Academy of Ophthalm


test

Referral
Abnormal red refle
Any other ocular a

small toys

Aversion to occlus

test

Strabismus
Nystagmus
Abnormal red refl
Any other ocular
Visual acuity of a

test

than 1 line dif


testing
Strabismus
Any other ocular

abnormality
Age 5 or older
Visual acuity
Visual acuity of
20/40 or less in one or both eyes
Corneal light reflex test
Strabismus
Cover/uncover test
Any other ocular
abnormality
Fundus examination
Source: Communication of the American Academy of Ophthalmology, San Francisco,
2001.
Treatment
cases of leukocoria
require prompt ophthalmologic
referral.
Therapy involves occlusion of the better-seeing eye.

This allows stimulation of visual centers in the brain


corresponding with the affected eye. The vulnerable
is
period for the development of amblyopia is up to
most common intraocular
approximately age 8. Beyond that period, amblyopia
ood, is a life-threatening cause
is unlikely to develop and treatment is unlikely to be
isease occurs in approximately 1
successful.
s, resulting in 300 new cases in

Differential Diagnos
Retinoblastoma, the
malignancy of childh
of leukocoria. The d
in 20,000 live birth
the United States ea

ch year. The genetic defect


occurs on the q!4 ba
nd of chromosome 13.
KEY POINTS
toma leads to death from brain

Untreated retinoblas
and visceral metast

asis in almost all cases.


1. Amblyopia represents a common and potentially
ties of the crystalline lens) occur
reversible cause of vision loss in children.
wborns, making them the most
2. Successful treatment depends on early recogocoria. They may be congeninition and referral for occlusion therapy and
elimination of predisposing conditions.
may be unilateral or bilateral.

Cataracts (opaci
in 1 of every 250 ne
common cause of leuk
tal or acquired and
Cataracts are often

genetically determined but


may result from met
abolic diseases or intrauterine
infections.
Retinopathy of p
rematurity (ROP) is a retinal vasLEUKOCORIA
ature infants that can also lead

cular disease of prem


to leukocoria. As m

any as 65% of neonates weighing


Leukocoria (white pupil, or absence of the red
birth are affected. Risk factors
reflex) in an infant or child may be caused by a
t less than 1250g, gestational
number of entities, ranging from isolated ocular
eks, mechanical ventilation, and
abnormalities to life-threatening systemic disease. All
al oxygen. Other causes of

less than lOOOg at


include birth weigh
age less than 32 we
need for supplement

--------------------------------------- 163
Cha
pter 18 / Ophthalmology
245
leukocoria include congenital glaucoma and ocular
ons
toxocariasis.

Clinical Manifestati
Chronic tearing in t

he absence of conjunctival injecClinical Manifestations

tion is the hallmark

of nasolacrimal duct obstruction.


The presence of muco
purulent discharge and tenLeukocoria may be detected by routine screening of
ial aspect of the lower lid sugthe red reflex in all neonates and, if found, requiregests
nfection of the nasolacrimal
prompt referral to an ophthalmologist. Infants at
. Other causes of excess tearing
high risk for the development of ROP should be
tation from allergens or congeniexamined by an ophthalmologist when discharged
from the nursery and again at 3 to 6 months of age.

derness over the med


s superimposed i
sac (dacryocystitis)
include chronic irri
tal glaucoma.
Treatment

Treatment
Successful therapy combines treatment of the underording to severity of symptoms.

Treatment varies acc


The obstruction reso

lves spontaneously by 1 year of


lying condition with attention to associated ams. Referral to an ophthalmologist
blyopia. Treatment for retinoblastoma includes
toms persist. Probing of the nasoenucleation (removal of the eye), radiation therapy,
chemotherapy, and cryotherapy. Prognosis is directly
is performed at 12 to 15 months
related to the size of the tumor at diagnosis, and cure
symptoms warrant earlier inter-

age in 90% of infant


is indicated if symp
lacrimal duct system
of age unless severe
vention. Rarely, sur

gery is required to create a patent


rates today approach 90%. Unilateral or bilateral
. Superimposed dacryocystitis
congenital cataracts may be surgically removed.
warm compresses and nasoThe visual prognosis for children requiring cataract
th the addition of systemic
extraction is not as good as that seen in adults,
because amblyopia or associated ocular abnormalities
t cases.
may limit the ultimate level of visual acuity. Most
cases of ROP regress spontaneously; however,
EY POINTS
cryotherapy performed at an intermediate stage of
ROP reduces progression to the vision-threatening
ct obstruction is a common cause
stages of disease. Infants with treated or regressed
nfants and neonates and typically
ROP remain at risk for the development of amblyneously.

tear drainage system


may be treated with
lacrimal massage, wi
antibiotics in selec
K
1. Nasolacrimal du
of tearing in i
resolves sponta
2. Referral is ind

icated if symptoms persist beyond 9


opia, strabismus, myopia, and glaucoma.
to 12 months of
age and for infants with recurrent
dacryocystitis.
KEY POINTS
1. The most common cause of leukocoria is congenital cataract.
2. All cases of leukocoria require prompt ophthalmoEON ATORUM

. _ .9.P.4J.H AL^JA N

logic referral.
3. All children at high risk for retinopathy of premam refers to conjunctivitis

Ophthalmia neonatoru
occurring within the

first month of life. Any ocular


turity should be seen by an ophthalmologist
nate requires evaluation because
before discharge from the nursery.
sent in the first few weeks of life.

discharge in the neo


tears are usually ab
Differential Diagnos

is
NASOLACRIMAL
DUCT OBSTRUCTION
thalmia neonatorum include

Common causes of oph


chemical irritation,

Chlamydia trachomatis, and


Nasolacrimal duct obstruction, a common cause
e. Chemical conjunctivitis can
of overflow tearing (epiphora), occurs in 6% of
rauma or by antibiotic proneonates. Obstruction is usually caused by failure of
rth to prevent gonococcal infecthe distal membranous end of the nasolacrimal ductiont
nfectious causes, including
to open.
(HSV), Staphylococcus aureus,

Neisseria gonorrhoea
be caused by birth t
phylaxis given at bi
. Less common i
herpes simplex virus

--------------------------------------- 164
246

Blueprints Pediatrics

Haemophilus infiuenzae, and Pseudomonas aeruginosa,


Y POINTS
typically manifest after the first week of life. Nasolacrimal duct obstruction should be considered in
in the neonate may represent
neonates with persistent conjunctival discharge.
tion or acquired infection.

KE

1. Conjunctivitis
chemical irrita
2. Chlamydia and g

onorrhea are the most common


infectious agen
ts.
Clinical Manifestations
3. Suspected gonoc
occal infection requires
Infants usually present with eyelid edema, conjuncent to prevent blindness.
tival hyperemia, and ocular discharge. Age at onset

emergent treatm

and clinical features may suggest the diagnosis, but


appropriate laboratory evaluation is required (Table
J y NCTIVjjis
18-2).

_ .i_N.FECTjqys_cp_N

Non-neonatal infecti

ous conjunctivitis ("pinkeye") is


Treatment and Prevention
hood and may be bacterial or

very common in child


viral in origin. The

infection causes inflammation in


Infants with suspected gonococcal, HSV, or P. aeruouter covering of the eye over
ginosa conjunctivitis should be referred to an ophus in particular is a frequent
thalmologist. Infants with conjunctivitis due to other
nctivitis.
causes require referral if signs worsen after 3 days of

the conjunctiva, the


the sclera. Adenovir
cause of viral conju

treatment or if symptoms persist longer than 3 days.


Parents and their sexual partners should be treated
is
for Chlamydia and gonorrhea in the usual manner.
tis, chemical conjunctivitis, and
The incidence of neonatal conjunctivitis has
l abrasions) may all present with
decreased dramatically since the introduction of
A careful history may alert the
ocular prophylaxis with silver nitrate. Currently eryter two conditions. Corneal abrathromycin, effective against both C. trachomatis and
y examination after instillation of
N. gonorrhoeas, is preferred.

Differential Diagnos
Allergic conjunctivi
trauma (e.g., cornea
red, irritated eyes.
clinician to the lat
sions are revealed b
fluorescein.

TABLE 18-2
Distinguishing Features of Ophthalmia Neonatorum
Features Chemical N. gonorrhoeae
C. trachomatis
Age at onset
24 hours
2 days to 8 weeks
Clinical features
Bilateral
Unilateral or bilateral
Serous discharge
Mucopurulent discharge
Conjunctival hyperemia
Conjunctival hyperemia

2-5 days
Bilateral
Purulent discharge
Marked eyelid edema
Chemosis

Complications
Corneal scarring

Self-limited

Sepsis
Meningitis

Pneumonia
Arthritis
Corneal ulceration
Blindness
Diagnosis
Exclude serious causes
Conjunctival Chlamydia culture

Conjunctival culture
on chocolate or

Direct
Thayer-Martin agar

immunofluorescent
antibody test
Treatment
None
Oral plus topical erythromycin

Intravenous ceftriaxone
or penicillin plus
saline lavage

--------------------------------------- 165
Cha
pter 18 / Ophthalmology
TABLE 18-3
Comparison

247

of Viral, Bacterial, and Allergic Conjunctivitis

Symptom

Viral

Bacterial

Mild

Mild to moderat

Clear

Purulent

Allergic
Pain
e

None
Discharge
Clear

Mild to copious
Mild to moderate
Prone to crusting
ng
No crusting
Itching
Usually absent
Present
Injection
Diffuse
Diffuse
Vision
Normal
Normal
Clinical Manifestations
sses; the value of ophthalmic

Mild to copious
Definite crusti
Absent
Diffuse
Normal
involves warm compre
antibiotics is quest

ionable.
Table 18-3 compares and contrasts the clinical
manifestations of viral, bacterial, and allergic
all areas of granulomatous
conjunctivitis.
the meibomian glands that may

Chalazions are sm
inflammation within
progressively enlarg

e. Warm compresses and ophthalmic antibiotic/s


teroid combinations often result
Treatment
t, excision may be required.
In practice, most cases of infectious conjunctivitis are
treated with a trial of antibiotic drops or ointment
ULIT[S _____ ........
for 5 to 7 days. Refractory cases require culture
results to guide therapy. Although both viral and bac-

in resolution; if no

JP..PERJpRBITAL_CELL

terial conjunctivitis are usually self-limited diseases,


is is caused by bacterial infection of
antibiotics have been shown to limit infectivity and
ounding skin anterior to the
decrease disease duration by about 2 days, presumbrous band that separates the

Periorbital cellulit
the eyelids and surr
orbital septum, a fi
subcutaneous lid fro

m the orbit itself.


ably due to frequent flushing. Some antibiotic drops
contain steroids to decrease inflammation (e.g.,
TobraDex); these must not be given if herpes

Pathogenesis

simplex virus 1 is thought to be the cause of the


infection, because there is an increased risk of more
to the area around the eye
severe disease and visual impairment.
e skin (Staphylococcus aureus]

Bacteria gain access


through breaks in th
or via extension fro

m infected sinuses, teeth, or


other upper respirat
ory structures (streptococci,
KEY POINT
ilus influenzae, etc.). The Hib

Bacteroides, Haemoph
vaccine has greatly

decreased the incidence of


1. Steroid drops must not be given if herpes simplex
ae type B infections.
virus 1 is thought to be the cause of the conjunc-

Haemophilus influenz

tival infection, because there is an increased risk


of more severe disease and visual impairment.

Differential Diagnos

is
Orbital cellulitis,
in which the

infection extends
behind the orbital

septum, is
a true emergency.
m HORDEOLUM AND CHALAZION
eye movement,
proptosis, and
(STYES)
ility accompany this disease. A

Severe pain with


decreased ocular mob
CT scan should be ob

tained to confirm the diagnoA hordeolum is an acute infection of the meibomian


-infected structures (e.g., sinuses),
glands, small fluid-secreting structures in the tarsal
ion. Complications include
plate of the lid. Localized tender swelling progresses
scesses, meningitis, and cavernous
to a point, which ruptures to the outside. Treatment

--------------------------------------- 166
248

Blueprints Pediatrics

sis, identify any co


and delineate extens
orbital and brain ab
sinus thrombosis.

Clinical Manifestations
me is the antibiotic of choice

induration. Cefuroxi
unless the infecting

organism is thought to be StaphyThe skin around the eye is indurated, warm, and
the case, a penicillinase-resistant
tender, although there is no true eye pain. Fever is
ycin should be started, dependvariably present. The physical exam may reveal sinus
vities. The patient may be released
or tooth tenderness, sore throat, or a point of entry
oral antibiotics when symptoms
on the skin. It is important to mark the area of

lococcus; if this is
penicillin or vancom
ing on local sensiti
with 7 to 10 days of
abate.

induration to assist in following progress.


KEY POINT
Treatment
1. Orbital celluli
tis,in which the infection extends
Intraventricular antibiotics should be begun as soon
tal septum, is a true emergency.
as possible and continued until near-resolution of

behind the orbi

--------------------------------------- 167
I
Orthopedics
Pediatricians and family practitioners require a basic
examiner should loo
k for any asymmetry in the
knowledge of orthopedic principles to treat injuries,
, with the examiner's fingers on
facilitate rehabilitation, and recognize the muscuer trochanters, both Barlow's
loskeletal manifestations of many systemic illnesses.
r dislocation of the hip with
The timely diagnosis and management of genetic,

gluteal folds. Then


the greater and less
test (posterosuperio
adduction and poste

rior pressure) and the Ortolani


congenital, developmental, and infectious bone and
with a resulting "click" as the
joint conditions in children can minimize potential
the joint) are essential parts of
deformities and loss of function.

maneuver (abduction
head relocates into
every newborn evalua

tion (Figure 19-1). In examining a somewhat older


infant, a Galeazzi sign should
be sought. By holdin
g the ankles with the knees
DEVELOPMENTAL HIP _DYSPLASIA
the examiner looks for any

bent and hips flexed,


foreshortening of th

e affected limb. Older infants

Pathogenesis
h limited hip abduction and

may also present wit


apparent shortening

of the involved extremity.


Developmental dysplasia of the hip (DDH) results
when contact between the acetabulum and the head
is noted in the lateral ileum on
of the femur is lost during intrauterine development,
reas the true acetabulum is dis-

A "false" acetabulum
hip radiographs, whe
torted and shallow.

If the physical examination is


most likely due to positioning of the fetus or restrichen a bilateral hip ultrasound
tion of fetal movement in utero.

suggestive of DDH, t
should be obtained a

nd a referral to orthopedics
given.
Epidemiology
DDH is more common in females, first-born chilTreatment
dren, and breech presentations. There is also an association with other anomalies, including clubfoot,
d dislocatable hips stabilize
congenital torticollis, metatarsus adductus, and infanwithin the first 4 weeks of
tile scoliosis. The severity of dysplasia ranges from

Most subluxatable an
without intervention
life. If treatment i

s indicated in children under


subluxatable (partial dislocation induced on examiavlik harness (which keeps
nation) to dislocatable (full dislocation induced on
flexed) may be prescribed.
examination) to dislocated (abnormally positioned
most of the time).
older patients. Patients who do

6 months of age, a P
the hip abducted and
Traction is used in
not respond to conse

rvative measures require open


reduction.
Avascular necrosi
s of the femoral head is the most
Clinical Manifestations
serious complication
and is more likely to occur
Early diagnosis results in a better outcome; therefore,
een left untreated for longer
a careful newborn examination is critical. First ththae

when the child has b


n 6 months.

--------------------------------------- 168
25
Chapter 3 / Cardiology
Figure 3-7 Ebstein's anomaly (with large nonrestrictive ductusFigure 3-8
Hypopla
stic left heart syndrome in a 24-hour-old
arteriosus). Typical anatomic and hemodynamic findings
patient with falling
pulmonary vascular resistance and a noninclude: (a) inferior displacement of the tricuspid valve into therestrictive du
ctus arteriosus. Typical anatomic and hemodyright ventricle, which may also cause subpulmonary obstruc-

tion; (b) diminutive muscular right ventricle; (c) marked namic findings inclu
de: (a) atresia or hypoplasia of the left
enlargement of the right atrium due to "atrialized" portion ofventricle, mitral
and aortic valves; (b) a diminutive ascending
right ventricle as well as tricuspid regurgitation; (d) right-to-leftaorta and t
ransverse aortic arch, usually with an associated
shunting at the atrial level (note arterial oxygen saturation ofcoarctation; (c)
coronary blood flow is usually retrograde from
the ductus arteriosu
s through the tiny ascending aorta; (d) sys78%); (e) a left-to-right shunt and pulmonary hypertension sec-temic arterial ox
ygen saturation (in Fio2 of 0.21) of 80%, reflectondary to a large patent ductus arteriosus supplying the pul-ing relatively bala
nced systemic and pulmonary blood
monary blood flow; (f) low cardiac output (note low mixed flows the pulmonary ar
tery and aortic saturations are equal
venous oxygen saturation in the superior vena cava).
(see text); (e) pulm
onary hypertension secondary to the nonCloherty JP, Stark AR. Manual of Neonatal Care, 4th ed. Philadelphia:
Lippincott-Raven, 1998:426.
rteriosus; (f) minimal left atrial hypertension;

restrictive ductus a
(g) normal systemic

cardiac output (note superior vena cava


oxygen saturation of
65%) and blood pressure (65/45).
Treatment
R. Manual of Neonatal Care, 4th ed. Philadelphia:

Cloherty JP, Stark A


Lippincott-Raven, 19

98:426.
PGEi may help increase pulmonary blood flow. Congestive heart failure may be treated with digoxin and
diuretics. Propranolol may be used to suppress
supraventricular tachycardia if present. Surgical
nting in the first week of life and
therapy to repair the abnormal tricuspid valve has
e of death from congenital
had poor results.
first month of life. In this syn-

cardiac lesion prese


the most common caus
heart disease in the
drome, there is hypo

plasia of the left ventricle, aortic


valve stenosis or at
resia, mitral valve stenosis or
CYANOTIC CONGENITAL HEART
a of the ascending aorta with

atresia, and hypoplasi


discrete coarctation

of the aorta. These lesions reduce


DISEASE: LESIONS WITH DUCTALPEPEN PENT .SYSTEM ic BLOOD _FLOW
low through the left side of the

or eliminate blood f
heart, causing an ob

ligatory left-to-right shunt at the


atrial level and a r
ight-to-left shunt at the ductus
Hypoplastic Left Heart Syndrome

arteriosus. Systemic

flow is completely ductal depenHypoplastic left heart syndrome (HLHS) (Figures 3erfusion is retrograde when
8 and 3-9) is the second most common congenital
itical aortic stenosis is present.

dent, and coronary p


aortic atresia or cr

--------------------------------------- 169
250

Blueprints Pediatrics
KEY POINT
rect position requires minimal

its anatomically cor


intervention.

Developmental dysplasia of the hip may be


demonstrated on physical examination by perons and Treatment
forming Barlow's test and the Ortolan! maneuver
as well as looking for asymmetry of the gluteal
folds and the Galeazzi sign.
(in-toeing of the forefoot

Clinical Manifestati
Metatarsus Adductus
Metatarsus adductus
without hindfoot abn

ormalities) is a common, relatively benign condit


ion caused by intrauterine positioning. As opposed
to clubfoot, dorsiflexion and
plantar flexion at t
he ankle joint are unrestricted.
FOOT DEFORMITIES
ing may be beneficial. Moderate

Mild passive stretch


cases may require th

e use of straight-laced shoes.


Foot deformities predispose children to difficulty
stent cases are treated with serial
walking, poor shoe fit, and pain. Some disorders
. Surgery is rarely indicated.
correct themselves as the child begins to ambulate;
others require bracing or surgical correction. In
(Congenital Clubfoot)
general, any congenital orthopedic condition of the
or clubfoot, is a rarer but more
foot that can be molded by the examiner's hands to
ty that includes medial rotation
Figure 19-1
Barlow (above) and Ortolani maneuvers.

More severe or persi


splinting or casting
Talipes Equinovarus
Talipes equinovarus,
debilitating deformi

--------------------------------------- 170
Chapter 19 / Orthopedics
251
of the tibia, flexion at the ankle, inversion of the foot,
and forefoot adduction. Without treatment, the foot

TABLE 19-1

becomes progressively more deformed, and ulceraosis of Limp by

Differential Diagn
Disease Category

tions develop when the child is old enough to limp.

Early intervention is essential for subsequent normal

Trauma or overuse

function and development. Initial treatment consists

Fracture

of serial casting; if not improved, some patients


ury
require surgical repair, preferably before the age of

Soft tissue inj

anticipated ambulation.
s

Infectious
Septic arthriti
Osteomyelitis
Lyme arthritis

KEY POINT
1. In general, any congenital orthopedic condition of

Discitis
Inflammatory
Transient synov

itis
the foot that can be molded by the examiner's
hands to its anatomically correct position requires

Rheumatic disea

minimal intervention.

Reactive arthri

se
tis
Developmental/Acq
uired
Developmental d
ysplasia of the hip
Avascular necro
sis
LIMP
emoral epiphysis

Slipped capital f
Neurologic

Limp is probably the most common musculoskeletal


phy
complaint prompting medical evaluation in children.

Muscular dystro

Peripheral neur
opathy
Pain, weakness, decreased range of motion, and leglength discrepancy all disrupt the normal gait.

Neoplasia
Bone tumors
Leukemia

Differential Diagnosis
ors

Spinal cord tum


Metabolic

The list of conditions that present with limp is exten-

Rickets

sive (Table 19-1); some are benign and self-limited,


Hematologic
whereas others result in significant morbidity.

Sickle cell dis

ease
Hemophilia
Clinical Manifestations

Other
Appendicitis

History
tory disease

Pelvic inflamma
Testicular tors

ion
The patient's age affects the differential diagnosis.
Infection is a common etiology in younger children,
whereas Legg-Calve-Perthes disease, slipped capital
femoral epiphysis, and juvenile rheumatoid arthritis
Each joint should be
examined for range of motion,
occur in older patients. Trauma is the most commoswellingn
, warmth,
erythema, and tenderness. Fraccause of limp at any age. The absence of pain sugtures produce point
tenderness and occasionally
gests weakness or instability. Pain may be severangulatione
. Neurol
ogic evaluation includes deep
(fracture, infection), constant, associated with activtendon reflexes, s
trength, and sensation. Extremities
ity (injury), acute, or chronic. Swelling and stiffnesars
e assessed for ade
quate perfusion and deformities.
are common in rheumatologic disease. Toxic synoviMuscle atrophy and
fasciculation may be present in
tis may follow a recent viral illness. Any history of
neuromuscular diseas
e.
bowel or bladder incontinence suggests spinal cord
compression.
Diagnostic Evaluatio
n
Physical Examination
All patients with si
gnificant limp should have plain
Watching the child walk is particularly importantfilms:
. An elevated
white blood count may indicate
Certain gaits are associated with specific disordersinfection.
; if great
er than 30,000/p.L, malignant

--------------------------------------- 171
Blueprints Pediatrics
252
marrow invasion should be considered. The erythrocyte sedimentation rate is increased in both infection
and rheumatologic disease. A bone scan reveals areas
of increased blood flow consistent with inflammation. An ultrasound is useful to evaluate for the presence of an effusion, especially when a septic joint is
considered. A computed tomography (CT) scan of

the limb is rarely helpful. However, magnetic resonance imaging (MRI) is a great modality for evaluating joints, cartilage, and soft tissue. Patients with
weakness should have electrolytes, calcium, serum
creatinine kinase, and urine myoglobin studies done;
electromyography and nerve conduction studies may
also be helpful. If the weakness is progressive and
Figure 19-2
ph of a slipped capital femoral epiphysis.
limited to the lower extremities, spinal cord coms 13-year-old boy demonstrates increased
pression must be ruled out with imaging studies
left femoral epiphysis with medial and
(i.e., MRI).
gulation of the femoral head on the

Radiogra

Frog-leg view in thi


radiolucency of the
perhaps posterior an
neck.

KEY POINTS
1. Trauma is the most common cause of limp in all
age groups.
nee. Limited internal rotation and
2. Plain films are a helpful screening tool.
present on examination.
3. Any evidence of neurologic involvement (weakness, bowel and/or bladder incontinence) necessitates aggressive workup to rule out spinal cord
compression.
is

is referred to the k
limb shortening are

Differential Diagnos
The differential dia

gnosis includes trauma, LeggCalve-Perthes diseas


e, toxic synovitis of the hip, and
Slipped Capital Femoral Epiphysis
Pathogenesis
Slipped capital femoral epiphysis (SCFE) is the
gradual or acute separation of the proximal femora
tion
growth plate, with the femur rotating externally
under the capital epiphysis. The cause is unknown
child's hips in the frog-leg
but may be immunologic or hormonal in origin.
the study of choice for epiphy-

avascular necrosis.
Diagnostil c Evalua
Radiographs with the
lateral position are
seal displacement (F

igure 19-2). Radiographs may


Antecedent trauma is not a contributing factor.
idening, decreased epiphyseal

show physeal plate w


height, and a Klein'

s line (line drawn along the


Epidemiology and Risk Factors
oes not intersect the lateral

femoral neck) that d


epiphysis.

SCFE typically occurs during the adolescent growth


spurt. The incidence is highest in patients who are
male and obese. Although usually asymmetric at
presentation, 25% of cases will eventually progress to
bilateral involvement.
treatment is prevention of

Treatment
The primary goal of
further misalignment

. Pin fixation is effective in


Clinical Manifestations
hronic cases generally require

the acute setting. C

osteotomy.
History and Physical Examination
Long-term compli
cations include avascular necroThe typical patient presents with a limp and pain,
rative changes similar to those seen
which may be centered in the hip or groin but often

sis and late degene


with osteoarthritis.

--------------------------------------- 172
Chapter 19 / Orthopedics
253
KEY POINTS
SEASE _____
1. Trauma is not a cause of SCFE.

_.OSGOpD-SCHLATTER P[

Osgood-Schlatter dis
ease involves inflammation,
2. The typical SCFE patient is an obese adolescent
ness over the tibial tuberosity.
male who presents with hip or knee pain and no
between the ages of 10 and 17,
history of trauma.
t growth spurt. Repetitive stress

swelling, and tender


It typically occurs
during the adolescen
and trauma may be co

ntributing factors. Pain is worsened with kneeling a


nd crawling, but relieved by rest.
Radiographs reveal i
rregularities of the tubercle
LEGG-CALyE-PERJHES DISEASE
aziness of the adjacent metaLegg-Calve-Perthes disease is defined as avascular
cases are mild and are treated
necrosis (ischemic compromise) of the femoral epication and stretching exercises.
physis. The etiology is unknown. Eventually, the
y require casting for up to 6
ischemic bone is resorbed and reossification occurs,
bidity is quite low.
with continued (but not necessarily normal) growth.

contour and possibly h


physeal border. Most
with activity modifi
More severe cases ma
weeks. Long-term mor

Legg-Calve-Perthes disease occurs more often in


males and younger children (ages 4-8).
SIS
Clinical Manifestations

_ J.PJOPATH 1C SCOLJO

Pathogenesis
A painless limp is the most common presenting
is found in otherwise healthy
complaint. If pain is present, it is often referred to
bones, muscles, and vertebral
the knee, clouding the diagnostic picture. Range of
unknown, but familial factors
motion is limited upon abduction, flexion, and inter-

Idiopathic scoliosis
children with normal
discs. The cause is

nal rotation. The differential diagnosis is similar to


le. Scoliosis, or lateral curvature,
that for slipped capital femoral epiphysis. Initial radiKyphosis is a curvature in the
ographic studies may appear normal; subsequent

definitely play a ro
is the most common.
sagittal plane.

films demonstrate epiphyseal radiolucency. A bone


scan may be helpful to detect early impairment in
the blood supply and fragmentation and flattening o

Epidemiologf

the femoral head.


dren display some degree of spinal
Treatment
creening is very important.

Five percent of chil


deformity. Routine s
Severe scoliosis req

uiring intervention occurs more


Treatment involves containing the fragile femoral
ogression of the curve is most
head within the acetabulum, preserving its spherical
lescent growth spurt.
contour, and maintaining normal range of motion.

often in females. Pr
rapid during the ado

Younger children with minimal involvement and full


range of motion may be observed. Orthotic bracing
Clinical Manifestati
ons
or surgery is necessary in older patients with significant changes in the femoral head. The amount
Examination
and area of ischemic damage affect the prognosis.
is not associated with back pain
Collapse of the femoral head is the most serious
ptoms warrant further investigaacute complication; long-term disability is related
xamination consists of two parts.
to abnormal or asymmetric growth.
examined from the rear while stand-

History and Physical


Idiopathic scoliosis
or fatigue; such sym
tion. The physical e
First, the child is
ing up. Shoulder gir

dle and iliac crest areas are noted


for symmetry and hei
ght. Then, the Adam's forward
KEY POINT
bending test is perf
ormed. The child bends forward
1. The typical patient with Legg-Calve-Perthes
the arms hanging freely. The
disease is a young male child who presents with a
h from in front and behind the
painless limp and knee pain.
alignment of the spinous processes

from the waist with


examiner should watc
patient to look for
and asymmetry of rib

height.

--------------------------------------- 173
254
Blueprints Pediatrics
Differential Diagnosis
and an abnormally small

monary complications
foramen magnum that

predisposes to brainstem
Occasionally, scoliosis may be due to neuromuscular
abnormalities or congenital deformities. Scoliosis
should not be confused with kyphosis, an increase in
the posterior convexity of the thoracic spine. Kyphosis is usually postural and responds well to specific
daily exercises; inflexible kyphosis may be caused
by wedge-shaped vertebral bodies (Scheuermann
disease) and may require bracing.
n deserve special attention

compression.

COMMON FRACTURES IN
CHILDREN
Fractures in childre
because their bones

are characteristically different


Treatment
. For one thing, they are more

from those of adults


porous, which limits

fracture propagation. Ligaments


Curvatures less than 25 need only be followed.
vely stronger than bones;
More pronounced deformity in a child who is still
cause sprains or tears in adults
growing should be treated with external bracing
n children. Fractures through the
until the growth spurt is completed. Bracing does not
ate require particular care,
reduce the curve, but it does halt progression and ibecauss
esult in deformity or limb-length
85% effective if used correctly. Unfortunately, compliance tends to be low. Curvature that is greater
than 40 to 50 after the growth spurt will continue
to progress; such patients require spinal fusion to
ons
reduce the curve and stabilize the spine. Curves of
50 or greater are associated with decreased vital
xamination
capacity and low functional pulmonary reserve.
ive for trauma in virtually all cases

and tendons are relati


injuries that would
can fracture bones i
epiphyseal growth pl
e they may r
discrepancy.
Clinical Manifestati
History and Physical E
The history is posit
of nonpathologic fra

ctures; caretakers who have


abused a child may n
ot offer this information.
Isolated point tende
rness occurs over the site of the
KEY POINTS
is variably present and may be
1. Scoliosis is more common in adolescent females
than in males.
2. Idiopathic scoliosis does not result in back pain or
fatigue.
sis
3. Bracing is recommended for curves greater than
25.
ccur when the force applied
4. Bracing halts curve progression; it does not

fracture. Angulation
quite subtle.
Differential Diagno
Greenstick fractures o
breaks one side of a

bone and bends the other. A fraccorrect the curvature already there.
the bone is broken through both

ture is complete if
sides. Spiral fractu

res are often the result of child


abuse. When a spira
l fracture is diagnosed, obtaining
a careful history o
f the event is warranted. Epiphyseal fractures disr
upt the growth plate, the weakest
ACHONDROPLASIA
d's skeletal system. Epiphyseal

portion of the chil

fractures are categ


orized according to the SalterAchondroplasia is a disorder of cartilage calcification
Harris classificati
on (Figure 19-3). Torus fractures or
and remodeling. Inheritance is autosomal dominant.
buckle fractures oc
cur at the metaphysis due to a
The physical appearance is strikingly characteristic:
compressive load th
at causes a buckle in a small area.
These patients are very short with proportionally
Stress fractures ar
e hairline cracks related to repetilarge heads. Long bones tend to be wide, short, and
tive activity and a
re usually seen in athletes. Pathocurved, and digits are short and stubby. Kyphoscollogic fractures res
ult when underlying disease
iosis and lumbar lordosis may be quite pronounced.
weakens the bone, a
s may occur in osteogenesis
Heterozygotes have fairly normal intelligence, sexual
imperfecta, maligna
ncies, long-term steroid use,
function, and life expectancy. Homozygotes fare lesinfections
, endocrin
e disorders, and some inborn
well, given their increased susceptibility to pulerrors of metabolis
m.

--------------------------------------- 174
255
Chapter 19 / Orthopedics
Type
Type III
(excellent prognosis)
(good prognosis)

Type II
(excellent prognosis)
Type IV Type V
(high risk for growth disturbance)

Figure 19-3

Epiphyseal fractures: Salter-Harris classification.

Treatment
ommon denominator in all vari-

brittle bones. The c


ants is the abnormal

synthesis of type I collagen,


Most fractures can be adequately treated with exteritutes about 90% of the bone
nal stabilization. Fractures that are unstable, mis-

which normally const

aligned, or through the growth plate require


ispersed in the teeth, ligaments,
operative reduction. In younger children, bony overrae. The most severe form is type

matrix but is also d


skin, ears, and scle
II, or fetal OI, whi

ch results in multiple intrauterine


growth at the site of the fracture may produce limb
and is uniformly fatal in the periangulation or asymmetric length if not correctly set.
tance is autosomal dominant in

and birth fractures


natal period. Inheri
most cases.

KEY POINTS
1. Fractures through the growth plate may result in
Clinical Manifestati
ons
deformity or leg-length discrepancy.
2. Spiral fractures suggest child abuse.
pends on the subclass of OI (Table

Clinical severity de
19-2). Some variants

cause death early in life; others


present with only mo
derately increased susceptibility to fractures. Bl
ue sclerae are a characteristic
qSJEOGENESIS IlyiPERFECTA
of the disease. Short stature is

feature in some forms


not uncommon as a re

sult of frequent recurrent fracOsteogenesis imperfecta (OI) describes a group oturesf


sociated with OI occasionally raise
closely related genetic disorders resulting in fragileth,
ld abuse.

. Fractures as
e suspicion of chi

--------------------------------------- 175
256

Blueprints Pediatrics
TABLE 19-2

Classification of Osteogenesis Imperfecta


Syndrome Orthopedic Manifestations
Life Expectancy

Nonorthopedic
Manifestations

Type 1
nductive
Type II
wth

Neonatal fractures; bow legs; kyphoscoliosis;


Generally shortened
joint laxity; mild short stature

Blue sclerae; co

Short, deformed limbs; severe bone fragility


Days

Intrauterine gro

hearing loss

retardation; s
tillbirth
Type III Neonatal fractures; severe bone fragility;

Blue sclerae

Infancy/childhood
lower limb deformities; short stature
Type IV

Increased susceptibility to fractures


Near normal

Treatment
r two-thirds of cases. Infection

Blue sclerae
and tibia account fo
usually begins in th

e metaphysis, an area of relative


Treatment involves standard fracture care, pneumatic bracing, and careful avoidance of even minor
phagocytes.
trauma.

blood stasis and few

Epidemiology and Ri
sk Factors
KEY POINTS
he neonatal period and again in

Incidence peaks in t
older children (ages

9-11), when it becomes more


1. Type II Ol is the most severe form, resulting in
predominant organism in all
intrauterine or perinatal death.
2. Patients with Ol types I, III, or IV may have blue
ylacoccus aureus. Osteomyelitis
sclerae.
treptococcal and Haemophilus

common in males. The

age groups is Staph


caused by group A s
influenzas infection

occurs in children as well. Group


B streptococci and E
scherichia coli are important
pathogens in the neo
nate. Patients with sickle cell
SUBLUXATION OF THE RADIAL
rly susceptible to Salmonella
HEAD
sionally, osteochondritis of the

disease are particula


osteomyelitis. Occa
foot may result from

puncture wounds through


Subluxation of the radial head, or nursemaid's
ases, the organisms involved are
elbow, is one of the most common injuries seen in
osa or S. aureus. Treatment may
young children. The history is often remarkable for a
bridement.
sudden strong jerking of the child's pronated hand,

sneakers. In these c
Pseudomonas aerugin
require surgical de

resulting in rapid extension at the elbow. The patient


holds the arm slightly flexed with the hand pronated.
ions
Motion at the elbow is limited. Treatment consists of
l Examination
holding the patient's elbow at 90 flexion and firmly

Clinical Manifestat

manipulating the forearm into supination.


h a history of fever and refusal to

Infants present wit

History and Physica

move the involved l

imb. Older patients also complain of localized


bone pain. The physical examination may reveal sof
t tissue swelling, limited range of
JP_ .OS JJEOM YELJTIS ______
a. Occasionally, sinus tracts will

motion, and erythem


drain purulent flui

d onto the skin surface.


Pathogenesis
Bone infections require early recognition and agsis
gressive treatment to effect a favorable outcome.

Differential Diagno

Hematogenous seeding is the usual source of origin;


d malignant invasion of the bone
trauma seems to increase susceptibility. The femur
th similar symptoms. Range of

Traumatic injury an
may also present wi

--------------------------------------- 176
Chapter 19 / Orthopedics
257
motion generally remains intact in patients with
osteomyelitis, as opposed to those with septic arthri-

_ .SEPTIC ARTHRITIS

tis and epiphyseal disorders.


Pathogenesis
Septic arthritis (pu
rulent infection of the joint space)
Diagnostic Evaluation
otentially more debilitating
White blood count is often within the normal range.

is more common and p


than osteomyelitis.

Only 50% to 60% of blood cultures are positive.


Aspiration of the involved bone is imperative for

Epidemiology

recovery identification, and sensitivity testing of the


causative organism, especially if initial blood cultures
hest in infants and young chilare negative. Radiographs are initially normal but
e hip is the most common site, and
demonstrate periosteal elevation or radiolucent
t likely pathogen. The knee is
necrotic areas in 2 to 3 weeks. Bone scans are posiin older children; S. aureus is still
tive within 24 to 48 hours. Markers of inflammation
, although streptococci and
are usually positive. An elevated C-reactive protein
ia are not uncommon. Neissevalue will be seen in 98% of cases and will return
be considered in the sexually
to normal within 7 days of effective treatment. In

The incidence is hig


dren. In infants, th
S. aureus is the mos
more often involved
the primary organism
gram-negative bacter
ria gonorrhoeae must
active adolescent, e

specially if multiple joints are


addition, the erythrocyte sedimentation rate will be

involved.

elevated in 90% of cases, but will require 3 to 4 weeks


to return to normal.
Clinical Manifestati
ons
Treatment
Examination
Treatment consists of intravenous or high-dose oral
sents as a painful joint, often
antibiotics for 4 to 6 weeks. Initially, broad-spectrum
, irritability, and refusal to bear
antistaphylococcal agents, such as oxacillin, are
on, range of motion is clearly
appropriate. Cefuroxime may be chosen if immurythema, warmth, and tenderness

History and Physical


Septic arthritis pre
accompanied by fever
weight. On examinati
limited; swelling, e
are also present to

varying degrees.
nization against H. influenzas type b is incomplete.
Treatment of neonates requires coverage for group
B streptococci and gram-negative bacilli. When the
is
organism has been recovered and sensitivities are

Differential Diagnos

available, therapy may be narrowed. Most patients do


thritis should be considered
not require surgery.
diagnosis. Toxic synovitis is a freAbscess formation within the metaphyseal shaft is
pain in children. It has not been
not uncommon. If the infection extends to the epito be an infectious condition,
physeal plate, growth deformities may occur. Septic
llows viral illnesses. The hip is

Osteomyelitis and ar
in the differential
quent cause of joint
definitively proven
although it often fo
most commonly involv

ed. In contrast to septic arthriarthritis is also a known complication.


is minimally limited, and the

tis, range of motion


white blood cell cou

nt, sedimentation rate, and


KEY POINTS
fever curve are usua
lly normal to slightly elevated. In
1. The peak incidence of osteomyelitis is bimodal
many causes of reactive or postin(neonatal period and ages 9-11).
hat may present in a similar
2. Only about half of blood cultures are positive,

addition, there are


fectious arthritis t
manner.

so aspiration of the bone yields invaluable


information.
3. The bone scan is more sensitive than plain films

Diagnostic Evaluatio

n
early in the disease process.
novial fluid usually yields a
4. S. aureus is the most common pathogen in all age
nt in excess of 25,000 and a
groups. It is also the most common pathogen in
The exception is N. gonorsickle cell patients, who are particularly susceptificult to recover; blood, cervical,
ble to Salmonella.
yngeal cultures may be more

Aspiration of the sy
white blood cell cou
pathologic organism.
rhoeae, which is dif
rectal, and nasophar
helpful.

--------------------------------------- 177
258
Blueprints Pediatrics
Treatment
Y POINTS
Delay in treatment may result in permanent
destructive changes and functional impairment. Intracause of septic arthritis in
venous antibiotic therapy remains the treatment of
ldren is S. aureus.
choice; conversion to oral therapy is appropriate when
must be considered in the sexually
sensitivities are known and symptoms substantially
nt.
improve. A septic hip is an orthopedic emergency that
requires surgical drainage and irrigation.

KE
1. The most common
infants and chi
2. N. gonorrhoeae
active adolesce

--------------------------------------- 178
Respiratory diseases rank as the second leading cause
of death in children younger than 4 years in the
Western world. Exchange of oxygen and carbon
dioxide depends on the adequate function of the
many components of pulmonary physiology.
disease of reversible airway
Changes in the upper or lower airways (obstructive
rized by bronchial hyperrespondiseases), compliance (restrictive lung diseases), venon, and mucous secretion.
tilation or perfusion of the lung parenchyma, or
abnormalities in control of ventilation can all lead to
results from smooth muscle
clinically significant pulmonary disease.
cur after allergic, environmental,

ASTHMA (REACTIVE
AIRWAYS DISEASE)
Pathogenesis
Asthma is a chronic
obstruction characte
siveness, inflammati
Bronchospasm, which
constriction, may oc
infectious, or emoti

onal stimuli. Common precipitants include cigare


tte smoke, upper respiratory
_O_BS_T_R_UCJJVE LUNG DISEASE
, dust mites, weather changes,

infections, pet dander


exercise, and season

al or food allergens. The inflam-

Inspiration is achieved by contraction of the


he airways has both an immedidiaphragm and accessory muscles of the thoracic
esponse; it is the latter that results
wall. The resulting negative intrathoracic pressure
way hyperresponsiveness characfacilitates air flow from outside the body into the
a exacerbation.
lungs. During inspiration, negative intrathoracic press classified based on the degree
sure tends to stent open intrathoracic airways (i.e.,
to initiation of appropriate
bronchi and bronchioles) and exacerbate collapse of
.
extrathoracic airways. With expiration, recoil of the
lung itself and of the thoracic wall creates positive
intrathoracic pressure leading to expiratory airflow.

matory response in t
ate and late-phase r
in the prolonged air
teristic of an asthm
Asthma severity i
of impairment prior
therapy (Table 20-1)
Epidemiology
Reactive airways dis

ease (RAD) is the most frePositive intrathoracic pressure contributes to lower


pulmonary disease in children,
airway collapse during expiration and tends to stent
s on the rise despite advances in
open the upper airways. The general result of these
ost common reason for hospitaleffects is that lower airway obstruction tends to
practice. Ninety percent of
present with signs and symptoms of air trapping (difore the age of 6 years. Boys are
ficulty with expiration), and upper airway obstructen as girls before adolescence, at
tion presents with evidence of difficulties with
rs become equal.
inspiration (Figure 20-1). These principles will be
evident in many of the diseases discussed in this
chapter.

quently encountered
and its prevalence i
therapy. It is the m
ization in pediatric
patients present bef
affected twice as of
which time the numbe
Risk Factors
Risk factors include

genetic predisposition, atopy,


cigarette smoke expo
sure, living in urban areas,
poverty, and African
-American race. Respiratory syn-

--------------------------------------- 179
26

Blueprints Pediatrics
surgery is available

. The stage I (or Norwood) palliation, which is perfo


rmed in the first week of life,
86%
50/40

allows the majority

of neonates to survive infancy.


The stage I procedur
e involves amalgamation of the
pulmonary artery and
aorta to provide unobstructed
systemic blood flow,

atrial septectomy, and modified


Blalock-Taussig shun
t to provide restrictive pulmonary blood flow. A
fter the stage I procedure, a
cavopulmonary anasto
mosis is performed at 4 to 6
months of age and a
modified Fontan procedure is
generally performed
at 2 to 4 years of age. Some
centers do not perfo
rm the stage I palliation and
proceed directly to
heart transplantation.
Interrupted Aortic A
rch
There are three type
s of interrupted aortic arch
(Figure 3-10): Type
A is interruption beyond the left
subclavian artery, t
ype B is interruption between the
left subclavian and
left common carotid arteries,
and type C is interr
uption between the left comAcute circulatory collapse following constriction ofmon carotid and t
Figure 3-9
he brachiocephalic arteries. In
the ductus arteriosus in hypoplastic left heart syndrome.Thesethis anomaly, syst
emic blood flow is dependent on
neonates are typically in shock with poor perfusion, tachycar-patency of the duc
tus arteriosus, which shunts blood
dia, acidosis and in respiratory distress. Note (a) the low cardiac
from the pulmonary a
rtery to the aorta. Interrupted
output (as evidenced by the low mixed venous oxygen saturation in the superior vena cava of 55%); (b) narrow pulse
aortic arch is often
associated with DiGeorge's
pressure; (c) elevated atrial and ventricular end-diastolic pres-syndrome, due t
o the 22qll microdeletion.
sure elevated left atrial pressure may cause pulmonary
edema (note left atrial saturation of 93%); (d) significantlyClinical Manifestat
ions
increased pulmonary blood flow, as reflected in an arterial Pulmonary edema occu
rs almost immediately. The
oxygen saturation (in Fi02 of 0.21) of 86%.
clinical presentatio
n is similar to that of critical
Cloherty JP, Stark AR. Manual of Neonatal Care, 4th ed. Philadelphia:
Lippincott-Raven, 1998:426.
coarctation of the a
orta.
Treatment
Clinical Manifestations
PGEj therapy should
begin immediately to maintain
systemic blood flow
via the right-to-left shunt
As the ductus closes, neonates with HLHS have
through the patent d
uctus arteriosus. Emergent

severely diminished systemic blood flow and present


in shock. They manifest signs of congestive heart
to reanastomose the aortic

surgery is necessary
segments.

failure with moderate cyanosis, tachycardia, tachypnea, pulmonary rales (from pulmonary edema), and
hepatomegaly. Poor or absent peripheral pulses and
vasoconstricted extremities are characteristic. The
L
cardiac examination reveals an S
3 and a loud single
S2. The ECG shows decreased R wave progression
across the precordium. The chest radiograph reveals
fects that result in increased
pulmonary edema.
include atrial septal defect,

ACYANOTIC CONGENITA
HEART DISEASE
Acyanotic cardiac de
pulmonary blood flow
ventricular septal d

efect, patent ductus arteriosus,


Treatment
and common atriovent
ricular canal. Acyanotic lePGEi should be started to maintain ductalpulmonary venous hypertension
dependent systemic blood flow. No corrective
of the aorta and aortic valve

sions that result in


include coarctation

--------------------------------------- 180
260

Blueprints Pediatrics
TABLE 20-1

Classification and Maintenance Treatment of Asthma


Severity Symptoms Maintenance Medications, Age < 5 Yr
Maintenance Medications, Age > 5 Yr
Preferred
Preferred
Alternative
Mild intermittent <2 days/wk and/or None
None
None
<2 nights/mo
Mild persistent >2 days/wk and/or Low-dose inhaled
Low-dose
Cromolyn, leukotriene
>2 nights/mo corticosteroid
inhaled
receptor antagonist,

Alternative
None

Cromolyn or
leukotriene
receptor

corticosteroid nedocromil, or
antagonist
sustained-release
theophylline
Moderate persistent Daily and/or >1 Low-dose inhaled
Low- to
Low- to medium-dose
night/wk corticosteroid
medium-dose
inhaled corticosteroid
and long-acting
inhaled
and either leukotriene
inhaled pY

Low-dose inhaled
corticosteroid and
either leukotriene
receptor

corticosteroid receptor antagonist or


and long-

agonist or

antagonist or

medium-dose

theophylline

theophylline

acting inhaled
inhaled
pYagonist
corticosteroid
Severe persistent Continual daily, High-dose inhaled
High-dose
None accepted
and frequent corticosteroid
inhaled
nighttime and long-acting
corticosteroid
inhaled pY
and longagonist and (if
acting inhaled
needed) oral
pYagonist
corticosteroid
and (if

None accepted

needed) oral
articosteroid
Inspiration
Expiration
cytial virus (RSV) infection necessitating hospitalization has also been associated with a higher
incidence of subsequent asthma.
Clinical Manifestations
Extrathoracic
obstruction
History and Physical Examination
The presentation of asthma is varied. The history
may be positive for wheezing with colds, decreased
exercise tolerance, or persistent nighttime coughing.
Children with acute attacks present in respiratory
distress with dyspnea, wheezing, subcostal retractions, nasal flaring, tracheal tugging, and a prolonged
Intrathoracic
expiratory phase as a result of obstruction of airflow.
obstruction
Cyanosis is uncommon. The absence of wheezing
with poorly heard breath sounds is an ominous sign,

indicating that the child's respiratory system is too


obstructed to move air. Mental status changes
Figure 20-1
f inspiration and expiration on the caliber
suggest advanced hypercarbia and impending respig extrathoracic and intrathoracic airway
ratory arrest.

--------------------------------------- 181
Chapter 20 / Pulmonology
Differential Diagnosis

261

When an infant presents with wheezing and respiratory distress, the differential diagnosis includes bronchiolitis, foreign body aspiration, gastroesophageal
reflux with aspiration, tracheoesophageal fistula,
and vascular sling. Anaphylaxis and angioneurotic
edema may cause wheezing at any age. Coughvariant asthma produces a chronic nighttime cough
similar to that accompanying postnasal drip, bronchitis, or cystic fibrosis; wheezing may or may not be
present.
Diagnostic Evaluation
The chest radiograph demonstrates significant hyperinflation and occasionally atelectasis (Figure 20-2).
CO 2 retention occurs early and may be quite dramatic; hypoxemia is usually less pronounced.
Treatment
With appropriate therapy and compliance, most
patients with mild asthma can remain symptom free
with few exacerbations. The most effective form of
treatment consists of removing inciting agents from

Effect o

of the airways durin


obstruction.

the child's environment. Cigarette smoke should be


strictly avoided. Limiting dust mite, mold, and pet
exposure is beneficial to patients with an allergic
component to their RAD.
The mainstays of medical maintenance therapy
are inhaled corticosteroids, p2-agonists, and leukotriene receptor antagonists. p2-Agonists such
as albuterol reduce smooth muscle constriction and
may be administered orally or via nebulization or
metered-dose inhalation. Longer-acting preparations
(salbutamol) are available for patients requiring daily
P2-agonist therapy. p

2-Agonists are effective in pre-

venting exercise-induced asthma if used 30 minutes


before vigorous activity. The abuse of bronchodilators may result in tolerance to their effects.
The advent of inhaled corticosteroid therapy has
had a remarkable impact on the treatment of RAD.
Aerosolized formulas are breathed directly into the
lungs, with a substantial decrease in systemic side
effects. Their use as a daily medication in persistent
diograph of a 3-year-old taken during

Figure 20-2

Chest ra

a RAD exacerbation s
hows severe hyperinflation, increased
and severe asthma has become the standard of
eter of the chest, a depressed diaphragm,
care. Increasing the dose of inhaled corticosteroids
atelectasis.
is becoming an important part of the initial response

anteroposterior diam
and several areas of

to an asthma exacerbation managed at home. A

--------------------------------------- 182
262 Blueprints Pediatrics
nebulized formulation of budesonide has recently
become available in the United States, making
Y POINTS
inhaled corticosteroids an option for younger chil-

KE
1. The three compo

nents of RAD are bronchospasm,


dren. Five-day oral pulse steroid treatment remains
mucous secretio
n, and inflammation.
one of the mainstays of treatment during an acute
2. Bronchodilators
are the treatment of choice in an
exacerbation.
acute asthma ex
acerbation.
Leukotriene receptor antagonists are oral medica3. Inhaled cortico
steroids and leukotriene inhibitors
tions recommended for the treatment of chronic
have improved s
ymptom control for patients with
moderate to severe asthma and may allow some
moderate to sev
ere asthma.
patients to reduce their dependence on (3
2-agonis 4t. The disappeara
nce of wheezing with increased
and daily inhaled steroid use. Cromolyn sodium is
respiratory dis
tress signals increased obstruction
another preventative medication that is being
rather than imp
rovement.
used less frequently since the advent of inhaled
5. The effects of
oral or intravenous corticosteroids
steroids. It works by stabilizing the mast cell memoccur 4 to 6 ho
urs after administration.
brane, preventing release of inflammatory mediators
such as histamine. It is available in nebulized and
metered-dose inhaler forms and is well tolerated,
with no known adverse effects. It is not helpful
during an acute attack, but is a good form of
history of intubatio
n, black race, and steroid
prevention.
The use of theophylline, once a commonly predependence.
scribed oral bronchodilator, has fallen out of favor as
a first-line treatment option. It has virtually no anti- .CYSTIC n BROS) s
inflammatory properties, is often poorly tolerated,
and requires frequent drug-level monitoring.
Children who present to the emergency departPathogenesis
ment in an acute asthma attack are initially assessed
for airway patency and ability to aerate. Pulse oximeCystic fibrosis (CF)
is a multisystemic inherited
disease characterize
d by disordered exocrine gland
try measurement is a simple, rapid screen for hypoxemia. Patients in severe respiratory distress require
function. The produc
t of the cystic fibrosis transregarterial blood gas measurements to assess the need
ulator (CFTR) gene i
s a cell membrane protein that
for supplemental oxygen and to recognize increasing
functions as a cAMPactivated chloride channel on
PaCO 2, a sign of impending respiratory failure. (A
the epithelial cells
of the respiratory tract, pancreas,
normal PaCO
2 in the face of tachypnea is an equally
sweat and salivary g
lands, intestines, and reproductive system. This ch
annel is nonfunctional in patients
ominous sign, because the PaCO
2 should be well
below 40 with a rapid respiratory rate.) Nebulized
with CF, so chloride
remains sequestered inside the
bronchodilators are administered continuously if
cell. Sodium and wat
er are drawn into the cell to
needed. Subcutaneous epinephrine or terbutaline
maintain ionic and o

smotic balance, resulting in relrapidly decreases airway reactivity. Corticosteroidsativ,


the cell surface and abnormally

e dehydration at
viscid secretions.

administered orally or intravenously, require 4 to 6


hours for a response but are indicated for treatment
of inflammation and prevention of the late-phase
response. Children who do not respond with
complete resolution of symptoms after severaCl
ugh autosomal recessive inherihours (i.e., children in status asthmaticus] or those
e frequency of 1 in 3500 live
who require ongoing oxygen therapy should bwhite
in 17,000 black infants. The gene
hospitalized for continued treatment and close
equency in other populations.
observation.
ne mutations [located to a gene
Despite advances in therapy, the mortality rate
7} have been described; 70%
for RAD in children has continued to rise over
utation at position 508 of CFTR.
the past two decades. Factors that increase the risk
ctancy is currently 31 years and
of death include noncompliance, delay in treatment,
ically in the past decade.

--------------------------------------- 183
Chapter 20 / Pulmonology
TABLE 20-2

263

Clinical Manifestations of Cystic Fibrosis


Respiratory
Nasal polyps
Sinusitis
Cough
Bronchiectasis
Clubbing/cyanosis
Recurrent pneumonia
Reactive airway disease
Hemoptysis
Pneumothorax
Cor pulmonale
Hepatobiliary

Epidemiology
F is acquired thro
tance, with a diseas
e births and 1
occurs with lower fr
Over 700 distinct ge
locus on chromosome
of patients have a m
The median life expe
has increased dramat

Obstructive neonatal jaundice


Portal hypertension
Cirrhosis
Gastrointestinal
Meconium ileus
Distal intestinal obstruction syndrome
Pancreatic insufficiency
Malabsorption
Failure to thrive
diograph in this adolescent male with
Pancreatitis
nstrates marked chronic disease and bleb
Diabetes

Figure 20-3

Chest ra

cystic fibrosis demo


formation.

Rectal prolapse
Duodenal ulcers
Other
Hyponatremic dehydration
Metabolic alkalosis
manifestations include pancreatic
Impaired fertility (males)
obstruction and rectal prolapse,

Gastrointestinal
insufficiency, bowel
diabetes, and hepati

c cirrhosis. Interference with


normal pancreatic en
zyme secretion leads to
decreased fat absorp
tion; parents may notice that the
Clinical Manifestations
arge and bulky. Failure to thrive is
History and Physical Examination
festation of CF in infants and

child's stools are l


the most common mani
children. In the neo

nate, meconium ileus is pathogThe most common presenting signs and symptoms

nomonic for CF.

of CF are listed in Table 20-2. All levels of the respiratory tract may be affected, including the nasal
passages, sinuses, and lower airways. Nasal polyps in
any pediatric patient should prompt further testing
Diagnostic Evaluatio
n
for CF. Opacification of the sinuses and sinusitis are

extremely common. Mucus stasis and ineffective


Recurrent lower airw
ay infection results in
clearance potentiate repeated bacterial pneumonias.
bronchiectasis, fibr
osis, parenchymal loss, and the
Frequent pathogens include Staphylacoccus aureus,
characteristic "bleb
" formation found on chest radiHaemophilus influenzae, and, most importantograph,
s [Figure 20
-3). Pulmonary function tests
Pseudomonas aeruginosa; 90% of patients acquire Pdemonstrat.
e both
obstructive and restrictive
aeruginosa, and it is seldom eradicated. Colonization
changes. The sweat c
hloride level remains the diagwith Burkholderia cepacia is particularly ominounostis
c test of choi
ce; a level greater than 60mEq/L
and is associated with accelerated pulmonary
is considered abnorm
al. Genetic and prenatal testing
deterioration and death. Digital clubbing is almost
are now available fo
r the 14 most common gene
universal.
mutations, accountin
g for 85% of cases.

--------------------------------------- 184
264
Blueprints Pediatrics
Treatment
Y POINTS
Chest physical therapy, exercise, and frequent coughis a disorder of exocrine gland funcing are helpful in mobilizing secretions. Bronthe lungs, sinuses, pancreas, sweat
chodilators and anti-inflammatory medications relax
smooth muscle walls, decrease airway reactivity,
ands, intestines, and reproductive
and curb tissue destruction. DNase (Pulmozyme),
autosomal recessive.
administered via nebulization, breaks down thick
far more prevalent in whites than in
DNA complexes present in mucus as a result of cell
destruction and bacterial infection. Normal growth
can often be achieved with pancreatic enzyme
ve is the most common presentation

KE
1. Cystic fibrosis
tion, affecting
and salivary gl
system.
2. Inheritance is
3. The disease is
other races.
4. Failure to thri
of CF in childr

en.
replacement, fat-soluble vitamin supplements, and
in the neonate is pathognomonic
high-calorie, high-protein diets. Nasogastric or gastrostomy tube feedings may be instituted if oral
at chloride level is diagnostic.
intake is inadequate.
s aggressive nutritional support,
Frequent disease exacerbations may be triggered
by viral or bacterial infections and are treated by
ol, chest physical therapy, and

5. Meconium ileus
for CF.
6. An elevated swe
7. Therapy involve
infection contr
dornase.

aggressive chest physical therapy, postural drainage,


and antibiotics, which may be given orally if the exac-

erbation is mild and the organisms are not resistant.


Usually, however, bacterial infections must be treated
with an aminoglycoside (e.g., tobramycin) and a
SEASE
semisynthetic penicillin or cephalosporin, depending
on organism sensitivities. Research aimed at proends from the nose to the
viding gene replacement therapy is currently under
e structures are intrathoracic and
way.
Obstruction or dysfunction of
Prognosis continues to improve with aggressive
s in the upper airway can lead to

UPPER AIRWAY DI
The upper airway ext
carina. Some of thes
some extrathoracic.
any of the structure
disease.

treatment of pulmonary exacerbations and optimal


nutritional support. Respiratory complications remain the major contributors to morbidity and mortality in CF.
Hemoptysis is an alarming development that may
he most proximal abnormality of
occur during pulmonary exacerbations in longauma during vaginal delivery can
standing disease. Frequent coughing and inflammaryngeal nerve damage with vocal
tion lead to erosion of the walls of bronchial arteries
ma from prolonged intubation can
in areas of bronchiectasis, and expectorated sputum
t long-term subglottic stenosis.
becomes streaked with blood. Blood loss of more
an leave the trachea or larynx
than 300mL/day or lOOmL/day for 3 consecutive
as laryngotracheal malacia. The
days is considered an emergency, often treated by
obstructed by congenital malarterial embolization.
emangiomas, laryngeal webs, or
Pneumothorax is another potentially lifeall hypopharynx (associated with
threatening complication that may occur in CF. It is
e) or a big tongue (in Down
characterized by the sudden onset of severe chessyndromet
so cause obstruction.
pain and difficulty breathing. Placement of a chest
n
tube results in rapid reexpansion, but over half of
pneumothoraces recur unless sclerosis or surgery is
emember that young infants are
performed.
ers. As a result, bilateral choanal
Progressive obstruction and hypoxia in advanced
significant cyanosis and is lifedisease can lead to chronic pulmonary hypertension
ateral, cyanosis may be evident
and right heart failure (cor pulmonale). For CF
Extrathoracic disease processes
patients with a life expectancy of 1 to 2 years, lung
, tachypnea, inspiratory retractransplantation is a potentially viable option.
ly apnea. Infants with intratho-

The Infant
Choanal atresia is t
the upper airway. Tr
lead to recurrent la
cord paralysis. Trau
result in significan
Immature cartilage c
floppy, referred to
upper airway may be
formations such as h
vascular rings. A sm
Pierre-Robin syndrom
) can al
Clinical Presentatio
It is important to r
obligate nose breath
atresia can lead to
threatening. If unil
only during feeding.
present with stridor
tions, or occasional

--------------------------------------- 185
Chapter 20 / Pulmonology
265
racic abnormalities may present with respiratory
ity and chronic hypercarbia
distress or wheezing. A hoarse or absent cry may
ckian syndrome. Much more
indicate vocal cord dysfunction.
however, is obstruction due to

associated with obes


is termed the Pickwi
common in children,

anatomic abnormaliti
es (large tonsils and adenoids,
Diagnostic Evaluation
macroglossia) or ins
ufficient airway tone (tracheoPulse oximetry can quickly assess the level of hypoxmalacia or laryngoma
lacia). Polysomnography, which
emia, but an arterial blood gas measurement may be
measures respiratory
effort, air flow, oxygenation, and
needed to evaluate the degree of respiratory comheart rate, can be
helpful in determining the type and
promise in an infant in respiratory distress. Inabilitseverity
y of the apne
ic events.
to pass a nasogastric tube is suggestive of choanal
Some children's s
ymptoms are relieved with
atresia. Lateral neck radiographs may demonstrate
removal of the adeno
ids or tonsils or both. Othersubglottic stenosis, but bronchoscopy may be needed
wise, treatment invo
lves overnight continuous posito confirm vocal cord abnormalities or laryngotrative airway pressure
(CPAP) or, in very severe cases,
cheal malacia. A chest radiograph demonstrating a
tracheosfomy.
right aortic arch should prompt consideration of a
vascular ring. A barium swallow may help delineate
this process.
- - AJ?NPA OF INFANC
Y
Treatment
Mild to moderate congenital stridor may be followed
the cessation of breathing for

Apnea is defined as
longer than 20 secon

ds or pauses of any duration


with close observation, but any degree of obstruction
r changes (cyanosis, pallor),
may be exacerbated by respiratory infections. Severe
responsiveness, or bradycardia.
respiratory distress mandates immediate endotraeurally mediated), obstructive, or
cheal intubation. Some disorders require a surgical
a diagnosis but a potentially dantracheostomy to bypass the obstruction on a longiring aggressive workup to
term basis. Choanal atresia and vascular rings are
the underlying cause.

associated with colo


hypotonia, decreased
It may be central (n
mixed. Apnea is not
gerous symptom, requ
determine and treat

repaired surgically.
Older Child
ons and Treatment
Obstruction of the upper airway in the older child
ening Events
may result from incomplete resolution of infant
of prematurity, apnea of incauses, but additional processes need to be considfancl-term infants. Often the disorder
ered. A number of infectious etiologies, including
ention after an apparent lifeepiglottitis, peritonsillar abscess, retropharyngeal
LTE). ALTEs are very frightenabscess, infectious mononucleosis, bacterial tra; the infant either stops breathing
cheitis, and croup, are important causes of upper
nd may be cyanotic or pale, hypoairway obstruction and are discussed in Chapter 12.
rouse, or choking and gagging. The
Anaphylaxis causes acute upper airway obstruction
ves that the child would have
and is addressed in Chapter 11. The most important
ntion (vigorous stimulation, carnoninfectious causes of upper airway obstruction in
tation). Infantile apnea can
older children are tonsillar and adenoidal hypertroses (Table 20-3).
phy and severe obesity. Chronic causes of obstruces treating the underlying distion tend to manifest as obstructive sleep apnea in
able cause can be found, the
the older child because the relaxed pharyngeal tone
on a home monitor that senses
during sleep exacerbates the obstruction.
thing) and heart rate and

Clinical Manifestati
Apparent Life-Threat
In contrast to apnea
y occurs in ful
comes to medical att
threatening event (A
ing to the caretaker
or is found apneic a
tonic, difficult to
observer often belie
died without interve
diopulmonary resusci
result from many cau
Management involv
order. When no treat
infant may be placed
chest movement (brea
sounds an alarm when

the child becomes apneic or


Obstructive Sleep Apnea
bradycardic; however
, home monitors have never
Symptoms of obstructive sleep apnea include restase the likelihood of sudden
less sleep, snoring or gasping, altered personality,
e (SIDS). In about half the
morning headache, and excessive daytime sleepinesscase.
nfancy, no predisposing condition
In older children and adults, obstructive sleep apnea

been proven to decre


infant death syndrom
s of apnea of i
is ever found.

--------------------------------------- 186
266

Blueprints Pediatrics
TABLE 20-3

sease
Causes of Apnea of Infancy
typically leads to interstitial

Interstitial Lung Di
Recurrent aspiration

Sepsis
Metabolic disorders
also result in obstructive lung

lung disease but may


disease, and thus it

may have symptoms associated


Meningitis
Electrolyte disorders
Acute chest syndrome in sickle
Pneumonia
Arrhythmias
ussed in Chapter 10. A number of
Bronchiolitis (RSV)
Aspiration
Seizures
Gastroesophageal reflux
ad to interstitial changes, includAirway obstruction
Idiopathic
tial lung disease, lymphocytic

with both processes.


cell disease is disc
rare diseases can le
ing chronic intersti
interstitial pneumon

itis, and sarcoidosis. Pulmonary


hemosiderosis involv
es an abnormal accumulation of
hemosiderin in the l
ungs as a result of diffuse alveolar hemorrhage. It m
ay be associated with cow's
KEY POINTS
nts or Goodpasture's syndrome in

milk allergy in infa


older children. Diag

nosis is based on the presence of


1. Apnea is a symptom, not a diagnosis.
crophages (siderophages) in
2. Home apnea monitors do not decrease the risk of
r gastric aspirates.
SIDS.

hemosiderin-laden ma
bronchial washings o
Clinical Manifestati

ons
Symptoms of restrict
ive lung disease typically reflect
limited pulmonary re
serve. Exercise intolerance,
RESTRICTIVE LUNG DISEASE
of breath are hallmarks.

dyspnea, and shortness


Space-occupying lesi

ons can be detected by chest


Restrictive lung diseases cause a decrease in most
decreased breath sounds over the
measurements of lung volume, including functional
hronic nature of many restrictive
residual capacity, tidal volume, and vital capacity.
ents at risk for developing symp-

auscultation noting
affected area. The c
lesions can put pati
tomatology of prolon

ged respiratory insufficiency.


Chest Wall Abnormalities
on may develop and be

Pulmonary hypertensi
detected by an accen

tuated second heart sound on


Pectus excavatum refers to a depression in the
ngers and toes may be noted.
sternum, and pectus carinatum to an outward deforons of pulmonary hemosiderosis

exam. Clubbing of fi
Clinical manifestati
include hemoptysis/h

ematemesis and a microcytic


mity. Severe congenital forms of these malformations
may result in restrictive lung disease as a result of

hypochromic anemia.

mechanical interference with normal respiration.


Severe scoliosis may have the same effect. Severe
obesity, in addition to being a risk for upper airway
N
obstructive disease, may also be a cause of restrictive
lung disease. Neuromuscular disease may manifest
itself as restrictive lung disease as a consequence of
in many diseases affecting
insufficient respiratory muscle strength.
em is ventilation and perfusion

VENTILATION-PERFUSIO
ABNORMALITIES
An important concept
the respiratory syst
matching. Alveoli th

at are actively involved in respiSpace-Occupying Lesions


adequate perfusion by local

ration need to have


capillary blood flow

. This is closely regulated by a


Any lesion that occupies intrathoracic space will
ators. Most important, the arteinterfere with normal pulmonary expansion if large
he alveolar capillaries are exquisenough. Pleural effusion, pericardial effusion, chyxygen tension. As a result, when
lothorax, hemothorax, pneumothorax, chest wall
ea of lung is compromised, local
tumors, mediastinal masses, cystic adenomatous malduced. The arterioles constrict,
formations, diaphragmatic hernias, and pulmonary
od is diverted to areas of lung
sequestrations may all compete with normal lung for
ntilation. When this system is
thoracic space.
will result.

number of local medi


rioles that supply t
itely sensitive to o
ventilation to an ar
oxygen tension is re
and consequently blo
engaged in active ve
disrupted, hypoxemia

--------------------------------------- 187
Questions
1. A 12-year-old male adolescent presents with a 1-month
4.
ant presents at your office with comhistory of fever, weight loss, fatigue, and pain and localpoor feeding, and fussiness.The physical
ized swelling of the midproximal femur. Which of the
cept for moderate dehydration, poor
following is the most likely diagnosis?
ritability. The white blood count is elea. Ewing's sarcoma
shift. The cerebrospinal fluid is unreb. osteosarcoma
sis of a catheterized specimen reveals
c. chronic osteomyelitis
white blood cells, and scant bacteria. You
d. benign bone tumor
ract infection. Which of the following is
e. eosinophilic granuloma
ate course of treatment?

A 4-month-old inf
plaints of fever,
exam is normal ex
perfusion, and ir
vated with a left
markable. Urinaly
red blood cells,
suspect urinary t
the most appropri
a. empiric intrav

enous antibiotic therapy


2. An obese adolescent male presents to your urgent care
ntibiotic therapy

b. empiric oral a

facility with a chief complaint of intermittent knee pain


c. fluid restrict
ion
for 2 weeks. He has no known history of trauma but does
d. surgical inter
vention
play soccer twice a week. He has had no fever or upper
e. delayed antibi
otic therapy based on culture results
respiratory symptoms. The knee exam is normal;
however, the hip exam demonstrates limited internal
5. A 15-year-old pat
ient with asthma presents to the emerrotation and mild tenderness. Which of the following is
gency room with s
hortness of breath. He has used his
the most likely cause of this patient's limp based on the
inhaler three tim
es in the past hour. His respiratory rate
history and exam?
is 34 with a puls
e oxygenation measurement of 92%.
a. Legg-Calve-Petthes disease
However, no wheez
ing is heard on exam. Which of the folb. osteomyelitis
lowing is the mos
t appropriate initial pharmacologic
c. septic arthritis
intervention?
d. Osgood-Schlatter disease
a. oral bronchodi
lators
e. slipped capital femoral epiphysis
b. nebulized bron
chodilators
3. A 1-month-old female infant,born at full term,is noted to c. nebulized crom
olyn
have a harsh holosystolic 3/6 heart murmur heard best at
d. intravenous st
eroids
the left lower sternal border.The child is not cyanotic and e. intravenous th
eophylline
does not have hepatomegaly or tachypnea at rest. The
child feeds without tachypnea or diaphoresis, and weight 6. A 3-month-old fem
ale infant presents to your emergency
gain is appropriate. There is no cardiomegaly on chest
room unresponsive
and with fever, tachypnea, bradycarradiograph. Which of the following is the most likely
dia, and hypotens
ion. What order should you follow in
diagnosis?
your initial asse
ssment?
a. ventricular septal defect
a. airway, breath
ing, circulation, disability, exposure
b. atria I septal defect
b. breathing, air
way, circulation, disability, exposure
c. patent ductus arteriosus
c. circulation, a
irway, breathing, exposure, disability
d. pulmonary stenosis
d. exposure, brea
thing, airway, circulation, disability
e. aortic stenosis
e. exposure, airw
ay, breathing, circulation, disability

--------------------------------------- 188
268
Blueprints Pediatrics
7. Preventive counseling should be an important part of
maly
every well-child visit. Which of the following statements
us pulmonary venous return with

b. Ebstein's ano
c. total anomalo

is true?
obstruction
a. Infants who are 20 pounds or heavier may ride in
d. tricuspid atr
esia with normally related great arteries
forward-facing car seats regardless of age,
e. tetralogy of
Fallot
b. Infants should be placed in the supine position for
sleeping.
c. When poisoning is suspected, parents should always 11. Peripheral pulmo
nicstenosis,atrial septal defect, ventricular septal defe
ct, chorioretinitis, hepatosplenomegaly,
give syrup of ipecac, regardless of the ingested subjaundice, and "b
lueberry muffin spots" are the clinical
stance.
manifestations t
ypically associated with which congenid. The most effective method of removing lead poisontal infection?
ing risk is to paint over lead-containing paint with
a. toxoplasmosis
paint manufactured after 1977.
b. syphilis
e. Driver education programs substantially reduce the
c. rubella
risk of accidents involving adolescents.
d. cytomegalovir
us
e. herpes simple
x virus 2
8. A 2-year-old boy presents to your office with a fever of
f. HIV
103F (39.4C) that has lasted for the past 5 days. You also
note bilateral conjunctivitis, dry red fissured lips, a maculopapular rash over the extremities and trunk, and
fl2y You are called t
o evaluate a newborn in the nursery.The
swelling of the hands and feet. Based on these findings
parents are very
concerned because the child's right foot
you make the diagnosis of Kawasaki's disease.What is the
points inward.Yo
u note that the foot is easily molded into
most appropriate initial therapy?
the correct anat
omic position; moreover, range of motion
a. corticosteroids
at the ankle is
normal. What is the most likely deformity?
b. antibiotics
a. medial tibia
torsion
c. cautious electrolyte replacement
b. developmental
hip dysplasia
d. dialysis
c. metatarsus ad
ductus
e. aspirin and intravenous immunoglobulin (IVIG)
d. talipes equin
ovarus
e. genu varum
9. A child presents to your office with a complaint of frequent short staring spells.These spells have been noticed
by both1 the parents and the child's preschool teacher.The13. A newborn infa
nt with suspected congenital heart
spells last only a few seconds each; however, the child is disease is noted
to have no thymic shadow on chest radinot responsive during the spells, and they are increasing ograph. Which of
the following is the most likely elecin frequency.The parents are concerned. Which of the fol- trolyte abnormal
ity?
a. hypocalcemia
lowing diagnostic procedures is most likely to yield a
b. hypercalcemia
definitive diagnosis?
c. hypokalemia
a. cerebrospinal fluid analysis
d. hyperkalemia
b. electroencephalogram
e. hypophosphate

mia
c. head CAT scan
d. muscle biopsy
e. magnetic resonance imaging
l is diagnosed with new-onset insulin-

14. A 3-year-old gir


dependent diabet

es mellitus. Which of the following lab10.: A full-term 4000-g male infant is noted to be cyanotic 6
is consistent with diabetic ketoacidosis?
hours after birth. He has increased pulmonary vascular
markings on chest radiograph without cardiomegaly. He
is tachypneic with good pulses and perfusion.There is no
ine
heart murmur, but there is a loud single S2.The electroblood pH
cardiogram is normal for a newborn. The preductal and
urea nitrogen
postductal oxygen saturation levels are 65%. A hyperoxia
test reveals a preductal right radial arterial blood gas15.
presents to your office with a chief comwhile breathing 100% O2 of 7.33/35/35/21 /-1.5.Which of
e headache and photophobia for 1 week.
the following congenital heart defects is most likely?
on arrival is 102.5T (39.2C).You notice
a. D-transposition of the great arteries with intact vennnular erythematous lesions with central
tricular septum
trunk and legs, consistent with erythema

oratory findings
a. hypoglycemia
b. hypercarbia
c. ketones in ur
d. high venous
e. normal blood
A7-year-old boy
plaint of sever
His temperature
several large a
clearing on his

--------------------------------------- 189
Questions
269
migrans. There is no known history of a tick bite. Which
cause he snores so badly that he freof the following is the most likely diagnosis?
eathing in his sleep and begins to gasp.
a. Lyme disease
ar quite large but not erythematous on
b. Rocky Mountain spotted fever
ild does not complain of throat pain.The
c. ehrlichiosis
tructive sleep apnea is confirmed after a
d. leptospirosis
is performed. What treatment is most
e. bacterial meningitis
ective in this patient?

room with him be


quently stops br
The tonsils appe
exam, and the ch
diagnosis of Obs
polysomnography
likely to be eff
a, continuous po

sitive airway pressure


16. Which of the following electrolyte abnormalities is cony
sistent with pyloric stenosis?
ntibiotic therapy
a. Na 134, K 4.8, Cl 114, bicarb 9, glucose 101
e tonsils and adenoids
b. Na 135, K 3.5, Cl 86, bicarb 37, glucose 69
c. Na 130, K5.0,C1102, bicarb 14, glucose 400

b. oxygen therap
c prophylactic a
d. removal of th
e. stimulants

d Na 128, K 6.0, Cl 95, bicarb 21, glucose 59


e. Na 150, K 6.0, Cl 110, bicarb 25, glucose 75
received no prenatal care presents in

21. A woman who has


active labor and

shortly delivers a small for gestational


17. A 5-year-old boy presents to the pediatrician with fever
admits to frequent cocaine use and
and new 3/6 systolic ejection murmur heard best at the
al intercourse before and during her
right upper sternal border. On extremity examination,
ysical exam, the newborn is noted to
splinter hemorrhages and petechiae are noted. Which of
er and spleen, marked lymphadenopathy,

age infant. She


unprotected sexu
pregnancy. On ph
have a large liv
and nasa.l disch

arge that your attending physician labels


the following is the most likely diagnosis based on the
Which test on the infant is most likely to
clinical description?
osis?
a. endocarditis

"the snuffles."
reveal the diagn
a. blood culture

b. rheumatic heart disease


d count
c. Kawasaki's disease

b. complete bloo

d. pericardia) effusion
ntigen
e. dilated cardiomyopathy
omegalovirus

c. hepatitis B a
d. urine for cyt
e. FTA-ABS

18. An infant presents at the emergency room following a


seizure that resolved without intervention. She has a 22. A 6-month-old ma
le infant presents to the pediatrician
history of a fever and 2 days of diarrhea. Her white blood
count is elevated, and a stool sample is full of mucus and
eart rate of 50. Physical examination
streaked with blood, but she appears nontoxic and is well
and there is no history of rash. On chest
hydrated. Which of the following pathogens is most
e is no cardiomegaly. Electrocardiogram
likely?
d ventricles. The family history reveals

with a resting h
reveals no rash,
radiograph, ther
revealed D-ioope
maternal systemi

c lupus erythematosus. Which of the


a. Shigella dysenterlae
ses is the most likely cause for the
b. Vibrio choleras
c. Giardia lamblia
d. Yersinia enterocotitica
mplete heart block
e. Salmonella typhi
sfunction

following diagno
bradycardia?
a. Lyme disease
b. congenital co
c. sinus node dy
d. cardiomyopath

y
19. Escherichia coti gastroenteritis is associated with which ofe. sinus bradyca
rdia
the following complications?
a. pseudoappendicitis
b. erythema nodosum
, vesicular, pustular lesions starting on
c. failure to thrive
reading to the extremities is the classic
d. cholera
hich of the following infections?
e. hemolytic uremic syndrome

23. Crops of papular


the trunk and sp
description of w
a. measles

b. erythema inf
ectiosum (fifth disease)
20. A 7-year-old child is referred to your office because of
c. roseola infan
tum
declining school performance.There is no known change
d. zoster (shing
les)
in the child's life stressors. The teacher reports that the e. rubella
child has been falling asleep in his classes. The grandth disease
mother notes that she has begun sleeping in the same

f. hand-foot-mou
g. chickenpox

--------------------------------------- 190
Chapter 3 / Cardiology

27
Ostium secundum defe

ct, seen in the midportion


of the atrial sept
um
Ostium primum defect
, located in the low atrial
septum
Sinus venosus defect
, found at the junction of the
right atrium and t
he superior or inferior vena cava
The degree of atr
ial shunting is dependent on the
size of the ASD and
the relative compliance of the
ventricles in diasto
le. Since right ventricular diastolic
compliance is usuall
y greater than left ventricular
diastolic compliance
, left-to-right shunting occurs
at the atrial level,
thus increasing flow across the
tricuspid and pulmon
ary valves and increasing

pulmonary blood flow


.
Clinical Manifestati
ons
Atrial septal defect
s are usually not associated with
symptoms, although t
here may be a history of slow
weight gain and freq
uent lower respiratory infections. On physical e
xamination, the precordium is
Figure 3-10
Interrupted aortic arch with restrictive patenthyperdynamic, and a r
ight ventricular heave is often
ductus arteriosus.Typical anatomic and hemodynamic findings present. A systolic
ejection murmur in the pulmonic
include: (a) atresia of a segment of the aortic arch between the
left subclavian artery and the left common carotid (the mostarea and a mid-diast
olic rumble in the lower right
common type of interrupted aortic arch "type B"); (b) a pos-sternal border reflect
the increased flow across the
terior malalignment of the conal septum resulting in a largepulmonary and tricus
pid valves. S
2 is widely and conventricular septal defect and a narrow subaortic area; (c) astantly split. On ch
est radiograph, the heart and main
bicuspid aortic valve occurs in 60% of patients; (d) systemicpulmonary artery ar
e enlarged and pulmonary vaspressure in the right ventricle and pulmonary artery (due to thecularity is incr
eased. The ECG often shows right venlarge, nonrestrictive ventricular septal defect); (e) increasedtricular hypertro
phy or right ventricular conduction
oxygen saturation in the pulmonary artery due to left-to-right
shunting at the ventricular level; (f) "differential cyanosis" withdelay. Rightaxis deviation is often seen in secundum
a lower oxygen saturation in the descending aorta due to a defects, whereas pri
mum defects have characteristic
right-to-left shunt at the patent ductus. Note the lower bloodextreme left-axis
deviation. The amount of right
pressure in the descending aorta due to constriction of the ventricle and left a
trium enlargement is directly
ductus; opening the ductus with PGE, results in equal upper
poportional to the s
ize of the left-to-right shunt.
and lower extremity blood pressures, but continued "differen-On echocardiogram,
the defect can be visualized, and
tial cyanosis."
Doppler flow mapping
demonstrates the direction of
Cloherty JP, Stark AR. Manual of Neonatal Care, 4th ed. Philadelphia:
Lippincott-Raven, 1998:426.

flow.
Treatment
Spontaneous closure

of small secundum ASDs is


stenosis. The acyanotic structural anomaly that
he majority of cases in the first year

likely to occur in t

results in relatively normal pulmonary blood flow is


um and sinus venosus ASDs do
pulmonary valve stenosis.
sly and must be addressed sur-

of life. Ostium prim


not close spontaneou
gically. The symptom

atic child with an ASD should


have the defect clos
ed as soon as possible. The timing
Atrial Septal Defects
of ASD repair in th
e asymptomatic infant or child is
Atrial septal defects account for 8% of congenital
In general, the defect should be
heart disease. There are three types of atrial septal
latory arrest is not needed and
defects:
of needing a blood transfusion

more controversial.
repaired when circu
when the likelihood

--------------------------------------- 191
270
Blueprints Pediatrics
24. A 4-year-old who has recently been started on potassurgeon called t
o consult notes that the anus appears
sium-sparing diuretics develops muscle weakness and
patent. You susp
ect meconium ileus. What genetic disortetany. His STAT serum potassium level is 7.7, with no
der is most cons
istent with this child's presentation?
hemolysis noted. An electrocardiogram is performed, and
a. cystic fibros
is
peaked T waves are noted. What is the most appropriate
b. phenylketonur
ia
initial treatment?
c. Tay-Sachs dis
ease
a. intravenous glucose
d. galactosemia
b. intravenous calcium gluconate
e. Wilson's dise
ase
c. intravenous 3% NaCI solution
d. hemodialysis
29. A 12-month-old m
ale infant presents with a hemoglobin
e. intravenous normal saline bolus
of 7.5 and a hem
atocrit of 22%. The mean corpuscular
volume is 65 and
the adjusted reticulocyte count is 1.0%.
25. An afebrile 5-year-old girl presents with tachycardia at
What is the most
likely cause of anemia in the child?
220 beats per minute. On electrocardiogram, a regular
a. iron deficien
cy anemia
narrow-complex tachycardia is seen. The rhythm conb. anemia of chr
onic disease
verts with one dose of adenosine intravenously to
c. transient ery
throcytopenia of childhood
normal sinus rhythm with preexcitation (delta waves)
d. thalassemia s
yndrome
. noted throughout the precordial leads. There is no care. parvovirus B1
9 aplastic crisis
diomegaly on chest radiograph. The narrow-complex
tachycardia is most likely consistent with which of the

following?
female child presents with blooda. Wolff-Parkinson-White syndrome
he stool is grossly positive on Hemoccult
b. idiopathic concealed bypass tract
f the following diagnoses is most likely?

30. An 18-month-old

c. sinus tachycardia
disease
d. atrial flutter
tear
e. atrial fibrillation
bowel disease

streaked stool.T
testing. Which o
a. anal fissure
b. peptic ulcer
c. Mallory-Weiss
d. inflammatory
e. necrotizing e

nterocolitis
26. A15-year-old girl presents to your emergency room with
a history of recent acetaminophen ingestion. What is the
most common significant morbidity associated with this 31.
presents to your office with a chief comingestion?
n face. On exam, you notice that heart,
a. cardiac arrhythmias
nal findings are normal. However, his
b. malignant hypertension
re quite edematous. You check a urine
c. seizures
is markedly positive for protein but
d. hepatotoxicity
blood. What is the most likely etiology
e. ineffective hemostasis
edema?

A 5-year-old boy
plaint of swolle
lung, and abdomi
hands and feet a
dipstick, which
demonstrates no
of this child's
a. urinary tract

infection
27. Which of the following statements concerning neural

b. renal mass
c. undiagnosed h

eart disease
tube defects is true?
d. minimal chang
e disease
a. A low maternal serum alpha-fetoprotein level is assoe. focal segment
al glomerulosclerosis
ciated with an increased risk of a neural tube defect in
the fetus.
b. There is no increased risk of a neural tube defect in a32. A 5-year-old b
oy presents pulseless, with ventricular
second child when the first child is born with an
tachycardia at 2
80 beats per minute on electrocardioencephalocele.
gram. Immediatel
y the child is intubated, ventilated, and
c. Maternal folic acid supplementation decreases the
successfully def
ibriHated. After defibrillation, an electroIncidence of neural tube defects.
cardiogram revea
ls a corrected QT interval of 500msec.
d. Children with spina bifida are invariably paralyzed in Which of the fol
lowing therapies is the mosf appropriate
their lower extremities.
chronic therapy
for long QT syndrome?
a. nadolol
28. You are called to the neonatal intensive care unit to eval- b. digoxin
uate a small newborn who has not passed meconium in
c. verapamil
the first 72 hours of life.There is no evidence of heart or d. lidocaine

lung disease, and the infant is feeding appropriately.The


asix)

e. furosemide (L

--------------------------------------- 192
Questions
271
33. A 3-year-old boy presents with an elbow hemarthrosis
37. A1500g 29-week-o
ld Asian male neonate was born preafter falling on his elbow. There is no history of sponta- maturely to a 28
-year-old G2P1001, serology-negative
neous bleeding. There is no history of epistaxis, gingival female by normal
spontaneous vaginal delivery. Apgar
bleeding, or cutaneous bruising. The child's maternal
scores were 5 an
d 7 at 1 and 5 minutes, respectively.The
grandfather had frequent spontaneous bleeding and
neonate is in si
gnificant respiratory distress, with poor air
hemarthroses after trauma on multiple occasions. Labomovement. The ne
onate is intubated, given surfactant,
ratory results revealed a prolonged PTT, normal FT, and a and taken to the
newborn intensive care unit (NICU) for
platelet count of 150,000.The factor VIII coagulant activ- further manageme
nt. A blood culture is sent soon after
ity (Vlllx) is low and the factor IX level is normal. What isarrival in the
NICU. Ampicillin and gentamicin are started
the mosf likely diagnosis?
empirically unti
l the blood culture result is known. Over
a. idiopathic thrombocytopenic purpura
the next 12 hour
s, the child is noted to have poor perfub. von Willebrand's disease
sion, hypotensio
n, decreased urine output, coagulation
c. vitamin K deficiency
tests consistent
with disseminated intravascular coagud. hemophilia A
lation, and bila
teral pulmonary infiltrates. Results of
e. liver disease
maternal vaginal
and rectal cultures for group B streptococci are unknow
n. Which of the following bacteria is
34. A 3-year-old boy presents to the pediatrician with
most likely to b
e responsible for the child's sepsis?
fever, pallor, anorexia, joint pain, petechiae, and
a. group B strep
tococci
hepatosplenomegaly. Which of the following is the mosf
b. Streptococcus
pneumonias
likely diagnosis?
c. Chlamydia tra
chomatis K
a. acute lymphoblastic leukemia
d. Staphylococcu
s epidermis
b. acute myelogenous leukemia
e. Staphylococcu
s aureus
c. juvenile chronic myelogenous leukemia
d. aplastic anemia
t 28 weeks' gestation is now 2 weeks
e. osteosarcoma
ric feeds are started. Forty-eight hours

38. A neonate born a


of age. Nasogast
after starting f

eeds, the neonate develops a distended


35. Which of the following statements about neuroblastoma
stool, pneumatosis intestlnalis, and
is true?
minal radiograph. Laboratory studies
a. Neuroblastoma is a benign tumor of the neural crest
topenia.The child becomes persistently
cells that form the adrenal cortex and the paraspinal
ite maximal medical therapy.The most
parasympathetic ganglion.
is:
b. The majority of neuroblastoma tumors occur in the

abdomen, bloody
free air on abdo
reveal thrombocy
hypotensive desp
likely diagnosis
a. sepsis

thoracic cavity.
eumonia
c. Neuroblastoma is the most common malignant tumor

b. aspiration pn

in infancy.
nterocolitis
d. In neuroblastoma of the abdomen, displacement of
ia
the kidney and distortion of the calyceal system often

d. necrotizing e

c. malrotation

e. jejuna! atres

occur.
39. Which of the fol
lowing is the proper initiation sequence
e. Most patients are treated with surgery alone, since
of sexual develo
pment in the male?
distant metastases are rare.
a. testicular en
largement, penile enlargement, height
growth spurt,
and pubic hair
36. A 6-week-old breast-fed infant presents to your office
b. pubic hair, t
esticular enlargement, penile enlargeone morning appearing quite well. The mother states
ment, height
growth spurt
that for the last week, the infant has had numerous
c. testicular en
largement, penile enlargement, pubic
periods of inconsolable crying lasting several hours each.
hair, height
growth spurt
Nothing seems to help. You find that most of the spells
d. penile enlarg
ement, height growth spurt, testicular
occur in the late afternoon and evening; between the
enlargement,
pubic hair
episodes, the baby looks and feeds quite well. What is the e. height growth
spurt, pubic hair, penile enlargement,
most likely diagnosis?
testicu lar e
n larg eme nt
a. otitis media
b. intussusception
who received Bactrim for otitis media
c. milk protein intolerance
emergency department with high fever;
d. colic
n the palms and soles, trunk, and the
e. malabsorption
s of the extremities; and inflammatory

40. A 6-year-old boy


presents to the
target lesions o
extensor surface

--------------------------------------- 193
272

rints Pediatrics
bullae on his mucous membranes. What type of hypersensitivity rash does this child have?
a. eczema
b. urticaria
lowing statements about polyhydramc. erythema multiforme
rome is associated with polyhydramnios.
d. Stevens-Johnson syndrome
e. toxic epidermal necrolysis
ramnios is more common than chronic

d. hyperkalemia
e. hyperglycemia
45. Which of the fol
nios is true?
a. Potter's synd
b. Acute polyhyd
polyhydramnio

s.
c. Lesions that
impair fetal swallowing are associated
41. A newborn male child has a flat facial profile, upslanted
with polyhydr
amnios.
palpebral fissures, epicanthal folds, a small mouth with a
protruding tongue, small genitalia, and simian creases on d. Polyhydramnio
s may result in postmaturity.
his hands. What chromosomal disorder does this child
e. Polyhydramnio
s is associated with fetal lung hypoplasia.
have?
a. trisomy 21
46. An 8-year-old bo
y presents with a 1-day history of emesis
b. trisomy 18
and periumbilica
l pain that has moved to the right lower
c. trisomy 13
d. Klinefelter's syndrome
quadrant. There
is no history of diarrhea. Abdominal
examination reve
als guarding and rebound tenderness.
e. Turner's syndrome
The white blood
cell count is elevated, at 20,000, with a
left shift. Whi
ch of the following is the most likely diag42. Trisomy 21 is associated with:
nosis?
a. malrotation
a. appendicitis
b. endocardial cushion defect
b. pancreatitis
c. cleft palate
c. viral gastroe
nteritis
d. renal disease
d. urinary tract
infection
e. sensorineural hearing loss
e. diabetes mell
itus
43. A 4-year-old male child presents with abrupt-onset
47. A 3-year-old boy
presents with violent episodes of interpetechiae and ecchymoses. Other than the skin findings,
mittent colicky
pain, emesis, and blood per rectum. A
the child appears well and is hemodynamically stable. No
tubular mass is
palpated in the right lower quadrant.The
splenomegaly is noted. A complete blood count reveals
abdominal radiog
raph reveals a dearth of air in the right
a normal white blood cell count, a normal hematocrit,
lower quadrant

and air-fluid levels consistent with ileus.


and a platelet count of 30,000. Large platelets are seen on Which of the fol
lowing procedures will best assist in diagthe peripheral smear. No premature white cell forms are
nosis and treat
ment?
seen on peripheral smear. The parent reports that the
a. esophagogastr
oduodenoscopy
b. rectal biops
y
child had a viral illness 2 weeks before presentation.
Which of the following, is the most likely diagnosis?
c. air contrast
or barium enema
a. isoimmune thrombocytopenia
d. stool culture
b. leukemia
e. colonoscopy
c. sepsis
48. A 4-week-old mal
e infant born at full term presents with
d. immune thrombocytopenic purpura
emesis, dehydra
tion, and poor weight gain.The pediatrie. hypersplenism
cian evaluating
the child palpates an olive-sized mass in
(44.J A 4-year-old child presents to the emergency room with
the child's epi
gastrium. She believes the neonate may
have pyloric st
enosis. Which of the following clinical prestupor and posturing. His mother reports that he has
sentations is m
osf consistent with pyloric stenosis?
been acting disoriented for the past 24 hours. She was
a. projectile n
onbilious emesis
not initially concerned because the child had similar
episodes of confusion with the high fevers he had a week
b. bilious emes
is
c. bloody diarr
hea
ago with his chickenpox, which she treated successfully
d. violent epis
odes of intermittent colicky pain and
with aspirin. You suspect possible Reye's syndrome.
emesis
Which of the following lab results is mosf supportive of
your diagnosis?
e. right lower
quadrant abdominal pain
a. hyperammonemia
49. Which of the fo
llowing statements is true!
b. hypernatremia
a. Ulcerative c
olitis typically is characterized by rectal
c. hypercalcemia
sparing.

--------------------------------------- 194
Questions
273
b. Ulcerative colitis typically is characterized by skip
shows gram-negative diplococci. You
3*= lesions.
e infant has ophthalmia neonatorum
c. Crohn's disease typically is characterized by
eria gonorrhoeas. What is the most approtransmural disease.
t?

junctival swab
suspect that th
caused by Neiss
priate treatmen

d. Crohn's disease typically is characterized by crypt


hromycin only
abscesses.
ical erythromycin
e. Having Crohn's disease dramatically increases the risk
ceftriaxone
of carcinoma of the colon.

a. topical eryt
b. oral and top
c. intravenous
d. no treatment
e. intravenous

acyclovir
50. Which imaging study is most likely to demonstrate vesicoureteral reflux?
visit,you are examining a child who is able

56. At a well-child
to understand t

wo-step commands, remove her own


a. renal ultrasound
b. voiding cystourethrogram
up and down stairs well without assisc. nuclear medicine scan
ld has reached the developmental miled. intravenous pyelography
ate for a child at which age?

shoes, and walk


tance. This chi
stones appropri
a. A 6-month-ol

d
e. abdominal CAT scan
b. A 12-month-o
ld
51. Given what you know about the pathophysiotogy of
ld
asthma, what medicine is most likely to address the
ld
underlying inflammation and prevent the "late-phase"
ld
response?
y comes in to see you. As you examine his
a. methylprednisolone
, you discover that he has had a history of
b. theophylline
c. albuterol
ses and an episode of Aspergillus pneud. cromolyn
nodeficiency should you consider?

c. A 24-month-o
d. A 36-month-o
e. A 48-month-o
57. A 5-year-old bo
medical history

two skin absces


monia.What immu

a. complement d
eficiency
e. terbutaline
b. DiGeorge's s
yndrome
52. What is the most significant serious complication arising
c. selective Ig
A deficiency
from Kawasaki's disease?
d. chronic gran
ulomatous disease
a. coronary aneurysms
e. HIV infectio
n
b. kidney failure
58. What does the p
resence of a positive antinuclear antic. arthritis
body liter in a
patient with juvenile rheumatoid arthritis

d. gastrointestinal bleeding

indicate?

e. hypertension
v a. an increased ri
sk of chronic disease
b. an increased
risk for the development of chronic
53. Which of the following findings is diagnostic for infantile
spasms?
ity of renal involvement
a. increased levels of protein in the cerebrospinal fluid

uveitis
c. the possibil

b. an asymmetric mass lesion


d. the extent o
f joint involvement
c. hypsarrhythmia
e. the presence
of skin findings
d. a generalized, symmetric three-per-second spike and 59. A 16-year-old f
emale patient presents with short stature
wave pattern
and no secondar
y sexual characteristics. What diagnosis
e. generalized brain edema
must be conside
red?
a. Turner's syn
drome
54. What is the most appropriate indication for using epib. isolated gro
wth hormone deficiency
nephrine (1:10,000)?
sease
a. ventricular ectopy
rt stature
b. asystole
sease
c. severe refractory metabolic acidosis and/or hyperkalemia
llowing vitamins or minerals needs to be
d. bradycardia caused by atrioventricular block
infants that are exclusively breast-fed?
e. supraventricular tachycardia

c. Cushing's di
d. familial sho
e. Addison's di

60. Which of the fo


supplemented in
a. iron
b. vitamin C

55. A 5-day-old infant develops bilateral conjunctiva! injection with purulent discharge. Gram's stain of the con-

c. vitamin D
d. calcium
e. folic acid

--------------------------------------- 195
Blueprints Pediatrics
274
61. A 7-year-old boy with a history of asthma is admitted to
rosis

d. tuberous scle

your general pediatric service with status asthmaticus.

e. neurofibromat

osis
After 2 days of oral prednisone and inhaled bronchodila-66.
l with vomiting and diarrhea presents to
tor therapy, he is ready for discharge. He has been
brought to the emergency department three times in the
partment with hypernatremic dehylast year with respiratory distress, and he reports needing
um sodium level measures 160mg/dL,
to use his p-agonist metered-dose inhaler at least twice
ficantly tachycardic with dry mucous
a week. Initiation of maintenance therapy with which of
or skin turgor.She appears listless,but
the following medications is appropriate?
re is normal. Which of the following
a. cromolyn sodium
opriate approach to this patient's
b. theophyline
c. inhaled corticosteroids
oluses of 10cc/kg of D5 water until the
d. leukotriene inhibitors
rt rate normalizes and her serum sodium
e. a long-acting p-agonist
patient's parents to give TOccoforal rehy62. A 10-month-old girl weighs 8kg. She needs to be NPO
every 5 to 10 minutes while you observe
overnight in preparation for sedation for a magnetic resn the emergency department. If the
onance imaging study. Which of the following would be
strate an understanding of this techappropriate maintenance fluids?
rge the patient home.
a. normal saline at 30 cc/hr
of 20cc/kg of normal saline until her vital
b. D10 water at 35 cc/hr
proved. Then calculate fluids to give
c. D5 normal saline with lOmEq KCI/L at 35 cc/hr
nd replacement fluids to correct the
d. D5 one-half normal saline with 20mEq KCI/L at
level to normal over 48 hours intra100 cc/hr
e. D5 one-fourth normal saline with 20mEq KCI/L at
half normal saline at one and a half times
35 cc/hr
nd admit for monitoring.

A 3-year-old gir
the emergency de
dration. Her ser
and she is signi
membranes and po
her blood pressu
is the most appr
rehydration?
a. Give serial b
patient's hea
is below 150.
b. Instruct the
dration fluid
the patient i
parents demon
nique, discha
c. Give boluses
signs have im
maintenance a
serum sodium
venously.
d. Start D5 onemaintenance a

e. Give boluses
of 20cc/kg of normal saline until her vital
63. You are called to evaluate a full-term newborn at 30
signs are sta
ble, then instruct the parents on oral rehyhours of age because she is jaundiced. Her unconjugated
dration thera
py and discharge the patient home.
bilirubin level is 15mg/dL, and her hematocrit is 48.
Which of the following is the most likely cause?
67. A 5-year-old boy
is brought to the emergency room after
a. echovirus hepatitis
having a 2-minut
e seizure at home.The parents describe
b. physiologic jaundice
initial twitchin
g of the right arm and then generalized
c. polycythemia
tonic-clonic act

ivity. On exam, the patient is now listless


d. ABO incompatibility
d has stable vital signs and no fever. You
e. biliary atresia
- to 10-mm hyperpigmented macules

but arousable an
notice several 5

scattered on the
patient's trunkand legs.Which of thefol64. A 12-year-old boy with Crohn's disease is admitted with
lowing is the mo
st likely diagnosis?
an exacerbation. He is complaining of abdominal pain
a. tuberous scle
rosis
and diarrhea. The most effective management in this
b. meningitis
acute setting is which of the following?
c. idiopathic se
izure
a. TNF alpha inhibitor
d. neurofibromat
osis
b. corticosteroids
e. Sturge-Weber
syndrome
c. metronidazole :
68. You are called t
o evaluate a newborn girl for intrauterine
d. sulfasalazine
growth retardati
on. You notice on exam that she is below
e. azathioprine
the fifth percen
tile for weight, length, and head circum65. A 1 -year-old is a new patient in your practice and you are ference. She als
o has hepatosplenomegaly.You obtain a
seeing him for well-child care. You note that he has an
head ultrasound
, which demonstrates periventricular
abnormally shaped skull. His head appears shortened
calcifications.
Which of the following is the most likely
from front to back and wide. He is developmentally
cause of these f
indings?
normal and has no significant past medical history. Of
a. herpes simple
x virus
the following diagnoses, which is most likely?
b. placental in
sufficiency
a. craniosynostosis
c. chorioamnion
itis
b. von Hippel-Lindau disease
d. trisomy 13
c. macrocephaly
e. cytomegalovi
rus

--------------------------------------- 196
Questions
275
69. An 8-year-old boy presents with ataxia, nystagmus, and
60 per minute. She has mild subcostal
head tilt What malignancy is most likely to cause this
diffuse wheezing. What is the most likely
child's symptoms?

piratory rate is
retractions and
diagnosis?

a. cerebella astrocytoma

a. croup

b. craniopharyngioma

b. epiglottitis

c. optic glioma
chomatis pneumonia */

c. Chlamydia tra

d. metastatic neuroblastoma
n body
e. acute lymphocytic leukemia

d. airway foreig
e. bronchiolitis

70. Which of the following statements is true regarding chil-73. A 6-year-old gi


rl is brought to you for evaluation of resdren with sickle cell disease?
piratory distres
s and right upper quadrant abdominal
a. Vaccinations are not required, because they receive
pain. You note t
he presence of fever, cough, and tachyppenicillin prophylaxis.
nea.On lung exam
, she has right lower lobe crackles and
b. Gallstones typically develop before the age of 3 years. egophony.Chest r
adiograph reveals right lower lobe conc. Episodes of dactylitis should be treated with antibisolidation with
a small pleural effusion. You make a
otics.
diagnosis of bac
terial pneumonia and attribute the
d. Hydroxyurea maintenance therapy decreases the
abdominal pain t
o pleuritic pain secondary to the pneunumber and severity of vasoocclusive crises.
monia with effus
ion. What is the most likely bacterial
e. Acute chest syndrome requires only supportive care.
cause of this ch
ild's'pneumonia?
a. Haemophilus i
nfluenzae type b
71. A 15-year-old boy presents with testicular tenderness
b. nontypeable H
aemophilus influenzae
that started approximately 2 hours ago. Physical exam
c. Staphylococcu
s aureus
reveals a swollen, diffusely tender testis with ipsilateral d. group B strep
tococci
scrotal edema and absent cremasteric reflex. You diage. Streptococcus
pneumoniae
nose testicular torsion. What is the most appropriate next
step?
74. You are examinin
g a 2-week-old infant whose mother is
a. scrotal MRI to confirm testicular torsion
HIV positive. Wh
at is the most appropriate test to deterb. surgical detorsion and fixation of the testis to the pos-mine the infant'
s HIV status?
terior scrotal envelope
a. HIV DMA PCR
c. oral erythromycin and intramuscular ceftriaxone for

b. HIV RNA PCR

presumptive treatment of infection by Chlamydia tra-

c. HIV IgG antib

ody
chomatis and Nelsseria gonorrhoeae

d. CD4 count

d. admission for observation and analgesia until the pain


hocyte count
resolves
e. orchiopexy to prevent infertility
disorder of carbohydrate metabolism, is

e. absolute lymp

75. Galactosemia, a

inherited in an
autosomal recessive fashion. What is the
72. A 6-month-old girl is referred for evaluation of her first risk of galactos
emia in a child whose parents are both

episode of wheezing. She was seen by her pediatrician


disorder?
earlier in the day and received treatment with nebulized

carriers for the


a. 100%

albuterol without effect. Her mother reports that the

b. 75%

infant has had rhinorrhea and cough for 3 days and now

c. 50%

seems to have difficulty breathing. Her temperature was

d. 25%

101F (38.2C) at home. On physical examination her res-

e. 0%

--------------------------------------- 197
nswers
1. a (Chapter 17)
at the left upper st
ernal border radiating to the left axilla is
The clinical description is most consistent with Ewing's
consistent with a le
ft patent ductus arteriosus.
sarcoma. Unlike osteosarcoma, Ewing's sarcoma tends to
involve systemic symptoms, such as fever, weight loss, and 4. a (Chapter 14)
fatigue. Ewing's sarcoma usually involves the diaphyseal
The patient discusse
d in this question has signs and sympportion of the long bones.The most common sites for Ewing's toms of significant
illness and probably will require parenteral
sarcoma are the midproximal femur and the bones of the
antibiotics, at leas
t initially, ideally in a hospital setting,
pelvis.The most common sites of osteosarcoma are the distal without delay. Oral
antibiotic therapy would be ineffective
femur, proximal tibia, and proximal humerus. Benign bone
and inadvisable. Sin
ce the patient is not taking fluids well,
tumors and eosinophilic granuloma are generally not painful.aggressive intraveno
us fluid therapy (rather than fluid restricChronic osteomyelitis may present with fever, pain, and local-tion) may be neces
sary. A surgical lesion is very unlikely,given
ized swelling, but weight loss is unlikely.
the presentation, al
though if the patient has a urinary tract
infection secondary
to an anatomic lesion, delayed surgery
2. e (Chapter 19)
may ultimately be re
quired.
Slipped capital femoral epiphysis (SCFE) is the gradual or
acute separation of the proximal femoral growth plate. The 5. b (Chapter 20)
cause is unknown, but trauma is not thought to be a factor inNebulized bronchodi
lator therapy, most appropriately
development of the condition. It typically occurs in obese albuterol, is the in
tervention of choice in this situation. pY
adolescent males during the growth spurt. Legg-CalveAgonists, such as al
buterol, reduce smooth muscle constricPerthes disease also presents with a limp, but these patientstion and can be inv
aluable for asthmatics in acute distress.
are typically younger (age 4-8 years). Osteomyelitis and
This patient is in s
evere distress; he is moving so little air that
septic arthritis are unlikely in the nonfebrile patient with thisno breath sound
s can be appreciated. Oral bronchodilators
duration of symptoms. Osgood-Schlatter disease presents
would take too long
to take effect in this situation. Cromolyn

with pain and swelling over the tibial tuberosity and does notis a form of preve
ntion but is not helpful during an acute
involve the hip.
attack. Intravenous
steroids may be appropriate in this case,
but would not be the
initial therapy because they take 4 to 6
3. a (Chapter 3)
hours to be effectiv
e. Theophylline has fallen out of favor for
A harsh holosystolic murmur heard best at the left lower
use in emergent situ
ations but may be employed later if the
disease does not res
pond to first-line therapies.
sternal border is most consistent with a ventricular septal
defect.The child does not have symptoms of congestive heart
failure (no cardiomegaly on chest radiograph, tachypnea or 6. a (Chapter 1)
diaphoresis with feeds, or hepatomegaly); therefore, the
The primary survey i
s the initial evaluation of the critically ill
defect is likely restrictive. A systolic ejection murmur at theor injured child
when life-threatening problems are identified
left upper sternal border is consistent with either an atrialand prioritized. Th
e proper order of the primary survey or
septal defect or pulmonic stenosis. A systolic ejection murmurinitial assessment
is airway, breathing, circulation, disability,
at the right upper sternal border is consistent with aortic and exposure. After
the primary survey is complete, resuscistenosis. A continuous "machinery-type" murmur heard best tation should occur
if the condition is life-threatening. Once

--------------------------------------- 198
Answers
277
the life-threatening issues are addressed, the secondary
hyperoxia test,the clinician should first
survey should be performed
diograph. If massive cardiomegaly is

than 50 mm Hg on the
examine the chest ra

noted, Ebstein's ano


maly is the most likely diagnosis. If
7. b (Chapter 2)
massive cardiomegaly
is ruled out, the pulmonary vascularity
Barring unusual medical conditions, infants should be placedshould be evaluated.
Increased pulmonary blood flow suggests the presence o
f D-transposition of the great arteries
on their backs to sleep rather than on their stomachs; this sig-with intact vent
ricular septum, whereas pulmonary edema
nificantly decreases the risk of sudden infant death syndromemay indicate the pr
esence of total anomalous pulmonary
(SIDS). In most states, infants must be both 20 pounds and 1venous return with o
bstruction.The remaining possible diagyear of age to ride in forward-facing car seats. For certain
ingestions, particularly those involving strong bases or hydro-noses (tetralogy
of Fallot, tetralogy of Fallot with pulmonary
carbons, syrup of ipecac is contraindicated. Lead-containingatresia, pulmonary a
tresia with intact ventricular septum, critpaint should be removed, rather than painted over, because ical pulmonary steno
sis, tricuspid atresia with normally

related great arteri


es) all have decreased pulmonary vascuof the continued risk of ingestion of chipped paint. Althoughlarity and normal o
r slightly enlarged cardiac silhouette on
entertaining and instructive, driver education courses do notchest radiograph. T
hese defects are differentiated by their
appear to decrease the risk of vehicle accidents involving axis of ventricular
depolarization and the presence or
adolescents.
absence of a heart m
urmur. Tricuspid atresia with normally
related great arteri
es has a superior axis, lying in the 270- to
8. e (Chapter 11)
0-degree quad rant.
Critical pulmonic stenosis and pulmonary
There is no definitive laboratory test for Kawasaki's disease, soatresia with in
tact ventricular septum both have axes in the
it remains a clinical diagnosis. The patient must have at least0- to 90-degree q
uadra nt.They are differentiated by the presfive of the following six findings on physical exam:
ence of the loud pul
monary ejection murmur heard in critical pulrnonic stenos
is. Similarly, tetralogy of Fallot and
1. Fever lasting 5 days or more
2. Bilateral conjunctivitis
with pulmonic atresia both have axes in the

tetralogy of Fallot

90- to 180-degree qu
adrant, and they are distinguished from
3. Specific changes of the lips or oral cavity or both
each other by the pu
lmonic stenosis murmur noted in tetral4. Changes of the peripheral extremities (including possibleogy Of FallOt. ,,..;
; ,..,', ,,., . : ;,:.;,..... ; ' .,'
-.,..;,:.;. ; ;.; .;
indurative edema of the hands and feet)
5. Acute cervical lymph node swelling

11. c (Chapter 13)

6. Polymorphous rash, primarily seen on the trunk


Congenital rubella i
s caused by rubella virus. Clinical maniKawasaki's disease is one of the few diseases of childhood infestations of conge
nital rubella include peripheral pulmonic
which aspirin therapy is appropriate. IVIG helps decrease thestenosis, atrial se
ptal defect, ventricular septal defect, ophincidence of coronary artery aneurysms. Electrolyte abnor- thalmologic defects
(cataracts, microphthalmia, glaucoma,
malities are not typical of Kawasaki's disease, so dialysis is notchorioretiniti
s), hepatosplenomegaly, jaundice, "blueberry
necessary. Although the etiology of Kawasaki's disease is notmuffin spots,"and f
ailure to thrive.Toxoplasmosis is caused by
known,antibiotics have not been found to alter the course orToxoplasma gondii, a
n intracellular protozoan parasite found
outcome of the illness.
in mammals and birds
, in particular cats. Feline stool and
undercooked meat are
the means by which transmission
9. b (Chapter 15)
occurs. Infected inf
ants suffer from intrauterine meningoen-

cephalitis and prese


nt with microcephaly, hydrocephalus,
The spells described are most consistent with petit mal
ioretinitis, intracerebral calcifications,
(absence) seizures. An electroencephalogram would demon-

microphthalmia, chor

strate the characteristic generalized, symmetric three-per- and seizures. Congen


ital syphilis results from Treponema palsecond spike and wave pattern. Cerebrospinal fluid analysis,lidum. Syphilis in t
he untreated pregnant woman may be
CAT scan, muscle biopsy, and magnetic resonance imaging
transmitted to the f
etus at any time, but fetal transfer is most
common during the fi
rst year of maternal infection. Neonates
would all show a normal result in a patient with petit mal symptomatic at birth
may exhibit nonimmune hydrops,
seizures as his or her only diagnosis.
thrombocytopenia, le
ukopenia, pneumonitis, hepatitis, rash,
and osteochondritis.
In the first year of life, affected infants
10. a (Chapter 3)
have intermittent fe
ver, osteochondritis, persistent rhinitis
The most likely congenital heart defect is D-transposition of(snuffles), hepatos
plenomegaly, lymphadenopathy, jaundice,
the great arteries with intact ventricular septum. Typically,and failure to thri
ve. Congenital cytomegalovirus (CMV) infecthere is increased pulmonary vascularity on chest radiograph,tion is the most co
mmon congenital infection in the newborn
a single S2, and no heart murmur. To differentiate among
in developed countr
ies. Most cases are clinically inapparent.
cyanotic congenital heart defects that present with a Pao2 lessLate sequelae suc
h as nerve deafness and learning disabilities

--------------------------------------- 199
Blueprints Pediatrics
278
may develop in 10% of clinically inapparent infections. The 15. a (Chapter 12)
syndrome of congenital CMV (cytomegalic inclusion disease) Many patients with L
yme disease do not give a history of a
is uncommon, occurring in 5% of infants with CMV infection. tick bite, presumabl
y because they are unaware of it. Cases of
Clinical manifestations include intrauterine growth retarda-Lyme disease are clu
stered around the Northeast, Midwest,
tion, intracerebral calcifications (usually periventricular),
and West Coast and p
eak during the summer and early fall.
chorioretinitis, microcephaly, jaundice, hepatosplenomegaly,The patient describe
d has meningeal symptoms; however,the
and purpura. Neonatal herpes simplex virus (HSV) infection characteristic rash
is the giveaway. Erythema migrans consists
generally occurs during the infant's transit through the
of erythematous macu
les progressing to annular lesions with
vaginal canal. Asymptomatic infection is rare. HSV infection
central clearing tha
t develop both at the inoculation site and
manifests itself in three distinct constellations of symptoms:secondary areas. T

he rash may be fleeting or last for several


Infants may have localized infection of skin, eye, mouth (SEMweeks. All of the o
ther pathogens listed can cause severe
disease); disseminated infection; or localized central nervousillness and mening
itis. Rocky Mountain spotted fever prosystem infection. Infants infected with HIV are, in the vast
majority of cases, asymptomatic at birth. During the first fewduces a maculopapu
lar rash that begins at the wrists and
ankles and spreads p
roximally; the lesions progress to a
months, infants develop thrush, lymphadenopathy, and
petechial stage. Ehr
lichiosis does not typically cause a rash.
hepatosplenomegaly. During the first year of life, common This particular rash
is not typical for leptospirosis. Meningosymptoms include recurrent refractory infection, severe
coccemia may cause a
petechial rash that is quite dissimilar
intractable diarrhea, and failure to thrive.
from erythema migran
s.
12. c (Chapter 19)
16. b (Chapter 7)
Metatarsus adductus (in-toeing of the forefoot without hind-Patients with pylori
c stenosis vomit gastric contents with high
foot abnormalities) is a common, relatively benign conditionconcentrations of hy
drochloric acid, the primary fluid in the
caused by intrauterine positioning. As opposed to talipes stomach. There are n
o small intestinal losses because the
equinovarus, range of motion at the ankle is unrestricted.
pylorus is too small
to allow retrograde propulsion. Thus,
Developmental hip dysplasia is most common in firstborn
the bicarbonate leve
l tends to be high, with a decrease in the
girls and may not be evident to the casual observer; Ortolanichloride concentrat
ion. Sodium and potassium measureand Barlow maneuvers demonstrate this lesion. Genu varum
ments are usually no
t affected until late in the presentation.
is a knee deformity and does not involve the ankle or foot.
17. a (Chapter 3)
13. a (Chapter 11)
Fever and new murmur
may be consistent with rheumatic
A newborn infant with suspected congenital heart disease
heart disease or end
ocarditis.The splinter hemorrhages and
and no thymic shadow on chest radiograph should be suspetechiae make endoc
arditis highly likely and rheumatic
pected of having DiGeorge's syndrome. DiGeorge's syndrome heart disease unlike
ly. Dilated cardiomyopathy may present
is a congenital T-cell deficiency resulting in increased sus-with new murmur, bu
t the murmur is generally due to atriceptibility to opportunistic infections from organisms such asoventricular valve
regurgitation, which has a blowing quality
fungi and Pneumocystis carinii. It typically presents early inand is heard best
at the left lower sternal border or apex. If
infancy with congenital heart disease, hypocalcemic tetany, ventricular thrombus
is associated with the dilated carand the absence of a thymic shadow on chest radiograph.
diomyopathy, splinte
r hemorrhages and petechiae may be
None of the other electrolyte abnormalities that are listed isnoted. Kawasaki's
disease patients present with high fever,
associated with DiGeorge's syndrome.
but murmur and splin
ter hemorrhages are not commonly
noted.

14. c (Chapter 6)
18. a (Chapter 12)
The child with diabetic ketoacidosis (DKA) usually reports
polyuria, polydipsia, fatigue, headache, nausea, emesis, andOf the options liste
d, Shigella is the most likely, given the
abdominal pain. When DKA occurs, ketones are formed in the history of a seizure
. Children with shigellosis can present with
blood and cleared in the urine. Hyperglycemia,and not hypo- neurologic manifesta
tions, including lethargy, seizures, and
glycemia, is typical. Primary metabolic acidosis with sec- mental status change
s, possibly as a result of a neurotoxin
ondary respiratory alkalosis is noted (decreased venous
elaborated by the or
ganism. Cholera causes "rice-water"
blood pH and hypocarbia). Dehydration results in an elevatedstools and leads qui
ckly to hypovolemic shock but does not
blood urea nitrogen level. When DKA is present,the patient iscause neurologic co
mplications.Giardiasis,the most common
total body potassium depleted from significant potassium
parasitic disease in
the United States, typically causes only
loss in the osmotic diuresis. Patients with DKA may be hyper-diarrhea without fe
ver. Yersinia can cause a pseudoappenkalemic, normokalemic, or hypokalemic at presentation.
dicitis. Salmonella
can invade the bloodstream and cause

--------------------------------------- 200
Answers
279
extraintestinal disease, including meningitis, arthritis, andunlikely. Cardiomyo
pathy is an unlikely cause of the complete
osteomyelitis; it is no more likely to cause seizures than anyheart block, given
the lack of cardiomegaly on chest radiother bacteria.
ograph. Sinus node d
ysfunction occurs usually secondary to
atrial suture lines
or atrial dilation.This child has no history of
19. e (Chapter 12)
surgery, and there i
s no evidence of atrial dilation on chest
radiograph or electr
ocardiogram. Sinus bradycardia is a
Both Shigella dysenteriae and Escherichia coli 0157:H7
normal variant commo
n among athletes.
produce an enterotoxin (Shiga or Shiga-like toxin) associated
with hemolytic uremic syndrome, a serious complication that
includes microangiopathic hemolytic anemia, nephropathy,

23. g (Chapter 5)

and thrombocytopenia. Pseudoappendicitis and erythema


nodosum are associated with Yersinia infections. Failure to Measles is caused by
a paramyxovirus and is characterized by
thrive can occur in small children with chronic giardiasis. malaise, high fever,
cough, coryza, conjunctivitis, Koplik's
Cholera is another cause of infectious diarrhea.
spots, and an erythe
matous maculopapular rash. Koplik's
spots are smaH,irreg
u!ar red spots with central gray or bluish-

white specks that ap


pear on the buccal mucosa. Rubella is
20. d (Chapter 20)
caused by rubella vi
rus and is characterized by mild fever and
This child has obstructive sleep apnea, most likely from
erythematous maculop
apular rash, with generalized lymenlarged tonsils or adenoids or both.This is easily diagnosedphadenopathy, espec
ially of the posterior auricular, cervical,
with a sleep study, which can also differentiate central fromand suboccipital no
des. Roseola infantum is caused by herobstructive sleep apnea. Removal of the obstructing tissue ispesvirus 6 and is c
haracterized by high fever followed by a
the treatment of choice in obstructive sleep apnea. Continu-maculopapular rash t
hat starts on the trunk and spreads to
ous positive airway pressure is more appropriate in cases ofthe periphery.The fe
ver typically resolves as the rash appears.
central sleep apnea. Oxygen therapy will not help if the
Erythema infectiosum
is caused by parvovirus B19 and is
patient is not breathing well to begin with. Antibiotic therapycharacterized by
marked erythema of the cheeks ("slapped
is not indicated because there is no infection to treat Stimu-cheek" appearance)
and an erythematous, pruritic, maculants have not been proven to be effective.
lopapular rash start
ing on the arms and spreading to the
trunk and legs. Hand
-foot-and-mouth disease is caused by
21. e (Chapter 12)
coxsackie A virus an
d is characterized by ulcers on the tongue
The newborn described in this scenario demonstrates signs and oral mucosa and
a maculopapular vesicular rash on the
hands and feet. Chic
kenpox is caused by varicella-zoster virus
and symptoms of congenital syphilis, characterized by
and is characterized
by fever and a pruritic papular, vesicular,
hepatomegaly, splenomegaly, mucocutaneous lesions, jaunpustular rash starti
ng on the trunk and spreading to the
dice, lymphadenopathy, and the characteristic "snuffles," a extremities.The infe
cted child is infectious until the last lesion
clear, copious nasal discharge. The mother's high-risk behav-is crusted over. Zo
ster, or shingles, is caused by reactivation of
iors suggest that multiple sexually transmitted diseases mayvaricella-zoster vir
us from the dorsal root ganglion and is
be present. Both RPR and VDRL tests are very likely to be pos-characterized by f
ever and painful pruritic crops of vesicles
itive, but the fluorescent treponemal antibody absorption along a dermatomal d
istribution in an individual with previ(FTA-ABS) test is a true treponemal test and is less likely toous varicella-zost
er infection.
result in a false-positive result. A complete blood count may
suggest infection but will not give the specific diagnosis. A
blood culture will be negative in this case. Newborns infected
with hepatitis B have a high likelihood of developing chronic24. b (Chapter 7)
disease but generally appear unaffected at birth. Most casesElectrocardiographic
changes associated with significant
of congenital cytomegalovirus are also clinically inapparent;hyperkalemia includ
e loss of P waves,"peaked"T waves, wide

however, 5% of those infected present with some constella- QRS complexes, and S
T segment depression. These changes
tion of intrauterine growth retardation, purpura, jaundice, may be seen at potas
sium levels of 7.0 or greater.Calcium gluconate does not rid
the body of potassium; however, it does
hepatosplenomegaly, microcephaly, intracerebral calcifica- stabilize the cardia
c cell membranes so that electrical activity
tions, and chorioretinitis.
is less likely to be
disrupted. In emergent situations, intravenous calcium gluco
nate is the best initial management of
22. b (Chapter 3)
hyperkalemia. Dialys
is is very effective at decreasing total
Congenital complete heart block is most likely given the
body potassium; howe
ver, it takes time to set up, so it is not a
maternal history of systemic lupus erythematosus. Because reasonable option in
emergent situations. Neither intrathere is no history of rash, Lyme disease causing complete venous glucose nor h
ypertonic NaCI solution is appropriate
heart block is unlikely. Tick exposure at this age is also in the management of
this patient.

--------------------------------------- 201
28
Blueprints Pediatrics
is low. After 6 months of age, both of these criteria
t elevated pulmonary arterial
are generally met. Subacute bacterial endocarditis
to congestive heart failure and
prophylaxis is not recommended for secundum atrial
patient with a large VSD with
septal defects but is indicated in primum and sinus
gy presents with shortness of
venosus atrial septal defects.
xertion, chest pain, and cyanosis.

a large shunt withou


pressures gives rise
growth failure. The
Eisenmenger physiolo
breath, dyspnea on e
The smaller the defe

ct, the louder the holosystolic


Ventricular Septal Defects
vascular resistance increases,

murmur. As pulmonary
the holosystolic mur

mur shortens and the pulmonary


The ventricular septal defects are the most common
eases in intensity. In the prescongenital heart defect, accounting for 25% of all
scular obstructive disease, a
congenital cardiac lesions. The five types of ventriave, ejection click, short systolic
cular septal defects are as follows:
stolic murmur of pulmonary

component of 82 incr
ence of pulmonary va
right ventricular he
ejection murmur, dia
valve insufficiency,

and loud, single S? are heard.


Muscular
mall defects may be normal or
Inlet
and a slight increase in pulConoseptal hypoplasia
ereas in large left-to-right

Chest radiograph for s


show mild cardiomegaly
monary vascularity, wh

Conoventricular
ncreased pulmonary vascularMalalignment
f the left atrium and left ven-

shunts cardiomegaly, i
ity, and enlargement o
tricle are seen. In

small defects the ECG is normal,


Muscular ventricular septal defects occur in the
VSD, left atrial, left ventricular,
muscular portion of the septum and may be single or
ertrophy is seen. Right ventricumultiple and located in the posterior, apical, or anteominates when pulmonary vasrior portion of the septum. The inlet VSD is an endohigh. On echocardiogram, the
cardial cushion defect and occurs in the inlet portion
ized, and Doppler flow mapping
of the septum beneath the septal leaflet of the
ection of flow.
tricuspid valve. Conoseptal hypoplasia VSDs are
positioned in the outflow tract of the right ventricle
beneath the pulmonary valve. The Conoventricular
e without intervention (40% by
VSD occurs in the membranous portion of the
ears), whereas the treatment for
ventricular septum. Malalignment VSDs result from
al closure before pulmonary vasmalalignment of the infundibular septum.
irreversible. Congestive heart
When the VSD is non-restrictive, pulmonary vasith digoxin, diuretics, and an
cular resistance (PVR) and systemic vascular resisng enzyme (ACE) inhibitor.
tance (SVR) determine shunt flow. When the PVR is
less than the SVR the shunt flow is left to right. Large
lar Canal
defects eventually result in pulmonary hypertension,
whereas small defects do not change PVR. The
ricular canal defect (Figure
amount of left ventricular and left atrial dilatation is
deficiency of the endocardial cushdirectly proportional to the size of the left-to-right
an ostium primum ASD and inlet
shunt. Right ventricular hypertrophy occurs when
tation of the mitral and tricuspulmonary vascular resistance increases. If left
trioventricular valve [CAW]).
untreated, the large VSD may result in elevated pulioventricular canal defect, the
monary arterial pressures and may lead to pulmonary
s directly to the top of the musvascular obstructive disease, and Eisenmenger's synventricular septum. As a result,
drome. In some cases of Eisenmenger's syndrome, the
ation beneath the atrioventricVSD shunt may reverse right to left. When the VSD
the right and left ventricles. The
is restrictive, shunt flow is left to right from the high
atrial level is an ostium primum
pressure LV to the lower pressure RV
e is cleft, and there may be some

whereas with a large


or biventricular hyp
lar hypertrophy pred
cular resistance is
defect can be visual
demonstrates the dir
Treatment
Most small VSDs clos
3 years, 75% by 10 y
large VSDs is surgic
cular changes become
failure is treated w
angiotensin-converti
Common Atrioventricu
The common atriovent
3-11), results from
ions and results in
VSD with lack of sep
pid valves (common a
In an incomplete atr
CAW leaflets attache
cular portion of the
there is no communic
ular valves between
communication at the
ASD. The mitral valv

degree of mitral reg


urgitation. In complete common
Clinical Manifestations
al, there is a CAW that is not
Clinical symptoms are related to the size of the
ular ventricular septum. As a
shunt. A small shunt produces no symptoms, whereas
arge inlet VSD located between the

atrioventricular can
attached to the musc
result, there is a l

--------------------------------------- 202
280
Blueprints Pediatrics
25. a (Chapter 3)
pmental delay and seizures in the first

presents with develo

year of life. Galact


osemia becomes evident soon after feedThe regular narrow-complex rhythm during tachycardia
ings start, manifest
ing as vomiting, growth failure, and
excludes atrial fibrillation, which is an irregular narrowcomplex rhythm. Flutter waves were not noted when adenohepatomegaly. Wilson
's disease presents with hepatitis,
sine was given, making a diagnosis of atrial flutter unlikely.usually after the
age of 5 years.
The preexcitation noted after conversion with adenosine is
consistent with Wolff-Parkinson-White (WPW) syndrome.The
fact that the tachycardia was narrow complex makes
29. a (Chapter 10)
The adjusted reticul
ocyte count (ARC) = [(measured hematthe tachycardia "orthodromic" reentrant tachycardia that
ocrit)/(normal hematoc
rit for age)] x reticulocyte count. An
travels down the atrioventricular node and up the bypass
ARC less than 2.0 su
ggests ineffective erythropoiesis, whereas
tract. If no preexcitation was noted after conversion with an ARC greater than
2.0 signifies effective erythropoiesis.
adenosine, then an idiopathic bypass tract would be more
likely. Sinus tachycardia is unlikely given the very fast rate, theAnemia caused
by a lack of production of red blood cells will
fact that the child is afebrile, and the fact that there is no evi-therefore hav
e an ARC less than 2.0, whereas anemias resultdence of cardiomyopathy.
ing from hemolysis o
r chronic blood loss will have an ARC
greater than 2.0.The
mean corpuscular volume (MCV) is used
to describe the anem
ia as microcytic, macrocytic, or normo26. d (Chapter 2)
cytic. All of the an
emias noted in the question result from
decreased red cell p
roduction and have an inadequate reticHepatotoxicity, manifested initially by elevated liver enzymes
and jaundice, may progress over several days to liver failureulocytosis (ARC < 2
.0). Decreased red cell production is due to
in people who have ingested large amounts of acetaeither deficiency of
hematopoietic precursors or bone
minophen when appropriate treatment is not sought.Cardiac marrow failure.The m
icrocytic anemia described in the quesarrhythmias can occur with anticholinergic or antiarrhythmiction is most likely
due to iron deficiency, which is not only the

ingestions. Acute iron overdose and other specific ingestionsmost common microcy
tic anemia, but also the most common
can cause seizures. Malignant hypertension and ineffective cause of anemia duri
ng childhood. It is most often seen
hemostasis are not associated with acetaminophen ingesbetween 6 and 24 mon
ths of age.Thalassemia syndromes are
tion.The blood acetaminophen level at 1 hour is not predic- also microcytic anem
ias but are less common than iron defitive of outcome, because timely intervention, even more thanciency anemia. Anemi
a of chronic disease may be microcytic
1 hour after ingestion, can prevent or ameliorate complica- or normocytic. Trans
ient erythrocytopenia of childhood is a
tions. However, the blood acetaminophen level at 4 hours
normocytic anemia th
at is an acquired red cell aplasia. Parafter ingestion is very predictive of outcome, because by thenvovirus B19 aplast
ic crisis is a normocytic anemia that results
the drug has been absorbed and is passing through the liver,from parvovirus B19
marrow suppression of erythropoietic
its primary organ of toxicity.
precursors.
27. c (Chapter 15)
30. a (Chapter 8)
Maternal folic acid supplementation decreases the incidence The most common caus
e of rectal bleeding in toddlers is an
of neural tube defects. A family history of neural tube defectsanal fissure. If
there were significant upper gastrointestinal
increases the risk slightly in subsequent children. A high tract bleeding from
peptic ulcer disease or Mallory-Weiss tear,
maternal serum alpha-fetoprotein level is associated with anthe child would have
melena instead of blood-streaked stool.
increased risk of neural tube defect in the fetus; low levels areInflammatory bo
wel disease and necrotizing enterocolitis
more predictive of Down syndrome. Children with spina
could both cause low
er gastrointestinal tract bleeding (hemabifida have wide variation in the level of lower extremity tochezia or blood-st
reaked stool) but are unlikely in an 18involvement.
month-old.
28. a (Chapter 20)
31. d (Chapter 14)
Meconium ileus is highly associated with cystic fibrosis, anEdema can be caused
by protein losses from the gastroinautosomal recessive disease with a frequency of about 1 in testinal tract, vasc
ulature, or kidneys. Congestive heart failure
2500 births. Infants with phenylketonuria are usually
will also result in
edema, but this etiology is rare in children.
detected on state newborn screening tests;those who are not Nephrotic syndrome i
s characterized by proteinuria, hypoalare diagnosed generally much later with mental retardation buminemia, hyperlipi
demia,and edema.Marked hematuria is
and behavioral problems. Tay-Sachs disease is a lipidosis, more common with the
glomerulonephritis syndromes. The
whereas galactosemia is a disorder of carbohydrate metabo- most common cause of
nephrotic syndrome in children, and
lism; neither presents with meconium ileus.Tay-Sachs diseasefortunately the most
benign, is minimal change disease.

--------------------------------------- 203
Answers
281
Although minimal change disease has a, good prognosis

reticuloendothelial

system infiltration, and penetration of


generally, it does require treatment; salt restriction and
tral nervous system and testicles). Marrow
corticosteroids are usually effective. Focal segmental
in crowding out of normal marrow blood
glomerulosclerosis is much less common, but the prognosis
ch then results in anemia (pallor)
is worse. Urinary tract infections do not cause edema,
(petechiae). Infiltration of the reticualthough mild proteinuria is possible. Most causes of renal
results in lymphadenopathy and
masses also do not cause edema or proteinuria; hematuria
Bone pain is due to expansion of the
arid hypertension are more typical.
uction of cortical bone by leukemic cells,

sanctuary sites (cen


infiltration results
cell precursors, whi
and thrombocytopenia
loendothelial system
hepatosplenomegaly.
marrow cavity, destr
or metastatic tumor.

Although fever and petechiae are consistent with aplasti


c anemia, bone pain, lymphadenopathy,
32. a (Chapter 3)
Beta-blocker therapy is the most appropriate chronic therapyand hepatosplenomega
ly are not.
for long QT syndrome. Nadolol minimizes the number of premature ventricular contractions (PVCs). Fewer PVCs decreases35. c (Chapter 17)
the risk of PVC R-wave depolarization on the vulnerable partNeuroblastoma is the
most common malignant tumor in
of the! wave, thereby decreasing the risk of ventricular tachy-infancy. Neurobla
stoma is a malignant tumor of the neural
cardia and ventricular fibrillation seen in long QT syndrome.crest cells that fo
rm the adrenal medulla and the paraspinal
Lidocaine would be an appropriate acute therapy at the time sympathetic ganglion
. Abdominal tumors account for 75% of
of the ventricular tachycardia to stabilize the myocardium. the neuroblastoma tu
mors (two-thirds adrenal medulla, onethird retroperitonea
l sympathetic ganglion).Thoracic tumors
33. d (Chapter 10)
account for 20% of n
euroblastoma tumors and tend to arise
The most likely diagnosis is hemophilia A. Hemophilia A is anfrom paraspinal gan
glion in the posterior mediastinum. NeuX-1inked disorder that is caused by deficiency of factor VIII.roblastoma of the
neck (5% of neuroblastoma tumors)
Hemophilia B is also an X-linked disorder and is caused by involves the cervica
l sympathetic ganglion. In neuroblastoma
factor IX deficiency. Hemophilias A and B are characterized byof the abdomen, th
ere is often displacement of the kidney
and minimal distorti
on of the calyceal system. This is in conspontaneous or traumatic hemorrhages, which can be subtrast to Wilms'tumor
, in which there is significant distortion
cutaneous, intramuscular, or within joints (hemarthroses). of the calyceal syst
em. Because 70% of children with neuroLife-threatening internal hemorrhages may follow trauma or
surgery.The PTT is prolonged, the PT is normal,and in hemo- blastoma have distan
t metastases, treatment generally
philia A the factor VIII coagulant activity (Vllkc) is decreased.involves surger

y (for tumor debulking) and chemotherapy.


Other than their factor replacement regimens, there is no distinguishable difference between hemophilias A and B. Idio- 36. d (Chapter 16)
pathic thrombocytopenic purpura is unlikely in this patient,The infant in this q
uestion most likely has colic, although sigsince the platelet count is normal at 150,000.With no historynificant disease sh
ould be ruled out with a good history and
of epistaxis, gingival bleeding, or cutaneous bruising, von physical exam. Colic
begins around age 3 weeks and can last
Willebrand's disease is unlikely. Hemarthroses are not typicalup to age 3 months
.lt is characterized by an infant who seems
for von Willebrand's disease. Vitamin K deficiency occurs ingenerally well durin
g most of the day but develops crying
the neonate who is exclusively breast-fed and has not
spells that last sev
eral hours at a time up to three times a
received prophylactic vitamin K injection after birth or in theweek.These tend t
o be in the evening hours.The infant is genchild with significant fat malabsorption. In vitamin K defi-erally inconsolable
during these spells. Formula changes have
ciency and in liver disease, there is a prolonged PT and normalnot been found to
ameliorate true colic. A 6-week-old breastfactor VIII coagulant activity. The most appropriate therapyfed infant is a litt
le young for both otitis media and intussusfor complications of hemophilia A is to infuse factor VIII ception; no fever is
present, and the symptoms have been
concentrate.
going on for too lon
g to be either of these conditions. Maiabsorption presents
with diarrhea and often failure to thrive,
34. a (Chapter 17)
neither of which is
present here. Milk protein intolerance is
The leukemias account for the greatest percentage of child- extremely unlikely i
n a breast-fed infant.
hood malignancies. Acute leukemias constitute 97% of all
childhood leukemias and are divided into acute lymphocytic 37. a (Chapter 13)
leukemia (ALL) and acute myelogenous leukemia (AML). ALL
Neonatal sepsis is g
enerally divided into early-onset sepsis,
accounts for 80% of all childhood acute leukemias. A historylate-onset sepsis, a
nd nosocomial sepsis. Staphylococcus
of fever, pallor, anorexia, bone pain, lymphadenopathy,
aureus is typically
a nosocomial infection found in preterm
petechiae, and hepatosplenomegaly is consistent with ALL. infants in the neona
tal intensive care unit from 7 days of life
Leukemic cell dissemination results in bone marrow failure, to discharge. It is
not a typical pathogen of early-onset sepsis.

--------------------------------------- 204
282
Blueprints Pediatrics
The neonate described has early-onset sepsis (birth to 7 daysJohnson syndrome is
the most severe form of erythema mulof life), which occurs after colonization with bacteria from thetiforme. Stevens
-Johnson syndrome is characterized by fever,
mother's genitourinary tract. The bacteria responsible for erythema multiforme

rash, and inflammatory bullae of two or


early-onset sepsis include group B streptococci, Escherichiamore mucous membrane
s (oral mucosa, lips, bulbar conjunccoli, Klebsiella pneumoniae.and Listeria monocytogenes. Grouptiva, anogenital ar
ea). Toxic epidermal necrolysis is the most
B streptococci are the most common cause of neonatal
severe form of cutan
eous hypersensitivity and is charactersepsis; sepsis caused by these organisms classically occurs ized by widespread s
kin erythema, tenderness, mucosal
with a bimodal distribution, early onset and late onset. Strep-involvement, and
sloughing of the epidermis. Eczema is not
tococcus pneumoniae sepsis typically occurs in infants and usually exacerbated
by medication exposure. Urticaria is the
school-age children rather than neonates. Chlamydia tramost common hypersen
sitivity reaction in the skin, is characchomatis classically causes conjunctivitis and afebrile
terized by hives, an
d may result from medication exposure.
pneumonia, whereas Staphylococcus epidermidis causes
bloodstream infections in neonates with central venous
41. a (Chapter 9)
catheters; neither causes fulminant neonatal sepsis.
The clinical descrip
tion is that of a patient with trisomy 21, or
Down syndrome.Common
dysmorphic facial features include
38. d (Chapter 13)
flat facial profile,
upslanted palpebral fissures, a flat nasal
Necrotizing enterocolitis refers to a process of acute intesti-bridge with epica
nthal folds, a small mouth with a protruding
nal necrosis after ischemic injury to the bowel and secondarytongue, micrognathi
a, and short ears with downfolding ear
bacterial invasion of the intestinal wall. Bowel ischemia as alobes. Other dysmo
rphic features are excess skin on the back
result of respiratory compromise in the preterm infant causesof the neck, microc
ephaly, a flat occiput (brachycephaly), short
bowel injury.The introduction of enteral feeding provides thestature, a short st
ernum, small genitalia, and a gap between
substrate for bacterial overgrowth. Bacterial invasion of thethe first and secon
d toes ("sandal gap toe"). Anomalies of the
bowel wall leads to tissue necrosis and perforation. Pneu- hand include single
palmar creases (simian creases) and
matosis intestinalis results from gas production in the bowelshort, broad hands
(brachydactyly) with fingers marked by an
wall and is pathognomonic for necrotizing enterocolitis. Pre-incurved fifth fing
er and a hypoplastic middle phalanx (climature infants with birth weights less than 2000g who have nodactyly). Features
of trisomy 18 include hypertonia, microbeen asphyxiated are the population at highest risk. Prenatalcephaly, corneal op
acities, micrognathia, and rocker bottom
factors associated with necrotizing enterocolitis include feet. Features of tr
isomy 13 include microcephaly, occipital
maternal age greater than 35, maternal infection requiring scalp defects, iris
coloboma, microphthalmia, cleft lip and
antibiotics, premature rupture of membranes, and cocaine
palate, and clenched
hands. Boys with Klinefelter's syndrome
exposure. Perinatal factors include maternal anesthesia,
do not have physical
features identifiable at birth that could
depressed Apgar score at 5 minutes, birth asphyxia, respira-lead to suspicion of
the disorder. Girls with Turner's syndrome
tory distress syndrome, and hypotension. Postnatal factors have a webbed neck,
low posterior hair line, wide-spaced
include patent ductus arteriosus, congestive heart failure, nipples, cubitus val

gus (wide carrying angle), and edema of


umbilical vessel catheterization, polycythemia, and exchangethe hands and feet.
transfusion.
42. b (Chapter 9)
39. a (Chapter 4)
Functional and struc
tural abnormalities in children with
The initiation sequence of sexual development in males is tes-trisomy 21 include
generalized hypotonia (obstructive sleep
ticular enlargement, penile enlargement, height growth
apnea), cardiac defe
cts (endocardia! cushion defects and
spurt,and pubic hair, whereas the sequence forfemales is the-septal defects are
seen in 50% of cases), gastrointestinal
larche, height growth spurt, pubic hair, and menarche.
anomalies (duodenal
atresia and Hirschsprung's disease),
Although the events of puberty occur in a predictable
atlantoaxial instabi
lity, developmental delay, moderate
sequence, the timing of the initiation and the velocity of themental retardation
, and hypothyroidism.There is a higher frechanges are highly variable among individuals.
quency of leukemia i
n children with trisomy 21 than in the
general population.
40. d (Chapter 5)
Erythema multiforme is an acute, self-limited, uncommon
43. d (Chapter 10)
hypersensitivity reaction that may be secondary to sulfa
The most likely diag
nosis is immune thrombocytopenic
drugs. Erythema multiforme is characterized by symmetric
purpura. Isoimmune t
hrombocytopenia is noted in newlesions evolving through multiple morphologic stages: ery- borns, not in childr
en. Isoimmune IgG antibodies are prothematous macules, papules, plaques, vesicles, and target duced against the fe
tal platelet when the fetal platelet crosses
lesions, with sparing of the mucosal surfaces. Stevensthe placenta and has
antigens that are not found on the

--------------------------------------- 205
Answers
283
maternal platelet.The maternal antibodies cross the placentaquadrant. The barium
or air enema results in hydrostatic
and attack the fetal platelets. Leukemia, sepsis, and hyper-reduction of the int
ussusception in 75% of cases.
splenism may all cause thrombocytopenia in the child's age
group, but are unlikely in this case.The white blood cell count48. a (Chapter 8)
is normal, and no immature white cells are seen on the
peripheral smear. Sepsis is unlikely, given that the child Projectile nonbiliou
s vomiting is the cardinal feature seen in
appears well and is hemodynamically stable. Hypersplenism virtually all patien
ts with pyloric stenosis. Physical findings
is unlikely when the spleen is normal on palpation.
vary with the severi
ty of the obstruction. The classic finding
of an olive-sized, m
uscular, mobile, nontender mass in the epigastric area occurs
in most cases. Dehydration and poor

44. a{Chapter15)
weight gain are comm
on when the diagnosis is delayed.
Reye's syndrome is much less common now that parents are
Hypokalemic, hypochl
oremic metabolic alkalosis with dehyinstructed to avoid aspirin in children. The most consistentdration is seen seco
ndary to persistent emesis in the most
laboratory abnormalities are hyperammonemia and elevated
severe cases.
hepatic enzymes, although glucose and electrolytes may be
abnormal as well. Hypercalcemia is not typical of Reye's

49. c (Chapter 8)

syndrome.
Crohn's disease typi
cally is associated with ileal and/or
colonic involvement
with skip lesions, rectal sparing, seg45. c (Chapter 13)
the ileum (string sign), granuloma,
Polyhydramnios is defined as an amniotic fluid volume
and transmural disease. The presence of
greater than 2 liters. Chronic polyhydramnios is more
eases the risk of colon cancer only slightly.
common than acute polyhydramnios. Polyhydramnios may
ypically is characterized by rectal involveresult in prematurity. Polyhydramnios is associated with
g, crypt abscesses, and diffuse superficial

mental narrowing of
intestinal fistula,
Crohn's disease incr
Ulcerative colitis t
ment, rectal bleedin
mucosal ulceration,

and its presence significantly increases


lesions that impair fetal swallowing, such as neural tube
ncer.
defects (anencephaly and myelomeningocele), abdominal

the risk of colon ca

wall defects (omphalocele and gastroschisis), esophageal or


duodenal atresia, and cleft palate, as welt as gestational dia-50. b (Chapter 14
)
betes, immune or nonimmune hydrops fetalis, multiple ges- A voiding cystoureth
rogram will demonstrate vesicoureteral
tations, and trisomy 18 or 21. Oligohydramnios is a decreasedreflux if it is pre
sent. Renal ultrasound and intravenous pyeamount of amniotic fluid and is associated with postmaturity,lography are benefi
cial for ruling out a renal mass, whereas
amniotic fluid leak, intrauterine growth retardation, and con-the nuclear medici
ne scan may show areas of renal scarring
genital anomalies of the fetal kidneys. Bilateral renal agenesisdem'onstrating p
revious episodes of pyelonephritis. CAT
results in Potter's syndrome, which is associated with renalscans may demonstrat
e dilated ureters but do not detect
anomalies, oligohydramnios, and pulmonary hypoplasia.
vesicoureteral reflu
x and are not generally part of the evaluation of uncomplicat
ed urinary tract infections in this age
46. a (Chapter 8)
group. ..,.-.. : .-.
.. . .. ... ...
The abdominal examination reveals peritoneal signs
(rebound tenderness and guarding) that are consistent with 51. a (Chapter 20)

appendicitis or pancreatitis, but not with viral gastroenteritis,Corticosteroids


,suchas prednisone and methylprednisolone,
urinary tract infection, or diabetes mellitus. In the latter threerequire 4 to 6
hours to take effect. However, they are very
diagnoses, the re may be some diffuse nonspecific abdominal important in the tre
atment of an acute exacerbation because
pain, but peritoneal signs are unlikely. The description of thethey address the
underlying inflammation and prevent the
movement of the pain from periumbilical to the right lower "late-phase" respons
e.Theophylline,atbuterol,andterbutaline
quadrant is typical for appendicitis. Pain from pancreatitis isare bronchodilato
rs that have virtually no anti-inflammatory
generally noted in the epigastric area, with radiation to theproperties. Cromoly
n is a mast cell stabilizer that is not effecback.
tive in acute asthma
exacerbations.
47. c (Chapter 8)
52. a (Chapter 11)
The history, physical examination, and abdominal radiograph The primary serious
complications of Kawasaki's disease are
are classic for a diagnosis of intussusception, the"telescoping"cardiac, includi
ng coronary vasculitis and aneurysm formaof a proximal segment of bowel into a more distal segment. tion. Prognosis is t
ied to cardiac involvement; cardiac instaIn cases of intussusception, barium enema demonstrates a
bility can produce a
rrhythmias, infarction, or congestive heart
"coiled spring" appearance to the bowel in the right lower failure within days
of presentation. Aneurysms and coronary

--------------------------------------- 206
Blueprints Pediatrics
284
artery disease persist and may result in death months to yearspresent with recur
rent bacterial infections and an increased
later. Patients with Kawasaki's disease may manifest sterilerisk of autoimmune d
isease. Patients with cell-mediated
pyuria; however, they are not at risk for kidney failure. Arthri-immunity will h
ave infections with opportunistic or lowtis, gastrointestinal bleeding, and hypertension are also grade organisms.
neither early nor late complications of Kawasaki's disease.
58. b (Chapter 11)
53. c (Chapter 15)
In patients with juv
enile rheumatoid arthritis, a positive
Hypsarrhythmia is the pattern seen on electroencephaloANA indicates an inc
reased risk for the incidence of chronic
gram in patients with infantile spasms. A characteristic gen-uveitis.These patie
nts require more frequent ophthalmologic
eralized, symmetric three-per-second spike and wave pattern examinations.
would be expected in a patient with absence seizures. Causes
of increased cerebrospinal fluid protein levels include Guil59. a (Chapter 6)
lain-Barre syndrome.
Turner's syndrome is
relatively common, with an incidence of
1 in 2500. Female pa
tients will present with short stature and
54. b (Chapter 1)
delayed puberty caus
ed by primary ovarian failure.Other stig-

Epinephrine is used for asystole, bradycardia, and/or ventric-mata, including we


bbed neck, a low hairline, and increased
ular fibrillation. Low-dose epinephrine increases systemic carrying angle, may
not be present. Patients with Cushing's
vascular resistance, chronotropy, and inotropy, thereby
syndrome will presen
t with other physical characteristics,
increasing cardiac output and systolic and diastolic blood including moon facie
s, buffalo hump,and abdominal striae. In
pressure. By increasing systolic blood pressure,cerebral bloodisolated growth ho
rmone deficiency and familial short
flow is increased; by increasing diastolic blood pressure, coro-stature, patient
s will not have delayed puberty. Patients with
nary perfusion is increased. Low-dose epinephrine may
Addison's disease pr
esent with fatigue, weakness, nausea, and
change fine ventricular fibrillation to coarse ventricular fibril-vomiting. In t
he acute setting, they may present with cardiolation and promote successful defibrillation.
vascular shock.
55. c (Chapter 18)
60. c (Chapter 16)
Ophthalmia neonatorum caused by Neisseria gonorrhoeas
Vitamin D is found i
n smaller amounts in breast milk when
typically presents at 2 to 5 days of life with bilateral conjunc-compared with i
nfant formulas. Infants that are exclusively
tival injection, purulent discharge, and eyelid edema. Appro-breast-fed need to
be supplemented with vitamin D. Iron is
priate treatment consists of either intravenous ceftriaxone oralso found in smal
ler amounts in breast milk; however, it has
penicillin plus saline lavage.Conjunctivitis caused by Chlamy-increased bioavail
ability, and supplementation is not
dia trachomatis is treated with oral and topical erythromycin.required. Vitamin
C, calcium, and folic acid are found in the
Chemical conjunctivitis does not require specific therapy. appropriate quantiti
es in breast milk.
Herpes simplex virus is treated with acyclovir.
61. c (Chapter 20)
56. c (Chapter 4)
The patient describe
d has persistent symptoms that require
A 6-month-old infant is able to sit well unsupported, reach frequent rescue medi
cations and visits to the emergency
and transfer objects with either hand, and babble. At 12
room. Inhaled cortic
osteroids have become the mainstay of
months, a child will learn to walk alone, use a pincer grasp,preventive medicine
s for moderate to severe asthma. Croand be able to say a few words. A 24-month-old child will walkmolyn sodium has l
argely fallen out of favor because it
up and down stairs and follow two-step commands. The 3requires frequent ad
ministration (three to four times daily)
year-old will be able to ride a tricycle, draw a circle, and useand is not as ef
fective as inhaled steroids.Theophylline is very
three word sentences. At 5 years old, a child will be able todifficult to dose b
ecause of its narrow therapeutic window.
hop and skip, tie shoes, and recognize colors.
Leukotriene inhibito
rs are an effective second-line agent that
should be considered
in addition to inhaled corticosteroids if
57. d (Chapter 11)
greater symptom cont
rol is needed. Long-acting p-agonists
may also play a role
if a patient requires daily bronchodilator
In phagocytic disorders, such as chronic granulomatous
therapy in the face

of other maintenance medications.


disease, patients present with recurrent skin infections and
abscess formation. The most common organisms include
Staphylococcus aureus, Pseudomonas, and fungi. Disorders of 62. e (Chapter 7)
humoral immunity involve frequent infections of the sinuses,Maintenance fluids a
re calculated by giving 100 cc/kg/day for
middle ear, and lung. In complement deficiencies, patients the first 10kg of bo
dy weight. For this patient, 800 cc over 24

--------------------------------------- 207
Answers
285
hours gives an hourly rate of approximately 33cc/hr. Sodium and hypernatremia. A
patient in shock should receive fluid
requirements are 2 to 3 mEq for every 100 cc of maintenance resuscitation with i
sotonic crystalloid such as normal saline or
fluids.Therefore, this patient needs 24mEq of Na in 24 hours'lactated Ringer's s
olution. It is not appropriate to give large
worth of maintenance fluids, giving a concentration of 30 volumes of free wate
r in the form of dextrose solution since
mEq per liter of fluid.There are 154mEq of sodium in a literthe water will drama
tically change electrolytes and will not
of normal saline, 77 mEq in a liter of one-half normal saline,be retained in the
vascular space as well as isotonic crystaland 38 mEq in a liter of one-fourth normal saline. Mainte- loid. Hypernatremia
needs to be corrected slowly over 48
nance potassium needs are approximately 2 mEq for every
hours in order to av
oid cerebral edema, a potentially devas100cc of maintenance fluids.This patient needs 16 mEq in 24 tating consequence o
f rapid correction. Oral rehydration
hours' worth of fluid, giving a concentration of 20mEq/L
refers to a techniqu
e of frequent small volumes of fluids with
Maintenance fluids should almost always contain dextrose to balanced electrolyte
s given by mouth. It is a very effective
try to prevent a catabolic state, usually 5% dextrose,althoughtechnique in the s
etting of mild to moderate dehydration,
neonates frequently require a 10% solution. The choice pro- but is not appropria
te for a patient in shock with abnormal
viding fluids closest to this child's needs is D5 one-fourthelectrolytes.
normal saline with 20 mEq of KCI/L at 35 cc/hr.
67. d (Chapter 15)
63. d (Chapter 13)
Although the most co
mmon cause of seizure is idiopathic
This infant's bilirubin is rising fasterthan 5 mg/dL per 24 hoursepilepsy, the h
yperpigmented macules {caf6 au lait spots) in
and is therefore likely pathologic rather than physiologicthi. s patient make ne
urofibromatosis the most likely diagnoHepatitis usually gives conjugated hyperbilirubinemia secsis- . It will be impor
tant to examine the patient for other signs
ondary to hepatocyte injury, and echovirus generally presentsof type 1 neurofibr
omatosis, including axillary freckling, neuwith other symptoms in addition to hyperbilirubinemia.The rofibromas, Lisch no
dules, optic gliomas, or bony abnormalihematocrit of 48 rules out polycythemia. Biliary atresia is a dis-ties. Tuberous

sclerosis is another neurocutaneous disorder


order of biliary secretion and therefore causes conjugated that may be associat
ed with seizures secondary to tubers
hyperbilirubinemia. Maternal antibodies to the infant's refound d in the brain.
The skin markings of tuberous sclerosis
blood cells, as seen in ABO incompatibility, is a relativelyinclude hypopigmente
d ash-leaf spots. Meningitis is unlikely
common cause of unconjugated hyperbilirubinemia. Thiwithous
t a fever or
other signs of infection. Sturge-Weber syninfant requires phototherapy and close monitoring of the
drome is generally a
severe neurologic disorder characterized
hemolytic process.
by a port-wine stain
on the face, mental retardation, and
visual impairment.
64. b (Chapter 8}
Corticosteroids remain the mainstay of therapy for acute

68. e (Chapter 13)

exacerbations of inflammatory bowel disease.Tumor necrosis Cytomegalovirus is l


ikely responsible for this syndrome
factor (TNF) alpha inhibitors are new medications for controlof intrauterine gro
wth retardation, hepatosplenomegaly,
of significant disease. Metronidazole is an antibiotic that is anand periventric
ular calcifications. Chorioretinitis, "blueberry
effective adjunct for Crohn's disease. Sulfasalaztne is the mostmuffin" rash, an
emia, thrombocytopenia, and jaundice may
commonly used maintenance medication for inflammatory
also be seen. It is
diagnosed by rapid antigen detection or
bowel disease. Azathioprine is an immunosuppressive medviral culture from t
he infant's urine. Herpes simplex virus is
ication used for control of chronic symptoms as a steroid- more likely to be ac
quired perinatally rather than as a consparing agent.
genital infection sy
ndrome, and growth retardation is not a
likely feature. Plac
enta! insufficiency is a much more common
65. a (Chapter 15)
cause of intrauterin
e growth retardation, but is not associated
The physical finding described in an otherwise normal child with the other findi
ngs described in this infant. Chorioamis the consequence of fusion of the coronal sutures, a form ofnionitis is a risk
factorfor early sepsis.Trisomy 13 is associated
craniosynostosis.von Hippel Lindau disease is a disorder char-with a number of p
hysical findings not seen in this infant,
acterized by vascular hamartomas.Macrocephaiy is a descrip- including cleft lip
or palate, polydactyly, hypotetorism,
tive term indicating a head circumference greater than 2
microphthalmos, and
overlapping fingers.
standard deviations above the mean. Tuberous sclerosis and
neurofibromatosis are neurocutaneous disorders with spe-

69. a (Chapter 17)

cific clinical criteria that do not include craniosynostosis.Cerebella astrocyto


ma and other infratentorial tumors often
66. c (Chapter 7}
present with deficit
s of balance or brainstem function. Head
tilt is a compensati
on for loss of binocular vision and is noted
The child described has significant dehydration with evi- with focal deficits

of cranial nerve III, IV, or VI, which cause


dence of compensated shock (tachycardia and listtessness)
eakness. Craniopharyngioma presents

extraocular muscle w

--------------------------------------- 208
286
Blueprints Pediatrics
with bitemporal hemianopia and pituitary dysfunction. Optic infancy. Although a
number of viruses can cause bronchioliglioma is more common in children younger than 2 years.
tis, respiratory syn
cytial virus (RSV) is most commonly isoAlthough children with optic glioma may have nystagmus,
lated. Only about 20
% of cases of bronchiolitis respond to
they also have exophthalmos and strabismus. Metastatic neu- p-agonist therapy.Cr
oup and epiglottis typically present with
roblastoma and acute lymphocytic leukemia do not usually
stridor. Chlamydia t
rachomatis is acquired perinatally, and
present with signs of cerebellar dysfunction, although chil-pneumonia caused by
this organism typically manifests at 2
dren with neuroblastoma on rare occasion develop opsoto 3 months of age.
clonus-myoclonus syndrome.
73. e (Chapter 12)
70. d (Chapter 10)
Hydroxyurea maintenance therapy has been shown to reduce
Streptococcus pneumo
niae is by far the most common cause
the number and severity of vasoocclusive crises in individu-of bacterial pneumon
ia in children. Since the introduction of
als with sickle cell disease.Children with sickle cell disease, likethe Haemophi
lus influenzae type b (Hib) vaccine, the inciall children, require all routine childhood vaccinations. dence of invasive Hi
b disease has decreased dramatically. Hib
Despite penicillin prophylaxis, children with sickle cell diseaserarely is seen
in the United States anymore. Nontypeable H.
are still at high risk of sepsis caused by Streptococcus pneu-influenzae, which
lacks the polysaccharide capsule, is a
moniae. These children require both the pneumococcal
common cause of otit
is media and sinusitis in children. Nonconjugate vaccine (7-valent) during infancy, and the pneu- typeable H. influenz
ae causes pneumonia only rarely in
healthy children. Pn
eumonia caused by Staphylococcus
mococcal polysaccharide vaccine (23-valent) at 4 to 6 years ofaureus usually is
associated with empyema or pneumatoage. Gallstones typically develop during adolescence as a celes. Listeria mono
cytogenes may cause pneumonia in
result of chronic hemolysis. Dactylitis, or hand-foot syndrome,neonates or in im
munosuppressed older children.
is the earliest manifestation of vasoocclusive disease. It is
caused by avascular necrosis of the metacarpal and
metatarsal bones and requires analgesics, not antibiotics. 74. a (Chapter 12)
Acute chest syndrome requires both supportive care (supplemental oxygen, red blood cell transfusions) and antibi- HIV DNA polymerase c
hain reaction (PCR) is the most effecotics.
tive way to test an
infant for HIV. HIV culture should also be
sent but requires 2
to 4 weeks for growth. HIV RNA PCR will
71. b (Chapter 14)
determine viral load

but is limited by the viral burden, and a


negative RNA PCR tes
t does not exclude HIV. Passively
Testicular torsion represents a surgical emergency. Delay inacquired maternal HI
V antibody may be present up to 18
surgical detorsion may result in irreversible testicular necro-months of age and
therefore cannot be used as an HIV screensis. Ultrasound may help confirm the diagnosis prior to
ing test in young ch
ildren. CD4 and absolute lymphocyte
surgery. MRI would unnecessarily delay surgical correction. counts may be helpfu
l in the evaluation, but they are not
Testicular torsion does not have a known infectious cause. appropriate diagnost
ic tests for HIV.
Although analgesia may be given to the child with suspected
testicular torsion, urgent surgical intervention is indicated.
Orchiopexy is the procedure used to correct undescended
75. d (Chapter 9)
testes.
The child has a 25%
chance of acquiring the autosomal recessive disorder. Becau
se each parent is a carrier for the disorder,
72. e (Chapter 12)
each parent has one
normal allele and one mutant allele.The
Bronchiolitis is a viral infection of the lower respiratory tract,probability of
the child receiving an affected allele is 0.5 from
which classically presents with new-onset wheezing in
each parent Therefor
e, the child has a 25% risk (0.5 x 0.5).

--------------------------------------- 209
Index
Note: Page numbers with an / indicate figures; those with a t indicate tables.
ABCDs, of melanoma, 56
milk, 75-76, 79t, 87t, 226, 266
ABCDEs, of primary survey, 1-5, 2f,
otitis media and, 139
267, 276-277
rashes from, 53-55
Abdomen
vaccinations and, 137t
abscess in, 137
ttj-antitrypsin deficiency, 181
neuroblastoma of, 237-238
Alpha-fetoprotein, 212, 233t, 280
pain in, 75-81, 90, 134, 156-157,
Alport's syndrome, 202t, 203-204
201, 283

ADHD. See Attention deficit


hyperactivity disorder
Adjusted reticulocyte count (ARC),
103, 280
Adrenal dysfunction, 64-69
arrhythmias with, 36t
demyelination and, 222
hyponatremia and, 71

trauma to, 2t
Alzheimer's disease, 97

shock from, 7t

Amblyopia, 243-245
Abetalipoproteinemia, 111

vomiting with, 79t

ABO incompatibility, 179, 185t, 189,


Amebiasis, 91

Adrenal hyperplasia, 95t

American Indians
274, 285
Abortion, 156, 196
alcoholism among, 195

Adrenocorticotropic hormone (ACTH)


adrenal hyperplasia and, 64-65
Cushing's syndrome and, 67

diabetes among, 59
Abruptio placentae, 164t, 165
jaundice and, 179
Absolute lymphocyte count (ALC),
Mongolian spots of, 195
126t
SIDS among, 14

deficiency of, 67-68, 190


precocious puberty and, 66
seizures and,.215

Absolute neutrophil count (ANC),


Amino acid metabolism disorders, 181,
126t
190

Adrenoleukodystrophy, 222
African Americans

Acetaminophen poisoning, 9t, 270, 280


Aminocaproic acid, 120, 122
N-Acetylcysteine, 9t
Amiodarone, 4t, 6f, 40f, 4If

asthma among, 259


cystic fibrosis and, 262

Achondroplasia, 60, 95t, 254


Amkacin, 174
Acne, 51-52
Amniocentesis, 96
Acquired immunodeficiency syndrome
Amoxicillin
(AIDS). See HIV disease
for Lyme disease, 161

diabetes among, 59
Ewing's sarcoma among, 241
G6PD deficiency among, 96t, 115
infant mortality rate among, 162
low-birth-weight infants of, 164

Acrocyanosis, 16
for otitis media, 139
ACTH. See Adrenocorticotropic
for pneumonia, 148

Mongolian spots of, 195


sickle cell disease among, 95t

for UTIs, 199, 201


hormone

SIDS among, 14

Acute chest syndrome, 113t, 114, 266


Amphetamines

SLE among, 132

for ADHD, 45
Acute lymphocytic leukemia (ALL),

thalassemia among, 108-109

229-232, 232t, 281


overdose of, 9t
short stature and, 61
Acute myelogenous leukemia (AML),
Ampicillin, 150, 174
229-232. See also Leukemia

Agammaglobulinemia, 54, 96t, 125, 230

AIDS. See HIV disease


Airway

Anaphylaxis, 2t, 7t, 125, 128, 265


Acute poststreptococcal
Androgens
glomerulonephritis (APGN),

management of, 1-3, 3f


obstruction of, 2t, 17t, 143, 207t,

140-141, 202t, 203-204


acne and, 51-52
for Fanconi's anemia, 118
Acute rheumatic fever (ARF), 141t,
as teratogen, 94t
251t

259, 266

Albinism, 105t
Alcohol abuse, 10, 13. See also

Acyclovir, 49, 156


Anemia, 103-118. See also specific types,

Substance abuse

e.g., Iron deficiency anemia aplastic,


Adam's bending test, 253
fetal effects of, 94t, 100, 164-165,
195-196
104t, 110, 112, 113t, 115, 117,
Addison's disease, 2t, 67-68, 72
120t, 230
Adenosine, 6f, 39, 40f
Alleles, 93-94
diagnosis of, 103-104, 104t, 105t,
Allergies, 128-130, 261
184, 185t
Adenovirus, 54, 145, 246
food, 54, 130-131
GER and, 81

--------------------------------------- 210
288

Blueprints Pediatrics
hemolytic, 96t, 111, 113, 119, 184 Apparent life-threatening events
diagnosis of, 26If
IBD with, 91, 109
(ALTEs), 265, 266t
treatment of, 260t, 261-262, 283
macrocytic, 104t, 115-118
Appendectomy, 77
vaccinations and, 136
microcytic, 104t, 106-110, 280
Appendicitis, 75, 77-78

Astrocytoma, 233t, 285-286


neonatal, 183-185, 185t
Asystole, 4t, 284
normocytic, 104t, 110-115

diagnosis of, 79t, 251t, 283

gastroenteritis vs., 151


Ataxia, 36t, 105t, 116, 222-223, 233t,
of chronic disease, 107t, 109-110
234
pernicious, 116
renal failure and, 209-210

IBD vs., 91

PIDre., 156

Ataxia-telangiectasia, 127, 222, 230, 235


Apt test, 184
types of, 104t, 185t
Atelectasis, 11, 145, 176
Arginine vasopressin receptor gene, 206
Anencephaly, 166, 212, 283
Atherosclerosis, 33, 59
Aneurysm, coronary, 32, 134-135,
Atrial fibrillation, 38, 39f, 41

Arnold-Chiari malformation, 212


Arrhythmias, 34-42, 37f-42f

283-284
Atrial flutter, 37-38, 39f
Angelman's syndrome, 99

anticholinergics and, 9t
apnea and, 266t

Atrial septal defect (ASD), 27-28, 32t


Angioedema, 95t, 130
arrhythmia with, 36t
cardiopulmonary arrest from, 2t
common atrioventricular canal
Angiotensin-converting enzyme (ACE)
inhibitors, 208
with, 28-29, 29f

causes of, 34
classification of, 35

rubella and, 169


Ankylosing spondylitis, 90

CPR and, 3-4, 4t

tachycardia with, 37
Ankyrin, 111
teratogens and, 94t
Anorexia nervosa, 66

GER and, 81
pacemakers for, 37, 38f, 39f
risks for, 36t

trisomies and, 97t


Anthracycline, 34, 36t
Atrioventricular (AV) block, 4t, 35-37,
shock from, 7t
Anticholinesterase, 221
37f, 39f
Anticoagulants, 41
Atrioventricular canal defect, 28-29,

sports and, 34

Arsenic, 111
Anticonvulsants, 188, 216, 217t
29f, 32t, 36t
poisoning from, 9t
Atropine, 2, 4t, 6f, 9t, 38, 39

Arteriovenous malformations (AVMs),

111, 219
thrombocytopenia from, 119
Attention deficit hyperactivity disorder
vitamin K deficiency from,
Arthritis, 54, 159t. See also Juvenile
rheumatoid arthritis

(ADHD), 43-46, 196, 214, 216,


123-124
231
Antidiuretic hormone (ADH), 60, 71

bacteremia and, 173

gonorrhea and, 156


Auspitz's sign, 53
Antidromic reentrant tachycardia

IBD and, 90

Autism, 45-46, 45t, 100


(ART), 37-38
Autoinfarction, 113
Antihistamines

Lyme, 160, 25It

Autosomal dominant disorders, 94, 95t


for erythema multiforme, 55

reactive, 132, 25It

poisoning from, 9t
Autosomal recessive disorders, 94 95,

rheumatoid, 61, 137

psoriatic, 53

95t, 117-118
for rhinitis, 129

septic, 50, 132, 137, 25It,

for urticaria, 54
Axonal injury, 217

257-258, 276
shingles and, 49

Antinuclear antibody (ANA), 131, 132t


Azathioprine, 91, 120
Anus
Azidothymidine, 115-116

SLE and, 132


Aryepiglottic folds, 144f

fissure of, 84-85, 87t, 280


Azithromycin, 156
imperforate, 191-192
Azotemia, 208-210
prolapse of, 263t
AZT (zidovudine), 158, 172
Aorta

Ash-leaf spots, 223

Asians

coarctation of, 17t, 26, 30f, 32t,


Bacteremia, 82, 138, 173. See also Sepsis
191,207t

jaundice and, 179


Mongolian spots of, 195
thalassemia and, 109

Bacteroides, 247
interrupted arch of, 17t, 26, 27f, Aspiration
100
Band 3 protein, 111
Aortic stenosis, 17t, 25-26, 30-32, 276
Barlow's maneuver, 249, 250f
Apgar score, 162, 163t
Bartonella henselae. See Cat-scratch

apnea and, 266t


dermatomyositis and, 134
foreign body, llf, 12t, 143,261
interstitial lung disease and, 266

disease
Apnea
meconium, 17t, 177-178
Bart's disease, 107
bronchiolitis with, 145
central, 17t
Battle's sign, 218
definition of, 265

pneumonitis and, 9t

Aspirin. See Nonsteroidal


anti-inflammatory drugs

B-cell immunodeficiency syndromes,


monitors for, 15, 186, 266
125-127, 126t
of infancy, 265, 266t

Asthma, 11, 79t, 129, 259-262

Beau lines, 105t


of prematurity, 185-186
Beckwith-Wiedemann syndrome, 166,
sleep, 99, 129, 228, 265, 269, 279

bronchiolitis and, 145


case study on, 267, 276, 283
classification of, 260t

183, 189, 194,239

--------------------------------------- 211
Index
289
Benzene, 117
neonatal, 17t, 185-188
Benzoyl peroxide, 52
infection of, 56, 60, 65
Beta-adrenergic blockers
tumors of, 61, 65, 79t, 230t,
233-234, 233t, 235t
arrhythmias from, 36t
for asthma, 260t, 261-262, 276,
Cephalexin, 50
284
Cephalohematoma, 162, 185t

Bronchiolitis, 145-146, 266t, 286


Bronchopulmonary dysplasia (BPD),
176

Bronchoscopy, 12
Brucellosis, 137

Brudzinski's sign, 149


bronchiolitis and, 145
Cerebellar astrocytoma, 233t, 285-286
hyperkalemia and, 72
Cerebellar ataxia, 222
for hypertension, 208

Bruton's agammaglobulinemia. See X-

linked agammaglobulinemia
Bulimia, 79t

Cerebral palsy, 213-214


for tachycardia, 39-41, 41f
child abuse and, 13
Bicarbonate, 4t, 6f, 73, 205

Burkitt's lymphoma, 231, 234

Burns, 12-13
classification of, 214t
Bike helmets, 10, 216
failure to thrive and, 227t
Biliary atresia, 180
strabismus with, 243

anemia with, 111


child abuse and, 12-14, 13f

Bilirubin metabolism, 179, 180f


Cerebral vascular accident (CVA).
Birth defects. See Genetic disorders
See Stroke

degrees of, 12

DIG and, 121


Birth trauma, 162-163
Cerebrospinal fluid (CSF)
Blalock-Taussig shunt, 23
CNS tumors and, 233t
hydrocephalus and, 212-213
Bleb formation, 263f
meningitis and, 149-150, 150t
Blindness, 167, 246t
syphilis and, 169
Bloom's syndrome, 230, 235
Cervix

treatment of, 12-13


Burrow's solution, 54

Cafe-au-lait spots, 105t, 118, 285

Calcinosis cutis, 133


Calcium channel blockers, 4t, 36t

Blueberry muffin spots, 168t, 169, 277


incompetent, 164t
Blue dot sign, 200
Calcium gluconate, 4t, 72, 279
inflammation of, 75
Campylobacter jejuni, 91, 151
Blue sclerae, 255, 256t
Chalazion, 247
Bochdalek hernia, 193
Chancre, 154-155

Candidiasis, 128, 157, 172-173


Caput succedaneum, 163

Boils, 50
Charcoal, activated, 8-10, 9t
Bone marrow function, 110, 115, 117
CHARGE syndrome, 100
Bone marrow transplant, 117-118, 127,
Chediak-Higashi syndrome, 105t
235
Chest wall abnormalities, 266
Bone tumors, 230t, 240-242, 251t, 254,
Cheyne-Stokes respirations, 213, 218
276
CHF. See Congestive heart failure (CHF)
Bordetella pertussis, 146
Chickenpox. See Varicella
Borrelia burgdorferi. See Lyme disease
Child abuse, 13-15, 254-255. See also
Botulinum toxin, 84, 214, 222
Sexual abuse

Carbamazepine, 116, 217t


Carbuncles, 50
Cardiac tamponade, 2t
Cardiogenic shock, 5, 7t
Cardiology, 16-42, 19f-42f
Cardiomyopathy, 7t, 34, 36t, 278
Cardiopulmonary resuscitation (CPR),
1-5, 2f-6f, 2t

Bowel obstruction
body marks with, 13f, 14f, 195
abdominal pain from, 75-76
burns and, 12-14, 13f

algorithm for, 3-5, 6f


drugs for, 4t
near-drownings and, 11

cardiopulmonary arrest from, 2t


failure to thrive and, 226-227
constipation and, 84
head trauma and, 216
diarrhea with, 82
risks for, 13

Car seats, 10

short stature and, 61


meconium and, 180, 263t, 280

Cat-scratch disease, 137-138, 148, 236

vomiting with, 78, 79t


Child neglect, 13, 226-227
Brachial plexus palsy, 163
Childproofing, 10

Cataracts, 101, 169, 244

C5 deficiency, 54
Cefixime, 156,201

Bradyarrhythmias, 35-37, 37f-39f.


Chlamydial infections, 14, 146, 147t,
See also Arrhythmias
156, 245-246, 246t

Cefotaxime, 150, 174, 201


Ceftriaxone, 150, 156, 246t, 273

Bradycardia, 4t, 186, 213, 218, 279, 284


conjunctivitis from, 174
Brain hemorrhages, 217-219, 218f, 218t Cefuroxime, 161,248,257
pneumonia from, 174-175, 286
Celiac disease, 61-62, 227t
Chloramphenicol, 117
Brain tumors, 61, 65, 79t, 243. See also
specific types, e.g., Pinealoma
Chloroma, 230

Cell-mediated immunity, 126-128, 126t


Cellulitis, 49-51, 54, 247-248

Breastfeeding, 225-226, 281


Choanal atresia, 17t, 264-265
anemia and, 184-185
Cholecystectomy, 103
jaundice and, 179-180
Cholecystitis
vegetarianism and, 116
GER and, 81
vitamin D and, 284
sickle cell anemia and, 113t
vitamin K and, 124
spherocytosis and, 112

Central nervous system (CNS). See also


Head trauma; Spinal cord

disorders of, 59, 79t, 84, 116,


212-224, 251t
cardiopulmonary arrest with,
l,2t

Breathing assessment, 1-2


Choledocholithiasis, 180
Bronchiectasis, 263t

hypertension and, 207t

--------------------------------------- 212
Chapter 3 / Cardiology

29
common atrioventricu

lar canal are the same as those


described for an ost
ium primum ASD.
Treatment
Surgical repair for
complete common atrioventricu95%
lar canal is usually
done within the first year of life.
Prior to surgical re
pair, congestive heart failure is
treated with digoxin
, diuretics, and an ACE inhibitor.
Complete heart block
occurs in 5% of patients
undergoing repair, a
nd residual mitral insufficiency is
often seen.
Patent Ductus Arteri
osus
Patency of the ductu
s arteriosus accounts for 10% of
congenital heart dis
ease. There is a high incidence in
premature neonates a
nd a 2:1 female predominance.
The ductus arteriosu
s connects the aorta and the left
pulmonary artery jus
t distal to the takeoff of the
left subclavian arte
ry from the aorta. The direction
of flow through a la
rge patent ductus arteriosus
depends on the relat
ive resistances in the pulmonary
Figure 3-11 Complete common atrioventricular canal.Typicaland systemic circuits.
In the non-restrictive patent
anatomic and hemodynamic findings include: (a) large atrial ductus arteriosus, a
s long as the systemic vascular
and ventricular septal defects of the endocardial cushion type;resistance is gre
ater than the pulmonary vascular
(b) single, atrioventricular valve; (c) pulmonary artery hypertension (due to large ventricular septal defect); (d) bidirectionalresistance, a
left-to-right shunt is present. If pulshunting (with mild hypoxemia) at atrial and ventricular levelmonary vascular re
sistance rises above systemic
vascular resistance,

a right-to-left shunt develops.


when pulmonary vascular resistance is elevated in the initial
neonatal period. With subsequent fall in pulmonary vascular Clinical Manifestati
ons
resistance, the shunt becomes predominantly left-to-right with
symptoms of congestive heart failure.
Symptoms are related
to the size of the defect and the
Cloherty JP, Stark AR. Manual of Neonatal Care, 4th ed. Philadelphia:direction o
f flow. A small patent ductus arteriosus
Lippincott-Raven, 1998:426.
causes no symptoms.
A large one with a left-to-right
shunt may result in
congestive heart failure, slowed
growth, and repeated
lower respiratory tract
CAW and the top of the muscular ventricular
infections. Reversal
of flow as a result of high
septum. In this defect, there is a left-to-right shunt at pulmonary vascular r
esistance causes shortness of
the atrial (ostium primum ASD) and ventricular level
breath, dyspnea on e
xertion, and cyanosis. In a large
(inlet VSD). Because of the increase in pulmonary
shunt, bounding puls
es, representing an aortic diablood flow, pulmonary hypertension and pulmonary
stolic runoff, are p
alpable. The murmur, often referred
vascular disease may develop over time.
to as a "machinery m
urmur," is continuous beginning
after S]; peaks at S
2, and trails off during diastole. The
Clinical Manifestations
chest radiograph of
a large patent ductus arteriosus
In complete common atrioventricular canal, congeswill show cardiomega
ly, increased pulmonary vascutive heart failure is seen early in infancy, with tachyplarity- , and left atr
ial and left ventricular enlargement.
nea, dyspnea, and poor feeding. On examinationTh,
e neonate with a
small patent ductus arteriosus has
a blowing holosystolic murmur is heard at the left
a normal ECG, wherea
s the neonate with a large
lower sternal border due to the VSD and some
patent ductus arteri
osus and a generous left-to-right
degree of common atrioventricular valve regurgitashunt shows left or
biventricular hypertrophy. Right
tion, and an S 2 with a widely fixed split is heard duventriculae
r hypertr
ophy predominates on ECG in
to the atrial septal defect. The ECG reveals left-axis
the presence of incr
eased pulmonary vascular resisdeviation, right atrial dilation, and left atrial dilation. tance. The patent du
ctus arteriosus is best seen on
The clinical manifestations of the incomplete
echocardiogram using
Doppler flow mapping.

--------------------------------------- 213
290

ints Pediatrics

Cholera, 151, 278-279


Costochondritis, 81
Cholestasis, 180
Co-trimoxazole, 201
Chondrodysplasia punctata, 94t
Coumadin. See Warfarin
Chordee, 199
Coxsackie virus
Chorioamnionitis, 149, 157, 164t
herpangina from, 140
Chorionic villus, 96, 165
hyperbilirubinemia and, 180
Chorioretinitis, 169, 171
myocarditis from, 33

Collier's sign, 233t


Color blindness, 96t
Coma, 5, 9t, 218, 219t
Combined immunodeficiency
syndromes, 127
Common variable immunodeficiency,
128
Complement disorders, 126t, 128, 133

CPR. See Cardiopulmonary resuscitation


Chromosomal disorders, 96-100, 97t,
181
Cradle cap, 195
cancer and, 239 240
aniopharyngioma, 60, 68, 233t
cleft palate and, 100, 194
Craniosynostosis, 224, 285

Concussion, 217
Cl esterase inhibitor deficiency, 130

Congenital infections, 167, 168t


retinoblastoma and, 244
Congestive heart failure (CHF), 16
Crigler-Najjar syndrome, 180
seizures and, 187
anemia with, 184
Crohn's disease. See also Inflammatory
Chromosome 22 deletion syndromes,
cardiopulmonary arrest from, 2t
bowel disease
19,26, 100, 127,278
coarctation of aorta and, 30
abdominal pain from, 75-76
Chronic granulomatous disease (CGD),
diagnosis of, 90-91, 283
127
GI bleeding from, 88, 90
Circumcision, 199-200
treatment of 91, 285

common atrioventricular canal and,


29f
Ebstein's anomaly and, 24
nephrotic syndrome and, 202t

Cromolyn sodium, 260t, 262, 276,


Cirrhosis, 121, 263t
Clavicle fracture, 163
283-284

pulmonic stenosis and, 31


renal failure and, 209

Croup, 143f
Cleft lip/palate, 93, 100, 194, 283
Diamond-Blackfan syndrome and,
Ctyptococcus neoformans, 148
105t, 117
Cryptorchidism, 97t, 199-200, 239
failure to thrive and, 227t
Currant jelly stools, 77, 82
otitis media and, 139

sickle cell disease and, 114


signs of, 103, 276
treatment of, 28

Conjunctivitis, 246-247, 247t

Cr

chlamydial, 174
Cushing's disease, 61-62, 67
pneumonia and, 147
Cushing's syndrome, 51-52, 58, 67
teratogens and, 94t
trisomy 13 and, 97t, 282
Cushing's triad, 213, 218
Clindamycin, 50
Cutis laxa syndrome, 94t

HSV, 171
Kawasaki's disease and, 135t
neonatal, 245-246, 246t, 284

pneumonia with, 147


Clinical Adaptive Test (CAT), 43
Cyanosis, 16-26, 17t, 19f-27f, 32t,
Clinical Linguistic and Auditory
185-186
Milestone Scale (CLAMS), 43
Cyclophosphamide, 120, 134

Conner's Parent and Teacher Scale, 44


Constipation, 76, 84-85

Constitutional delay, 60, 66


Clonidine, 45
Cyclosporin, 91, 117, 207t
Clostridhtm botulinum, 222
Cystic fibrosis, 262-264, 263f, 263t,

Contrast media allergy, 54

Contusion, 217
280
Clostridium difficile, 82, 84, 91, 151 Copper, 106, 111
bronchiolitis and, 145
Clostridium perfringens, 79t
Clotrimazole, 511
constipation and, 84-85

Corneal problems, 55, 168t, 171, 246t


Coronary artery disease, 33, 134,

dehydration and, 69
Club foot, 93, 166, 250-251, 278
diagnosis of, 262-263
Glutton joints, 169
diarrhea and, 82

283-284

Cor pulmonale, 113t, 263t, 264


failure to thrive and, 227t
CMV. See Cytomegalovirus
gastroenteritis and, 151
Coagulopathies, 14, 87-88, 118-124,

Corpus callosum agenesis, 97t


Corticosteroids. See also Steroids

219
hyperbilirubinemia and, 180
acne and, 52
inheritance of, 951
Cocaine. See also Substance abuse
pneumonia with, 147
arrhythmias from, 36t
rhinitis and, 129
maternal use of, 196
short stature and, 61
necrotizing enterocolitis and, 182
poisoning from, 9t
sinusitis and, 140
treatment of, 263-264

for asthma, 260t, 261-262, 283,


284
bronchiolitis and, 145
for eczema, 54, 130
hypertension and, 207t

prematurity and, 164t


vitamin K deficiency in, 123
Colic, 76-78, 226, 281
Cystitis, 200-201
Colitis, 82, 84, 87, 226

for IBD, 91
for myasthenia gravis, 221
production of, 64f, 65

pseudotumor cerebri and, 220


Cytomegalovirus (CMV), 119, 137,
Collagen vascular diseases
for seizures, 215
147t
arrhythmias with, 36t
hepatitis and, 152
constipation with, 84
for Wegener's granulomatosis, 134
mononucleosis and, 141-142
hemostasis and, 118
Corticotropin. See Adrenocorticotropic
neonatal, 168t, 170-171, 278-279,
hormone
leukemia and, 230
285

--------------------------------------- 214
Index
Dactylitis, 113t, 286
short stature with, 60
Danazol, 52
strabismus with, 243
Dane particle, 153
Doxy cy dine

291
Diarrhea, 69, 76, 82-84, 150-151
bloody, 76, 82-83, 90
complications of, 82
definition of, 82

Deafness. See Hearing loss


for acne, 52
diagnosis of, 82, 83t
for PID, 156
Deferoxamine, 9t
for Rocky Mountain spotted fever,
Dehydration, 69-71
160
cardiopulmonary arrest from, 2t
for syphilis,
diagnosis of,
Driver education,
diarrhea and,

155
70
10
82

estimation of, 70t


Drowning, 11
Drug abuse. See Substance abuse
hospitalization for, 84
treatment of, 70-71, 285
Drug overdoses, 2t, 8
vomiting and, 78

hepatitis and, 152


HIV disease and, 172
infectious, 87t

malabsorption and, 226


treatment of, 82-84
withdrawal syndrome and, 197

Diazepam, 188
Diazoxide, 191, 208

Dubin-Johnson syndrome, 181


2,7-Dichlorofluorescin (DCF) assay, 128
Dementia, 105t, 116
Duchenne muscular dystrophy. See
Denver II developmental test, 43
Muscular dystrophy

Didanosine (ddl), 158

Diepoxybutane, 118
Depression, 14, 75
Duodenal atresia, 78, 81, 97, 192-193,
Dermatitis. See also Rash
Diet, 116,216,225-228
283
Diethylstilbestrol, 94t, 164t
atopic, 53-54, 118, 129-130
Duodenal ulcer, 263t
contact, 54
Dysautonomia, 207t
seborrheic, 53-54, 195

DiGeorge's syndrome. See Chromosome

Dyskeratosis congenita, 105t


violaceous, 133
Dysmenorrhea, 75
Dermatology, 48-56

Digibind, 41f

Dysrhythmias. See Arrhythmias


Dermatomes, 49

Digoxin

22 deletion syndromes

Digitalis, 36t, 41f

Dermatomyositis, 133-134

antidote for, 4If

arrhythmias with, 36t


Ebstein's anomaly, 18, 24-25, 25f

for Graves' disease, 63


intoxication of, 79t

arrhythmia with, 36t


diagnosis of, 133
diagnosis of, 17t, 277
treatment of, 133-134
tachycardia with, 37-38

for tachycardia, 39-41, 40f, 4If


Dimercaprol, 10, 115

Desmopressin acetate (DDAVP), 122,


teratogens and, 94t
211
treatment of, 25
Developmental delay, 43, 150, 211

Diphenhydramine, 130

ECG. See Electrocardiography


Developmental dysplasia of hip (DDH),
Echo virus, 33
93, 249, 250f, 25It

Diplopia, 213, 221, 234

Diphtheria vaccine, 137t

Disequilibrium syndrome, 210


Eclampsia. See Preeclampsia
Developmental milestones, 43, 44t, 158, Disomy, 99
Eczema, 53-54, 118, 129-130, 282
284. See also Growth

Disseminated intravascular coagulation


Ehlers-Danlos syndrome, 118
Dexamethasone suppression test, 67
Ehrlichiosis, 137, 160
Dextroamphetamine, 45, 61

(DIG), 120-121
anemia with, 104t, 111, 119, 184,
185t

Eisenmenger's syndrome, 28
Dextrose, 4t
Elbow, nursemaid's, 256
Diabetes insipidus, 60, 69-70, 206

causes of, 121


HSVand, 171

Electrocardiography (ECG)
Diabetes mellitus, 57-59, 263t
for aortic stenosis, 31, 32t
dehydration and, 69
failure to thrive and, 227t
for atrial septal defect, 27, 32t

hyperbilirubinemia and, 180

leukemia and, 230


neonatal sepsis and, 173

for atrioventricular canal defect,


gestational, 166, 179-180, 183, 283
29, 32t
obesity and, 228

thrombocytopenia with, 120

treatment of, 121


for bradyarrhythmias, 35-36, 37f
Diabetic ketoacidosis (DKA), 2t, 57, 73,
vitamin K deficiency vs., 124t
for cardiomyopathy, 34
278
Distributive shock, 5, 7t
for coarctation of aorta, 30, 32t
treatment of, 59
for cyanotic neonate, 16, 18
vomiting with, 79t
DKA. See Diabetic ketoacidosis
for hyperkalemia, 72f
Double bubble sign, 192
Dialysis, 210, 279
for myocarditis, 33-34
Down syndrome, 97, 280, 282
for patent ductus arteriosus, 29, 32t
Diamond-Blackfan syndrome
duodenal atresia and, 192
anemia with, 115, 184
for pulmonary atresia, 23
diagnosis of, 104t, 105t, 117
for pulmonic stenosis, 31, 32t
erythroblastopenia and, 110
for tachyarrhythmias, 37-42,

hyperbilirubinemia and, 181


incidence of, 97
leukemia with, 230

39f-42f
leukemia and, 230
for tetralogy of Fallot, 24, 32t
treatment of, 117

microcephaly and, 224

polycythemia with, 183


for transposition of great arteries,
Diaper rash, 51, 54, 195

polyhydramnios with, 166

20, 32t

--------------------------------------- 215
292

rints Pediatrics
for truncus arteriosus, 19
rectal bleeding from, 87t
atresia of, 192f, 283
for ventricular septal defect, 28,
vomiting with, 79t
fistula of, 147, 192f
32t
Enterovirus, 148, 149t, 152
inflammation of, 76, 78, 87t
for WPW syndrome, 39
varices of, 86, 87t, 88
Enuresis, 210-211

Electroencephalography (EEC), 1 88,


Ethosuximide, 217t
214-216, 220, 277
Evan's syndrome, 120t

Environmental medicine, 93, 94t,

221-222
Electrolyte imbalances, 71-73
Ewing's sarcoma, 230t, 240-241, 267,
apnea and, 266t
276
arrhythmias and, 36t

Ependymoma, 233t

Epidermophyton, 51
Epidural hematoma, 217-219, 218f,

Exanthems, 48-49,
diarrhea and,
Exophthalmos, 63,
renal failure

159t. See also Rash


82
233t
and, 208-210

seizures and, 1 87

218t
Epiglottitis, 143-144, 144f

shock and, 7t, 285


Factor VIII, 121-122,281
Epilepsy, 214-216. See also Seizures
Elliptocytosis, 111, 185t
Factor IX, 121-122, 124

Epinephrine, 4t, 6f, 38, 284

Failure to thrive (FIT), 226-227, 227t,


Emesis. See Vomiting

for angioedema, 130

Emphysema, 1 75
263t

for AV block, 39
Episcleritis, 90

GER and, 81
Encephalitis
giardiasis and, 151
cardiopulmonary arrest from, 2t
Hirschsprung's disease and, 85
exanthems and, 48
phagocytic disorders and, 127

Epistaxis, 120
Epstein-Barr virus (EBV)

herpes simplex, 220


renal tubular acidosis and, 204
measles and, 159t
short stature and, 61
meningitis and, 149
thalassemia and, 108
seizures and, 187

erythema multiforme and, 54


fever with, 137-138
hemolytic anemia and, 111

hepatitis from, 152


Familial short stature, 60
vomiting with, 79t

hyperbilirubinemia and, 180

Encephalocele, 212
Fanconi's anemia, 105t, 115, 117-118,

leukemia and, 230

184,230
Encephalopathy, 220

lymphoma and, 235

bilirubin, 181
Fanconi's syndrome, 73f, 204-205
causes of, 220t
Fecal impaction, 84-85
HIV disease and, 158
Fecaliths, 77
hypertensive, 207t
Feeding issues, 225-226
seizures and, 187

meningitis from, 148


mononucleosis from, 141-142,
142f
rash with, 105t
thrombocytopenia with, 119

urticaria from, 54
Femoral head necrosis, 114, 249, 251t,
shingles and, 49
252
SEE and, 132t
Erb's palsy, 163
vaccinations and, 137t
Fetal alcohol syndrome, 94t, 100,

Ergotamine, 219

195-196
Encopresis, 84-85
Fetal distress, 164t
Endocarditis, 32-33, 278

Erythema
infectiosum, 48-49, 159t, 279

fever with, 137


Fetal hydantoin syndrome, 238

migrans, 149, 160, 278

Fetal hydrops, 48
hypertension and, 207t

multiforme, 54-55, 282

Libman-Sacks, 133
Fever of unknown origin (FUO),
meningitis and, 149
136-138
nephrotic syndrome and, 202t
Fifth disease, 48-49, 159t, 279
prevention of, 33
Fitz-Hugh-Curtis syndrome, 157
treatment of, 33

nodosum, 151, 279


toxicum neonatorum, 195
Erythroblastosis fetalis, 164t, 166, 179
Erythromycin

for acne, 52
Fluid imbalances, 69-71, 70t, 208-210,

Endocrine disorders, 57-68


284-285
arrhythmias with, 36t

for conjunctivitis, 175, 246


for pneumonia, 148

CNS tumors and, 234


Flumazenil, 9t
pyloric stenosis and, 80
Fluorescein, 246
fractures from, 254

Erythropoiesis, 180f

neonatal, 189-191, 190f


Folicacid, 116-117,212,280
Escherichia coli
for anemia, 104t, 112, 113t
Endometriosis, 75
for ICB, 91
Endotracheal tubes, 2, 144

breastfeeding and, 226

Folliculitis, 50
Enteritis, 82

HUS and, 119, 151, 279

gastroenteritis and, 151

Enterocolitis. See also Necrotizing


meningitis and, 149t, 174
Fontan operation, 23, 36t
enterocolitis
osteomyelitis from, 256
Food poisoning, 79t
appendicitis vs., 77
sepsis and, 282
Foot
diagnosis of, 76
Eskimos, 95t
deformities of; 93, 166, 250-251,
GI bleeding from, 88
278
Esmolol, 40f
hematochezia with, 86
Down syndrome and, 97, 282
radiation, 91
polyarteritis nodosa of, 134

Esophagus

--------------------------------------- 216
Index
293
rocker-bottom, 971
Genital abnormalities, 64-65, 94t,
HACEK organisms, 33
swelling of, 113t, 114
98-99, 196, 199, 239
Haemophilus influenzas
Foreign body aspiration, llf, 12t, 143, Genital herpes, 155-156
B-cell deficiencies and, 125
261
Gentamicin, 174, 201
bronchiolitis from, 145
Formulas, infant, 225-226
conjunctivitis from, 246
GER. See Gastroesophageal reflux
Fractures, 25It, 254-255
endocarditis from, 33
child abuse and, 13-14, 254-255

Germ cell tumors, 233

epiglottitis from, 144


Gestational age, 165-166, 175
meningitis from, 136, 148, 149t
clavicle, 163
osteomyelitis from, 256
epiphyseal, 254, 255f
spiral, 14, 254
otitis media from, 139
types of, 254
periorbital cellulitis from, 247
pneumonia from, 147t, 286
Fragile X syndrome, 99-100

Giardiasis, 91, 151,278-279

Gilbert's syndrome, 180


Glasgow Coma Scale, 5, 218, 219t

Glaucoma, 169, 223, 245

sickle cell anemia and, 113t, 114


Free erythrocyte protoporphyrin (FEP), Glioma, 65, 233t, 285-286
vaccine for, 50, 136, 137t, 144, 148
104
Glomerulonephritis, 201-205, 202t,
Halo nevus, 56
Friedreich's ataxia, 36t, 222
280
Halothane, 52
Frog-leg view, 252f
Hamartomas, 223
Fructose intolerance, 190

Henoch-Schonlein purpura and,


134
hypertension and, 207t

Hand
Fructosemia, 181
poststreptococcal, 140-141, 202t,
absent thumb of, 118
Furuncles, 50
polyarteritis nodosa of, 134

203-204
renal failure and, 209t, 210

swelling of, 113t, 114


Gabapentin, 217t
triphalangeal thumb of, 117
Glomerulosclerosis, 201-203, 281
Gait disturbances, 251
Glossitis, 105t
Hand-foot-and-mouth disease, 49, 140,
Galactosemia, 95t, 101, 181, 190, 226,
279
Glucagon, 191
275, 286
Hashimoto's thyroiditis, 63
Galeazzi sign, 249
Hay fever, 128-129

Glucocorticoids, 58, 68, 191. See also

Steroids
Gallstones, 112, 113t, 286
Headaches, 79t, 219-220
Gamma interferon therapy, 128
Head trauma, 216-219
Ganciclovir, 170

Glucose-6-phosphate dehydrogenase
(G6PD) deficiency, 114-115
anemia with, 104t, 111, 113t, 184,

bike helmets and, 10, 216


185t
cardiopulmonary arrest from, 2t
Gardnerella, 157
diabetes insipidus after, 60
Gastric lavage, 8

diagnosis of, 115

hyperbilirubinemia and, 180


hypertension and, 207t
Gastroenteritis, 79t, 82, 150-151
precocious puberty and, 65
abdominal pain from, 75
shock from, 7t
diagnosis of, 76, 78, 150-151
failure to thrive and, 227t
Hearing loss

inheritance of, 96t


treatment of, 115

Glycogen storage disease (GSD), 36t,


101, 190

CMVand, 170
treatment of, 151
CNS tumors and, 233t
Gastroenterology, 75 91
meningitis and, 150

Glycogen synthetase deficiency, 190


Goiter, 94t, 189

Gastroesophageal reflux (GER), 81-82


neurofibromatosis and, 223
apnea and, 186, 266t
osteopetrosis and, 105t
complications of, 81
otitis media and, 139
cystic fibrosis and, 214
rubella and, 169
diagnosis of, 81
syphilis and, 154, 169
failure to thrive and, 227t
teratogens and, 94t

Gonadotropin, 65-66
Gonorrhea, 14, 156, 245-246, 257,
284
Goodpasture's syndrome, 266

Gottron's papules, 133


pneumonia with, 147
Heart block, 35-37, 36t, 39f, 279
treatment of, 76, 81-82
Heart disease
vomiting vs., 78, 79t

Gower's sign, 96t, 221

Graves' disease, 62-63, 189

Griseofulvin, 5It
acquired, 32-33
Gastrointestinal (GI) disorders, 75-91
congenital, 93, 277
bleeding with, 86-89, 87t, 89f
Growth. See also Developmental
acyanotic, 26-32, 29f-31f
neonatal, 179-182, 180f, 181f
milestones
arrhythmias and, 36t
intrauterine, 165-166
Gastroschisis, 166, 194, 283
cardiopulmonary arrest from, 2t
Gaucher's disease, 95t, 102, 120t, 181,
cyanotic, 16-26, 19f-27f

short stature and, 60-61


sickle cell anemia and, 113t

diagnosis of, 16-18, 17t


222
Genetic disorders, 93-102
shock from, 7t

Growth hormone (GH), 58, 61,

190-191
teratogens and, 94t
cystic fibrosis and, 262-264
Guillain-Barre syndrome (GBS), 49,
tracheoesophageal fistula and,
hyperbilirubinemia and, 181
seizures and, 187
191
thalassemia and, 108t
trisomies and, 97t

221, 284
Gummas, 155

--------------------------------------- 217
294

Blueprints Pediatrics

functional, 33-34
Human herpesvirus 6 (HHV-6), 48,
hypoxia test for, 17t
159t
incidence of, 16

vaccines for, 123, 137t, 153


vomiting with, 79t

Hernia
Hunter's syndrome, 102
ischemic, 7t
Huntington's disease, 95t
rheumatic, 32, 36t, 38
Hurler's syndrome, 102
Heart transplant, 7, 26, 34
HUS. See Hemolytic-uremic syndrome
Heimlich maneuver, 12
Hutchinson's syndrome, 238
Helmets, bike, 10,216

Bochdalek, 193
diaphragmatic, 178, 193, 266
incarcerated, 75, 78

inguinal, 168t, 200


Heroin, 196-197

Hemangioma, giant, 111, 120t


Hutchinson teeth, 169

Herpangina, 140

Hyaline membrane disease. See


Hematochezia, 86-87, 103, 280
Respiratory distress syndrome
Hematocrit, 16, 183

Herpes simplex virus (HSV)


eczema and, 54
encephalitis and, 220

Hydralazine, 208
Hematology, 103-124, 183-185, 185t
Hydrocarbon poisoning, 9t, 277
Hematuria
Hydroceles, 200
anemia and, 103
familial, 203
Hydrocephalus, 212-213

genital infection of, 155-156


meningitis and, 149t

neonatal, 168t, 171, 245-246, 278

stomatitis from, 140


Arnold-Chiari malformation and,
glomerulonephritis and, 203-204
212
Hemoglobin, 107-109, 108t, 159t

Herpes zoster virus, 49, 75, 172, 279

Hemolytic-uremic syndrome (HUS),


cardiopulmonary arrest from, 2t

Heterotaxy, 17t
Hip dysplasia, 93, 249, 250f, 25It

CNS tumors and, 233t, 234


104t, 111, 119, 120t
intraventricular hemorrhage and,
Escherichia coli and, 119, 151,
187
279
meningitis and, 150
hematochezia with, 86, 87t
precocious puberty and, 65
IBD vs., 91

Hirschsprung's disease

constipation with, 84-85


diagnosis of, 85-86
Down syndrome and, 97
hyperbilirubinemia and, 180

strabismus with, 243


nephrotic syndrome and, 202t
toxoplasmosis and, 168t
signs of, 105t
treatment of, 213
Hemophilia, 96t, 121-123, 158, 251t
Hydronephrosis, 75, 198, 238-239.
case study on, 271, 281

neuroblastoma and, 238

rectal bleeding from, 87t


treatment of, 86

vomiting with, 78
See also Kidney disease
treatment of, 121-122
Hispanics
Hydrops fetalis, 37, 164t, 166, 184, 283
von Willebrand's disease vs., 122t
Hemoptysis, 263t, 264
Hydroxylase deficiencies, 64-65

diabetes among, 59
SLE among, 132

Hydroxyurea, 114, 286


Hemorrhoids, 105t
Hydroxyzine, 54
Hemosiderosis, 266

Histiocytosis X, 54

Hemostasis, 118. See also


Hyperactivity. See Attention deficit

Histoplasmosis, 236
HIV disease, 127, 136, 157-158, 167,

hyperactivity disorder
Coagulopathies
Hyperbilirubinemia, 214
Hemothorax, 17t, 266
Henoch-Schonlein purpura (HSP), 134,
cephalohematoma and, 162
neonatal, 179-181, 180f, 182t
202t
polycythemia and, 183
glomerulonephritis and, 203
Hypercalcemia, 84, 207t

172, 278
Addison's disease and, 67
breastfeeding and, 226

diagnosis of, 158, 286


failure to thrive and, 227t

hematochezia with, 86

fever with, 137

IBD vs., 91
Hypercholesterolemia, 33
intussusception with, 77
Hypercortisolism, 61
rectal bleeding from, 87t
Hyperinsulinism, 189, 191
renal failure and, 209t
Hyperkalemia, 72
signs of, 75, 118
arrhythmias with, 36t
Heparin, 121
cardiopulmonary arrest from, 2t

hemophilia and, 122


lymphoma and, 235, 236
risks for, 158
sexual abuse and, 14
short stature and, 61
thrombocytopenia with, 119,120t
treatment of, 158

diagnosis of, 72, 205f


Hepatic failure, 79t, 101, 280
Hepatitis, 152-154, 279
drugs for, 4t
abdominal pain from, 75
ECG of, 72f
renal failure and, 209-210
aplastic anemia from, 117
renal tubular acidosis and, 204
chickenpox and, 49
treatment of, 72
course of, 153t
diagnosis of, 152-153
Hypermagnesemia, 4t
hyperbilirubinemia and, 180
Hypermethioninemia, 181

Hives. See Urticaria


HLHS. See Hypoplastic left heart

syndrome
Hodgkin's disease, 230t, 236-237
Holliday-Seger formula, 69

Homocystinuria, 100-101
Hordeolum, 247

IBD and, 90
Hypernatremia, 5, 72, 285
risks for, 152
Hyperparathyroidism, 207t
treatment of, 153-154

Horner's syndrome, 231, 238

types of; 152f


Hyperpigmented lesions, 55-56

HSV. See Herpes simplex virus

Howell-Jolly bodies, 116

--------------------------------------- 218
Index
295
Hypertension, 206-208, 207t
Islet cells, 57-58, 189
Cushing's triad and, 213, 218
Isoniazid, 52
cystic fibrosis and, 2631
Isoproterenol, 38, 39
Hypertensive crisis, 208

sinusitis and, 140


vaccinations and, 137t
Immunology, 125-128, 126t

Impetigo, 13, 50

Isotretinoin, 94t
Hyperthermia, 5, 207t

Imprinting disorders, 99

Hyperthyroidism, 62-64. See also


Inborn errors of metabolism.
Jacksonian seizures, 215
Thyroid disorders
Janeway lesions, 33
Hypertrophic subaortic stenosis, 34
Jaundice, 179-181, 180f
Hyperviscosity syndrome. See
anemia and, 103, 105t, 184
Polycythemia
cystic fibrosis and, 263t

See Metabolic disorders


Increased intracranial pressure, 207t

arrhythmias from, 36t


CNS tumors and, 233t
head trauma and, 218

Hypocalcemia
hepatitis and, 152
arrhythmias with, 36t
sickle cell disease and, 113
cardiopulmonary arrest from, 2t

meningitis and, 149


vomiting and, 78, 79t

Jones criteria for rheumatic fever, 14It


DiGeorge's syndrome and, 19
Indians. See American Indians
Juvenile rheumatoid arthritis (JRA),
drugs for, 4t
Indomethacin, 30
131-132,284
Hypogammaglobulinemia, 118, 125-126
Infectious diseases, 136-161. See also
anemia with, 109
Hypoglycemia
specific types, e.g., Staphylococcal
diagnosis of, 131-132, 131t
cardiopulmonary arrest from, 2t
infections
fever with, 137
definition of, 189
Inflammatory bowel disease (IBD), 86,
treatment of, 132
glycogen storage diseases and, 101
87t, 90-92, 280
growth hormone deficiency and, 61
Kallmann's syndrome, 66
neonatal, 189-191

anemia with, 91, 109

complications of, 90
Kasabach-Merritt syndrome, 111
seizures with, 187
Kawasaki's disease, 32, 134-135
treatment of, 4t, 59, 191
abdominal pain from, 75
Hypogonadism, 66
arrhythmias with, 36t
Hypokalemia, 72-73, 204, 205f
complications of, 135-136,
constipation and, 84
283-284

diagnosis of, 90-91, 283


fever with, 137-138

gastroenteritis vs., 151


PIDvs., 156
short stature and, 61-62
treatment of, 91, 285

evaluation of, 73f


diagnosis of, 134, 135t, 277, 278
Hypomagnesemia, 36t
Insulin-dependent diabetes mellitus
fever with, 137
0DDM], 57-59
Hyponatremia, 71, 263t
Kayser-Fleischer rings, 105t
cardiopulmonary arrest from, 2t
Keratitis, 55, 168t, 171
desmopressin acetate and, 122

diagnosis of, 58
pathogenesis of, 57
prognosis for, 59

Kernicterus, 179, 181, 214


Hypoplastic left heart syndrome
Kernig's sign, 149
(HLHS), 17t, 25-26, 25f, 26f, 94t

risks for, 57-58

treatment of, 58-59


Ketogenic diet, 216
Hypospadias, 199, 239
Kidney
Hypothermia, 2t, 5

Insulin pump, 58

Interferon, 153-154
agenesis of, 105t, 118, 283
Hypothyroidism, 61-62, 189. See also
biopsy of, 209
Thyroid disorders
cancer of, 223, 238, 240

Intestinal obstruction. See Bowel

obstruction
Hypovolemic shock, 5, 7t, 113
dysplasia of, 198
Hypoxia test, 16-18, 17t
horseshoe, 97t, 118, 239

Intrathoracic pressure, 259, 260f

Intrauterine growth retardation,


stones in, 75, 90, 203-204
Hypsarrhythmia, 215, 284

165-166

transplant of, 203-204, 210


Intraventricular hemorrhage (IVH),
Kidney disease, 75, 198-211, 238. See
IBD. See Inflammatory bowel disease
also Renal failure
Ichthyosis vulgaris, 54
anemia and, 110
Ideal body weight (IBW), 225
failure to thrive and, 227t
Idiopathic thrombocytopenic purpura
hypertension and, 207t

186-188
Intubation, 2, 4t
Intussusception, 75, 77-78, 82, 88, 283

Inuit, 95t
(ITP), 119-120, 120t, 230
neonatal sepsis and, 173
Ileus. See Bowel obstruction
polycystic, 95t, 97t, 198, 206,

Ipecac syrup, 8, 277

Iron deficiency anemia, 106-107, 107t,

238-240
Immunodeficiencies, 125-130, 126t. See
small for gestational age and, 165
also specific types, e.g., HIV disease

270, 280
pica and, 103

bronchiolitis and, 145 146


eihauer-Betke stain, 184

Kl
signs of, 105t

failure to thrive and, 227t


Klein's line, 252
leukemia and, 230
Klinefelter's syndrome, 66, 98-99,
lymphoma and, 235
282
otitis media and, 139

thalassemia vs., 109


treatment of, 106-107

Iron overload, 9t, 109


Irritable bowel syndrome, 76

Kobner phenomenon, 52
pneumonia with, 147

--------------------------------------- 219
296

Blueprints Pediatrics

Koplik's spots, 48, 159t, 279


Measles, 48, 159t
Krabbe's disease, 102
atypical, 160

anemia with, 104t, 116


hyperbilirubinemia and, 181
vitamin K deficiency vs., 124t

thrombocytopenia with, 119


Kussmaul respirations, 58
treatment of, 49
Kyphosis, 253-254
vaccine for, 48, 137t

Lorazepam, 188
Lordosis, 254

Labyrinthitis, 79t
Meckel's diverticulum
diagnosis of, 90
Lactoferrin, 225-226
GI bleeding and, 87t, 89-90
Lamotrigine, 217t
hematochezia with, 86

Low birth weight (LEW)

anemia and, 184


failure to thrive and, 226, 227t
hyperbilirubinemia and, 181,

Large for gestational age, 166


incidence of, 89
Laryngotracheomalacia, 1 7t, 264
vomiting with, 78
Meconium
Lead poisoning, 79t, 277
aspiration of, 17t, 177-178
anemia and, 104t, 105t, 106

182t
intraventricular hemorrhage and,

186
meningitis and, 149

constipation with, 84-85


necrotizing enterocolitis and, 182
bowel obstruction from, 180, 263t,
laboratory findings in, 107t
prematurity and, 163-164
280
pica and, 103
small for gestational age and, 165
Hirschsprung's disease and, 86
prevention of, 10
Lucey-Driscol syndrome, 180
Medulloblastoma, 233t, 234
signs of, 105t
Lumbar puncture
Megacolon, 84, 85, 90-92
Learning disabilities, 43-46, 196, 214,
hydrocephalus and, 213
Megaloblastosis, 115
216, 231
for meningitis, 149
Melanin, 101
Legg-Calve-Perthes disease, 251-253,
for pseudotumor cerebri, 220
Melanoma, 55-56
276
for syphilis, 155
Melena, 86-88, 103
Leiner's disease, 54
Lung diseases, 259-266. See also specifi
c
types, e.g., Asthma
Menarche, 47f
Lennox-Gastaut syndrome, 215, 217t
Meningitis, 148-150
Leptospirosis, 137
Lung transplant, 264
age and, 148, 149t
Lyme disease
Lesch-Nyhan syndrome, 96t, 115
apnea and, 186, 266t
Leukemia, 229-232, 281
bacteremia and, 138, 173
congenital, 184
cardiopulmonary arrest from, 2t
diagnosis of, 230-231
diagnosis of, 149, 150t, 161

arthritis from, 132, 25It

diagnosis of, 160-161, 278


fever with, 137
heart block from, 35, 37

Diamond-Blackfan syndrome and,


diarrhea and, 83
117
mortality with, 150
Down syndrome and, 97
neonatal sepsis and, 1 73
Fanconi's anemia and, 118
otitis media and, 139
fever with, 137
risks for, 148-149
incidence of, 120t, 229
seizures and, 187
limp with, 25It
shingles and, 49
prognosis for, 23 2t
treatment of, 149-150, 174
risks for, 230
vomiting with, 79t
thrombocytopenia with, 120t
Meningocele, 212
treatment of] 231-232

meningitis with, 148, 149t


treatment of, 161

Lymphocytic interstitial pneumonitis


(LIP), 158
Lymphoma, 238, 240
Burkitt's, 231, 234
fever with, 137
HIV disease and, 158
Lysosomal storage disorders, 102

Meningoencephalitis, 49, 167, 169


Leukocoria, 244-245
Meningomyelocele, 84, 166
Leukodystrophies, 222

Macrocephaly, 224, 234


Malaria, 111, 235

Menstruation
Leukoplakia, 105t
Mallory-Weiss tear, 78
abdominal pain from, 75
Leukotriene receptor antagonist, 260t, Malnutrition
acne and, 52
261-262, 284
anemia and, 103, 105t
anemia and, 103
Levetiracetam, 217t
diarrhea and, 82
Mental disorders, 75, 226, 227t, 233t
Leydig cell tumor, 65
failure to thrive and, 227
Mental retardation
short stature from, 60
Angelman's syndrome and, 99
Libman-Sacks endocarditis, 133
Malrotation of intestines, 78, 80-81, 19
2
child abuse and, 13
Lidocaine, 4t, 6f, 4If
Limp, 251-252, 25It
cystic fibrosis and, 214

Mannosidosis, 102

Maple syrup urine disease, 190


Down syndrome and, 97
Lipid metabolism disorders, 181, 219
failure to thrive and, 227t
Listeria monocytogenes, 147t, 149t, 282,Marfan's syndrome, 95t, 101-102
fetal alcohol syndrome and, 196
286
Mastoiditis, 139
fragile X syndrome and, 99-100
McArdle's disease, 101
homocystinuria and, 101-102
Lithium, 10, 52, 94t, 206
Liver cancer, 180
intraventricular hemorrhage and,

McCune-Albright syndrome, 65
Mean corpuscular volume (MCV), 104,

187
Liver disease, 79t, 101, 280
PKU and, 101

280

--------------------------------------- 220
Index
297
Prader-Willi syndrome and, 99
Neisseria meningitidis, 148-149, 149t
rubella and, 169
complement disorders and, 128
syphilis and, 154
sepsis from, 14

Mobitz type block, 35-36, 37f


Molecular cytogenic disorders, 99-100

Moles. See Nevi


tuberous sclerosis and, 223

Mongolian spots, 13, 195

sickle cell anemia and, 113t


6-Mercaptopurine, 91
Nephritic syndromes, 202t
Mononucleosis, 141-142
Metabolic acidosis
Nephritis, 132, 209
bicarbonate for, 4t

abdominal pain from, 75

cardiopulmonary arrest from, 2t


Nephroblastoma, 240

Hodgkin's disease and, 236


strep throat vs., 140

Nephrolithiasis, 75, 90, 203-204


cystic fibrosis and, 263t
Nephropathy, 198-211. See also Kidney
dehydration and, 70
diabetes and, 57
disease
diabetic, 59
diagnosis of, 73
IgA, 203
hyperchloremic, 204-205, 205f
potassium imbalance and, 72-73
sickle cell anemia and, 113t
renal failure and, 209-210
Nephrotic syndrome, 201-203, 202t,

Moraxella catarrhalis, 139

Moro reflex, 163

Morphine, 24, 197


Motor vehicle accidents, 10-11

Movement disorders, 216


Mumps, 159t

shock from, 7
207t, 280
treatment of, 73
Neural tube defects, 93, 94t, 212,

vaccine for, 137t

Muscle weakness, 221-222


280
Metabolic alkalosis, 73-74
dehydration and, 70
constipation and, 84
hypochloremic, hypokalemic, 78,
meningitis and, 149
81
polyhydramnios and, 166, 283

Muscular dystrophy, 221, 251t


arrhythmias with, 36t
cardiomyopathy with, 34
inheritance of, 96t

Metabolic disorders, 100-102


Neuroblastoma, 120t, 137, 230t,
anemia with, 115
237-240,281,286
apnea and, 186, 266t
Neurofibromatosis, 95t, 207t, 223t,
cataracts and, 244
233t, 238, 285
encephalopathy and, 220

Mustard procedure, 37
Myasthenia gravis (MG), 221
Mycobacterium avium, 158

Mycobacterium tuberculosis, 147t, 148


Neuropathy, 59, 116, 212-224, 251t. See
failure to thrive and, 227t
Mycoplasma hominis, 157

also Central nervous system


fractures from, 254
Nevi
hyperbilirubinemia and, 181

Mycoplasma pneumoniae, 147t

hypoglycemia and, 189-190


acquired, 56

erythema multiforme and, 54


hemolytic anemia and, 111

congenital, 55-56
seizures and, 187-188
pigmented, 98
vomiting with, 79t

meningitis and, 148


sickle cell anemia and, 113t

port-wine, 223
Metatarsus adductus, 250, 268, 278
Nicotinamide adenine dinucleotide
Methadone, 196-197
phosphate (NADPH), 114-115

Myelomeningocele, 93, 149, 212, 283

Myocarditis, 33-34
Methemoglobinemia, 16, 17t
Niemann-Pick disease, 102, 181, 222
Methicillin, 136, 174

arrhythmias with, 36t

cardiopulmonary arrest from, 2t


Nikolsky's sign, 50, 55
Methimazole, 63
Nissen fundoplication, 81
Methylmalonic aciduria, 115, 190

heart block from, 35


measles and, 159t
tachycardia with, 37

Nitroblue tetrazolium (NET) test,


Methylphenidate, 45, 61
128
Methylprednisolone, 283
Nitrofurantoin, 115, 199
Microcephaly, 224
Non-Hodgkin's lymphoma (NHL), 230t,
anemia and, 105t, 118
234-236
CMV and, 168t
ataxia-telangiectasia and, 127
ethanol and, 941, 196
hemolytic anemia with, 111
rubella and, 169
risks for, 235
trisomies and, 97t, 282
treatment of, 235-236

Myotonic dystrophy, 95t

Naloxone, 4t, 9t, 197


Narcotics. See Opiates
Nasolacrimal duct obstruction, 245-246
Native Americans. See American Indians

Near-drowning, 11
Microphallus, 61
Nonsteroidal anti-inflammatory drugs
Microphthalmia, 169
(NSAIDs)

Neck
neuroblastoma of, 237

Microsporum, 51
for arthritis, 132

webbed, 117, 284

coagulopathies and, 123


Migraine, 79t, 220
GI bleeding from, 87t
Milia, 195
for Kawasaki's disease, 135, 277

Necrotizing enterocolitis (NEC)


constipation with, 84
neonatal, 182,271,282

Milk allergy, 75-76, 79t, 87t, 226, 266


poisoning by, 79t
Minimal change disease (MCD),
Reye's syndrome and, 49, 220, 283
201-202, 202t, 280-281
urticaria from, 54

rectal bleeding with, 87t, 280


substance abuse and, 84
Nedocromil, 260t

Minocycline, 52
Norwalk virus, 79t
Neisseria gonorrhoeae, 14, 156, 245-246,
Mittelschmerz, 75
Nursemaid's elbow, 256

257, 284

--------------------------------------- 221
298

Blueprints Pediatrics

Nutrition, 116, 216, 225-228


Peptic ulcer disease [PUD), 81, 87t

Otitis media, 138-140


ataxia and, 222

Nystagmus, 233t, 234, 244t


Periarteritis nodosa, 36t
Pericarditis
Obesity, 227-228
cardiopulmonary arrest from, 2t
astrocytoma and, 233t
constrictive, 202t
diabetes and, 59
GER and, 81
hypertension and, 208
SLE and, 132t
macrosomia and, 166
Periorbital cellulitis, 247-248
pseudotumor cerebri and, 220
restrictive lung disease and, 266
Peritonitis
sleep apnea and, 265
cardiopulmonary arrest from, 2t
dialysis and, 210
slipped capital femoral epiphysis
nephrotic syndrome and, 202
and, 252
treatment of, 77
Obsessive-compulsive disorder (OCD),
vomiting and, 78, 79t

B-cell deficiencies and, 125

breastfeeding and, 225


treatment of, 139
vomiting with, 79t
Ovaries
cancer of, 238, 240
cysts in, 66, 75, 156
streak, 98
torsion of, 75

Oxacillin, 50

Pacemakers, 37, 38f, 39f

216
Palivizumab, 145
Persistent pulmonary hypertension of
Obstipation, 84
newborn (PPHN), 17t, 18, 164,
Obturator sign, 77
177-179, 183
Oligohydramnios, 93, 166, 283
Pertussis, 137t, 146
Omphalocele, 97t, 166, 193-194, 283
Pervasive developmental disorder
Oncology, 229
(PDD), 45-46, 45t

Pancreas
annular, 192
inflammation of, 78, 79t, 81,
159t, 180, 263t
removal of, 191
Pancytopenia, 103, 117-118

Ophthalmia neonatorum, 245-246,


Pesticide poisoning, 9t
246t, 284
Peutz-Jeghers syndrome, 105t
Ophthalmology, 243-248
Peyer's patches, 77

Panophthalmitis, 55
Papilledema, 220, 234
Parainfluenza virus, 143, 145, 147t

Opiates. See also Substance abuse


Phagocytic disorders, 126t, 127-128
constipation from, 84
Phakomatoses, 223
naloxone for, 4t, 9t, 197

Paralysis, 221-222
Parental imprinting disorders, 99

for shingles, 49
Pharyngitis, 78, 79t, 142. See also
Parinaud's syndrome, 233t
Strep throat
urticaria from, 54
Phenobarbital, 188, 197, 216, 217t
withdrawal syndrome and, 196-197

Paroxysmal nocturnal hemoglobinuria


(PNH), 111

Oral contraceptives, 52, 207t


vitamin deficiencies from, 116,
Parvovirus, 48, 159t, 184
123-124
Oral rehydration therapy, 70-71,
t,Phenothiazines, 9t, 84
151
Orchitis, 159t
Phenylbutazone, 117

aplastic crisis from, 110, 112, 113


280
rash from, 105t
thrombocytopenia and, 119

Phenylketonuria (PKU), 95t, 101


Organophosphate poisoning, 9t
Phenytoin, 188, 217t
Ornithine transcarbamylase (OTC)
6
deficiency, 96t, 102
anemia from, 115

Patent ductus arteriosus, 29-30, 32t, 27


prostaglandin EI for, 18, 20, 26
rubella and, 168t, 169

for status epilepticus, 216


Orotic aciduria, 115
tachycardia and, 41f

teratogens and, 94t

Orthodromic reentrant tachycardia


as teratogen, 94t
(ORT), 37-40, 39f
vitamin deficiencies from, 116,

tracheoesophageal fistula and, 191


treatment of, 30

Orthopedics, 249-258
123-124
Ortolani's maneuver, 249, 250f
Pheochromocytoma, 58, 207t, 223
Osgood-Schlatter disease, 253, 276
Phototherapy, 181, 182t
Osier nodes, 33
Physostigmine, 9t
Osier-Weber-Rendu syndrome, 105t
Pica, 103

trisomies and, 97t


Pavlik harness, 249
Pectus carinatum, 266
Pectus excavatum, 266

Pelvic inflammatory disease (PID),


Osmotic fragility test,-112
Pickwickian syndrome, 99, 265
Osteogenesis imperfecta, 13, 254-256,
PID. See Pelvic inflammatory disease
256t
Pierre-Robin syndrome, 264

75-76, 78, 79t, 156-157, 251t


Penicillamine, 94t

Penicillin
Osteogenic sarcoma, 241-242
Pinealoma, 65, 233t, 234
Osteomyelitis, 50, 83, 113t, 241, 25It,
Pink eye, 246-247, 247t
276

for conjunctivitis, 246t


hemolytic anemia and, 111
for periorbital cellulitis, 248

Pin worms, 201


diagnosis of, 256-257
Pituitary tumor, 61, 66, 67, 68, 234
treatment of, 257

resistance to, 136

Osteopetrosis, 105t, 184


PKU. See Phenylketonuria

for strep throat, 141

sickle cell disease and, 114, 286

for syphilis, 155, 169


Placenta previa, 164t
Osteosarcoma, 230t, 241, 276
Platelet disorders, 118-124
Otitis externa, 139

Pepper's syndrome, 238

--------------------------------------- 222
Index
299
Pneumatosis intestinalis, 182, 282
Psoas sign, 77
Pneumococcal vaccine, 137t
Psoriasis, 52-54

chromosomal disorders and, 96


CMV and, 171
ectopic, 75, 156

Pneumocystis carinii pneumonia (PCP),

HIV disease and, 158

Ptosis, 221
127, 158, 172,278
Puberty, 46f-47f, 46t, 282
Pneumonia, 17t, 146-148
definition of, 46
age and, 146-147, 147t
delayed, 60-61, 66, 113t, 284

Turner's syndrome and, 98


vaccinations and, 137t
vomiting and, 78, 79t

apnea and, 186, 266t


Premature rupture of membranes
precocious, 60, 65-66, 66, 233t
B-cell deficiencies and, 125
(PROM), 164t, 182
Pulmonary atresia, 17t, 18, 22-23, 23f
chlamydial, 174-175,286
Pulmonary embolism, 7t, 81
cystic fibrosis and, 263t
Prematurity, 163-164
diagnosis of, 147-148, 286
Pulmonary fibrosis, 113t

anemia of, 184, 185t


apnea of, 185-186

Pulmonic stenosis, 17-18, 31-32,


diarrhea and, 83
168-169, 276-277
GERand, 81

bronchiolitis and, 145

measles and, 159t


Pulse oximetry, 18
risks for, 147
Pulsus alternans, 34
treatment of, 148

causes of, 164t

vomiting and, 78, 79t


Pyelonephritis, 200-201, 206, 283

developmental delays and, 43

child abuse and, 13


cryptorchidism and, 200

failure to thrive and, 227t


diarrhea and, 83
walking, 148
hypertension and, 207t
Pneumonitis, 9t
PID vs., 156
CMV, 170
renal failure and, 209t
exanthems and, 49

formula feedings for, 225


gastroschisis and, 194
GER and, 81
hyperbilirubinemia and, 179, 181,

Pyloric stenosis, 78, 80-81, 227t, 269,


lymphocytic interstitial, 266
278, 283
meconium aspiration, 17t, 177
Pneumothorax, 17t, 175, 177, 263t, 264
Pyloromyotomy, 80

182t

hypothyroidism and, 189


intraventricular hemorrhage and,

Pyoderma gangrenosum, 90
Poisoning, 8-10, 105t, 180, 277. See also
Pyridoxine deficiency, 106, 187
specific types, e.g.. Lead poisoning
acetaminophen, 9t, 270, 280
Pyrimethamine, 167
anemia with, 111

186-187
necrotizing enterocolitis and,
182
patent ductus arteriosus and, 29

Pyruvate carboxylase deficiency, 190


antidotes for, 9t
Pyruvate kinase deficiency, 111,180,
polyhydramnios and, 166, 283
suicide attempts by, 8
184, 185t
Poliomyelitis, 136, 137t, 221

RDSand, 175-176

Polyarteritis nodosa, 75, 134


QT syndrome, 36t, 38, 41f, 281

sepsis and, 173-174

retinopathy of, 176, 244-245

vaginosis and, 157


Polycythemia, 17t, 183
Quinidine, 111, 119
hyperbilirubinemia and, 180
Quinine, 119
hypoglycemia and, 189

Priapism, 113t, 114, 183


Primaquine, 115

macrosomia and, 166


Procainamide, 6f, 41f
Raccoon eyes, 218, 238
Polydactyly, 93, 97t
Radial head subluxation, 256
Polyhydramnios, 164t, 166, 191-194,

Prolonged QT interval syndromes, 36t,


38, 41f, 281

283
Rash. See also Dermatitis
Prolonged rupture of membranes, 149
allergic, 53-55
Polymyositis, 133
bacterial, 50-51
Polyps, 87t, 129, 263
butterfly, 105t, 132t

PROM. See Premature rupture of


membranes

diaper, 51, 54, 195


Pompe's disease, 36t
fungal, 51
Porphyria, 207t

Propionibacterium acnes, 51

Postmaturity syndrome, 164-166, 177,


hepatitis with, 152

Propranolol

Lyme disease, 149, 160, 278


183, 283
polymorphous, 135t
Potter's syndrome, 166, 283
Rocky Mountain spotted fever, 160,

Propionic acidemia, 190

for Graves' disease, 63


hypoglycemia and, 189
for tachycardia, 40f

PPHN. See Persistent pulmonary


278
hypertension of newborn
strep throat with, 140
Prader-Willi syndrome, 99, 224
syphilitic, 154-155, 168t

Propylthiouracil (PTU), 63, 189

Prostaglandin EI (PGE,), 18, 20, 26


Protein C, 95t, 123

viral, 48-49, 159t


Pralidoxime, 9t

RDS. See Respiratory distress syndrome


Prednisone, 134, 283
Protein S, 123
Preeclampsia, 120t, 207t
Reactive airway disease. See Asthma

Proteinuria, 202

Rectal bleeding, 77, 86, 87t, 280


nephrotic syndrome and, 202t

Prune belly syndrome, 84

prematurity with, 164t


Pseudomonas, 126t, 139, 246, 263
Rectal prolapse, 263t
small for gestational age and, 165 Pseudoseizures, 216
Red reflex, 244t
Pregnancy
Reed-Sternberg cells, 236
Pseudotumor cerebri, 220

--------------------------------------- 223
Chapter 1 / Emergency Management: Evaluation of the Crit
ically III or Injured Child
Infant Older Child
Percu
taneous peripheral IV
Airway
Determine unresponsiveness
Call for help
Position patient supine
Support head and neck
No (after 90 sec)
Head tilt/chin lift or jaw thrust
No blind finger sweeps
i
Breathing
2 initial breaths
Intraosseous needle
Then: 20 breaths/min Then: 15 breaths/min
(1st choice for < 5 y.o.)
Circulation
or
Saphenous vein cutdown
Check brachiai pulse Check carotid pulse
or
Activate EMS System
Central venous access
Compression location:
Compression location:
1 finger breadth below

lower 1/3 of sternum

Yes

intermammary line on
I
sternum
inger's 10-20 ml/kg and
Compression method:
Compression method:
urther volume administration
Hands encircle chest or
1 or 2 hands on sternum
tion drugs
2 fingers on sternum
Compression depth:
Compression depth:
access management during cardiopul0.5-1"
1-1.5"
.
Compression rate: 100/min
Compression rate:

Lactated R
titrate f
Resuscita

Figure 1-3

Vascular

monary resuscitation

80-100/min
Compression:ventilation ratio = 5:1
Reassessment: Palpate pulse every 10 cycles
, venous blood gas, electrolyte

complete blood count

and chemistry panel,


and blood glucose) is obtained
Figure 1-2 Basic CPR in infants and children.
at the time of vascula
r access. If ingestion is a possiModified from Nichols DG, Vaster M, Lappe DG, et al. Golden Hour: Thebility, ser
um and urine toxicology and an acetaHandbook of Advanced Pediatric Life Support. St. Louis: Mosby Yearbook,
minophen and salicyl
ate level may be obtained.
1991:2,128.
th tachydysrhythmias (SVT, VT)

In the patient wi
therapeutic decision

s are based on whether the


are an insensitive indicator, because hypotension is a
ically stable or unstable.
late finding in hypovolemia. Children are more likely
hycardia (SVT)
to present with asystole than with an arrhythmia,
unless they have an underlying cardiac electrical
le: Vagal maneuvers, adeno-

patient is hemodynam
Supraventricular Tac

Hemodynamically stab
sine (AV reciproc

ating tachycardia), amiodarone


abnormality. Cardiorespiratory monitors are helpful
ardia).
for specifying the electrical activity of the heart.

(automatic tachyc

Hemodynamically unst
able or SVT refractory to
If pulselessness is noted on examination of the
hronized cardioversion 0.50 to
brachiai pulse in the infant or the carotid pulse in the
d to 2J/kg if initial cardioversion
child, chest compressions should be started. Vascular

medications: Sync
l.OJ/kg; increase

is unsuccessful.

access management during cardiopulmonary resuscitation is outlined in Figure 1-3. Once access has been
dia (VT)
established, initial fluid resuscitation with lactated
le: Lidocaine, amiodarone, or
Ringer's solution or normal saline should be given as
treat hypomagnesemia and/or
a 20mL/kg bolus as quickly as possible. If necessary,
darone and procainamide
these boluses should be repeated. However, if there
d together because they both
is no response or the patient has suffered acute blood
terval and both may cause
loss, consider a lOmL/kg infusion of albumin, crystalloid, or type O-negative whole blood. If hypoable or VT refractory to
tension due to hemorrhage is suspected, gaining
hronized cardioversion 0.50 to
proximal control of the hemorrhage is critical.
d to 2J/kg if initial cardioversion
Optimally, a full set of screening tests (including

Ventricular Tachycar
Hemodynamically stab
procainamide, and
hypokalemia. Amio
should not be use
prolong the QT in
hypotension.
Hemodynamically unst
medications: Sync
l.OJ/kg; increase
is unsuccessful.

--------------------------------------- 224
30
rints Pediatrics
Treatment
a female, Turner's syndrome must
Indomethacin is often effective in closing the patent
bstruction is usually located in

the aorta occurs in


be considered. The o
the descending aorta

, at the insertion site of the


ductus arteriosus in the premature neonate by
decreasing PGE, levels. A patent ductus arteriosus
he aortic valve is bicuspid in
usually closes in the first month of life, but for
tral valve anomalies may also

ductus arteriosus. T
80% of cases, and mi
be present. The coar

ctation results in mechanical


those that do not, surgical ligation by thoracotomy
or video-assisted thoracoscopic surgery, or coil
the proximal and distal aorta
embolization by catheterization is curative.
t ventricular afterload. Congestive

obstruction between
and in increased lef
heart failure develo

ps in 10% of cases in infancy.


Coarctation of the Aorta
ons
Coarctation of the aorta (Figure 3-12) accounts for
femoral pulses are often weak
8% of congenital heart defects and has a male-toto upper extremities or are

Clinical Manifestati
On examination, the
and delayed relative
absent and there is of

ten upper extremity hyperfemale predominance of 2:1. When coarctation of


th critical coarctation have

tension. Neonates wi

ductal-dependent sys
temic blood flow and may
present with circula
tory collapse. Flow across the
coarctation may prod
uce a systolic ejection murmur
heard at the apex. O
n chest radiograph, the aortic
knob is enlarged; on
ECG, right ventricular hypertrophy is seen in th
e neonate, and left ventricular
hypertrophy is seen
in the older child. The echocardiogram is used to v
isualize the defect and to check
for abnormalities of
the aortic valve, mitral valve, and
left ventricular per
formance.
Treatment
Palliation may be ac
complished via balloon dilation
angioplasty, stent p
lacement, or by surgical end-toend anastomosis, sub
clavian flap repair, patch repair,
or graft placement.
Aortic Stenosis
In aortic stenosis (
Figure 3-13), the valvular tissue is
thickened and often
rigid. Most commonly, the valve
is bicuspid, with a
single fused commissure and an
eccentric orifice. T
he stenotic valve produces a presCoarctation of the aorta in a critically ill neonatesure gradient be
Figure 3-12
tween the left ventricle and the aorta
with a nearly closed ductus arteriosus. Typical anatomic andthat results in left
ventricular hypertrophy and,
hemodynamic findings include: (a) "juxtaductal" site of the
coarctation; (b) a bicommissural aortic valve (see in 80% ofover time, decreased
compliance and ventricular
patients with coarctation); (c) narrow pulse pressure in theperformance.
descending aorta and lower body; (d) a bidirectional shunt at
the ductus arteriosus. As in critical aortic stenosis (see Fig. 3-13)Clinical Ma
nifestations
there is an elevated left atrial pressure, pulmonary edema,a left-The neonate wi
th aortic stenosis may present with
to-right shunt at the atrial level, pulmonary artery hyperten-cardiovascular col
lapse or with a soft murmur. The
sion, and only a moderate (30-mm Hg) gradient across the archlevel of symptomato
logy is related to the severity of
obstruction. The low measured gradient (despite severe
anatomic obstruction) across the aortic arch is due to low the stenosis and the
ventricular function. The
cardiac output.
neonate with critica
l aortic stenosis has ductalCloherty JP, Stark AR. Manual of Neonatal Care, 4th ed. Philadelphia:dependent s

ystemic blood flow and may present with


Lippincott-Raven, 1998:426.
after the ductus closes. If ven-

circulatory collapse

--------------------------------------- 225
300

rirrts Pediatrics

Rehydration therapy, 70-71, 151


Selective IgA deficiency, 128
Reiter's syndrome, 53, 157, 161
Selective serotonin reuptake inhibitors
Renal cell carcinoma, 223, 238, 240
(SSRIs), 45
Renal failure. See also Kidney disease;
SEM disease, 171,278
Nephropathy
Senning procedure, 37
acute, 208-209, 209t
Sepsis, 5, 7, 79t, 138
cardiopulmonary arrest from, 2t

Rocky Mountain spotted fever, 137,


159-160
Rosacea, 52
Roseola, 48-49, 159t, 279
Ross procedure, 31
Rotavirus, 79t, 82, 151

Roth spots, 33
apnea and, 266t
chronic, 209
diarrhea and, 82
glomerulonephritis and, 203
DIG and, 121
HUS and, 119
hypoglycemia and, 189
hyperkalemia and, 72
neonatal, 173-174, 271,
hypertension and, 207t
281-282

Rotor's syndrome, 181

Rubella, 48, 159t, 184,279


congenital, 168t, 169-170,
277-278
thrombocytopenia with, 119

intrinsic, 208
nosocomial, 173
polycythemia and, 183
thrombocytopenia with, 120t
short stature and, 61
Septic shock, 2t, 5, 7t
SLE and, 132-133

treatment of, 49

vaccine for, 137t


Rubeola. See Measles

UTls and, 201


Serum sickness, 54
vomiting with, 79t
Sacroiliitis, 90
Setting sun sign, 213
Renal nuclear scans, 209
Saddle nose, 169
Severe combined immunodeficiency
Renal tubular acidosis (RTA), 204-205, Salmonellosis, 79t, 82-84, 113t, 150,
disease (SCIDJ, 127
Sexual abuse, 13-14. See also Child
205f 227t
abuse
Renal vein thrombosis, 202t

256, 278-279
Salter-Harris classification, 255f

Replogle tube, 191-193


Sexual development, 46f-47f 46t
Sandal sign, 97, 282
Sexually transmitted diseases (STDs),
Respiratory distress syndrome (RDS)
154-158,257,284
croup and, 143
neonate with, 17t, 175-176, 186
arthritis and, 257
neuroblastoma and, 238
child abuse and, 14

Sanfilippo's syndrome, 102


Sarcoidosis, 137

Sarcoma, 230t, 238-242, 267, 276

Scabies, 54
congenital, 168-169, 168t, 277,
PPHNand, 178
279
sepsis with, 173
Scarlet fever, 140-141
conjunctivitis and, 245-246, 246t
Respiratory syncytial virus (RSV), 143, Schilling test, 116
145-146, 147t, 259-260, 266t, 286
Lyme disease and, 161

Schizophrenia, 206

thrombocytopenia with, 119


Respiratory tract infections, 125, 145, Sclerosing cholangitis, 90
treatment of, 155, 169
264-265
Sclerosis, tuberous, 95t, 187, 223
Shagreen patches, 223
Restrictive lung disease, 113t, 266
Scoliosis, 253-254, 266
Shaken infant syndrome, 14
Retinoblastoma, 95t, 230t, 239, 244
Shiga-like toxin, 151, 279
Retinoic acid, 52

Seat belts, 10

Seborrheic dermatitis, 53-54, 195


Shigellosis, 84, 150-151, 278-279
Retinopathy
Shingles, 49, 75, 172,279
anemia and, 105t, 113t
Shock, 5-7
diabetic, 59
neurofibromatosis with, 223
dehydration and, 70-71, 70t
of prematurity, 176, 244-245
etiologies of, 5-7, 7t

Seizures, 214-216,285
apnea and, 266t
cardiopulmonary arrest from, 2t
classification of, 215t

CNS tumors and, 233t


GI bleeding and, 88
Reye's syndrome, 49, 220, 283
sepsis and, 173-174
Rhabdomyomas, 223
sickle cell disease and, 113

cystic fibrosis and, 214


definition of, 214
diagnosis of, 215-216, 277

Rhabdomyosarcoma, 230t, 238, 240


treatment of, 7
Rheumatic fever, 1411, 251t
vaccinations and, 137t

febrile, 214-216

Rheumatic heart disease, 32, 36t, 38


Short stature, 60-61, 117-118, 255,

Jacksonian, 215
meningitis and, 150

256t
Rheumatoid arthritis, 61, 137. See also
Arthritis
Shoulder dystocia, 163, 166
Rheumatology, 131-135, 131t, 132t,
Shwachman-Diamond syndrome, 230
135t, 230
S1ADH. See Syndrome of inappropriate
Rhinitis, 128-129, 129
secretion of antidiuretic hormone
Ribavirin, 145
Sickle cell disease, 111-114, 184

naloxone and, 197


neonatal, 187-188, 197
poisoning and, 9t
roseola and, 159t
sickle cell disease and, 114
sodium and, 71-72
treatment of, 216, 217t

abdominal pain from, 75


Rickets, 60, 204, 226, 25It

tuberous sclerosis and, 223

Rickettsia rickettsii, 159


clinical features of, 105t, 112-114,
types of, 188
113t, 219
Riley-Day syndrome, 138
diagnosis of, 104t, 206, 251t
Ristocetin cofactor assay, 123
gastroenteritis and, 150

vaccinations and, 137t


vomiting with, 79t

--------------------------------------- 226
Index
301
hyperbilirubinemia and, 180
Staphylococcal scaled skin syndrome,
encephalopathy and, 220
inheritance of, 95t
50,55
fetal effects of, 94t, 100, 164-165,
small for gestational age and, 165 Status asthmaticus, 262
195-196
treatment of, 114, 136, 286
hypoglycemia and, 189
Status epilepticus, 216
SIDS. See Sudden infant death syndrome Steatorrhea, 82
neonatal seizures and, 187
Single-gene disorders, 93-96
prematurity and, 164t
Steeple sign, 143f
withdrawal syndrome and, 196-197
Single ventricle, 32t
Steroids. See also Corticosteroids
Succimer, 10
Sinus bradycardia, 35, 37f
ACTH deficiency from, 68, 191
Succinylcholine, 2
Sinusitis, 129, 140, 263t
Sudden infant death syndrome (SIDS),
B-cell deficiencies and, 125

for angioedema, 130


for arthritis, 132

2t, 14-15,265
fever with, 137
Sinus tachycardia, 37, 39f, 40f
prevention of, 15, 186, 266, 277

for dermatomyositis, 134


diabetes and, 58
for eczema, 54, 130

RDS and, 176


Skin, eye, mouth (SEM) disease, 171,
risks for, 14t, 196
278

fractures from, 254

Skin grafts, 12, 125


Suicide attempts, 8, 14

for nephrotic syndrome, 202-203


production of, 64f, 65

Sulfadiazine, 167
Skin tags, 93
Sulfonamides, 136
Skull abnormalities, 224, 285
aplastic anemia from, 117
SLE. See Systemic lupus erythematosis
erythema multiforme from, 54-55
Sleep apnea, 99, 129, 228, 265, 269,
G6PD deficiency and, 115
279
thrombocytopenia from, 119
Slipped capital femoral epiphysis
Superior vena cava syndrome, 231
(SCFE), 251t, 252f, 267, 276
Supraventricular tachycardia (SVT),
Small for gestational age, 165-166,
3-4, 37-41, 39f, 40f
183
Surfactant, 175-176

for psoriasis, 53
short stature and, 61
for surfactant, 175
Stevens-Johnson syndrome, 55, 282
Stillbirths, 162
Stomatitis, 90, 140
Stomatocytosis, 111
Strabismus, 243, 244t

Strep throat, 140-142


Smallpox, 136
Syndrome of inappropriate secretion of
Snake bites, 111
antidiuretic hormone (SIADH), 71,

diagnosis of, 76
rheumatic fever and, 14It
urticaria from, 54

207t
Snuffles, 154, 168t, 277, 279
Soy intolerance, 79t
Synovitis, 132, 251t, 252, 257
Spasmus nutans, 216
Syphilis, 154-155

Streptococcal infections
arthritis and, 257
B-cell deficiencies and, 125

congenital, 168-169, 168t, 277,


Spectrin, 111
279
Spherocytosis, 95t, 111-112, 185t

endocarditis from, 33

epiglottitis from, 144


Lyme disease and, 161
Spinabifida, 94t, 212, 280
sexual abuse and, 14
Spinal cord. See also Central nervous
thrombocytopenia with, 119
system

glomerulonephritis after, 140-141,


202t, 203-204
Henoch-Schonlein purpura and,

treatment of, 155, 169


134
deformity of, 253-254, 266
Systemic lupus erythematosis (SLE),
injury of, 1, 7t
132-133
tumors of, 221-222, 230, 251t
abdominal pain from, 75
Spinal muscle atrophy (SMA), 221
arrhythmias with, 36t, 279

osteomyelitis from, 256


otitis media from, 139
periorbital cellulitis from, 247
pneumonia from, 146, 147t, 286

arthritis with, 132


Spitz nevus, 56

resistant, 136

Sprue, celiac, 51-52, 227t


diagnosis of, 132-133, 132t

sepsis from, 14, 173-174,282

fever with, 137-138


Staphylococcal infections
hemolytic anemia with, 111
arthritis and, 257
neonatal thrombocytopenia and,
conjunctivitis from, 245
119, 120t
eczema and, 54

sickle cell anemia and, 113t, 114,


286
Streptomycin, 94t

endocarditis from, 33
nephrotic syndrome and, 202t

String sign, 91
Stroke, 208, 219

rash with, 105t


osteomyelitis from, 256
treatment of, 133
otitis media from, 139

anemia and, 105t


leukemia and, 231

periorbital cellulitis from, 247-248


pneumonia from, 147t, 286
Tachyarrhythmias, 37-42, 39f-42f
rashes from, 50-51
Tachycardia, 4t, 37-42, 39f, 4If, 42f,
resistant, 136
Tacrolimus, 54
sepsis from, 14, 173, 281
Talipes equinovalgus. See Club foot
sickle cell anemia and, 113t
Tanner stages, 46f-47f, 46t, 62, 66
ventriculoperitoneal shunt and,
Tapeworms, 116
213

sickle cell disease and, 113t, 114


Sturge-Weber syndrome, 223, 285
70
Styes, 247
Subdural bleed, 217-219, 218f, 218t
Substance abuse, 10, 195-197

vomiting with, 79t


Tay-Sachs disease, 95t, 102, 222

--------------------------------------- 227
302

Blueprints Pediatrics

breastfeeding and, 226

child abuse and, 13

T-cell disorders, 126-127, 126t


tachycardia with, 37
Teeth, 941, 169
teratogens and, 94t

hyperbilirubinemia and, 180


hypertension and, 207t
polycythemia with, 183

treatment of, 20
Telangiectasia, 105t
Treponema pallidum. See Syphilis
Teratogens, 93, 94t, 195

short stature and, 61-62

teratogens and, 94t


Trichomoniasis, 157
Testicular cancer, 199-200
treatment of, 189
Trichophyton, 51
Testicular torsion, 75, 200, 25It, 275, Tiagabine, 217t
286
Tricuspid atresia, 17t, 18, 21-22, 22f,
Tick-born illnesses, 159-161, 221
277
Testitoxicosis, 65

Tics, 216

Tetanus vaccine, 137t


Tricyclic antidepressants (TCAs)
Tetracycline, 52, 94t, 220
antidote for, 4If

Tinea infections, 5It, 53

Tinnitus, 9t
arrhythmias from, 36t
Tetralogy of Fallot, 18, 23-24, 24f, 32t,Tobacco use, 129, 139, 165, 259, 261
poisoning from, 9t
277
Trimethadione, 94t
arrhythmias and, 36t
Toilet training, 84, 211
chromosome 22 deletion
Trimethoprim, 115, 158, 172

Tongue, strawberry, 135t

Trisomy 13, 97t, 98, 183, 282


syndromes and, 100

Tonsillitis, 79t, 265, 269, 279

pulmonary atresia with, 17t


Trisomy 18, 97t, 166, 181, 183, 282
teratogens and, 94t
Trisomy 21. See Down syndrome
Tet spells, 23-24

Topiramate, 217t
TORCH infections, 167, 168t

Torsade de pointes, 41f


Truncus arteriosus, 17t, 19f
Thalassemia, 104t, 107-109, 120t, 184, Total anomalous pulmonary venous
Tuberculosis
280
connection (TAPVC), 17t, 20-21,
Addison's disease and, 67
genetic defect in, 108t
anemia with, 109
signs of, 105t, 107t, 185t
21f, 122
breastfeeding and, 226
Tourette's syndrome, 216
treatment of, 109
fever with, 137

Toxic epidermal necrolysis, 55, 282

Thalidomide, 94t
pneumonia with, 147t
Theophylline, 9t, 72, 79t, 81, 260t, 262,Toxocariasis, 245
Tuberous sclerosis, 95t, 187, 223
Toxoplasmosis, 167, 168t, 277
276, 284
Tularemia, 137
Thiamin deficiency, 105t
Tumor lysis syndrome, 231
Thiopental, 2

Hodgkin's disease and, 236


mononucleosis and, 141-142
thrombocytopenia with, 119

Turner's syndrome, 30, 60-62, 66, 98,


Third spacing, 7t
282, 284
Thrombocytopenia, 118 119
mpanic membrane perforation, 139
anemia with, 116

Tracheal obstruction, 11, 12t


Ty
Tracheoesophageal fistula, 147,
191-192, 192f

Tyrosine, 101
causes of, 120t
Tyrosinemia, 181
CMV and, 168t
Tyrosinosis, 190
polycythemia and, 183
rubella and, 48

Transcobalamin II deficiency, 15

Transcutaneous oxygen monitor


(TCOM), 18
Transfusions

Ulcer
Thrombocytopenia-absent radius (TAR)
duodenal, 263t
syndrome, 118, 120t
longitudinal, 91
Thrombocytopenic purpura, 49, 119,
272, 282-283
peptic, 81, 87t

anemia and, 111, 185


for hyperbilirubinemia, 181, 182t
partial exchange, 183

sickle cell anemia and, 113t


Ulcerative colitis. See also Inflammatory
Thrombotic thrombocytopenic purpura
twin-twin, 165, 183-184
bowel disease
(TTP), 111, 119
abdominal pain from, 75-76
Thymus
Transient erythroblastopenia of
childhood (TEC), 110,280
diagnosis of, 90-92, 283
agenesis of, 100, 278
treatment of, 91
immunodeficiencies and, 127
Ureteropelvic junction (UPJ)
removal of, 221
obstruction, 198
Thyroid disorders, 62-64

Transient hypogammaglobulinemia of
infancy, 126

Transplants. See also Skin grafts


bone marrow, 117-118, 127, 235

Urethral valves, 199

anemia with, 104t, 116


Urinary tract infections (UTIs), 76,
arrhythmias with, 36t
200-201, 283
cardiopulmonary arrest and, 2t

heart, 7, 26, 34
lung, 264

constipation and, 84-85


bacteremia with, 173
delayed puberty and, 66
constipation and, 84
diabetes and, 58
enuresis and, 211

rejection of, 125


renal, 203-204, 210
thymic, 127
Transposition of the great arteries

failure to thrive and, 227t


diagnosis of, 189, 190f
vesicoureteral reflux and, 199
Down syndrome and, 97

(TGA), 18, 20f, 32f, 268, 277

failure to thrive and, 227t


vomiting with, 79t
fever with, 137
Urologic disorders, 198-211
goiter with, 94t, 189
Urticaria, 54, 130,282

arrhythmia with, 36t


differential diagnosis for, 17t
macrosomia and, 166

--------------------------------------- 228
Index
303
Uterus
UTI with, 201
bicornuate, 164t
withdrawal syndrome with, 197
fibroids of, 93
von Gierke's disease, 101
Uveitis, 55, 90, 131,284

Ventilation-perfusion abnormalities, 266


Ventricular fibrillation, 4t, 39f, 42f
Ventricular-peritoneal shunt, 78, 79t

Ventricular septal defect (VSD), 32t,


von Hippel-Lindau disease, 223
267, 276
von Recklinghausen's disease.
Vaccine (s)
See Neurofibromatosis
contraindications to, 137t
von Willebrand factor (vWF), 123
hepatitis, 123, 1371, 153

common atrioventricular canal


with, 28-29, 29f
pulmonic atresia and, 22

Hib, 50, 136, 137t, 144, 148, 247


von Willebrand's disease, 95t, 122t, 123,
measles, 48, 137t
281
mumps, 137t
Vulvovaginal infections, 157
pertussis, 137t, 146

rubella and, 169

pneumococcal, 137t
Warfarin, 94t, 123-124

treatment of, 28

transposition of great arteries and,


121

trisomies and, 97t

polio, 136, 137t, 221


Waterhouse-Friderichsen syndrome,
rubella, 137t, 169-170
67
schedule of, 136, 137t
Wegener's granulomatosis, 134
sickle cell disease and, 114, 286
Wenckebach phenomenon, 35-36,
tetanus, 137t
37f
varicella, 49

types of, 28
Ventricular tachycardia, 4t, 39f, 4If,
42
Ventriculoperitoneal shunt, 213

Vesicoureteral reflux, 199f, 201, 209,

VACTERL syndrome, 191


Werdnig-Hoffmann disease, 221

283
Vincristine, 84

Whooping cough. See Pertussis


Vagal maneuvers, 39
Wilms' tumor, 230t, 238, 239-240
Vagal nerve stimulator, 216
Vaginosis, 157, 201
Wilson's disease, 95t, 105t, 181

Virilization, 64, 67, 94t

Vision screening, 243, 244t


Vitamin B! deficiency, 105t

Wiskott-Aldrich syndrome, 54, 118,


Valproic acid, 94t, 116, 217t
120t, 127,235
Valvular disease
Withdrawal syndrome, 196-197
acquired, 32-33
anemia and, 111
Wolff-Parkinson-White (WPW)
arrhythmias and, 36t
syndrome, 24, 36t, 37-40, 280
congenital, 30-32, 3If, 32t, 34

Vitamin B12 deficiency, 91, 104t, 105t,


116,225

Vitamin C deficiency, 118


Vitamin D deficiency, 60, 204, 226, 284
Vitamin E deficiency, 105t, 184-185

tachycardia with, 38
X-linked agammaglobulinemia, 125,
Vitamin K deficiency, 87t, 123-124,
128, 230
Vancomycin, 136, 150, 248
X-linked disorders, 95-96, 96t,
Variable penetrance, 94

124, 124t, 281


Vitiligo, 56, 105t, 116

Varicella, 49, 137t, 159t, 171-172, 180,Vocal cord paralysis, 17t


114-115, 118, 121, 127
279
X-linked lymphoproliferative disease,
Volvulus, 75, 78, 80-81, 87t
235
Varicella-zoster immune globulin, 172 Vomiting, 78-80, 79t
Varicocele, 200
Yersinia enterocolitica, 91, 151, 278-279
Vasculitis, 118, 134-135, 207t, 219

bilious, 76, 80, 85, 192


bloody, 76, 80-81, 86-87, 103
complications of, 78

VATER syndrome, 100

dehydration from, 69-70

Zidovudine, 158, 172


Vecuronium, 2
Zoster, 49, 75, 172, 279. See also
Vegetarianism, 116, 225
Varicella

pregnancy and, 78
projectile, 80-81, 283

--------------------------------------- 229
Chapter 3 / Cardiology

31
Treatment
If intervention is r

equired, relief of the aortic valve


gradient may be acco
mplished by open surgical
valvotomy or by ball
oon valvuloplasty. Both surgical
valvotomy and balloo
n valvuloplasty may result in
progressive aortic r
egurgitation that may require
aortic valve replace
ment with a mechanical, homograft, or autograft
valve (Ross procedure).
Pulmonic Stenosis
Pulmonic valve steno
sis accounts for 5% to 8% of
congenital heart def
ects. The pulmonary commissures are fused, the
valve is domed and has a small
central opening, and
there is poststenotic dilatation
of the main pulmonar
y artery. The valve is bicuspid
or dysplastic in 10%
of cases. Right ventricular hypertrophy occurs over t
ime as the ventricle attempts to
maintain cardiac out
put. In critical pulmonic stenosis, a decrease in t
he compliance of the right ventricle will increase ri
ght atrial pressure and may open
Figure 3-13 Critical valvular aortic stenosis with a closedthe foramen ovale, pr
oducing a small right-to-left
ductus arteriosus.Typical anatomic and hemodynamic findings
include: (a) a morphologically abnormal, stenotic valve; (b) post-shunt.
stenotic dilatation of the ascending aorta; (c) elevated left

ventricular end diastolic pressure and left atrial pressures con-Clinical Manife
stations
tributing to pulmonary edema (mild pulmonary venous and
Most patients are as
ymptomatic. Severe to critical
arterial desaturation); (d) a left-to-right shunt at the atrial levelpulmonary s
tenosis may cause dyspnea on exertion
(note increase in oxygen saturation from superior vena cava toand angina. Rightsided congestive heart failure is
right atrium); (e) pulmonary artery hypertension (also secondary to the elevated left atrial pressure); (f) only a modestrare, except in i
nfants with critical pulmonic steno(25-mm Hg) gradient across valve.The low measured gradient sis who may have duc
tal-dependent pulmonary
(despite severe anatomic obstruction) across the aortic valve isblood flow. Char
acteristically, the ejection click of
due to a severely limited cardiac output, as evidenced by thepulmonic stenosis v
aries with inspiration, and a harsh
low mixed venous oxygen saturation (45%) in the superior vena
systolic ejection mu
rmur is heard at the left upper
cava.
sternal border. In s
evere stenosis, a thrill and right
Cloherty JP, Stark AR. Manual of Neonatal Care, 4th ed. Philadelphia:ventricular
heave are palpable. On chest radiograph,
Lippincott-Raven, 1998:426.
heart size and pulmo
nary vascularity are normal,
but the pulmonary ar
tery segment is enlarged. On
ECG, the degree of r
ight ventricular hypertrophy and
right-axis deviation
correlates with the degree of
tricular function is maintained, a harsh systolic ejection murmur is heard at the right upper sternal
alvular gradient and the degree of
border and is preceded by an ejection click heard best
pertrophy can be measured by

stenosis. The transv


right ventricular hy
echocardiogram.

at the left lower sternal border. If ventricular function is compromised, there may be significant stenosis with only a soft murmur appreciated. On chest

Treatment

radiograph, poststenotic dilatation of the ascending


is accomplished by balloon
aorta is present, and in severe cases, pulmonary
stenotic valve. Indications for
edema can be seen. The ECG may show left veninclude a right ventricular
tricular hypertrophy, and a strain pattern of ST
n 50mm Hg or symptoms of

Definitive treatment
valvuloplasty of the
pulmonary valvotomy
pressure greater tha

depressions and inverted T waves may be seen. The


ve heart failure.
valvular lesion, the degree of stenosis, and left venapter has focused on the evaluatricular function are all seen on echocardiogram.
neonate and the most common

right-sided congesti
Thus far, this ch
tion of the cyanotic

--------------------------------------- 230
32

rints Pediatrics
TABLE 3-3
Classic Findings for the 10 Most Common Congenital Heart Lesions
Lesion

CG

Presentation Physical

X-ray
Examination

Atrial septal defect


Murmur
ild RVH
CE, T PBF
Ventricular septal defect Murmur, CHF
VH, RVH
+CE,tPBF
Patent ductus
Murmur, +CHF
VH, +RVH
CE, t PBF
AV canal defect
Murmur, +CHF
Superior" axis
CE, T PBF
Pulmonic stenosis
Murmur, cyanosis
CE,NL,orlPBF
Tetralogy of Fallot
Murmur, cyanosis
VH
+CE, 1 PBF
Aortic stenosis
Murmur, CHF
+CE, NL, PBF
Coarctation of aorta
Hypertension
VH
+CE, NL, PBF
Transposition of the
Cyanosis
VH
CE,NL,ortPBF
great arteries

Fixed split S2

Holosystolic murmur

Continuous murmur

Holosystolic murmur

"

Click, SEM

RVH

SEM

Click, SEM

LVH

iFemoral pulses

Marked cyanosis

Single ventricle
(Variable)
(Variable)
(
Variable)
(Variable)
CE, cardiac enlargement; CHF, congestive heart failure; LVH, left ventricular
hypertrophy; NL, normal;PBF, pulmonary blood flowRVH, right ventricular
hypertrophy; SEM, systolic ejection murmur.
cyanotic and acyanotic congenital heart defects.
oronary artery aneurysms,
Before moving to acquired structural heart diseasewit,
l for occlusion or rupture, that
functional heart disease, and arrhythmias, see Table
fe-threatening. Coronary artery
3-3, which lists the classic findings for the 10 most
ring the subacute phase [llth
common congenital heart lesions.
t 30% of cases but regress

the development of c
h their potentia
makes the disease li
aneurysms develop du
to 25th day) in abou
in most patients. Ea

rly therapy with intravenous


immunoglobulin decre

ases the incidence of coronary


ACQUIRED STRUCTURAL
ss than 10%. High-dose aspirin
HEART DISEASE
likelihood of late aneurysms. The

artery aneurysms to le
therapy lessens the
echocardiogram is us

ed to assess ventricular function


Rheumatic Heart Disease
rdial fluid and coronary artery

and visualize perica


aneurysms. A thoroug

h discussion of Kawasaki's
Rheumatic heart disease results from single or muldisease is found in
Chapter 11.
tiple episodes of acute rheumatic fever. Mitral regurgitation is the most common lesion found. Aortic
insufficiency is also commonly found with or without
Endocarditis
mitral regurgitation. Mitral stenosis is less common
and usually is the end result of multiple attacks of

Pathogenesis

acute rheumatic fever. Least common is aortic stenois is a microbial infection of the
sis. The tricuspid and pulmonary valves are almost
h it may occur on normal
never affected. Symptoms are proportional to the
docarditis is much more likely
degree of valvular damage. Rheumatic fever is disally abnormal valves, valves
cussed in Chapter 12.
fever, acquired valvular

Bacterial endocardit
endocardium. Althoug
valves, bacterial en
to occur on congenit
damaged by rheumatic
lesions (mitral valv

e prolapse), and prosthetic


replacement valves a
s a consequence of turbulent
Kawasaki's Disease
that may precipitate bacterial
Cardiac effects may include pericarditis, myocarditis,
a previous episode of endoand transient rhythm disturbances. However, it is
ipulation or infection, instru-

blood flow. Factors


endocarditis include
carditis, dental man

--------------------------------------- 231
33
Chapter 3 / Cardiology
mentation of the gastrointestinal or genitourinary
s, myocardial abscess formatract, intravenous drug abuse, an indwelling central
prosthetic valve disease.
venous catheter, and prior cardiac surgery.
ocarditis is necessary for high-

embolic complication
tion, or refractory
Prevention of end

In children, alpha hemolytic streptococci [Streptoiotic regimens to prevent endococcus viridans) and Staphylococcus aureus are the
al, respiratory, gastrointestinal, or
most common etiologic agents. S. viridans accounts
ures include oral amoxicillin or
for approximately 67% of the cases, whereas S.
n and gentamicin prior to the
aureus is present in about 20% of cases. When infec-

risk patients. Antib


carditis during dent
genitourinary proced
parenteral ampicilli
procedure.

tion complicates cardiac surgery Staphylococcus epidermidis, gram-negative bacilli, and fungi should be
considered. Gram-negative organisms cause about
5% of cases of endocarditis in children and are more
Y POINTS
likely in neonates, immunocompromised patients,

KE

and intravenous drug abusers. Among the HACEK


ongenitally abnormal valves, valves
(Haemophilus, Actinobacillus, Cardiobacterium, Eikematic fever, acquired valvular
nella, Kingella) organisms, which are a rare cause of
valve prolapse), or prosthetic
endocarditis, Haemophilus influenzae is the most
ves are at increased risk for
common, frequently affecting previously damaged

1. Patients with c

valves.
streptococci (S. viridans) and

2. Alpha hemolytic

damaged by rheu
lesions (mitral
replacement val
endocarditis.

S. aureus are t
he most common etiologic agents
in endocarditis
.
Clinical Manifestations
Fever is the most common finding in children with
bacterial endocarditis. Often, a new or changing
Coronary Artery Dise
ase
murmur is auscultated. Children with endocarditis
usually display nonspecific symptoms such as chest
ase is rare in childhood, but the
pain, dyspnea, arthralgia, myalgia, headache, and
ess appears to begin early in life.
malaise. Embolic phenomena such as hematuria with
at progression of atherosclerotic
red cell casts and transient ischemic attack or stroke
d by genetic factors (familial
may be present. Other embolic phenomena, such as
) and lifestyle (cigarette smokRoth spots, splinter hemorrhages, petechiae, Osier
l diet, high-saturated-fat diet].
nodes, and Janeway lesions, are relatively rare in
me habits are formed during

Coronary artery dise


atherosclerotic proc
There is evidence th
lesions is influence
hypercholesterolemia
ing; high-cholestero
Because many lifeti

children with bacterial endocarditis.


rtunity exists for prevention of

childhood, the oppo


coronary artery dis

ease.
Diagnostic Evaluation
Laboratory studies include a complete blood count,
erythrocyte sedimentation rate (ESR), c-reactive
SEASE
protein (CRP), and urinalysis. Multiple blood cultures increase the probability of discovering the

_FUNATIONAL HEART DI

Myocarditis

pathogen. Positive blood cultures, an elevated ESR,


elevated CRP, hematuria, and anemia are most often
rditis in North America refound. The echocardiogram is used to define vegetaection of the myocardium, pretions or thrombi in the heart.
ruses (coxsackie B virus and

Most cases of myoca


sult from viral inf
dominantly enterovi
echovirus). It is u

nclear whether myocardial damage


from viral myocardi
tis results from direct viral invaTreatment
ne antibody response.
Medical management consists of 6 weeks of intraions
venous antibiotics directed against the isolated
pathogen. Surgery is indicated for endocarditis when
gree of damage to the
medical treatment is unsuccessful, refractory cons may be asymptomatic and the
gestive heart failure exists, or there are serious
de only by finding ST- and T-

sion or an autoimmu
Clinical Manifestat

Depending on the de
myocardium, patient
diagnosis may be ma

--------------------------------------- 232
34

Blueprints Pediatrics

wave changes on an ECG done for an unrelated

Treatment

reason, whereas others may present with fulminant


Medical therapy incl
udes inotropic agents and
congestive heart failure. Common symptoms include
ove myocardial contractility and
fever, dyspnea, fatigue, and chest pain (usually due to
rload on the weakened ventricle.
a secondary pericarditis). Signs include tachycardia,
reload and hopefully improve
evidence of congestive heart failure, and
ving the dilated ventricle to a
S3 ventricular gallop. The ECG often reveals ST-

vasodilators to impr
to decrease the afte
Diuretics decrease p
cardiac output by mo

more favorable posit


ion on the Frank-Starling curve,
segment depression and T-wave inversion, as well as
edications are used to control
arrhythmias and conduction defects. The chest radintricular arrhythmias. If medical
ograph varies from mild to marked cardiomegaly.
transplantation may be necessary.
Echocardiogram denotes dilated or hypocontractile

and antiarrhythmic m
potentially fatal ve
therapy fails, heart

ventricles, or both. Pericardial effusion may be


present. Endomyocardial biopsy may be indicated in
Hypertrophic Cardiom
yopathy
select cases to confirm diagnosis. Viral etiology
should be evaluated by viral culture and PCR from
thic hypertrophic subaortic
the throat, stool, blood, and pericardial fluid, if
ic cardiomyopathy is an autosopresent.
disorder in which the ventric-

Also known as idiopa


stenosis, hypertroph
mal dominant genetic
ular septum is thick

ened, resulting in left ventricular


outflow tract obstru
ction. In the thickened stiff left
Treatment
function is well preserved, but sysTherapy for patients with viral myocarditis is sup-

ventricle, diastolic

portive to maintain perfusion and oxygenation. Treat


mpromised. Abnormal motion of

tolic function is co
the mitral valve res

ults in mitral insufficiency.


ventricular arrhythmias, conduction abnormalities,
and congestive heart failure as indicated. Intravenous
ons
immunoglobulin is given to minimize further

Clinical Manifestati

damage to the myocardium. The prognosis for


pnea on exertion, chest pain,
patients with myocarditis depends on the extent of
s often a bisferious pulse (double
myocardial damage.
tion is hindered by septal

Symptoms include dys


and syncope. There i
peaked) because ejec
obstruction, a ventr

icular gallop (S

3), and murmurs


indicative of mitral

regurgitation and left ventricular


Dilated Cardiomyopathy
ction. ECG illustrates left-axis

outflow tract obstru


deviation, left vent

ricular hypertrophy, and possible


Dilated or congestive cardiomyopathy is characteres consistent with ischemia or
ized by myocardial dysfunction and ventricular

ST- and T-wave chang


strain. The echocard

iogram is diagnostic.
dilatation. Although usually an idiopathic disorder, it
can be caused by neuromuscular disease (Duchenne
muscular dystrophy) or drug toxicity (anthracy-

Treatment
Therapy is centered

around preventing fatal ventricclines). Dilation of the left ventricle results in


congestive heart failure. An increase in left atrial
decreasing the stiffness of
pressure, pulmonary venous pressure, and pulmonary
ith negative inotropic medicapillary wedge pressure results in pulmonary
cium channel blockers, and
edema.
king agents. The avoidance of

ular arrhythmias and


the left ventricle w
cations, such as cal
beta-adrenergic bloc
competitive sports i

s essential because sudden death


Clinical Manifestations
significant risk.
Symptoms include dyspnea, orthopnea, and parox-

during exertion is a

ysmal nocturnal dyspnea. Eventually, right heart


failure with dependent edema occurs, and a pulsus

__ARRHYTH__MIAS

alternans may be noted. On cardiac examination, a


right ventricular heave and an 83 gallop are found.
ren are much less common than
The ECG reveals rhythm disturbances, left ventricujust as life threatening. Arrhythlar hypertrophy, and nonspecific ST- and T-wave
orders of impulse formation,
ischemic changes. Ventricular function is evaluated
or both and are generally classiby echocardiogram.

Arrhythmias in child
in adults but can be
mias result from dis
impulse conduction,
fied as follows.

--------------------------------------- 233
Chapter 3 / Cardiology

35
bradycardia, and sin

us pauses. Bradycardias due


KEY POINTS
include first-degree heart
1. Most cases of myocarditis in North America result
heart block, and third-degree
from viral infection of the myocardium.
ck. Second-degree heart
2. Dilated or congestive cardiomyopathy is characided into Mobitz type I block
terized by myocardial dysfunction and ventricular
type II block, and fixed-ratio

to conduction block
block, second-degree
(complete) heart blo
block is further div
(Wenckebach), Mobitz

dilatation; it is usually idiopathic.


) block.
3. Therapy for dilated cardiomyopathy includes
is
inotropic agents to improve left ventricular con-

atrioventricular (AV

tractility and vasodilators to decrease afterload.


e rhythm strips of various
Diuretics decrease preload, and antiarrhythmic
bradycardia is associated with
medications are used to control potentially fatal
, hypoxia, central nervous system
ventricular arrhythmias.
ased intracranial pressure,
4. In hypertrophic cardiomyopathy, the ventricular
rkalemia, hypothermia, drug
septum is thickened, resulting in left ventricular
n, beta-blockers, calcium channel
outflow tract obstruction.
atrial surgery. It is also a normal
5. Therapy for hypertrophic cardiomyopathy is centhletic teenagers. The ECG
tered around preventing fatal ventricular arrhythave with normal AV conduction
mias and decreasing the stiffness of the left
OObpm in the neonate and 60bpm
ventricle with negative inotropic medications.
When sinus bradycardia becomes

Figure 3-14 shows th

Differential Diagnos

bradycardias. Sinus
increased vagal tone
disorders with incre
hypothyroidism, hype
intoxication (digoxi
blockers), and prior
finding in healthy a
reveals a normal P w
at rates less than l
in the older child.
too slow, sinus paus

es or escape rhythms may occur.


The escape rhythms m
ost often seen include ectopic
atrial bradycardia o
r ectopic atrial rhythm, junctional
Brady arrhythmias
ional rhythm, or a slow idiovenSinus node dysfunction
hythm.
Conduction block
block usually results from

bradycardia or junct
tricular ventricular r
First-degree heart
slowing of atriovent

ricular conduction at the level of


Tachyarrhythmias
ssociated with increased vagal tone,
Narrow QRS
er administration, infectious
Wide QRS
arditis, Lyme disease), hypotherPremature Beats
ormalities (hypo/hyperkalemia,
Atrial
pomagnesemia), congenital

the AV node. It is a
digoxin and beta-block
etiologies (viral myoc
mia, electrolyte abn
hypo/hypercalcemia, hy
heart disease (ASD,

atrioventricular canal defect,


Ventricular
VC, and L-transposition of
Bradyarrhythmias are the result of either depresr "corrected transposition"),
sed automaticity or block of an impulse, whereas
cardiomyopathy. First-degree

Ebstein's anomaly, TAP


the great arteries o
rheumatic fever, and

tachyarrhythmias or premature beats arise from


rized on ECG by PR interval
abnormal impulse formation caused by enhanced
and rate. The rhythm is regular,
automaticity, a reentrant circuit, or triggered activity.
nus node, and has a normal QRS
Arrhythmias may result from congenital, functional,
or acquired structural heart disease; electrolyte
rt block refers to episodic interdisturbances (potassium, calcium, and magnesium);
conduction:
drug toxicity; poisoning; or an acquired systemic
disorder. Table 3-4 lists etiologies predisposing
ebach) denotes progressive
children to arrhythmias.

AV block is characte
prolongation for age
originates in the si
morphology.
Second-degree hea
ruption of AV nodal

Mobitz type I (Wenck

prolongation of t
he PR interval over several beats
until a QRS is dr
opped. This cycle repeats itself
Bradyarrhythmias
he number of beats in a cycle may

often, although t
not be constant.

The QRS configuration is normal.


As already stated, bradyarrhythmias result from sinus
is rhythm are the same as those for
node dysfunction or conduction block. Bradycart block.
dias due to sinus node dysfunction include sinus
used by abrupt failure of atribradycardia, junctional bradycardia, ectopic atrial
uction below the AV node in the

Etiologies for th
first-degree hear
Mobitz type II is ca
oventricular cond

--------------------------------------- 234
36

Blueprints Pediatrics
TABLE 3-4
Factors Predisposing to Dysrhythmias
Congenital heart disease
Supraventricular dysrhythmias: Ebstein's anomaly (may also present with WPW
syndrome), atrial septal defects, atrial
surgery, L-transposition of the great arteries, after Fontan operation
Ventricular dysrhythmias: aortic valve disease, pulmonary valve disease, aft
er tetralogy of Fallot repair, anomalous left
coronary artery, RV dysplasia
Heart block (varying degrees): after open-heart surgery (Ebstein's anomaly,
L-transposition of the great arteries,
common atrioventricular canal, VSD repair); congenital complete heart bloc
k (idiopathic, associated with maternal
systemic lupus erythematosus, L-transposition of the great arteries)
Isolated conduction system disorders
WPW syndrome
Prolonged QT interval syndromes
Associated with systemic illness
Infectious myocarditis
Kawasaki's disease

Idiopathic dilated or hypertrophic cardiomyopathy


Friedreich's ataxia (atrial tachycardia or fibrillation)
Muscular dystrophies (Duchenne, periodic paralysis)
Glycogen storage diseases (Pompe's disease)
Collagen vascular diseases (rheumatic carditis, systemic lupus erythematosus
, periarteritis nodosa, dermatomyositis)
Endocrine disorders (hyperthyroidism, adrenal dysfunction)
Metabolic and electrolyte disturbances (hypomagnesemia, hyperkalemia, hypoca
lcemia, hypoxia)
Lyme disease
Drug toxicity
Chemotherapeutic agents (anthracyclines)
Tricyclic antidepressants
Cocaine
Digitalis, beta-adrenergic blockers, calcium channel blockers
Asthma medications (sympathomimetics)
Other causes
Blunt chest trauma (myocardial contusion)
Increased intracranial pressure
bundle of His-Purkinje fiber system. It is a more
diac recordings a
re required to distinguish the site
serious bradycardia than first-degree heart block or
of injury. Patien
ts may progress to complete heart
Wenckebach because it can progress to complete
block.
heart block. On ECG, there is sudden AV conduction failure with a dropped QRS after a normal P
Third-degree hear
t block occurs when no atrial
wave. No preceding PR interval prolongation is
impulses are conduct
ed to the ventricles. The atrial
seen in normal conducted impulses.
rhythm and rate are
normal for the patient's age, and
Fixed-ratio AV block is an arrhythmia in which the
the ventricular rate
is slowed markedly (40-5 5 bpm).
QRS complex follows only after every second
If an escape rhythm
arises from the AV node fjunc(third or fourth) P wave, causing 2:1 (3:1 or 4:1)
tional rhythm), the
QRS interval is of normal duraAV block. There is a normal PR interval in contion, but if an esca
pe rhythm arises from the distal
ducted beats. There is usually a normal or slightly
His bundle or Purkin
je fibers, the QRS interval is proprolonged QRS. Fixed-ratio block results from
longed (idioventricu
lar rhythm). Congenital comeither AV node or His bundle injury, and intracarplete AV block can b
e an isolated abnormality or can

--------------------------------------- 235
37
Chapter 3 / Cardiology
PI P, P

Tachyarrhythmias

Sinus bradycardia 7**


H 4Narrow-complex tachy
cardias have a QRS morphol-

ogy similar or ident


ical to that of normal sinus
First-degree AV block
most, but not all, SVTs (some

rhythm. They include


SVTs have a widened

QRS). Narrow-complex tachycardias may be due t


o increased automaticity or from
p , P

a reentrant circuit.

Narrow-complex tachycardias
-JL^-*.
Mobitz type 1 *
icity include sinus tachy(Wenckebach phenomenon)

due to increased automat

cardia, ectopic atri


al tachycardia, junctional ectopic
tachycardia, and atr
ial fibrillation. Narrow-complex
tachycardias caused
by reentrant mechanisms are
Mobitz type II
categorized as ortho
dromic reentrant tachycardia
(ORT) or antidromic
reentrant tachycardia (ART). In
p? p PP p
es down the AV node and up
2:1 AV block
nce the ventricles are depolarized

n, P P

R. ORT the SVT propagat


the bypass tract. Si
in the normal fashio

n, down the AV node, the QRS


complex is narrow. I
n ART the SVT propagates down
R R R R
PPPPlPPPlPP | P
up the AV node. Since the venComplete (third-degree)

the bypass tract and

tricles are depolari


zed down the bypass tract, the
AV block
QRS is widened. Narr
ow-complex AV reciprocating
Figure 3-14' Bradyarrhythmias.
AV node reentrant tachycardia,

tachycardias include
WPW syndrome orthodr

omic tachycardia (accessory


pathway not conceale
d on ECG delta wave), orthodromic atrioventricu
lar reciprocating tachycardia
be associated with L-transposition of the great arter(accessory pathway c
oncealed on ECG no delta

ies, atrioventricular canal defect, or maternal lupus


wave), sinoatrial re
entrant tachycardia, and atrial
erythematosus. Other causes include open-heart
surgery (especially after large ventricular septal
lex tachycardias are relatively
defect closure), cardiomyopathy, or Lyme disease.
ly.

flutter. Narrow-comp
well tolerated acute
Conversely, wide-

complex tachycardias, defined as


Newborns with congenital complete heart block may
tachycardias with a
QRS more than 0.12 seconds, are
present with hydrops fetalis.
a medical emergency.
Wide-complex tachycardias
Treatment
tachycardia, ventricular fibrilla-

include ventricular
tion, WPW syndrome a

ntidromic reentrant tachycarNo intervention is necessary for sinus bradycardia if


dia, and orthodromic
SVT with aberrancy.
cardiac output is maintained. A management algorithm for sinus bradycardia is shown in Figure 3-15.
Differential Diagnos
is
No treatment is necessary for first- or secondFigure 3-17 shows th
e rhythm strips of the various
degree heart block (Mobitz type I). Mobitz type II,
fixed-ratio AV block, and third-degree heart block all
uses of tachyarrhythmia are as
require pacemaker placement. In Mobitz type II and

tachycardias. The ca

fixed-ratio AV block, prophylactic pacemaker insercardias


tion is essential to protect the patient should he or
ever, stress, dehydration, and
she progress to complete heart block with inadequate

Narrow-Complex Tachy

cardiac output away from medical care.


-sinus tachycardia SVT):
If the child with complete heart block is hemofrom a concealed bypass tract
dynamically unstable, transcutaneous or transvenous
ode reentrant tachycardia,
pacing can be performed acutely, and permanent
Ebstein's anomaly (associtransvenous or epicardial pacemaker placement can
drome), L-transposition of the

follows.

Sinus tachycardia: F
anemia
ORT (most common non
Most cases result
causing ORT, AV n
WPW syndrome ORT,
ated with WPW syn

be performed later. Third-degree heart block is

great arteries

managed with either ventricular demand pacing or


al surgery (D-TGA s/p Mustard/
AV sequential pacing. Figure 3-16 is a management
, ASD s/p repair Hemi-Fontan,
algorithm for AV block.
tis, structural heart disease with

Atrial flutter: Atri


Senning procedure
Fontan), myocardi

--------------------------------------- 236
38

Blueprints Pediatrics
Sinus bradycardia
Diagno

sis
-f-

< 2 yr HR

< 90
0 yr-11 yr
HR < 80
11yr HR<60
normal QRS
pattern
Atropine

Determine

0.02 mg/kg

cause

Epinephrine

Observation

increa

sed
intracranial

pressure?
Yes

1
Isoproterenol

Treat ICP

Treat
infusion begin at
hypertensive crisis,
0.05-0.1 |ig/kg/min
Hydralazine

Hyperventilation,
Mannitol
See Chapter 8

Labetolol
Nitroprusside
Pacing
(transvenous or
transcutaneous)
Determine cause
Figure 3-15

Management algorithm for sinus bradycardia.

dilated atria (Ebstein's anomaly, tricuspid atresia,


rdia
rheumatic heart disease of the mitral valve), severe
dia: Congenital or acquired
tricuspid regurgitation

Wide-Complex Tachyca
Ventricular tachycar
heart disease res

ulting in ventricular dilation or


Atrial fibrillation: Most often seen with left atrial
ntricular suture line, drug ingesenlargement (rheumatic heart disease of the mitral
rome ART
valve, VSD, systemic to pulmonary artery palliative
tion: Terminal rhythm that develshunt placement); other causes that result in right
, ischemia, or high-voltage elecatrial or biatrial enlargement include Ebstein's
edisposing factors include WPW
anomaly, WPW syndrome, and myocarditis
QT syndrome

hypertrophy or ve
tion, or WPW synd
Ventricular fibrilla
ops after hypoxia
trical injury; pr
syndrome and long

--------------------------------------- 237
Chapter 3 / Cardiology

39
AV block

1st-degree block

Prolonged PR interval

No treatment

(see text)
Mobitz I - (Wenckebach) _
PR interval progressively
prolonged until QRS dropped
If hemodynami
cally
Mobitz II - dropped QRS,

unstable, gi

ve
without preceding prolongation

epinephrine 10

ng/kg,
of PR interval, QRS normal

atropine 0.02

mg/kg
Fixed-ratio AV
block

QRS follows every

2:1, 3:1, etc.

ous
ous
Figure 3-16

2nd or 3rd P wave


If refractory

to medications
Complete dissociation of
Permanent
atrial and ventricular activity,
transvenous or
ventricular rate much slower
epicardial
than atrial rate (40-55 beats/min)
pacemaker
Management algorithm for AV block.

Sinus tachycardia
es from vagal maneuvers to phar-

Temporary
pacemaker
transcutane
or transven

tachycardia progress

macotherapy to cardi
oversion. Vagal maneuvers
enhance vagal tone t
o slow conduction in the AV node
Orthodromic
termination of the arrhythmia.
reentrant
sed in infants by applying ice to
tachycardia
er children through carotid
Atrial flutter
o keep the infant's airway unob-

and often result in


Vagal tone is increa
the face, and in old
massage; make sure t
structed when applyi

ng ice to the face. If vagal


maneuvers are ineffe
ctive in stable narrow-complex
tachycardia, adenosi
ne is given to block the AV node
to break a reentrant
SVT whose circuit involves the
AV node (AV node ree
ntrant tachycardia, WPW
Ventricular
oncealed bypass tract ORT).
tachycardia

syndrome with ORT, c

Adenosine will be in
effective on a narrow-complex
Ventricular
ults from increased automaticity
fibrillation
nism that does not involve the

tachycardia that res


or a reentrant mecha
AV node (sinus tachy

cardia, ectopic atrial tachycarTachydysrhythmias.


Figure 3-17
c tachycardia, atrial flutter, or

dia, junctional ectopi


sinoatrial reentrant

tachycardia). If adenosine returns


the child to normal
sinus rhythm and WPW is not suspected (no delta wav
e seen after conversion of tachyTreatment
cardia), the child i
s started on digoxin to reduce the
Narrow-Complex Tachycardia
s. If adenosine reveals WPW synTreatment of sinus tachycardia involves correcting the
ted after conversion of tachyunderlying cause of the tachycardia. Figure 3-18 outblocker, because the use of digoxin
lines a management algorithm for supraventricular
and speed up conduction over
tachycardia. Treatment for stable narrow-complex
y in an antidromic fashion and

--------------------------------------- 238

risk of future event


drome (delta wave no
cardia), use a betacan slow the AV node
the accessory pathwa

Blueprints Pediatrics
Pulseless VT or VF: Nonsynchronized defibrillao note pupillary response, level
tion [2J/kg, followed by 4J/kg if unsuccessful,
d localizing findings.
followed by 4J/kg if unsuccessful) is indicated.
Epinephrine is administered if resuscitation is
unsuccessful after three electrical shocks, followed
by shock again, 4J/kg. Precede subsequent defib-

ination is performed t
of consciousness, an

Secondary Survey
The secondary survey

includes a head-to-toe physirillation attempts with intravenous lidocaine,


amiodarone, or epinephrine.
rder to determine the extent of

cal examination in o

injury and further p


rioritize treatment. The patient's
For a full discussion of drug physiology, indicalevel of consciousne
ss is assessed using the Glasgow
tions, dosage, route of administration, effects, and
Coma Scale (see Tabl
e 15-5). In preparation for the
side effects, see The Harriet Lane Handbook osecondarr
y survey,
the patient should be undressed.
Golden Hour: The Handbook of Advanced Pediatric
Because of children'
s large surface-to-body mass
Life Support. Table 1 -2 describes the indications anratiod
, they cool rap
idly, and passive heat loss can be
effects of each drug.
problematic. Exposur
e (hypo- or hyperthermia)
For disability, a rapid screening neurologic exammust be detected and
dealt with promptly.
TABLE 1-2
Drugs Used in Pediatric Cardiorespiratory Resuscitation
Drug Indication
Effect
Atropine
Bradycardia and
Increases hear
t rate and conduction through the
atrioventricular block
atrioventric
ular node by decreasing vagal tone
Bicarbonate
Severe refractory metabolic
Increases bloo
d pH
acidosis and/or hyperkalemia
Elemental calcium
Hypocalcemia, hyperkalemia,
Increases myoc
ardial contractility, increases
(calcium gluconate or
hypermagnesemia, and calcium
ventricular
excitability, and increases conduction
calcium chloride)
channel blocker overdose
velocity th
rough the myocardium
Dextrose
Hypoglycemia
Increases bloo
d glucose level
Epinephrine (1:10,000)
Asystole, bradycardia, pulseless
Increases syst
emic vascular resistance, chronotropy,
VT,VF
and inotrop
y, thereby increasing cardiac output
and blood p
ressure (increasing diastolic blood
pressure in
creases coronary artery perfusion
pressure)
Epinephrine (1:1000)
Pulseless arrest after above
Same as above
dose or as first dose down

endotracheal tube if no
vascular access available
Lidocaine
Ventricular ectopy, VT,VF
fractory pulseless VT and VF more

Helps make re
susceptible

to cardioversion, may suppress


hemodynamic
ally stable VT, and decreases the
likelihood
of recurrence of ventricular ectopy
Amiodarone
Atrial (refractory SVT) and
Blocks Na, K,
and Ca channels and beta-receptors in
ventricular arrhythmias
the myocard
ium as well as alpha- and
(refractory pulseless VT,
beta-recept
ors in the vascular periphery
refractory VF, hemodynamically
stable VT)
Narcan
Presumed or known opiate
Rapid reversa
l of opiate effect
intoxication
Drugs that can be given by endotracheal tube include lidocaine, atropine, Narc
an, and epinephrine (high dose).
SVT, supraventricular tachycardia; VF, ventricular fibrillation; VT, ventricul
ar tachycardia.

--------------------------------------- 239
40

Blueprints Pediatrics
Supraventricular tachycardia
H-H-H-HHUnstable

Stable

Heart failure, hypotension


Vagal maneuvers
Cardioversion

Adenosine 50 ng IV bolus

0.25-2.0 J/kg

Double dose up to 400 |ag

or

i.e., 100, 200, 400

Overdrive pacing
No conversion

No conversion

unstable

stable

Rhythm
conversion
Check diagnosis
Chronic management
digoxin
(assuming WPW

s
yndrome not present)
or |3-blocker
(If
WPW syndrome present)
Options
Digitalization or
Propranolol (0.5 mg/kg) IV or
Esmolol 500 u.g/kg/min over
1 min, then 50 ng/kg/min IV
(titrate to effect) or
Amiodarone 2.5 mg/kg
IV and/or overdrive pacing
Management algorithm for supraventricular tachycardia.

Figure 3-18

result in ventricular fibrillation secondary to atrial fib- congestive heart fai


lure or hypotension, cardioversion
rillation or some other fast atrial arrhythmia. For this
or transesophageal o
verdrive pacing is indicated. Synreason, digoxin should be avoided to treat ORT assochronized cardiove
rsion is required to avoid the inadciated with WPW syndrome. Propranolol is an effecvertent development
of ventricular fibrillation.
tive and safe alternative to digoxin in ORT associated
In unstable atria
l flutter, synchronized cardioverwith WPW syndrome. When unstable narrowsion or overdrive pa
cing is used when rapid intercomplex tachycardia is present and the patient has
vention is necessary
because of congestive heart

--------------------------------------- 240
Chapter 3 / Cardiology

41
Ventricular

tachycardia
3 or more beats
QRS usually widened
AV dissociation
Rate: 150-300/minute
Fusion beats
Normotensive
1.ABCs

Lidocaine 1 mg/kg x 2 IV load


2. Synchronized cardioversi
on

10-50 jig/kg/min
(0.25-2.0 J/kg)
procainamide 10 mg/kg IV load
(repeat)
over 30-60 minutes
amiodarone 2.5 mg/kg IV
[Do not use amiodarone
and procainamide together both prolong the QT interval
and both may cause hypotension]
Lidocaine 1 mg/kg IV x 2
followed by
Lidocaine 20-50 (ig/kg/mi

n
Specific Rx for drug-induce
d VT:
Digibind for digitalis
NaHCO 3 for tricyclic antidep
ressants
Phenytoin for cocaine, digoxin, or
bupivacaine
p-blockers or magnesium sulfate (5
0 mg/kg IV)
Figure 3-19

Management algorithm for

for torsade de pointes

ventricular tachycardia.
failure. Once cardioversion has occurred, digoxin,
ulation is needed before convertbeta-blockers, procainamide, amiodarone, sotalol, or
crease the risk of embolization of
a quinidine/digoxin combination may be given to
l clots. An alternative to anticoaguhelp prevent recurrences. If the child is hemodyageal echocardiography to assess
namically stable, he or she should be loaded with
ts are seen, cardioversion may
digoxin and then given procainamide in an attempproceedt
with a slightly increased risk of
to convert the arrhythmia. It is critical to load
tive to anticoagulation. Quiniwith digoxin before giving procainamide, because
or amiodarone can be effective in
procainamide has vagolytic activity that could
sion of atrial fibrillation, and
inadvertently increase the ventricular rate and cause
namide are good long-term mainacute hemodynamic deterioration.
ronized cardioversion converts
If atrial fibrillation has been present for more than

a few days, anticoag


ing the rhythm to de
possible intra-atria
lation is transesoph
for clots. If no clo
, although
thromboembolism rela
dine, procainamide,
pharmacologic conver
quinidine and procai
tenance drugs. Synch
most cases to sinus

rhythm.

--------------------------------------- 241
42

rintg Pediatrics
orthodromic SVT with

aberrancy as though the


Ventricular fibrillation
lar tachycardia. Hypotensive
Ventricular tachycardia

patient has ventricu


or unresponsive pati

ents should be treated immediately with cardi


opulmonary resuscitation and
synchronized cardiov
ersion. After cardioversion,
Chaotic

sinus rhythm can be

maintained with intravenous


ventricular pattern

lidocaine or amiodar

one. Normotensive patients


with acute-onset ven
tricular tachycardia can be
treated with intrave
nous lidocaine or amiodarone in
an attempt to break
the arrhythmia without
cardioversion.
Children with ven
tricular fibrillation should
receive CPR and must
be defibrillated with nonsynNon-synchronized
ion. Giving epinephrine may
defibrillation 2 J/kg
on into coarse fibrillation and allow

chronized cardiovers
turn fine fibrillati
successful defibrill

ation. The management algorithms


for ventricular tach
ycardia and ventricular fibrillation/pulseless ventr
icular tachycardia are outlined in
Defibrillation 4 J/kg
re 3-20, respectively.
Defibrillation 4 J/kg

Figure 3-19 and Figu


K

EY POINTS
1. Bradyarrhythmia
s with widened QRS complexes
are likely to b
e escape rhythms from the His
bundle or Purki
nje system (idioventricular
rhythm) and are
at high risk for progression to
complete heart
block.
2. Symptomatic sin
us bradycardia, second-degree

Lidocaine 1 mg/kg IV or
bitz type II and fixed-ratio AV
Amiodarone 2.5 mg/kg IV and repeat d
rd-degree heart block all need
Epinephrine 100 ng/kg IV/IO/

heart block (Mo


block), and thi
pacing.
3. Narrow-complex

tachycardias tend to be well tolerated acutely,


whereas wide-complex tachycarContinue CPR
ered a medical emergency.

dias are consid


4. Treat wide-comp

lex tachycardia due to SVT (WPW


Figure 3-20 Management algorithm for ventricular
RT or SVT with aberrancy) as
fibrillation.
ent has ventricular tachycardia.

syndrome with A
though the pati
5. When treating S

VT, rule out WPW syndrome,


Wide-Complex Tachycardia
atment for WPW-associated SVT is
Treat wide-complex ventricular tachycardia due to
that for non-WPW SVT.
WPW syndrome with antidromic conduction or

because the tre


different from

--------------------------------------- 242
Chapter
Development
_ DEVELOPMENTAL _MILE_STON E_S
best indicator of future intellec-

Language is the
tual achievement.

Language development is divided


Neurologic, intellectual, and physical development
receptive and expressive, each
in infants and children each occur in an orderly
te DQ.
and sequential manner. Table 4-1 lists the normal pronts require age-adjusted parameters
gression of developmental milestones. The informaeir developmental achievement.
tion is subdivided into gross motor, visual motor (or
age, a child's age should take into
fine motor-adaptive), language, and social milestones.
tional age at birth. For example, at
The two developmental screens most commonly
h checkup, a former premature
used by pediatricians are the Denver II developmental
weeks' gestation should be able to
screening test and the Clinical Adaptive Test (CAT)/
propriate for a 6-month old.
Clinical Linguistic and Auditory Milestone Scale
(CLAMS). The Denver II divides streams of development into gross motor, fine motor-adaptive, language,
DEVELOPMENTAL
and personal-social. The CAT rates problem-solving and
visual motor ability, and the CLAMS assesses language
development from birth to 36 months of age.

into two streams,


assigned a separa
Premature infa
when assessing th
Until 2 years of
account the gesta
his or her 9-mont
infant born at 28
perform skills ap

ilt VARIATIONS IN
PATTERNS

Attention Deficit
Hyperactivity Disorder
Sometimes the developmental process does not
progress appropriately, and developmental disabilihyperactivity disorder (ADHD) is
ties may be suspected. Abnormal development can
ed of inattention, hyperactivity,
be subdivided into developmental delay, dissociation,
o the extent that the behavior is
and deviancy. Developmental delay refers to a pernconsistent with the developmenformance significantly below average in a given skill
child. ADHD may be found in 5% of
area. A developmental quotient (DQ) below 70
boys in elementary school. Up to
constitutes developmental delay. The DQ reflects the
cted with ADHD as a child will
child's rate of development: DQ = (developmental
ymptoms into adulthood.
age -H chronological age) x 100.
ations
Developmental dissociation refers to a substantial
difference in the rate of development between two
f ADHD to be made, a child must
skill areas. An example of a developmental discrepoutlined in the DSM-IV (Table 4ancy between gross motor and language developof ADHD requires the presence of
ment is a child with isolated mental retardation
ractivity, and impulsiveness in mulwhose gross motor development is normal. Develop(e.g., in school and at home). The
mental deviancy refers to nonsequential developpresent for at least 6 months and
ment within a given area of skill. For example, the
nt by age 7. However, the signs of
development of hand preference at 12 months is a
ized in settings that are able
departure from normal sequence and may be related
ate reinforcement, are new to the
to an abnormality of the other extremity.
hly supervised. With this in mind, .a

Attention deficit
a syndrome compos
and impulsivity t
maladaptive and i
tal stage of the
girls and 10% of
70% of those affe
have persistent s
Clinical Manifest
For a diagnosis o
meet the criteria
2). The diagnosis
inattention, hype
tiple environments
symptoms must be
are usually prese
ADHD may be minim
to provide immedi
child, or are hig

--------------------------------------- 243
44

Blueprints Pediatrics
TABLE 4-1
Commonly Quizzed Developmental Milestones

Age Gross Motor Fine (Visual) Motor


Social
1 month
Raises head slightly
to
Fixes on face
from prone

Language
Follows with eyes to
midline only; tight
grasp

Alerts/startles
sound

2 months
vely
3 months

Smiles responsi

Recognizes parent
Holds head up,
Hands open at rest
Reaches for familiar
steady
objects or people
4-5 months
Rolls front to back,
Grasps with both
e
Enjoys observing
back to front; sits
hands together
environment
well supported
6 months

Sits well
Recognizes strangers
unsupported

Transfers hand to

Coos

Orients to voic

Babbles

hand; reaches with


either hand

9 months
ada/

Crawls, cruises, pulls Uses pincer grasp;


Plays pat-a-cake
to stand
fingerfeeds

Begins to use"d
mama"; unders

tands
"no"
12 months
Walks alone
than
Imitates; comes when

Throws, releases
objects

follows

called; cooperates

ands
15 months

with dressing
Walks backward;

1-8 words other


"dada/mama";
one-step comm

creeps upstairs
18 months
Runs
parts
Plays around (not with)
other children
Squats and recovers

21 months
ations
24 months
Walks well up and
tep
Parallel play
down stairs
anger

Builds 2-block
tower; scribbles
Feeds self (messily)

Points to body

with utensils

when asked

Builds 5-block tower

Two-word combin

Removes clothing

Understands 2-s
commands; str
understands V

2 of
speech
30 months
Throws ball
noun
Knows first, last
overhand
names
3 years
Pedals tricycle
s; uses
Group play; shares

Appropriate pro
use
Draws a circle

3-word sentence
plurals, past

tense;
stranger unde
rstands
% of speech

4 years

Alternates feet going


Imaginative play
down stairs; skips

5 years
me

Catches ball; dresses

Knows colors

alone
Ties shoes

Prints first na

Plays cooperative
games; understands
"rules" and abides by

them
child may not display any signs of ADHD when in
ly used rating scales are the
the pediatrician's office.
Teacher's Rating Scale
Assessment
er's Parent and Teacher Scale.

teachers. Two common


ADD-H Comprehensive
and the revised Conn
A complete physical

exam should be performed, but


To assess a child with possible ADHD, a physician
, physical, and neurologic exams
must rely on information obtained from parents and

normally the sensory


are normal.

--------------------------------------- 244
Chapter 4 / Development

45
TABLE 4-3

Diagnostic Criteria for Attention Deficit


a for Pervasive Developmental
Hyperactivity Disorder

Diagnostic Criteri

Symptoms of Inattention
ial Interactions
Failing to give attention to detail
behaviors
Difficulty completing tasks
ionships
Difficulty organizing activities
terest
Avoids activities that require sustained mental effort
reciprocity
Easily distracted by external forces
munication
Forgetful in daily activities

Impairments in Soc

Disorder

Lack of nonverbal
Lack of peer relat
Lack of showing in
Lack of emotional
Impairments in Com

Developmental lang
uage delay
Symptoms of Hyperactivity
a conversation with others
Fidgets and squirms
language
Unable to remain in position
y

Unable to sustain
Use of repetitive
Lack of social pla

Feelings of restlessness
Presence of Stereo
typical Behaviors
Unable to enjoy activities quietly
ce to rituals
Talks excessively
r mannerisms
Symptoms of Impulsivity
objects
Difficulty waiting turn

Inflexible adheren
Stereotypical moto
Preoccupation with

Adapted from Ameri


can Psychiatric Association. Diagnostic and
Interrupts others
statistical manual
of mental disorders, 4th ed. Washington, DC:
These symptoms should be present in two or more settings and resultAmerican Ps
ychiatric Association, 1994.
in impaired functioning. In addition, the symptoms must be present
prior to the patient reaching 7 years of age.
Adapted from American Psychiatric Association. Diagnostic and
statistical manual of mental disorders, 4th ed. Washington, DC: Ameri-given in
isolation, and at least once a year the patient
can Psychiatric Association, 1994.
deserves a trial off
medications.
Pervasive Developmen
tal Disorder
Management
tal disorder (PDD) represents
The treatment program for ADHD requires a multic nonprogressive developmental
disciplinary approach. Emotional supports should be
ng impairments in social interacmade available for the patient and parents. A behavand behavior. Autism is a form
ior management program must be developed to assist
in 2 to 6 children per 1000
both the parents and teachers with discipline. The
times more common in males.
patient's academic needs should be met; up to 25%
cause has been identified. Most
of children with ADHD will also have a learning disween 18 months and 3 years of
ability. Comorbid conditions are common and may
n be present from infancy
include aggression problems, oppositional defiant dis].
order, conduct problems, and mood disorders.

Pervasive developmen
a spectrum of chroni
disabilities involvi
tion, communication,
of PDD. FDD is seen
children and is four
No single underlying
children present bet
age, but symptoms ca
(impaired attachment

Clinical Manifestati
ons
Occasionally, pharmacologic treatments are necessary. First-line therapy consists of psycho-stimulants,
ve significant speech and lan-

Children with PDD ha

including methylphenidate and dex-troamphetablems with social interactions.


mine. They work by increasing the availability of
e contact, do not give reciprodopamine and norepinephrine in the CNS. Side
nd do not engage in pretend
effects include insomnia and anorexia; sometimes tics
any children have an attachment
and dyskinesias may develop. Seventy percent to 80%
unusual objects and may display
of patients will respond to the initial dose of these
or. The DSM-IV criteria are
medications. However, nonstimulant medications,

guage delays and pro


They have limited ey
cal communication, a
play. In addition, m
and fascination with
stereotypical behavi
listed in Table 4-3.

such as selective serotonin reuptake inhibitors and


clonidine, are sometimes necessary when the patient

Management

does not respond or cannot tolerate stimulant medlogic treatment available for
ication. Pharmacologic treatment should never be
ill benefit from medication

There is no pharmaco
PDD. Some children w

--------------------------------------- 245
46

Blueprints Pediatrics
TABLE 4-4
Secondary Sex Characteristics: Tanner
Breast Development
Stage I Preadolescent; elevation of papilla only

Stage II Breast bud; elevation of breast and papilla as small mound; enlargeme
nt of areolar diameter (11.15 + 1.10)
Stage III Further enlargement and elevation of breast and areola;no separation
of their contours (12.15 + 1.09)
Stage IV Projection of areola and papilla to form secondary mound above level
of breast (13.11 1.15)
Stage V Mature stage; projection of papilla only due to recession of areola to
general contour of breast (15.33 + 1.74)
Note: Stages IV and V may not be distinct in some patients
Genital Development (Male)
Stage I Preadolescent; testes, scrotum, and penis about same size and proporti
on as in early childhood
Stage II Enlargement of scrotum and testes, skin of scrotum reddens and change
s in texture; little or no enlargement
of penis (11.64 1.07)
Stage III Enlargement of penis, first mainly in length; further growth of test
es and scrotum (12.85 + 1.04)
Stage IV Increased size of penis with growth in breadth and development of gla
ns; further enlargement of testes and
scrotum and increased darkening of scrotal skin (13.77 + 1.02)

Stage V Genitalia adult in size and shape (14.92 1.10)


Pubic Hair (Male and Female)
Stage I Preadolescent; vellus over pubes no further developed than that over a
bdominal wall (i.e., no pubic hair)
Stage II Sparse growth of long, slightly pigmented downy hair, straight or onl
y slightly curled, chiefly at base of penis
or along labia (Male: 13.44 + 1.09. Female: 11.69 1.21)
Stage III Considerably darker, coarser and more curled; hair spreads sparsely
over junction of pubes (Male: 13.9 + 1.04.
Female: 12.36 1.10)
Stage IV Hair resembles adult in type; distribution still considerably smaller
than in adult. No spread to medial surface
of thighs (Male: 14.36 + 1.08. Female: 12.95 1.06)
Stage V Adult in quantity and type with distribution of the horizontal pattern
(Male: 15.18 + 1.07. Female: 14.41 +
1.12)
Stage VI Spread up linea alba:"male escutcheon"
designed to target specific symptoms such as impulAge (years)
sivity and hyperactivity. The mainstays of treatment
14 15 16 17
are behavioral therapy, improving communication,

9 10 11 12 13

and providing parental support. The best prognostic


indicator of future success is the extent of language
development present during the preschool years.
Height s
purt
Testicular volume
(cc)*
_ _S_EXU AL_DEVELp_PM_ENT ........
to 6 8 to 10 10 to 15 15 to 25
Adolescence refers to the passage from childhood to

Genitalia si
ze zm
adulthood, whereas puberty refers to those biologic
e)
changes that lead to reproductive capability. The

(Tanner stag

events of puberty occur in a predictable sequence,


Pubic hai
r
but the timing of the initiation and the velocity of
tage)
the changes are highly variable among individuals. The integration of the pubertal changes into

(Tanner s

the adolescent's self-concept is crucial to normal


1
12 13
14
15
16
17

10

Age (years)
adolescence.
In males, the initiation sequence of sexual develof pubertal events in the average
opment is testicular enlargement, followed by penile

Figure 4-1

Sequence

American male.

--------------------------------------- 246
Chapter 4 / Development 47
Age (years)
g system is used to determine
10
11
12 13
14
the pubertal process. Tanner stages

The Tanner stagin


15

16

17

where a child is in
for the male genital

ia, female breasts, and male and


female pubic hair ar
e shown in Table 4-4. Pubertal
abnormalities are ad
dressed in Chapter 6.
Height spurt
KE
Y POINTS
Menarche
velopmental assessments are

1. Two separate de
more predictive

than a single assessment, and


Breast 2m*m3
be performed in all areas of devel(Tanner stage)
s for delay, dissociation, or

testing should
opment to asses
deviancy.

Pubic hair BB i
gram for ADHD requires a mul(Tanner stage)
approach.

2. The treatment pro


tidisciplinary
3. Pervasive devel

opmental disorder (PDD) repre8 9 10 11 12 13 14 15 16 17


m of chronic nonprogressive
Age (years)
isabilities involving impairments

sents a spectru
developmental d
in social inter

action, communication, and behavFigure 4-2 Sequence of pubertal events in the average
form of PDD.
American female.
uberty occur in a predictable

ior. Autism is a
4. The events of p
sequence, but t

he timing of the initiation and the

velocity of the
changes are highly variable among
enlargement, height growth spurt, and pubic hair.
This progression is shown in Figure 4-1.
sequence of sexual development for
In females, the order of pubertal events in sexual
ular enlargement, penile enlargedevelopment is thelarche [breast buds), followed by
owth spurt, and pubic hair,
height growth spurt, pubic hair, and menarche.
uence for females is thelarche
Figure 4-2 illustrates these changes.
height growth spurt, pubic hair, and

individuals.
5. The initiation
males is testic
ment, height gr
whereas the seq
(breast buds),
menarche.

--------------------------------------- 247
Chapter
Dermatology
VIRAL EXANTHEMS
tnatally acquired rubella is con-

complications. Pos
firmed by serologi

c testing. The diagnosis of rubella is


Clinical Manifestations
cause the symptoms are mild and may

often difficult be
be confused with t

hose of enteroviral infection,


Certain viral exanthems are characteristic for partiosis, infectious mononucleosis, mild
cular viral illnesses.
Although measles is uncommon in developed
et fever.

roseola, toxoplasm
measles, and scarl
Roseola infantu

m is a common, acute disease


countries where vaccines are used, it continues to be
ng children caused by human hera major health problem worldwide. The incubation
. The illness begins with an abrupt
period is 8 to 12 days after initial exposure to
d by temperatures of 103 F to 106F
the paramyxovirus; there are no signs or symptoms
ersist for 1 to 5 days. During the
during this stage. A prodrome follows, consisting of
enerally appears well and has no physmalaise, high fever, cough, coryza, and conjunctivitis.
xplain the fever. Around the third or
Within 2 to 3 days of the onset of symptoms, Koplik's
ess, a maculopapular rash appears on
spots (small irregular red spots with central gray or
bluish white specks) appear on the buccal mucosa.
ads peripherally. The fever typically

of infants and you


pesvirus 6 (HHV-6)
fever characterize
(39.4-41.0C) that p
fever, the child g
ical findings to e
fourth day of illn
the trunk and spre
resolves as the ra

sh appears. Initially, leukocytosis up


About 5 days after the onset of symptoms, an eryeft shift may exist, but by the second
thematous maculopapular rash erupts on the head

to 20,000 with a l
day of illness, le

ukopenia and neutropenia may be


and spreads caudally, lasting about 4 to 5 days.
ns are uncommon, although febrile
Diagnosis is made by the distinctive history and
due to the rapid increase in temcharacteristic clinical findings; however, it may be
e onset of infection.
confirmed by serologic testing. Severe complications
iosum (fifth disease] is a mild,
include acute encephalitis, resulting in brain damage,
emic illness caused by the DNAand subacute sclerosing panencephalitis.
Rubella is generally innocuous when acquired postrus B19. It primarily occurs in

noted. Complicatio
seizures may occur
perature during th
Erythema infect
self-limited, syst
containing parvovi
epidemics. Usually

there is no prodrome, and fever


natally, but when a fetus is infected during gestation,
ow grade. The rash progresses
the results can be devastating. For details on congenies. It begins as a marked erythema
tal rubella, see Chapter 13. Rubella is caused by rubella
ch gives a "slapped cheek" appearvirus, an RNA togavirus. Clinical manifestations in
ous, pruritic, maculopapular rash
postnatally acquired rubella are absent in many cases.
arms and spreads to the trunk and
There is no prodrome during the incubation period of
age is characterized by fluctuations
14 to 21 days. When symptoms do occur, rubella is
the maculopapular rash and usually
characterized by an erythematous, maculopapular, discrete rash, with generalized lymphadenopathy and
. Fluctuations occur with tem-

may be absent or l
through three stag
of the cheeks, whi
ance. An erythemat
then starts on the
legs. The third st
in the severity of
lasts 2 to 3 weeks
perature changes a

nd exposure to sunlight. Comslight fever. The rash rarely lasts longer than 5 days.
arthritis, hemolytic anemia, and
Fever may accompany the onset of rash. Transient polrvovirus B19 infection during
yarthralgia and polyarthritis are common in adolesiated with fetal hydrops and death
cents. Encephalitis and thrombocytopenia are rare

plications include
encephalopathy. Pa
pregnancy is assoc
of the fetus.

--------------------------------------- 248
Chapter 5 / Dermatology
49
Hand-foot-and-mouth disease is a common acute
nied by fever and malaise. A
disease of young children during the spring and
hen appears in crops confined
summer caused by coxsackie A viruses. There is
stribution and clears in 7 to 14
usually a prodrome of fever, anorexia, and oral pain,
ast as long as 4 weeks, however,
followed by crops of ulcers on the tongue and orawitl
for weeks or months.
mucosa and a maculopapular vesicular rash on the

nerve and is accompa


vesicular eruption t
to the dermatomal di
days. The rash may l
h pain persisting
Other complicatio

ns from zoster include enhands, feet, and occasionally the buttocks. Diagnocephalopathy, ase
ptic meningitis, Guillain-Barre synsis is made by the history and the constellation odromef
, pneumonitis
, thrombocytopenic purpura,
symptoms.
cellulitis, and arth
ritis.
Varicella (chickenpox) is a highly contagious
disease caused by primary infection with varicellazoster virus. It is usually a mild, self-limited disease

Treatment

in normal children. Its severity can range from a few


es, treatment is mainly suplesions and a low-grade fever to hundreds of lesions

In uncomplicated cas

and a temperature up to 105F (40.6C). Fatal disd with acetaminophen or


seminated disease may occur in immunocompro. (Ibuprofen is contraindicated
mised children. After an incubation period of 10 to
spected, because of increased risk
21 days, there is a prodrome consisting of mild fever,
lulites.) Aspirin should be
malaise, anorexia, and occasionally a scarlatiniform
irin therapy for fever in the
or morbilliform rash. The characteristic pruritic rash
nfection is associated with Reye's
occurs the following day, appearing first on the trunk
g associated with fifth disease,
and then spreading peripherally. The rash begins as
s zoster is treated with an antired papules and develops rapidly into clear vesicles
. During chickenpox, daily
that are about 1 to 2mm in diameter. The vesicles
water reduces the risk of secthen become cloudy, break, and form scabs. The
ection. Herpes zoster can be quite
lesions occur in widely scattered "crops," so there are
cs are sometimes needed.
usually several stages of lesions present at the same
ildren who are exposed to
time. Vesicles often occur on mucous membranes.
la-zoster virus infection are
Patients are infectious from 24 hours before the
er immune globulin within 96

portive. Fever is treate


ibuprofen and fluids
when varicella is su
of streptococcal cel
avoided, because asp
setting of a viral i
syndrome. The itchin
varicella, and herpe
histamine medication
bathing in lukewarm
ondary bacterial inf
painful, and narcoti
Immunocompromised ch
someone with varicel
given varicella-zost
hours of the exposur

e and observed closely. Acyclovir


appearance of the rash until all the lesions are
treatment of both varicella and
crusted, which usually occurs 1 week after the onset
ndicated in immunocompromised
of the rash.
ovir may be considered for use in
Chickenpox is a clinical diagnosis. In unclear cases,
12 years, children with chronic
a Tzanck prep, looking for multinucleated giant cells,
ho have received steroids for any
can be performed on a vesicle, or a pharyngeal swab

is effective in the
zoster; its use is i
patients. Oral acycl
patients older than
disease, and those w
reason. Administrati

on of the varicella vaccine within


or swab of vesicular fluid can be sent for viral culture.
may prevent or lessen disease.
Alternatively, acute and convalescent sera can be
ailable for the prevention of
tested for a fourfold increase in antibody titer. Progressive varicella with meningoencephalitis, hepatitis,
d varicella (see Chapter 12).
and pneumonitis may occur in immunocompromised

72 hours of exposure
Immunizations are av

measles, rubella, an

children and is associated with a 20% mortality rate.


Immunization with varicella vaccine has reduced the
frequency of this infection in the United States.
Zoster (shingles) represents a reactivation of variY POINTS
cella-zoster virus infection and occurs predominantly
are generally benign and are
in adults who previously have had varicella and have
atically.
circulating antibodies. After chickenpox, varicellare differentiated by history and
zoster virus retreats to the dorsal root ganglion; as a
.
result, it follows a dermatomal distribution when
hickenpox are contagious from 24
reactivated. Although herpes occurs in children, it is
e onset of rash until all lesions
uncommon in those younger than 10 years. An attack
er.
of zoster begins with pain along the affected sensory

KE
1. Viral exanthems
treated symptom
2. The exanthems a
rash appearance
3. Children with c
hours before th
have crusted ov

--------------------------------------- 249
Chapter 1 / Emergency Management: Evaluation of the Crit
ically III or Injured Child
KEY POINTS
rates and multiorgan
system dysfunction results.
When this process ha
s caused irreparable functional
1. No matter what the cause of cardiorespiratory
loss in essential or
gans, the terminal or irreversible
arrest, the algorithms outlined for pediatric basic
stage of shock is re
ached.
and advanced cardiac life support should be
The types of shoc
k include hypovolemic, cardiofollowed. A primary survey (Airway, Breathing,
genic, distributive,
and septic. Hypovolemic shock
Circulation, Disability, Exposure) is followed by a
results from decreas
ed intravascular volume, which
secondary survey.
results in decreased
venous return and myocardial
2. Approximately half of the causes of pediatric
preload. Because of
the reduction in myocardial

arrest are due to respiratory arrest, which can be


resultant decrease in stroke
brought about by upper airway obstruction, lower
ut, and blood pressure. This is
airway obstruction, restrictive lung disease, or an
logy of shock in children.
etiology that results in inadequate gas exchange.
is the result of "pump failure."
3. The CPR algorithm is summarized in Figure 1-4.
lume results in diminished
4. If resuscitation does not establish cardiac output,
ypotension.
the following mechanical or metabolic causes
k results from an abnormality in
should be investigated: hypothermia, tension
leads to maldistribution of a
pneumothorax, hemothorax, cardiac tamponade,
olume and a state of relative
profound hypovolemia, profound metabolic
of peripheral pooling, preload
imbalance, toxin ingestion, and closed head injury.
a decrease in stroke volume,

preload, there is a
volume, cardiac outp
the most common etio
Cardiogenic shock
Inadequate stroke vo
cardiac output and h
Distributive shoc
vasomotor tone that
normal circulatory v
hypovolemia. Because
is reduced, causing
cardiac output, and

blood pressure. Systemic vascular resistance is al


so decreased due to vasomotor dysfunction. Because bo
th systemic vascular resistance
and cardiac output a
re reduced, severe hypotension
SHOCK
lts when certain pathogens infect
Shock is a syndrome characterized by the inability of
compensated stage of septic
the circulatory system to provide adequate nutrients
ed by decreased vascular resisto meet the body's metabolic demands. Children,
shock), whereas in the late
especially neonates, will initially try to compenhypovolemia from third
sate by becoming tachycardic. Hypotension, a late
lure due to myocardial depresfinding, leads to cellular hypoperfusion, metabolic
parent. Compensated septic
acidosis, and cellular death. Three relationships
m" sepsis, and uncompensated
explain hypotension in shock:
rred to as "cold" sepsis.

results.
Septic shock resu
the blood. The early
shock is characteriz
tance (distributive
uncompensated phase,
spacing and pump fai
sion becomes more ap
shock is called "war
septic shock is refe
The most common e

tiologies for each type of


Blood pressure (cardiac output x systemic vascuble 1-3.
lar resistance)
Cardiac output (stroke volume x heart rate)

shock are listed in Ta

Clinical Manifestati
ons
Stroke volume (determined by preload [ventricular end diastolic volume], afterload
Examination

History and Physical

[systemic vascular resistance], and myocardial


ocus on potential causes. Concontractility)
ock if there is a history of vom-

The history should f


sider hypovolemic sh
iting, diarrhea, pol

yuria, burns, trauma, surgery,


The three stages of shock are compensated,
eding, intestinal obstruction, long
uncompensated, and irreversible. In the compensated
or pancreatitis. A history of constage, homeostatic mechanisms maintain essential
e, arrhythmias, or chemotherapy
organ perfusion. Blood pressure, urine output, and
tration may indicate cardiocardiac function all seem to be normal. In the uncomutive shock should be contempensated stage, homeostatic mechanisms fail because
a history of toxic ingestion,
of ischemia, endothelial injury, and the elaboration of
or spinal cord injury. In additoxic materials. Eventually, cellular function deterioromised patient who presents

gastrointestinal ble
periods in the sun,
genital heart diseas
(Adriamycin) adminis
genic shock. Distrib
plated when there is
anaphylaxis, or head
tion, any immunocomp

--------------------------------------- 250
50

Blueprints Pediatrics
tion. These bacteria

are normal flora of the skin, and


BACTERIAL RASHES
ument allows entry into the
Bacterial rashes of the skin are common and are in
. The location of the infection
most cases the result of group A beta-hemolytic
e in rare cases the cellulitis may
streptococcal or Staphylococcus aureus infection.
ying osteomyelitis, septic arthri-

a break in the integ


dermis and epidermis
is important, becaus
arise from an underl
tis, sinusitis, or d

eep wound infection. Before the use


Clinical Manifestations
enzae type b (Hib) vaccine, H.

of Haemophilus influ
influenzas type b wa

s a significant pathogen resulting


Bullous impetigo, which is caused by S. aureus,
lulitis by hematogenous spread.
begins as red macules that progress to bullous (fluidcellulitis is now rarely seen.
filled) eruptions on an erythematous base. These
ccus pneumoniae is the most comlesions range from a few millimeters to a few cenenously spread cellulitis.
timeters in diameter. After the bullae rupture, a clear,
d S. pneumoniae often affects
thin, varnish-like coating forms over the denuded
ital area. Cellulitis of the face,
area. S. aureus can be cultured from the vesicular
it results from trauma or
fluid. Bullous impetigo lesions can be mistaken for
can result from all the

in many cases of cel


H. influenzae type b
Currently, Streptoco
mon cause of hematog
Hematogenously sprea
the face and periorb
depending on whether
hematogenous spread,

cigarette burns, raising the suspicion of abuse.


group A beta-hemolytic strepNonbullous impetigo, which is caused by both
S. pneumoniae, or H. influenzae
group A beta-hemolytic streptococci and S. aureus,
begins as papules that progress to vesicles and then
to painless pustules measuring about 5mm in
diameter with a thin erythematous rim. The pustules
rupture, revealing a honey-colored thin exudate
mpetigo can be treated topically
that then forms a crust over a shallow ulcerated base.
ent. Bullous impetigo and
Local lymphadenopathy is common with streptoif the lesions are numerous, are
coccal impetigo. Fever is uncommon. The causative
-generation cephalosporin such as
organism can usually be isolated from the lesions.
drug that is effective against
Staphylococcal scalded skin syndrome, which is
nd group A streptococci. The
caused by exfoliative isolates of S. aureus, is most
any honey-colored crusts with
common in infancy and rarely occurs beyond age 5.
presses.
Onset is abrupt, with diffuse erythema, marked skin
cases of Staphylococcal scalded
tenderness, and fever. Within 12 to 24 hours of onsetski,
th an oral antistaphylococcal
superficial flaccid bullae develop and then rupture
with severe cases should be
almost immediately, leaving a beefy red, weeping
ey had a second-degree burn,
surface. Exfoliation is caused by a toxin and may
d management and intravenous
affect most of the body, and there is usually a posiycin.
tive Nikolsky's sign (separation of the epidermis on
iculitis responds to aggressive hylight rubbing). The initial focus of Staphylococcal
pirocin, whereas folliculitis of
infection may be minor or not apparent. Unruptured
usually recalcitrant and needs an
bullae contain sterile fluid.
cal drug. Simple furunculosis is
Folliculitis is an infection of the shaft of the hair
eat. Larger and deeper furuncles
follicle. Superficial folliculitis is common and easily
ed and drained. After drainage,
treated. Deep forms of this infection include furunal mupirocin treatment.
cles (boils) and carbuncles. Furuncles begin as superld cellulitis can be treated
ficial folliculitis and are most frequently found in
tic, such as cephalexin or amoxiareas of hair-bearing skin that are subject to friction
id. Those with severe infection
and maceration, especially the scalp, buttocks, and
c streaking or lymphadenopaaxillae. Carbuncles are an accumulation of furuncles.
zed and given a parenteral

pathogens mentioned:
tococci, S. aureus,
type b.
Treatment
Limited nonbullous i
with mupirocin ointm
nonbullous impetigo,
treated with a first
cephalexin, an oral
both staphylococci a
caretaker can remove
twice-daily cool com
Mild to moderate
n are treated wi
medication. Children
treated as though th
with meticulous flui
oxacillin or clindam
Superficial foll
giene and topical mu
the male beard is un
oral antistaphylococ
treated with moist h
may need to be incis
they need only topic
Children with mi
with an oral antibio
cillin-clavulanic ac
who have lymphangiti
thy may be hospitali

Cellulitis is a localized, acute inflammation of the


or periorbital cellulitis (see Chapter
skin characterized by erythema, pain, and warmth.
ted with intravenous ampicillinCellulitis in children is most often caused by group
sion to the hospital for observaA beta-hemolytic streptococcal or S. aureus infecr periorbital cellulitis is present

antibiotic. Facial
18) usually is trea
sulbactam and admis
tion. When orbital o

--------------------------------------- 251
Chapter 5 / Dermatology
51
or a peripheral skin cellulitis results in lymphadypopigmented oval scaly lesions
enopathy or lymphangitic streaking, a blood culturoe
part of the back, chest, and
should be sent to determine whether bacteremia iproximas
Christmas tree distribution.
present.
uals tend to have hypopig-

superficial tan or h
n the neck, upper
l arms in a
Dark-skinned individ
mented lesions durin

g the summer, when uninfected


skin darkens from su
nlight exposure. Treatment is
KEY POINTS
e shampoo.

with selenium sulfid


Diaper rash may r

esult from atopic dermatitis,


1. S. aureus and group A beta-hemolytic streptomatitis, or primary or secondary
cocci cause most bacterial skin infections.
2. Because of the Hib vaccine, S. pneumoniae has
ection. Eighty percent of diaper
replaced H. influenzae as the most common
than 4 days are colonized with
pathogen in hematogenously spread cellulitis.
apular lesions with peripheral
3. The child with peripheral cellulitis with lymolds and satellite lesions are typical
phadenopathy or lymphangitic streaking and the
rash. Topical nystatin is the firstchild with orbital or periorbital cellulitis should
oice.
have a blood culture sent to determine whether

primary irritant der

Candida albicans inf


rashes lasting more
Candida. Fiery red p
scales in the skin f
for candidal diaper
line treatment of ch

bacteremia is present.
JP..ACNE....
Pathogenesis
.AUPERF[CJAL_FyNGAL_RASH_ES___.__
d by enlargement of sebaceous

Acne vulgaris is cause


glands, increased se

bum production, proliferation of


Essentially three fungal organisms cause superficial

Propionibacterium ac

nes, and secondary inflammatory


tinea infections: Trichophyton, Microsporum, and
predilection for face, chest, and
Epidermophyton. Tinea infections and their treatss from comedones (white-

changes. There is a
back. Lesions progre
heads), to open come

dones [blackheads), to pustules,


ments are discussed in Table 5-1.
es (cysts), and finally to atrophic
Tinea versicolor, another type of yeast infection
rs. Androgens are the stimulus
caused by Malassezia furfur, is characterized by
development and secretion.

to papules, to nodul
and hypertrophic sca
for sebaceous gland
At puberty, hormonal

stimuli lead to increased


growth and developme
nt of sebaceous follicles.
TABLE 5-1
severe acne often have high

Female patients with


levels of circulatin

g androgens.
Common Tinea Infections and Their Treatments
Infection

Treatment
Epidemiology

Tinea capitis (scalp)

Oral griseofulvin,4-6

weeks
n, self-limited, multifactorial
Selenium sulfide shampoo
ceous follicles, noted during the
to decrease infectivity;
ns may begin as early as 8 to 10
does not eradicate
ence increases steadily throughinfection
then decreases in adulthood.
Tinea corporis (body)
Topical antifungals (e.g.,
develop acne at a younger age
"ringworm"
clotrimazole) for at
disease affects boys 10 times
least 4 weeks; oral
use of higher androgen levels.
griseofulvin if refractory
teenage boys have severe acne.
Tinea cruris (genitocrural) Same as tinea corporis
"jock itch"
Tinea pedis (foot)

Acne is a very commo


disorder of the seba
teenage years. Lesio
years of age. Preval
out adolescence and
Although girls often
than boys do, severe
more frequently beca
In fact, 15% of all

Risk Factors
Same as tinea corporis,

"athlete's foot"
plus proper foot
male gender, puberty, oily comhygiene
yndrome, or any other process

Risk factors include


plexion, Cushing's s
that results in incr

eased androgens.

--------------------------------------- 252
52
Blueprints Pediatrics
Clinical Manifestations
ptives with low levels of andro-

cases, oral contrace


gens may also be hel

pful by suppressing androgen


History
It is important to determine when the acne started

production.
To maximize the t

herapeutic benefits, combinaand whether there is a family history of acne. A full


lly prescribed. Mild acne with few
menstrual history should be taken to determine
with benzoyl peroxide and
whether there is a correlation between the onset of
Mild acne generally responds to
menses and the patient's acne exacerbations. It is also
ring.
important to discuss the patient's skin care, includ-

tion therapy is usua


comedones is treated
topical antibiotics.
therapy without scar

Many comedones an
d some papules and pustules
ing how the patient's acne has been treated in the
f moderate acne. Therapy includes
past. Many drugs cause acne. Corticosteroids, androetinoin, and topical or oral
gens, danazol, iodides, and bromides often exacerbate
s a variable response to treatment,
acne. Other possible stimuli include isoniazid,
ssibility with this severity of acne.
lithium, halothane, vitamin B
]2, and hyperalimenta-

are characteristic o
benzoyl peroxide, tr
antibiotics. There i
and scarring is a po

Severe acne is ch
aracterized by inflammatory
tion. These drugs are not directly comedogenic but
ysts, abscesses, and scarring.
"prime" the follicular epithelium to the comedogenic
f topical therapy and sebaeffects of sebum.
ive agents, including estrogens,

papules, pustules, c
Treatment consists o
ceous gland-suppress
steroids, and retino

ic acid (Accutane). Because of its


Physical Examination
teratogenicity, a ne
gative pregnancy test must be
Distribution, morphology, and severity of lesions
eks of initiating retinoic acid
should be recorded. It is important to differentiate
eption must be used from 1
common acne from nodulocystic acne.
nth after therapy. Accutane

obtained within 2 we
therapy, and contrac
month before to 1 mo
therapy usually last

s 4 to 5 months.
Differential Diagnosis

The differential diagnosis for acneiform rashes


includes acne vulgaris, drug-induced acne, Cushing's
KEY POINT
syndrome or other pathologies that increase endogenous steroid secretion, and perioral dermatitis.
way to treat acne; combination

1. There is no one
therapy works b

est.
Rosacea, an acneiform eruption of the central face
and neck, is sometimes confused with acne, but it is
primarily seen in adults.
Treatment

PSORIASIS

Treatment should be individualized depending on

Pathogenesis

the patient's gender and the severity, type, and distribution of lesions.
The pathogenesis of
psoriasis is unknown. A multiBenzoyl peroxide works by decreasing the colofactorial inheritan
ce pattern has been proposed.
nization of P. acnes and decreasing the developmenChildret
n with HLA
type C6 are clearly more
of microcomedomes by lessening the concentration
likely to develop th
e disease. Histologically, there is
of surface free fatty acids. Topical retinoids (e.g.,
hyperproliferation o
f the epidermis, and epidermal
Retin-A) have strong anticomedogenic activity;
turnover time is not
ed to be distinctly accelerated
however, side effects may limit use and include
in those affected. T
he rash usually appears at sites of
dryness, burning, and, most important, photosensiphysical, thermal,
or mechanical trauma. This is
tivity. The use of sunscreen with a protective factor
known as the Kobner
phenomenon, a diagnostic
(SPF) of at least 15 is necessary. Topical and systemifeaturc
e of the dise
ase.
antibiotics are used to prevent and decrease colonization of P. acnes. Topical antibiotics are also available in combination with benzoyl peroxide. The

Epidemiology

systemic antibiotics used include tetracycline, doxyred by some to be an adult disease,


cycline, minocycline, and erythromycin. In some
in before the age of 10, and 35%

Psoriasis is conside

--------------------------------------- 253
Chapter 5 / Dermatology

53

but 10% of cases beg

before the age of 20. Fifty percent of children with

Treatment

psoriasis have a positive family history for the disease.


If psoriasis is present during adolescence, it is likely
ma, is characterized by remissions
a lifelong disease.
he most important aspect of

Psoriasis, like ecze


and exacerbations. T
treating psoriasis i

s to educate the patient and family


that the disease is
a recurrent one that cannot be
Risk Factors
trolled with conscientious

cured but can be con

therapy. No matter w
here the rash is or its severity,
HLA inheritance is part of the mode of transmission;
the goal of psoriasi
s therapy is to keep the skin well
therefore, a positive family history is a significant risk hydrated. Tar prepar
ations may be added to the daily
factor.
bath or used as an o
intment. For more severe cases,
natural sunlight or
ultraviolet B (UVB) light may be
used in conjunction
with the tar lubricant. For small
Clinical Manifestations
areas of involvement
, fiuorinated steroids may be
History and Physical Examination
successful; the leas
t potent but effective dose should
be used, because adr
enal suppression can occur.
The nonpruritic rash consists of erythematous
papules that coalesce to form plaques with sharply
demarcated borders and a silvery or yellow-white
scale. The scales tend to build up into layers, and their
EY POINTS
removal may result in pinpoint bleeding (Auspitz's
sign). The rash is usually symmetric, with plaques
t be cured and is characterized by
appearing over the knees, elbows, scalp, and genital
exacerbations that can be conarea. These are sites of repeated trauma. The scalp
nscientious therapy.
frequently has a thick, adherent scale with alopecia
s at skin points of repeated trauma,

1. Psoriasis canno
remissions and
trolled with co
2. Psoriasis occur
and the rash is

nonpruritic.
at sites of involvement. The nails often demonstrate
sts of keeping the skin well
punctate stippling or pitting, detachment of the nail
plate (onycholysis), and accumulation of subungual
ar preparations that help hold
debris. Examination of the palms and soles reveals
skin.

3. Treatment consi

hydrated with t
moisture in the

scaling and fissuring. Psoriatic arthritis may also be


present.
Differential Diagnosis

ALLERGIC RASHES

The differential diagnosis for a psoriatic rash in chilczema)


dren includes uncommon disorders such as Reiter's

Atopic Dermatitis (E

syndrome, pityriasis rubra pilaris, and lichen planus.


czema) is a common skin disorReiter's syndrome, in contrast to simple psoriasis, has
hildhood and affects 5% of chil-

Atopic dermatitis (e
der of infancy and c
dren before the age

of 5. Seventy percent of affected


a psoriatic-like rash that involves the mucous memdegree relatives exhibiting some
branes. In some severe cases in which the rash is
ease, and 30% to 50% of children
also accompanied by arthritis, the lesions of the
is go on to develop allergic rhinimucous membrane are the main differentiating point
ximately 60% of affected chilbetween psoriasis and Reiter's syndrome. Occasiondermatitis within the first year
ally, atopic dermatitis may be confused with psoria-

children have firstform of allergic dis


with atopic dermatit
tis or asthma. Appro
dren develop atopic

sis; however, eczema is pruritic and psoriasis is not.


hin the first 5 years of life.
Scalp lesions may be confused with seborrheic

of life, and 90% wit

dermatitis or tinea capitis.


ons

Clinical Manifestati
History and Physica

l Examination
The rash is characte
rized by erythema, edema,
papules, and weeping
in the active phase. Scales and
Diagnostic Evaluation
develop later. Severe pruritus is
The diagnosis is a clinical one. Skin biopsy reveals a
ma. The itching is a constant
hyperplastic epidermis.
an "itch-scratch-itch cycle." If

lichenification may
the hallmark of ecze
feature that creates

--------------------------------------- 254
Blueprints Pediatrics
54
there is no pruritus, it is unlikely that the rash is
xyzine), emollients (Eucerin
atopic dermatitis. Cellulitis can often be superimsteroids or other immunomoduposed on a base of eczema. S. aureus and Staphyloolimus).
coccus pyogenes are the usual bacterial agents. Herpes
simplex infection can also complicate atopic der-

antipruritics (hydro
cream), and topical
lators (topical tacr

matitis, leading to a
herpeticum.
the most common type
The three clinical
ction in the skin and

diffuse eruption as eczema

Urticaria
Urticaria (hives) is

of
phases are as follows:
affects up to

hypersensitivity rea
20% of children at s

ome time. It is IgE mediated


Phase I infantile eczema (2 months-2 years):
duction of an agent to which
Rash appears on the face, neck, scalp, trunk, and
s been previously sensitized.
extensor surfaces of extremities and progresses to
une-mediated urticaria
phase II in one-third of patients.
illin), food (fish, eggs, peanuts,
Phase II childhood eczema (2-10 years): Rash is
tors (cold, light, heat), blood
present on flexor surfaces predominantly (antecuand infections (Epstein-Barr
bital, popliteal, neck, wrists, sometimes hands and
feet), and one-third progress to adolescent eczema.
reptococcal pharyngitis). NonimPhase III adolescent eczema: Hands (mostly),
occur after first exposure to
eyelids, neck, feet, and flexor areas have rash.
in, opiates, or contrast media.
Atopic dermatitis tends to remit and exacerbate.
ons

and results from reintro


the immune system ha
Common causes of imm
include drugs (penic
chocolate), physical fac
and blood products,
virus, hepatitis, st
munologic urticaria can
such agents as aspir
Clinical Manifestati
An urticarial rash c

onsists of wheals, which are


Typically, the eruptions become milder with age, and
ruritic areas of edema of the
longer remissions occur. Triggers may include excessh evolves over several hours
sive bathing and hand washing, occlusive clothing
le day. The diagnosis is clinical
(especially wool), sweating, stress, and possibly food
eristic appearance and, when
allergy (eggs, milk, seafood, nuts, wheat, or soy).
of exposure. The presence of con-

raised, pale, pink p


upper dermis. The ra
or perhaps in a sing
and based on charact
possible, a history
current arthritis an

d fever suggests the diagnosis of


Differential Diagnosis
Some of the more common rashes that must be
differentiated from eczema include seborrheic dermatitis, diaper dermatitis, contact dermatitis, scabies,
tating exposure is the key to prepsoriasis, drug reactions, fungal infections, and
ichthyosis vulgaris.
sses can be applied to pruritic
Eczematous lesions are not exclusively due to
may be given antihistamines
atopic dermatitis, because a variety of immunodefiic medication may be used to
ciencies can cause similar rashes. These include
arthralgias or arthritis can be
Wiskott-Aldrich syndrome, agammaglobulinemia,
en.
Leiner's disease (C5 deficiency), and histiocytosis X.
Treatment

serum sickness.
Treatment
Avoiding the precipi
vention. Cold compre
areas, and the child
by mouth. Antiprurit
relieve itching, and
treated with ibuprof
Erythema Multiforme

The most important aspect of treating eczema is to


is an acute, self-limited,
interrupt the itch-scratch cycle. The families need to
ction that is uncommon in
be educated that the disease is a recurrent one that
ologic agents include viral

Erythema multiforme
hypersensitivity rea
children. Common eti
infection (herpesvir

us, adenovirus, Epstein-Barr


cannot be cured but can be controlled with conscineumoniae infection, drug ingesentious therapy. Therapy is directed at controlling
fa drugs), immunizations, and food
dryness, inflammation, and pruritus. General measures include avoiding extremes of temperature and
humidity, chemicals, strong soaps, certain foods,
ons
wool, and synthetic materials.
Severe atopic dermatitis is treated with wet
me, there is a symmetric
compresses soaked in aluminum acetate solution
ons evolving through multiple
(Burrow's solution), oatmeal baths (Aveeno bath),
erythematous macules, papules,

virus), Mycoplasma p
tion (especially sul
reactions.
Clinical Manifestati
In erythema multifor
distribution of lesi
morphologic stages:

--------------------------------------- 255
55
Chapter 5 / Dermatology
plaques, vesicles, and target lesions. The lesioncaines
, diphenhydram
ine, and Maalox (aluminum
change over days, not hours. Erythema multiformhydroxidee
, magnes
ium hydroxide) are comforting.
tends to occur over the dorsum of the hands and feetBecaus,
e corneal ul
ceration, keratitis, uveitis, and
palms and soles, and extensor surfaces of extremitiespanophthalmiti,
s
are possible, an ophthalmology
but may spread to the trunk. Burning and itching arconsultatioe
n is re
commended.
common. Systemic manifestations include fever,
Children with tox
ic epidermal necrolysis are
malaise, and myalgias.
treated as though th
ey had a full-body second-degree
Stevens-Johnson syndrome is the most severe
burn. Fluid therapy
and reverse barrier isolation are
form of erythema multiforme. There is a prodrome
critical to survival
.
for 1 to 14 days of fever, malaise, myalgias, arthralgias, arthritis, headache, emesis, and diarrhea. This is
followed by sudden onset of high fever, erythema
Y POINTS
multiforme skin lesions, and inflammatory bullae of

KE

1. Allergic rashes
are a spectrum of hypersensitivity
two or more mucous membranes (oral mucosa, lips,
ning in severity from urticaria to

reactions worse

bulbar conjunctiva, and anogenital area). In the most


severe cases, involvement of most of the gastroinorme to Stevens-Johnson syntestinal, respiratory, or genitourinary tracts may be
epidermal necrolysis.
seen. Untreated, this syndrome has a mortality rate
onic disease that cannot be cured

erythema multif
drome to toxic
2. Eczema is a chr
but in which re

missions and exacerbations can be


of approximately 10%.
conscientious therapy directed at
Toxic epidermal necrolysis is the most severe form
ch-scratch cycle.
of cutaneous hypersensitivity, considered by some to

controlled with
stopping the it

be a variant of Stevens-Johnson syndrome. Although


e most common type of hypersensiits occurrence in children is rare, it is associated
in the skin and affects one in five
with a 30% mortality rate. The pathogenesis is not

3. Urticaria is th
tivity reaction
children.
4. Stevens-Johnson

syndrome is erythema multiwell understood, but most cases are secondary to


mucosal bullae, whereas toxic
medications, especially sulfa drugs, anticonvulsants,

forme with oral

and nonsteroidal anti-inflammatory agents. Onset is


lysis is similar to staphylococcal
acute, with high fever, a burning sensation of the
that both result in sloughing of
mucous membranes, and/or oral and conjunctival
ayer.
erythema and erosions. The presentation of the skin

epidermal necro
scalded skin in
the epidermal l

resembles that of staphylococcal scalded skin, with


widespread erythema, tenderness, blister formation,
and detachment of the epidermis causing denuda
IONS
tion (positive Nikolsky's sign). Mucous membrane

_-

involvement is severe and the nails may be shed. SysWitof melanoma increasing, it is very
temic complications include elevated liver enzymesimportan,
tify suspicious lesions and underrenal failure, and fluid and electrolyte imbalancestan.
Children with fair skin, excessive
Sepsis and shock are frequent causes of death.
sun
ltiple nevi are at increased risk
for

HYPERPIGMENTED LES

h the incidence
t to iden
d risk factors.
exposure, and mu
skin cancer.

Treatment
For uncomplicated erythema multiforme, symptomatic treatment and reassurance are all that is

Congenital Nevi

necessary. Oral antihistamines, moist compresses, anCongenitad


l nevi a
re classified based on their size.
oatmeal baths are helpful. The lesions resolve over a
Large or giant nevi
are greater than 20 cm, small nevi
1- to 3-week period, with some hyperpigmentationar.
e less than 2 cm,
and intermediate nevi are between
The use of corticosteroids is controversial.
2 and 20cm. Research
has shown that there is an illTreatment of the patient with Stevens-Johnsodefinen
d but increas
ed risk of melanoma in patients
syndrome includes hospitalization with barrier isolawith congenital nev
i. Congenital nevi must be foltion, fluid and electrolyte support, the treatmenlowet
d annually for
changes and may require comof common secondary infection of the skin, moisplett
e excision. Gia
nt nevi have an increased risk of
compresses on bullae, and colloidal baths. For oramelanoml
a (betwee
n 5% and 15%). There is also an
mucosal lesions, mouthwashes with viscous lidoassociation with n
eurocutaneous melanosis, so

--------------------------------------- 256
56
Blueprints Pediatrics
patients with lesions over the head and spine require
an MRI to evaluate for CNS involvement.

Prevention
A large amount of ch

ildhood sun exposure and


frequent sunburns ar
e associated with increased risk
Common Acquired Nevi
of moles and skin cancer. Sun

for the development


protection with a su

nblock having an SPF of 15 or


Many children will go on to develop nevi, reaching a
UVA light is recommended.
maximum number in early adulthood. Patients with
more than 15 common acquired moles have an
increased risk for melanoma in the future. Moles
need to be assessed by using the ABCDs. Moles with
Y POINTS
asymmetry, irregular borders, variations in color, and
diameter larger than 6 mm have atypical features and
e assessed for asymmetry, irregu-

more against UVB and

KE
1. Moles need to b
lar borders, co

lor, and size.


may require excision.
t UVB and UVA light is required to
A Spitz nevus is a smooth pink to brown domesk of melanoma.
shaped papule. These nevi are benign, but may need
to be removed if they grow rapidly. A halo nevus is
a mole with a depigmented ring around it. These
lesions are benign, but are associated with the presence of vitiligo or melanoma at another site.

2. Sunblock agains
decrease the ri

--------------------------------------- 257
Chapter
DIABETES MELLITUS
oma and death. The most common

may progress to c
cause of DKA in t

he known diabetic is inadequate


Insulin-Dependent Diabetes
he condition can also be triggered
Mellitus (Type 1)
ance, which is exacerbated by

insulin dosing. T
by insulin resist
an intercurrent i

llness or extreme physiologic


Pathogenesis
y, new-onset diabetics present
Diabetes mellitus is a chronic metabolic disorder
severe complication of DKA
characterized by hyperglycemia and abnormal
ebral edema.
energy metabolism due to absent or diminished
DKA, the other major complication
insulin secretion. Insulin-dependent diabetes melypoglycemia from insulin overlitus (IDDM) type 1 results from lack of insulin
aloric intake, or increased exercise
production in the B cells of the pancreas. Although
tant increase in calories.
the precise etiology of IDDM is unknown, genetic,
autoimmune, and environmental factors have all
been implicated.
Risk Factors
After 90% of B-cell function has been destroyed,
common endocrine disease in
loss of insulin secretion becomes clinically significant.
ing in 1 in 500 children and
With the loss of insulin, the major anabolic hormone,
main risk factor for IDDM is a
a catabolic state develops, which decreases glucose
utilization and increases glucose production by
he presence of DR3 and DR4 major

stress. Frequentl
in DKA. The most
management is cer
In addition to
seen in IDDM is h
dose, decreased c
without a concomi

Epidemiology and
IDDM is the most
childhood, occurr
adolescents. The
family history. T
histocompatibilit

y antigens increases the lifetime


gluconeogenesis and glycogenolysis. The lack of
idual developing IDDM, as does
insulin prevents glucose from entering the cell, and
gree relative with IDDM. There is a
hyperglycemia results. The production of ketoacids
mong identical twins. The presis brought about by an increase in the catabolic
mediators glucagon, epinephrine, growth hormone
t cell antibodies in 85% of individu-

risk for an indiv


having a first-de
50% concordance a
ence of anti-isle
als with recent-o

nset IDDM and the increased


(GH), and cortisol. These messengers trigger lipolyer autoimmune diseases in children
sis, fatty acid release, and ketoacid synthesis. When
e case for an autoimmune
the blood glucose concentration exceeds 180 mg/dL,
ironmental role in disease pathe resultant glycosuria causes an osmotic diuresis

appearance of oth
with IDDM make th
etiology. The env

with increased urine output (polyuria). If insulin


s unclear. No particular virus has

thogenesis remain

been determined t
o be directly responsible.
deficiency is severe, ketones are produced in
significant quantities, the blood's native buffering
capacity is overwhelmed, and diabetic ketoacidosis
(DKA) results.
Clinical Manifest
ations
DKA is characterized by hyperglycemia, metaHistory and Physi
cal Examination
bolic acidosis (ketoacidosis), dehydration, and leA history of newonset weight loss, polydipsia,
thargy. It is a medical emergency that, in severe casespolyphagia,
, an
d polyuria is consistent with type

--------------------------------------- 258
58

Blueprints Pediatrics

1 diabetes mellitus. The physical examination is


suspected DKA, the serum
generally normal in type 1 diabetes mellitus unless
n is grossly elevated, and the
DKA is present.
PCO
2 are low. Metabolic
When DKA is suspected in a child with known
s results in diminished pH, and
IDDM, important historic information includes the
bolic acidosis is a compensatory
usual insulin dose, the last insulin dose, the child's
s and a drop in serum PCO
2.
diet over the previous day, and whether the child
ic diuresis, blood urea nitrogen
has been ill and emotionally or physically stressed.
e is loss of phosphate, calcium,
The child with DKA appears acutely ill and suffers
ugh there is a total body loss of
from moderate to profound dehydration. Symptoms
assium may be low, normal, or
include polyuria, polydipsia, fatigue, headache,
on the level of acidosis. When
nausea, emesis, and abdominal pain. The child's
protons move from the extracelmental status may vary from confused to comatose.
ntracellular space and potassium
On physical examination, tachycardia and hyperpnea
cellular space to the extracellu(Kussmaul respirations) are generally noted. There
n electroneutrality. Until the
may be a fruity odor to the breath because of
eversed with insulin, the urine is
the ketosis. Intravascular volume depletion may be so
; until the serum concentration of
marked that hypotension may be detected. Although
180mg/dL, the urine is positive
cerebral edema is rare, it often is fatal. Changing

In children with
glucose concentratio
venous pH and serum
acidosis from ketosi
the response to meta
respiratory alkalosi
Because of the osmot
is elevated and ther
and potassium. Altho
potassium, serum pot
even high depending
acidosis is present,
lular space to the i
moves from the intra
lar space to maintai
catabolic state is r
positive for ketones
glucose falls below
for glucose.

mental status, unequal pupils, decorticate or decerebrate posturing, and/or seizures indicate cerebral

Treatment

edema. Early identification and aggressive management of increased intracranial pressure are pivotal to
of treatment of new-onset
improve outcome.
reverse the catabolic state
Symptoms of hypoglycemia are due to catesulin therapy and to restore
cholamine release (trembling, diaphoresis, flushing,
e losses.
and tachycardia) and to cerebral glucopenia (sleepiDM is treated through insulin
ness, confusion, mood changes, seizures, and coma).
xercise, psychological support,

The immediate goals


IDDM and DKA are to
through exogenous in
fluid and electrolyt
The child with ID
replacement, diet, e
and regular medical

follow-up. Patient education has


a vital role. Curren
t therapy requires frequent blood
Differential Diagnosis
nd carbohydrate counting. The
Secondary diabetes may occur when there is insulin
o tailor insulin dosing based on
antagonism from excess glucocorticoids (Cushing's
d the current meal. The newly
syndrome or iatrogenic), hyperthyroidism, pheochroequires 0.5 to 1.0 unit/kg of
mocytoma, GH excess, or with medications such as
t diabetics will take insulin two
thiazide diuretics.
. It is customary to give

glucose monitoring a
patient learns how t
the glucose level an
diagnosed diabetic r
insulin per day. Mos
to three times a day
two-thirds of the to

tal daily dose before breakfast


and one-third before
dinner, and the human
Diagnostic Evaluation
etween short-acting regular
Two random blood glucose levels greater than
iate-acting NPH insulin. An
200mg/dL are consistent with a diagnosis of IDDM.
become available. This
If early IDDM is suspected, a 2-hour postprandial
unt of insulin throughout
blood glucose concentration is the first value to
doses of short-acting insulin given
become abnormal. A fasting blood glucose concentrames of medical, surgical, or
tion greater than 126mg/dL and a 2-hour postpranditional insulin may be
dial blood glucose concentration greater than
hemoglobin levels should be
200mg/dL are suggestive of IDDM. Islet cell
nths to assess average glycemic
antibodies in the serum may be found in the
new-onset insulin-dependent diabetic; poorly
ccurs, a child may ingest a

insulin is divided b
insulin and intermed
insulin pump has now
delivers a basal amo
the day, with bolus
at meal times. At ti
emotional stress, ad
needed. Glycosylated
monitored every 3 mo
control.
If hypoglycemia o

controlled diabetics have high levels of glycosylated


o increase the serum glucose
hemoglobin.
e child is vomiting, Monogel

carbohydrate snack t
concentration. If th

--------------------------------------- 259
Chapter 6 / Endocrinology
59
instant glucose or cake icing may be applied to the

Epidemiology

buccal mucosa to provide glucose. If the child is


% to 3% of all diabetes in
stuporous or having a seizure, intravenous glucose or
he incidence is increasing
intramuscular glucagon may be given.
prevalence of obesity. Most cases
DKA is a medical emergency. Initial fluid resuscidolescence around the onset of
tation is accomplished by giving a normal saline or lac-

NIDDM accounts for 2

tated Ringer's solution, lOmL/kg intravenous bolus.


is highest in Native Americans,

puberty. Prevalence

children. However, t
because of the high
occur during early a

African Americans, a
nd Hispanics. Genetic susceptiWhile the fluid bolus is running in, the total fluid
however, environmental factors,
deficit is calculated based on the amount of dehydraysical inactivity, and diet, play a
tion. The fluid deficit should be replaced over a 48-

bility is important;
including obesity ph
major role.

hour period. The level of hyperglycemia is assessed


and an insulin drip is started at 0.1 unit/kg/hr.The goal
Exam
is to decrease the serum glucose 50 to lOOmg/dL/hr.
e asymptomatic at presentation.
A glucose level that falls too quickly could precipitate
toms similar to those of type
cerebral edema. When serum glucose approaches 250

History and Physical

to 300mg/dL, dextrose should be added to normal


s usually a positive family history.

1 diabetics. There i

Many patients will b


Others may have symp

On physical examinat
ion, obesity is noted, with a
saline and the electrolyte solution to avoid hypo) usually greater than 85%.
glycemia. Acidosis and ketone production corrects
h NIDDM is acanthosis nigriwith insulin therapy. Until there is adequate insulin,
on involving hyperpigmentation
the body will continue to produce ketoacids. Frequent
e skin folds, found primarily on
monitoring of blood glucose level, electrolytes, and
.
acid-base status is crucial.

body mass index (BMI


Often associated wit
cans, a skin conditi
and thickening of th
the back of the neck

Treatment

Prognosis
tay of treatment is insulin

Currently, the mains


therapy. Many oral h

ypoglycemic agents have not yet


The Diabetes Control and Complications Trial has
ren; their use is primarily
demonstrated that intensive management and tight
arch is needed in this area. In
glycemic control will reduce the risk of complicatherapy, lifestyle changes in
tions by 50% to 75%. Complications from IDDM
e particularly important.
include mlcrovascular disease of the eye (retinopa-

been tested in child


anecdotal. More rese
addition to medical
diet and exercise ar

thy), kidney (nephropathy), and nerves (neuropathy). Micro vascular disease is generally not seen until
the child has been insulin dependent for a minimum
EY POINTS
of 10 years. Accelerated large vessel atherosclerotic
tus is a chronic metabolic disorder
disease can lead to myocardial infarction or stroke.
y hyperglycemia and abnormal
Diabetic children should have annual urine collecsm caused by absent or
tions to screen for microalbuminuria, annual ophlin secretion or action at the
thalmologic examinations, and annual screening for
.
hyperlipidemia.
nt diabetes mellitus (IDDM) type

K
1. Diabetes melli
characterized b
energy metaboli
diminished insu
cellular level
2. Insulin-depende
1 results from

a lack of insulin production in the B


cells of the p
ancreas.
Noninsulin-Dependent Diabetes
w-onset weight loss, polydipsia,

3. A history of ne
polyphagia, an

d polyuria is consistent with type 1


Mellitus (Type 2)
Pathogenesis
tus.

diabetes melli
4. Long-term compl

ications from IDDM include


Noninsulin-dependent diabetes mellitus (NIDDM)
disease (retinopathy, nephropathy,
type 2 is a polygenic condition that results from a
) and accelerated large vessel
relative insulin resistance. This insulin resistance
c disease.
initially causes a compensatory increase in insulin
of NIDDM cases in children is
secretion; however, with time there is a progressive
decline in the glucose-stimulated insulin secretion.

microvascular
and neuropathy
atheroscleroti
5. The percentage
rising.

--------------------------------------- 260
Blueprints Pediatrics
Collapsed child
breathing?
No
Yes
I
Open airway
Mouth to mouth
Call for help
I
Breathing?
-> Maintain airway
Give O2
Ventilation adequate?
Chest rises with

No >

Maneuvers fo

r
mouth to mouth?

obstr

ucted airways
(chokin
g/laryngoscopy)
Continue ventilation *-

No

Give O2, bag-mask


Endotracheal intubation
Yes
Peripheral pulses?

Yes

Blood

pressure?
Apex audible?
I
Low
Normal
Give volume
Ensure adequacy of ventilation
challenge
Close monitoring
Chest compressions
Differential diagnosis?
I

Establish I

V/cutdown
Intravenous in situ? s infusion

No

*" Intraosseou

Epinephrine/
bicarbonate
i
Yes.
VT
Bradycardia/

reading?
Pulsele3SSVT
VF

asystole
/Hemodynamically
Hemodynamically

He

modynamically
unstable/

Hemodynamically

VT refractory

stable

unstable/
stable
SVT refractory

to meds
to meds
Lidocaine
Cardioversion
Amiodarone

Cardioversion
Atropine

DefibrillateVagal maneuvers
Adenosine

Epinephrine
Procainamide

Amiodarone

Pacemaker
Treat hypomagnesemia
Treat hypokalemia
Figure 1-4- Cardiopulmonary resuscitation algorithm.VT, ventricular tachycardia;
VF, ventricular fibrillation; SVT, supraventricular
tachycardia.

--------------------------------------- 261
Blueprints Pediatrics
DIABETES INSIPIDUS
tional delay. Eighty percent of

60

stature and constitu


cases of short statu

re are attributable to these


In diabetes insipidus, there is loss of antidiuretic
ic causes may result in either
hormone secretion from the posterior pituitary gland
proportionate short stature.
and an inability to concentrate the urine. Diabetes
lt in proportionate short stature
insipidus can occur after head trauma or with a brain
ent than those of disproportumor or central nervous system infection. Surgical
e.
interruption of the pituitary stalk during craniosult in disproportionate short
pharyngioma removal often results in diabetes
ong bones predominantly and

two causes. Patholog


disproportionate or
Etiologies that resu
are much more preval
tionate short statur
Disorders that re
stature affect the l

insipidus. Only rarely is diabetes insipidus an isolated


ch is caused by activated vitamin
idiopathic disorder.
hondroplasia, an autosomal

include rickets, whi


D deficiency, and ac
dominant disorder.
Diseases that cau

se proportionate short stature


Clinical Manifestations
er a prenatal or postnatal

may result from eith


insult to the growth

process. Prenatal etiologies


The child with diabetes insipidus has abrupt-onset
growth retardation, placental
polydipsia and polyuria. If the cause of the diabetes
erine infections, teratogens, and
insipidus is a brain tumor impinging on the pituitary
ities. The most common
gland, focal neurologic signs and visual abnormalities
ities that result in short
may be noted.
21 and Turner's syndrome.
The increased urine output may reach 5 to
lude malnutrition, chronic sys10 L/day with a urine specific gravity and urine osmolality that are quite low. Over time, serum sodium and
hosocial deprivation, drugs, and
serum osmolality increase, as hemoconcentration
Common endocrine defects that
occurs from free water loss. In unclear cases, the water
ure include hypothyroidism,
deprivation test is used to document diabetes
deficiency, glucocorticoid
insipidus. Demonstration of antidiuretic hormone
us puberty.
(ADHJ secretion is critical in differentiating ADHdeficient diabetes insipidus from nephrogenic diais
betes insipidus, a rare X-linked recessive disease in
which the collecting ducts do not respond to ADH.
al short stature establish growth

include intrauterine
dysfunction, intraut
chromosomal abnormal
chromosomal abnormal
stature are trisomy
Postnatal causes inc
temic diseases, psyc
endocrine disorders.
result in short stat
growth hormone (GH)
excess, and precocio
Differential Diagnos
Children with famili
curves at or below t

he fifth percentile by the age of


2. They are otherwis
e completely healthy, with a
Treatment
ination. These children have a

normal physical exam


normal bone age, and

puberty occurs at the expected


DDAVP, an ADH analogue, is given intranasally,
is usually found in at least one
intravenously, or orally to stimulate the kidneys to
nheritance is complex and the
retain water and reverse the polyuria, polydipsia, and
may be more distant.
hypernatremia.
stitutional delay grow and

time. Short stature


parent, but height i
diminutive ancestor
Children with con
develop at or below

the fifth percentile at normal


KEY POINTS
his results in a curve parallel to

growth velocities. T

the fifth percentile


. Puberty is significantly delayed,
1. In diabetes insipidus, there is loss of ADH secreelay in- the bone age. Because
tion and an inability to concentrate the urine.
to enter puberty at the usual
2. Diabetes insipidus can occur after head trauma or
ture and sexual immaturity are
with a brain tumor or central nervous system
infection.
ir peers enter puberty. Family

which results in a d
these children fail
age, their short sta
accentuated when the
members are usually

of average height, but there


is often a history o
f short stature in childhood
and delayed puberty.
The parents of children
SHORT STATURE
delay should be counseled that

with constitutional
their child's growth

is a normal variant and that the


Short stature is a common concern of parents.
ture to the height expected for
Normal causes include familial (genetic) short

child will likely ma


their family.

--------------------------------------- 262
61
Chapter 6 / Endocrinology
GH deficiency accounts for approximately 5% of
stations of Turner's syndrome,
cases of short stature referred to endocrinologistswhic.
in detail in Chapter 9, is short
Children with classic GH deficiency grow at a
l manifestations of Turner's
diminished growth velocity, less than 5 cm/yr, and
es be subtle. Given that the
have delayed skeletal maturation. A history of birth
s syndrome is 1 in 2500 females,
asphyxia or neonatal hypoglycemia or physical
ryotype testing are indicated
findings of microphallus or midline defects arie
cent with short stature and
suggestive of idiopathic GH deficiency. GH defivated gonadotropins, indicating
ciency secondary to hypothalamic or pituitary tumor
ure, and a 45,XO karyotype are
usually is associated with other neurologic or visual
impairments. In an older child with more recent
ation of certain medications
onset of subnormal growth, the index of suspicion
rowth. Such drugs include
for tumor should be high.
etamine (Dexedrine), and mePrimary hypothyroidism causes marked growth
in).
failure because of a diminished growth velocity and

One of the manife


h is discussed
stature. The clinica
syndrome can sometim
incidence of Turner'
gonadotropins and ka
n the female adoles
delayed puberty. Ele
primary ovarian fail
diagnostic.
Chronic administr
may result in poor g
steroids, dextroamph
thylphenidate (Rital

skeletal maturation. Thyroxine (T

4j, triiodothyro-

nine resin uptake (T


3RU), thyrotropin (TSH), and
ons
thyroid antibodies should be measured, even in

Clinical Manifestati

the absence of symptoms, to rule out any degree

History

of hypothyroidism when evaluating short stature.


information includes the child's

Important historical
prenatal and birth h

istory, the pattern of growth, the


Primary hypothyroidism is treated with levothyroxdisease, long-term medication
ine (Synthroid).
of developmental milestones,
Cushing's disease is a rare cause of short stature.
ubertal patterns of the patient's
Hypercortisolism, from either exogenous steroid
. Obtaining and evaluating
therapy or endogenous oversecretion, may have a
harts are vitally important. A
profound growth-suppression effect. Usually, other
tory, including what, how, and
stigmata of Cushing's syndrome are present if growth
fed, is also required.
suppression has occurred.

presence of chronic
use, the achievement
and the growth and p
parents and siblings
the child's growth c
thorough feeding his
by whom the child is

Chronic systemic diseases can result in short


stature from lack of caloric absorption or increased
metabolic demands. Cyanotic heart disease, cystiThc
sical examinations done on chilfibrosis, poorly controlled diabetes mellitus, chronic
ure are normal. It is critical to
renal failure, HIV infection, and severe rheumaght and weight on the appropritoid arthritis are disorders that increase metaage. In addition to height, arm
bolic demands and diminish growth. Alternativelyspa,
ower-body segment ratio is meainflammatory bowel disease, celiac sprue, and cystic
athologic disproportionate causes
fibrosis can reduce caloric absorption and produce
young children, the head circumshort stature.
be evaluated to check for failure
Some children who live in emotionally or physitoen with failure to thrive, weight
cally abusive or neglectful environments develop
ished and the head circumferfunctional GH deficiency. Children with psychoso. When examining the child with
cial deprivation may have bizarre behaviors that
hysician may find dysmorphic feainclude food hoarding, pica, and encopresis, as welturel
suggestive of a particular syndrome.
as immature speech, disturbed sleep-wake cycles, and

Physical Examination
e majority of phy
dren with short stat
plot the child's hei
ate growth curve for
n and upper-to-l
sured to check for p
of short stature. In
ference should also
thrive. In childr
and height are dimin
ence is often spared
short stature, the p
s in a pattern
The integument shoul

d be examined for cyanosis


an increased pain tolerance. Clinically, they resemble
indicating potential
congenital heart disease, abnorchildren with GH deficiency, with marked retardamal pigmentation not
ed in Cushing's syndrome, the
tion of bone age and pubertal delay. If GH testingstigmat is
a of hypothy
roidism, and bruises and poor
done while the child remains in the hostile environhygiene indicative o
f psychosocial deprivation.
ment, there is a blunted GH response; when the
The thyroid is palpa
ted to determine its size, its
child is removed from the deprived environment,
consistency, and the
presence of thyroid nodules.
GH testing reverts to normal and catch-up growth
The lungs and heart
are examined to identify chronic
is noted.
cardiopulmonary dise
ase. Abdominal tenderness or

--------------------------------------- 263
62
Blueprints Pediatrics
bloating may indicate inflammatory bowel disease oir
MRI of the brain should be
celiac sprue. Tanner staging for both boys and girlorderes
nitiating GH therapy. GH therapy
must be documented to help differentiate among
hood because of its effects on
familial short stature, constitutional delay, and
metabolism. If puberty is
precocious puberty. A thorough neurologic and
4 years, the addition of sex
funduscopic examination may reveal underlying
idered, both to augment the
central nervous system disease that may result in
H and to stimulate secondary
GH deficiency.

n most children. An
d prior to i
is needed into adult
bone mass and lipid
delayed beyond age 1
steroids may be cons
growth response to G
sexual development.
Primary hypothyro

idism is treated with levothyroxine (SynthroidJ.


After several weeks of therapy,
Diagnostic Evaluation
the growth velocity
generally returns to normal, and
Because most cases of short stature result from
over time there may
be some catch-up growth.
either familial short stature or constitutional delay,
Unlike GH therapy, l
evothyroxine therapy does not
diagnostic studies are generally not necessary
promote catch-up gro
wth.
unless abnormalities are found on exam. A bone age
To manage the sho
rt stature associated with
(anteroposterior x-ray of the left wrist) assessmenCushing't
s disease,
the physician must identify and
helps to delineate familial short stature from constitreat the etiology.
Girls with short stature caused
tutional delay. An advanced bone age likely indicatebs
y Turner's syndrom
e may receive GH to increase
precocious puberty; a normal bone age, familial shortheit
r final adult he
ight. Short stature caused by psy-

stature; and a delayed bone age, constitutional delay.


chosocial deprivatio
n is treated by removing the
Thyroid function tests must be done to rule ouchilt
d from the envir
onment. Short stature caused by
hypothyroidism. Urinalysis and renal function testmedications
s is re
versed by discontinuing the offendare needed to rule out chronic renal disease. A
ing medication.
complete blood count with differential and an
erythrocyte sedimentation rate may reveal evidence
of chronic systemic infection. The child's nutritional
KE
Y POINTS
status can be examined through the serum albumin
1. Eighty percent
of cases of short stature result
and total protein counts. A screen for insulin-like
from normal gro
wth and development and are
growth factor-1 (IGF-1) and insulin-like growth
factor binding protein~3 (IGF-BP3J may be ordered
due to either f
amilial (genetic) short stature or
to look for GH deficiency. If a chromosomal anomaly
constitutional
delay.
is considered, obtaining a karyotype may be helpful.
2. Pathologic caus
es may result in either disproportionate or prop
ortionate short stature; proporA magnetic resonance image of the head may identify
tionate short s
tature is more prevalent than
a hypothalamic or pituitary process that is resulting in
disproportionat
e short stature.
decreased GH secretion from the pituitary.
3. The most common
pathologic etiologies of proportionate shor
t stature include GH deficiency,
Treatment
primary hypothy
roidism, Cushing's disease,
The child with familial short stature has few therachronic systemi
c diseases, psychosocial deprivapeutic options. For most children with constitutional
tion,Turner's s
yndrome, and medications.
delay, reassurance that the child's short stature is a
__THYROID DYSFUNCTIpN
normal variant suffices. In some select patients with
_______
no signs of puberty by age 14 years, a 4- to 6-month
treatment with the appropriate sex hormone may
Hyperthyroidism
help to modestly increase stature and pubertal
development for psychological support until true
Most cases of hypert
hyroidism in children are caused
pubertal development begins.
by Graves' disease.
Other causes include a hyperChildren with GH deficiency are managed with
functioning "hot" th
yroid nodule or acute supbiosynthetic human GH by subcutaneous injection
purative thyroiditis
. Graves' disease, an autoimmune
every day or by a depot form of growth hormone thadisordert
, is cause
d by circulating thyroid-stimulating
is given 1-2 times per month. Accelerated growth
immunoglobulins bind
ing to thyrotropin receptors
velocity on GH treatment results in catch-up growth
on thyroid cells, wh
ich results in diffuse hyperplasia

--------------------------------------- 264
Chapter 6 / Endocrinology
63
and increased levels of free T
4. Neonatal Graves'
c thyroiditis that results in
disease follows transplacental passage of maternal
on of the thyroid gland.
thyroid-stimulating immunoglobulins.
thyroidism include panhypopi-

a chronic lymphocyti
autoimmune destructi
Other causes of hypo
tuitarism, ectopic t

hyroid dysgenesis, administration


Clinical Manifestations
ations, and surgical or radioacSymptoms include a voracious appetite (without
for treatment of hyperthyweight gain or with weight loss), heat intolerance,
ce of hypothyroidism in girls is
emotional lability, restlessness, excessive sweating,
han in boys. There is often a
frequent loose stools, and poor sleep. Exophthalmofamils
raves' disease or Hashimoto's thyis uncommon in children. Older children may
ren present at adolescence; it is
complain of palpitations. There is often a change
hyroiditis before 5 years of age.
in behavior and school performance. On physical
examination, the child may be flushed, fidgety, and
ons
warm, with proptosis, a hyperactive precordium,
ppear after the first year of life
resting tachycardia, and a widened pulse pressure.
olerance, diminished appetite,
The thyroid gland is generally enlarged, smooth, firm
pation. Physical findings include
(but not hard), and nontender. Often a fine tremor
delayed puberty, immature
is noted, and proximal muscle weakness is present.
arse puffy facies, dry thin hair,
Acute-onset tachycardia, hyperthermia, diaphoresis,
endon reflexes with a delayed
fever, nausea, and vomiting indicate thyroid storm
(malignant hyperthyroidism), which can be lifetests reveal a depressed total T
4
threatening but is rare in children.
and a depressed T
3RU level.
Infants with neonatal Graves' disease tend to stare,
idism is present, an elevated
are jittery and hyperactive, and have an increased
ion is noted. If secondary
appetite and poor weight gain. Tachycardia is usually
esent, the TSH level may be
present, and thyromegaly may be palpable.
r elevated. The detection of
In hyperthyroidism, T4 levels are elevated, T 3RU is
es indicates an autoimmune
elevated, and TSH is suppressed.

of antithyroid medic
tive iodine ablation
roidism. The inciden
four times greater t
y history of G
roiditis. Most child
unusual to develop t

Clinical Manifestati
Symptoms generally a
and include cold int
lethargy, and consti
slow linear growth,
body proportions, co
dry skin, and deep t
relaxation time.
Thyroid function
serum concentration
If primary hypothyro
serum TSH concentrat
hypothyroidism is pr
depressed, normal, o
thyroid autoantibodi
basis for disease, w

hereas palpation of a thyroid


nodule should prompt
evaluation with a thyroid
Treatment

scan.

Medical therapy for congenital hyperthyroidism


is the administration of propylthiouracil (PTU).

Treatment

Neonatal Graves' disease generally resolves over the


with synthetic levothyroxine
first several months of life. In the infant hemodyded and adjusted to maintain
namically compromised by hyperthyroidism, par4 levels, normal TSH levels,
enteral fluids, digoxin, and propranolol may be
ent. Thyroid function tests
necessary.
frequently.
PTU, methimazole, or radioiodine may be used to

Thyroid replacement
(Synthroid) is provi
normal serum free T
growth, and developm
should be monitored

treat Graves' disease and must be titrated carefully


because too high a dose can result in hypothyroidism.
Fifty percent of children with Graves' disease have a
EY POINTS
spontaneous remission and may be taken off antithyroid medication after 12 to 24 months of treatment.
yperthyroidism in children
Those children who do not have remission of their
raves'disease, which is an
disease will continue on the antithyroid drug and
ced thyroid hyperplasia.
Synthroid is added to prevent hypothyroidism.
'disease results from

1. Most cases of h
are caused by G
autoimmune-indu
2. Neonatal Graves
transplacental

passage of maternal thyroidstimulating imm


unoglobulins.
Hypothyroidism
rthyroidism,T4 levels are elevated,

3. In primary hype
T3RU is elevate

d, and TSH is suppressed.


Congenital hypothyroidism is discussed in Chapter
13. The most common cause of juvenile or acquired
Continued
hypothyroidism is Hashimoto's thyroiditis, which is

--------------------------------------- 265
64

Blueprints Pediatrics
Adrenal cortex
Testis

Cholesterol
20, 22-D
A5 Pregnenolone

17-OHase17a-Hydroxypregnenolone

3|i-HSD

3P-HSD
17-OHase

Progesterone
21-OHase
11-Deoxycorticosterone
11-OHase

17,20-D

17a-Hydroxyprogesterone
l-OHase
11-Deoxycortisol
11-OHase

Corticosterone
18-OHase

Cortisol

18-Hydroxycorticosterone
Aldosterone
Mineralocoreticoids
Glucocorticoids
Androgens
A schematic of steroidogenesis in the adrenal cortex.
Figure 6-1
inherited as an auto
somal recessive trait and tends to
4. Medical therapy for Graves'disease consists of
sic salt-wasting 21-hydroxylase
propylthiouracil administration.
ilizing 21-hydroxylase deficiency.
5. The most common cause of juvenile or acquired
eded to produce aldosterone and
hypothyroidism is Hashimoto's thyroiditis, which
lase deficiency results in a buildis a chronic lymphocytic thyroiditis that results in
of aldosterone and cortisol.
autoimmune destruction of the thyroid gland.
roxyprogesterone increases,
6. Thyroid function tests in hypothyroidism reveal
lized to dehydroepiandroa decreased T4 serum concentration, decreased
nedione. Both forms of
T3RU, and elevated serum TSH concentration.
ciency result in decreased cortisol
7. Hypothyroidism is treated with synthetic
etion, increased corticotropin
levothyroxine.
d 17-hydroxyprogesterone and

occur as either clas


deficiency or as vir
21-Hydroxylase is ne
cortisol. 21-Hydroxy
up of the precursors
Specifically, 17-hyd
which is then metabo
sterone and androste
21 -hydroxylase defi
and aldosterone secr
(ACTH), and increase
17-hydroxypregnenolo

ne.
_.ADRENAL DYSFUNCTION
ciency accounts for 5% of the

11-Hydroxylase defi
cases of congenital

adrenal hyperplasia and is also

Congenital Adrenal Hyperplasia


somal recessive trait. Similar to

inherited as an auto
21 -hydroxylase defi

ciency, 11 -hydroxylase deficiency


The clinical characteristics of congenital adrenal
on of aldosterone and cortisol.
hyperplasia depend on which enzyme in the pathway
erts 11 -deoxycortisol to cortisol
of steroidogenesis is deficient. See Figure 6-1 for a
one to corticosterone in the
schematic of steroidogenesis in the adrenal cortex.
With reduction or absence of
21 -Hydroxylase deficiency accounts for 90% of the
rtisol and aldosterone precursors
cases of congenital adrenal hyperplasia. The disease is
nted to androgen synthesis.

impairs the producti


11 -Hydroxylase conv
and deoxycorticoster
aldosterone pathway.
11 -hydroxylase, co
build up and are shu

--------------------------------------- 266
65
Chapter 6 / Endocrinology
Clinical Manifestations
Y POINTS
In congenital 21-hydroxylase deficiency, female
infants are born with ambiguous genitalia. Clideficiency accounts for 90% of
toromegaly and labioscrotal fusion may result in
ngenital adrenal hyperplasia.
erroneous male sex assignment. There is normal
1-hydroxylase deficiency, female
ovarian development, and internal genital structures
n with ambiguous genitalia,
are female. Male infants born with the defect have
fants born with the defect have
no genital abnormalities. Symptoms of emesis, salt
rmalities.
wasting, dehydration, and shock develop in the first
21-hydroxylase deficiency, symp2 to 4 weeks of life. Hyponatremia and hyperkalemia
salt wasting, dehydration, and
result from lack of aldosterone, and hypoglycemia
n the first 2 to 4 weeks of life.
results from decreased levels of cortisol. Worsening
f congenital adrenal hyperpiasia
hyponatremic dehydration culminates in shock
menting elevated levels of

KE

1. 21 -Hydroxylase
the cases of co
2. In congenital 2
infants are bor
whereas male in
no genital abno
3. In salt-wasting
toms of emesis,
shock develop i
4. The diagnosis o
is made by docu
17-hydroxyproge

sterone in the serum.


and acidosis in severe cases. The diagnosis of 21hydroxylase deficiency includes
hydroxylase deficiency is made by documenting
neralocorticoid therapy.
elevated serum levels of 17-hydroxyprogesterone.
In 11 -hydroxylase deficiency, there is overproduction of deoxycorticosterone, which has mineralo-

5. Therapy for 21cortisol and mi

corticoid activity and results in hypernatremia,


tropin dependent or gonadotrohypokalemia, and hypertension. Diagnosis is based

may be either gonado

on the measurement of increased levels of 11 -deoxye central (gonadotropincortisol and deoxycorticosterone in the serum or
s puberty is more common in
their tetrahydrometabolites in the urine. Serum
Precocious puberty in girls is usually
androstenedione and testosterone are also elevated,
in boys there is a greater incidence
and renin and aldosterone levels are depressed.
mors causing gonadotropin-

pin independent. Tru


dependent) precociou
girls than in boys.
idiopathic, whereas
of CNS pathology. Tu
dependent precocious

puberty (GDPPJ include


Treatment
and hamartomas. Other causes

gliomas, pinealomas,
of GDPP include hydr

ocephalus, head injury, and


Therapy for 21-hydroxylase deficiency includes
em infection or congenital
cortisol and mineralocorticoid therapy. Cortisol
therapy reduces ACTH secretion and overproduction
pendent precocious puberty
of androgens, and mineralocorticoid administration is
rare and is seen in McCuneadjusted to normalize serum renin levels. Surgical
olyostotic fibrous dysplasia of
correction of female genital abnormalities is accomocious puberty in boys (familial
plished early. The linear growth and sexual developdig cell tumors, and ectopic HCG
ment of children with 21-hydroxylase deficiency
sms such as hepatic and pineal
must be monitored closely. Undertreatment, as
indicated by elevated 17-hydroxyprogesterone,
che refers to isolated early breast
androstenedione, and renin levels and by accelerated
al age of onset is 12 to 24
advancement of skeletal maturity, leads to excessive
elarche is likely due to small
growth, premature sexual hair growth, and virilizaestrogen from the prepubertal
tion of the child. Ultimately, undertreatment may
sed sensitivity to low levels of
lead to premature epiphyseal fusion and adult
ubertal female. Premature
short stature. Overtreatment with cortisol supthe early appearance of sexual
presses growth and may cause symptoms of
hypercortisolism.
of 8 in girls and the age of 9 in

central nervous syst


malformation.
Gonadotropin-inde
(GIPP) is extremely
Albright syndrome (p
bone), familial prec
testitoxicosis), Ley
production by neopla
tumors.
Precocious thelar
development. The usu
months. Premature th
transient bursts of
ovary or from increa
estrogen in the prep
adrenarche refers to

hair before the age


boys. This benign co

ndition is due to early maturation of adrenal andr

ogen secretion.
Precocious Puberty
ons
True precocious puberty is defined as secondary sex
che, gonadotropin and serum
characteristics presenting in girls before the age of
in the prepubertal range, and
7.5 years and in boys before the age of 9 years and
ration and advancing skeletal

Clinical Manifestati
In precocious thelar
estrogen levels are
linear growth accele

--------------------------------------- 267
66
rintg Pediatrics
maturation are not present. This nonprogressive,
of 14 or the failure to complete
benign condition is distinguished from true precors from the onset of puberty.
cious puberty by the normal growth rate and bone
is the cause for 90% to 95% of
age noted with premature thelarche.
dren the bone age is normal,
In premature adrenarche, the levels of adrenal
puberty will simply appear late.
androgens are normal for pubertal stage but elevated
ositive family history.
for chronologic age. The child's bone age is usually
slightly advanced. Children with premature adrenaris
che must be evaluated for other causes of increased
delay puberty in both sexes.
androgen production, such as congenital adrenal
e due to primary gonadal failure
hyperplasia, polycystic ovarian syndrome, or adrenal
hypogonadism. Examples of
tumor. In children with evidence of significant andros syndrome or autoimmune
gen effect (advanced bone age, growth accelerationovaria,
girls) and Klinefelter's syndrome
and acne), measurement of adrenal steroids and
otropic hypogonadism is due to
androgens before and after ACTH administration is
ry axis dysfunction. Examples
used to identify those with congenital adrenal
yndrome, isolated gonadotropic
hyperplasia.
amic and pituitary tumors,
The clinical manifestations of GDPP include preanorexia nervosa. Other
mature development of secondary sexual characincluding hypothyroidism may
teristics and an accompanying growth spurt. If the
GDPP is secondary to pathology of the central
ons
nervous system, then focal neurologic signs are often
present. Diagnosis is based on advanced bone age
ical exam should include an
and pubertal levels of gonadotropins and estrogen
h trends, the timing of puberty

teristics at the age


genital growth 5 yea
Constitutional delay
cases. In these chil
growth is slow, and
There is usually a p
Differential Diagnos
Systemic disease can
Pubertal delay may b
or hypergonadotropic
this include Turner'
n failure (in
(in boys). Hypogonad
hypothalamic/pituita
include Kallmann's s
deficiency, hypothal
hypopituitarism and
endocrine disorders
also delay puberty.
Clinical Manifestati
The history and phys
examination of growt

or testosterone. A pubertal pattern of elevated


ers, and an assessment of the
gonadotropins after infusion of gonadotropinnner staging. Laboratory evaluareleasing hormone (GnRH) is indicative of GDPP. In
luding a bone age, testosterone
GIPP, gonadotropins are low and GnRH has no
, gonadotropins, FSH and LH,
effect on gonadotropin levels.
id function testing. Screening to

in other family memb


patient's current Ta
tion is helpful, inc
and estradiol levels
prolactin, and thyro
look for systemic di

sease is also indicated.


Treatment
Premature thelarche is a benign condition that does
not require any treatment. Premature adrenarche
itutional delay, a short course of
that is not caused by congenital adrenal hyperplasia,
needed to initiate pubertal
tumor, or polycystic ovarian syndrome is also a
ocial support is also important.
benign condition.
adism is determined to be the
GDPP is treated with injections of long-acting
id replacement is initiated at the
preparations of GnRH. GnRH analogues suppress
ty and continued for a lifetime.
gonadotropin release and thereby decrease secondary
sex characteristics, slow skeletal growth, and prevent
the fusion of long bone epiphyseal plates. GIPP is
managed by treating the underlying disease process.
EY POINTS

Treatment
In the case of const
sex steroids may be
development. Psychos
If permanent hypogon
etiology, sex stero
normal time of puber

K
1. True precociou

s puberty is defined as secondary


Pubertal Delay
stics presenting in girls before the

sex characteri
age of 7.5 yea

rs and in boys before the age of 9


Pubertal delay is characterized by a delay in the onset
be either gonadotropin dependent
of puberty or in the rate of progression through
n independent.
normal sexual development. In females, this refers to
onadotropin-dependent)
the absence of secondary sex characteristics at the
erty is more common in girls
age of 13 or the absence of menarche 5 years from
Precocious puberty in girls is usually
the onset of sexual development. In males, pubertal
ereas precocious puberty in boys
delay denotes the absence of secondary sex charac-

years and may


or gonadotropi
2. True central (g
precocious pub
than in boys.
idiopathic, wh

--------------------------------------- 268
67
Chapter 6 / Endocrinology
is often due to tumors of the central nervous
CT scans of the adrenal glands
system.

of the pituitary and


is helpful to determ

ine if additional pathology exists.


3. The clinical manifestations of gonadotropin-

Treatment

dependent precocious puberty (GDPP) include


premature development of secondary sexual
re surgical removal. Similarly,
characteristics and an accompanying growth
perplasia is treated with surgical
spurt.
itary adenoma. Trans-sphenoidal
4. GDPP is treated with injections of long-acting
most effective method of
preparations of gonadotropin-releasing hormone.
. Perioperative stress dosing
5. The most common cause of pubertal delay is
s needed to avoid adrenal

Adrenal tumors requi


bilateral adrenal hy
excision of the pitu
microsurgery is the
microadenoma removal
of glucocorticoids i
insufficiency. Posto

peratively, the patient may


constitutional delay.
icoid deficiency in addition to

develop mineralocort
the glucocorticoid d

eficiency.
Cushing's Syndrome
Cushing's syndrome is a constellation of symptoms
Y POINTS
and signs that result from high cortisol levels. It is
ome is a constellation of
due to either endogenous overproduction of cortisol
gns that result from high
or excessive exogenous treatment with pharmacoand is due to either endogenous
logic doses of cortisol. Endogenous causes include
Cushing's disease and adrenal tumors. Cushing's
of cortisol or excessive exogenous
disease, also known as bilateral adrenal hyperplasia,
pharmacologic doses of cortisol.
is the most common etiology of Cushing's syndrome
se is the most common noniatroin children older than 7 years. In most instances, it is
Cushing's syndrome.
caused by a microadenoma of the pituitary gland
ns and symptoms of Cushing's

KE
1. Cushing's syndr
symptoms and si
cortisol levels

overproduction
treatment with
Cushing's disea
genic cause of
2. The classic sig
syndrome includ

e "moon" facies, buffalo hump,


resulting in ACTH oversecretion. Rarely, in the
, abdominal striae, acne, slow
young child or infant, a malignant carcinoma of the
nsion, and muscle weakness.
adrenal gland is seen. Most adrenal tumors that cause

truncal obesity
growth, hyperte

Cushing's syndrome are adenomas. Ectopic ACTH


secretion may occur with some tumors; however, this

Addison's Disease

is exceedingly rare in children.


Addison's disease, o

r primary adrenal insufficiency,


Clinical Manifestations
acquired and results in
The classic signs and symptoms of Cushing's
ecretion. Depending on the
syndrome include slow growth with pubertadiseasl
ere may be a concomitant decrease
arrest, "moon" facies, buffalo hump, truncal obesity,
se. In the newborn, primary
abdominal striae, acne, hyperpigmentation, hypery may be due to adrenal hypoplatension, fatigue, muscle weakness, and emotional ansiad
veness, adrenal hemorrhage, or
mental changes. Most adrenal tumors are virilizing.
with sepsis (WaterhouseInitial laboratory studies include documentation
e). In older children and adoof an elevated serum cortisol level and an increased
adrenal insufficiency is most
24-hour urine free cortisol test. If hypercortisolism is
alone or in association with
demonstrated, the dexamethasone suppression test ianothes
endocrinopathy such as thyperformed to document the presence of Cushing's
berculosis, hemorrhage, fungal
syndrome. Dexamethasone is given in the latinfectione
ic infiltration, and HIV infection
evening, and a cortisol level is measured the nexmat
truction of the adrenal gland.
morning. Failure of the dexamethasone to suppress
is an X-linked recessive
the morning cortisol level is consistent with
in fatty acid metabolism that
Cushing's syndrome. A prolonged dexamethasone
nsufficiency and progressive
suppression test is used to differentiate Cushing's
on.
disease from an adrenal tumor. When evaluating a
imary adrenal insufficiency, secchild with Cushing's syndrome, obtaining MRI scans
ficiency is due to ACTH defi-

may be congenital or
decreased cortisol s
e process, th
in aldosterone relea
adrenal insufficienc
, ACTH unresponsi
ischemic infarction
Friderichsen syndrom
lescents, autoimmune
common. It may occur
r autoimmune
roiditis or IDDM. Tu
, neoplast
y also cause des
Adrenoleukodystrophy
disorder of long-cha
results in adrenal i
neurologic dysfuncti
In contrast to pr
ondary adrenal insuf

--------------------------------------- 269
68
Blueprints Pediatrics
ciency.The most common cause of ACTH deficiency
is chronic steroid therapy that results in suppression
known as addisonian crisis, is a
of pituitary ACTH. Pituitary tumors and craniodition that should be treated
pharyngioma also result in depressed pituitary
ction of electrolyte abnormaliACTH secretion from either destruction of the
is required immediately with
pituitary or pituitary compression.
l saline and stress dose

Treatment
Adrenal crisis, also
life-threatening con
without delay. Corre
ties and dehydration
5% dextrose in norma
intravenous glucocor

ticoids.

Clinical Manifestations
ent consists of maintenance
Symptoms from primary adrenal insufficiency include
orticoids and mineralocorticoids.
weakness, nausea, vomiting, weight loss, headache,
ose is increased during times of
emotional lability, and salt craving. Physical findings
ss to avoid adrenal insufficiency.
include postural hypotension and increased pigmentation over joints and on scar tissue, lips, nipples, and
the buccal mucosa. The postural hypotension and salt
craving are due to lack of aldosterone, whereas the
Y POINTS
increased pigmentation is due to increased ACTH
insufficiency may be congenital
secretion. Melanocyte-stimulating hormone is a byresults in decreased cortisol
product of the ACTH biosynthetic pathway. Adrenal
eas secondary adrenal insufficrisis is characterized by fever, vomiting, dehydration,
o ACTH deficiency.
and shock. It may be precipitated by intercurrent
rimary adrenal insufficiency
illness, trauma, or surgery.
s, nausea, vomiting, weight loss,
Electrolyte abnormalities include hyponatremia,
ostural hypotension, and increased
hyperkalemia, hypoglycemia, and mild metabolic
acidosis from dehydration. An elevated baseline
is is characterized by fever, vomitACTH with a concurrent low cortisol level is conn, and shock. It may be precipisistent with primary adrenal insufficiency. The serum
urrent illness, trauma, or surgery.
cortisol level by definition is low and is unresponsive
ormalities found in adrenal
to injection of ACTH (corticotropin stimulation
hyponatremia, hyperkalemia,
test). If the corticotropin stimulation test is abnornd metabolic acidosis from
mal, a prolonged ACTH stimulation test is necessary
to rule out secondary adrenal insufficiency.

Long-term managem
doses of oral glucoc
The glucocorticoid d
acute metabolic stre

KE
1. Primary adrenal
or acquired and
secretion, wher
ciency is due t
2. Symptoms from p
include weaknes
salt craving, p
pigmentation.
3. An adrenal cris
ing, dehydratio
tated by interc
4. Electrolyte abn
crisis include
hypoglycemia, a
dehydration.

--------------------------------------- 270
pi I 11 lj
Electrolyte, and
pH Management
A human is born with 90% of his or her body weighwelt
l as a carbohydr
ate source. In general, one-fourth
as water. Body composition changes dramatically
to one-half normal s
aline with 5% dextrose (10%
over the first year of life as muscle mass increases. By
in infants) and 20mE
q/L KCl meets maintenance
1 year of age, a child's total body water approacheglucoss
e and electr
olyte needs.
the adult level of 60% body weight. Electrolyte
homeostasis, fluid distribution, and pH balance are

critical to the maintenance of normal physiology. The


younger the patient, the more intolerant he or she is
to challenges to these systems.

__P_EHY_D_RATION_
Dehydration in the p

ediatric patient is usually secondary to vomiting o


r diarrhea. Infants and toddlers
are particularly sus
ceptible because of the limited
_ MAINTENANCE _FL_U IDS
ability of the immat
ure kidney to conserve water and
The amount of fluid needed to maintain normal body
ause of the child's dependence

electrolytes and bec


on caretakers to mee

t his or her needs.


function is directly related to caloric expenditure,
which in turn is related to a child's weight. The
Holliday-Seger method is useful for calculating
maintenance fluids: lOOmL/kg/day for the first 10
ons
kg, plus 50mL/kg/day for the next 10kg, plus

Clinical Manifestati
History

25mL/kg/day for each additional kg thereafter. For


practical purposes, it is often more useful to
arifies the differential and procalculate hourly rate using 4mL/kg/hr (first 10kg
ncerning the acuity, source,
body weight] + 2mL/kg/hr (second 10kg body
d lost. Recent weight loss and
weight] + 1 mL/kg/hr (each additional kilogram].
ut are important indicators of
An example of calculating maintenance fluid
ency. The color, consistency,
requirements for a 16-kg child follows.
e of stool and/or emesis may

A careful history cl
vides information co
and quantity of flui
decreased urine outp
the degree of defici
frequency, and volum
influence initial di

agnostic and therapeutic measures.


Daily rate: (lOOmL/kg/day x 10kg) + (50mL/kg/day
ical illnesses may present
x6kg) = 1300mL/day
ation, including diabetes, metaHourly rate: 1300mL/day divided by 24hr/day
tic fibrosis, and congenital adrenal
= 54mL/hr
a in the presence of physical
Short-cut method: (4mL/hr x 10kg) + (2mL/hr x
n may indicate diabetes mellitus,
6kg) = 52mL/hr
or renal tubular acidosis. Children

Many chronic med


acutely with dehydr
bolic disorders, cys
hyperplasia. Polyuri
signs of dehydratio
diabetes insipidus,
who are neglected or

refuse to drink because of


For each lOOcc of maintenance fluids, a chilseverd
al pain may also develop signifineeds 3mEq of sodium and 2mEq of potassium, as

--------------------------------------- 271

e oropharynge
cant dehydration.

Chapter 1 / Emergency Management: Evaluation of the Crit


ically III or Injured Child
TABLE 1-3
hepatic, renal) and
preventing or correcting metabolic abnormalities
arising from cellular hypoperfuThe Etiologies of Shock
sion. Management of
hypoxia reduces the level of
Hypovolemic
metabolic acidosis.
Correcting metabolic acidosis
Water and electrolyte losses Distributive
results in better ce
llular function, myocardial perforHemorrhage
Anaphylaxis
mance, and decreased
systemic and pulmonary vasPlasma losses (third spacing) Neurologic injury (head
cular resistance.
Cardiogenic
or spinal cord)
Hypovolemic shock
is treated with normal saline
Congenital heart disease
Drug toxicity
or lactated Ringer's
solution (see Chapter 7 for
Ischemic heart disease
Septic
details). If hemorrh
age is the cause of the hypoCardiomyopathies
Infection
volemia, type O-nega
tive, cross-matched whole
Arrhythmias
Miscellaneous
blood or packed red
cells may be given. In cardioInfections
Pulmonary embolism
genic shock resultin
g from a congenital heart defect,
Adrenal insufficiency
surgery, balloon ang
ioplasty or valvuloplasty, surgical
valvotomy or inotrop
ic support may be indicated.
Children with severe
ischemic injury to the heart,
dilated cardiomyopat
hy, or myocarditis may need a
with a history of fever and is ill appearing may be in
heart transplant. In
distributive shock due to anaseptic shock.
phylaxis, intravenou
s steroids, Benadryl (diphenhySerial vital signs are critical in the diagnosis and
dramine), subcutaneo
us epinephrine, and albuterol
management of children with shock. In early "warm"
nebulizers are emplo
yed. Sometimes intubation for
compensated septic shock, vasodilation, warm
laryngospasm and vas
opressors for intractable hypoextremities, tachycardia, a widened pulse pressure,
tension are needed.
Septic shock is treated with
and adequate urine output are seen. In contrast,
vasopressors, fluids
, and broad-spectrum antibiotics.
symptoms of hypovolemic, cardiogenic, and late
Antibiotics are cons
idered a resuscitation medication
"cold" uncompensated septic shock include vasoconfor septic shock.
striction, tachycardia, cold extremities, poor peripheral pulses, altered consciousness, pallor, sweating,
ileus, and oliguria.
KE
Y POINTS
Diagnostic Evaluation
1. Determine the c
ategory of shock and whether the
patient has ear

ly or late manifestations.
During the stabilization period, the clinician must
ck accounts for most cases of
determine into which category of shock the patient's
illness falls. Any patient with shock should be placed
shock, blood pressure depression
on a cardiac monitor. The level of tachycardia is the
ng, and the level of tachycardia is the
best determinant of the level of intravascular depletion or vasomotor abnormality. Hypotension is a late
measure of intravascular fluid
finding and occurs only after 40% of the intravascular

2. Hypovolemic sho
shock.
3. In hypovolemic
is a late findi
most sensitive
status.
4. In septic shock

, antibiotics are a resuscitation


volume has been depleted. Diagnostic tests are detertheir administration should not
mined on the basis of the specific causes suspected.
Treatment
The treatment of shock is aimed at ensuring perfusion of critical vascular beds (coronary, cerebral,

medication and
be delayed.

--------------------------------------- 272
70

Blueprints Pediatrics
TABLE 7-1
Clinical Estimation of Degree of Dehydration
Mild

Moderate

<5%

5-10%

Heart rate
greatly increased
Respiratory rate
increased
Blood pressure
decreased
Skin

increased

increased

normal

normal

normal

normal (orthostasis)

Capillary refill
>3 seconds
Mucous membranes
dry
Anterior fontanelle
depressed
Eyes

<2 seconds

2-3 seconds

normal/dry

dry

normal

depressed

Tearing
absent
Appearance
sunken
Mental status
depressed
Lab values

normal/absent

absent

normal

sunken

normal

altered

Severe
Weight loss
>10%
Vital signs

Urine osmolarity
600mOsm/L
maximal
Urine specific gravity
1.020
maximal
Blood urea nitrogen
<20
high
Blood pH
normal
moderate/profound acidosis
Stage of shock
not in shock
uncompensated shock
Physical Examination
m bicarbonate concentration is

800 mOsm/L
1.025
elevated
mildly acidotic
compensated shock
Usually, the seru
decreased secondary

to metabolic acidosis. However,


There is no single physical exam or laboratory finding
that will accurately assess a patient's degree of dehymay result in alkalosis and a high
dration (see Table 7-1). It is important to remember
a result of acid loss from gastric
that a child's primary mechanism of compensating
nificant dehydration, perfusion of
for decreased plasma volume is tachycardia; hypotenmpaired. This will be reflected
sion is a very late and ominous finding.
serum blood urea nitrogen (BUN)

protracted vomiting
bicarbonate level as
secretions. With sig
the kidneys may be i
in elevations of the
and creatinine (Cr)

levels as glomerular filtration rate


falls. A BUN/Cr rati
o greater than 20 is consistent
Diagnostic Evaluation
e.
Serum electrolyte levels help guide the choice of

with prerenal failur


Treatment

fluid composition and rate of replacement. Dehydration may be isotonic, hypotonic (hyponatremic),
rapy (ORT) is the preferred
or hypertonic (hypernatremicj, depending on the
o moderate dehydration. The
nature of the fluid lost and the replacement fluids
ation recommends that
provided by the caretaker.
mEq/L sodium, 20mEq/L
Isotonic dehydration is the most common form
glucose. Commercial preparaand suggests that either compensation has occurred
te these concentrations (e.g.,
or that water losses roughly parallel sodium losses.
able. Free water may precipitate
Hypotonic (hyponatremic) dehydration is defined by
contraindicated. ORT is para serum sodium less than 130mEq/L. Children who
nsive, requiring small volumes of
lose electrolytes in their stool and are supplemented
quently. Administered correctly,
with free water or very dilute juices may present in
.
this manner. Hypertonic (hypernatremic) dehydra-

Oral rehydration the


treatment for mild t
World Health Organiz
solutions contain 90
potassium, and 20g/L
tions that approxima
Pedialyte) are avail
hyponatremia and is
ticularly labor inte
fluid given very fre
it is very effective
Severe dehydratio

n leads to life-threatening
tion (Na > 150mEq/L) is uncommon in children, but
hildren in hypovolemic shock
implies an excessive loss of free water compared with
kg intravenous boluses of
electrolyte loss (e.g., diabetes insipidus).
al saline or Ringer's lactate) until

hypovolemic shock. C
should receive 20mL/
isotonic fluid (norm

--------------------------------------- 273
Chapter 7 / Fluid, Elect
rolyte, and pH Management
71
their condition stabilizes (see Chapter 1). Clinical
found hyperglycemia or elecestimation of degree of dehydration and serum
due to an underlying pathologic
electrolyte studies tailor subsequent management.
tic ketoacidosis) may require more
Most deficits are replaced over 24 hours, with half
nt discussed elsewhere in this
given in the first 8 hours and the rest over the next

Patients with pro


trolyte disturbances
process (e.g., diabe
specialized manageme
review.

16 hours. One important exception is the child


with hypernatremic dehydration, in whom the deficit
should be replaced over 48 to 72 hours to prevent

HYPONATREMIA

excessive fluid shifts and brain edema. Ongoing losses


(usually in stool) are replaced milliliter for milliliter
sodium level less than
with intravenous fluid comparable in electrolyte
in the face of decreased,
content with that being lost.
total body sodium content. In
For example, an 18-kg infant with a normal serum

Hyponatremia (serum

sodium judged to be 10% dehydrated has lost an


ommon setting is dehydration.
estimated 2000mL of fluid (lOOOmL = 1kg). Half
syndrome of inappropriate
the deficit is replaced over the first 8 hours, with the
retic hormone (SIADH), water
balance given over the next 16 hours. Maintenance
or congestive heart failure, and
therapy must also be included. The child received a
y.
20-mL/kg bolus initially.

children, the most c

1. 2000mL + 2 = lOOOmL (one-half the total


ons
deficit); 360mL (20mL/kg) has already been
Examination
replaced. Therefore, 640 mL is given over the first

Clinical Manifestati

8 hours at 80 mL/hr. This should be added to the


ical manifestations depends on

The severity of clin

130mEq/L) may occur


normal, or increased

Other causes include


secretion of antidiu
intoxication, renal
adrenal insufficienc

History and Physical

56mL/hr the child requires to meet maintenance


dium in the extracellular space
needs. Rate = 80mL/hr + 56mL/hr = 136mL/hr.
ge from normal. Falling levels
2. The second half (lOOOmL) is replaced over
ral days are better tolerated
the next 16 hours (63 mL/hr) along with the
norexia and nausea are early,

both the level of so


and the rate of chan
that occur over seve
than rapid losses. A
nonspecific complain

ts. Neurologic findings include


maintenance rate (56mL/hr). Rate = 63mL/hr
and decreased deep tendon
+ 56 mL/hr = 119 mL/hr.
nd respiratory arrest are life-

confusion, lethargy,
reflexes. Seizures a
threatening complica

tions.
The composition of the replacement fluid varies
depending on the initial laboratory values. Replacement (and maintenance) fluid should be potassium
on
free until the patient urinates. Bicarbonate or acetate
p of hyponatremia includes
therapy may be indicated if the pH and serum
glucose, blood urea nitrogen and
bicarbonate levels remain dangerously low after the
initial boluses.
molality, liver function tests,

Diagnostic Evaluati
The laboratory worku
serum electrolytes,

creatinine, serum os
protein, and lipid l

evels. The measured serum sodium


needs to be correcte
d in the setting of hyperglycemia.
For every lOOmg/dL
rise in glucose (above "normal"
lOOmg/dL), add 1.6m
Eq Na

+ to the measured value.


KEY POINTS
Na) and specific gravity (USG) also
1. Children are more susceptible to severe dehydra.
tion than adults.

Urine sodium (U
assist in diagnosis

2. The history and physical examination are the best


determinants of the degree of dehydration.
3. Dehydration may be isotonic, hypotonic, or
ed with fluid resuscitation as
hypertonic.
. Hyponatremia due to other
4. When calculating fluid needs, remember to
d restriction and treatment of the
replace previous losses, keep up with ongoing
The cautious use of 3% hyperoutput, and provide maintenance therapy.
ted to life-threatening situations

Treatment
Dehydration is treat
discussed previously
causes requires flui
underlying disorder.
tonic saline is limi
(i.e., intractable

seizures).

--------------------------------------- 274
72

Blueprints Pediatrics
HYPERNATREMIA

SERUM K

Depressed ST segment
Hypernatremia is uncommon in children in the
Diphasic T wave
absence of dehydration (discussed earlier]. Signs
Prominent U wave
and symptoms include muscle weakness, irritability

<2.5mEq/L

and lethargy. Seizures and coma are the major


Normal
complications.
>6.0mEq/L
Tall T wave
Normal serum potassium values range from 3.5 to
Long PR interval
5.0mEq/L; a measurement of 5.5mEq/L or greater
Wide QRS duration
is considered hyperkalemia. In children, the most
Tali T wave
common cause of an abnormally high potassium

>7.5mEq/L

level is artifactual, due to the hemolysis of red


Absent P wave
>9.0mEq/L
Sinusoidal wave
cells during sample collection. Transcellular shifts
in hydrogen ions increase serum potassium without
Figure 7-1

ECG findi

ngs of hyperkalemia.
changing total body content; for every unit reduction
in arterial pH, plasma potassium increases 0.2 to
0.4mEq/L. Disorders and medications that interfere
with renal excretion of the electrolyte precipitate
true hyperkalemia.
ssion (see Figure 7-1).

and ST segment depre


Ventricular fibrilla

tion and cardiac arrest occur at


serum levels greater
than 9mEq/L.
Differential Diagnosis
Common

causes

of hyperkalemia

elude the

following

Treatment
Calcium gluconate pr

otects the heart by stabilizing


Acidosis
ane. Infusion of sodium bicar-

the cardiac cell membr


bonate or insulin (a

nd glucose) drives potassium into


Severe dehydration
Potassium-sparing diuretics
ange resins (e.g., Kayexalate)
Excessive parenteral infusion
he only measures that actually
Renal failure
the body.
Other less common but important conditions to

the cells. Cation exch


and hemodialysis are t
remove potassium from

consider include the following:


K
EY POINTS
Adrenal corticoid deficiency (i.e., Addison's
hanges associated with hyperdisease)
peaked T waves, disappearing P
Renal tubular acidosis
ng of the QRS complex.
Massive crush injury
include calcium gluconate,
Beta-blocker or theophylline intoxication
e or insulin/glucose, cation

1. Progressive ECG c
kalemia include
waves, and wideni
2. Treatment options
sodium bicarbonat
exchange resins

, and hemodialysis.
Clinical Manifestations
Paresthesias and weakness are the earliest symptoms;
flaccid paralysis and tetany occur late. Cardiac

HYPOKALEMIA

involvement correlates with specific progressive


ECG changes; T-wave elevation ("peaking") is folHypokalemia in the p
ediatric population is usually
lowed by loss of P waves, widening QRS complexesencountere,
d in ca
ses of alkalosis secondary to

--------------------------------------- 275
Chapter 7 / Fluid, Elect
rolyte, and pH Management
73
vomiting, administration of loop diuretics, or diabetic
lower than expected,
there is primary respiratory
ketoacidosis. Signs and symptoms include weaknessalkalosis,
.
tetany, constipation, polyuria, and polydipsia. Muscle

breakdown leading to myoglobinuria may compromise renal function. ECG changes are noted at levels
ons
less than or equal to 2.5mEq/L; cardiac arrhythmias

Clinical Manifestati

can occur and are more likely if the patient is being


t consistent clinical finding in

Hyperpnea is the mos


metabolic acidosis;

other signs and symptoms are


treated with digitalis. Blood pressure changes and
lying disorder. Important laboraurine electrolyte content assist in diagnosis (Figure
serum electrolytes, blood urea
7-2). Treatment consists of correcting pH (when
increased) and replenishing potassium stores.
glucose, venous or arterial blood

related to the under


tory studies include

nitrogen, creatine,
gas, and urine dipst

ick for pH and glucose. The difference between the


sums of the measured cations
(Na + + 1C) and anio
ns (Cl + HCCV), termed the
_ .METABOLIC ACIDOSIS
12 4; Table 7-2 lists condi-

anion gap, is normally


tions associated wit

h changes in the anion gap.


The extracellular fluid pH (hydrogen ion concentration) is kept in a very narrow range, largely as a result
of the bicarbonate buffer system. Hydrogen ions
(H +) combine with HCO

Treatment

3~ to form H 2CO 3, which in

turn breaks down to water and CO


nistration of sodium bicarbon-

2 (which can be

The intravenous admi


ate should be reserv

ed for cases in which the serum


expired through the lungs). The addition of excessive
and the cause is unknown
H +, the loss of HCO
3~, or abnormal pulmonary
rse. Patients receiving alkali
function can all affect this buffering system and

pH is less than 7.0


or difficult to reve

lead to acid-base disturbances.


therapy require freq
uent pH, sodium, potassium, and
Metabolic acidosis results from the loss of HCO
3~ calcium monitoring;
complications include alkalosis,
or the addition of H
+ in the extracellular fluid. It is hypokalemia, hyperna
tremia, and hypocalcemia.
the most common acid-base disorder encountered
in the pediatric population. In the presence of a
metabolic acidosis, the following formula predicts
IS _
the expected PaCO
2: PaCO 2 = 1.5 x HCCV + 8 (2).

.METABOLIC, ALKALOS

If the measured PaCO


2 is higher than expected,
is much less common than
then there is a primary respiratory acidosis. If it is
. "Contraction" alkalosis results

Metabolic alkalosis
acidosis in children
Ye

s
Blood pres
sure
tUrine pota
ssium
Renal tubular
acidosis

Skin losses
Fanconi's sy

ndrome

Gl losses
Congenital adrenal hyperplasia
High carbohydrate die
Cushing's syndrome
Enema/laxative abuse
xcess mineralocorticoid
Anorexia nervosa

ndrome
ics
cs

Barterr's sy
Antibiot
Diureti
Alkaios

is
Evaluation of
Figure 7-2
sulin
hypokalemia.

Increased in

--------------------------------------- 276
74

rints Pediatrics
TABLE 7-2
Changes in the Anion Gap
Increased Anion Gap Normal Anion Gap

Decrea

sed Anion Gap


Hypokalemia

Hypernatremic dehydration Hyperkale

Hypocalcemia

Renal tubular acidosis

Hyperc

Hypomagnesemia

Hyperalimentation

Hyperm

mia
alcemia
agnesemia
Hyperphosphatemia

Hypoal

Diarrheal dehydration

Lithiu

buminemia
m poisoning
Lactic acidosis
Diabetic ketoacidosis
Salicylate poisoning
Renal failure
Methanol poisoning
Uremia

from the loss of fluid high in H + or Cl , as may occur


Y POINTS
with protracted gastric vomiting or chronic thiazide
or loop diuretic administration. Patients with cystic
sis is a relatively common disorder
fibrosis may develop metabolic alkalosis due to
tients.
excessive electrolyte losses in the sweat. Volume
co 2 = 1.5 x HCCV + 8 (2) can
expansion and chloride replacement correct the
h between primary and secondary
alkalosis unless it is associated with disorders of
sis.
mineralocorticoid excess (e.g., renal artery stenosis);
um bicarbonate) should be used
potassium supplements are necessary in these
sis is severe or difficult to correct.
cases.

KE

1. Metabolic acido
in pediatric pa
2. The equation Pa
help distinguis
metabolic acido
3. NaHCO

3 (sodi

only when acido

--------------------------------------- 277
Chapter
ABDOMINAL PAIN
dairy food. Sickle cell disease, ulcer-

with exposure to
ative colitis, an

d Crohn's disease are chronic condiAbdominal pain is one of the most common sympin is a major symptom. More
toms the pediatrician sees, and it has a complex difrarde abdominal migraines, seizures,
ferential diagnosis. Abdominal pain may be acute or
sease, and malignancy, including
chronic/recurrent (at least three episodes within 3
as solid tumors.
months), and it may represent a surgical or medical
s the most common surgical cause
condition. Chronic/recurrent abdominal pain occurs
. Intussusception is an important
in approximately 10% of children 5 to 15 years old,
that presents with intermittent but
and less than 10% of these cases result from an
triking lethargy. Incarcerated hernia,
organic cause.
bstruction, and testicular torsion

tions in which pa
e causes inclu
Hirschsprung's di
leukemia as well
Appendicitis i
of abdominal pain
pediatric disease
severe pain and s
volvulus, bowel o
represent surgica

l emergencies. Trauma can lead to


significant intra
-abdominal injury and pain.
Differential Diagnosis
uction at any level is an important
Infectious conditions (including bacterial and viral
gastroenteritis) are the most common cause of
eteropelvic obstruction, hydro-

Urologic obstr
consideration. Ur
nephrosis, and re

nal stones can cause significant pain.

abdominal pain. Mesenteric lymphadenitis may


uses are an important part of the
cause persistent pain following an infection. Group
nosis in adolescent girls. Pregnancy
A streptococcal infections, urinary tract infections,
considered, especially if symptoms
and lower lobe pneumonias may also present with
abdominal pain. Pelvic inflammatory disease (PID) is
th an ectopic pregnancy. Dysmen-

Gynecologic ca
differential diag
should always be
are consistent wi
orrhea, ovarian c

ysts, mittelschmerz, PID, cervicitis,


an important consideration in adolescent females.
d ovarian or adnexal torsion are all
Viral hepatitis, infectious mononucleosis, and herpes
s in this population.
zoster are more uncommon infections that may need
uses of abdominal pain are unto be considered.
Noninfectious medical diseases are less common
n. True malingering is unusual, as

endometriosis, an
potential problem
Psychiatric ca
common in childre
are conversion di

sorders. However, many children


and include both primary gastrointestinal and geniominal pain in the setting of
tourinary pathology and systemic diseases. Cholecysy in the context of school, and mild
titis, pancreatitis, gastritis, and peptic ulcer disease
also can be seen in children with
are uncommon in children, but warrant consideration. Abdominal pain is a primary feature in HenochSchonlein purpura, but also may be seen in other
vasculitides, including Kawasaki's disease, polyarteriations
tis nodosa, and lupus erythematosus. If the pain is
recurrent, the differential diagnosis must be
expanded. Constipation and functional abdominal
d localize the pain and determine
pain are frequent complaints evaluated by a pediatemporal characteristics and its exactrician. Lactase deficiency results in recurrent pain
eviating factors. With "inflammatory"

do experience abd
stress, especiall
intermittent pain
depression.
Clinical Manifest
History
The history shoul
its quality and
erbating and all

--------------------------------------- 278
76

Blueprints Pediatrics

pain, the child tends to lie still, whereas with


motion tenderness is consistent
"colicky" pain, the child cannot remain still. Colicky

performed. Cervical
with PID.

pain usually results from obstruction, whereas


inflammatory pain is caused by an infected or perforated organ or viscus. It is important to ascertain
n
whether the child has any drug or food allergies or
strategy is dictated by the history

Diagnostic Evaluatio
The diagnostic test

has had previous abdominal surgeries. After laparotomy, small bowel obstruction becomes more
physical examination. If the cause
likely. Pain may be accompanied by anorexia,
ht to be a surgical one, then a
nausea, emesis, diarrhea, or constipation. Bilious
n should be obtained. Of the
emesis indicates obstruction (or less commonly,
te or chronic/recurrent
ileus), whereas bloody emesis points to an upper
ical causes are the most likely to
GI source (esophagitis, gastritis, or duodenitis).
tervention.
Bloody or mucinous diarrhea suggests bacterial
count with manual differential,
enterocolitis.
nd chemistries, amylase, lipase,
Stooling characteristics are important, because
tion, urinalysis, and radiographic
constipation is a common etiology of chronic abdomrformed if there has been
inal pain. Dysuria and abdominal pain are indicative
an acute surgical condition is
of a urinary tract infection, whereas sore throat and
uld also be typed for possible

and findings of the


of the pain is thoug
surgical consultatio
common causes of acu
abdominal pain, surg
require immediate in
A complete blood
serum electrolytes a
stool guaiac examina
studies should be pe
abdominal trauma or
suspected. Blood sho
transfusion. A bariu

m swallow with upper gastroinabdominal pain implicate pharyngitis. There may be


, a pH probe, or an endoscopic
a history of trauma. Obtaining a good sexual history
sed to evaluate for reflux.
in the adolescent is critical. If there is a history of
iral gastroenteritis is the most
vaginal discharge and fever, PID should be condies need be performed, but if
sidered. Inquiring about ill contacts can give helpful
tis is being considered, stool
clues to the diagnosis, because viral gastroenteritis is
or culture. Group A streptococquite contagious and very common. A family history
PID require appropriate cultures.
of lactose intolerance, Crohn's disease, ulcerative
of constipation, abdominal radicolitis, or irritable bowel syndrome increases the likeated. To diagnose a urinary tract
lihood of these diagnoses because they are genetically
sis and urine culture should be
based. Changes in the child's environment (home,

testinal examination
examination may be u
When uncomplicated v
likely cause, no stu
bacterial enterocoli
should be obtained f
cal pharyngitis and
In some severe cases
ographs may be indic
infection, a urinaly
performed.

friends, school) or behavior (poor school performance, increasingly argumentative) may suggest that
the abdominal pain is not the result of organic
disease.

Treatment
Treatment is directe

d at the underlying cause of

Physical Examination
the pain. Surgical p
roblems are treated accordingly.
The goal of the abdominal examination is to ascerGroup A streptococca
l pharyngitis, urinary tract
tain whether the child has an abdominal process thainfectionst
, and PID
require appropriate antibiotics.
requires surgical intervention. Watching the child
Individuals with lac
tase deficiency benefit from a
walk, climb onto the examination table, and interaclactose-fret
e diet o
r exogenous lactase replacement.
with both parents and staff before formally examinPatients with reflux
esophagitis benefit from small,
ing the child's abdomen helps one to gain an apprefrequent meals (ra
ther than infrequent large ones),
ciation for the degree of incapacitation or emotionasittinl
g upright for 3
0 minutes after a meal or sleepoverlay that may be present. The abdomen should bine
g at a 45-degree a
ngle after eating, avoidance of late
inspected, auscultated, and palpated. Peritoneal signs
evening meals, a pro
kinetic agent, and an H
2-blocker
include rebound tenderness, guarding, psoas or obtuand/or proton pump a
ntagonist. Children with
rator signs, and rigidity of the abdominal wall. Unlesabdominas
l pain e
xacerbated by stress require
the diagnosis is thought to be uncomplicated viral
patience, reassuranc
e, and in rare cases professional
gastroenteritis, a rectal examination should be
psychiatric assistan
ce. Constipation can be treated
performed to detect tenderness or hard stool anwitd
h prune juice, s
enna, Colace, mineral oil, or
to obtain stool for guaiac testing. If the patient is an
lactulose. In some c
ases, disimpaction, cathartics, or
adolescent female, a pelvic examination should be
enemas may be requir
ed.

--------------------------------------- 279
Ch
apter 8 / Gastroenterology

77
in perforation, the

patient should be given ampicillin,


KEY POINTS
gentamicin, and metr
onidazole (Flagyl) to treat peri1. Determine whether the pain is acute or chronic/
nal flora. The mortality rate rises
recurrent and whether a medical, surgical, or
erforation.
nonorganic disorder is most likely.
2. If the patient is an adolescent female, genitourinary pathology must be considered, and a pelvic

tonitis from intesti


significantly with p

KE
Y POINTS
exam should be performed.
1. Appendicitis is
the most common indication for
abdominal surge
ry in childhood.

2. Fever, emesis,
anorexia, and diffuse periumbilical
Appendicitis
itially; the pain and abdominal
Appendicitis is the most common indication for
lize to the right lower quadrant
abdominal surgery in childhood. Appendicitis results
al peritoneum becomes inflamed.
from bacterial invasion of the appendix, which is
nd tenderness, and obturator and
more likely when the lumen is obstructed by a
commonly found.
fecalith, parasite, or lymph node. Appendicitis occurs
most frequently in children between 10 and 15 years
of age. Less than 10% of patients are younger than
5 years of age.

pain develop in
tenderness loca
when the pariet
Guarding, rebou
psoas signs are

Intussusception

Intussusception resu
lts from telescoping of one part
Clinical Manifestations
of the intestine int
o another. Intussusception causes
Classically, fever, emesis, anorexia, and diffuse periimpaired venous retu
rn, bowel edema and ischemia,
umbilical pain develop. Subsequently, pain and
necrosis, and perfor
ation. It is one of the most
abdominal tenderness localize to the right lowecommor
n causes of
intestinal obstruction in infancy.
quadrant as the parietal peritoneum becomes
Most intussusception
s are ileocolic; the ileum invagiinflamed. Guarding, rebound tenderness, and obturanates into the colo
n at the ileocecal valve. A previtor and psoas signs are commonly found. The appenous viral infection
may cause hypertrophy of the
dix tends to perforate about 36 hours after pain
Peyer's patches or m
esenteric nodes, which are
begins. The incidence of perforation and diffuse
hypothesized to act
as the lead point in intussuscepperitonitis is higher in children younger than 2 years,
tion. A specific lea
d point is identified in only about
when diagnosis may be delayed. Atypical presenta5%of cases but sho
uld be sought in neonates or in
tions are common in childhood, especially with
children older than
5 years. A lead point is virtually
retrocecal appendicitis, which may present with perinever demonstrated
in children older than neonates
umbilical pain and diarrhea. Retrocecal appendicitis
but younger than 2 y
ears. Recognizable lead points
usually does not induce right lower quadrant pain
in intussusception i
nclude Meckel's diverticulum, an
until after perforation. Bacterial enterocolitis caused
intestinal polyp, ly
mphoma, or a foreign body. Intusby Campylobacter and Yersinia may mimic appensusception has als
o been associated with Henochdicitis because both can result in right lower quadSchonlein purpura (H
SP), but in this setting is
rant abdominal pain and tenderness. Diagnosis of
usually ileal-ileal.
It can be very difficult to distinappendicitis is established clinically by history and by
guish this surgical
cause from the nonsurgical
physical examination, which should include a rectainflammatorl
y abdo
minal pain seen in HSP.

examination to detect tenderness or a mass. A moderately elevated white blood cell count with a left
ons
shift is often seen in appendicitis. A plain film of the
irritability, colicky pain, and
abdomen may demonstrate a fecalith. Abdominal
sed with relatively normal
ultrasound may demonstrate the inflamed appendixperiods,
eeding occurs in 80% of patients
but computed tomography scans have a higher yield.
he form of the classic "currant

Clinical Manifestati
Violent episodes of
emesis are intersper
. Rectal bl
but only rarely in t
jelly" stools (stool

s containing bright red blood and


Treatment
mucus). The degree o
f lethargy demonstrated by the
Laparotomy and appendectomy should be perchilg. A tubular mass is palpable in
formed before perforation. When appendicitis resultabous
nts. A plain abdominal film may

d may be strikin
t 80% of patie

--------------------------------------- 280
78
Blueprints Pediatrics
show a paucity of gas in the right lower quadrant or
ons
evidence of obstruction with air-fluid levels. A
barium enema or air enema demonstrates a coiledspring appearance to the bowel, which is diagnostic.
ry should differentiate between
Stool should be tested for occult blood.
pitting up" (gastroesophageal

Clinical Manifestati
History
In infants the histo
true vomiting and "s
reflux) and whether

the emesis is acute or chronic.


Treatment
e (bloody or bilious), amount,
Fluid resuscitation with normal saline or lactated
esis are important. Emesis
Ringer's solution is usually necessary. Hydrostatic
g in the infant is probably gasreduction with barium enema or pneumatic reduc. If the emesis is projectile and
tion with air enema is successful in 75% of cases if
months old, pyloric stenosis must
performed in the first 48 hours, and is successful
weight gain and emesis may indiin 50% of cases if performed in the first 48 hours.
s or metabolic disorder. Macrolide
Peritoneal signs are an absolute contraindication to
n to cause emesis and diarrhea;

Frequency, appearanc
and timing of the em
shortly after feedin
troesophageal reflux
the child is 1 to 3
be considered. Poor
cate pyloric stenosi
antibiotics are know
chemotherapeutic age

nts and some toxic ingestions


this procedure. Laparotomy and direct reduction
s. If the child has a ventricuis indicated when reduction by enema is either
, vomiting may be a sign of shunt
unsuccessful or contraindicated. The immediate
eased intracranial pressure.

commonly cause emesi


lar-peritoneal shunt
obstruction and incr

recurrence rate is about 15%. When a specific lead


or headache or both may indipoint is identified, the recurrence rate is higher.
process. Diarrhea, emesis, and

Emesis with seizure


cate an intracranial
fever are seen with

gastroenteritis. Fever, abdominal


pain, and emesis are
typical for appendicitis, whereas
KEY POINTS
bdominal pain are seen with
1. Most intussusceptions are ileocolic, in which the
on. Emesis and syncope may result
ileum invaginates into the colon at the ileocecal
valve.
2. Violent episodes of irritability, colicky pain, and
ion, the initial assessment
emesis are interspersed with relatively normal
child's vital signs and hydration
periods. Rectal bleeding may occur, but only rarely
mptoms of dehydration are
in the form of the classic "currant jelly" stools.
7. A bulging fontanelle or
3. Hydrostatic reduction with barium enema or
es increased intracranial prespneumatic reduction with air enema is successful
the emesis. Emesis is common in
in 75% of cases.
is. The lung fields should be aus-

bilious emesis and a


intestinal obstructi
from pregnancy.
Physical Examination
On physical examinat
should focus on the
status. Signs and sy
discussed in Chapter
papilledema implicat
sure as the cause of
infectious pharyngit
cultated for crackle

s or an asymmetric examination
to rule out pneumoni
a. Emesis and vaginal discharge
in the female adoles
cent warrant a pelvic examinaEMESIS
PID. The abdominal examination

tion to evaluate for


should focus on bowe

l sounds and the presence of


Vomiting is one of the most common presenting
ss, or masses. Hypoactive bowel
symptoms in pediatrics and can be caused by both
ileus or obstruction, whereas
gastrointestinal and nongastrointestinal pathologies.
unds suggest gastroenteritis.
Complications of severe, persistent emesis include
emesis may indicate intussusdehydration and hypochloremic, hypokalemic
cy. Tenderness on exam is
metabolic alkalosis. Forceful emesis can result in a
dicitis, pancreatitis, cholecystitis,
Mallory-Weiss tear of the esophagus at the gastroesophageal junction or erosion of the gastric cardia;
chronic emesis can result in distal esophagitis.
on
Differential Diagnosis
studies depend on the suspected

distention, tenderne
sounds may indicate
hyperactive bowel so
Abdominal mass with
ception or malignan
suggestive of appen
peritonitis, or PID.
Diagnostic Evaluati
Specific laboratory
cause. Appropriate

cultures and a complete blood


Table 8-1 lists the most common causes of vomiting
ifferential should be sent if an

count with manual d

in the pediatric population.


deemed likely and the vomiting is

infectious cause is

--------------------------------------- 281
Ch
apter 8 / Gastroenterology
79
TABLE 8-1
Differential Diagnosis of Vomiting in Children
Infectious

Gastrointestinal: Infa

Viral gastroenteritis (especially

Gastroesophageal reflu

rotavirus and Norwalk virus)

Cow or soy milk protei

Bacterial enterocolitis/sepsis

Bowel obstruction"

nt
x
n intolerance
Hepatitis

Duodenal atresia

Food poisoning

Pyloric stenosis

Staphylococcus aureus

Malrotation with or

Clostridium perfringens

Incarcerated hernia

Salmonella

Intussusception

without volvulus

Pelvic inflammatory disease

Meckel's diverticulu

Peritonitis

Hirschsprung's disea

Pharyngitis

Gastrointestinal: Chil

Pneumonia

Appendicitis

Otitis media

Bowel obstruction

m with torsion
se
d

Tonsillitis
Malrotation
Urinary tract infection

Incarcerated hernia

Metabolic

Intussusception

Diabetic ketoacidosis

Meckel's diverticulu

Inborn errors of metabolism

Adhesions

m with torsion
Adrenal insufficiency

Post-traumatic obstr

uction13
Renal failure

Pancreatitis

Hepatic failure

Hepatitis

Central Nervous System

Cholecystitis

Increased intracranial pressure


Ventricular-peritoneal shunt

Respiratory
Reactive airway diseas

e
malfunction

Oncology

Meningitis
Chemotherapeutic agent
s
Encephalitis

Toxic Ingestion

Labyrinthitis
Migraine

Salicylates
Theophylline

Reye's syndrome

Caustic agents

Seizure

Digoxin

Tumor
Lead
Gynecologic

Emotional

Pregnancy
"Psychogenic"
Bulimia
"Malrotation with or without volvulus is much more likely i
n an infant than in a child.
b From duodenal hematoma, ruptured viscus, or superior mes
enteric artery syndrome.
significant. A chest radiograph will rule out pneushould be obtained t
o rule out urinary tract infection
monia. If a surgical process within the abdomen is
and assess degree of
dehydration.
considered, upright and supine abdominal films
should be obtained, along with a complete blood

Treatment

count and electrolyte and chemistry panels. Amylase


and lipase should be sent to detect pancreatitis. IIff
the cause appears
to be a self-limited nonsurgical
vomiting is prolonged or the patient is significantly
infectious process (
viral gastroenteritis or bacterial
dehydrated, electrolytes will help guide replacemenenterocolitist
) and
the patient is not significantly
therapy. An ammonia level, serum amino acids, and
dehydrated, outpatie
nt therapy is indicated. Oral
urine organic acids should be sent if metabolic
rehydration therapy,
which is discussed in Chapter 7,
disease is suspected. Urinalysis and urine culture
is recommended for d

ehydrated infants. For older

--------------------------------------- 282
Poisoning, Burns,
and Injury
Prevention
Nowhere does the old adage "an ounce of prevention
is
is worth a pound of cure" resonate more true than

Differential Diagnos
The possibility of t

oxicologic ingestion should be


in pediatrics. Together, accidents and injuries are the
tient presenting with altered
largest cause of morbidity and mortality in children.
behavior changes, seizures,
When an untoward event occurs, timely evaluation
and treatment may limit disability and preserve
.
quality of life.

considered in any pa
mental status, acute
arrhythmias, or coma
Diagnostic Evaluatio

n
Screening studies sh
ould include a pulse oxygenation
check, dextrose-stic
k, electrocardiogram, serum
electrolytes and osm
olarity, and a venous blood
Poisoning is one of the more common pediatric
Blood and urine toxicology
medical emergencies, resulting in over 2 million
emergency visits a year. About 80% of childhood poihelpful; the clinician should
sonings occur in children younger than 5 years.
nces in particular are screened
These tend to involve only one substance and may
denote either accidental ingestion or (more rarely)
abuse by caretakers. Adolescents account for the
remaining 20%; such ingestions are usually intentional, represent a suicide attempt or gesture, and
may involve multiple substances. Recreational drug
based on the estimated

gas to determine pH.


screens are variably
inquire which substa
for.
Treatment
Treatment should be
maximal potential do

se ingested. Children with siguse in this older population can result in unintenand patients who are medically
tional but fatal overdoses.

nificant ingestions
unstable require dil

igent observation and management of their airway


, breathing, and circulation.
Induction of emesis
with syrup of ipecac is approClinical Manifestations
if the substance was recently

priate in some cases


ingested.* Gastric l

avage both removes and dilutes


History and Physical Examination

The history should include the substance ingested,


ll fragments recovered by either

stomach contents. Pi
method may aid in di

agnosis. Activated charcoal by


when, how much, and subsequent behavior. The
tube minimizes absorption by
characteristic clinical manifestations and treatment
e and hastening its elimination;
of the most common poisonings in children and
adolescents are discussed in Table 2-1.

mouth or nasogastric
binding the substanc

* Ipecac is specific
ally contraindicated for ingestions of
hydrocarbons and cau
stic acids/bases.

--------------------------------------- 283
80
;prints Pediatrics
children, fluids should be encouraged, with cautious
Y POINTS
advancement to a soft, bland diet as tolerated. Children who are severely dehydrated or unable to effecs is an important cause of gastric
tively orally hydrate themselves should be admitted
ion and emesis in the first 2
to the hospital.
with a peak incidence at 2 to 4
A surgical consultation must be obtained if indicated. If ventricular-peritoneal shunt malfunction is
ilious vomiting is the cardinal
believed to be causing emesis, obtain a computed
disorder.
tomography of the head, a shunt series, and a neurohould take place as soon as the
surgical consultation.
lies have been satisfactorily

KE
1. Pyloric stenosi
outlet obstruct
months of life,
weeks of life.
2. Projectile nonb
feature of this
3. Pyloromyotomy s
metabolic anoma

corrected.
KEY POINTS
1. Most cases of emesis are caused by gastroeMalrotation and Volv
ulus
sophageal reflux, acute gastroenteritis, or systemic
disorders such as tonsillitis, otitis media, or urinaryMalrotation occurs w
hen the small intestines abnortract infection.
mally rotate in uter
o, resulting in malposition in
2. Most children with uncomplicated viral gastroenthe abdomen and abno
rmal posterior fixation of the
teritis and mild dehydration can be treated as
mesentery. When the
intestine attaches improperly
outpatients with oral rehydration therapy.
to the mesentery, it
is at risk for twisting on its
vascular supply; the
twisting phenomenon is called
volvulus. The most c
ommon age of presentation is
Pyloric Stenosis
under 1 month.
Pyloric stenosis is an important cause of gastric
Clinical Manifestati
ons

outlet obstruction and vomiting in the first 2 months


lways includes bilious emesis. In
of life. Peak incidence occurs at 2 to 4 weeks of life,
st history of attacks is occasionally
with an incidence of 1 in 500 infants. Male infants
xamination may reveal abdominal
are affected 4:1 over female infants, and pyloric
ained emesis or stool, and shock.
stenosis occurs more frequently in infants with a
s typically show gas in the
family history of the condition. Recent evidence
ty of air in the intestine. An
suggests that erythromycin therapy may precipitate
al series with small bowel followpyloric stenosis.
diagnosis by illustrating the

The history almost a


older children, a pa
elicited. Physical e
distention, blood-st
Abdominal radiograph
stomach with a pauci
upper gastrointestin
through confirms the

abnormal position of
the ligament of Treitz and the
Clinical Manifestations
Projectile nonbilious vomiting is the cardinal feature
cecum. A positive st
ool guaiac examination is a poor
of the disorder. Physical findings vary with the severprognostic sign, ind
icating significant bowel ischemia.
ity of the obstruction. Dehydration and poor weighTreatmentt
gain are common when the diagnosis is delayed.
Hypokalemic, hypochloremic metabolic alkalosis
Operative correction
of the malrotation and the
with dehydration is seen secondary to persistent
volvulus should be u
ndertaken as soon as possible,
emesis in the most severe cases. The classic finding of
K
EY POINTS
an olive-sized, muscular, mobile, nontender mass in
the epigastric area occurs in most cases. Visible
1. Malrotation occ
urs when the intestines abnorgastric peristaltic waves may be seen. Ultrasonogramally rotate in
utero, resulting in malposition in
phy reveals the hypertrophic pylorus.
the abdomen and
abnormal posterior fixation of
the mesentery.
When the intestine attaches
Treatment
improperly, it
is at risk for volvulus.
Initial treatment involves nasogastric tube placement
2. An upper gastro
intestinal series with small bowel
and correction of dehydration, alkalosis, and elecfollow-through
confirms the diagnosis by controlyte abnormalities. Pyloromyotomy should take
firming the abn
ormal position of the ligament of
place as soon as the metabolic anomalies have been
Treitz and the
cecum.
satisfactorily corrected.

--------------------------------------- 284
Ch
81
apter 8 / Gastroenterology
because bowel ischemia, metabolic acidosis, and
y and whether it radiates and is

location and severit

sepsis can progress quickly to death.


tent. Burning epigastric or chest

constant or intermit
pain is probably ref

lux in the adolescent, especially


if it occurs after m
eals when the patient lies down.
_ GASTROESO PH AGIE AL_R_EF LUX
Physical Examination
Gastroesophageal reflux (GER) is the regurgitation
hysical examination of the child
of stomach contents into the esophagus due to an
l reflux is normal. In severe
incompetent lower esophageal sphincter. A small
present with failure to thrive.
degree of reflux is common in all infants, and it is

In most cases, the p

only infants who have moderate to severe chronic


n
reflux that tend to come to the pediatrician's atten-

Diagnostic Evaluatio

tion. In this group, complications include failure to


d reflux is made by the characthrive, aspiration pneumonia, esophagitis, choking or
moderate or severe reflux, the diagapneic episodes, hematemesis, anemia, and chronic
confirmed by barium swallow
fussiness.
estinal examination, pH probe

The diagnosis of mil

with gastroesophagea
cases, infants will

teristic history. In
nosis of GER may be
with upper gastroint
placement in the eso

phagus, or upper gastrointestiDifferential Diagnosis


vere reflux or projectile emesis is

nal endoscopy. If se
present in the small

infant, gastric (pyloric stenosis)


Incompetence of the lower esophageal sphincter may
nal stenosis or atresia, malrotabe the result of prematurity, esophageal disease,
obstruction should be considobstructive lung disease, overdistention of the
ltrasound and barium swallow
stomach caused by overeating, or medication (theom normal anatomy and normal

or intestinal (duode
tion with volvulus)
ered. An abdominal u
are useful to confir
gastric emptying.

phylline). If the infant is having forceful emesis or


ld to moderate reflux generally
projectile vomiting, reflux is not the most likely
cause, and the differential diagnosis for emesis just
complete blood count and
discussed should be considered.
n severe reflux, a hypochloremic,
The differential diagnosis for GER in the adolesc alkalosis may exist; these
cent may include pneumonia, costochondritis, periive and may have pyloric stenosis
carditis, pulmonary embolism, arrhythmias, ischemia

The child with mi

has an unremarkable
electrolyte panel. I
hypokalemic metaboli
children fail to thr
rather than GER.

due to an anomalous coronary artery, pancreatitis,


ination is abnormal in the prescholecystitis, peptic ulcer disease, and anxiety.
est radiograph should be obtained

If the chest exam


ence of reflux, a ch
to look for aspirati

on pneumonia or changes due to


recurrent aspiration
.
Clinical Manifestations
History

Treatment

It is important to determine if the infant is "spitting


uld receive small, frequent
up" or having projectile emesis and if the emesis is
ght position and be maintained
bloody or bilious. One of the most common causes
position for at least 20 minutes
of GER is overfeeding, so a careful history should
ds should be thickened with
include what formula the infant eats, how it is mixed,
sures fail, metoclopramide may be
how much the infant eats during each feeding, and
ric motility and increase the rate
how often the child is fed. If the emesis is indepenIf esophagitis is suspected, an H 2dent of meals, it is probably not reflux. A history of
idine) or a proton pump inhibitor
coughing, gagging, and arching of the back with
ay be useful.
extensor posturing during feeding may result from
dical management fails, a Nissen
direct aspiration, whereas the presence of these
e necessary. In this procedure,
symptoms soon after feeding may suggest GER. In
omach is wrapped around the
severe reflux, the infant may have poor weight gain.
increase lower esophageal sphincIn the older child, GER is often manifested as

Infants with GER sho

epigastric abdominal or chest pain. Define the pain's


adolescents with reflux should

feedings in the upri


in the prone head-up
after a feeding. Fee
cereal. If these mea
used to improve gast
of gastric emptying.
blocker (e.g., ranit
(e.g., omeprazole) m
In cases where me
fundoplication may b
the fundus of the st
distal esophagus to
ter pressure.
Older children or

--------------------------------------- 285
82
Blueprints Pediatrics
also have small, frequent meals, eat slowly, and mainsevere disease. Cer
tain medications, especially antitain the upright position after meals. Meals after 7
biotics and chemothe
rapeutic agents, may cause diarP.M. should be discouraged, and antacids may be
rhea. Viral gastroen
teritis is highly contagious, so sick
useful.
contacts are likely.
If a close contact of the child has
contact with raw pou
ltry, salmonella should be
considered. Foul-sme
lling diarrhea that floats in the

KEY POINTS
atorrhea and may result from cystic
1. Most cases of gastroesophageal reflux occur in
bsorption from other causes.
the infant and adolescent populations and will
not require medical intervention.
f dehydration are discussed in
2. Most infants with moderate GER respond to small,
itical in the evaluation of a
frequent feedings in the upright position, thicka. An attempt should be made to
ened feeds with rice cereal, and maintenance of
of dehydration in order to
the prone head-up position for at least 20
minutes after feeding.
bdominal examination focuses
3. The most common symptoms of GER in the
the presence of distention, tenadolescent are burning epigastric pain and
Hypoactive bowel sounds point
chest pain.
ction. Hyperactive sounds are

toilet is likely ste


fibrosis or fat mala
Physical Examination
Signs and symptoms o
Chapter 7 and are cr
patient with diarrhe
determine the degree
guide therapy. The a
on bowel sounds and
derness, or masses.
to intestinal obstru
consistent with gast

roenteritis. Abdominal mass


with diarrhea could
indicate intussusception or
malignancy.
DIARRHEA
Diagnostic Evaluati
on
Diarrhea is defined as an increase in the frequency
and the water content of stools. Viral gastroenteritis
ild with diarrhea, inspecting the
accounts for 70% to 80% of acute diarrhea in North
evaluation and the treatment plan.
America. The complications of acute diarrhea
y of blood or mucous or both in

When evaluating a ch
stool is critical to
If there is a histor
the stool, bacterial

cultures should be obtained.


include dehydration, electrolyte and acid-base disavirus and adenovirus are available.
turbance, bacteremia and sepsis, and malnutrition
% of infant diarrhea during the
in chronic cases. Enteritis refers to small bowel
inflammation, whereas colitis refers to large bowel
athogen is being considered and the
inflammation.

Rapid tests for rot


Rotavirus causes 65
winter months.
If a bacterial p
child is younger th

an 3 months, a blood culture


should be performed
because the incidence of secDifferential Diagnosis
rom salmonella enterocolitis is

ondary bacteremia f
high in this age gr

oup. When there is a history of


Table 8-2 lists the most common causes of diarrhea
le antibiotic use, a Clostridium
in the pediatric population of the Western world.
ay should be sent. Stool ova and

long-term or multip
difficile toxin ass

parasites should be
tested for children with chronic
diarrhea, for those
with a history of foreign travel or
Clinical Manifestations
for immunocompromised
History

recent camping, and


children with diarr

hea. If the child appears toxic, or


The history should ascertain whether the diarrhea is
dehydration is noted, a complete
acute or chronic/recurrent and establish the frenual differential, electrolyte
quency, appearance (bloody, mucosal, currant jelly),
alysis is indicated. Urinary tract
amount, consistency, and color of the diarrhea.
ted by urine dipstick, urine
Dietary indiscretions and manipulations may result
ne culture.
in diarrhea. Small infants will have diarrhea when
they are fed concentrated formula. If the child has
traveled out of the country, consider a parasitic or
bacterial enterocolitis. Weight loss or lack of weight
iral gastroenteritis without siggain in association with diarrhea indicates more
n, the current recommendations

moderate to severe
blood count with ma
panel, and urine an
infection is evalua
microscopy, and uri

Treatment
For uncomplicated v
nificant dehydratio

--------------------------------------- 286
Ch
83
apter 8 / Gastroenterology
TABLE 8-2
Differential Diagnosis of Diarrhea in Children
Acute Diarrhea
rrent Diarrhea
Intra-intestinal Infections
Viral gastroenteritis
emic syndrome
Rotavirus
Enterovirus
lein purpura
Adenovirus
Norwalk agent
Bacterial enterocolitis
Shigella
Salmonella
ium
Yersinia
inal
Campylobacter
E. col! (enteroinvasive/enteropathogenic)
intolerance
C. difficile
N. gonorrhoeae
olitis

Chronic Recu
Renal
Hemolytic ur
Vasculitis
Henoch-Schon
Infectious
Parasites
Amoebiasis
Giardiasis
Cryptosporid
Gastrointest
Cow/soy milk
Overfeeding
Ulcerative c
Crohn's dise

ase
C. trachomatis

Hirschsprung

's disease
Extra-intestinal Infections

Lactase defi

ciency
Otitis media
Irritable bo
wel disease
Urinary tract infection
Gastrointestinal

Encopresis
Excessive fr

Intussusception

Cystic fibro

Appendicitis
Hyperconcentrated infant formula
Cystic fibrosis

Celiac sprue
Allergy
Food allergi

uctose intake
sis

es
Toxic Ingestion
Iron, mercury, lead, fluoride ingestion
Medication Induced
Any antibiotic, chemotherapeutic agents
are to feed through the diarrhea. The continuation of
appear toxic, the in
fant can be reexamined and
normal feedings results in less intestinal denudement,
observed at home. If
the stool culture is positive and
improved nutritional absorption, and a faster return
the infant is febril
e, the infant's age determines
to a normal stooling pattern. If the infant is also vomtherapy:
iting, replace one feed with Rice-Lyte or Pedialyte to
The infant younger t
calm the stomach and then return to normal feeds.
han 3 months is admitted to
Often, the parents need to give smaller feedings more
the hospital; a b
lood culture is obtained, and intrafrequently to accommodate the intestinal irritation
venous antibiotic
s are started. A lumbar puncture
from the gastroenteritis and to minimize emesis.
and urinalysis sh
ould also be considered in this age
Infants who do not tolerate their regular formula but
group.
are not significantly dehydrated or toxic appearing
The infant older tha
n 3 months is admitted to the
may be orally rehydrated at home. See Chapter 7 for
hospital; a blood
culture should be sent, but antidetails on oral rehydration therapy.
biotics may be wi
thheld pending the results of the
For the infant 0 to 12 months old with diarrhea
blood culture.
for more than 5 days, with suspected enterocolitis or
Any infant with a po
sitive stool culture who
exposure to salmonella, a stool culture should be perlooks toxic or ha
s a positive blood culture is
formed. A blood culture should be performed if the
admitted for intr
avenous antibiotics and evaluation
infant is younger than 3 months. If the stool culture
for pyelonephriti
s, meningitis, pneumonia, and
is positive and the infant is afebrile and does not
osteomyelitis.

--------------------------------------- 287
84

Blueprints Pediatrics
Older children with viral gastroenteritis should be
training but is usually caused by

conflicts in toilet

encouraged to drink isotonic fluids. Any fluid with a


pain on defecation,
which creates a fear of defecation
high carbohydrate load should be diluted with wateran.
d further retenti
on. Voluntary withholding of stool
Admission is indicated for the child who is more
increases distention
of the rectum, which decreases
than 5% dehydrated and cannot effectively orally
rectal sensation, ne
cessitating an even greater fecal
rehydrate himself. See Chapter 7 for details on
mass to initiate the
urge to defecate. Complications
intravenous rehydration.
of stool retention i
nclude impaction, abdominal pain,
Viral gastroenteritis requires no pharmacologic
overflow diarrhea re
sulting from leakage around the
therapy. Antidiarrheal medications are contraindifecal mass, anal fis
sure, rectal bleeding, and urinary
cated because they may cause toxic megacolon. In
tract infection caus
ed by extrinsic pressure on the
general, antibiotics are not indicated for bacterial
urethra.
enterocolitis. Exceptions include colitis caused by
Encopresis, which
is daytime or nighttime soiling
Salmonella typhi, Shigella, and C. difficile. A summarby formey
d stools
in children beyond the age of
of the bacterial pathogens and their treatment is
expected toilet trai
ning (4-5 years), is another
given in Chapter 12. Parasitic gastrointestinal infeccomplication of cons
tipation. In older children, it is
tions should be treated with the appropriate antimiimportant to ask
specifically about soiling, because
crobial. Antibiotic-related diarrhea remits when the
such information may
not be expressed due to
offending antibiotic is discontinued. Intussusception
embarrassment. These
children are unable to sense
is treated by hydrostatic reduction with barium
the need to defecate
because of stretching of the
enema, air enema, and/or surgery.
internal sphincter b
y the retained fecal mass.
Organic causes of
failure to defecate include
decreased peristalsi
s, decreased expulsion, and
anatomic malformatio
n. Organic etiologies are delinKEY POINTS
1. The most common cause of diarrhea in children is
eated in the followi
ng section.
viral gastroenteritis.
2. Bacteremia is more likely in infants younger than
Differential Diagno
sis
3 months with bacterial enterocolitis.
N anorganic
Functional constipa
3. Most children with uncomplicated viral gastroention (intentional withholding)
teritis or bacterial enterocolitis can be rehydrated
Dysfunctional toile
t training
orally.
4. Do not use antidiarrheal medications in children
Organic
with acute diarrhea.
Dietary: Low-fiber
diet, inadequate fluid intake
5. Feed through diarrhea in infants. Recovery is
Gastrointestinal: F
unctional ileus, Hirschsprung's

faster because there is less sloughing of the


nosis, rectal abscess or fissure, stricintestinal mucosa.
crotizing enterocolitis (NEC),

disease, anal ste


ture following ne
collagen vascular

diseases
Drugs or toxins: Le
ad, narcotics, phenothiazines,
vincristine, anti
cholinergics
CONSTIPATION
ngomyelocele, tethered spinal

Neuromuscular: Meni
cord, infant botu

lism, absent abdominal muscles


(prune belly synd
rome)
Constipation is defined as infrequent passage of hard,
ibrosis, hypothyroidism, hypodry stools. Constipated infants fail to empty the
cemia
colon completely with bowel movements and over
oidism
time stretch the smooth muscle of the colon, resulting in a functional ileus. In contrast to constipation,
obstipation is the absence of bowel movements.
ions
Beyond the neonatal period, the most common cause
(90%-95%) of constipation is due to voluntary withl Examination
holding or functional constipation. Intentional withed by constipation is often
holding is often noted from the very beginning of
t. The pain may be accompanied
toilet training. A family history of similar problems
ting is unusual. Stools are hard,
is often obtained. Stool retention may be due to
and infrequent. Particular foods can

Metabolic: Cystic f
kalemia, hypercal
Endocrine: Hypothyr

Clinical Manifestat
History and Physica
Abdominal pain caus
diffuse and constan
by nausea, but vomi
difficult to pass,

--------------------------------------- 288
Ch
apter 8 / Gastroenterology 85
exacerbate constipation. Discussion of the psycholyethylene glycol-electrolyte
logical state of the child will help determine whether
ful osmotic cathartic. In some
voluntary withholding is the most likely diagnosis. A
pation due to psychological causes
medication history is essential. If a history of diarrhea
or psychotherapy.
or fecal spotting alternating with periods of constipation exists, a diagnosis of Hirschsprung's disease or
encopresis should be entertained.
Y POINTS
On examination, the abdomen is diffusely uncomdefined as infrequent passage of
fortable rather than tender, and the left colon may be
s. Constipated patients fail to com-

needed. GoLYTELY (po


solution) is a power
severe cases, consti
requires counseling

KE
1. Constipation is
hard, dry stool

easily palpable and full of feces. Rectal examination


he colon with bowel movements
usually reveals a rectal vault full of stool. Fissure or
tretch the smooth muscle of the
any other rectal processes can make defecation

pletely empty t
and over time s
colon, resultin

g in a functional ileus.
painful, so direct examination is warranted.
cate resulting from organic causes

2. Failure to defe
may be due to d

ecreased peristalsis, decreased


Diagnostic Evaluation
anatomic malformation.

expulsion, and
3. In infancy, con

stipation is commonly associated


If the diagnosis is unclear, a plain abdominal film can
be helpful, because a colon full of stool makes the
re.
diagnosis of constipation. If hypothyroidism is conatal period, the most common
sidered, free T 4, TSH, and T 3RU levels are indicated.
of constipation is voluntary

with anal fissu


4. Beyond the neon
cause (90%-95%)
withholding or

functional constipation.
If hypokalemia or hypocalcemia is a potential cause,
be treated with a diet or a mild
an electrolyte and chemistry panel may be obtained.
When Hirschsprung's disease is suspected, a rectal
for a short time.
mucosal biopsy is required to make the diagnosis. A
lead level assists in diagnosing plumbism as the cause
of constipation. Genetic testing or a sweat test can
confirm suspected cystic fibrosis.
DIS EASE _ _ _
Treatment

5. Most cases can


stool softener

HI RSCH S PRU N G'S

Hirschsprung's disea
se, or congenital aganglionic
Most children with functional constipation can be
megacolon, occurs in
1 in 5000 children and results
treated through dietary changes. The child's fluid
from the failure of
the ganglion cells of the myenintake should be increased, the amount of simple carteric plexuses to mi
grate down the developing colon.
bohydrates Qunk food) decreased, and the amount oAf
s a result, the ab
normally innervated distal colon
fiber and bulk in the diet (leafy vegetables, cerealsremain)
s tonically
contracted and obstructs the flow
increased; the child should begin daily ingestion of
of feces. Hirschspru
ng's disease is three times more
undiluted prune juice or apple juice. Senna or Colace
common among boys an
d accounts for 20% of cases
should be reserved for children in whom dietary
of neonatal intestin
al obstruction. In 75% of cases,
measures are insufficient. The routine use of laxatives
the aganglionic segm
ent is limited to the rectosigor enemas is discouraged.
moid colon, whereas
15% extend beyond the splenic
The constipated child with impaction may be
flexure.
manually disimpacted or may receive a Fleet enema
with a stool softener (Colace), osmotic agent (lactu-

lose, mineral oil, Miralax), or peristalsis inducer


ons
(senna). Anal fissures are treated by softening the
be suspected in any infant who
stools, avoiding the insertion of objects in the anus
um within the first 24 hours of
(thermometer), keeping the rectum as clean as poss repeated rectal simulation to
sible, and applying petroleum jelly locally with each
ts. In the first month of life,
diaper change. Hirschsprung's disease should be
evidence of obstruction with
managed in consultation with a pediatric surgeon or
s vomiting, and abdominal disgastroenterologist or both.
es, particularly those with short
In children with cystic fibrosis and those who have
cm) involvement, the diagnosis
received vincristine, constipation can be so persistent
childhood. In the older child,
and intractable that GoLYTELY cleanouts are
y be seen, as well as intermittent

Clinical Manifestati
The diagnosis should
fails to pass meconi
life and who require
induce bowel movemen
the neonate develops
poor feeding, biliou
tention. In some cas
segment (less than 5
goes undetected into
failure to thrive ma

--------------------------------------- 289
86
rints Pediatrics
bouts of intestinal obstruction, enterocolitis with
_
LEEDING _____
bloody diarrhea, and, occasionally, bowel perforation,
sepsis, and shock.
eding may be acute or chronic,
Stool that is palpable throughout the abdomen
, and may manifest itself as
and an empty rectum on digital examination are
hezia, or melena. There are a
most suggestive of the disease. Abdominal radiograph
s in childhood that cause gasshows distention of the proximal bowel and no gas
ng.
or feces in the rectum. Barium enema may demons to the emesis of fresh or old
strate a transition zone between the narrowed abnorointestinal tract. Fresh blood
mal distal segment and the dilated normal proximal
ltered to a "ground coffee"
bowel. Anal manometry demonstrates failure of the

.GASTROINTESTINAL B

Gastrointestinal ble
gross or microscopic
hematemesis, hematoc
plethora of disorder
trointestinal bleedi
Hematemesis refer
blood from the gastr
becomes chemically a
appearance within 5

minutes of exposure to gastric


internal sphincter to relax with balloon distention of
s the passage of fresh (bright
the rectum. Rectal biopsy revealing no ganglion cells
blood from the rectum. The
and hypertrophied nerve trunks is necessary for the
e colon, although upper gastroindiagnosis.
ing that has a rapid transit time

acid. Hematochezia i
red) or dark maroon
source is usually th
testinal tract bleed
can also result in h

ematochezia. Melena is shiny, jet


Treatment
that are guaiac positive. It results

black, tarry stools


usually from upper g

astrointestinal bleeding; the


Hirschsprung's disease is treated surgically in two
cally altered during passage
stages. The first stage involves the creation of a diverting colostomy with the bowel that contains ganglion
cells, thus permitting decompression of the ganglionis
containing bowel segment. In the second stage, the
aganglionic segment is removed by pulling the gangnosis for gastrointestinal bleedglionic segment through the rectum. This procedure
ided into upper and lower gasis postponed until the infant is 12 months old or
etiologies. Upper gastrointestinal

blood has been chemi


through the gut.
Differential Diagnos
The differential dia
ing is generally div
trointestinal tract
bleeding occurs at a

site proximal to the ligament of


delayed for 3 to 6 months when the disease has been
r gastrointestinal bleeding occurs
diagnosed in an older child. The mortality rate for
this disorder is low in the absence of enterocolitis;
the ligament. Although hematememajor complications include anal stenosis (5%-10%)
ointestinal bleeding can be seen
and incontinence (l%-3%).
ildren from esophagitis or gastritis,

Treitz, whereas lowe


at a site distal to
sis from upper gastr
in critically ill ch
or in children with

portal hypertension from


esophageal varices,
most gastrointestinal bleeding in
KEY POINTS
lower tract and manifests as

children is from the


rectal bleeding. Tab

le 8-3 lists the most common


1. Hirschsprung's disease results from the failure of
eding by age. Minor bleeding prethe ganglion cells of the myenteric plexuses to
ked with blood after stool is passed
migrate down the developing colon. As a result,
o an anal fissure or polyp. Inflam-

causes of rectal ble


sents as stool strea
and is usually due t
matory diseases, suc

h as inflammatory bowel disease


the abnormally innervated distal colon remains
tonically contracted and obstructs the flow of
colitis, result in diarrheal stool
feces.
uses of hematochezia include
2. The diagnosis should be suspected in any infant
isease, Meckel's diverticulum,
who fails to pass meconium within the first 24
drome, Henoch-Schonlein

or infectious entero
mixed with blood. Ca
inflammatory bowel d
hemolytic uremic syn
purpura, and infecti

ous enterocolitis. Table 8-4 lists


hours of life and who requires repeated rectal
simulation to induce bowel movements.
and symptoms of the major
3. In the first month of life, evidence of obstruction

the associated signs


causes of gastrointe

stinal bleeding.
includes poor feeding, bilious vomiting, and
abdominal distention.
Clinical Manifestati
ons
4. Rectal biopsy revealing no ganglion cells and
hypertrophied nerve trunks is necessary for the
diagnosis.
efine the onset and duration of

History
It is important to d
bleeding, color (bri

ght red versus dark maroon versus

--------------------------------------- 290
Ch
apter 8 / Gastroenterology
TABLE 8-3

87

Causes of Rectal Bleeding


Newborn

by Age of Patient
Infant to 2 Yr

2Yrto Preschool

Anal fissure

Infectious diarrhe

Milk colitis

Polyp

Infectious diarrhea

Anal fissure

Intussusception

Meckel's diverticu

Polyp

Intussusception

Meckel's diverticulum

HUS

Preschool to
Adolescence
Vitamin K deficiency
a
IBD
Ingested maternal blood
Infectious diarrhea
Cow/soy milk enterocolitis
Peptic ulcer
Infectious diarrhea
lum
Esophageal varices
Necrotizing enterocolitis
Polyp
Hirschsprung's disease

HSP
Less Frequent Causes
Volvulus
Anal fissure
Duplication cyst
HUS
Vascular malformation
HSP
Stress ulcer
Esophagitis

Esophagitis

PUD

HUS

Esophageal varices

Duplication cyst

IBD

PUD

Esophagitis

Vascular malformation
HSP, Henoch-Schonlein purpura; HUS,hemolytic uremic syndrome; IBD,nflammatory
bowel disease; PUD, pepticulcer disease.
TABLE 8-4
Diagnosis of Gastrointestinal Bleeding
Site

Cause

Signs and Symptoms

Upper Medications Ingestion of ASA, other NSAIDs


Varices Splenomegaly or evidence of liver disease
Esophagitis Dysphagia, vomiting, dyspepsia, irritability in infants
Lower PUD Epigastric pain, meal-related, may be increased at night; family his
tory
Fissure Bright red blood on surface of stool; pain, constipation; f
issure often visible on
anal eversion
Colonic polyps Bright red blood on surface of stool; painless
Milk colitis Blood mixed with stool, diarrhea; patient may have hyp
oproteinemia, edema
Meckel's Painless bleeding, mixed with stool; often a lot of blood
diverticulum
IBD Diarrhea, fever, abdominal pain, poor growth, associated system
ic signs and symptoms
Bacterial colitis Abdominal pain, diarrhea, fever, antibiotics
HSP Joint pain, purpura, abdominal pain, nephritis (casts, RBCs in
urine)
HUS Diarrhea, renal failure, thrombocytopenia, microangiopathic hem
olytic anemia
Intussusception Intermittent abdominal pain, vomiting, pallor,"red
currant jelly" stool, right-sided mass
ASA, acetylsalicylic acid; HSP, Henoch-Schonlein purpura; HUS, hemolytic uremi
c syndrome; IBD, inflammatory bowel disease; NSAID, nonsteroidal
anti-inflammatory drug; PUD, peptic ulcer disease; RBC, red blood cell.
tarry black), rate (brisk versus gradual), and type of
ing, ingestion of ulcerogenic
bleeding (hematochezia, hematemesis, melena,
nonsteroidal anti-inflammatory
blood-streaked stool). Some chronic medical condid a family history of liver disease
tions result in gastrointestinal bleeding, including
ase. For lower gastrointestinal
previous gastrointestinal surgery, liver disease,
ire about diarrhea, infectious conesophagitis, peptic ulcer disease, inflammatory bowel
l, antibiotic or chemotherapeutic
disease, history of milk colitis, history of colonic
n with large or hard stools and
polyps, or coagulopathy.
defecation.
For upper gastrointestinal bleeding, ask specifically
food history is important,

about forceful vomit


drugs (salicylates,
drugs, steroids), an
or peptic ulcer dise
tract bleeding, inqu
tacts, foreign trave
use, and constipatio
difficult or painful
A 24- to 48-hour

--------------------------------------- 291
Blueprints Pediatrics
88
because multiple episodes of "red" vomitus or diartinal bleeding unlikely, although

of upper gastrointes

rhea could result from the ingestion of red fluids or


l ulcers may bleed only distally.
foods (Kool-Aid, beets, red Jello, Tylenol elixir).
itive bright red blood or "coffee
Melena is not always due to blood in the stool; it can
ally clear indicates upper gasoccur in children who have ingested iron, bismuth,
ng that has remitted. Persistent
blackberries, or spinach.
blood indicates active bleeding
Physical Examination
ive intravenous fluid manageThe immediate priority when examining a child with
ient, a thorough history and
gastrointestinal bleeding is to determine if hypowith consideration of the agevolemia exists from an acute bleed. Vital signs should
usually lead to diagnosis. Gastric
be examined for orthostatic changes or for evidence
y in children with minor or nonaof shock (tachycardia, tachypnea, hypotension). The
l bleeding. The precise diagnosis
earliest sign of significant gastrointestinal bleeding is
pper or lower endoscopy.
a raised resting heart rate. A drop in blood pressure
y diarrhea, stool should be sent for
is not seen until at least 40% of the intravascular
ing to look for WBCs and stool
volume is depleted. Dermatologic abnormalities such
ate with bloody stool, necroas petechiae and purpura indicate coagulopathy,
must be considered, and an
whereas cool or clammy skin with pallor is suggesvaluation for sepsis should be
tive of shock or anemia. On abdominal examination,
lowed maternal blood is susevaluate for evidence of masses (a right lower
quadrant mass may be due to Crohn's disease or
of gastrointestinal bleeding, the
intussusception), tenderness (epigastric tenderness
d on the child's stool to
suggests peptic ulcer disease, right lower quadrant
al blood from the blood of the
tenderness may be due to Crohn's disease or infecod is noted and there is a worstious enterocolitis), and hepatosplenomegaly and
ination, a chest radiograph
caput of medusa (evidence of portal hypertension
onary hemorrhage. A

occasionally duodena
Return of guaiac-pos
grounds" that eventu
trointestinal bleedi
return of bright red
and mandates aggress
ment.
In the stable pat
physical examination
related causes will
lavage is unnecessar
cute gastrointestina
is usually made by u
If there is blood
methylene blue stain
culture. In the neon
tizing enterocolitis
abdominal film and e
performed. When swal
pected as the cause
Apt test is performe
differentiate matern
neonate. If oral blo
ening pulmonary exam
may demonstrate pulm
Meckel's scan can be

performed when Meckel's


and risk of varices). Capillary refill (thenar eminence
ected.
in neonates and infants) should be assessed on the
extremity examination. On rectal examination, look
for anal fissure, which is best seen by spreading the
buttocks and everting the anal canal (most fissures
d with severe bleeding or hypoare located at the 6 and 12 o'clock positions),

diverticulum is susp
Treatment
In the unstable chil
volemia, follow the

primary and secondary surveys as


perform a stool guaiac examination, feel for hard
1. Remember, normal hemoglostool, and look for a dilated rectum in children with
es not rule out severe acute
chronic constipation or anal fissure.
ilution takes up to 12 hours in

outlined in Chapter
bin or hematocrit do
bleeding; full hemod
the acutely bleeding

patient. Intravenous normal


Diagnostic Evaluation
actate at 20mL/kg boluses should
Unless the source of bleeding is clearly from the
atient is stable. Type O-negative
nasopharynx, an anal fissure, or hemorrhoids, a come reserved for the unstable
plete blood count with manual differential, coagulaleeding that cannot quickly be
tion studies, and a type and cross should be sent.
l. The most common error in
If the bleeding source is unclear and the patient is
ild with severe gastrointestiunstable, the clinician should use gastric lavage to
equate volume replacement.
determine whether the bleeding is from the upper or
e finding; fluid resuscitation
lower gastrointestinal tract. A well-lubricated nasoy the level of tachycardia.
gastric or orogastric tube of the largest bore possible
without heavy bleeding or signs
should be placed, and the stomach lavaged with
d be evaluated and treated
room-temperature normal saline until lavage fluid is
ticular diagnosis.
clear. Iced saline may cause hypothermia and should
rates a useful algorithm for the
be avoided. Esophageal varices are not a contraindiement of gastrointestinal
cation to the placement of a nasogastric or orogastric
on causes of gastrointestinal
tube. Return of clear lavage fluid makes the diagnosis
rticulum, ulcerative colitis,

saline or Ringer's l
be given until the p
whole blood should b
patient with acute b
brought under contro
management of the ch
nal bleeding is inad
Hypotension is a lat
should be governed b
The stable child
of hypovolemia shoul
according to the par
Figure 8-1 illust
evaluation and manag
bleeding. Three comm
bleeding Meckel's dive

--------------------------------------- 292
C
hapter 8 / Gastroenterology

89
ABCs + Hemodynamic stabilization
Fluid resuscitation
History and physical (H & P)

Upper tract bleeding

Lowe

r tract bleeding
Gastric lavage
Gastroccult-positive
Gastroccult-negative

emesis or
nasogastric aspirate
nasogastric aspirate
I
siooi guaiac :
Room temperature saline lavage
Hemoccult-positive Hemoccult-negative
stool stool
Bismuth
ron
i
Red foods and
Clear
Not clear
Fecal leukocytes f ooc| coloring
Antacids
Consider IV infusion
of H2 blockers and
Proton pump blookers vasoactive agents
Positive
Upper endoscopy Upper endoscopy

Negative

Consider based on H & P


Stool culture/ova and parasites
Upper Glseries

Abdominal film
Upper Gl with smal

Discrete sourc

bowel follow-through
Barium enema

Diffusesource
Colonoscopy
Positive

Negative
Meckel's scan
Tagged RBC scan

Thermocoagulation H

2 blockers

Treat

Consider
Sclerotherapy Proton pump blockers
accordingly
colonoscopy
Antacids
Figure 8-1
eeding.

Algorithm for evaluation and management of gastrointestinal tract bl


and Crohn's disease a

re discussed in the following


KEY POINTS
sections.

1. Upper gastrointestinal bleeding occurs at a site


Meckel's Diverticul
um
proximal to the ligament of Treitz, whereas lower
gastrointestinal bleeding occurs distal to the
um, the vestigial remnant of
ligament.
ic duct, is the most common
2. Most gastrointestinal bleeding in children is from
rointestinal tract. It is present in
the lower Gl tract and manifests as rectal bleeding.
ulation and is located within
3. The earliest sign of significant gastrointestinal
cal valve in the small intestine.
bleeding is a raised resting heart rate. A drop in
of bleeding from the diverticublood pressure is not seen until at least 40% of
f age. Heterotopic tissue, usually
the intravascular volume is depleted.
s more common in symptomatic

Meckel's diverticul
the omphalomesenter
anomaly of the gast
2% to 3% of the pop
100cm of the ileoce
The peak incidence
lum is at 2 years o
gastric, is 10 time
cases because of ac

id secretion and ulceration.

--------------------------------------- 293
Chapter 2 / Poisoning, Bu
rns, and Injury Prevention
TABLE 2-1
Signs, Symptoms, and Treatment of Specific Pediatric Poisonings
Substance Clinical Manifestations
ntidote/Treatment
Acetaminophen
Nausea/vomiting, anorexia, pallor,
A/-acetylcysteine
diaphoresis; may progress over days
gastric emptying if <2hr
to jaundice, abdominal pain, liver
since ingestion; activated
failure
charcoal if <4hr since ingestion.

A
A:
T:

Draw blood level at 4hr and use


available nomogram to assess
risk of hepatotoxicity. If toxic,
start oral A/-acetylcysteine and
continue for 72 hr
Anticholinergics
: physostigmine for atropine
(atropine, tricyclic
and antihistamines
antidepressants,

Fever, mydriasis, flushing, dry skin,

tachycardia, hypertension, cardiac


arrhythmias, delirium, psychosis,

: NaC02, MgSO 4 for tricyclic


antihistamines,
antidepressants
phenothiazides)
Aspirin
: gastric emptying if <6hr

convulsions, coma

Fever, hyperpnea, vomiting, tinnitus,

lethargy, coma
since ingestion, activated
charcoal, cathartics; fluid and
electrolyte management
Cholinergics
Nausea/vomiting, sweating, meiosis,
: pralidoxime chloride
(organophosphates
salivation, lacrimation, bronchorrhea,
: gastric lavage, activated
and other pesticides)
urination, defecation, weakness,
charcoal; prophylactic atropine
muscle fasciculations, paralysis,

A
T

confusion, coma
Hydrocarbons
: Prevent aspiration

Fever, nausea/vomiting,

gastrointestinal bleeding,
(Aspiration results in chemical
confusion, coma
pneumonitis and significant lung
tissue damage!) No gastric
emptying techniques are
necessary.
Iron
: deferoxamine chelation

Vomiting, diarrhea,
gastrointestinal bleeding,

A
T

: emesis induction, gastric


cyanosis, seizures, coma,
lavage, cathartics
metabolic acidosis
Opiates
: naloxone

Pinpoint pupils, bradypnea,

hypotension, hypothermia,

: evaluate and secure airway


stupor, coma
as needed; gastrointestinal
decontamination if appropriate;
naloxone
Sedatives/hypnotics
: flumazenil for

Nystagmus, meiosis or mydriasis,

hypothermia, hypotension,
benzodiazepines
bradypnea, confusion, ataxia, coma
: evaluate and secure airway if

needed; maintain hemodynamic


stability; activated charcoal with
cathartic; supportive care
Sympathomimetics
Fever, mydriasis, tachycardia,
: gastric emptying, activated
(amphetamines,
hypertension, sweating, delirium,
charcoal, cathartics; sedatives
cocaine,
psychosis, tremor, myoclonus,
for severe agitation; control
theophylline)
convulsions
of hypertension; ample fluids

--------------------------------------- 294
90
Blueprints Pediatrics
Clinical Manifestations
occurs equally in males and
The most common presentation of Meckel's diverric patients are adolescents, but
ticulum is painless rectal bleeding. Eighty-five
een reported in infancy.
percent of patients with Meckel's diverticulum have
melena, 10% will develop intestinal obstruction from
ons
intussusception or volvulus, and 5% suffer from
n, recurrent fever, weight loss,
painful diverticulitis mimicking appendicitis. The
mon manifestations in Crohn's
diagnosis is made by performing a Meckel's scan. The
arrhea is common, it is not unitechnetium-99 pertechnetate scan, preceded by presease. Rectal bleeding is noted in
pentagastrin stimulation or a histamine H
2-receptor
Crohn's disease. Abdominal pain
antagonist (cimetidine), identifies the ectopic
ere in Crohn's disease than in

whites and Jews and


females. Most pediat
both diseases have b
Clinical Manifestati
Crampy abdominal pai
and diarrhea are com
disease. Although di
versal in Crohn's di
only 35% of cases of
tends to be more sev
ulcerative colitis,

may be diffuse, and is frequently


acid-secreting cells creating the hemorrhage in the
ower quadrant. Perianal disease
diverticulum.
s, fissures, fistulas, or abscesses.
Treatment
t gain, and delayed growth

worse in the right l


may produce skin tag
Anorexia, poor weigh
occur in 40% of pati

ents.
Definitive treatment is surgical resection.
Most children wit
h ulcerative colitis exhibit
bloody mutinous diar
rheal stool (100%), abdominal
KEY POINTS
smus (75%). Ninety percent of
1. Meckel's diverticulum, the vestigial remnant of the
d to moderate disease. Mild
omphalomesenteric duct, is the most common

pain (95%), and tene


patients exhibit mil
disease is defined a

s less than six stools per day, no


anomaly of the gastrointestinal tract.
d no hypoalbuminemia, whereas
2. The most common presentation of Meckel's divergreater than six stools per day,

fever, no anemia, an
moderate disease has
fever, anemia, and h

ypoalbuminemia. Severe disease


ticulum is painless rectal bleeding.
h high fever, abdominal ten-

may be fulminant wit


derness, distention,

tachycardia, leukocytosis, hemorrhage, severe anemia


, and more than eight stools per
Inflammatory Bowel Disease
and intestinal perforation are

day. Toxic megacolon


rare complications.

After 10 years of disease, there is


Inflammatory bowel disease (IBD) is a generic term
1% to 2% per year for the develfor Crohn's disease and ulcerative colitis, which are
Table 8-5 compares Crohn's
chronic inflammatory disorders of the intestines.
ve colitis.
Ulcerative colitis produces diffuse superficial
equelae, similar in both diseases,
colonic ulceration and crypt abscesses. It involves thmae
ompany gastrointestinal symprectum in 95% of patients, with or without contiguyarticular arthritis, ankylosing
ous extension higher in the colon. Ulcerative colitis
sclerosing cholangitis, chronic
does not affect the small intestine.
croiliitis, pyoderma gangrenosum,
The pathology of Crohn's disease involves transphrolithiasis, aphthous stommural inflammation in a discontinuous pattern,
, recurrent iritis, and uveitis.
which results in skip lesions. Crohn's disease may
ve colitis involves the rectum in
involve any part of the gastrointestinal tract (mouth
octosigmoidoscopy and biopsy are
to anus). The process is ileocolic in 60% of cases,
tion of the mucosa in ulcerative
involves the small intestine in 30% of cases, and
use superficial ulceration and easy
impairs the colon in only 10% of cases. Fibrosis is
disease, direct visualization and
transmural, and strictures are common. Granulomas
cal area are not always possible.
are observed in up to 30% of patients. Internal or
mination with a double airexternal fistula formation occurs in up to 40% of
ma demonstrates diffuse colonic
patients.
olyp formation in ulcerative
Although the exact etiology of these disorders is
nation should be delayed in
not known, a combination of genetic, environmental,
ely active disease to avoid precippsychological, infectious, and immunologic mecha-

a cumulative risk of
opment of carcinoma.
disease and ulcerati
Extraintestinal s
y precede or acc
toms and include pol
spondylitis, primary
active hepatitis, sa
erythema nodosum, ne
atitis, episcleritis
Because ulcerati
95% of patients, pr
indicated. Visualiza
colitis reveals diff
bleeding. In Crohn's
biopsy of the ileoce
Radiographic exa
contrast barium ene
lesions and pseudop
colitis. This exami
patients with sever
itating toxic megac

olon. Crohn's disease often reveals


nisms have been implicated. IBD is most common in
c involvement with skip lesions,

ileal and/or coloni

--------------------------------------- 295
Ch
apter 8 / Gastroenterology
91
TABLE 8-5
5-Aminosalicylic compounds have

quickly increasing.
long been a mainstay

of anti-inflammatory treatment.
Comparison of Crohn's Disease and Ulcerative
ole as anti-inflammatory agents
Colitis
Aggressive nutritional support
Crohn's Ulcerative
ing) is important for growth,
Feature Disease Colitis
ve anti-inflammatory effects and

Antibiotics have a r
in Crohn's disease.
(including tube feed
but also seems to ha
symptom control in C

rohn's disease. Corticosteroids


Malaise, fever,
Common
mmatory and immunosuppressive
weight loss
a mainstay of management. Pure
Rectal bleeding
Sometimes
Abdominal mass
Common
nclude 6-mercaptopurine, azaAbdominal pain
Common
in A, and methotrexate.
Perianal disease
Common
, therapy is chosen in an effort to
Heal involvement
Common
tom control with minimum

Common

have both anti-infla


effects, and remain

Usual
Rare

immunosuppressives i

Common

thioprine, cyclospor

Rare
None (backwash

ileitis)
esult, immunosuppressive agents
Strictures
Common
Unusual
e severe illness, but may be
Fistula
Common
Unusual
e long-term steroid use. New
Skip lesions
Common
Not present
being developed and evaluated
Transmural
Usual
Not present
ry specific components of the
involvement
Crypt abscesses
Unusual
Usual
. Infliximab is an example of a
Granulomas
Common
Not present
ed antibody directed against
Risk of cancer
Slightly
Greatly
r alpha, and it shows promise in
increased
increased
ficant Crohn's disease.
rectal sparing, segmental narrowing of the ileum
(string sign], and longitudinal ulcers.
Y POINTS
Anemia is common and usually is associated with

As a general rule
achieve maximum symp
side effects. As a r
are reserved for mor
necessary to decreas
biologic agents are
that are aimed at ve
inflammatory cascade
genetically engineer
tumor necrosis facto
the control of signi
KE
1. Ulcerative coli

tis produces diffuse superficial


iron deficiency. Megaloblastic anemia secondary to
ion and crypt abscesses. It involves
folate and vitamin B12 deficiency may also be present.
5% of patients, with or without
An elevation of the erythrocyte sedimentation rate is
nsion higher in the colon. Ulceraseen in about 50% of cases of ulcerative colitis and
es not affect the small intestine.
in 80% of Crohn's disease cases. Hypoalbuminemia,
amination with a double aircaused by poor protein intake, is common in indienema demonstrates diffuse
viduals with severe symptoms. Serum aminotransand pseudopolyp formation in
ferase levels are increased if hepatic inflammation is
tis.
a complicating feature. Stool examination reveals
tis places the child at high risk for
blood and fecal leukocytes with a negative stool
of colon cancer.
culture.
f Crohn's disease involves trans-

colonic ulcerat
the rectum in 9
contiguous exte
tive colitis do
2. Radiographic ex
contrast barium
colonic lesions
ulcerative coli
3. Ulcerative coli
the development
4. The pathology o
mural inflammat

ion in a discontinuous pattern,


Differential Diagnosis
n skip lesions. Crohn's disease may
The differential diagnosis of IBD includes chronic
t of the gastrointestinal tract
bacterial or parasitic causes of diarrhea, appendicitis,
.
hemolytic uremic syndrome, Henoch-Schonlein
amination with a double airpurpura, and radiation enterocolitis. Enteric infecenema in Crohn's disease demontions include C. difficile, Campylobacter jejuni,
nd/or colonic involvement with skip
Yersinia enterocolitica, amebiasis, and giardiasis.
sparing, segmental narrowing of the

which results i
involve any par
(mouth to anus)
5. Radiographic ex
contrast barium
strates ileal a
lesions, rectal
ileum (string s

ign), and longitudinal ulcers.


Treatment
lammatory bowel disease is aimed
Treatment of inflammatory bowel disease is aimed
ximum symptom control with
at control of inflammation and suppression of the
fects.
immune system. The variety of agents available is

6. Therapy for inf


at achieving ma
minimum side ef

--------------------------------------- 296
92

Blueprints Pediatrics
Because anorexia and increased nutrient losses in
gery is indicated in ulcerative
the stool are common in children with IBD, adequate
s fulminant colitis with severe
calories and protein are essential. Oral supplementsbloo,
megacolon, intractable disease

Crohn's disease. Sur


colitis when there i
d loss or toxic

nasogastric tube feedings, and, in some severe cases,


roid requirement, steroid toxiccentral venous hyperalimentation are necessary.
or colonic dysplasia. Because
Vitamin and mineral supplementation, especially
s restricted to the colon, colectomy
iron, may be required.
is performed in Crohn's disease
Patients with ulcerative colitis for more than
hage, obstruction, perforation,
10 years need annual colonoscopy and rectal
tion, or ureteral obstruction. In
biopsy because of the high risk of colon cancegeneralr
ve management is warranted
development.
he diseased bowel is not curaSurgery is eventually needed in 25% of patients
ase. Recurrence rates of up to
with ulcerative colitis and 70% of children with
ed after segmental resection.

with a high-dose ste


ity, growth failure,
ulcerative colitis i
is curative. Surgery
when there is hemorr
severe fistula forma
, conservati
because removal of t
tive in Crohn's dise
50% have been report

--------------------------------------- 297
Genetic
Disorders
Structural birth defects are categorized as minor or
trauterine forces such as uterine
major. Minor birth defects such as skin tags, inner
dramnios may cause fetal conepicanthal folds, and rudimentary polydactyly are of

defect. Abnormal in
fibroids or oligohy
straint, resulting

in club foot or hip dysplasia. Table


little physiologic significance. Approximately 15% of
common teratogenic drugs and
newborn infants have at least one minor anomalythei;
0.5% of infants have three or more minor anomalies.
In contrast, major birth defects such as cleft palate,
myelomeningocele, and congenital heart disease have
EY POINTS
an adverse effect on the infant. Major birth defects

9-2 lists the most


r effects.
K
1. Environmental

factors cause 10% of birth defects.


occur in 2% to 3% of all newborns. The probability
nts, high-dose radiation, maternal
of having a major birth defect increases as the
number of minor anomalies present increases (Table
rders, mechanical forces, and drugs
9-1). Birth defects can be caused by environmental
as teratogens.
or genetic factors. Sporadic disorders are birth defects
xposure before 12 weeks'gesta-

2. Infectious age
metabolic diso
can all serve
3. A teratogenic e
tion affects or

ganogenesis and tissue morphocaused by unknown factors.


as an exposure thereafter retards

genesis, where
fetal growth an

d central nervous system


JNVmONMENTAL FACTORS

development.

Environmental factors are known to cause at least


10% of all birth defects. Teratogens are environmental agents that cause congenital developmental
GENETIC FACTORS
anomalies by interfering with embryonic or fetal
organogenesis or growth. Exposure to a teratogeGenetin
c disorders
can be classified as disorders of
before implantation (days 7 to 10 postconceptionsingl)
e genes, chrom
osomes, parental imprinting,
can either have no effect or can result in loss of thane
d molecular cytog
enics. Advances in molecular
embryo. To disrupt organogenesis, a teratogen musgenetict
s have blurr
ed the distinction among these
be present before 12 weeks' gestation. Any teratocategories.
genic exposure after 12 weeks' gestation predominantly affects growth and central nervous system
development.
SI NGLE-GENE .DISORD
ERS
Teratogens include intrauterine infections
(Chapter 13), high-dose radiation, maternal metaNormal human cells h
ave 46 chromosomes (22 pairs
bolic disorders (Chapter 13), mechanical forces, and
of autosomes and 1 p
air of sex chromosomes). Chrodrugs. The most common maternal metabolic disormosomes contain gene
s, which occur in pairs at a
der that has teratogenic potential is diabetes mellisingle locus or site
on specific chromosomes. These
tus; 10% of infants of diabetic mothers have a birth
paired genes, called
alleles, determine the genotype

--------------------------------------- 298
94 Blueprints Pediatrics
of an individual at that locus. If the genes at a spet disorders are rare and are
cific locus are identical, the individual is homozythal. A mutant gene usually is
gous; if they are different, the individual is
arent with the same condition.
heterozygous. More than 3000 different single-gene
ected parents' offspring is 50% for
disorders have been described and are classified by
times an individual is the first
their mode of inheritance (autosomal dominant,
o display a trait due to spontaautosomal recessive, or X-linked).
a spontaneous mutation has

of autosomal dominan
usually severe or le
inherited from one p
The risk for the aff
each pregnancy. Some
person in a family t
neous mutation. When
occurred in a fetus,

the risk of recurrence in a subsequent pregnancy is t


he same as the chance of the
Autosomal Dominant Disorders
occurring de novo. Autosomal
Autosomal dominant disorders are expressed after
cause conditions that manifest
alteration of only one gene in the pair (usually coding
ing degrees of severity among
for a structural protein). Homozygous disease states

spontaneous mutation
dominant genes often
themselves with vary
affected individuals

, a phenomenon known as variable expressivity or


variable penetrance. Table 9-3
lists some of the mo
st important autosomal dominant diseases. Other
chapters discuss some of these
TABLE 9-1
Incidence of Major Anomalies in the Presence of
Minor Anomalies

diseases in detail.
Autosomal Recessive

Disorders
Number of Minor Incidence of Major Anomalies
Anomalies (%)
disorders are expressed after

Autosomal recessive
alteration of both t

he maternal and paternal genes of


1
coding for an enzyme}. Because
3
nzyme activity is adequate under
20
a person with only one mutant

a gene pair (usually


half of the normal e
most circumstances,

gene is not affected


, whereas individuals who are
TABLE 9-2
Common Teratogenic Drugs
Drug Results
Warfarin (Coumadin)
Hypoplastic nasal bridge, chondrodysplasia punct
ata
Ethanol
Fetal alcohol syndrome, microcephaly, CHD (septa
l defects, patent ductus arteriosus)
Isotretinoin (Accutane)
Facial and ear anomalies, congenital heart disea
se
Lithium
CHD (Ebstein's anomaly, atrial septal defect)
Penicillamine
Cutis laxa syndrome
Phenytoin (Dilantin)
Hypoplastic nails, intrauterine growth retardati
on, cleft lip and palate
Radioactive iodine
Congenital goiter, hypothyroidism
Diethylstilbestrol
Vaginal adenocarcinoma during adolescence
Streptomycin
Deafness
Testosterone-like drugs
Virilization of female
Tetracycline
Dental enamel hypoplasia, altered bone growth
Thalidomide
Phocomelia, CHD (tetralogy of Fallot, septal def
ects)
Trimethadione
Typical facies, CHD (tetralogy of Fallot, transp
osition of the great arteries, hypoplastic
left heart)
Valproate
Spina bifida
CHD, congenital heart disease.

--------------------------------------- 299
Cha
95
pter 9 / Genetic Disorders
TABLE 9-3
Examples of Autosomal Dominant Diseases
Autosomal Dominant Disease Frequency Chromosome

Comments

Achondroplasia
1:25,000
4p 80% new mutations; proximal
limb shortening
Adult polycystic kidney disease 1:1200
16p Renal cysts, intracranial a
neurysm
Hereditary angioedema
1:10,000
11q Deficiency of C1 esterase i
nhibitor; episodic edema
Hereditary spherocytosis
1:5000
8p, 14q See Chapter 10; some va
riants autosomal recessive
Huntington's disease
1:2500
4p Dementia, chorea
Marfan's syndrome
1:20,000
15q Aortic root dilatation, ta
ll stature
Myotonic dystrophy
1:25,000
19q Muscular weakness, cardiac
arrhythmias
Neurofibromatosis
1:3000
17q 50% new mutations; cafe au
lait spots
Protein C deficiency
1:15,000
2p Hypercoagulable state
Retinoblastoma
1 :15,000
13q See Chapter 18
Tuberous sclerosis
1 :30,000
9q, 16p "Ash-leaf" spots; seizu
res
von Willebrand's disease
1:100
12p See Chapter 10
p, short arm of chromosome; q, long arm of chromosome.
Examples of Autosomal Recessive Diseases
Autosomal Recessive Disease Frequency
Chromosome Comment
s
Congenital adrenal
1:5000-1:15,000; 6p
P
renatal diagnosis possible
hyperplasia
1:700 in Yupik Eskimos
Cystic fibrosis
1:2000 (Caucasians) 7q
S
ee Chapter 20
Galactosemia
1:60,000 9p
C
arbohydrate metabolism disorder
Gaucher's disease
1:2500 (Ashkenazi Jews) 1 q
L
ysosomal storage disorder
Infantile polycystic kidney
1:14,000 6p
R
enal and hepatic cysts, hypertension
disease
Phenylketonuria
1:14,000 12q
A
mino acid metabolism disorder
Sickle cell disease
1:625 (African Americans) 11p
S
ee Chapter 10
Tay-Sachs disease
1:3000 (Ashkenazi Jews) 15q
L
ysosomal storage disorder
Wilson's disease
1:200,000 13q
D
efective copper excretion
p, short arm of chromosome; q, Jong arm of chromosome.
homozygous for a defective gene have the disorder.
X-Linked Disorders
Both parents of a child with an autosomal recessive
disorder are usually heterozygous for that gene, and
X-linked disorders,
which are usually recessive, occur
each child of such a couple has a 25% risk of inherwhen a male inherits
a mutant gene on the X chroiting the disorder. Table 9-4 lists the more common
mosome from his moth
er. The affected male, termed
autosomal recessive disorders.
hemizygous for the g
ene, has only a single X chroMost inborn errors of metabolism, with the excepmosome and, therefor
e, a single set of X-linked
tion of ornithine transcarbamylase (OTC) deficiency,
genes. The mother of
the affected individual is
are autosomal recessive disorders. Inborn errors of
heterozygous for tha

t gene, because she has both a


metabolism are discussed later in this chapter.
and a mutant one. She may

normal X chromosome

--------------------------------------- 300
96

rints Pediatrics

be asymptomatic or demonstrate mild symptoms of


DISORDERS
the disorder due to lyonization, in which only one
X chromosome is transcriptionally active in each
Chromosomal disorder
s are responsible for pregcell. Recurrence risk for X-linked disorders differs
nancy loss, congenit
al malformation, and mental
depending on which parent has the abnormal generetardation.
. Althou
gh more than 50% of firstAn affected father will pass the defective X chrotrimester pregnancy
losses are due to chromosomal
mosome on to his daughters, creating carriers for the
% of newborn infants have chrodisorder; his sons will not be affected. A mother with
s. Most chromosomal defects
an abnormal X chromosome is a carrier, and there is
gametogenesis, so that an infant
a 50% chance she will pass the abnormal chromoh a chromosomal abnormality
some to her progeny. Daughters who receive the
mily history. Chromosomal
abnormal X chromosome will be carriers for the

imbalances, only 0.6


mosomal abnormalitie
arise de novo during
can be conceived wit
without any prior fa

abnormalities can al
so be passed from parent to offdisease, and sons will have the disease. Table 9-5 lists
s, there is often a family history
the most common X-linked disorders.
ous abortions or a higher than

spring. In such case


of multiple spontane
chance frequency of

children with chromosomal


problems. Disorders
of chromosome number may
involve autosomes or
sex chromosomes. Birth defects
caused by autosomal
abnormalities are generally
KEY POINTS
se caused by sex chromosome

more severe than tho


abnormalities. Numer

ic defects of the autosomes


1. Single-gene defects are classified by their mode
hromosomes 21, 18, and 13.
of inheritance into autosomal dominant, autosomosome numerical abnor-

include trisomy of c
Examples of sex chro

mal recessive, and X-linked disorders.


s syndrome and Klinefelter's
2. In autosomal dominant disorders, the phenomenon of incomplete penetrance results in variable
btaining chromosomal studies
expression of the defective gene.
of a suspected chromosomal
3. Genes defective in autosomal dominant disorders
rgan system malformations, sigtypically code for structural proteins, whereas
al delay or mental retardation
those in autosomal recessive disorders code for
explanation, short stature or
enzymes.
narche in girls, infertility or a
4. Most inborn errors of metabolism, with the noted
spontaneous abortions, ambiguexception of ornithine transcarbamylase defivanced maternal age. Fetal karyociency, are autosomal recessive disorders.
lished through amniocentesis

malities are Turner'


syndrome.
Indications for o
include confirmation
syndrome, multiple o
nificant development
without an alternate
extremely delayed me
history of multiple
ous genitalia, or ad
typing may be accomp
or chorionic villus

sampling.
TABLE 9-5
X-Linked Diseases
X-Linked Disease

Frequency

Comments

Bruton agammaglobulinemia 1:100,000


unoglobulins; recurrent infections
Chronic granulomatous disease 1:1,000,000
ing by phagocytes; recurrent infections
Color blindness 1:100,000

Absence of imm

Duchenne muscular dystrophy 1:3600


e weakness; Gower's sign
Glucose-6-phosphate dehydrogenase 1:10 (African
d hemolytic anemia
deficiency Americans)

Proximal muscl

Hemophilias A and B 1:10,000


Lesch-Nyhan syndrome 1:100,000
ism disorder; self-mutilation
Ornithine transcarbamylase deficiency
der; hyperammonemia

Defective kill

Oxidant-induce

See Chapter 10
Purine metabol
Urea cycle disor

--------------------------------------- 301
Ch
apter 9 / Genetic Disorders 97
Autosomal Trisomies
) with an incurved fifth finger

hands (brachydactyly
(clinodactyly) and h

ypoplastic middle phalanx, and


Trisomy21

an excessive gap bet

ween the first and second toes


Trisomy 21, or Down syndrome, is the most common
er features include short stature,
autosomal chromosomal abnormality in humans,
a, cardiac defects (endocardial
with an incidence of 1 per 700 live births. The risk
septal defects are seen in 50% of
of Down syndrome increases with advancing maternal age; it is 1 in 365 for mothers 35 years of age and
inal anomalies (duodenal atresia

("sandal sign"). Oth


generalized hypotoni
cushion defects and
cases), gastrointest
and Hirschsprung's d

isease), hypothyroidism, and


1 in 50 for those 45 or older. Of children with Down
IQ range 35-65). Leukemia is
syndrome, 95% have three number 21 chromosomes
in children with trisomy
(47 total chromosomes), which results from chroal population. During the third
mosomal nondisjunction during maternal meiosis.
an Alzheimer-like dementia
Four percent have translocation of a third number 21
ved medical, educational, and
chromosome to another chromosome (46 total
t, life expectancy for patients
chromosomes). One-third of translocation cases are
ow extends well into
familial, meaning that one of the parents has a balanced translocation involving one number 21 chromosome and another chromosome. One percent of
children with Down syndrome have chromosome
1 per 8000 live births. Eighty
mosaicism, with some cells having two number 21
ult from meiotic nondisj unction
chromosomes (46 total chromosomes) and some
ith advanced maternal age. The
cells having three number 21 chromosomes (47 total
partial (involving only a
chromosomes). The mosaicism results from a mitotic
osome) or mosaic, caused by
division error that occurred during embryonic
ion in the zygote. Chromosome
development.
cause of trisomy 18 is extremely
Common dysmorphic facial features include
ce should prompt karyotyping of
brachycephaly (flat occiput), flat facial profile,
de an inherited defect. Clinical
upslanted palpebral fissures, small ears, flat nasal
isomy 18 are shown in Table 9-6.
bridge with epicanthal folds, and a small mouth with
tients with trisomy 18 is
a protruding tongue. Anomalies of the hand include
die before reaching 1 month of
single palmar creases (simian creases), short, broad
1 year of age.
TABLE 9-6
Key Features of Trisomy 13 and Trisomy 18
Trisomy 13
omy 18

mental retardation (
20 times more common
21 than in the gener
and fourth decades,
develops. With impro
vocational managemen
with Down syndrome n
adulthood.
Trisomy 18
Trisomy 18 occurs in
percent of cases res
and are associated w
remaining 20% may be
portion of the chrom
mitotic nondisj unct
translocation as the
rare, and its presen
the parents to exclu
manifestations of tr
The prognosis for pa
extremely poor: 30%
age, and 90% die by

Tris

Head and neck


inent occiput

Microcephaly with sloping forehead

Prom

Cutis aplasia of scalp


Narr
ow bifrontal diameter of forehead
Microphthalmia
Lowset, malformed ears
Cleft lip and palate
Micr
ognathia
Chest and abdomen
Congenital heart disease (VSD, ASD, PDA)
Cong
enital heart disease (VSD, ASD, PDA)
Omphalocele
Shor
t sternum
Extremities
Clenched hands with overlapping fingers
Clen
ched hands with overlapping fingers
Polydactyly
Rock
er-bottom feet
Polycystic kidney or other renal defects
Hors
eshoe kidney
Other
Cryptorchidism
Lack
of subcutaneous fat
Agenesis of corpus callosum
ASD, atrial septal defect; PDA, patent ductus arteriosus; VSD, ventricular sep
tal defect

--------------------------------------- 302
Blueprints Pediatrics
98
Trisomy 13
ons
Trisomy 13 occurs in 1 per 10,000 live births but
include lymphedema of the
constitutes 1% of all spontaneous abortions. Apield-shaped chest, widely spaced
proximately 75% of surviving cases are the result of
a webbed neck, low hairline,
meiotic nondisjunction and are associated with
eased carrying angle), short
advanced maternal age. The risk with advanced
e pigmented nevi. Additional
maternal age is much less that for trisomy 21. Twenty
e gonadal dysgenesis, gonadopercent of children with trisomy 13 have 46 chroalies, congenital heart disease,
mosomes with a translocation of a third chromosome
is, and learning disabilities.
13 to another chromosome. One-fourth of translopresent in 100% of patients,
cation cases are familial, meaning that one of the
rimary amenorrhea and lack
parents has a balanced translocation involving one
ent due to loss of ovarian horchromosome 13 and another chromosome. The
e appropriately infantile at
remaining 5% of children with trisomy 13 have
ring childhood and become
mosaicism; some cells have 46 chromosomes with
puberty. In mosaics with a Y chro-

Clinical Manifestati
Dysmorphic features
hands and feet, a sh
hypoplastic nipples,
cubitus valgus (incr
stature, and multipl
abnormalities includ
blastoma, renal anom
autoimmune thyroidit
Gonadal dysgenesis,
is associated with p
of pubertal developm
mones. The gonads ar
birth but regress du
"streak" ovaries by
mosome in one of the

ir cell lines, gonadoblastoma is


two number 13 chromosomes, and some cells have
rophylactic gonadectomy is
47 chromosomes with three number 13 chromoatients. Renal anomalies, usually
somes. The mosaicism results from a mitotic division
g system or horseshoe kidney,
error that occurred during embryonic development.
e with Turner's syndrome. ConClinical manifestations of trisomy 13 are shown in
e occurs in 20% of patients, and
Table 9-6. Prognosis for patients with trisomy 13 is
de coarctation of the aorta,
extremely poor: 50% die before reaching 1 month of
age, and 90% die by 1 year of age.
bicuspid aortic valve. As a conse-

common. Therefore, p
necessary in these p
duplicated collectin
occur in 40% of thos
genital heart diseas
common defects inclu
aortic stenosis, and
quence of having onl

y one functional X chromosome, females with T


urner's syndrome display the
Sex Chromosome Abnormalities
x-linked disorders as males. The

same frequency of se
diagnosis is made by

karyotype. Because of their


Sex chromosome anomalies involve abnormalities in
s suspected of having Turner's
the number or structure of the X or Y chromosomes
X karyotype in the peripheral
or both.
opsy will be necessary to make

mosaicism, some girl


syndrome have a 46,X
blood, and a skin bi
the diagnosis.
Short stature has

been successfully treated using


Turner's Syndrome
. Secondary sexual characTurner's syndrome occurs in 1 per 5000 live births.
ter estrogen and progesterone
Approximately 98% of fetuses with Turner's synentioned earlier, gonadectomy is
drome expire in utero; only 2% are born. Advancing
with a Y chromosome cell line.
maternal age does not increase the risk, because
ion of a few mosaics, women
Turner's syndrome is caused by defective embryonic
me cannot become pregnant.
cell division that occurs after fertilization. Therefore,
the recurrence risk for parents who have a child with
me
Turner's syndrome is no higher than that of the
general population.
me, caused by an extra X chroSeveral genotypes can cause the Turner's pheno1000 newborn males, 20% of
type. In 60% of cases, the karyotype is 45,XO, in
and 1 in 250 men over 6 feet
which the female lacks an X chromosome. Another
is XXY in 80% of cases and

human growth hormone


teristics develop af
administration. As m
indicated in mosaics
With the rare except
with Turner's syndro
Klinefelter's Syndro
Klinefelter's syndro
mosome, affects 1 in
aspermic adult men,
tall. The karyotype
mosaic (XY/XXY) in 2

0%. Recurrence risk is the


15% of individuals are mosaics with a genotype

same as the initial

risk in the general population.


of 45,XO/46,XX;
45,XO/46,XX/47,XXX; or
45,XO/46,XY Mosaic individuals may have fewer
Clinical Manifestati
ons
physical stigmata of Turner's syndrome. In the
The physical stigmat
a of Klinefelter's syndrome are
remaining 25% of cases, there are two X chromonot obvious until pu
berty, at which time males are
somes but the short (p) arm of one of the X chroincompletely masc
ulinized. They have a female body
mosomes is missing.
habitus with decreas
ed body hair, gynecomastia, and

--------------------------------------- 303
Ch
apter 9 / Genetic Disorders
99
small phallus and testes. Infertility results from
hat leads to severe central obesity.
hypospermia or aspermia. Affected males are usually
onstantly unless food is locked
taller than average relative to their families, and their
d obstructive sleep apnea and
arm span can be greater than their height. There is
mplications (pickwickian synan increased incidence of learning difficulties, but the
There is mild mental retardaaverage IQ is 98. Gonadotropin levels are usually elestic impulse control problems.
vated because of inadequate testosterone levels.
atient, strict dietary control is
Testosterone therapy during adolescence may
ult to enforce. Although those
improve secondary sexual characteristics and prevent
rmal life spans, complications of
gynecomastia.
ructive sleep apnea and diabetes

trollable appetite t
These children eat c
away. Obesity-relate
cardiorespiratory co
drome) may develop.
tion with characteri
For the average p
attempted but diffic
affected can live no
obesity such as obst
mellitus often lead

to earlier death.
PARENTAL IMPRINTING

Angelman's Syndrome

DISORDERS
Approximately 60% of
patients with Angelman's
Imprinting refers to different phenotypes resulting
odeletion on the maternal
from the same genotype, depending on whether a
ion of 15qll-13) and a
mutation-marked chromosome is inherited from the
mosome 15. The other 40%

syndrome have a micr


chromosome 15 (delet
normal paternal chro
have two normal copi

es of paternally derived chromother or father. Uniparental disomy is the term


non known as uniparental
used when both chromosomes of a pair have been
syndrome results from lack of
inherited from only one parent. Prader-Willi and

mosome 15, a phenome


paternal disomy. The
a maternal copy of c

hromosome 15.
Angelman's syndromes are examples of imprinting,
and some cases are also examples of uniparental
ons
disomy.

Clinical Manifestati

Dysmorphisms seen in
Angelman's syndrome include
maxillary hypoplasia
, large mouth, prognathism, and
Prader-Willi Syndrome
nts are severely mentally retarded,

short stature. Patie


with impaired or abs

ent speech and inappropriate


Prader-Willi syndrome occurs in 1 per 15,000 newr. Jerky arm movements, ataxic
borns and is associated with an interstitial deletion
k result in marionette-like moveof the long arm of chromosome 15 (deletion of
s designation as the "happy
15qll-13). Approximately 70% of those affected
have a chromosome deletion in the paternally
ny patients have seizures.
derived chromosome 15 and a normal maternal chro-

paroxysms of laughte
gait, and tiptoe wal
ments, leading to it

puppet" syndrome. Ma

mosome 15. The remaining 30% have a normalMOLECULAR CYTOGENIC


appearing chromosome complement with two copies

DISORDERS

of maternal chromosome 15. This is known as uniparental maternal disomy, and the syndrome results
from the lack of a paternal copy of chromosome 15.

Fragile X Syndrome

Recurrence risk is 1 in 100, unless the chromosome


ed form of mental retardation
15 deletion results from a parental translocation,
1000 males, is the first example
which is extremely rare.

Fragile X, an X-link
that occurs in 1 in

of a trinucleotide r
epeat disorder. The gene involved,
Clinical Manifestations
ive in brain and sperm. In normal

called FMR-1, is act


individuals, the DNA

trinucleotide CGG is repeated


Dysmorphisms include narrow bifrontal diameter,
e start of this gene. Those
almond-shaped eyes, a down-turned mouth, and
e X have over 200 CGG repeats.
small hands and feet. Short stature and hypod its name because a cytogenetgonadotropic hypogonadism with small genitalia and
eakage occurs at a specific fragile
incomplete puberty are seen. These children suffer

about 30 times at th
affected with fragil
The disorder receive
ically detectable br
site on the X chromo

some. Currently, Southern blot


from severe hypotonia, which is associated with
rase chain reaction (PCR) are
feeding difficulties and failure to thrive in infancy. By
e number of CGG repeats. Clinseveral years of age, these children develop an unconicamay include macrosomia at birth,

analysis and polyme


used to determine th
l manifestations

--------------------------------------- 304
BLUEPRINTS
PEDIATRICS
Third Edition
Bradley S. Marino, MD, MPP
Assistant Professor of Anesthesia
Department of Anesthesia and Critical Care Medicine
Assistant Professor of Pediatrics
Department of Pediatrics
University of Pennsylvania
Division of Cardiology and Critical Care Medicine
The Children's Hospital of Philadelphia
Philadelphia, Pennsylvania
Katie S. Fine, MD
Private Pediatrician
North Raleigh Pediatric Group
Raleigh, North Carolina
Julia A. McMillan, MD
Professor of Pediatrics
Johns Hopkins University School of Medicine
Vice Chair for Education
Department of Pediatrics
Johns Hopkins Hospital
Baltimore, Maryland
Blackwell
Publishing

--------------------------------------- 305
2004 by Blackwell Publishing
Blackwell Publishing, Inc., 350 Main Street, Maiden, Massachusetts 02148-5018, U
SA
Blackwell Publishing Ltd, 9600 Garsington Road, Oxford OX4 2DQ, UK
Blackwell Science Asia Pty Ltd, 550 Swanston Street, Carlton, Victoria 3053, Aus
tralia
All rights reserved. No part of this publication may be reproduced in any form o
r by any
electronic or mechanical means, including information storage and retrieval syst
ems, without
permission in writing from the publisher, except by a reviewer who may quote bri
ef
passages in a review.
03 04 05 06 5 4 3 2 1

ISBN: 1-4051-0333-7
Library of Congress Cataloging-in-Publication Data
Marino, Bradley S.
Blueprints pediatrics / Bradley S. Marino, Katie S. Fine, Julia A. McMillan. 3rd
ed.
p. ; cm. (Blueprints)
Includes index.
Rev. ed. of: Blueprints in pediatrics, c2001.
ISBN 1-40510-333-7 (pbk.)
1. Pediatrics Outlines, syllabi, etc.
[DNLM: 1. Pediatrics Examination Questions. WS 18.2 M339b 2003] I. Marino,
Bradley S. Blueprints in pediatrics. II. Fine, Katie S. (Katie Snead) III. McMil
lan, Julia A.
IV. Title. V. Series.
RJ48.3 .M37 2003
618.92'00076-dc21
2002154156
A catalogue record for this title is available from the British Library
Acquisitions: Nancy Anastasi Duffy
Development: Amy Nuttbrock and Selene Steneck
Production: Debra Lally
Cover design: Dick Hannus
Interior design: Mary McKeon
Typesetter: SNP Best-set Typesetter Ltd., Hong Kong
Printed and bound by Capital City Press in Burlington, VT
For further information on Blackwell Publishing, visit our website:
www.blackwellpublishing.com
Notice: The indications and dosages of all drugs in this book have been recommen
ded in the medical literature
and conform to the practices of the general community. The medications described
and treatment prescriptions
suggested do not necessarily have specific approval by the Food and Drug Adminis
tration for use in the diseases
and dosages for which they are recommended. The package insert for each drug sho
uld be consulted for use
and dosage as approved by the FDA. Because standards for usage change, it is adv
isable to keep abreast of

revised recommendations, particularly those concerning new drugs.

--------------------------------------- 306
Table of Contents
Associate Editors vi
Preface vii
Acknowledgments viii
Introduction ix
Abbreviations x
1 Emergency Management: Evaluation of the Critically III or Injured Child 1
2 Poisoning, Burns, and Injury Prevention 8
3 Cardiology 16
4 Development 43
5 Dermatology 48
6 Endocrinology 57
7 Fluid, Electrolyte, and pH Management 69
8 Gastroenterology 75
9 Genetic Disorders 93
10 Hematology 103
11 Immunology, Allergy, and Rheumatology 125
12 Infectious Disease 136
13 Neonatology 162
14 Nephrology and Urology 198
15 Neurology 212
16 Nutrition 225
17 Oncology 229
18 Ophthalmology 243
19 Orthopedics 249
20 Pulmonology 259
Questions 267
Answers 276
Index .287

--------------------------------------- 307
ASSOCIATE EDITORS
David A. Munson, MD
Chief Resident
Department of Pediatrics
Division of General Pediatrics
The Children's Hospital of Philadelphia
Philadelphia, Pennsylvania
Jeanine Cooley Ronan, MD, MS
Chief Resident
Department of Pediatrics
Division of General Pediatrics
The Children's Hospital of Philadelphia
Philadelphia, Pennsylvania
SamirS. Shah, MD
Fellow
Department of Pediatrics
Divisions of General Pediatrics and Immunologic and Infectious Diseases
The Children's Hospital of Philadelphia
Philadelphia, Pennsylvania

--------------------------------------- 308
Preface
n 1997, the first five books in the Blueprints series were published as board
review for medical
I students, interns and residents who wanted high-yield, accurate clinical conte
nt for USMLE
Steps 2 & 3. Six years later, we are proud to report that the original books and
the entire
Blueprints brand of review materials have far exceeded our expectations.
The feedback we've received from our readers has been tremendously helpful an
d pivotal
in deciding what direction the third edition of the core books will take. The st
udent-to-student
approach was highly acclaimed by our readers, so resident contributors have been
recruited to
ensure that the third edition of the series continues to provide content and an
approach that
made the original Blueprints a success. It was suggested that the review questio
ns should reflect
the current format of the Boards, so new board-format questions have been includ
ed in this
edition with full explanations provided in the answers. Our readers asked for an
enhanced art
program, so a second color has been added to this edition to increase the useful
ness of the
figures and tables.
What we've also learned from our readers is that Blueprints is more than just
Board review
for USMLE, Steps 2 & 3. Students use the books during their clerkship rotations
and subinternships. Residents studying for USMLE Step 3 often use the books for reviewing
areas that
were not their specialty. Students in physician assistant, nurse practitioner, a
nd osteopath programs use Blueprints either as a companion or in lieu of review materials writte
n specifically
for their areas.
However you use Blueprints, we hope that you find the books in the series inf
ormative
and useful. Your feedback and suggestions are essential to our continued success
. Please send
any comments you may have about this book or any book in the Blueprints series t
o
blue@blackwellpub.com.
The Publisher
Blackwell Publishing
VII

--------------------------------------- 309
Acknowledgments
his book is a tribute to our patients. Each day we are remind
ed how truly precious chilT
dren are and what an honor it is to care for them. We are fo
rever grateful to our colleagues,
both resident and faculty, whose limitless understanding and supp
ort allow us to pursue projects such as this. We owe special thanks to Brian Stidham, MD, f
or his erudite chapter of pediatric ophthalmology. Finally, we would like to thank our families
, without whose support,
patience, and encouragement none of this would be possible.
B.M.
K.F.
J.M.
VIII

--------------------------------------- 310
Introduction
his book is an attempt to help the nonpediatrician understand that infants,
children, and
T
adolescents are not simply small adults. Congenital defects, the underdevelo
ped immune
system, and conditions reflecting abnormalities in organ development all play an
important
role in the care of pediatric patients. In some cases, the diseases of children
are different from
those seen in adults; often, the differences lie in the mode of presentation.
The physicians who wrote this book attempted to organize their knowledge into
a form
that is concise, complete, and clear. They relied on the most current sources in
the pediatric
literature to provide the reader with both important facts and an understanding
of the context
in which pediatric medical care is delivered. Although they learned from the lit
erature, it is
their patients who taught them the importance of what they learned.
Julia A. McMillan, MD

--------------------------------------- 311
Abbreviations
ABG arterial blood gas FEV
ory volume
ACTH adrenocorticotropic hormone FTA-ABS
eponemal antibody
AIDS acquired immunodeficiency
syndrome FVC
apacity
ALL acute lymphocytic leukemia G6PD

forced expirat
fluorescent tr
absorption
forced vital c
glucose-6-phos

phate
ALT alanine transaminase
AMP adenosine monophosphate Gl
al
ANA antinuclear antibody Hb
AP anteroposterior Hib
fluenzae type b
ARDS adult respiratory distress syndrome HIV
ficiency virus
ASD atrial septal defect HLA
e antigen
ASO anti-streptolysin 0 IFA
ent antibody
AST aspartate transaminase Ig
AZT zidovudine IM
BUN blood urea nitrogen INH
CAT computed axial tomography IVC
cava
CAW common atrioventricular valve IVIG
munoglobulin
CBC complete blood count JRA
toid arthritis
CDC Centers for Disease Control and JVP
pressure
Prevention KUB
/bladder
CF cystic fibrosis LDH
ogenase
CHF congestive heart failure LFTs
tests
CK creatine kinase LP
e
CNS central nervous system L/S
hingomyelin (ratio)
CSF cerebrospinal fluid LV
CT computed tomography LVH
ar hypertrophy
CXR chest x-ray MMR
rubella
DIC disseminated intravascular MR(I)
ance (imaging)
coagulation NG
DMD Duchenne's muscular dystrophy NPO
thing by mouth)
DTP diphtheria/tetanus/pertussis NSAID
nti-inflammatory
DTRs deep tendon reflexes
DVT deep venous thrombosis PCR
in reaction
EBV Epstein-Barr virus PDA
arteriosus
ECG electrocardiography PFTs
tion tests
ECMO extracorporeal membrane PMI
al intensity
oxygenation PPD
in derivative
EEG electroencephalography PT
me
ELISA enzyme-linked immunosorbent PTT

dehydrogenase
gastrointestin
hemoglobin
Haemophilus in
human immunode
human leukocyt
immunofluoresc
immunoglobulin
intramuscular
isoniazid
inferior vena
intravenous im
juvenile rheuma
jugular venous
kidneys/ureter
lactate dehydr
liver function
lumbar punctur
lecithin-to-sp
left ventricle
left ventricul
measles/mumps/
magnetic reson
nasogastric
nil per os (no
nonsteroidal a
drug
polymerase cha
patent ductus
pulmonary func
point of maxim
purified prote
prothrombin ti
partial thromb

oplastin time
assay RBC
EMG electromyography RF
tor
ESR erythrocyte sedimentation rate RPR
eagin

red blood cell


rheumatoid fac
rapid plasma r

--------------------------------------- 312
RSV respiratory syncytial virus UA urinalysis
RV right ventricle URI upper respiratory infection
RVH right ventricular hypertrophy US ultrasound
SIDS sudden infant death syndrome VMA vanillylmandelic acid
s/p status post VSD ventricular septal defect
T3RU triiodothyronine resin uptake vWF von Willebrand factor
T4 thyroxine WBC white blood cell
TSH thyroid-stimulating hormone

--------------------------------------- 313
USMLE Review
More than just Board review for USMLE, Steps 2 & 3, Blueprints can
clerkship rotations and subinternships,
and are especially helpful in studying in areas for Step 3 outside
ialty.
Concise and accurate clinical core content covers all you need
e USMLE and rotations
Includes USMLE style questions with full explanations provided

help you in
of your spec
to know for th
in the answers

Updated with the help of residents to maintain a student-to-student approach


Key Points in every section highlight the most important, high-yield informat
ion for each topic
Color-enhanced design to increase the usefulness of figures and tables
Here's what your colleagues are saying about Blueprints...
"I think the Blueprints series are excellent introductory texts to a clerkship.
I really like the clean 'look' of the series
and the 2-column template makes it easy to read late at night."
Jim Chen, Class of 2003, Boston University School of Medicine
"The Blueprints are the best booksput there. Everyone in my class uses them."
James Fletcher, Class of 2003, Eastern Virginia Medical School
"The Key Points section is a good way to summarize the most important informatio
n. Each chapters' organization
is in an easy to understand format making it better for quick review during the
clerkship and while preparing for
USMLE 2."
Nitin Gupta, Class of 2002, East Carolina University

"This was a solid knowledge base that was concise yet thorough. I was able to co
ver a lot of material in a short
amount of time."
Nicole Lacz, Class of 2004, UDMNJ - New Jersey Medical School
Blueprints :

M<*~ ;
Blueprint

Emerge"
Obstetrics & Cynecology
Blueprints
Pediatrics

Cardiology
Blueprints
Psychiatry
U5MLE

Blueprints
Blueprints in L
Family Medicine
Medicine
ISBN 1-14051-0333-7
Visit us at www.blackwellpublishing.com!
Blackwell
Publishing

You might also like